You are on page 1of 412

To my family,

who always stood beside me in every phase of my life.


Love You!

Special thanks to my friends, students,


teachers & my competitors
This book mightn’t have been completed without them.
Preface to Forth Edition
I feel a great sense of pride and enthusiasm in presenting forth edition of my
book. This book has primarily been written with the aim of meeting the needs
and interest of C.A. final students.

Every topic has been dealt precisely and to the point in a simple and
understandable language. Things have been explained with proper reasoning,
wherever possible. A good number of practical problems have been provided
to understand the theoretical aspect. Many new exercises from have been
introduced with hints for all difficult exercises. The book covers around past
25 year questions of CA, CWA & CIMA examination till Nov. ‘10. While
carrying out changes, the general approach to the subject, with an inclusion
of a variety of practical problems, and the lucidity of presentation of the
subject matter of the previous edition have been retained and an endeavor
has been made to give a lot more content to the user-both students and
instructors. I have deliberately kept this book in question bank format, to
help students in solving the problem on their own. This helps in gaining
confidence. Solutions of this book are separately provided. While making this
book I have tried to cover full new course syllabus & have inserted both new
course syllabus & old syllabus, to make aware about the differences between
2 syllabuses.

Human efforts are not perfect. In spite of my best efforts, I am aware of


possible errors and omissions that escaped my notice. I shall, therefore, be
extremely thankful to the learned ones who will extend their cooperation by
sending their valuable criticism, suggestions and observations for further
improvement of the book. I am reachable at paraggupta_ca@yahoo.co.in.

Dec.’ 2010 CA. Parag Gupta


 
No part of this book may be reproduced or utilized in any form or by 
any means, electronic or mechanical including photocopying, recording 
or by any information storage and retrieval system, without permission 
in writing from author. 
 
 
 
 
 
 
For any  Costing & O.R. related query, you may call me during 9:00 p.m.­
11:00 p.m. @ +91 9891432632 or mail me at 
paraggupta_ca@yahoo.co.in.  
 
For registration enquiry, any other query, etc. call +91 9968875529, 
+91 11 47665555 
 
 
 
 
 
 
 
For solutions of Costing, join world’s largest free consultancy group of 
CA Final students: 
http://groups.yahoo.com/group/costingbyparaggupta 
 
 
 
 
 
Price: Rs. 350/‐ 
 
 
 
 
 
 
 
 
 
 
This book is meant for educational & learning purposes. The author of the book has/have taken all responsible care to ensure that the contents of the book do not violate 
any existing copyright or other intellectual property rights of any person in any manner whatsoever. In the even the author has/have been unable to track any source 
and if any copyright has been inadvertently infringed, please notify the author in writing for corrective action. 
Syllabus - New Course
Paper 5: Advanced Management Accounting
(One paper – Three hours – 100 marks)

Level of Knowledge: Advanced knowledge

Objective:
To apply various management accounting techniques to all types of organizations for planning, decision
making and control purposes in practical situations.
To develop ability to apply quantitative techniques to business problems

1. Cost Management
(a) Developments in the business environment; just in time; manufacturing resources planning; (MRP);
automated manufacturing; synchronous manufacturing and back flush systems to reflect the importance of
accurate bills of material and routings; world class manufacturing; total quality management.
(b) Activity based approaches to management and cost analysis
(c) Analysis of common costs in manufacturing and service industry
(d) Techniques for profit improvement, cost reduction, and value analysis
(e) Throughput accounting
(f) Target costing; cost ascertainment and pricing of products and services
(g) Life cycle costing
(h Shut down and divestment.

2. Cost Volume Profit Analysis


(a) Relevant cost
(b) Product sales pricing and mix
(c) Limiting factors
(d) Multiple scarce resource problems
(e) Decisions about alternatives such as make or buy, selection of products, etc.

3. Pricing Decisions
(a) Pricing of a finished product
(b) Theory of price
(c) Pricing policy
(d) Principles of product pricing
(e) New product pricing
(f) Pricing strategies
(g) Pricing of services
(h) Pareto analysis

4. Budgets and Budgetary Control


The budget manual, Preparation and monitoring procedures, Budget variances, Flexible budgets, Preparation of
functional budget for operating and non-operating functions, Cash budgets, Capital expenditure budget,
Master budget, Principal budget factors.

5. Standard Costing and Variance Analysis


Types of standards and sources of standard cost information; evolution of standards, continuous-
improvement; keeping standards meaningful and relevant; variance analysis; disposal of variances.
(a) Investigation and interpretation of variances and their inter relationship
(b) Behavioural considerations.

6. Transfer pricing
(a) Objectives of transfer pricing
(b) Methods of transfer pricing
(c) Conflict between a division and a company
(d) Multi-national transfer pricing.

7. Cost Management in Service Sector


CA. Parag Gupta Ph.: +91 11 47665555 Paraggupta_ca@yahoo.co.in Costing & O.R.
World’s largest CA Final student’s consultancy group: http://groups.yahoo.com/group/costingbyparaggupta
8. Uniform Costing and Inter firm comparison

9. Profitability analysis - Product wise / segment wise / customer wise

10. Financial Decision Modeling


(a) Linear Programming
(b) Network analysis - PERT/CPM, resource allocation and resource leveling
(c) Transportation problems
(d) Assignment problems
(e) Simulation
(f) Learning Curve Theory
(g) Time series forecasting
(h) Sampling and test of hypothesis

Important things to be noted:

1. Marginal Costing (incld. Marginal v/s Absorption) has been deleted from New Course, although
students are expected to comprehensive knowledge about applicability of CVP analysis. {As per
Mr. R Devrajan (Last Director-Board of Studies), since Syllabus of New Course has specifically
deleted Marginal costing, there is no need to issue any specific notification in this regards. For
any clarification, students may compare both new & old course syllabuses}.
2. Topics newly introduced have been written in “Times New Roman” Font & are underlined.
3. It is advisable to students to read topics like “Balance Scorecard’ for their examination (although
they have been specially deleted from New course syllabus), because ICAI is still asking
questions from these topics in New course examinations.

CA. Parag Gupta Ph.: +91 11 47665555 Paraggupta_ca@yahoo.co.in Costing & O.R.
World’s largest CA Final student’s consultancy group: http://groups.yahoo.com/group/costingbyparaggupta
Syllabus - Old Course
Paper 5 : Cost Management
(One paper – Three hours – 100 marks)

Level of Knowledge: Expert Knowledge

Objectives: To gain expert knowledge of:


a) use of costing data for decision-making and control, and
b) emerging modern cost management concepts.

Contents:
1. Cost concepts in decision-making; Relevant cost, Differential cost, Incremental cost and Opportunity
cost.
2. Objectives of a Costing System; Inventory valuation; Creation of a Database for operational control;
Provision of data for Decision-Making.
3. Marginal Costing; Distinction between Marginal Costing and Absorption Costing; Break-even
Analysis, Cost-Volume-Profit Analysis. Various decision-making problems.
4. Standard Costing and Variance Analysis.
5. Pricing strategies: Pareto Analysis
6. Target costing, Life Cycle Costing
7. Costing of service sector.
8. Just-in-time approach, Material Requirement Planning, Enterprise Resource Planning, Total Quality
Management and Theory of constraints.
9. Activity-Based Cost Management, Bench Marking; Balanced Score Card and Value-Chain Analysis.
10. Budgetary Control; Flexible Budgets; Performance budgets; Zero-based budgets.
11. Measurement of Divisional profitability pricing decisions including transfer pricing.
12. Quantitative techniques for cost management, Linear Programming, PERT/CPM, Transportation
problems, Assignment problems, Simulation, Learning Curve Theory.

CA. Parag Gupta Ph.: +91 11 47665555 Paraggupta_ca@yahoo.co.in Costing & O.R.
World’s largest CA Final student’s consultancy group: http://groups.yahoo.com/group/costingbyparaggupta
Contents
Question Papers: Pages
New Course-Nov.’10 i - iv
Old Course-Nov.’10 v-x

Chapters:

Cost Accounting & Management:


1. CVP Analysis 1 - 17
2. Activity-based costing management 16 - 33
3. Target Costing, Value Chain analysis & Life 34 - 59
Cycle Costing
4. Service Sector 60 - 72
5. Standard Costing & Variance Analysis:
(i) Variance Analysis 73 - 114
(ii) Investigation of Variance 115 - 117
(iii) Balance Scorecard 117 - 122
6. Budget & Budgetary Control:
(i) Key Factor & Product Mix Decision 123 - 131
(ii) TOC, Throughput A/cing & Synchronous 131 - 135
Manufacturing
(iii) Budgetary Control 135 - 153
7. Transfer Pricing 154 - 181
8. Decision Making:
(i) Relevant Costing 182 - 206
(ii) Make or Buy 206 - 213
(iii) Subcontracting 214 - 216
(iv) Export Pricing 216 - 217
(v) Shut Down Point & Divestment Strategy 217 - 221
(vi) Inventory Decision 221 - 223
(vii) Miscellaneous (e.g. Joint Cost, D.C.F, etc.) 223 - 233
9. Miscellaneous Theory Chapters:
(i) Total Quality Management (TQM) 234 - 245
(ii) Pricing Decisions & Pareto Analysis 246 - 254
(iii) Benchmarking 255 - 257
(iv) JIT & Backflushing 258 - 265
(v) MRP I, MRP II & ERP 265 - 268
(vi) Computer Aided Manufacturing & Business 268 - 268
Process Re-engineering
(vii) Uniform Costing, Inter-firm Comparision & 269 - 276
D.P.P

Operations Research:
10. Linear Programming Problems 277 - 294
11. The Transportation Problem 295 - 306
12. The Assignment Problem 307 - 314
13. Network Analysis-PERT/CPM 315 - 335
14. Simulation 336 - 343
15. Learning Curve Theory 344 - 351
16. Time Series Analysis & Forecasting 352 - 365
17. Sampling and Test of Hypothesis 366 - 383

Tables:
1. Normal Table a
2. Student’s t Distribution b
3. Chi-Square Distribution c
4. F Table for alpha = 0.10 d
5. F Table for alpha = 0.05 e
6. Log Table f
7. Antilog Table h
i

Nov. 2010 -New Course Question Paper


Question No.1 is compulsory.
Answer any five from the remaining six questions.
Working notes should form part of the answer
Marks
1(a) A potato chips manufacturing company decided that the mean net weight per pack of its 5
product must be 90 grams. A random sample of 16 packets yields a mean weight of 80 grams
with standard deviation of 17.10 grams. Test the hypothesis that the mean of the whole
universe is less than 90, use level of significance of (a) 0.05(b) 0.01.

(b) What are the steps involved in Zero-base budgeting? 5

(c) G Ltd. produces and sells 95000 units of 'X' in a year at its 80% production capacity. The 5
selling price of product is `8 per unit. The variable cost is 75% of sales price per unit. The fixed
cost is `3,50,000. The company is continuously incurring losses and management plans to
shut-down the plant. The fixed cost is expected to be reduced to `1,30,000. Additional costs of
plant shut-down are expected at `15,000.

Should the plant be shut-down? What is the capacity level of production of shut-down point?

(d) H Ltd. manufactures three products. The material cost, selling price and bottleneck resource 5
details per unit area s follows :

Product X Product Y Product Z


Selling price (`) 66 75 90
Material and other variable cost (`) 24 30 40
Bottleneck resource time (minutes) 15 15 20

Budgeted factory costs for the period are `2,21,600. The bottleneck resources time available is
75120 minutes per period.

Required:
(i) Company adopted throughput accounting and products are ranked according to 'product
return per minute'. Select the highest rank product.
(ii) Calculate throughput accounting ratio and comment on it.

2(a) E Ltd. manufactures and sells four types of products under the brand names A, B, C and D. 12
On a turnover of `30 crores in 2009, company earned a profit of 10% before interest and
depreciation which are fixed. The details of product mix and other information are as follows :

Products Mix % to total sales P/V ratio (%) Raw Material as %


on sales value
A 30 20 35
B 10 30 40
C 20 40 50
D 40 10 60

Interest and depreciation amounted to `225 lakhs and `115.50 lakhs respectively. Due to
increase in prices in the international market, the company anticipates that the cost of raw
materials which are imported will increase by 10% during 2010. The company has been able
to secure a license for the import of raw materials of a value of `1,535 lakhs at 2010 prices. In
order to counteract the increase in costs of raw materials, the company is contemplating to
revise its product mix. The market survey report indicates that the sales potential of each of
the products: 'A', 'B' and 'C' can be increased upto 30% of total sales value of 2009. There
was no inventory of finished goods or work in progress in both the year.

You are required to :


Set an optimal product mix for 2010 and find the profitability.

CA. Parag Gupta Ph.: +91 11 47665555 Paraggupta_ca@yahoo.co.in Costing & O.R.
World’s largest CA Final student’s consultancy group: http://groups.yahoo.com/group/costingbyparaggupta
ii

(b) List out the remedies available for difficulties experienced during implementation of PRAISE. 4

3(a) A company is engaged in manufacturing of several products. The following data have been 10
obtained from the record of a machine shop for an average month:

Budgeted
No. of working days 24
Working hours per day 8
No. of direct workers 150
Efficiency One standard hour per clock hour
Down time 10%
Overheads Fixed `75,400
Variable `90,720
The actual data for the month of August 2010 are as follows :
Overhead Fixed `78,800
Variable `70,870
Net operator hours worked 20500
Standard hours produced 22550
There was a special holiday in August 2010.

Required:
(i) Calculate efficiency, activity, calendar and standard capacity usages ratio.
(ii) Calculate all the relevant fixed overhead variances.
(iii) Calculate variable overheads expenditure and efficiency variance.

(b) A firm makes two products X and Y, and has a total production capacity of 16 tonnes per day. 6
X and Y are requiring the same production capacity. The firm has a permanent contract to
supply at least 3 tonnes of X and 6 tonnes of Y per day to another company. Each tonne of X
require 14 machine hours of production time and each tonne of Y requires 20 machine hours
of production time. The daily maximum possible number of machine hours is 280. All the firm's
output can be sold, and the profit made is `20 per tonne of X and `25 per tonne of Y.

Required:
Formulate a linear programme to determine the production schedule for maximum profit by
using graphical approach and calculate the optimal product mix and profit.

4 Attempt any four : 4

(a) The following information is given by Z Ltd.:


Margin of safety `1,87,500
Total cost `1,93,750
Margin of safety 7500 units
Break-even sales 2500 units

Required:
Calculate Profit, P/V Ratio, BEP Sales (in `) and Fixed Cost.

(b) Explain the major components of balanced score card. 4

(c) List the 5 steps involved in the methodology of critical path analysis. 4

(d) Calculate the selling price per unit to earn a return of 12% net on capital employed (net of tax 4
@ 40%). The cost of production and sales of 80000 units are :
Variable cost including material cost `9,60,000
Fixed overheads `5,00,000
The fixed portion of capital employed is `12 lakhs and the varying portion is 50% of sales
turnover.

CA. Parag Gupta Ph.: +91 11 47665555 Paraggupta_ca@yahoo.co.in Costing & O.R.
World’s largest CA Final student’s consultancy group: http://groups.yahoo.com/group/costingbyparaggupta
iii

(e) What are the steps involved in carrying out Monte Carlo Simulation model? 4

5(a) 11
Fruitolay has decided to increase the size of the store. It wants the information
about the probability of the individual product lines : Lemon, grapes and
papaya. It provides the following data for the 2009 for each product line :

Lemon Grapes Papaya


Revenues `79,350.00 `2,10,060.00 `1,20,990.00
Cost of goods sold `60,000.00 `1,50,000.00 ` 90,000.00
Cost of bottles returned ` 1,200.00 `0.00 `0.00
Number of purchase orders 36 84 36
placed
Number of deliveries received 30 219 66
Hours of shelf stocking time 54 540 270
Items sold 12600 110400 30600

Fruitlay also provides the following information for the year 2009:
S.No. Activity Description of Activity Total Costs (`) Cost allocation
basis
1. Bottle returns Returning of empty 1,200.00 Direct tracing to
bottles to the store product line
2. Ordering Placing of orders of 15,600.00 156 purchase
purchases orders
3. Delivery Physical delivery and 25,200.00 315 deliveries
the receipts of
merchandise
4. Self stocking Stocking of 17,280.00 864 hours of
merchandise on store time
shelves and ongoing
restocking
5. Customer Assistance provided to 30,720.00 153600 items
Support customers including sold
bagging and checkout

Required :
(i) Fruitolay currently allocates store support costs (all costs other than the cost of goods sold)
to the product line on the basis of the cost of goods sold of each product line. Calculate the
operating income and operating income as the percentage of revenue of each product line.
(ii) If Fruitolay allocates stores support costs (all costs other than the cost of goods sold) to the
product lines on the basis of ABC system, calculate the operating income and operating
income as the percentage of revenue of each product line.
(iii) Compare both the systems.

(b) Discuss various forecasting methods using time series. 5


6 (a) A company has three plants located at A, B and C. The production of these plants is absorbed 8
by four distribution centres located at X, Y, W and Z. The transportation cost per unit has been
shown in small cells in the following table:

Distribution Centers X Y W Z Supply


Factories (Units)
A 6 9 13 7 6000
B 6 10 11 5 6000
C 4 7 14 8 6000
18000
Demand (Units) 4000 4000 4500 5000 17500

Find the optimum solution of the transportation problem by applying Vogel's Approximation
CA. Parag Gupta Ph.: +91 11 47665555 Paraggupta_ca@yahoo.co.in Costing & O.R.
World’s largest CA Final student’s consultancy group: http://groups.yahoo.com/group/costingbyparaggupta
iv

Method.

(b) Mention the data required to operate the material requirement planning system. 4

(c) "Customer profile is important in charging cost." Explain this statement in the light of customer 4
costing in service sector.

7(a) A company has two divisions: Division A and Division B. Both divisions of the Company 12
manufacture the same product but located at two different places. The annual output of
division A is 6000 tons (at 80% capacity) and that of division B is 7500 tons (at 60 % capacity).
The basic raw material required for production is available locally at both the places, but at
division A, it is limited to 4000 tons per annum at the rate of `100 per ton, at division B, it is
limited to 8000 tons per annum at the rate of `110 per ton. Any additional requirement of
material will have to be purchased at the rate of `125 per ton from other markets
at either of division. Variable costs per ton at each division remain constant. For every 1000
tons of output, 800 tons raw material is required. The details of other costs of the divisions are
as follows :

Division A Division B
Other Variable costs of output 122 per ton 120 per ton
Fixed Cost per annum 3,80,000 6,00,000

Required:
(i) Calculate variable cost per ton for each division's product and decide ranking in order of
preference.
(ii) The company desires to fully utilize the available local supplies of raw material to save the
overall variable cost of production; keeping the total production of both the divisions putting
together is the same as at present level. Calculate the quantity of production (output) that
could be transferred between the two divisions and overall saving in variable cost.
(iii) After considering the option (ii), how the balance capacity should be utilized if company is
working at L00% capacity, and also calculate selling price per ton if company mark up 10% on
full cost of each division's product.

(b) Explain distinctive features of learning curve theory in manufacturing environment. 4

CA. Parag Gupta Ph.: +91 11 47665555 Paraggupta_ca@yahoo.co.in Costing & O.R.
World’s largest CA Final student’s consultancy group: http://groups.yahoo.com/group/costingbyparaggupta
v

Nov. 2010 -Old Course Question Paper


Question No.1 is compulsory.
Answer any five from the remaining six questions.
Working notes should form part of the answer
Marks
1(a) The standard cost for producing 180 kgs of a product whose raw material inputs are A and B is 5
given below:

Standard Cost (`)


Material A 60 kgs @ `10 per kg 600
Material B 140 kgs @ `2 per kg 280
880

The actual prices of A and B were `12 and `8 per kg respectively. Consumption of B was 108
kg. The actual output at 80% yield was 144 kg.

Calculate the following direct material variances:


(i) Mix variance
(ii) Yield variance
(iii) Price variance
(iv) Usage variance

(b) Sportswear Ltd. manufactures sportswear shirts and shorts. The production budget for these 5
two products has to be prepared for the next three months, November 2010, December 2010
and January 2010.

The following information is given :


(i) Sales volume every month will be 2% more than the previous month's volume for each
product.
(ii) The company carries stock of finished garments sufficient to meet 40% of the next month's
sale.
(iii) Closing stock for October 2010 was 6000 shirts and 8000 shorts.

You are required to prepare the production budget for each product for November, December
2010 and January 2011.

( c) A factory has a special offer to produce 4 units of a labour intensive product by using its 5
existing facilities after the regular shift timings. The product can be produced by using only
overtime hours which entails normal rate plus 25%, so that usual production is not affected.
Two workers are interested in taking up this additional job every evening after their usual shift
is over. One is an experienced man who has been working on a similar product. His normal
wages are `48 per hour. The other worker is a new person who earns `42 an hour as normal
wages. He can be safely considered to have a learning curve ratio of 90% for this work. The
company wants to minimize labour cost for the order and only one person is to be chosen for
the job. The experienced man will take 20 hours for the first unit while the new worker will take
30 hours for the first unit. Evaluate who should be chosen for the job.

(d) You are given the following linear program. Introduce appropriate variables and restate the 5
problem to set up the simplex tableau. (Do not attempt further solution.)
Maximise:
8x 1 + 4x 2 – 3x 3 + 10x 4
s.t.
2x 1 – x 2 + x 3 + 2x 4 ≥ 40
3x 1 – x 2 + x 4 ≤ 90
2x 1 + x 2 + x 4 = 60
x1, x2, x3, x4 ≥ 0

2(a) M Ltd. makes two products, X and Y, in their respective divisions. Each unit of Y needs one 9
CA. Parag Gupta Ph.: +91 11 47665555 Paraggupta_ca@yahoo.co.in Costing & O.R.
World’s largest CA Final student’s consultancy group: http://groups.yahoo.com/group/costingbyparaggupta
vi

unit of X. Divisions X and Y are profit centres and can function according to their divisional
interests.

In the external domestic market, X can sell either 6000 units at `1,000 per unit or 5000 units at
`1,120 per unit.

X has a production capacity of 7000 units, with each unit requiring 2 hours. Y also has a
production and demand of 7000 units.

Y can buy product X from outside as follows:

Order Quantity Price for the entire order


(Units) (Rs/u)
6001-7000 900
4001-6000 920
2001-4000 1,000
0-2000 1,120

Y resorts to bulk purchase to avail maximum possible discount.

(i) There is an export order (that may either be fully accepted or fully rejected) for X to supply
800 units @ `900 per unit.

(ii) There is an offer to hire out X's capacity of 1600 hours at `130 per hour. The hiring offer
may either be fully accepted or fully rejected.

(iii) Y will not buy from X at any price more than it will incur in the outside market. Y does not
place restrictions on quantities to be supplied by X, provided its pricing condition is not
violated.

Given that any one or more of the offers may be accepted, what will be X's best strategy?
What will be the corresponding transfer price?
[A detailed cost statement is not essential. Only figures relevant for decision making are
required to be considered under each analysis.]

(b) State the pricing strategy that you would advise in the following situations which are 7
independent of each other :

(i) A new product is to be launched. It has had high promotional expenditure and its demand in
the market is not known.
(ii) A new product is to be launched. It is to be mass manufactured.
(iii) A product which has an external market demand is to be transferred to another division of
the same company. For the external market, variable selling costs of `10 per unit and fixed
selling costs amounting to `10 lacs p.a. are incurred. These costs are not applicable to
divisional transfers. The divisional transfer can take up only 20% of the output produced.
(iv) A special one-time order for the use of idle capacity is offered. This order will not impact
the existing sales of the company. The product has competition in the market.
(v) There is stock of a discontinued product. It has severe competition and the product is
perishable.

3(a) ABC Ltd. Manufactures four products A, B, C & D in the same factory. The following 9
information is given for a certain period :

Product A B C D
Good output (No. of units) 720 600 480 504
Average yield (%) 80 80 96 90
Machine hours per unit of input 4 3 2 1

CA. Parag Gupta Ph.: +91 11 47665555 Paraggupta_ca@yahoo.co.in Costing & O.R.
World’s largest CA Final student’s consultancy group: http://groups.yahoo.com/group/costingbyparaggupta
vii

The plant works such that after machining, the defectives in each run are automatically
segregated and dumped separately in a container. The good units pass through the process
and are further checked for quality by the inspectors of quality control who charge by the
number of batches inspected.
The total production and selling overheads of the company are the following for the period :
`
Machine operation and maintenance 66,375
Set up costs 19,200
Stores receiving 21,400
Inspection 24,000
Finished goods - packing / dispatch 14,400

The following additional information is given :

(i) A material requisition is made for every 25 units of input.


(ii) Machines need to be set up and tuned after each production run.
(iii) Production is in batches of 24 good units for all the products.
(iv) Units of A and B are packed in boxes that have 24 units capacity each and C & D are
packed in smaller boxes of 12 units capacity. The smaller box costs half the price of
the bigger box.
Each box contains only one type of product. There is no product mix up in packing.

Choose appropriate activity cost drivers for each overhead cost and calculate the overhead
cost per unit of good output for each of the products under the ABC system.

(b) At the end of activity 6-7, a product is to be launched and the date has been announced for the 7
inaugural function, based on the normal duration of activities as given in the network below.
Activities have been subcontracted by the project manager to contractors A, B, C, D, E, F, G
and H as indicated in the table below. Each subcontractor offers a discount on his contract
price for each day given to him in addition to the normal days indicated in the network. What
will be the maximum discount that the project manager may earn for the company without
delaying the launch of the product?

Activity Contractor Discount (`)/ Day


1–2 A 300
1–3 B 200
1–4 C 1,200
2–5 D 500
3–5 E 400
4–6 F 1,000
5–6 G 600
6–7 H 500

4(a) The manager of a hotel providing lodging facilities wants to expand his services to include 7
manual booking (reservation or cancellation) of railway tickets for his clients. He does not want
to have electronic booking due to operational difficulty. He has the following information:-

(`)/ month
Proportion of rent allocated for office space 4,000
General Telephone expenses allocated to this service 2,400

CA. Parag Gupta Ph.: +91 11 47665555 Paraggupta_ca@yahoo.co.in Costing & O.R.
World’s largest CA Final student’s consultancy group: http://groups.yahoo.com/group/costingbyparaggupta
viii

Proportion of security charges/ maintenance expenses allocated 1,600


Salary to person exclusively doing the booking of tickets 20,000
Mobile phone charges exclusive to person booking ticket 3,000
Share of general miscellaneous fixed expenses allocated 1,000
Conveyance incurred to book tickets (to and fro charges to the 4,000
nearest booking station)[fixed per month]

The manager estimates that there will be 2,500 bookings per month for 3 months of peak
season, 1,000 bookings per month for 2 months of moderate business and 700 bookings per
month during the remaining period. He cannot charge more than the prevailing rate of `30 per
booking charged by other agents.
Calculate the total cost per booking.
What is the estimated profit the manager hopes to achieve for the full year?
What should be the average minimum volume to justify the setting up of the new service ?
(Detailed break-up of monthly revenues or costs is not essential.)

(b) A manufacturing company makes 4 products that are sold through 8 regional offices 9
countrywide. The products pass through 3 production processes in a factory. A separate
market research division monitors outside competition. This division is outside the sales
management hierarchy.
As a management accountant, suggest some routine reports for performance measurement to
be made to :
(a) The Sales Management
(b) The Works Manager

5(a) A company has 3 factories F 1 , F 2 and F 3 , which supply the same product to 5 agencies A 1 , 10
A 2 , A 3 , A 4 and A 5 . Unit production costs, shipping costs and selling prices differ among the
different sources and destinations and are given below:

F1 F2 F3
Production Cost (`/ unit) 28 35 29
Production Capacity (No. of units) 110 240 125

Agencies A1 A2 A3 A4 A5
Selling Price `/u. 40 48 42 45 41
Demand (No. of units) 80 100 75 45 125

Shipping Costs `/u.


A1 A2 A3 A4 A5
F1 3 9 8 12 8
F2 6 10 6 2 5
F3 3 10 3 6 8

(i) Set up the initial transportation matrix for minimisation.


(ii) After doing (i) above, you are given the following additional information:
(a) 40 units must be transported from F 2 to A 2 as per an earlier agreement made by F 2
with A 2 ’s customer. This quality is included in the figures given for total production
and demand at these locations.
(b) Not more than 30 units may be sent from F 1 to A 1 , since the transporter’s vehicle
lacks space in this route.

Incorporating conditions (a) and (b) above, obtain the initial solution by Vogel’s Approximation
Method. (Do not attempt to continue for the full and final solution)

(iii) After doing the initial solution as in (ii) above, you are informed that the route from F 2
to A 1 is blocked by sudden flooding of the roads.
Without actual re-calculation, briefly explain how your solution is likely to be affected.

(b) The selling price per unit of a product is `14. For the forthcoming period, the demand will be 6

CA. Parag Gupta Ph.: +91 11 47665555 Paraggupta_ca@yahoo.co.in Costing & O.R.
World’s largest CA Final student’s consultancy group: http://groups.yahoo.com/group/costingbyparaggupta
ix

only 5,000 units. The fixed expenses at 50% activity (5,000 units) will be `30,000. The
company is thinking of shutting down operations, in which case an additional amount of `2,000
will have to be incurred for shutting down and only `20,000 of the above fixed costs can be
avoided.
What should be the variable cost per unit to recommend a shut down?

6 (a) Aero Ltd. has hired an aircraft to specially operate between cities A and B. All the seats of the 12
aircraft are economy class.
The following information is available :

Seating capacity of the aircraft =320 passengers


Average number of passengers per flight =240 passengers
Average one way fare from A to B =`5,000 per passenger
Variable fuel costs per flight from A to B `90,000
Food Cost `300 per passenger
(no charge to passenger)
Commission to travel agents 10% of the fare
(All tickets are through agents)
Annual lease costs allocated to each flight `2,00,000
Ground services, baggage handling / check- in `40,000
services costs per flight A to B
Flight crew salaries per flight A to B `48,000

There is an offer from another airlines operator, Mid Air Ltd. for a stop-over at destination D,
which is on the way from A to B. Due to this, the flight will operate from A to D, then D to B.
The following terms are to be considered for the stop-over:

50 seats will be booked by Mid Air at `2,500 per ticket, whether or not Mid Air is able to sell
them to its customers. No agent’s commission is payable on these tickets.
60 new passengers will be booked by Aero's travel agents for travel from A to D at a fare of
`2,000 per passenger.
Since the stop-over wastes more time, 25 of Aero’s original passengers from A to B will drop
out and seek other airlines which fly directly from A to B.
Due to the stop-over, fuel costs will increase from `90,000 to `1,35,000, Additional airport
landing/ baggage handling charges of `19,000 per stop-over will have to be incurred by Aero
Ltd.
Aero Ltd. will have to serve snacks to all passengers in the D to B sector at no charge to
passenger. Each snack will cost Aero Ltd. `200. This will be in addition to the original food at
`300 served in the A to D sector.

You may assume that fuel costs are not affected by the actual number of passengers in a
flight. You may ignore non-financial considerations, additional wear and tear to aircraft due to
extra landing / take-off.

Without considering Mid Air’s offer,


(i) What is the profit earned by Aero Ltd. per flight from A to B?
(ii) What is the break-even number of passengers for each flight from A to B?
Considering the effects of Mid Air’s offer,
(iii) Evaluate whether Aero should accept the offer.
(A detailed profitability statement is not essential, a relevant cost-revenue analysis would
suffice)

(b) How can simulation be applied in practical situations? 4

7 Answer any four of the following : 4


each
(a) Discuss the impact of JIT systems on overhead costs.

(b) What are benefits of Enterprise Resource Planning?

CA. Parag Gupta Ph.: +91 11 47665555 Paraggupta_ca@yahoo.co.in Costing & O.R.
World’s largest CA Final student’s consultancy group: http://groups.yahoo.com/group/costingbyparaggupta
x

(c ) A company’s four products M, N, O and P are in the market. Identify the phase of life cycle for
each product with a brief reason.

M : There is a lot of competition. Quantity sold has been increasing at 10%, 8% and 7% in the
last 3 years.

N : Until last year, N had no competition. Suddenly the company finds 4 new products very
similar to N in the market. However, N continues to have good sales.

O : There is intense competition. Achieving targeted sales is becoming increasingly difficult.


Hence the company is introducing slightly modified features in the fresh production.

P : Huge inventory of P is available. P is being sold, but there are many products in the market
which are priced lesser than P, but have the same utility as P.

(d) Three different salesmen X, Y and Z are to be assigned three different regions A, B and C so
that the company's revenue is maximised. The following matrix gives the sales revenue :

X Y Z
A 10 60 30
B 20 30 15
C 60 40 10

You are required to use the assignment technique to maximize revenue.

(e) TP Ltd. produces a product which passes through two processes - cutting and finishing.
The following information is provided :

Cutting Finishing
Hours available per annum 50,000 60,000
Hours needed per unit of product 5 12
Fixed operating costs per annum excluding direct 10,00,000 10,00,000
material

The selling price of the product is `1,000 per unit and the only variable cost per unit is direct
material, which costs `400 per unit. There is demand for all units produced.
Evaluate each of the following proposals independent of each other:
(i) An outside agency s willing to do the finished operation of any number of units
between 5,000 and 7,000 at `400 per unit.
(ii) An outside agency is willing to do the cutting operation of 2,000 units at `200 per
unit.
(iii) Add itional equipment for cutting can be bought for `10,00,000 to increase the
cutting facility by 50,000 hours, with annual fixed cost increased by `2 lacs.

CA. Parag Gupta Ph.: +91 11 47665555 Paraggupta_ca@yahoo.co.in Costing & O.R.
World’s largest CA Final student’s consultancy group: http://groups.yahoo.com/group/costingbyparaggupta
CVP Analysis

COST BEHAVIOR

THE NATURE OF COSTS: Before one can begin to understand how a business is going to perform over
time and with shifts in volume, it is imperative to first consider the cost structure of the business. This requires
drilling d own into the specific types of costs that are to be incurred and trying to understand their unique
attributes.
VARIABLE COSTS: Variable costs will vary in direct proportion to changes in the level of an activity. For
example, direct material, direct labor, sales commissions, fuel cost for a trucking company, and so on, may be
expected to increase with each additional unit of output.

Direct Cost per


Units
Material (`) unit(`)
10000 40000 4
20000 80000 4
30000 120000 4
40000 160000 4

Variable Costs Variable Cost per unit

200000 5
Material Cost Per Unit
Direct Material (Rs.)

4
150000
3
100000
2
50000 1

0 0
10000 20000 30000 40000 10000 20000 30000 40000
Units Produced Units Produced

FIXED COSTS: The opposite of variable costs are fixed costs. Fixed costs do not fluctuate with changes in
the level of activity. Examples include administrative salaries, rents, property taxes, security, networking
infrastructure support, and so forth. Observe that the fixed cost per unit will decline with increases in
production. This attribute of fixed costs is important to consider in assessing the scalability of a business
proposition.

Types of fixed costs: For planning purposes, fixed costs can be viewed as either committed or discretionary.

Committed fixed costs - Relate the investment in facilities, equipment & basic organizational structure.
Examples of such costs include depreciation of buildings and equipment, taxes on real estate, insurance and
salaries of top management and operating personnel. They have two key characteristics:
 They are long term
 They can’t be significantly reduced even for short periods of time without seriously impairing the
profitability or long run goals of the organization. Even if operations are interrupted or cut back,
the committed fixed costs will still continue largely unchanged. During a recession, for example, a
firm shall not usually discharge key executives or sell of key facilities.
Discretionary fixed costs - Usually arise from annual decisions by management to spend in certain fixed cost

CA. Parag Gupta Ph.: +91 11 47665555 Paraggupta_ca@yahoo.co.in Costing & O.R.
World’s largest CA Final student’s consultancy group: http://groups.yahoo.com/group/costingbyparaggupta
Basics of cost volume profit analysis -2-

areas. The most important characteristics of discretionary cost is that management is not locked into a
decision regarding such costs. They can be adjusted from year to year or even perhaps during the course of a
year if circumstances may demand such a modification. Examples are: Advertising, R & D, Public relations,
Management development programs, Internships for students.

Two key differences exist between discretionary & committed fixed costs:
 The planning horizon for discretionary fixed costs is short term.
 Discretionary fixed costs can be cut for short periods of time with minimal damage to the long run
goals or the organization.
Rent per
Units
Factory Rent (`) unit(`)
10000 50000 5.00
20000 50000 2.50
30000 50000 1.67
40000 50000 1.25
Fixed Costs (Rs.) Fixed Cost per unit

60000 6.00
Factory Rent (Rs.)

50000 5.00
40000 4.00
30000 3.00
20000 2.00
10000 1.00
0 0.00
10000 20000 30000 40000 10000 20000 30000 40000
Units Produced Units Produced

During a long period of time, virtually all costs tend to behave like variable costs. Within a shorter time
periods, costs will be fixed or variable in relation to changes in activity. The shorter the time period, the
greater the probability that a particular cost will be fixed. Consider a time period of one year, the costs of
providing the firm’s operating capacity such as depreciation and salaries of senior plant managers are likely to
be fixed in relation to changes in activity. Plant investment and abandonment decisions should not be based
on short-term fluctuations in demand within a particular year. Capacity costs will tend to be fixed in relation to
changes of activity within short-term periods such as one year. However, over long-term periods of several
years, significant changes in demand will cause capacity costs to change.

RELEVANT RANGE
The "relevant range" is the anticipated activity level at which you will perform. Any pricing data outside of this
range is irrelevant and need not be considered. This enhanced concept of variable cost is portrayed in the
following graphic:

Cost behavior often changes outside of the relevant range of activity due to a change in the fixed costs. When
volume increases to a certain point, more fixed costs will have to be added. When volume shrinks
significantly, some fixed costs could be eliminated. Fixed costs that behave in this fashion are also called
semi-fixed or step fixed costs
For example, you are buying frozen pizza in a box from the grocery store. One pizza can feed 3 guys. Step-
fixed cost is the cost of the pizza, it is sold by the box, and there is no partial pizza to be sold. The relevant
range is the 3 guys, once it's more than 3; it goes up to the next level->need another box of pizza. When you
have 4 guys waiting to eat pizza, you need to buy 2 pizzas, not 1. Similarly, If housekeeping staff can clean
CA. Parag Gupta Ph.: +91 11 47665555 Paraggupta_ca@yahoo.co.in Costing & O.R.
World’s largest CA Final student’s consultancy group: http://groups.yahoo.com/group/costingbyparaggupta
Cost Accounting & Management -3-

no more than 10 rooms each, a count of 51 guests would require six staff. If nine more guests arrive to bring
the total count to 60, the number of housekeeping staff needed is still only six. The next guest after that will
require going to the next “step”, or seven staff.

Note: Difference between Step fixed-cost function and Step variable-cost function is that the cost remains the
same in step fixed-cost function over wide ranges of the activity in each relevant range though in step
variable-cost function it remains same over narrow ranges of the level of activity in each relevant range.

MIXED COSTS: Many costs contain both variable and fixed components. These costs are called mixed or
semi-variable costs. If you have a phone, you probably know more than you wish about such items. Phone
agreements usually provide for a monthly fee plus usage charges for excess minutes, internet expense and
so forth. With a mixed cost, there is some fixed amount plus a variable component tied to an activity. Mixed
costs are harder to evaluate, because they change in response to fluctuations in volume. But, the fixed cost
element means the overall change is not directly proportional to the change in activity.

Methods for segregation of Mixed Cost:

1. Graphical Method (Scattered Graph) - The visual fit method or scatter-graph method requires that all
recent, normal data observations be plotted on a cost (Y-axis) versus activity (X-axis) graph. A line is
then drawn that is a best fit for the data points. When the line is extended to cross the Y-axis (at zero
units of activity), there is a "fairly accurate estimate of fixed costs for the period". The slope can also
be calculated to give another reasonably accurate estimate of the variable cost per product. To
compute the variable cost per unit, the slope of the line is determined by choosing two points and
dividing the change in their cost by the change in the units of activity for the two points selected.

2. High-Low Method (Range Method) - uses the total costs incurred at the high and low levels of activity
to classify mixed costs into fixed and variable components. The difference in costs between the high
and low levels represents variable costs.

Changes in Total Costs s


Variable Cost per unit =
High Minus Low Activity Level

CA. Parag Gupta Ph.: +91 11 47665555 Paraggupta_ca@yahoo.co.in Costing & O.R.
World’s largest CA Final student’s consultancy group: http://groups.yahoo.com/group/costingbyparaggupta
Basics of cost volume profit analysis -4-

The fixed cost can be found by subtracting the total variable cost at either the high or the low activity
level form the total cost at that activity level.

3. Comparison by period (Level of Activity Method) – This method is same as Range Method except
high & low activities we arbitrarily choose any two activity levels.

4. Least Squares Method – This method uses mathematical approach to determine the components of
variable & fixed expenses. The following regression equation for a straight line can be used to
express the relationship between a mixed cost & the level of activity:

Y = a + bX

We will solve following equations to yield the values of parameters a and b of the above equation.

ΣY = Na + b ΣX
ΣXY = a ΣX + b ΣX2
 Y = the total mixed cost
 a = the total fixed cost
 b = the variable cost per unit of activity
 X = the level of activity
 N = No. of activities

This equation makes it very easy to calculate what the total mixed cost would be for any level of
activity within the relevant range.

5. Analytical Method (Accounting Method): Each account under consideration is classified as either
variable or fixed based on the analyst’s prior knowledge of how the cost behaves.

Question 1: Briefly explain the methods of separating semi-variable costs into their fixed and variable
elements. (6 Marks) May/00

Question 2: Distinguish between ‘committed fixed costs’ and ‘discretionary fixed cost’ (5 Marks) May/96

Question 3: From the following information in respect of the semi - variable expenses obtain the fixed and
variable elements using the following methods.
a. Level of activity method. b. High low method.
d. Scatter Graph method e. Least squares method

Month Machine Hours Semi-variable maintenance expense (`)


January 400 2800
February 300 2600
March 200 2400
April 600 3200
May 500 3000
June 800 3600

Marginal Costing: The accounting system in which variable cost are charged to cost units and fixed costs of
the period are written off in full against the aggregate contribution. (CIMA’s Official Terminology). Variable
costing & Contribution Approach are other names of marginal costing.
It may be defined as the technique of presenting cost data wherein variable costs and fixed costs are shown
separately for managerial decision-making. It should be clearly understood that marginal costing is not a
method of costing like process costing or job costing. Rather it is simply a method or technique of the analysis
of cost information for the guidance of management which tries to find out an effect on profit due to changes
in the volume of output.

Product Cost : A product cost is the sum of the costs assigned to a product for a specific purpose. In Financial
accounting courses, it is a concept used in applying the cost plus approach to product pricing in which only

CA. Parag Gupta Ph.: +91 11 47665555 Paraggupta_ca@yahoo.co.in Costing & O.R.
World’s largest CA Final student’s consultancy group: http://groups.yahoo.com/group/costingbyparaggupta
Cost Accounting & Management -5-

the costs of manufacturing the product are included in the cost amount to which the markup is added. The
three components of manufacturing costs: direct materials, direct labor, and factory overhead costs.
Generally, inventoriable(manufacturing) costs are called product costs.

Manufacturing costs Product costs Recorded as an asset


(inventory) in the
balance sheet and
becomes and expense
in P&L A/c when the
product is sold.

Non-manufacturing costs
Recorded as an
Period costs expense in P&L A/c
in current accounting
period.

Period Costs: These are all costs in the income statement other than cost of goods sold. Period costs are
treated as expense of the A/cing period in which they are incurred because they are expected to benefit
revenues in that period and are not expected to benefit revenues in future periods (or because there is not
sufficient evidence to conclude that such benefits exists). In manufacturing organization all manufacturing
costs are regarded as product costs & all non-manufacturing costs are regarded as period costs. In
Merchandising sector, such as retailing, the cost of goods purchased is regarded as product costs & all other
costs such as administration & selling and distribution expenses are considered as period costs. R&D cost,
Design costs, Marketing costs, distribution costs, customer-service costs are some other examples of period
costs.

Marginal Cost: The cost of one unit of product or service which would be avoided if that unit were not
produced or provided. (CIMA’s Official Terminology)
Note: In this context, a unit is usually either a single article or a standard measure such as the liter or
kilogram, but in certain circumstances is an operation, process or part of an organization.

The marginal cost of a product –“is its variable cost”. This is normally taken to be; direct labour, direct
material, direct expenses and the variable part of overheads.

Presentation of Cost Data under Marginal Costing and Absorption Costing

Following presentation of two Performa shows the difference between the presentation of information
according to absorption and marginal costing techniques:

Absorption Costing
ABC Ltd.
Income Statement
For the year ended ….
(`‘000)
Sales yyy
Cost of Goods Sold:
Direct material consumed xxxx
Direct labour cost xxxx
Variable manufacturing overhead xxxx
Fixed manufacturing overhead xxxx
Manufacturing Cost incurred during the year (a.k.a.Gross Factory Cost) xxx

CA. Parag Gupta Ph.: +91 11 47665555 Paraggupta_ca@yahoo.co.in Costing & O.R.
World’s largest CA Final student’s consultancy group: http://groups.yahoo.com/group/costingbyparaggupta
Basics of cost volume profit analysis -6-

Opening Work-in-Progress xxxx


Less: Closing Work-in-Progress xxxx
Total cost of goods manufactured xxxxx
Add: Op. stock of finished goods (valued at total cost of previous year) xxxx
Less: Cl. stock of finished goods (valued at total cost of current year) xxxx yyyy
Gross profit/Margin (i.e. Sales-Cost of goods sold) yyyyy
Less: Operating Costs:
These are Selling and distribution costs (Both Fixed & Variable) xxxx
period costs Administration costs, etc. (Both Fixed & Variable) xxxx yyyy
Operating Income zzzz
Less: Under-absorption of Fixed factory overhead yyyy
Add: Over-absorption of Fixed factory overhead yyyy
Operating Income zzzzz

Marginal Costing
ABC Ltd.
Income Statement (a.k.a. Contribution Income Statement)
For the year ended ….
(`‘000)
Sales yyy
Total variable cost: xxxx
Direct material consumed xxxx
Direct labour cost xxxx
Variable manufacturing overhead xxxx
Variable cost of goods produced xxxxx
Add: Op. stock of finished goods (valued at Total Var. Cost of previous year) xxxx
Less: Cl. stock of finished goods (valued at Total Var. Cost of current year) xxxx
Variable Cost of Goods Sold xxxx
Add: Variable administration, selling and dist. overhead xxxxx
Total variable cost yyyy
Contribution Margin (Sales - Total variable cost) yyyyy
Less: Fixed operating costs (Production, administration, selling and distribution) yyyy
Operating Income zzzz

1. Net Income/Profit = Operating Income – Non Operating Expenses (e.g. Income Taxes, Interest, etc.)
2. As Non Operating Expenses are generally not given in question, we take Operating Income as Net Profit

Question 4: State the distinction between Marginal Costing and Absorption Costing. (7 Marks) Nov/01

Absorption Costing Marginal Costing


1. It is a total cost technique i.e. both variable and Here only variable costs are charged to product,
fixed costs are charged to products, processes or processes or operations. Fixed costs are charged as
operations. period costs to the profit statement of the same
period in which they are incurred.
2. Fixed factory overheads are absorbed by the The cost of production under this method does not
production units on the basis of a predetermined include fixed factory overheads and therefore, the
fixed factory overhead recovery rate based on normal value of closing stock comprises of only variable
capacity. Under/over absorbed overheads are costs. No part of the fixed expenses in included in the
adjusted before arriving at the figure of profit for a value of closing stock and carried over to the next
particular period. period.
3. Inspire of best possible forecast and equitable Since fixed overheads are not included in the cost of
basis of apportionment/allocation of fixed costs, production, therefore the question of their under/ over
under or over recovery of fixed overheads generally recovery does not arise.
arises.
4. Managerial decisions under this costing technique Here decisions are made on the basis of contribution
CA. Parag Gupta Ph.: +91 11 47665555 Paraggupta_ca@yahoo.co.in Costing & O.R.
World’s largest CA Final student’s consultancy group: http://groups.yahoo.com/group/costingbyparaggupta
Cost Accounting & Management -7-

are based on profit i.e. excess of sales value over i.e. excess of sales price over variable costs. This
total costs, which may at times lead to erroneous basis of decision making results in optimum
decisions. profitability.
Cost-Volume-Profit Analysis
Cost-volume-profit (CVP) analysis is used to determine how changes in costs and volume affect a
company's operating income and net income. In performing this analysis, there are several assumptions
made, including:
 Sales price per unit, Variable costs per unit & total fixed costs are known & constant (Within relevant
range & time period) & if represented graphically they are linear in behavior (representing straight
lines).
 Other variables like production efficiency, production methods, and price levels remain constant.
 Everything produced is sold.
 All the company's costs, including manufacturing, selling, and administrative costs, be identified as
variable or fixed
 Revenue & Costs are only affected because activity changes.
 If a company sells more than one product, they are sold in the same mix (i.e. constant sales mix).
 All revenues & costs can be added, subtracted & compared without taking into account time value of
money.
Contribution margin and contribution margin ratio
Key calculations when using CVP analysis are the contribution margin and the contribution margin
ratio. The contribution margin represents the amount of income or profit the company made before
deducting its fixed costs. Said another way, it is the amount of sales available to cover (or contribute to)
fixed costs. When calculated as a ratio, it is the percent of sales available to cover fixed costs. Once
fixed costs are covered, the next rupee of sales results in the company having income. The contribution
margin (a.k.a. contribution) is sales revenue minus all variable costs. To calculate the contribution
margin ratio, the contribution margin is divided by the sales or revenues amount. Contribution Margin
Ratio is also popularly known as Profit Volume Ratio (P/V Ratio) & Contribution Sales Ratio (C/S
Ratio).
Single product contribution income statement

Units XXXX
(`‘000)
Sales XXX
Less: Variable cost XXX
Contribution XXX
Less: Fixed cost XXX
Profit XXX

Multi-product contribution income statement

Particulars A B C Total
Sales XXX XXX XXX XXX
Less: Variable cost XXX XXX XXX XXX
Contribution XXX XXX XXX XXX
Less: Specific Fixed cost XXX XXX XXX XXX
Total XXX XXX XXX XXX
Less: General/Common fixed cost XXX
Profit
XXX
Formulae for Calculation:
 Profit (Operating Income) = Sales – Variable Costs – Fixed Costs
 Contribution = Sales – Variable costs
 Contribution = Fixed Cost + Profit
 Sales – Variable cost = Fixed cost + Profit
 P/V ratio (or C/S ratio) = Contribution ÷ Sales
CA. Parag Gupta Ph.: +91 11 47665555 Paraggupta_ca@yahoo.co.in Costing & O.R.
World’s largest CA Final student’s consultancy group: http://groups.yahoo.com/group/costingbyparaggupta
Basics of cost volume profit analysis -8-

= Contribution per unit ÷ Selling price per unit


= Change in Contribution ÷ Change in Sales
= Change in Profit ÷ Change in Sales
= Profit ÷ Margin of Safety Sales
= 1 – Variable Cost Ratio
 Profit = (Sales × P/V ratio) - Fixed Cost = P/V ratio × Margin of Safety sales(`)
= Contribution p.u. × Margin of safety (in units)

Break-even point
The break-even point represents the level of sales where net income equals zero. In other words, the
point where sales revenue equals total variable costs plus total fixed costs, and contribution margin
equals fixed costs. Variable costs represent all variable costs including costs classified as
manufacturing costs, selling expenses, and administrative expenses. Similarly, the fixed costs represent
total manufacturing, selling, and administrative fixed costs.
 Break Even point (in units) = Fixed Cost ÷ Contribution per unit
Break-even point (in rupees): The break-even point in sales rupees is calculated by dividing total fixed
costs by the contribution margin ratio.
 Break Even Sales (in sales value) = Fixed Cost ÷ P/V ratio
 Break Even Sales (in sales value) = Variable Costs + Fixed Costs
Once the break-even point in units has been calculated, the break-even point in sales rupees may be
calculated by multiplying the number of break-even units by the selling price per unit. This also works in
reverse. If the break-even point in sales rupees is known, it can be divided by the selling price per unit
to determine the break-even point in units.

Profit Graph

CA. Parag Gupta Ph.: +91 11 47665555 Paraggupta_ca@yahoo.co.in Costing & O.R.
World’s largest CA Final student’s consultancy group: http://groups.yahoo.com/group/costingbyparaggupta
Cost Accounting & Management -9-

Contribution Break Even Chart

Question 5: A company manufactures a single product having a marginal cost of `0.75 per unit. Fixed
Cost is `15000 per annum. The market is such that up to 40000 units can be sold at a price of `1.50 per
unit, but any additional sale must be made at Re. 1 per unit. Company has a planned profit of `25000.
How many units must be made and sold.

[Ans.: 80000]
Question 6 (Sensitivity analysis): The Super co. owns and operates six outlets in and around Kansas City.
You are given the following corporate budget data for next year:

(`)
Revenue 1,00,00,000
Fixed Costs 17,00,000
Variable Costs 82,00,000

Variable costs change with respect to the number of units sold.

Required:
Compute the budget operating income for each of the following deviations from the original budget data.
(Consider each case independently.)
a. A 10% increase in contribution margin, holding revenues constant.
b. A 10% decrease in contribution margin, holding revenues constant.
c. A 5% increase in fixed costs.
d. A 5% decrease in fixed costs.
e. An 8% increase in units sold.
f. An 8% decrease in units sold.
g. A 10% increase in fixed costs and 10% increase in units sold.
h. A 5% increase in fixed costs and 5% decrease in variable costs.

[Ans.: 280000,(80000),15000,185000,244000,(44000),110000,425000]
Question 7 (Sensitivity analysis): If labour costs and material cost are likely to go up by 10% and 5%
respectively per unit, what is the percentage increase necessary in selling price to keep the P/V of 20% as
before, assuming that the ratio between material and labour is 3:2, and variable overheads is nil.

[Ans.: 7%]

Question 8: H Ltd. produces Pens and Pencils. The company’s budget for 2008 includes the following data:

CA. Parag Gupta Ph.: +91 11 47665555 Paraggupta_ca@yahoo.co.in Costing & O.R.
World’s largest CA Final student’s consultancy group: http://groups.yahoo.com/group/costingbyparaggupta
Basics of cost volume profit analysis - 10 -

Pens Pencils
Unit Selling Price (`) 10 5
Contribution Margin ratio (%) 40 60

The budget is designed to show a figure of profit or loss for each product, after apportioning joint fixed costs
of `100000 in proportion to the number of units of each product sold.
For 2008, Pens are budgeted to show a profit of `14000, and pencils a loss of `2000. The number of units of
each product sold is expected to be equal.
You are required to write a report to the managing director of H Ltd. advising him on the basis of the
information given whether to implement any of the following three proposals:
(i) to increase the price of the pencil by 25%, in the expectation that the price elasticity of demand
over this range of prices will be unity;
(ii) to make changes to the production process that would reduce the joint fixed costs by 12.5% and
increase the variable costs of each product by 10%;
(iii) to introduce both the above changes.

[Ans.: (i) Increase in contribution = `6400; (ii) Decline in profit = `300; (iii) Increase in profit = `6740]

Targeted income
CVP analysis is also used when a company is trying to determine what level of sales is necessary to reach a
specific level of income, also called targeted income. To calculate the required sales level, the targeted
income is added to fixed costs, and the total is divided by the P/V Ratio to determine required sales rupees, or
the total is divided by contribution per unit to determine the required sales level in units.

Fixed Costs + Target Operating Income


Required Sales Revenue in Rupees =
P/V Ratio

Fixed Costs + Target Operating Income


Required Sales in Units =
Contribution Margin per unit

This calculation of targeted income assumes it is being calculated for a division as it ignores income taxes. If
a targeted net income (income after taxes) is being calculated, then income taxes would also be added to
fixed costs along with targeted net income.

Fixed Costs + Target Net Income + Income


Required Sales Revenue in Rupees = Tax
P/V Ratio

Fixed Costs + Target Net Income + Income Tax


Required Sales in Units =
Contribution Margin per unit
Where, Target Operating Income = Target Net Income + Income Taxes
If Rate of Income Tax is known & Value of Income Tax not known then,

Target Net Income


Target Operating Income =
1 - Tax Rate

Question 9: X Ltd. is a recently formed company, manufacturing vehicles. Its cost structure is such that on
sale of every `2,000, it spends `1400/-.In 2002, when the total sales revenue was `10,00,000/-, it sustained
loss of `2,00,000/- You are required to compute the break even point. If the minimum net profit to be earned
is `2,00,000/- in order to justify the survival, what must be sales revenue?

[Ans.:1666667 & 2333333]


Question 10(Volume analysis): The following figures for profit & sales are obtained from the accounts of X
Co. Ltd.

CA. Parag Gupta Ph.: +91 11 47665555 Paraggupta_ca@yahoo.co.in Costing & O.R.
World’s largest CA Final student’s consultancy group: http://groups.yahoo.com/group/costingbyparaggupta
Cost Accounting & Management - 11 -

Year Sales Profit


` `
2002 20,000 2,000
2003 30,000 4,000
(i) Find out Contribution Sales Ratio.
(ii) What is Break Even Sales
(iii) Find out the sales to earn a profit of `6000 in 2004.
(iv) What is the profit when sales are `12000 in 2004?

[Ans.:0.2,10000,40000,400]

Margin of Safety:
Margin of safety is the difference between the sales or productions at a particular level of activity and the
break even sales a production. A large margin of safety indicates the soundness of the business and
correspondingly a small margin of business indicates a not too-sound position.
Margin of safety can be improved by lowering the fixed cost and variable costs, increasing the volumes of
sales and production, increasing the selling prices or changing the product mix resulting into a better overall
Profit/Volume ratio.
 Margin of safety Sales = Sales at selected activity – BEP Sales
 Margin of safety Sales = Profit at selected activity ÷ P/V ratio
 Margin of safety (%) = Margin of Safety Sales X 100 ÷ Sales at selected activity
 Margin of safety (%) = 1 – BEP (%)

Question 11(Single product marginal cost sheet): A company producing a single article sells it at `10
each. The marginal cost of production is `6 each and fixed cost is `400 per annum.
Calculate
(a) The P/V ratio;
(b) The break-even sales;
(c) The sales to earn a profit `500;
(d) Profit at sales `3,000;
(e) New break-even point if sales price is reduced by 10%.
(f) MOS when the profit earned in `200 and PVR – 40%.

[Ans.: 0.4, 1000, 2250, 800, 1200, 500]


Question 12: From the following particulars, you are required to calculate:
(i) P/V Ratio
(ii) BEP for sales;
(iii) Margin of Safety;
(iv) Profit when sales are `2,00,000/-
(v) Sales required to earn a profit of `40,000/-

Year Sales Profit


I `2,40,000 18,000
II `2,80,000 26,000

You may make plausible assumptions. Also evaluate the effect on II year’s profit of
(a) 20% decrease in sales quantity.
(b) 20% decrease in sales quantity accompanied by 10% increase in sales price and reduction of `3,500/- in
fixed costs.

[Ans.: 0.20, 150000, 130000, 10000, 350000, (a) Reduction in profit 11200; (c) Increase in profit 14700]
Question 13: The following information is given by Z Ltd.:

Margin of safety `1,87,500


Total cost `1,93,750

CA. Parag Gupta Ph.: +91 11 47665555 Paraggupta_ca@yahoo.co.in Costing & O.R.
World’s largest CA Final student’s consultancy group: http://groups.yahoo.com/group/costingbyparaggupta
Basics of cost volume profit analysis - 12 -

Margin of safety 7500 units


Break-even sales 2500 units

Required:
Calculate Profit, P/V Ratio, BEP Sales (in `) and Fixed Cost. (4 Marks) Nov./10-N.C.

[Ans.: `56250, 0.30, `62500, `18750]


Question 14: A single product company furnishes the following data:

Year 2007 Year 2008


Sales 24,00,000 ?
P/V Ratio 33-1/3 % 30%
Margin of Safety 25% 40%
While there was no change in volume of sales in year 2008, the selling price was reduced. Calculate sales,
fixed cost and profit for year 2008.

[Ans.: Sales: 2285714, Fixed Cost: 411429, Profit: 274285]

Angle of Incidence:
It is the angle of intersection (θ) between the sales & the total cost lines. It indicates the profit earning capacity
of the concern at a certain level of sales production. The larger the angle of incidence the more is the profit
earning capacity & vice versa. It also provides an indication as to what extent the output & sales price may be
varied to attain a desire level of profit. It gives an easy & clear idea to the profitability under different levels of
activities & also for different product mix & is a simple visual aid to find out profit earning capacity without
going in for any calculation.

Curvilinear CVP analysis


In CVP analysis, the usual assumption is that the total sales line and variable cost line will have linear
relationship, i.e. these lines will be straight lines, and however, in actual practice it is unlikely to have a linear
relationship for two reasons, namely:
--- After the saturation point of existing demand the sales value may show a downward trend.
--- The average unit variable cost declines initially, reflecting the fact that, as output increases the firm
will be able to obtain bulk discounts on the purchase of raw materials and can also benefit from division of
labour. When the plant is operated at further higher levels of output, due to bottlenecks and breakdowns the
variable costs per unit will tend to increase. Thus the law of increasing costs may operate and the variable
cost per unit may increase after reaching a particular level of output.
In such cases, the contribution will not increase in linear proportion on the phenomenon of diminishing
marginal productivity; the total cost line will not be straight, as assumed but will be of curvilinear shape. This
situation will give rise to two break even points. The optimum profit is earned at the point where the distance
between sales and total cost is the greatest.

CA. Parag Gupta Ph.: +91 11 47665555 Paraggupta_ca@yahoo.co.in Costing & O.R.
World’s largest CA Final student’s consultancy group: http://groups.yahoo.com/group/costingbyparaggupta
Cost Accounting & Management - 13 -

Since Marginal Costing has been shifted from Final to (I)PCC, this book only basic concepts
of CVP & doesn’t contains certain important topics like Marginal vs Absorption, Composite
BEP, BEP with semi-variable cost, BEP with limiting factor, Cash BEP, Multiple BEP, etc.
Students are expected to have a comprehensive knowledge of concepts of these topics before
they initiate themselves towards advance studies for the final examination.

Practice Questions on BEP

Composite BEP i.e. more than one product with common fixed costs
(i) With out limiting factor (Multi Products)
BEP (in units) = Fixed cost ÷ Weighted Average contribution p.u.
Where, Weighted Av. Contribution p.u. = Σ [Sales Mix(%) × Contribution p.u.]
Also, BEP (in `) = Fixed cost ÷ composite P/V ratio
Where, composite P/V ratio = Cumulative Contribution ÷ Cumulative Revenue
But, when sales mix in rupee is given
BEP (in `) = Fixed cost ÷ composite P/V ratio
Where, composite P/V ratio = Σ [Sales Mix × P/V Ratio]
(ii) With limiting factor:
Find contribution per limiting factor & give rank. Find total contribution from 1st rank product.
Calculate the amount of fixed cost still to recover. Whether it can be recovered by 2nd rank product or
not?

Question 1(Sales mix and BEP): Aravind Ltd. manufactures and sells four products under the brand names
A, B, C & D. the following details are provided in respect of the products.

Product A B C D
% in Sales Value 30 40 20 10
% of Variable cost to selling price 60 70 80 30

The total budgetary sales (100%) are `10, 00,000 p.m. fixed costs are `2, 50,000 p.m.
The company’s new sales manager, Aravind has suggested a change in sales mix keeping the total sales at
`10, 00,000 per month. His suggestion is as under:

Product A B C D
% in Sales Value 25 40 30 5

(1) Calculate the break-even point for the Company, under the existing sales mix.
(2) Compute the effect of implementing the suggested change in sales mix.
(3) Explain the reasons for the effect of change in sales mix despite total sales and fixed cost being the
same.

[Ans.: 714286, BEP will change to 793651]


Question 2(Sales mix and BEP): The budgeted results of A Co. Ltd. include:

Product Sales value (`) P/V ratio


A 50,000 50%
C 80,000 40%
O 1,20,000 30%

Fixed overhead for the period `1,00,000.

The directors are worried about the results of the company. They have requested you to prepare a statement
showing the amount of loss expected and recommend a change in the sales of each product or in total mix
which will eliminate the expected loss.

[Ans.: 7000, New BEP Sales 85000, 80000, 85000]

CA. Parag Gupta Ph.: +91 11 47665555 Paraggupta_ca@yahoo.co.in Costing & O.R.
World’s largest CA Final student’s consultancy group: http://groups.yahoo.com/group/costingbyparaggupta
Basics of cost volume profit analysis - 14 -

Question 3: A multi- product company has the following costs and output data for the last year

Product
X Y Z
Sales mix (in value) 40% 35% 25%
Selling price `20 `25 `30
Variable cost per unit `10 `15 `18
Total fixed costs `1,50,000
Total sales `5,00,000
The company proposes to replace product Z with product S.

Estimated cost and output data are:

Product

X Y Z
Sales mix (in value) 50 % 30% 20%
Selling price `20 `25 `28
Variable cost per unit `10 `15 `14
Total fixed costs `1,60,000
Total sales `4,50,000
Analyze the proposed change and suggest what decision the company should take.
Also state the break even point for the company as a whole in the two situations.

[Ans.: Continue Product Z; 340909 & 340426]


Question 4: XYZ Ltd. sells three Products A,B & C. The following information is provided:

Particulars A B C
Sales Volume (units) 7000 5000 6000
Selling Price per unit (`) 10 8 5
Variable Cost per unit (`) 5 6 2.50

Fixed Cost p.a. `40000. State the break even point for the company as a whole.

[Ans.: BES 93334]

Break-even point in case of step cost: In some cases, some costs tend to behave as fixed for
production within batches though are variable with quantity of batches, in such cases we will calculate Break-
even level of units on batches of production.
Question 5(Multiple break even points): A firm sells its product at `25 per unit. Its Cost behavior for various
production ranges is:

Units of Cumulative fixed Variable Cost per


production Cost Unit
0 –16,000 2,50,000 16.00
16,001 – 60,000 3,50,000 17.00
60,001 and above 5,00,000 20.00

CA. Parag Gupta Ph.: +91 11 47665555 Paraggupta_ca@yahoo.co.in Costing & O.R.
World’s largest CA Final student’s consultancy group: http://groups.yahoo.com/group/costingbyparaggupta
Cost Accounting & Management - 15 -

Identify the break- even point(s) in units.

[Ans.: 43750 & 100000]


Question 6(Multiple Break even points): Kalyan University conducts a special course on ‘Computer
Applications’ during summer. For this purpose, it invites applications from graduates. An entrance test is
given to the candidates and based on the same, a final selection of a hundred candidates is made. The
entrance test consists of four objective type of Examination and is spread over four days, one examination
per day. Each candidate is charged a fee of `50 for taking up the entra nce test. The following data was
gathered for the past two years:

Statement of Net Revenue from the Entrance Test for the course on “Computer Application”

Year 1 Year 2
(`) (`)
Gross Revenue (Fees collected) 1,00,000 1,50,000
Costs
Valuation 40,000 60,000
Question booklets 20,000 30,000
Hall rent at `2, 000 per day 8,000 8,000
Honorarium to Chief Administrator 6,000 6,000
Supervision charges (1 supervisor for every 100 candidates at 4,000 6,000
`50/- per day)
General Administration Expenses 6,000 6,000
Total Cost 84,000 1,16,000
Net revenue 16,000 34,000

Required to compute:
(a) The budgeted net revenue if 4,000 candidates take up the entrance test in Year 3.
(b) The break even number of candidates.
(c) The number of candidates to be enrolled if the net income desired is `20,000/- .

[Ans.: 52000, 1120, 2230]


Question 7: A hospital operates a 40 bed capacity special health care April 7, 2003 department. The said
department levies a charge of `425 per bed day from the patient using its services. The data relating to fees
collected and costs for the year 2001 are as under:
`
Fees collected during the year 3495625
Variable costs based on patient days 1357125
Departmental fixed costs 622500
Apportioned costs of the hospital administration charges 1000000

Based the above, nursing staff were employed as per the following scale at `48000 per annum per nurse.
Annual Patient days No. of Nurses required
Less than 5000 3
5000-7000 4
7000-9000 6
Above 9000 8

The projections for the year 2002 are as under:


-The costs other than apportioned overheads will go up to 10%.
-The apportioned overheads will increase by `250000 per annum.
-The salary of the nursing staff will increase to `54000 per annum per nurse.
The occupancy of the bed capacity is not likely to increase in 2002 and consequently the management is
actively considering a proposal to close down the department. In that event, the departmental fixed costs can
be avoided.

Required:
CA. Parag Gupta Ph.: +91 11 47665555 Paraggupta_ca@yahoo.co.in Costing & O.R.
World’s largest CA Final student’s consultancy group: http://groups.yahoo.com/group/costingbyparaggupta
Basics of cost volume profit analysis - 16 -

(i) Present situation to show the profitability of the department for the years 2001 and 2002.
(ii) Calculate the:
- Break even bed capacity for the year 2002.
- Increase in fee per bed day required to justify continuance of the department. (12 Marks) Nov/02

[Ans.: (i) Profit for 2001 is `2,28,000, For 2002 loss is `255962; (ii) BEP is 9720 bed days, Increase in fee per
bed days `31.12]

Question 8: PQ Ltd has been offered a choice to buy a machine between A and B
You are required to compute:
(a) Break even point for each of the machines
(b) The level of sales at which both machines earn equal profits
(c) The range of sales at which one is more profitable than the other
The other relevant data is as given below:

Machine A Machine B
Annual output in units 10,000 10,000
Fixed cost 30,000 16,000
Profit at above level of production 30,000 24,000

The market price of the product is expected to be `10 per unit.

[Ans.: 5000 & 4000; 7000]


Question 9: Satish Enterprises are leading exporters of Kid’s Toys. J Ltd. of USA have approached Satish
Enterprises for exporting a special toy named “Jumping Monkey”. The order will be valid for next three years
at 3000 toys per month. The export price of the toy will be $4.
Cost data per toy is as follows: `
Material 60
Labour 25
Variable Overheads 20
Primary packing per toy 15
The toys will be packed in lots of 50 each. For this purpose a special box, which contain the 50 toys will have
to be purchased, cost being `400 per box.
Satish Enterprises will also have to import a special machine for making the toys. The cost of the machine is
`2400000 and duty thereon will be at 12%. The machine will have an effective life of 3 years and depreciation
is to be charged on straight line method. Apart from depreciation, annual fixed overheads are estimated at
`400000 for the first year with 6% increase in the second year. Fixed overheads are incurred uniformly over
the year.
Assuming the average conversion rate to be `50 per $, you are required to:
(i) Prepare monthly and yearly profitability statements for the first year and second year assuming
the production at 3000 toys per month.
(ii) Compute monthly and yearly break-even units in respect of the first year.
(iii) In what contingency can there be a second break-even point for the month and for the year as a
whole?
(iv) Have you any comments to offer on the above? (16 Marks) Nov./99

[Ans.: (i) 108000, 1296000, 106000, 1272000 (ii) 1500 & 18000 (iii) 1505 & 18005]
Question 10: Navbharat Commerce College, Bombay has six sections of B.Com and two sections of M.Com
with 40 and 30 students per sections respectively. The college plans one day pleasure trip around the city for
the students once in an academic session during break to visit park, zoo, planetarium and aquarium.

A transporter used to provide the required number of buses at a flat rate of `700 per bus for the aforesaid
purpose. In addition, a special permit fee of `50 per bus is required to be deposited with city Municipal
Corporation. Each bus is 52 seater. Two seats are reserved for teachers who accompany each bus. Each

CA. Parag Gupta Ph.: +91 11 47665555 Paraggupta_ca@yahoo.co.in Costing & O.R.
World’s largest CA Final student’s consultancy group: http://groups.yahoo.com/group/costingbyparaggupta
Cost Accounting & Management - 17 -

teacher is paid daily allowance of `100 for the day. No other costs in respect of teachers are relevant to the
trip.

The approved caterers of the college supply breakfast, lunch and afternoon tea respectively at `7, `30 and `3
per student.

No entrance fee is charged at the park. Entrance fees come to `5 per student both for the zoo and the
aquarium. As regards planetarium, the authorities charges block entrance fees as under for group of students
of educational institutions depending upon the number of students in a group:

Number of students in a group Block Entrance Fee


Upto 100 `200
101-200 `300
201 & above `450

Cost of prizes to be awarded to the winners in different games being arranged in the park depend upon the
strength of students in a trip. Cost of prizes to be distributed are;

Number of students in a Trip Cost of Prizes


Upto 50 `900
51 – 125 `1050
126 – 150 `1200
151 – 200 `1300
201 – 250 `1400
251 and above `1500

To meet the above costs the college collects `65 from each student who wish to join the trip. The college
releases subsidy of `10 per student in the trip towards it.

You are required to :


(a) Prepare a tabulated statement showing total costs at the levels of 60, 120, 180, 240, and 300
students indicating each item of cost.
(b) Compute average cost per student at each of the above levels.
(c) Calculate the number of students to break even for the trip as the college suffered loss during the
previous year despite 72% of the total students having joined the trip. (19 Marks) May/97

[Ans.: (a) 5850, 9600, 13500, 17400, 21150 (b) 97.50, 80, 75, 72.50, 70.50 (c) BEPs :145, 180, 220, 255]

CA. Parag Gupta Ph.: +91 11 47665555 Paraggupta_ca@yahoo.co.in Costing & O.R.
World’s largest CA Final student’s consultancy group: http://groups.yahoo.com/group/costingbyparaggupta
Activity-based cost
management

Cost Object: It is an item for which cost measurement is required e.g. a product, a job or a customer.

Direct Costs: Those costs that can be specifically & exclusively identified with a particular cost object. Direct
costs can be accurately traced because they can be physically identified with a particular object whereas
indirect costs cannot. Prime cost refers to the direct costs of the product and consists of direct material costs
+ Direct labour costs + Direct expenses. Direct costs are different from Variable costs & may include fixed
costs. Cost of hiring machine for producing a specific product is an example of a direct expense although it
fixed in nature.

Cost tracing
Direct
Costs
Traditional
costing Cost
system Objects
Indirect Cost allocation/
apportionment
Costs ABC
system
Indirect Costs: They can’t be identified specifically and exclusively with a given cost object. The salaries of a
factory supervisors can’t be specifically identified with a particular product hence these are classified as
indirect.

Activities: Activities comprise of units of work or tasks. For example, purchase of materials is an activity
consisting a series of tasks like purchase requisition, advertisement inviting quotations, identification of
suppliers, placement of purchase order, follow-up etc.

Types of Activities: Activities basically fall into four different categories, known as the manufacturing cost
hierarchy. These categories were first identified by Cooper in 1990 and help to determine the type of activity
cost driver required. The categories are:
(i) Unit level activities (a.k.a. Volume related activities): These are activities for which the consumption of
resources can be identified with the number of units produced. E.g. Use of indirect materials, Inspection or
testing of every item produced or say every 100th item produced, Indirect consumables, etc.
(ii) Batch level activities: The costs of some activities (mainly manufacturing support activities) are driven by
the number of batches of units produced. These are activities related to setting up of a batch or a production
run. The costs of such activities vary with the number of batches made, but is fixed for all units within that
batch. E.g. Production scheduling, Material movement, Machine set up costs, Inspection of products – like
first item of every batch, etc.
(iii) Product-sustaining activities or service sustaining activities: The costs of some activities (often once only
activities) are driven by the creation of a new product line and its maintenance. These are activities performed
to support different products in the product line. E.g. Designing the product, Producing parts to a certain
specification, Advertising costs, if advertisement is for individual products, etc.

CA. Parag Gupta Ph.: +91 11 47665555 Paraggupta_ca@yahoo.co.in Costing & O.R.
World’s largest CA Final student’s consultancy group: http://groups.yahoo.com/group/costingbyparaggupta
Cost Accounting & Management - 19 -

(iv) Facility-sustaining or business-sustaining activities: These are activities necessary for sustaining the
manufacturing process and cannot be directly attributed to individual products. E.g. Maintenance of buildings,
Plant security, Production manager’s salaries, Advertising campaigns promoting the co., etc.

Value Added and Non-value Added activities


Value Added Activites(VA) Non Value Added Activites (NVA)
These are activities necessary for the performance of These are additional and extraneous activities,
the process. not fully necessary for the performance of the
process.
These represent work that is valued by the external These represent work that is not valued by the
or internal customer. external or internal customer.
They improve the quality or function of a product. NVA activities do not improve the quality or
Hence, the customers are usually willing to pay for function of a product or service but they can
the service. VA activities result in “Cost” and not in adversely affect costs and prices. NVA
losses. activities create waste, result in delay of some
sort, add cost to the products or services for
which the customer is not willing to pay.
Example: Making product more versatile for certain Examples: Moving materials and machine set
other uses. up for a production run.

Cost Driver: It is the factor that causes a change in the cost of an activity. Instead of using the term
‘allocation bases’ or ‘overhead allocation rates’ the term cost driver is used in ABC system. They are
classified into:
 Resource Cost Driver: It is a measure of the quantity of resource consumed by activity. It is used to
assign the cost of a resource to an activity or cost pool. An example of a resource cost driver is the
percentage of total square feet occupied by an activity. This factor is used to allocate a portion of the
cost of operating the facilities to the activity.

 Activity Cost Driver: It is a measure of the frequency and intensity of demand, placed on activities
by cost objects. It is used to assign activity costs to cost objects. Activity cost drivers can be
transaction drivers (e.g. No. of purchase orders processed, no. of customer orders processed, etc.)
as well as duration drivers (it represent amount of time required to perform an activity e.g. Setup
hours, inspection hours, etc.).

Common activities and associated cost drivers:


Major Activities Associated Costs Cost Driver
Processing purchase orders Labour cost for workers determining order Number of purchase
for materials and parts quantities, contracting vendors, and preparing orders processed
purchase orders
Handling material and parts Labour cost for workers handling material and Number of material
parts, depreciations of equipment used to move requisitions
material and parts (e.g. depreciation of fork lift
trucks), etc.
Inspecting incoming material Labour cost for workers performing inspections, Number of receipts
and parts depreciation of equipment used to test strength
of materials, tolerances, etc.
Inspecting finished goods Labour cost for finished goods inspectors, Number of assembly
depreciation of equipment used to test whether labour hours
finished goods meet customer specifications,
etc.
Setting up equipment Labour cost for workers involved in setups, Number of setups
depreciation of equipment used to adjust
equipment
Producing goods using Depreciation of manufacturing equipment Number of machine
manufacturing equipment hours
Supervising assembly workers Salary of assembly supervisors Number of assembly
labour hours
Packing customer orders Labour cost for packing workers, cost of Number of boxes
packing materials, etc. shipped
CA. Parag Gupta Ph.: +91 11 47665555 Paraggupta_ca@yahoo.co.in Costing & O.R.
World’s largest CA Final student’s consultancy group: http://groups.yahoo.com/group/costingbyparaggupta
Activity based cost management - 20 -

Traditional Cost Accounting


It arbitrarily allocates overheads to the cost objects. Total Company’s overhead is allocated based on volume
based measure (taking time as base factor) i.e. labour hours or machine hours. Here the main assumption is
that there is a relationship between overhead & volume based measure.

Activity Based Costing


ABC is not a method of costing, but a technique for managing the organization better. It is a one-off exercise
which measures the cost and performance of activities, resources and the objects which consume them in
order to generate more accurate and meaningful information for decision-making. ABM draws on ABC to
provide management reporting and decision making.

It is more accurate cost management methodology. It focuses on indirect costs (overhead). It traces rather
than allocates each expense category to the particular cost object. It converts “indirect” expenses to “direct”.

ABC Basic Premise


o Cost objects consume activities.
o Activities consume resources.
o This consumption of resources is what derives costs.
o Understanding this relationship is critical to successfully managing overhead.
Cost Drivers

Volume Based Non-Volume Based


They assume that a product’s consumption They are in contrast of volume based cost
of overhead is directly related to units drivers. Non-volume based activities are not
produced. performed each time a unit of the product or
E.g. Direct Machine hours, If volume is service is produced. E.g. number of
increased by 10%, machine house will production runs for production scheduling &
increase by 10% hence energy cost will the number of purchase orders for the
increase by 10%. purchasing activity.

Remember, Using only volume-based cost drivers to assign non-volume related overhead costs can result in
the reporting of distorted product costs.

Product Diversity - Difference in product size, product complexity, size of batches and set-up times cause
product diversity.

Product cost distortion occurs when both of these conditions occur:


a) Non-Volume based overhead costs are a large proportion of total overhead costs.
b) Product diversity applies.

When & Why to use ABC


o Overhead is high-New production techniques have resulted in the increase of the proportion of
support service costs in the total cost of delivering value to customers. ABC improves the accuracy of
accounting for support service costs.
o Products are diverse-There is product and customer proliferation. Demand on resources by products /
customers differ among product / customers. Therefore, product / customer profitability can be
measured reasonably accurately, only if consumption of resources can be traced to each individual
product / customer
o Costs of errors are high-The costs associated with bad decisions have increased substantially.
o Competition is stiff- Fierce competitive pressure has resulted in shrinking profit margin. ABC helps to
estimate cost of individual product or service more accurately. This helps to formulate appropriate
marketing / corporate strategy.

Steps involved in Activity Based Costing:

CA. Parag Gupta Ph.: +91 11 47665555 Paraggupta_ca@yahoo.co.in Costing & O.R.
World’s largest CA Final student’s consultancy group: http://groups.yahoo.com/group/costingbyparaggupta
Cost Accounting & Management - 21 -

Step 1: Identify the various Activities within the organization.


Only significant activities shall be considered for decision-making purposes.

Step 2: Relate the Overheads to the Activities using Resources Cost Drivers
 Overheads will be related to Support and Primary Activities
 Resources Cost Drivers, i.e. the quantity of resources used by an activity is used for this purpose.
 All costs will be identified under the activities, thus creating Activity Cost Pools/Cost Buckets.

Step 3: Determine the Activity Cost Drivers for each Activity Cost Pool
 Activity cost drivers used to relate the overheads collected in the cost pools to cost objects (products)
should be determined.
 This is based on the factor that drives the consumption of the activity, i.e. the answer to the question:
what causes the activity to incur costs? For example in production scheduling, the driver will be
number of batches ordered.
Activity Cost Pool
Step 4: Calculate Activity Cost Driver Rate (i.e. ) & relate costs to products.
Activity Cost Driver
 Activity Cost Driver Rates are computed for each activity, just like overhead absorption rates.
 The rates will be multiplied by the different amounts of each activity that each product/other cost
object consumes, so as to ascertain its cost.

A comparison of traditional & ABC systems

Traditional Absorption Costing Activity Based Costing


Overheads are first related to departments Overheads are first related to activities or grouped into
cost centres (Production and Service Cost Cost Pools (rather than departments).
Centres)
Only two types of activities viz. Unit Level All levels of activities in the manufacturing cost
Activities and Facility Level Activities are hierarchy viz. Unit Level, Batch Level, Product Level
identified. and Facility Level are identified.
Direct labour & machine hours are the Many different types of second stage cost drivers are
allocation bases that are normally used. used, including non-volume-based drivers.
This method relates overheads to cost centres This method relates overheads to the causal factor i.e.
i.e. locations. It is not realistic of the behaviour driver. Thus, it is more realistic of cost behaviour.
of costs.
Overhead Rates can be used to ascertain Activity Cost Driver Rates can be used to ascertain
cost of products only. cost of products and also cost of other cost objects
such as customer segments, distribution channels. etc.

Activity-based Management

Activity-based management (ABM) is a method of identifying and evaluating activities that a business
performs using activity-based costing to carry out a value chain analysis or a re-engineering initiative to
improve strategic and operational decisions in an organization. Activity-based costing establishes
relationships between overhead costs and activities so that overhead costs can be more precisely allocated to
products, services, or customer segments. Activity-based management focuses on managing activities to
reduce costs and improve customer value. To implement ABM only 1st three of the four stages for designing
an activity-based product costing system are required. They are:
 Identify the various Activities within the organization.
 Relate the Overheads to the Activities using Resources Cost Drivers
 Determine the Activity Cost Drivers for each Activity Cost Pool
Thus final stage of assigning activity costs to products can be omitted & ABC can solely be adopted for cost
management without activity-based product costing. Alternative, organizations can design an activity based
system that incorporates both ABM & ABC.

Activity Based Costing Activity Based Cost Management


ABC refers to the technique of determining the It refers to the management philosophy that
costs of activities and the cost of output that focuses on the planning, execution and
CA. Parag Gupta Ph.: +91 11 47665555 Paraggupta_ca@yahoo.co.in Costing & O.R.
World’s largest CA Final student’s consultancy group: http://groups.yahoo.com/group/costingbyparaggupta
Activity based cost management - 22 -

those activities produce. measurement of activities as the key to


competitive advantage.
The aim of ABC is to generate improved cost The ABM is a much broader concept. Its aim is
data for use in managing a company’s to use information generated by ABC, for
activities. effective business processes and profitability.

Question 1: Explain the concept of cost drivers indicate what you will consider as cost drivers for the
following business function:
Research & development; and Customer service. (4 Marks) Nov./98
Question 2: What is activity based costing? (4 Marks) May/00
Question 3: Explain the concept of activity based costing. How ABC system supports corporate
strategy? (4 Marks) Nov./05
Question 4: Differentiate between ‘Value-added’ and ‘Non-value-added’ activities in the context of
Activity-based costing. Give examples of Value-added and Non-value-added activities. (4 Marks) May/06
Question 5: What are the areas in which activity based information is used for decision making?
(4 Marks) Nov./00
Question 6: What is the fundamental difference between Activity Based Costing System (ABC) and
Traditional Costing System? Why more and more organizations in both the manufacturing and non-
manufacturing industries are adopting ABC? (10 Marks) Nov./07
Question 7: Give two examples for each of the following categories in activity based costing:
(i) Unit Level activities
(ii) Batch Level activities
(iii) Product Level activities
(iv) Facility Level activities (3 Marks) Nov./02[Adapted] & (4 Marks) Nov/06
Question 8: Why are conventional product costing systems more likely to distort product costs in highly
automated plants? How do activity based costing deal with such a situation? (4 Marks) May/06
Question 9: Traditional Ltd. is a manufacturer of a range of goods. The cost structure of its different products
is as follows:

Particulars Product A Product B Product C


Direct materials 50 40 40 `/u
Direct labour @ `10 `/hour 30 40 50 `/u
Production overheads 30 40 50 `/u
Total Cost 110 120 140 `/u
Quantity produced 10,000 20,000 30,000 Units

Traditional Ltd. was absorbing overheads on the basis of direct labour hours. A newly appointed management
accountant has suggested that the company should introduce ABC system and has identified cost drivers and
cost pools as follows:

Activity Cost Pool Cost Driver Associated Cost


Stores Receiving Purchase Requisitions 2,96,000
Inspection Number of Production runs 8,94,000
Despatch Orders Executed 2,10,000
Machine Setup Number of setups 12,00,000

The following information is also supplied:

Details Product A Product B Product C


No. of Setups 360 390 450
No. of Orders Executed 180 270 300
No. of Production runs 750 1,050 1,200
No. of Purchase Requisitions 300 450 500

CA. Parag Gupta Ph.: +91 11 47665555 Paraggupta_ca@yahoo.co.in Costing & O.R.
World’s largest CA Final student’s consultancy group: http://groups.yahoo.com/group/costingbyparaggupta
Cost Accounting & Management - 23 -

You are required to calculate activity based production cost of all the three products.

[Ans.: A - `1504940; B – `2485060; C – `3710000] (5 Marks)-June/09-N.C.


Question 10: A company manufactures three types of products namely P, Q & R. The data relating to a
period are as under:

P Q R
Machine hour per unit 10 18 14
Direct labour hour per unit @ `20 4 12 8
Direct material per unit ` 90 80 120
Production (units) 3000 5000 20000

Currently the company uses traditional costing method and absorbs all production overheads on the basis of
machine hours. The machine hour rate of overhead is `6 per hour.

The company proposes to use activity based costing system and the activity analysis is as under:

P Q R
Batch size (units) 150 500 1000
Number of purchase orders per batch 3 10 8
Number of inspections per batch 5 4 3

The total production overheads are analyzed as under:

Machine set up costs 20%


Machine operation costs 30%
Inspection costs 40%
Material procurement related costs 10%

Required:
i. Calculate the cost per unit of each product using traditional method of absorbing all production
overheads on the basis of machine hours.
ii. Calculate the cost per unit of product using activity based costing principles.

[Ans.: (i) 230, 428, 364 (ii) 427, 425, 335.20] (7 Marks) Nov./08-O.C.
Question 11: The following are Product Nova Shaft’s data for next year budget:

Activity Cost Driver Cost Driver Cost pool


volume/year
Purchasing Purchase orders 1500 `75000
Setting Batches produced 2800 `112000
Materials handling Materials movements 8000 `96000
Inspection Batches produced 2800 `70000
Machining costs Machine hours 50000 `150000
Purchase orders 25
Output 15000 units
Production batch size 100 units
Materials movement per batch 6
Machine hours per unit 0.1

Required:
(i) Calculate the budgeted overhead costs using activity based costing principles.

(ii) Calculate the budgeted overhead Costs using absorption costing (absorb overhead using machine hours).

(iii) How can the company reduce the ABC for Product Nova Shaft? (9 Marks) June/09-O.C.

CA. Parag Gupta Ph.: +91 11 47665555 Paraggupta_ca@yahoo.co.in Costing & O.R.
World’s largest CA Final student’s consultancy group: http://groups.yahoo.com/group/costingbyparaggupta
Activity based cost management - 24 -

[Ans.: (i) `26300; (ii) `15090


(iii) Ways in which the company can reduce the ABC for product Nova Shaft:
 Reduce the number of batches by increasing the batch size which will then reduce the setting up
overhead, materials handling and inspection costs.
 Reduce the number of purchase orders
 Innovate ways of speeding up production so that the machining hours are reduced.]
Question 12: Biscuit Ltd. Manufactures 3 types of biscuits, A, B and C, in a fully mechanised factory. The
company has been following conventional method of costing and wishes to shift to Activity Based Costing
System and therefore wishes to have the following data presented under both the systems for the month.

Inspection cost `p.m. 73,000


Machine – Repairs & Maintenance `p.m. 1,42,000
Dye cost `p.m. 10,250
Selling overheads `p.m. 1,62,000

Product A B C
Prime cost (`per unit) 12 9 8
Selling price (`Per unit) 18 14 12
Gross production (units/production run) 2,520 2,810 3,010
No. of defective (units / production run) 20 10 10
C
Inspection (No. of hours / production run) 3 4 4
Dye cost / production run (`) 200 300 250
No. of machine hours / production run 20 12 30
Sales – No. of units / month 25,000 56,000 27,000

The following additional information is given:

(i) No accumulation of inventory is considered. All good units produced are sold.

(ii) All manufacturing and selling overheads are conventionally allocated on the basis of units sold.

(iii) Product A needs no advertisement. Due to its nutritive value, it is readily consumed by diabetic patients of
a hospital. Advertisement costs included in the total selling overhead is `83,000.

(iv) Product B needs to be specially packed before being sold, so that it meets competition. `54,000 was the
amount spent for the month in specially packing B, and this has been included in the total selling overhead
cost given.

You are required to present product-wise profitability of statements under the conventional system and the
ABC system and accordingly rank the products. (11 Marks) May/08

[Ans.: Conventional : Net profit(Rank) is A-57959(II), B- 77403(I) & C- 10467(III), ABC : Net profit(Rank) is A-
84813 (I), B-61237 (II) & C- (220)(III)]
Question 13: Bombay steel Ltd. manufacture four products, namely A,B,C and D, using the same plant and
process. The following information relates to a production period:

Product Volume Material cost Direct labour Machine Labour cost


per unit (`) per unit time per unit per unit
A 500 5 ½hours ¼hours 3
B 5,000 5 ½hours ¼hours 3
C 600 16 2 hours 1hours 12
D 7,000 17 1½hours 1½hours 9

Total production overhead recovered by the cost accounting system is analysed under the following heading:

Factory overhead applicable to machine-oriented activity 37,424


CA. Parag Gupta Ph.: +91 11 47665555 Paraggupta_ca@yahoo.co.in Costing & O.R.
World’s largest CA Final student’s consultancy group: http://groups.yahoo.com/group/costingbyparaggupta
Cost Accounting & Management - 25 -

Set-up costs are 4,355


Cost of ordering materials 1,920
Handling materials 7,580
Administration for spare parts 8,600

These overhead costs are absorbed products on a machine hour rate of `4.80 per hour giving an overhead
cost per product of:

A=`1.20 B=`1.20 C=`4.80 D=`7.20

However, investigation into the production overhead activities for the period reveals the following totals:

Product Number of set- Number of Number of times Number of


ups material orders material was handled spare parts
A 1 1 2 2
B 6 4 10 5
C 2 1 3 1
D 8 4 12 4
You are required:
(I) to compute an overhead cost per unit per product using activity based costing, tracing overheads to
production units by means of cost drive.
(II) to comment briefly on the difference disclosed between overheads traced by the present system and
those traced by activity based costing.

[Ans.: Value of overhead in Traditional: 1.2, 1.2, 4.8, 7.2; Value of overhead in ABC: 5.63, 2.49, 6.76, 5.79]
Question 14: Family Store wants information about the profitability of individual product lines: Soft drinks,
Fresh produce and Packaged food. Family store provides the following data for the year 2002-03 for each
product line:

Soft drinks Fresh Packaged


produce food
Revenues `7,93,500 `21,00,600 `12,09,900
Cost of goods sold `6,00,000 `15,00,000 `9,00,000
Cost of bottles returned `12,000 `0 `0
Number of purchase orders placed 360 840 360
Number of deliveries received 300 2,190 660
Hours of shelf-stocking time 540 5,400 2,700
Items sold 1,26,000 11,04,000 3,06,000

Family store also provides the following information for the year 2002-03:

Activity Description of Activity Total cost Cost-allocation Base


Bottles returns Returning of empty `12,000 Direct tracing to soft
bottles drink line
Ordering Placing of orders for `1,56,000 1,560 purchase orders
purchases
Delivery Physical delivery and `2,52,000 3,150 deliveries
receipt of goods
Shelf stocking Stocking of goods on `1,72,800 8,640 hours of shelf-
store shelves and on- stocking time
going restocking
Customer Support Assistance provided to `3,07,200 15,36,000 items sold
customers including
check-out

Required:

CA. Parag Gupta Ph.: +91 11 47665555 Paraggupta_ca@yahoo.co.in Costing & O.R.
World’s largest CA Final student’s consultancy group: http://groups.yahoo.com/group/costingbyparaggupta
Activity based cost management - 26 -

i) Family store currently allocates support cost (all cost other than cost of goods sold) to product lines
on the basis of cost of goods sold of each product line. Calculate the operating income and operating
income as a % of revenues for each product line.
ii) If Family Store allocates support costs (all costs other than cost of goods sold) to product lines using
and activity based costing system, calculate the operating income and operating income as a % of
revenues for each product line.
iii) Comment on your answers in requirements (i) and (ii).

[Ans.: (i) `13500, `150600, `39900; 1.70%, 7.17%, 3.30% (ii) `85500, `12600, `105900; 10.78%, 0.60%,
8.75%] (12 Marks) May/03-PEII, (11 Marks) Nov./10-N.C.[Adapted]
Question 15: Having attached a CIMA course on activity based costing (ABC) you decide to experiment by
applying the principles of ABC to the four products currently made and sold by your company. Details of the
four products and relevant information are giving below for one period:

Product A B C D
Output in units 120 100 80 120
Cost per unit: ` ` ` `
Directs material 40 50 30 60
Direct labour 28 21 14 21
Machine hours (per unit) 4 3 2 3

The four products are similar and are usually produced in production runs of 20 units and sold in batches of
10 units. The production overhead is currently absorbed by using a machine hour rate, and the production
overhead for the period has been analysed as follows:
(`)
Machine department costs(rent, business, rates, depreciation and supervision) 10,430
Set-up costs 5,250
Stores receiving 3,600
Inspection / Quality control 2,100
Materials handling and dispatch 4,620

You have ascertained that the” cost drivers” to be used are as listed below for overhead drivers” to costs
Shown:

Cost Cost Driver


Set-up costs Number of production runs
Store receiving Requisitions raised
Inspection / quality control Number of production runs
Materials handling and dispatch Orders executed

The number of requisitions raised on the stores was 20 for each product and the number of orders executed
was 42, each order being for a batch of 10 of a product.

You are required:

a) to calculate the total cost for product if all overhead costs are on machine hour basis;
b) to calculate the total costs for product, using activity based costing ;
c) to calculate and list the unit product costs from your figures in (a) and (b) above, to show the
differences and to comment briefly on any conclusions which may be drawn which could have pricing
and profit implications. (CIMA London Nov/91) & (12 Marks) CA PEII May/05-Adapted

[Ans.: (a) Machine hour basis: 17760, 13100, 6720, 16920; (b) ABC: 16331, 13257, 7984, 16928; (c)
Machine hour basis: 148, 131, 84, 141; ABC:136.09, 132.57, 99.80,141.07,]
Question 16: XYZ Ltd. manufactures four products, namely A, B, C and D using the same plant and process.
The following information relates to a production period:

CA. Parag Gupta Ph.: +91 11 47665555 Paraggupta_ca@yahoo.co.in Costing & O.R.
World’s largest CA Final student’s consultancy group: http://groups.yahoo.com/group/costingbyparaggupta
Cost Accounting & Management - 27 -

Product A B C D
Output in units 720 600 480 504
Cost per unit: ` ` ` `
Direct Material 42 45 40 48
Direct labour 10 9 7 8
Machine hours per unit 4 hrs. 3 hrs. 2 hrs. 1 hr

The four products are similar and are usually produced in production runs of 24 units and sold in batches of
12 units. Using machine hour rate currently absorbs the production overheads. The total overheads incurred
by the company for the period is as follows:

`
Machine operation and Maintenance cost 63,000
Setup costs 20,000
Store receiving 15,000
Inspection 10,000
Material handling and dispatch 2,592

During the period the following cost drivers are to be used for the overhead cost:

Cost Cost driver


Setup cost No. of production runs
Store receiving Requisition raised
Inspection No. of production runs
Material handling and dispatch Orders executed

It is also determined that:

· Machine operation and maintenance cost should be apportioned between setup cost, store receiving and
inspection activity in 4:3:2.

· Number of requisition raised on store is 50 for each product and the no. of order executed is 192, each order
being for a batch of 12 of a product.

Required:
(i) Calculate the total cost of each product, if all overhead costs are absorbed on machine hour rate basis.
(ii) Calculate the total cost of each product using activity base costing.
(iii) Comment briefly on differences disclosed between overheads traced by present system and those traced
by activity base costing. (11 Marks) Nov/04

[Ans.: (a) A – `89280, B – `64800, C – `39840, D – `37296, (b) A – `69750, B – `60825, C - `47100, D –
`53541]
Question 17 [Activity Based Budgeting]: A bank offers three products, viz., deposits, Loans and Credit
Cards. The bank has selected 4 activities for a detailed budgeting exercise, following activity based costing
methods.

The bank wants to know the productwise total cost per unit for the selected activities, so that prices may be
fixed accordingly.

The following information is made available to formulate the budget

Activity Present Estimation for the budget period


Cost (`)
(i) ATM Services:
(a) Machine Maintenance 4,00,000 (all fixed; no change)
(b) Rents 2,00,000 (fully fixed; no change)
(c) Currency Replenishment 1,00,000 (expected to double during budget period)
Cost (This activity is driven by no. of ATM
CA. Parag Gupta Ph.: +91 11 47665555 Paraggupta_ca@yahoo.co.in Costing & O.R.
World’s largest CA Final student’s consultancy group: http://groups.yahoo.com/group/costingbyparaggupta
Activity based cost management - 28 -

transactions)
(ii) Computer Processing 5,00,000 (Half this amount is fixed and no change is
expected )
(The Variable portion is expected to
increase to three times the current level).
This activity is driven by the number of
computer transactions.
(iii) Issuing Statements 18,00,000 Presently, 3 lac statements are made. In
the budget period, 5 lac statements are
expected;
For every increase of one lac statements,
one lac rupees is the budgeted increase
(this activity is driven by the number of
Statements)
(iv) Customer Inquiries 2,00,000 Estimated to increase by 80% during the
budget period. (This activity is driven by
telephone minutes.)

The activity drivers and their budgeted quantities are given below:

Deposits Loans Credit Cards


No. of ATM Transactions 1,50,000 - 50,000
No of Computer Processing 15,00,000 2,00,000 3,00,000
No. of Statements to be issued 3,50,000 50,000 1,00,000
Telephone Minutes 3,60,000 1,80,000 1,80,000

The bank budgets a volume of 58,600 deposit accounts, 13,000 loan accounts, and 14,000 Credit Card
accounts.
You are required to:
i) Calculate the budgeted rate for each activity.
ii) Prepare the budgeted cost statement activity wise.
Find the budgeted product cost per account for each product using (i) and (ii) above. (12 Marks) Nov./09-N.C.

[Ans.: (a) 4, 0.50, 4, 0.50; (b) 800000, 1000000, 2000000, 360000 (c) 50, 30, 60]

Question 18: XYZ Ltd. produces and sells sophisticated glass items – ‘A’ and ‘B’. In connection with
both the products the following informations are revealed from the cost records for the month February,
2008:

Product A B
Output (in units) 60,000 15,000
Sales (`) 37,80,000 20,55,000
Cost structure:
Direct material (`per unit) 18.75 45.00
Direct Wages (`per unit) 10.00 13.00
Direct labour hours 30,000 hours 9,750 hours
No. of quantity produced per batch 240 50
Setup time per batch 2 hours 5 hours

The Indirect costs for the month are as under:

`
Cleaning and maintenance wages 2,70,000
Designing Costs 4,50,000
Set up costs 3,00,000
Manufacturing operation’s costs 6,37,500
Shipment costs 81,000
Distribution costs 3,91,500
Factory administration costs 2,55,000
CA. Parag Gupta Ph.: +91 11 47665555 Paraggupta_ca@yahoo.co.in Costing & O.R.
World’s largest CA Final student’s consultancy group: http://groups.yahoo.com/group/costingbyparaggupta
Cost Accounting & Management - 29 -

At present the company adopts the policy to absorb indirect costs applying direct labour hour basis and
enjoying a good position in the market with regard to Product B, but facing a stiff price competition with
regard to Product A. The cost Accountant of the company, after making a rigorous analysis of the data,
decided to shift from the absorption technique based on direct labour hours to activity cost driver basis
and also to treat cleaning and maintenance wages as direct cost.

The cost accountant identified `1,20,000 for product A and the balance of cleaning and maintenance
wages for Product B.

The data relevant to activities and products are as follows:

Product Product
Activity Cost driver A B
Designing: Square feet 30 sq. ft. 70 sq. ft.
Manufacturing Moulding machine 9,000 hrs. 3,750 hrs.
operation’s: hours
Shipment: Number of Shipments 100 100
Distribution: Cubic feet 45,000 cu. ft. 22,500 cu. ft.
Setup of moulding Setup hours
machine:
Factory administration: Direct labour hours

You are required:

(i) to compute the total manufacturing cost and profits of both the products by applying direct labour
basis of absorption, assuming cleaning and maintenance cost as indirect,
(ii) to compute the total manufacturing cost and profits of both the products by applying activity based
costing, assuming cleaning and maintenance cost as indirect
(iii) to compare the results obtained from (i) and (ii) and give your opinion on the decision of cost accountant.

[Ans.: Cost per unit: Traditional- 58.75, 97; ABC- 51.38, 126.48; Profit per unit: Traditional- 4.25, 40; ABC-
11.62, 10.52] (10 Marks) May/08-PEII

[Hint: Cleaning and maintenance cost is to be apportioned treating labour hours as cost drivers.]
Question 19: During the last 20 years, KL Ltd’s manufacturing operation has become increasingly automated
with Computer-controlled robots replacing operators. KL currently manufactures over 100 products of varying
levels of design complexity. A single, plant-wide overhead absorption rate, based on direct labour hours, is
used to absorb overhead costs.

In the quarter ended March, KL’s manufacturing overhead costs were:

(`‘000)
Equipment operation expenses 125
Equipment maintenance expense 25
Wages paid to technicians 85
Wages paid to Store men 35
Wages paid to despatch staff 40
310

During the quarter, the company reviewed the Cost Accounting System and concluded that absorbing
overhead costs to individual products on a labour hour absorption basis is meaningless. Overhead costs
should be attributed to products using an Activity Based Costing (ABC) system and the following was
identified as the most significant activities:

(i) Receiving component consignments from suppliers


(ii) Setting up equipment for production runs
(iii) Quality inspections
(iv) Dispatching goods as per customer’s orders.
CA. Parag Gupta Ph.: +91 11 47665555 Paraggupta_ca@yahoo.co.in Costing & O.R.
World’s largest CA Final student’s consultancy group: http://groups.yahoo.com/group/costingbyparaggupta
Activity based cost management - 30 -

It was further observed that, in the short-term, KL’s overheads are 40% fixed and 60% variable.
Approximately, half the variable overheads vary in relating to direct labour hours worked and half
vary in relation to the number of quality inspections.Note

Equipment operation and maintenance expenses are apportioned as:


Component stores 15%, manufacturing 70% and goods dispatch 15%

Technician’s wages are apportioned as :


Equipment maintenance 30%, set up equipment for production runs 40% and quality inspections 30%.

During the quarter :


(i) a total of 2000 direct labour hours were worked (paid at `12 per hr.)
(ii) 980 components consignments were received from suppliers
(iii) 1020 production runs were set up
(iv) 640 quality inspections were carried out
(v) 420 orders were dispatched to customers.

KL’s production during the quarter included components R, S and T. The following information is available:

Component Component Component


R S T
Direct labour Hrs worked 25 480 50
Direct Material `1,200 `2,900 `1,800
Component Consignments Recd. 42 24 28
Production runs 16 18 12
Quality Inspections 10 8 18
Orders (goods) dispatched 22 85 46
Quantity produced 560 12,800 2,400

Required:
(1) Calculate the unit cost of R, S and T components, using KL’s existing cost accounting system.
(2) Explain how an ABC system would be developed using the information given. Calculate the unit cost of
components R, S and T using ABC system. (11 Marks) May/05 & CIMA Stage 3 [Adapted]

[Ans.: (1) Cost per unit (Under existing cost system)


R S T
9.60 6.49 4.23

(2) Cost per unit (Under ABC cost system)


R S T
18.682 2.08 5.82]
[Note.: Since this question is only a part question of question asked in CIMA, this paragraph is of no use.]

Question 20: A company manufactures several products of varying levels of designs and models. It uses a
single overhead recovery rate based on direct labour hours. The overheads incurred by the company in the
half of the year are as under:
`
Machine operation expenses
10,12,500
Machine maintenance expenses
1,87,500
Salaries of technical staff
6,37,500
Wages and salaries of stores
2,62,500
staff

During this period, the company introduced activity based costing system and the following significant
activities were identified:
- receiving materials and components
- set up of machines for production runs
- quality inspection.

CA. Parag Gupta Ph.: +91 11 47665555 Paraggupta_ca@yahoo.co.in Costing & O.R.
World’s largest CA Final student’s consultancy group: http://groups.yahoo.com/group/costingbyparaggupta
Cost Accounting & Management - 31 -

It is determined that:
- The machine operation and machine maintenance expenses should be apportioned between stores and
production activity in 20:80 ratio.
- The technical staff salaries should be apportioned between machine maintenance, set up and quality
inspection in 30:40:30 ratio.

The consumption of activities during the period under review are as under:

Direct labour hours worked 40,000


Direct wage rate `6 per hour
Production set-ups 2,040
Material and component consignments received from suppliers 1,960
Number of quality inspections carried out 1,280

The data relating to two products manufactured by the company during the period are as under:
Products
P Q
`6,000 4,000
Direct materials costs
960 100
Direct labour hours
48 52
Direct material consignments received
36 24
Production runs
30 10
Number of quality inspections done
15,000 5,000
Quantity produced (units)

A potential customer has approached the company for the supply of 24,000 units of a component K to be
delivered in lots of 3,000 units per quarter. The job will involve an initial design cost of `60,000 and the
manufacture will involve the following per quarter:

Direct materials costs `12,000


Direct labour hours 300
Production runs 6
Inspections 24
Number of consignments of direct materials to be received 20

The company desires a mark up of 25% on cost.

Required:

(i) Calculate the cost of product P and Q based on the existing system of single overhead recovery rate.
(ii) Determine the cost of product P and Q using activity based costing system.
(iii) Compute the sales value per quarter of component K using activity based costing system.

[Ans.: Product P Product Q


(i) Cost per unit (`) 4.144 1.97
(ii) Cost per unit (`) 3.58 7.31
(iii) Selling price per unit of K is `14.34] (12 Marks) May/03 & May/07-RTP
Question 21: Amar and Naveen architects, have been using a simplified costing system in which all
professional labour costs are included in a single direct cost category, professional labour and all overhead
costs are included in a single indirect cost category, professional support, and allocated to jobs by using
professional labour hours as the allocation base. Consider two clients: Host Restaurant, which required 25
hours of design work for a new addition, and Pizza Hut, which required plans for a new floor that took 40
hours to draw. The firm has two partners, who each earn a salary of `1,50,000 a year, and four associates,
who each earn `60,000 per year. Each professional has 1,500 billable hours per year. The professional
support is `10,80,000, which consists of `7,00,000 of design support and `3,80,000 of staff support. Host

CA. Parag Gupta Ph.: +91 11 47665555 Paraggupta_ca@yahoo.co.in Costing & O.R.
World’s largest CA Final student’s consultancy group: http://groups.yahoo.com/group/costingbyparaggupta
Activity based cost management - 32 -

Restaurant job required five hours of partner time and 20 hours of associate time. Pizza Hut job required 30
hours of partner time and 10 hours of associate time.

Required:

(i) Prepare job cost sheets for Host Restaurant and Pizza Hut using a simplified costing system with one
direct and one indirect cost pool.
(ii) Prepare job cost sheets for the two clients, using an activity based costing system with two direct cost
categories – partner labour and associate labour – and two indirect cost categories – design support and staff
support. Use professional labour in Rupees as the cost allocation base for design support and professional
labour hours for staff support.
(iii) Determine the amount by which each job was under – or overcosted, using the simplified costing system.

[Ans.: (i) Host Restaurant – `4500 & Pizza Hut – `7200; (ii) Host Restaurant – `4041 & Pizza Hut – `9496;
(iii) The simplified costing system overcosted Host Restaurant job by `459 and undercosted Pizza Hut job by
`2,296.] RTP-May/05
Question 22: ABC electronics makes audio player model ‘AB 100’. It has 80 components. ABC sells 10,000
units each month at `3,000 per unit. The cost of manufacturing is `2,000 per unit or `200 lakhs per month for
the production of 10,000 units. Monthly manufacturing costs incurred are as follows:

(`Lakhs)
Direct material costs 100.00
Direct manufacturing labour costs 20.00
Machining costs 20.00
Testing costs 25.00
Rework costs 15.00
Ordering costs 0.20
Engineering costs 19.80
200.00

Labour is paid on piece rate basis, therefore, ABC considers direct manufacturing labour cost as variable
cost.

The following additional information is available for ‘AB 100’


(i) Testing and inspection time per unit is 2 hours.
(ii) 10 per cent of ‘AB 100’ manufactured are reworked.
(iii) It currently takes 1 hour to manufacture each unit of ‘AB 100’
(iv) ABC places two orders per month for each component. A different supplier supplies each component.
ABC has identified activity cost pools and cost drivers for each activity. The cost per unit of the cost driver for
each activity cost pool is follows:

Manufacturing Description of activity Cost driver Cost per unit of


Activity cost driver
1. Machine costs Machining components Machine hours of `200
capacity
2. Testing costs Testing components and Testing hours `125
finished products. (Each unit of
‘AB 100’ is tested individually)
3. Rework costs Correcting and fixing errors Units of ‘AB 100’ `1,500 per unit
and defects reworked
4. Ordering costs Ordering of components Number of orders `125 per order
5. Engineering Designing and managing of Engineering hours `1,980 per
costs products and processes engineering hour

Over a long-run horizon, each of the overhead costs described above vary with chosen cost drivers. In
response to competitive pressure ABC must reduce the price of its product to `2600 and to reduce the cost by
at least `400 per unit. ABC does not anticipate increase in sales due to price reduction. However, if it does
not reduce price it will not be able to maintain the current sales level.

CA. Parag Gupta Ph.: +91 11 47665555 Paraggupta_ca@yahoo.co.in Costing & O.R.
World’s largest CA Final student’s consultancy group: http://groups.yahoo.com/group/costingbyparaggupta
Cost Accounting & Management - 33 -

Cost reduction on the existing model is almost impossible. Therefore, ABC has decided to replace ‘AB 100’ by
a new model ‘AB 200’, which is a modified versions of ‘AB 100’. The expected effect of design modifications
are as follows:
(i) The member of components will be reduced to 50.
(ii) Direct material costs to be lower by `200 per unit.
(iii) Direct manufacturing labour costs to be lower by `20 per unit.
(iv) Machining time required to be lower by 20 per cent.
(v) Testing time required to be lower by 20 per cent.
(vi) Rework to decline to 5 per cent.
(vii) Machining capacity and engineering hours capacity to remain the same.
ABC currently outsource the rework on defective units.
Required:
(i) Compare the manufacturing cost per unit of ‘AB 100’ and ‘AB 200’.
(ii) Determine the immediate effect of design change and pricing decision on the operating to apply to ‘AB
200’.
Ignore income tax, Assume that the cost per unit of each cost driver for ‘AB 100’ continues to apply to ‘AB
200’. (15 Marks) May/02
[Ans.: (i) Total manufacturing cost per unit of ‘AB 100’ `2000 and ‘AB 200’ is `1614.25; (ii) Operating income
per month will be reduced by `1050000]
[Hint: It is to be assumed that the total available engineering hours is used for manufacturing ‘AB 200’ model
of audio player]

CA. Parag Gupta Ph.: +91 11 47665555 Paraggupta_ca@yahoo.co.in Costing & O.R.
World’s largest CA Final student’s consultancy group: http://groups.yahoo.com/group/costingbyparaggupta
Target Costing, Value
analysis & LCC

T ARGET COSTING
Meaning

Target Costing is defined as “a structured approach to determine the cost at which a proposed product with
specified functionality and quality must be produced, to generate a desired level of profitability at its
anticipated-selling-price”

Steps in Target Costing approach to pricing:


Set target selling price based on customer
expectations and sales forecast

Establish profit margin based on long-term


profit objectives and projected volumes

Determine target (or allowable) cost per unit


(target selling price less required profit

Compare with Estimate the current cost of new product

Establish cost reduction targets for each


component and production activity, using value
engineering and value analysis

1. Setting of target selling price: The setting of target selling price of a product which customers are
prepared to pay, depend on many factors like design specifications of the product, competitive
conditions, customer’s demand for increased functionality and higher quality projected production
volume, sales forecasts etc. A concern can set its target selling price after taking into account all of
the aforesaid factors.
2. Determination of target costs: Target profit margin may be established after taking into account
long-term profit objectives and projected volume of sales. On deducing target profit margin from
target selling price, target cost is determined.
3. Estimate the actual cost of the product: Actual cost of the product may be determined after taking
into account the design specifications, material cost and other costs required to produce the product.
4. Comparison of estimated cost with actual cost: In case the estimated cost of the product is higher
than that of the target cost of the product then the concern should resort to cost reduction methods
involving the use of Value Engineering / Value Analysis tools.

VALUE ENGINEERING (VE)


Value Analysis entails studying the activities that are involved in producing the product to detect non-value
adding activities that may be eliminated or minimized to save costs, but without reducing the functionality or
quality of the product. Value Analysis is used to analyze and understand the detail of specific situations. It is
used to find a focus on key areas for innovation.

CA. Parag Gupta Ph.: +91 11 47665555 Paraggupta_ca@yahoo.co.in Costing & O.R.
World’s largest CA Final student’s consultancy group: http://groups.yahoo.com/group/costingbyparaggupta
Cost Accounting & Management - 35 -

Value Engineering involves searching for opportunities to modify the design of each component or part of a
product to reduce cost, but without reducing the functionality or quality of the product. It is used in reverse
(called Value Engineering) to identify specific solutions to detail problems. It is particularly suited to physical
and mechanical problems, but can also be used in other areas.

Value Analysis Value Engineering


Indicates application on the product that is into Indicates application on the product at its design
manufacturing. stage.
All factors come together Including workers, It is always done by a specific product design
subcontractors, engineers to make a team with total (engineers) team.
experience and knowledge.
It may change the present stage of the The changes are executed at the initial stages only.
product or operation.

Scope
Value Engineering and Value Analysis help identify costs into
a) Value-Added Cost and
b) Non Value-Added Cost. The objective is to retain (if possible, reduce) value-added cost, while totally
avoiding or eliminating non-value added costs.

Value-added cost: A value-added cost is a cost that, if eliminated, would reduce the value or utility
(usefulness) customers obtain from using the product or service.

Non value-added cost: A non value-added cost is a cost that, if eliminated would not reduce the value or
utility customers obtain from using the product or service. It is a cost that the customer is unwilling to pay to
the company.

Some issues analyzed during VE review are:


1. Elimination of unnecessary functions from the production process:
 This involves a detailed review of the entire manufacturing process to see if there are any steps that add
no value to the product, e.g. interim quality review before further processing and final quality check.
 By eliminating unnecessary or duplicate functions, the firm can reduce their associated direct or
overhead costs from the total product cost.
 The possible repercussions of elimination of any intermediate production function should be carefully
analyzed. The engineering team must be careful to develop work-around steps that eliminate the need
for the original functions.
2. Elimination of unnecessary product qualities:
 The product quality should be studied with reference to the nature of its use, longevity of product’s
useful life.
 If some unnecessary quality e.g. excessive degree of sturdiness in consumable item (as opposed to a
durable item) can be eliminated, it should be done in order to save significant material and other product
costs.
 However, visible reduction in durability or reliability cannot be stretched too far. Hence any designs that
have had their structural integrity reduced must be thoroughly tested to ensure that they meet all design
standards.
3. Design Minimization:
 This involves the creation of a design that uses fewer parts or has fewer features.
 This approach is based on the assumption that a minimal design is easier to manufacture and assemble
Also, with fewer parts to purchase, less procurement expenses is associated with the product.
 However, sometimes it would be less expensive to settle for a few extra standard parts that are more
easily and cheaply obtained, rather than customized pre-fabricated parts, which complicate the
assembly process.
4. Better product Design to suit manufacturing process:
 This is also known as Design For Manufacture and Assembly (DFMA) and involves the creation of a
product design that can be created in only a specific manner. For example, a toner cartridge for a laser
printer is designed so that it can be successfully inserted into the printer only when the sides of the
cartridge are correctly aligned with the printer opening; all other attempts to insert the cartridge will fail.

CA. Parag Gupta Ph.: +91 11 47665555 Paraggupta_ca@yahoo.co.in Costing & O.R.
World’s largest CA Final student’s consultancy group: http://groups.yahoo.com/group/costingbyparaggupta
Target costing, Value analysis & Life Cycle Costing - 36 -
(including techniques of profit improvement, cost reduction)

 When used for the assembly of an entire product, this approach ensures that a product is not incorrectly
manufactured or assembled, which would call for a costly disassembly or product recalls from customers
who have received defective goods.
5. Substitution of Parts:
 This is also called as Component Parts Analysis. This approach encourages the search for less
expensive components or materials that can replace more expensive parts currently used in a product
design.
 Substitution of new parts is encouraged since new materials are being developed every year.
 However parts substitution must be accompanied by a review of related changes elsewhere in the
design and the consequent impact on total costs.
 This also involves allied analysis on tracking the intentions of suppliers to continue production of parts in
the future. If parts are not available, they must be eliminated from the product design.
6. Combination of Steps:
 Sometimes, a careful review of all processes associated with a product reveals that some steps can be
eliminated, other steps can be consolidated, or that several can be accomplished by one person, rather
than having people in widely disparate parts of the production process to perform them. This is also
known as Process Centering.
 By combining steps, transfer and queue time can be eliminated from the production process, which in
turn reduces the chances of damage during transfers.
7. Search for better way of doing things:
 This seeks to answer a basic question – is there a better way?
 It strikes at the core of the cost reduction issue. It is a more general attempt to start from scratch and
build a new product or process that is not based in any way on pre-existing ideas.
 Improvements resulting from this technique tend to have the largest favorable impact on cost reductions
but can also be the most difficult for the organization to adopt, especially if it has used other designs or
systems for production.

Role of a firm’s Suppliers in its Value Engineering or Cost Reduction drive


Value Engineering also involves calling on the services of a company’s suppliers to assist in the cost
reduction effort. Suppliers of materials can have significant role in value engineering due to the following
reason:
1. Suppliers can contribute information on enhanced types of technology of materials.
2. Suppliers specialize in areas that a company has no information about and can share product
expertise.
 They may have also conducted extensive value engineering for the components they
manufacture, resulting in advanced designs that a Company may be able to incorporate into
its new products.
 Suppliers may have also redesigned their production processes, or can be assisted by a
company’s engineers in doing so, producing cost reductions or decreased production waste
that can be translated into lower components costs for the company.

KAIZEN COSTING
Kaizen Costing refers to the ongoing continuous improvement program that focuses on the reduction of
waste in the production process, thereby further lowering costs below the initial targets specified during the
design phase. It is a Japanese term for a number of cost reduction steps that can be used subsequent to
issuing a new product design to the factory floor.
The initial VE review may not be complete and perfect in all costs aspects. There may be further chances of
waste reduction, cost and time reduction and product improvement. Such continuous cost reduction technique
is call as kaizen costing.
The review of product costs under the target costing methodology is not reserved just for the period up to the
completion of design work on a new product. On the contrary, there are always opportunities to control costs
after the design phase is completed, though these opportunities are fewer than during the design phase.

Kaizen Costing Process: Activities in kaizen costing include elimination of waste in production, assembly
and distribution processes, as well as the elimination of work steps in any of these areas. Thus kaizen costing
is really designed to repeat many of the value engineering steps for as long as a product is produced,
constantly refining the process and thereby stripping out extra costs at each stage.

CA. Parag Gupta Ph.: +91 11 47665555 Paraggupta_ca@yahoo.co.in Costing & O.R.
World’s largest CA Final student’s consultancy group: http://groups.yahoo.com/group/costingbyparaggupta
Cost Accounting & Management - 37 -

Savings from Kaizen Costing: The cost reductions resulting from kaizen costing are much smaller than
those achieved with value engineering. But these are still significant since competitive pressures are likely to
force down the price of a product over time, and any possible cost savings allow a company to still attain its
targeted profit margins Mille continuing to reduce cost.

Multiple Versions of Products - Continuous Kaizen Costing: Multiple improved versions of products can be
introduced to meet the challenge of gradually reducing costs and prices. The market price of products
continues to drop over time, which forces a company to use both target and kaizen costing to reduce costs
and retain its profit margin.

However, prices eventually drop to the point where margins are reduced, which forces the company to
develop a new product with lower initial cost and for which kaizen costing can again be used to further reduce
costs. This pattern may be repeated many times as a company forces its costs down through successive
generations of products.

The exact timing to switch to a new product is easy to determine well in advance since the returns from
kaizen costing follow a trend line of gradually shrinking savings. Since prices also follow a predictable
downward track, plotting these two trend lines into the future reveals when a new product version must be
ready for production.

Advantages of Target Costing:


1. Innovation: It reinforces top-to-bottom commitment to process and product innovation and is aimed at
identifying issues to be resolved.
2. Competitive Advantage: It enables a firm to achieve competitive advantage over other firms in the
industry. The firm, which achieves cost reduction targets realistically, stands to gain in the long run.
3. Market Driven Management: It helps to create a company’s competitive future with market-driven
management for designing and manufacturing products that meet the price required for market success.
4. Real Cost Reduction: It uses management control systems to support and reinforce manufacturing
strategies and to identify market opportunities that can be converted into real savings to achieve the best
value rather than simply the lowest cost.

Limitations of Target Costing:


1. Time Factor: The development process can be lengthened to a considerable extent since the design
team may require a number of design iterations before it can devise a sufficiently low-cost product
that meets the target cost and margin criteria.
2. Responsibility for Cost Reduction: A large amount of mandatory cost cutting can result in finger-
pointing in various parts of the company, especially if the cost reduction targets are not equitably
shared. For example the industrial engineering staff will not be happy it is required to completely alter
the production layout in order to generate cost savings, while the purchase staff is not required to
make any cost reductions through supplier negotiations. Strong inter-personal and negotiation skills
are required on the part of the project manager, to avoid this problem.
3. Co-ordination: Having representatives from a number of departments on the design team can
sometimes make decision-making difficult, as there are too many opinions regarding design issues.
Resolving this difficulty requires a strong team manager, as well as a long-term commitment on the
part of a company to weed out those who are not willing to act in the best interests of the team.

Cost Accountant’s role in a Target Costing Environment:


1. Cost Estimation: To provide other members of the design team a running series of cost estimates
based on initial design sketch, activities based costing reviews of production processes, and “best
guess” costing information from suppliers based on estimated production volumes.
2. Permissible Cost Ranges: To provide estimates within a high-low range costs, since preliminary
data will necessarily be vague. However, this estimated cost range should be tightened as more
information becomes available.
3. Capital Budgeting Analysis: To cater to capital budgeting requests generated by the design team
based on types of equipment needed for the anticipated product design, product revenues and costs,
rates of return etc. The Cost Accountant should also be able to answer questions regarding
uncertainties and risk analysis.

CA. Parag Gupta Ph.: +91 11 47665555 Paraggupta_ca@yahoo.co.in Costing & O.R.
World’s largest CA Final student’s consultancy group: http://groups.yahoo.com/group/costingbyparaggupta
Target costing, Value analysis & Life Cycle Costing - 38 -
(including techniques of profit improvement, cost reduction)

4. Cost Principles Explanation: To work with the design team to help it understand the nature of
various costs (such as cost allocations based on an activity-based costing system), as well as the
cost-benefit trade-offs of using different design or cost operations in the new product.
5. Review of Cost Reduction Targets: To track the gap between the current cost and the target cost
that is the design team’s goal providing an itemization of where cost savings have already been
achieved and where there has not been a sufficient degree of progress.
6. Final Review and Feedback: To compare a product’s actual cost to the target cost after the design
is completed, and for as long as the company sells the products is a necessary step because
management must know immediately if costs are increasing beyond budgeted levels and why these
increases are occurring.

Impact of target costing on profitability


Assured Profit by constant review: Target Costing places detailed continuing emphasis on product costs
throughout the life cycle of every product that it is unlikely that a company will experience runaway costs. Also
the management is completely aware of costing issues since it receives regular reports from the cost
accounting members of all design teams.
Price Determination and Consequent Cost Control: Target Costing improves profitability through precise
targeting of the correct prices at which the company feels it can field a profitable product in the marketplace
that will sell in a robust manner. The traditional cost-plus approach revolves around designing a product,
determining its cost, adding a profit margin and failing to understand why its resoundingly high price does now
attract buyers. Thus, target costing results not only in better cost control but also in better price control.

Target costing can have positive impact on profitability, depending on the commitment of management to its
use, the constant involvement of cost accountants in all phases of a product’s life cycle, and the type of
strategy a company follows. Target costing is really part of a larger concept called concurrent engineering,
which requires participants from many departments to work together on project teams. It is indeed one of the
most proactive systems found in the entire range of accounting knowledge.

Features of Target Costing Data:


1. Non-Traditional: The traditional sources of cost data is a central accounting data base consisting of
accounts payable, billings, bills of material, and inventory records. These do not provide information
required for Target Costing.
2. Futuristic: Target Costing Data is essentially futuristic, as it is associated with new and improved
products, new designs and manufacturing processes.
3. Poorly defined: The data for Target Costing project is more poorly defined information.
The Cost Accountant has to start from scratch in order to estimate costs. In earlier stages of product
designs, “best possible guesses” may have to be used.

Steps involved in implementing a Target Costing System:


1. Create a Project Charter: Project Charter is a document, approved by top management that describes
its goals and what it is authorized to do. This Charter is based on the corporate mission statement and
related goals. Written approval of Project Charter by the top management provides the target costing
effort with a strong basis of support and direction in all subsequent efforts.
2. Obtain a Management Sponsor: Management Sponsor is an individual belonging to top management.
His role will be to support the initiative in all respects, to obtain funding, to co-ordinate with other
members of top management, to eliminate problems in a timely manner.
3. Obtain a Budget: The funding should be based on a formal allocation of money through the corporate
budget. The fund should be given unreservedly to the target costing effort.
4. Assign a Strong Team Manager: The Target Costing Team involves the active participation of many
members with diverse backgrounds. A strong Team Manager is required to bring the group together as
a smooth functioning team focused on key objectives. He should be skilled in dealing with management,
the use of project tools and working with a diverse group of people. This manager should be a full-time
employee, so that his or her complete attention can be directed towards the welfare of the project.
5. Enroll Full-time Participants: It is essential that the members of the team be devoted to it full-time
rather than trying to fulfill other commitment elsewhere in the company at the same time. They should
have a single focus on ensuring the success of the target- costing program.
6. Use Project Management Tools: Target costing can be a highly complex effort especially for high-cost
products with many features and components. The team should use all available project management

CA. Parag Gupta Ph.: +91 11 47665555 Paraggupta_ca@yahoo.co.in Costing & O.R.
World’s largest CA Final student’s consultancy group: http://groups.yahoo.com/group/costingbyparaggupta
Cost Accounting & Management - 39 -

tools, such as Microsoft Project (for tracking the completion of specific tasks) a company database
containing various types of costing information and a variety of product design tools.

Question 1: List the steps involved in target costing process with the help of a block diagram.
(6 Marks)Nov./06
Question 2: What is Target Costing? It is said that implementation of the target costing technique
requires intensive marketing research. Explain why intensive marketing research is required to
implement target costing technique (9 Marks)Nov./07
Question 3: How can value analysis achieve cost reduction? (5 Marks) Nov./09-N.C.
Ans,: Value analysis can do cost reduction in the following manner:
 By identifying and removing unnecessary components in a product which had utility earlier.
 By introducing component substitution at a lesser cost without affecting the quality of the product.
 By simplifying the product design.
 By introducing alternative methods with less cost but improved efficiency.
Question 4: What are the benefits of a target costing system? (3 Marks) May/10-O.C.

Question 5: Discuss, how target costing may assist a company in controlling costs and pricing of products.
(4 Marks)Nov./08-O.C.
Answer: Target costing may assist control of costs and pricing of product as under:
1. Target costing considers the price that ought to be charged by a company to achieve a given
market share.
2. Target costing should take life cycle costs in to consideration.
3. If there is a gap between the target cost and expected cost, ways and means of reducing or
eliminating it can be explored.
4. The target cost may be used for controlling costs by comparison
Question 6: What is Target Costing and what are the stages to the methodology? (4 Marks) Nov./00
Answer: Target Costing: It is a management tool used for reducing a product cost over its entire life
cycle. It is driven by external Market factors. Marketing management prior to designing and introducing a
new product determines a target market price. This target price is set at a level that will permit the
company to achieve a desired market share and sales volume. A desired profit margin is then deducted
to determine the target maximum allowable product cost. Target costing also develops methods for
achieving those targets and means to test the cost effectiveness of different cost-cutting scenarios.
Stages to the methodology.
1. Conception (planning) Phase: Under this stage of life cycle, competitors products are to be
analysed, with regard to price, quality, service and support, delivery and technology. The features
which consumers would like to have like consumer value etc. established. After preliminary testing,
the company may be asked to pinpoint a market niche, it believes, is under supplied and which
might have some competitive advantage.
2. Development phase: The design department should select the most competitive product in the
market and study in detail the requirement of material, manufacturing process along with
competitors cost structure. The firm should also develop estimates of internal cost structure based
on internal cost of similar products being produced by the company. If possible the company
should develop both the cost structures (competitors and own) in terms of cost drivers for better
analysis and cost reduction.
3. Production phase: This phase concentrates its search for better and less expensive products,
cost benefit analysis in different features of a product priority wise, more towards less expensive
means of production, as well as production techniques etc.
Question 7: A company has the capacity of production of 80,000 units and presently sells 20,000 units at
`100 each. The demand is sensitive to selling price and it has been observed that every reduction of
`10 in selling price the demand is doubled. What should be the target cost at full capacity if profit margin
on sale is taken as 25%?
What should be the cost reduction scheme if at present 40% of cost is variable with same % of profit? If
Rate of Return is 15%, what will be maximum investment at full capacity?
[Ans.: `60 p.u.; Cost Reduction is `12 lakhs; Investment: `10666667]
[Hint: Whenever there is a scope of increase in capacity & fixed cost is provided on p.u. basis, we
assume fixed cost p.u. is on full capacity]

CA. Parag Gupta Ph.: +91 11 47665555 Paraggupta_ca@yahoo.co.in Costing & O.R.
World’s largest CA Final student’s consultancy group: http://groups.yahoo.com/group/costingbyparaggupta
Target costing, Value analysis & Life Cycle Costing - 40 -
(including techniques of profit improvement, cost reduction)

Question 8: Sterling Enterprises has prepared a draft budget for the next year as follows:

Quantity 10000 units


Sales price per unit 30
Variable costs per unit: Direct Materials 8
Direct Labour 6
Variable overhead (2 hrs × Re. 0.50) 1
Contribution per unit 15
Budgeted Contribution 1,50,000
Budgeted Fixed costs 1,40,000
Budgeted Profit 10,000

The Board of Directors is dissatisfied with this budget, and asks a working party to come up with an
alternate budget with higher target profit figures.

The working party reports back with the following suggestions that will lead to a budgeted profit of
`25,000. The company should spend `28,500 on advertising, & put the target sales price up to `32 per
unit. It is expected that the sales volume will also rise, inspite of the price rise, to 12,000 units.

In order to achieve the extra production capacity, however, the work force must be able to reduce the
time taken to make each unit of the product. It is proposed to offer a pay and productivity deal in which
the wage rate per hour in increased to `4. The hourly rate for variable overhead will be unaffected.
Ascertain the target labour time required to achieve the target profit.

[Ans.: Time reduction p.u. 0.25 per hour]


Question 9: AML Ltd. Is engaged in production of three types of ice-cream products: Coco, Stawberry and
Vanilla. The company presently sells 50,000 units of Coco @ `25 per unit, Stawberry 20,000 @ `20 per unit
and Vanilla 60,000 units @ `15 per unit. The demand is sensitive to selling price and it has been observed
that every reduction of `1 per unit in selling price, increases the demand for each product by 10% to the
previous level. The company has the production capacity of 60,500 units of coco, 24,200 units of strawberry
and 72,600 units of vanilla. The company marks up 25% of the cost of product.

The company management decides to apply ABC analysis. For this purpose it identifies four activities and the
rates as follows:
Activity Cost rate

Ordering `800 per purchase order


Delivery `700 per delivery
Shelf stocking `199 per hour
Customer Support and assistance `1.10 p.u. sold.

The other relevant information for the products are as follows:

Coco Stawberry Vanilla


Direct Materials p.u. (`) 8 6 5
Direct wages p.u. (`) 5 4 3
No. of purchase order 35 30 15
No. of deliveries 112 66 48
Shelf stocking hours 130 150 160

Under the traditional costing system, store support costs are charged @ 30% of prime cost. In ABC these
costs are coming under customer support and assistance.Note

Required

CA. Parag Gupta Ph.: +91 11 47665555 Paraggupta_ca@yahoo.co.in Costing & O.R.
World’s largest CA Final student’s consultancy group: http://groups.yahoo.com/group/costingbyparaggupta
Cost Accounting & Management - 41 -

(i) Calculate target cost for each product after a reduction of selling price required to achieve the sales equal
to the production capacity.
(ii) Calculate the total cost and unit cost of each product at the maximum level using traditional costing.
(iii) Calculate the total cost and unit cost of each product at the maximum level using activity based costing.
(iv) Compare the cost of each product calculated in (i) and (ii) with (iii) and comment on it.

[Ans.: (i) 18.4, 14.4, 10.4 (ii) 1022450, 314600, 755040; 16.90, 13.00, 10.40 (iii) 985320, 368670, 738100;
16.286, 15.24, 10.16] (12 Marks) May/10-N.C.
[Note: On calculation of total overhead costs under traditional & ABC system, costs are same i.e. `4,82,790,
hence we will ignore the line “In ABC these costs are coming under customer support and assistance.” written
in question.-Also clarified in examiner comment’s by ICAI]
Question 10 [Target Costing]: X Ltd. is engaged in the production of four products: A, B, C and D. The price
charged for the four products are `180, `175, `130 and `180 respectively. Market research has indicated that
if X Ltd can reduce the selling prices of its products by `5, it will be successful in getting bulk orders and gain
a significant share of market of those products. The company's profit markup is 25 per cent on cost of the
product. The relevant information of products are as follows:

Products A B C D
Output in units 600 500 400 600
Cost per unit:
Direct material (in `) 40 50 30 60
Direct labour (in `) 28 21 14 21
Machine hours (per unit) 4 3 2 3

The four products are usually produced in production runs of 20 units and sold in batches of 10 units. The
production overhead is currently absorbed by using a machine hour rate, and the total of the production
overheads for the period has been analysed as follows:
(`)
Machine department costs 52,130
Setup costs 26,250
Stores receiving 18,000
Inspection/Quality control 10,500
Material handling and dispatch 23,100

The cost drivers to be used for the overhead costs are as follows:
Cost Cost drivers
Setup costs Number of production runs
Store receiving Requisitions raised
Inspection/Quality control Number of production runs
Materials handling and dispatch Order executed

The number of requisitions raised in the stores was 100 for each product and the number of orders executed
was 210, each order being for a batch of 10 units of a product.

You are required :


i) To compute the target cost for each product.
ii) To compute total cost of each product using activity based costing.

Compare target cost and activity based cost of each product and comment whether the price reduction is
profitable or not. (12 Marks) Nov.’09-O.C.

[Ans.: (a) Target Cost: 140, 136, 100, 140; (b) Total Cost as per ABC: 81648, 66280, 39904, 84636]
[Hint: This question is exactly same as Question 15 of “Activity based cost management”]
Question 11 [ABC + Target Costing]: Computo Ltd. manufactures two parts ‘P’ and ‘Q’ for Computer
Industry.

P : annual production and sales of 1, 00,000 units at a selling price of `100.05 per unit.
CA. Parag Gupta Ph.: +91 11 47665555 Paraggupta_ca@yahoo.co.in Costing & O.R.
World’s largest CA Final student’s consultancy group: http://groups.yahoo.com/group/costingbyparaggupta
Target costing, Value analysis & Life Cycle Costing - 42 -
(including techniques of profit improvement, cost reduction)

Q : annual production and sales of 50,000 units at a selling price of `150 per unit.

Direct and Indirect costs incurred on these two parts are as follows:
(`in thousand)
P Q Total
Direct Material cost (variable) 4,200 3,000 7,200
Labour cost (variable) 1,500 1,000 2,500
Direct Machining cost (See Note)* 700 550 1,250
Indirect Costs:
Machine set up cost 462
Testing cost 2,375
Engineering cost 2,250
16,037

Note: Direct machining costs represent the cost of machine capacity dedicated to the production of each
product. These costs are fixed and are not expected to vary over the long-run horizon.
Additional information is as follows:

P Q
Production Batch Size 1,000 units 500 units
Set up time per batch 30 hours 36 hours
Testing time per unit 5 hours 9 hours
Engineering cost incurred on each product 8.40 lacs 14.10 lacs

A foreign competitor has introduced product very similar to ‘P’. To maintain the company’s share and profit,
Computo Ltd. has to reduce the price to `86.25. The company calls for a meeting and comes up with a
proposal to change design of product ‘P’. The expected effect of new design is as follows:

· Direct Material cost is expected to decrease by `5 per unit.


· Labour cost is expected to decrease by `2 per unit.
· Machine time is expected to decrease by 15 minutes; previously it took 3 hours to produce 1 unit of ‘P’. The
machine will be dedicated to the production of new design.
· Set up time will be 28 hours for each set up.
· Time required for testing each unit will be reduced by 1 hour.
· Engineering cost and batch size will be unchanged.

Required:

(a) Company management identifies that cost driver for Machine set-up costs is ‘set up hours used in batch
setting’ and for testing costs is ‘testing time’. Engineering costs are assigned to products by special study.
Calculate the full cost per unit for ‘P’ and ‘Q’ using Activity-based costing.
(b) What is the Mark-up on full cost per unit of P?
(c) What is the Target cost per unit for new design to maintain the same mark up percentage on full cost per
unit as it had earlier? Assume cost per unit of cost drivers for the new design remains unchanged.
(d) Will the new design achieve the cost reduction target?
(e) List four possible management actions that the Computo Ltd. should take regarding new design.
(16 Marks) May/06
[Ans.: (a) P –`87, Q- `146.74; (b) 15%; (c) `75; (d) No, as the cost of new design is `77.36 per unit.
(e) (i) Value engineering and value analysis to reduce the direct material costs.
(ii) Time and motion study in order to redefine the direct labour time and related costs.
(iii) Exploring possibility of cost reduction in direct machining cost by using appropriate techniques.
(iv) Identification of non-value added activities and eliminating them in order to reduce overheads.]

Question 12 [ABC + Target Costing]: IBM Ltd. Manufactures and sells computers peripherals to several
retail outlets throughout the country. Amar is the manager of the printer division. Its two largest selling printers
are P1 & P2. The manufacturing cost of each printer is calculated using IBM’s activity based costing system.
IBM has one direct manufacturing cost category (direct materials) and the following five indirect
manufacturing cost pools.
CA. Parag Gupta Ph.: +91 11 47665555 Paraggupta_ca@yahoo.co.in Costing & O.R.
World’s largest CA Final student’s consultancy group: http://groups.yahoo.com/group/costingbyparaggupta
Cost Accounting & Management - 43 -

Indirect manufacturing cost pool Allocation Base Allocation Rate (`)


1. Materials handling No. of parts `1.20 per part
2. Assembly management Hours of assembly time `40 per hour of
assembly time
3. Machine insertion of parts No. of machine inserted parts `0.70 per machine
inserted part
4. Manual insertion of parts No. of manually inserted parts `2.10 per
manually inserted part
5. Quality testing Hours of quality testing time `25 per testing
hour.

Product characteristics of P1 and P2 are as follows:


Product P1 P2
Direct materials costs `407.50 `292.10
Number of parts 85 46
Hours of assembly time 3.2 1.9
Number of machine – inserted parts 48 31
Number of manually inserted parts 36 15
Hours of quality testing time 1.4 1.1

A foreign competitor has introduced products very similar to P1 and P2. Given their announced selling prices
and to maintain his company’s market share and profits, Amar estimates the P1 to have manufacturing cost of
approximately `680 and P2 to have a manufacturing cost of approximately `390. He calls a meeting of
product designers and manufacturing personnel at the printer division. They all agree to have the `680 and
`390 figures become target costs for designed version of P1 and P2 respectively. Product designers examine
alternative ways of designing printer with comparable performance but lower costs. They come up with the
following revised designs for P1 and P2 (termed P1 – REV and P2 – REV, respectively)

Particulars P1 – REV P2 – REV


Direct materials cost `381.20 `263.10
Number of parts 71 39
Hours of assembly time 2.1 1.6
Number of machine – inserted parts 59 29
Number of manually – inserted parts 12 10
Hours of quality testing time 1.2 0.9

Required:
 Compute the present costs of products P1 and P2 using ABC system.
 Compute the manufacturing costs of P1 – REV and P2 – REV. How do they compare with the `680
and `390 target costs? (RTP-May/05, May/07, Nov./07 & Nov./08-O.C.)

[Ans.: P1: Present Cost-`781.70, Estimated Cost-`646.90 & Target Cost-`680; P2: Present Cost-`504.00,
Estimated Cost-`437.70 & Target Cost-`390]

VALUE CHAIN ANALYSIS

Competitive advantage for a company means not just matching or surpassing their competitors, but
discovering what the customers want and then profitably satisfying and even exceeding their expectations. As
barriers to global trade are diminishing, customers can acquire the best of what they want, at an acceptable
price, from anywhere in the world.

A strategic tool to measure the importance of the customer’s perceived value is value chain analysis.

Classification of business activities for VCA purpose:


Porter classified business activities into (a) Primary or Line activities and (b) Support activities
a) Primary activities are directly involved in transforming inputs into outputs and delivery and after
sales support to output. They are handled by line functions in an organisation. They include:
CA. Parag Gupta Ph.: +91 11 47665555 Paraggupta_ca@yahoo.co.in Costing & O.R.
World’s largest CA Final student’s consultancy group: http://groups.yahoo.com/group/costingbyparaggupta
Target costing, Value analysis & Life Cycle Costing - 44 -
(including techniques of profit improvement, cost reduction)

 Material handling and warehousing


 Transforming inputs into final product
 Order processing and distribution Communication, pricing and channel management, and
 Installation, Repair and parts replacement
b) Support activities are activities that support primary activities. They are handled by the
organisation’s staff functions and include:
1. Procurement – purchasing of raw materials, supplies and other consumable items as well as
assets.
2. Technology Development – know-how, procedures and technological inputs needed in every
value chain activity.
3. Human resource management – Selection, promotion and placement, appraisal, rewards;
management development and labour / employee relations.
4. Firm Infrastructure – general management, planning, finance, accounting, legal government
affairs and quality management.

Industry Value Chain.


Industry Value Chain refers to the series of activities, which add value to the product supplied to the industry.
It starts with the value-creating processes of suppliers, who provide the basic raw materials and components.
It continues with the value creating processes of different classes of buyers or end-use consumers and
culminates in the disposal and recycling of materials.

Firm’s competitive advantage


1. To survive and prosper in an industry, a firm must meet two criteria
 Supply what customers want to buy, and
 Survive competition
2. A firm’s overall Competitive Advantage is derived from the difference between Value Offered to
Customers and Cost of creating that customer value.
3. This Competitive advantage takes two possible forms
 Differentiation Advantage and
 Low-Cost Advantage.
A comparative analysis of these forms is given below:

Differentiation Advantage Low - Cost Advantage


It occurs when customers perceive that a firm’s A firm enjoys a relative low-cost advantage if its total
product offering is of higher quality, involves less risk costs are lower than the market average.
and/or outperforms competing products offered by
competitors. Customers are thus willing to pay a
premium price for this product.
Gained by: Gained by:
 Ability to deliver goods & services in timely  Access to low cost raw material
manner  Innovative process technology
 Producing better quality  Access to distribution channels
 Provision of after-sales support services  Economies of scale
 Offering a wider range of goods and services  Superior operating management etc.
etc.
Advantage can be exploited by: Advantage can be exploited by:
 Increasing prices to offsets the improvement  Pricing the products lower than it’s
in customer benefits thus maintaining current competitor’s so as to gain market share and
market share; or maintain current profitability or,
 Pricing below the ‘full-premium’ level in order  Matching with the price of competing
to build market share. products and increase its profitability
Superior relative differentiation position offers the Superior relative cost position offers customers
customers better value for an equivalent price. equivalent value for a lower price.

Use of VCA to assess competitive advantage


A company can gain competitive advantage not just by matching or surpassing its competitors, but
discovering what the customers want and then profitably satisfying and even exceeding, customer
expectations. This is done by a concept called Value Chain analysis (VCA).

CA. Parag Gupta Ph.: +91 11 47665555 Paraggupta_ca@yahoo.co.in Costing & O.R.
World’s largest CA Final student’s consultancy group: http://groups.yahoo.com/group/costingbyparaggupta
Cost Accounting & Management - 45 -

VCA can be used to better understand which segment, distribution channels, price points, product
differentiation, selling propositions and value chain configurations will yield the firms the greatest competitive
advantage.

Use of VCA to assess competitive advantage involves the following analyses:

1. Internal Cost analysis – to determine the sources of differentiation (including the cost) within internal
value-creating processes
2. Internal Differentiation analysis – to understand the sources of differentiation (including the cost)
within internal value-creating processes; and
3. Vertical Linkage analysis – to understand the relationships and associated costs among external
suppliers and customers in order to maximize the value delivered to customers and to minimize cost.

The following actions and steps are involved in the above analyses:

Stage Description
1. Internal Cost Analysis:
 Identify the firm’s value creating processes
 Determine the portion of the total cost of the product or services attributable to each value-
creating process.
 Identify the cost drivers for each process.
 Identify the links between processes
 Evaluate the opportunities for achieving relative cost advantage.
2. Internal Differentiation Analysis:
 Identify the industry’s value creating processes
 Evaluate differentiation strategies for enhancing customer value
 Determine the best sustainable differentiation strategies
3. Vertical Linkage Analysis
 Identify the industry’s value chain and assign costs, revenues and assets to value-
creating processes.
 Diagnose the cost drivers for each value-creating process; and
 Evaluate the opportunities for sustainable competitive advantage

Features of this analysis:

1. Not mutually exclusive – Firms begin by focusing on their internal operations and gradually
widening their focus to consider their competitive position within their industry.
2. Continuous – VCA is a continuous process of gaining competitive advantage not a one- time affair.
3. Part of Strategic Planning: VCA is a process of gathering, evaluating and communicating
information for business decision-making.

Steps involved in Internal Cost Analysis (Stage 1)

1. Identify the firm’s value-creating processes


 Traditionally, the firm organizes itself into departments based on cost, revenue profit and
investment centers. These centers are useful for control but are not very useful for identifying
value-creating processes.
 Adopting a process perspective requires a horizontal view of the organization beginning with
product inputs and ending with outputs and customers.
 Processes are structured and measured sets of activities designed to produce a specified output
for a particular customer or market.
 Emphasising process means focusing not on what work is done but on how work is done within
the organization.
 Process Structure differs from traditional hierarchical structure and shows how the organization
delivers customer value. While it is not possible to measure or improve hierarchical structure in
any absolute sense, processes lend themselves to such measures as cost, time, output quality
and customer satisfaction.

CA. Parag Gupta Ph.: +91 11 47665555 Paraggupta_ca@yahoo.co.in Costing & O.R.
World’s largest CA Final student’s consultancy group: http://groups.yahoo.com/group/costingbyparaggupta
Target costing, Value analysis & Life Cycle Costing - 46 -
(including techniques of profit improvement, cost reduction)

2. Determine the portion of total cost of the product/service attributable to each value creating
process:
 A full-cost approach provides the best estimate of life-cycle costs for evaluating the strategic cost
advantage of a firm’s value-creating process.
 For estimating the full cost of each value-creating activity, full utilization of the capacity of the
activity or its practical capacity is normally used. Facility managers and equipment vendors are
useful sources of capacity estimates. When cost vary dramatically, companies should seek more
information for a more realistic long-term estimate of capacity.
3. Identify the cost drivers for each process
 By listing cost drivers, a firm can assign priorities among its cost improvement initiatives.
 In order to determine its relative cost advantage, a firm should also know the cost factors of its
competitors.
 Multiple cost drivers are identified for each value-creating process. These may be classified into
(a) Structural cost drivers (covering aspects such as Scale, Scope, learning, Technology and
Complexity etc.) and (b) Executional Cost Drivers (Capacity utilization, plant layout, Product
design, Employee participation, Supplier and customer liaison, etc.)

Structural Cost Drivers Executional Cost Drivers


 They consist of organizational factors that  They capture a firm’s operational decisions
determine the economic structure driving the on how and best to employ its resources to
cost of firm’s product. achieve its goals objectives.
 These cost drivers reflect a firm’s long-term  These cost drivers are determined by
firm in its decisions, which position the management policy, style and culture. They
industry and marketplace. are comparatively short term.
 Structural cost drivers may change.  Executional cost drivers may improve.
 Example: Large Pharmaceutical Companies  Example:-Worker empowerment and
Economies of scale that lower their unit flattened continuous organization help many
enjoy costs for expensive R&D. firms in their improvement efforts.

4. Identify the links between processes


 Activities within a value chain are interdependent and hence firms must identify value chain
linkages among interdependent activities that may impact their total cost.
 Cost improvement programs in one value chain process may lower or increase costs and/or
revenues in other processes. Transfer of goods and services from one value
 Chain process to another increases cost. Eliminating these transfer or costs has an impact on
overall costs in the subsequent chain.
 Such linkage offer sustainable competitive advantage, because of their subtle, complex and
inimitable nature.

5. Evaluate the opportunities for achieving relative cost advantage


 Using the value chain approach, a company goes beyond simple across-the-board cuts and
attempts to lower cost and improve efficiency within each value-creating process. For instance a
company might negotiate lower costs of process inputs such as wages or purchases, or evaluate
make-or-buy options.
 Reducing process input costs may consist of measures such as negotiating lower wages, moving
production to countries cheaper labour costs, entering into long term contracts with suppliers at
reduced prices, etc. Companies also buyer-seller partnerships to gain advantages in cost quality,
flexibility, delivery and, technology.
 Using Pareto Analysis company should prioritize its value-creating processes since 20% of value
creating processes often account for 80% of total costs.

Steps in Internal Differentiation Analysis (Stage 2)

i. Identify the customers’ value-creating processes:


To pursue a superior differentiation strategy, a firm’s processes must enhance the value of its
customers. Therefore, a firm should carefully study the value-creating processes of its customers.
ii. Evaluate differentiation strategies for enhancing customer value:

CA. Parag Gupta Ph.: +91 11 47665555 Paraggupta_ca@yahoo.co.in Costing & O.R.
World’s largest CA Final student’s consultancy group: http://groups.yahoo.com/group/costingbyparaggupta
Cost Accounting & Management - 47 -

This involves identification of the value-creating processes that distinguishes a firm’s products or
services from those of its competitors. This can be achieved in the following areas:
a. Product Features - that are aesthetically or functionally superior
b. Marketing Channels – that provide levels of responsiveness, convenience, variety and
information.
c. Service and Support – tailored to meet end-user and channel member (sophistication and
urgency or need).
d. Brand / lineage Positioning – that lends greater appeal to the company’s products on critical
selection criteria.
e. Price: including both net purchase price and cost savings available to the customer through the
financial services market.
iii. Determine the best sustainable differentiation strategies:
In order to prioritize its processes as sources of differentiation, a company must determine what
attributes of each process enhance customer value. The more unique a firm’s resources and skills,
the more sustainable is its differentiation advantage over competitors.

Steps in Vertical Linkage Analysis (Stage 3)

Vertical linkage analysis is a much broader application of internal cost and differentiation analysis that
includes all upstream and downstream value-creating processes throughout the industry. It considers all links
from the source of raw materials to the disposal and/or recycling of the product. It involves the following steps:
1. Identify the industry’s value chain and assign costs, revenues and assets to value creating
processes.
 The firm should identify the vertical linkages in the industry value chain, for example, the
petroleum industry consist of numerous value creating processes or activities, including
exploration, production, refining, marketing and distribution, which define the value chain for this
industry.
 One firm may participate in all parts of this value chain; another firm may participate in only a few.
 The information systems to identify and analyse these subtle relationship should be developed
 Costs, Revenues and Assets of each value-creating process may be determined based on
relevant cost approach, use of market prices, transfer prices, current replacement cost of assets,
etc.
2. Diagnose the cost drivers for each value-creating process:
 Different cost determinants should be identified for each value-creating process.
 Direct labour-based measures may be suitable for labour-intensive activities while operating
hours may be appropriate for machine based activities.
3. Evaluate the opportunities for sustainable competitive advantage:
 Sufficient qualitative information usually exists on a firm’s major value-creating processes and the
strategies for each. Such information will be difficult to obtain and may also be unreliable.
 Using benchmarking processes and by understanding how other companies compete in each
process of the industry value chain, a firm can use the qualitative analysis to seek
 out competitive niches even if financial data are unavailable.
 To evaluate the opportunities for competitive advantage in the global marketplace, firms need to
consider such things as a country’s values, political climate, environmental concerns, trade
relations, tax laws, inflation rates and currency fluctuations.

Strategic frameworks required for value chain analysis

Value chain analysis requires a strategic framework of focus for organizing internal and external information,
for analyzing information, and for summarizing findings and recommendations.

Recent concepts from strategists and organization experts lead to three strategic frameworks for
VCA.
1. Industry structure analysis:
2. Core competencies analysis and
3. Segmentation analysis

1. Industry Structure Analysis of Michael Porter

CA. Parag Gupta Ph.: +91 11 47665555 Paraggupta_ca@yahoo.co.in Costing & O.R.
World’s largest CA Final student’s consultancy group: http://groups.yahoo.com/group/costingbyparaggupta
Target costing, Value analysis & Life Cycle Costing - 48 -
(including techniques of profit improvement, cost reduction)

It is a five-factor model to organise information about an industry structure to evaluate its potential
attractiveness. Under this model, the profitability of an industry or market is measured by the long
term return on investment of the average firm depends largely on the following five factors that
influence profitability.
a) Bargaining power of buyers:
 The degree of buyer power generally depends on:
 Customer concentration (higher concentration of customers means greater negotiation
leverage).
 Propensity for customers to integrate backward (higher propensity for backward integration
means greater bargaining leverage);
 Costs of switching suppliers (lower switching costs means greater leverage for the buyer) and
 Number of alternative suppliers (higher alternatives indicate greater customer leverage.)
b) Bargaining power of suppliers:
Just as powerful buyers can squeeze profits by putting downward pressure on prices, suppliers
squeeze profits by increasing input costs. The same factors that determine the power of buyers also
determine the power of suppliers. The bargaining power of suppliers and buyers relative to the firm
depends on the relationships between their value chains. Identifying the specific activities involved
and the nature of their strengths and relationships can give important insights into the power balance
between the buyer and seller, and how it may be altered for, the firm’s benefit.
c) Threat of substitute products or services:
 Profit potential is determined by the maximum price that customers are willing to pay, which
in turn depends on the availability of substitutes.
 Products with few substitutes command higher prices than products with many close
substitutes since customer will prefer switching in the latter case.
 A thorough understanding of the value chains of buyers as they relate to the firm’s product
can help in assessing (and combating) the threat of substitution.
d) Threat of new entrants:
 When an industry is earning a return on invested capital above the cost of capital, that
industry will act as a magnet to firms outside the industry.
 Unless the entry of new firms is barred, the rate of profit must fall to the competitive level.
 Even the mere threat of entry may be sufficient to ensure that established firm’s constrain
their prices to the competitive level.
e) Intensity of competition:
 Markets experiencing rapid growth typically see less intense competition
 Rival companies can usually satisfy profitability and growth without having to take market
shares from their competitiors.

2. Core Competencies Analysis


Core competencies are created by superior integration of technological, physical and human resources,.
They represent distinctive skills as well as intangible, invisible, intellectual assets and cultural capabilities.
Cultural capabilities refer to the ability to manage change, the ability to learn and team working.
Organisations should be viewed as a bundle of a few core competencies, each supported by several
individual skills.
Core competence based diversification reduces risk and investment and increases the opportunities for
transferring learning and best practice across business units.

A core competence is identified by the following tests:


 Leverage test - Does it provide potential access to a wide variety of markets?
 Value enhancement test – Does it make a significant contribution to the customer’s perception
regarding benefits of the end product?
 Imitability test – Can it be imitated? Does it reduce the threat of imitation by competitors?

Applying the VCA approach to core competencies for competitive advantage includes the following
steps:

 Validate core competencies in current businesses: Core competencies should tie together the
portfolio of end products and help the firm excel in dominating its industry. Core competencies need
to be continually validated, due to continuous technological development over time.

CA. Parag Gupta Ph.: +91 11 47665555 Paraggupta_ca@yahoo.co.in Costing & O.R.
World’s largest CA Final student’s consultancy group: http://groups.yahoo.com/group/costingbyparaggupta
Cost Accounting & Management - 49 -

 Export or leverage core competencies to the value chains of other existing businesses: The
same set of core competencies can be exploited in multiple businesses by exporting core
competencies to the value chains of other existing businesses.
 Use core competencies to reconfigure the value chains of existing businesses: While firms may
manage their existing value chains better than their competitors, sophisticated firms work harder on
using their core competencies to reconfigure the value chain to improve payoffs. Otherwise
competitors may, exploit opportunities.
 Use core competencies to create new value chains: With strong core competencies in its existing
businesses, an organization can seek new customers by developing new value chains.
3. Segmentation Analysis (Stage 3)
Industries are sometimes collections of different market segments. Vertically integrated industries are
examples of a string of natural businesses from the source of raw material to end use by the final
consumer. Not all firms ill an industry participate in all segments.
Segmentation analysis will reveal competitive advantages and disadvantages of different segments. A
firm may use this information to decide whether to exit the segment, to enter a segment, reconfigure one
or more segments, or embark on cost reduction/differentiation programs.

Using the value chain approach for segmentation analysis, Grant recommended five steps:
1. Identify segmentation variables and categories:
The market may be divided into a number of segments using appropriate bases. Some approaches to
define market segments are:

Based on customer characteristics Based on Product


Geographic Use-type
Type of organization Usage
Size of firm Benefits sought Prices sensitivity
Life-style customers Competition Brand Loyalty
Age
Occupation

2. Construct a segmentation matrix: After customer and product-related variables have been selected
for identifying different segments, a segmentation matrix can be developed. Two or more dimensions
may be used to partition an industry. For example restaurants could be divided into four dimensions;
types of cuisine, price range, type of service (e.g. sit-down, buffet, cafeteria, take-out, fast food) and
location.
3. Analyse segment attractiveness: Competitive assessments using industry structure analysis or
core competencies analysis can also be used to evaluate the profitability of different segments.
However the competitive focus shifts to an analysis of the different segments. In addition, the
interrelationship among segments must be carefully considered.
4. Identify key success factors for each segment: Quality, delivery, customer
satisfaction market share, profitability and return on investment are common measures of corporate
success. In this regard, each segment must be assessed using the most appropriate key success
factors. Cost and differentiation advantages should be highlighted by these measures. Examination of
differences among segments in buyers’ purchase criteria reveal clear differences in key success
factors.
5. Analyse attractiveness of broad versus narrow segment scope: The competitive advantage of
each segment may be identified in terms of low cost and/or differentiation. Sharing costs across
different market segments may provide a competitive advantage. Taxing a narrow segment focus
may leave a firm vulnerable to competitors. A segment justifying a unique strategy may be of
worthwhile size to support a business strategy.

Limitations of Value Chain Analysis

1. Non-availability of data: Internal data on costs, revenues and assets used for value chain analysis
are derived from financial information of a single period. For long-term strategic decision-making,
changes in cost structures, market prices and capital investments etc. may not be readily available.
2. Identification of stages: Identifying stages in an industry’s value chain, is limited by the ability to
locate at least one firm that participates in a specific stage. Breaking a value stage into tow or more
stages when an outside firm does not compete in these stages is strictly judgment.
CA. Parag Gupta Ph.: +91 11 47665555 Paraggupta_ca@yahoo.co.in Costing & O.R.
World’s largest CA Final student’s consultancy group: http://groups.yahoo.com/group/costingbyparaggupta
Target costing, Value analysis & Life Cycle Costing - 50 -
(including techniques of profit improvement, cost reduction)

3. Ascertainment of cost, revenues and assets: Finding the costs revenues and assets for each
value chain activity poses/gives rise to serious difficulties. There is no scientific approach and much
depends upon trial and error and experimentation methods.
4. Identification of cost drivers: Isolating cost drivers for each value-creating activity, identifying value
chain linkages across activities, and computing supplier and customer profit margins present serious
challenges.
5. Resistance from employees: Value chain analysis is not easily understandable to all employees
and hence may face resistance from employees as well as managers.
6. Science Vs. Art: Value chain analysis is not an exact science. It is more “art” than preparing precise
accounting reports. Certain judgments and factors of analysis are purely subjective and differ from
person to person.

Impact of VCA on Organisational and Managerial Accounting

Management Accountants should recognize that the traditional, functional, internally oriented information
system is inadequate for the firm engaged in global competition. In order to facilitate Value chain analysis,
there should be a change in focus for management accounting. The Management Accountant’s role will be
significant in the following areas:
1. Need for education, training and awareness: Management Accountants should bring the
importance of customer value to the forefront of management strategic thinking. They should take the
initiate to bring the value chain message to major players in the firm through seminars, articles, value
chain examples and company-specific applications.
2. Exploring for information: VCA requires expertise in internal operators and information and also
demands a great deal of external information. Management accountants must seek relevant financial
and non-financial information from sources outside the organization.
3. Creativity: Management accountants must integrate databases and potential sources of timely
information on competitive forces confronting the business. This calls for innovation and creativity in
gathering and analyzing information for management decisions.
4. System design: Designing internal and external in to assist managers in planning, monitoring and
improving value-creating processes is another challenge facing management accountants.
5. Cooperation: Management accountants should solicit support from all senior mangers for allocating
resources to develop and improve value chain-oriented informationsystems. The management
accountant should ensure that the top management is committed to value chain analysis and the
organizational changes necessary for its successful implementation.

Difference between traditional management accounting & value chain analysis.


Traditional Management Accounting Value Chain analysis
Focus Internal External
Perspective Seeks cost reduction in Value added Seeks competitive advantage based on entire
process i.e. Sale Price- Cost of Raw set linked activities from suppliers to end-use
materials. customs.
Number of A single cost driver is adopted. Cost is Multiple cost drivers are adopted i.e:
Cost Drivers generally based on volume production  Structural drivers (e.g.scale, scope,
and sales. experience, technology and complexity)
 Executional drivers (e.g., participation
management and plant layout.
Use of Cost Application at the overall firm level A set of unique cost drivers is used for each
Driver (cost-volume-profit analysis) value activity.
Cost Application at the overall firm level A set of unique cost drivers is used for each
Containment (cost-volume-profit analysis) value activity.
Philosophy
Cost Seeks adhoc cost reduction solutions View cost containment as a function of the cost
Preferences by focusing on variance analysis, drivers regulating each value activity.
performance evaluation based on  Exploit linkages with suppliers
financial and quantitative data  Exploit linkages with customers
 Exploit process linkages within the firm
 “Spend to save”
Nature of Data Focus on control of manufacturing Focus on gaining advantage and not only on cost
CA. Parag Gupta Ph.: +91 11 47665555 Paraggupta_ca@yahoo.co.in Costing & O.R.
World’s largest CA Final student’s consultancy group: http://groups.yahoo.com/group/costingbyparaggupta
Cost Accounting & Management - 51 -

costs control and reduction.


Benchmarking Partially present Inter- finance Focus on full-fledged benchmarking learning
comparison, if any, is generally from competitors but exploiting one’s own
restricted to financial and operational strengths gain advantage
information.
Insights for Limited to some extent Identify cost drivers at the individual activity level
Strategic and develop cost/differentiation advantage either
Decisions. by controlling those drivers better than
competating by reconfiguring the value chain,
Quantify and assess “supplier power” and “buyer
power” a exploit linkages with suppliers and
buyers.

Question 13: What steps are involved in value chain analysis approach for assessing competitive
advantages? (4 Marks) May/05

Question 14: What is the concept of ‘Value-chain’ and why is it important for Cost Management?
(4 Marks) May/06
Ans.: Value chain is the linked set of value creating activities from the basic raw materials and components
sources to the ultimate end use of the product or service delivered to the customer.
The six business functions contained in the value chain are (i) Research and Development, (ii) Design (iii)
Production (iv) Marketing (v) Distribution and (vi) Customer service.
The objective of value chain is to serve as means of increasing the customer satisfaction and managing costs
effectively. Coordination of the individual parts of the value chain activities creates conditions to improve
customer satisfaction in terms of cost efficiency, quality and delivery. A firm which performs value chain
activities more efficiently and at a lower cost than its competitors will be able to gain competitive advantage.
The following methodology should be adopted.
1. The firm should identify the industry value chain and then assign costs, revenues and assets to
value activities.
2. Diagnose the cost drivers regulating each value activity.
3. Develop sustainable cost advantage either by controlling cost drivers better than competitors or
by reconfiguring the chain value.
By analyzing costs, revenues and assets in each activity systematically a company can achieve low cost.
Thus value chain helps managers in deciding how to apply the organization’s valuable physical and
human resources to each linked process so as to achieve cost effectiveness.
Question 15: “Cost can be managed only at the point of commitment and not at the point of incidence.
Therefore it is necessary to manage cost drivers to manage cost.” Explain the statement with reference to
structural and executional cost drivers. (5 Marks) Nov/07
Question 16: Define the term ‘value-chain’. Mention three useful strategic frameworks of the value-chain
analysis. (4 Marks) June/09-O.C.
Question 17: Differentiate between ‘Traditional Management Accounting’ and ‘Value Chain Analysis in the
strategic framework’. (5 Marks) Nov./08-N.C.

Question 18: Explain with a diagram the value chain activities within the firm with suitable classifications
under primary and support activities and also the industry value chain indicating what the end use consumer
pays for.
Ans.:

CA. Parag Gupta Ph.: +91 11 47665555 Paraggupta_ca@yahoo.co.in Costing & O.R.
World’s largest CA Final student’s consultancy group: http://groups.yahoo.com/group/costingbyparaggupta
Target costing, Value analysis & Life Cycle Costing - 52 -
(including techniques of profit improvement, cost reduction)

Industry Value Chain Value chain activities within the firm


Primary Activities Support Activities

ROD
Supplier
value Procurem
sumer pays for profit margin throughout

chain ent
Design
Firm Z
x y value
chain Technology
Production Development

Distributi
on value Marketing
chain
Human Resource
Management
End use consumer

Distribution
Buyer
value
chain Firm infrastructure

Service

Disposal
Recycle
value
chain

LIFE CYCLE COSTING

As per CIMA, Life-Cycle Costing (also known as Whole Life Costing) is the practice of obtaining over their life-
times, the best use of physical asset at the lowest total cost to the entity.

Life cycle costing (LCC), aims at cost ascertainment of a product, project etc. over its projected life. It is a
system that tracks and accumulates the actual costs and revenues attributable to cost object from its
inception to its abandonment.

Sometimes the terms, “cradle-to-grave costing” and “womb-to-womb costing” convey the meaning of fully
capturing all costs associated with the product from its initial to final stages. It is an approach used to provide
a long term picture of product line profitability, feedback on the effectiveness of life cycle planning cost data to
clarify the economic impact of alternatives chose in the design, engineering phase etc. It is also considered as
a way to enhance the control of manufacturing costs. The thrust of product life cycle costing is on the
distribution of costs among categories changes over the life of the product, as does the potential profitability
of a product. Hence it is important to track and measure costs during each stage of a product’s life cycle.

Product life cycle is a pattern of expenditure, sale level, revenue and profit over the period from new idea
generation to the deletion of product from product range.

Each product has a life cycle. Product life cycle spans the time from initial R&D on a product to when
customer servicing and support is no longer offered for the product. The life cycle of a product vary from a few
months to several years. For products like motor vehicles this time span may range from 5 to 7 years. For

CA. Parag Gupta Ph.: +91 11 47665555 Paraggupta_ca@yahoo.co.in Costing & O.R.
World’s largest CA Final student’s consultancy group: http://groups.yahoo.com/group/costingbyparaggupta
Cost Accounting & Management - 53 -

some basic pharmaceuticals, the time span may be 7 to 10 years. In case of cameras, photocopying
machines etc. the life is more than 100 years.

Phases in product life cycle

The four identifiable phases in the Product Life Cycle are (a) introduction (b) Growth (c) Maturity and (d)
Decline. A comparative analysis of these phases is given below:

Particulars Introduction Growth Maturity Decline


Phase I II III IV
Rise in sales Sales level off
Initial stages, Rise in sales levels
Sales Volume levels at decreasing and then start
hence low at increasing rates.
rates. decreasing.

High levels to
Retention of high Prices fall closer Gap between
Prices of cover initial costs
level prices except in to cost, due to effect price and cost is
Products and promotional
certain cases* of competition further reduced.
expenses.

Highest, due to
Total expenses Ratio reaches a
Ratio of effort needed to Reduced sales
remain the same, normal % of sales.
promotion inform potential promotional efforts
while ratio is reduced Such normal %
expenses to customers, launch as the product is no
due to increase in becomes the
Sales products, distribute longer in demand.
sales. industry standard.
to customers etc.

Starts
Negligible and Entry of a large Fierce disappearing due to
Completion
insignificant number of competitors Competition withdrawal of
products

Normal rate of Declining profits


NIL due to Increase at a rapid profits since costs due to price
Profits
heavy initial costs. pace. and prices are competition, entry of
normalized. new products etc.

Manufacturing cost/ Plants reused/


R & D cost/Design Promotional cost/ Distribution costs/ sold/scrapped
Costs
cost. Capacity costs Product support /related costs.
cost.
* In the growth stage, the firm will maintain the prices at the high levels, in order to realize maximum profits.
Price reduction will not be undertaken unless (a) the low prices will lead

The major characteristics of product life-cycle concept are as follows:


1. The products have finite live and pass through the cycle of development, introduction, growth,
maturity decline and deletion at varying speeds.
CA. Parag Gupta Ph.: +91 11 47665555 Paraggupta_ca@yahoo.co.in Costing & O.R.
World’s largest CA Final student’s consultancy group: http://groups.yahoo.com/group/costingbyparaggupta
Target costing, Value analysis & Life Cycle Costing - 54 -
(including techniques of profit improvement, cost reduction)

2. Product cost, revenue and profit patterns tend to follow predictable courses through the product life
cycle, Profits first appear during the growth phase and after stabilizing during the maturity phase
declines at the point of deletion.
3. Profit per unit varies as products move through their life cycles.
4. Each phase of the product life cycle poses different threats and opportunities that give rise to different
strategic actions.
5. Products require different functional emphasis in each phase-such as all R&D emphasis in the
development phase and a cost control emphasis in the decline phase.
6. Finding new uses or getting the present users to increase their consumption, may extend the life of
the product.

Stages in products life cycle


The life cycle of manufactured product will consist of the following stages:
1. Market Research – what customer wants, how much he is prepared to pay for it & how many he will
buy.
2. Specification – Conversion of market research into a detailed specification. Max. Permissible
manufacturing cost, functionalities, no. required & delivery date is provided to engineering
department.
3. Design – Design engineers produce drawings & process schedules in accordance with the
specification provided.
4. Prototype Manufacture - Prototypes are produced on the basis of these drawings, which help in
developing the product & ascertaining that whether the product meets the specifications.
5. Development – It is the period of testing & changing of initial product manufactured till it meets the
laid down specifications.
6. Tooling – If product meets the required specifications & is profitable it is tooled up for production i.e.
building of production line, building expensive jigs & buying special purpose tools leading in very large
initial investment.
7. Manufacture – Involves purchase of raw material & bought out components, arranging for labour &
facilities for manufacture.
8. Selling & Distribution – Money is spend on campaign to sell the product & product is distributed to
sales outlets & to customers.
9. After Sales Support – Ensuring availability of spares & expert servicing throughout life of product.
10. Decommissioning & replacement – After the life of the product, the plant used to produce the product
is sold/scrapped/or disposed of in a manner, which is suitable.

Features/Characteristics of Product Life Cycle Costing:


Product life cycle costing is important due to the following features:

1. Product life cycle costing involves tracing of costs and revenues of each product over several
calendar periods throughout their entire life cycle. Costs and revenues can be analysed by time
periods, but the emphasis is on costs and revenue accumulation over the entire life cycle for each
product.
2. Product life cycle costing traces research and design and development costs, incurred to individual
products over their entire life cycles, so that the total magnitude of these costs for each individual
product can be reported and compared with product revenues generated in later periods.

Life cycle costing therefore ensures that costs for each individual product can be reported and compared with
product revenues generated in later periods. Hence the costs are made more visible.

Importance of Product Life cycle:


(i) When non-production costs like costs associated with R & D, design, marketing, distribution and
customer service are significant, it is essential to identify them for target pricing, value engineering
and cost management. For example, a poorly designed software package may involve higher costs
on marketing, distribution and after sales service.
(ii) There may be instances where the pre-manufacturing costs like R & D and design are expected to
constitute a sizeable portion of life cycle costs. When a high percentage of total life cycle costs are
likely to be so incurred before the commencement of production, the firm needs an accurate
prediction of costs and revenues during the manufacturing stage to decide whether the costly R &
D and design activities should be undertaken.

CA. Parag Gupta Ph.: +91 11 47665555 Paraggupta_ca@yahoo.co.in Costing & O.R.
World’s largest CA Final student’s consultancy group: http://groups.yahoo.com/group/costingbyparaggupta
Cost Accounting & Management - 55 -

(iii) Many costs are locked in at R & D and design stages. Locked in or Committed costs are those
costs that have not been incurred at the initial stages of R & D and design but that will be incurred
in the future on the basis of the decisions that have already been taken. For example, the adoption
of a certain design will determine the product’s material and labour inputs to be incurred during the
manufacturing stage. A complicated design may lead to greater expenditure on material and labour
costs every time the product is produced. Life cycle budgeting highlights costs throughout the
product life cycle and facilitates value engineering at the design stage before costs are locked in.
Total life-cycle costing approach accumulates product costs over the value chain. It is a process of
managing all costs along the value chain starting from product’s design, development,
manufacturing, marketing, service and finally disposal.

Benefits of Life cycle costing:


1. The life cycle concept results in earlier actions to generate revenue or to lower costs than otherwise
might be considered.
2. Better decisions should follow from a more accurate and realistic assessment of revenues and costs,
at least within a particular life cycle stage.
3. Life cycle thinking can promote long-term rewarding in contrast to short-term profitability rewarding.
4. The life cycle concept helps managers to understand acquisition costs vs. operating and support
costs, i.e. to find a correct balance between investment costs vs. operating expenses.

Here are some of the most often cited LCC limitations:


1. It is not an exact science, everyone gets different answers and the answers are neither wrong nor
right—only reasonable or unreasonable. LCC experts do not exist because the subjects are too broad
and too deep.
2. It models operate with limited cost databases and the cost of acquiring data in the operating and
support areas is both difficult to obtain and expensive to acquire.
3. It requires a scenario for: how the money expenditure model will be constructed for acquisition of
equipment, how damage will occur, etc.—most details require extensive extrapolations and obtaining
facts is difficult.
4. It results are not good budgeting tools. They’re effective only as comparison/trade-off tools and
producing good LCC results requires a project team approach because specialized expertise is
needed.

Product Life Cycle Costing is considered important due to the following reasons:
1. Time based analysis: Life cycle costing involves tracing of costs and revenues of each product over
several calendar periods throughout their life cycle. Costs and revenues can be analysed by time
periods. The total magnitude of costs for each individual product can be reported and compared with
product revenues generated in later periods.
2. Overall Cost Analysis: Production costs are accounted and recognized by the routine accounting
system. However non-production costs like R&D, design, marketing, distribution, customer service
etc. are less visible on a product-by-product basis. Product Life Cycle Costing focuses on recognizing
both production and non-production cost.
3. Pre-production Costs analysis: The development period for R&D and design is long and costly. A
high percentage of total product costs may be incurred before commercial production begins. Hence,
the company needs accurate information on such costs for deciding whether to continue with the
R&D or not.
4. Effective Pricing Decisions: Pricing Decisions, in order to be effective, should include market
consideration on the one hand and cost considerations on the other. Product Life Cycle Costing and
Target Costing help analyse both these considerations and arrive at optimal price decisions.
5. Better Decision Making: Better decisions should follow from a more accurate and realistic
assessment of revenues and costs, at least within a particular life cycle stage.
6. Long Run Wholistic view: Product life cycle thinking can promote long-term rewarding in contrast to
short-term profitability rewarding. It provides an overall framework for considering total incremental
costs over the entire life span of a product, which in turn facilitates analysis of parts of the whole
where cost effectiveness might be improved.
7. Life Cycle Budgeting: Life Cycle Budgeting. i.e. Life Cycle Costing with Target Costing principles,
facilitates scope for cost reduction at the design stage itself. The Company stands to benefit since
costs are avoided before they are committed or locked in.

CA. Parag Gupta Ph.: +91 11 47665555 Paraggupta_ca@yahoo.co.in Costing & O.R.
World’s largest CA Final student’s consultancy group: http://groups.yahoo.com/group/costingbyparaggupta
Target costing, Value analysis & Life Cycle Costing - 56 -
(including techniques of profit improvement, cost reduction)

8. Review: Life cycle Costing provides scope for analysis of long term picture of product line profitability,
feedback on the effectiveness of life cycle planning and cost data to clarify the economic impact of
alternatives chosen in the design, engineering phase etc.

Question 19: What is Product Life-cycle Costing? Describe its characteristics & benefits.
(5 Marks)Nov/04, (5 Marks) June/09-N.C. & (4 Marks) May/10-O.C.
Question 20: What is total-life-cycle costing approach? Why is it important? (4 Marks)May/06
Question 21: What is product life cycle costing? What are the costs that you would include in product life
cycle cost? (4 Marks)May/07
Question 22: A company’s four products M, N, O and P are in the market. Identify the phase of life cycle for
each product with a brief reason.

M : There is a lot of competition. Quantity sold has been increasing at 10%, 8% and 7% in the last 3 years.

N : Until last year, N had no competition. Suddenly the company finds 4 new products very similar to N in the
market. However, N continues to have good sales.

O : There is intense competition. Achieving targeted sales is becoming increasingly difficult. Hence the
company is introducing slightly modified features in the fresh production.

P : Huge inventory of P is available. P is being sold, but there are many products in the market which are
priced lesser than P, but have the same utility as P. (4 Marks) Nov./10-O.C.

[Ans.: M – Maturity; N – Growth; O – Decline; P – Maturity phase but since P’s inventory is huge it might be
in Decline phase.]

Question 23: Meena is a news reporter and feature writer for an economic daily. Her assignment is to
develop a feature article on ‘Product Life-cycle Costing’, including interviews with the Chief Financial Officers
(CFO) and operating managers. Meena has been given a liberal budget for travel so as to research into
company’s history, operations, and market analysis for the firm she selects for the article.
Required :

Meena has asked you to recommend industries and firms that would be good candidates for the article. What
would you advice? Explain your recommendations. (3 Marks) June/09-O.C.

[Ans.: The product life cycle span the time from the initial R & D on a product to when customer service and
support is no longer offered for that product.
Life Cycle Costing technique is particularly important when:
(a) High percentage of total life-cycle costs are incurred before production begins and revenue are earned
over several years and
(b) High fraction of the life cycle costs are locked in at the R & D and design stages.
Meena should identify those industries and then companies belonging to those industries where above
mentioned feature are prevalent. For example, Automobile and Pharmaceutical Industries companies like
Tata Automobile, M&M, Ranbaxy and Dabur will be good candidates for study on product life cycle costing.]

It is advisable to go through Chapter “Capital Budgeting” of Strategic


Financial Management, before attempting practical questions of LCC.

Question 24: Indo Gulf Fertilizers Ltd. Supports the concepts of the terotechnology or life cycle costing for
new investment decisions covering its engineering activities. The finalized of the philosophy is now well
established and its principles extended to all other areas of decision making.

The company is to replace a number of its machines and the Production Manager is to run between the ‘X’
machine, a more expensive machine with a life of 12 years, and the ‘W’ machine with a life of 6 years. If the
‘W’ machines chosen it is likely that it would be replaced at the end of 6 years by another ‘W’ machine. The

CA. Parag Gupta Ph.: +91 11 47665555 Paraggupta_ca@yahoo.co.in Costing & O.R.
World’s largest CA Final student’s consultancy group: http://groups.yahoo.com/group/costingbyparaggupta
Cost Accounting & Management - 57 -

pattern of maintenance and running costs differs between the types of machine and relevant data are shown
below:
(`)
X W
Purchase price 19,000 13,000
Trade-in-value 3,000 3,000
Annual repair costs 2,000 2,600
Overhaul costs (p.a) (at year 8) 4,000 (at year 4) 2,000
Estimated financing costs averaged over machine life (p.a) 10% 10%

You are required to recommend, with supporting figures, which machine to purchase, stating any
assumptions made.PVIFA (10,6) = 4.36, PVIFA (10,12) = 6.81
(10 Marks) ICWA Dec./08 & (10 Marks) ICWA June/10
[Ans.: Machine X-Life 12 years: Annualized equivalent is `4,925/- & Machine W-Life 12 years: Annualized
equivalent is `5,508/-; Recommendation: Purchase machine ‘X’]
[Hint.: Whenever life of two machines, products, etc. are not equal, we are supposed to take decision on the
basis of annualized equivalent value of NPV instead of just NPV]
Question 25 [ABC+Life Cycle Costing]: Activities have been identified and the budget quantifies for the
three months ended 31 March,2001 as follows:-

Activities Cost Driver Unit Basis Units of Cost Driver Cost (`000)
Product Design Design hours 8000 2000(see note 1)
Purchasing Purchase order 4000 200
Production Machine hours 12000 1500(See note 2)
Packing Volume (Cu.m.) 20000 400
Distribution Weight (Kg.) 120000 600

Note1: This includes all design costs for new products released this period.

Note 2: This includes a depreciation provision of `300000 of which `8000 applies to 3 months depreciation on
a straight-line basis for a new product (NPD).The reminder applies to other products.

New product NPD is included in the above budget. The following additional information applies to NPD.
(1) Estimated total output over the product life cycle: 5000 units (4 years life cycle)
(2) Product design requirement: 400 design hours.
(3) Output in quarter ended 31 March 2001:250 units.
(4) Equivalent batch size per purchase order: 50 units.
(5) Other product unit data: production time 0.75 machine hours: volume 0.4 cu. Meters: weight 3 kg.

Required:-

Prepare a unit overhead cost for product NPD using an activity based approach which includes an
appropriate share of life cycle costs using the information provided.

[Ans.: Total cost is `144.60]


[Hint: Depreciation of NPD is `1,28,000 for total life]
Question 26: A housewife is looking at ways of producing domestic hot water and considers possibility an
electric immersion heater having an installation cost of `160 and estimated annual electrical charges of `200,
and a gas boiler with an installation cost of `760 with fuel bills of `80.
Assuming yourself as a consultant to this cost-conscious-housewife, advise her suitably by comparing-
two systems, on the basis of (i) total expenditure, and (ii) present value, over a 5-year period. Take interest at
9 per cent.
What will be your recommendation if you consider both the equipments for a 8 years period?

[Ans.: Total cost-5 Years:-> Heater: `1160, Boiler: `1160; 8 Years:-> Heater: `1760, Boiler: `1400; Present
Value- 5 Years:-> Heater: `938, Boiler: `1071; 8 Years:-> Heater: `1267, Boiler: `1203]

CA. Parag Gupta Ph.: +91 11 47665555 Paraggupta_ca@yahoo.co.in Costing & O.R.
World’s largest CA Final student’s consultancy group: http://groups.yahoo.com/group/costingbyparaggupta
Target costing, Value analysis & Life Cycle Costing - 58 -
(including techniques of profit improvement, cost reduction)

Question 27: Gemini enterprises currently makes as many units of part No. X-248 as it needs. Sen, general
manager of Gemini enterprises, has received a quotation from another company for making part No. X-248.
Zedco will supply 1,000 units of parts No.X-248 per year at `50 per units. Zedco can begin supply on 1st
July,1998 and continue for 5 year, after which Gemini will not need the part. Zedco can accommodate any
change in Gemini’s demand for the part and will supply it for `50 regardless of quantity.
Shah, the controller of Gemini enterprises, reports the following costs for manufacturing 1,000 units of part
No. X-248:
`
Direct materials 22,000
Direct labour 11,000
Variable manufacturing overhead 7,000
Depreciation on machine 10,000
Product and process engineering 4,000
Rent 2,000
Allocation of general plants overhead costs 5,000
Total costs 61,000

The following additional information is available:

a. Part X-248 is made on a machine used exclusively for its manufacture. The machine was acquired on
1st July,1997 at a cost of `60,000. The machine has a useful life of six year and a zero terminal
disposal price. Depreciation is calculated on straight-line basis.
b. The machine could be sold today for `15,000.
c. Product and process engineering costs are incurred to ensure that the manufacturing process for part
No. X-248 works smoothly. Although these costs are fixed in the short run, with respect to units of
part No. X-248, they can be saved in the long run if part No.X-248 is no longer produced. If part No.X-
248 is outsourced, product and process engineering costs of `4,000 will be incurred for 1997-98 but
not thereafter.
d. Rent costs of 2,000 are allocated to products on the basis of the floor space used for manufacturing
the product. If part number X-248 is discontinued, the space currently used to manufacture it would
become available. The company could then use the space for storage purpose and saves `1,000
currently paid for outside storage.
e. General plant overhead costs are allocated to each department on the basis of direct manufacturing
labour costs. The costs will not change in total. But no general plant overhead will be allocated to part
number X-248 if the part is outsourced.
Assume that Gemini requires a 12% rate of return for this project. The following information may be useful:

Year Present value factor at 12%


0 1.000
1 0.893
2 0.797
3 0.712
4 0.636
5 0.567
Required:

1. Should part No.X-248 be outsourced? Prepare a quantitative analysis.


2. State any sensitivity analysis that seems to be advisable. Do not perform any sensitivity
calculations.
3. Sen is particularly concerned about his bonus for 1997-98. The bonus is based on accounting
income of Gemini enterprises. What decision will Sen make if he wants to maximize his bonus for
1997-98? (16 Marks) May/98
[Ans.: NPV is `(3025), so it is desirable to manufacture the part internally]
Question 28: S engineering company is considering to replace or repair a particular a particular machine,
which has just broken down. Last year this machine cost `20,000 to run and maintain. These costs have been
increasing in real terms in recent years with the age of the machine. A further useful life of 5 year is expected,
if immediate repair of `19,000 are carried out. If the machine is not repaired it can be sold immediately to
realize about `5,000(ignore loss/gain on such disposal).

CA. Parag Gupta Ph.: +91 11 47665555 Paraggupta_ca@yahoo.co.in Costing & O.R.
World’s largest CA Final student’s consultancy group: http://groups.yahoo.com/group/costingbyparaggupta
Cost Accounting & Management - 59 -

Alternatively, the company can buy a new machine for `49,000 with an expected life of 10year with no
salvage value after providing depreciation on straight line basis. In this case, running and maintenance cost
will reduce to `14,000 each year and are not expected to increase much in real term for a few years at least.
S engineering company regard a normal rate of 10% p.a. after tax as a minimum requirement on any new
investment. Considering capital budgeting techniques, which alternative which you choose? Take corporate
tax rate of 50% and assume that depreciation on straight line basis will accepted for tax purpose also.
Given cumulative present value of Re. 1 p.a. at 10% for 5 year `3.791,10 year `6.145.

[Ans: Present value of cash outflow: Alt I – `12506; Alt II – `11710]

CA. Parag Gupta Ph.: +91 11 47665555 Paraggupta_ca@yahoo.co.in Costing & O.R.
World’s largest CA Final student’s consultancy group: http://groups.yahoo.com/group/costingbyparaggupta
Service Sector

Services sector companies provide their customers with services or intangible products. The activities of
service sector may be used for both: (i) Provision of services to outside customers (ii) Provision of services
internally. The types of service that may be provided, by service sector are of diverse nature and they have
their own peculiarities, requirements, different cost accounting treatment but the general principles of costing
discussed in earlier chapters relating to manufacturing sector will also apply to service sector.

MAIN CHARACTERISTIC OF SERVICE SECTOR:


1. Activities are labour intensive: The activities of service sector generally are labour intensive. The direct
material cost is either small or non-existent. For example, cost of stationery used by a professional
consultant for expressing an opinion in back and white, for a client will be small or even non-existent in
case he gives verbal opinion. In the preceding example direct labour cost content of a service is
significant.
2. Cost-unit is usually difficult to define: The selection of cost unit usually, for service sector is difficult to
ascertain as compared to the selection of cost unit for manufacturing sector. The following table provides
some examples of the cost units for service sector.
(A) To External Customer Cost Unit
i. Hotel Bed nights available, Bed night occupied
ii. School Student hours, Full time students
iii. Hospital Patient per day, Room per day
iv. Accounting firm Charged out client hours
v. Transport Passenger km., quintal km.
(B) Internal Services Cost Unit
i. Staff canteen Meals provided, No. of staff
ii. Machine maintenance Maintenance hours provided to user department
iii. Computer department Computer time provided to user department

3. Product costs in service sector: Costs are classified as product or period costs in manufacturing sector
for various reasons. These are:
 To determine the unit manufacturing costs so that inventories can be valued and selling prices created
and verified.
 To report production costs on income statement
 To analyse costs for control purposes.

Differences between manufacturing and service sector.


1. The difference between manufacturing and service sector is that in service sector there is no physical
product that can be stored, assembled and valued. Services are rendered and cannot be stored up or
placed in a vault.
2. In service sector the cost of material is insignificant.
3. For computing unit cost of services the most important cost would be professional’s labour cost.
4. The direct labour cost is traceable to service rendered.
5. In addition to labour cost the service sector like manufacturing sector incurs various overhead cost. In
service sector those overhead costs, which are incurred for offering a service, are classified as
service overheads (like factory overhead in manufacturing sector).
6. Examples: Rendering a loan service, representing someone in court of law or selling an insurance
policy are typical services performed by professionals.
CA. Parag Gupta Ph.: +91 11 47665555 Paraggupta_ca@yahoo.co.in Costing & O.R.
World’s largest CA Final student’s consultancy group: http://groups.yahoo.com/group/costingbyparaggupta
Cost Accounting & Management - 61 -

Collection of costing data in service sector


1. Classification: Costs are accumulated under various heads for control purpose and for decision
making.
2. Grouping: Costs are then grouped under fixed costs and variable costs.
3. Put in a particular format: They are then put in a format, which depends upon the nature of industry
and the need of the management.
4. Period: For preparing a cost sheet under operating costing, costs are usually accumulated for a
specified period viz. a month, a quarter or year etc.
5. Composite cost units: Often composite cost units such as passenger km, bed, nights etc. are used
by these organizations for ascertaining the cost of these services respectively.

Different costing methods used in service sector:


1. Job Costing method: In job costing method the cost of a particular service is obtained by assigning
costs to a distinct identifiable service. In service sector like Accounting firm, Advertising campaigns
etc. job costing method is used.
2. Process Costing method: In process costing system the cost of a service is obtained by assigning
costs to masses of similar unit and then computing unit cost on an average basis. Retail banking,
Postal delivery, Credit card etc. uses process costing method.
3. Hybrid costing method: Many companies uses a method of costing which is neither job costing nor
process costing method. They in fact uses a hybrid costing method which combines elements of both
job costing and process costing methods.

Job costing method in service sector: The two costs, which are incurred in service sectors, are:
 Direct labour
 Service overheads

For ascertaining the price of a service provided by service sector if job-costing method is followed, the costs
for each job are to be monitored continuously.

There are two main uses of this job costs information:


1. To guide decisions on job pricing
2. To assist in cost planning and cost control

The five steps, which are generally adopted for assigning costs to individual jobs, are as follows:
1. Identify the job that is chosen as cost object: For instance, litigation work for Motorola India Ltd.
By Dua & Associates by assuming that work requires 100 budgeted hours of professional labour.
2. Identify the direct cost categories for the job: In the above example the professional hours
required for doing litigation work is a direct cost.
3. Identify indirect costs (overheads) associated with the job: This step requires identifications of
indirect costs incurred for providing services. These costs may include the costs of support labour,
computer time, travel, telephone/fax machine, photocopying etc.
4. Select the cost allocation base to be used in assigning each indirect cost to the job: This step
requires the selection of cost allocation base that has a cause and effect relationship between
changes in it and changes in the level of indirect costs. The allocation base suitable for allocating
indirect cost of law firm is professional labour hours.
5. Identify the rate per unit of the cost allocation base used to allocate cost to the job: The
budgeted indirect cost allocation rate is computed by using following formula.
Indirect Cost Pool
Cost Driver

Customer Costing in service sector: The customer costing is a new approach to management. The theme
of this approach is customer satisfaction. In some service industries such as public relations, the specific
output of industry may be difficult to identify and even more difficult to quantify. Customer costing analyses
the costs incurred to earn the revenues from customers. A customer cost hierarchy, categorizing the costs
relating to the customers into different cost pools on the basis of different types of cost drivers or different
degrees of difficulty in determining cause and effect relationship, is used. Further where there are multiple
customers, identifying support activities i.e. common costs with particular customer may be more problematic.
In such cases, it is important to cost customers. An ABC analysis of customers profitability provides valuable
CA. Parag Gupta Ph.: +91 11 47665555 Paraggupta_ca@yahoo.co.in Costing & O.R.
World’s largest CA Final student’s consultancy group: http://groups.yahoo.com/group/costingbyparaggupta
Service Sector - 62 -

information to help management in pricing customer. Consider a banking sector A bank’s activities for
customer will include the following types of activities. These are:
a) Stopping a cheque
b) Withdrawal of cash
c) Updation of pass book
d) Issue of duplicate pass book
e) Returning a cheque because of insufficient funds
f) Clearing of a customer cheque.

Different customers or categories of customers use different amount of these activities and so customer
profiles can be build up and customer can be charged according to the cost to serve them.
For example:
1. In Computer Institute the cost of providing a course for enrolled students may be determined by a
variety of factors, such as type of course (Oracle or Java) and the level of course (introduction or
advanced).
2. A hotel may have activities that are provided for specific types of customers such as well laid
gardens, swimming pool and a bar. Older guest may appreciate and use the garden, families the
swimming pool and business guests the bar. If the activities are allocated to relevant guest a correct
cost per bed occupied can be calculated for each type of category.
3. A courier service where the costs to serve the customers vary with the type of service selected by
customers (how fast the package is to be delivered), the destination, weight, size of package and
whether the package is to be collected from customer’s location or will be dropped at the office of the
courier firm. The firm could use an efficient system of internet strategy to accomplish the tasks like
preparing labels at the customer’s site, arranging for pick up, dropping at the destination and tracking
and tracing packages. The system calculates the freight charges, invoices customers daily and
produces customized reports.

For customer costing purpose, the costs are divided into following categories. These are
1. Customer Specific costs: These are direct and indirect cost of providing service to customer plus
customer related cost assigned to each customer. For example cost of express courier service to a
client/customer who requests overnight delivery of some agreement.
2. Customer-line categories: These are the cost which are broken into the broad categories of
customers and not individual customer.
3. Company costs: These are those costs which are not allocated to either customer line or individual
customers but charge to company. The example is the cost of advertisement to promote sale of
service.

Pricing by service sector


1. The service sector follows a different approach for pricing their service. Although a service has no
physical existence it must be priced and billed to customers.
2. Most service organizations use a form consisting of time and material pricing to arrive at the price of a
service.
3. Service companies such as appliance repair shops, automobile repair business calculate their prices
by using two computations, one for labour and other for materials and parts.
4. A mark up percentage is used to add the cost of overhead to the direct cost of labour, materials and
parts. If materials and parts are not part of service being performed, then only direct labour costs are
used as basis for determining price.
5. For professionals such as accountants and consultants direct labour costs and apportioned overhead
and indirect costs are considered for pricing.

Question 1: Give an appropriate cost unit for each of the following service sectors:
(i) Hotel
(ii) School
(iii) Hospital
(iv) Accounting firm
(v) Transport
(vi) Staff Canteen
(vii) Machine maintenance
(viii) Computer Department (4 Marks) June/09-N.C.

CA. Parag Gupta Ph.: +91 11 47665555 Paraggupta_ca@yahoo.co.in Costing & O.R.
World’s largest CA Final student’s consultancy group: http://groups.yahoo.com/group/costingbyparaggupta
Cost Accounting & Management - 63 -

Question 2: Explain which features of the Service organization may create problem for the application of
activity-based costing. (4 Marks) May/05

[Ans.: The following may create problem for adoption of ABC system in service organization –
(i) Facility sustaining costs (such as property, rents etc.) represent a significant portion of total costs and
may only be avoidable if the organization ceases business. It may be impossible to establish appropriate cost
drivers.
(ii) It is often difficult to define products where they are of intangible nature. Cost objects can therefore be
difficult to specify.
(iii) Many service organizations have not previously had a costing system and much of the information
required to set up a ABC system will be non-existent. Therefore introduction of ABC may be expensive.]
Question 3: Write short notes on pricing by service sector. (3 Marks) Nov./03 & (4 Marks) Nov./05
Question 4: How will you apply customer costing in service sector? Explain with the help of a suitable
example. (4 Marks) May/06
Question 5: Discuss with examples, the basic costing methods to assign costs to services. (5 Marks) May/07
Question 6: Explain the main characteristics of Service sector costing. (5 Marks) June/09-O.C.
Question 7: "Customer profile is important in charging cost." Explain this statement in the light of customer
costing in service sector. (5 Marks) Nov./09-N.C.
Question 8: The Bharat Hotel has annual fixed costs applicable to rooms of `7,50,000 for a 300-room hotel
with average daily room rates of `50 and average variable costs of `10 for each room rented.

The hotel operates 365 days per year. It is subject to an Income-tax rate of 40%.

You are required to:


i) Calculate the number of rooms the Hotel must rent to earn a net Income after tax of `6,00,000, and
ii) Compute the Break-even Point in terms of number of room rented. Dec./99-ICWA

[Ans.: (i) 43750 Room days (ii) 18750 Room days]


Question 9: A hotel operated by a company has 180 single rooms and 60 double rooms. The rent of the
double rooms is set at 160% of the rent of the single rooms. The operational costs per day per room are
estimated as under :

Single Rooms Double Rooms


` `
Variable costs 300 500
Fixed costs 500 780

The average occupancy of both the single rooms and double rooms is expected to be 85% throughout a year
of 365 days. In fixing the room rent, the company desires to earn a margin of safety of 20%. The hotel has to
pay a tax of 20% on its tariff.

Required:

(i) Calculate the tariff per day per (1) Single room and (2) Double room.
(ii) The hotel intends to reserve the normal occupancy of 12 single rooms for one of its valued corporate
customers at a discount (excluding tax) of 10% of the rent. What increase in the occupancy of the remaining
single room days is required to compensate the loss arising from the discount. (7 Marks) May/04

[Ans.; (i) (1) `1155 approx. (2) `1848 approx.; (ii) 551 room-days approx.]
[Hint.: The margin of safety is 20%, therefore break even point is 80%
At 80% that is at BEP the revenue (`2,60,61,000 × 80%+ `4,24,42,200) = `6,32,91,000
∴Total desired revenue = 6,32,91,000 ÷ 80% = `7,91,13,750]
[Note.: Suggested answers of ICAI has forgotten to multiply Total Variable cost by 80% & has wrongly
calculated, revenue at BEP as (`2,60,61,000 + `4,24,42,200) = `6,85,03,200]
CA. Parag Gupta Ph.: +91 11 47665555 Paraggupta_ca@yahoo.co.in Costing & O.R.
World’s largest CA Final student’s consultancy group: http://groups.yahoo.com/group/costingbyparaggupta
Service Sector - 64 -

Question 10: Elegant Hotel has a capacity of 100 single rooms and 20 double rooms. It has a sports centre
with a swimming pool, which is also used by persons other than resident of the hotel. The hotel has a
shopping arcade at the basement and a specialty restaurant at the roof top. The following information is
available:

(i) Average occupancy: 75% for 365 days of the year.


(ii) Current costs are:
(iii)
Variable cost Fixed cost
`/per day `/per day
Single Room 400 200
Double Room 500 250

(iv) Average sales per day of restaurant `1,00,000; contribution is at 30%. Fixed cost `10,00,000.
(v) The sports centre/swimming pool is likely to be used by 50 non-residents daily; average
contribution per day per non-resident is estimated at `50; fixed cost is `5,00,000 per annum.
(vi) Average contribution per month from the shopping arcade is `50,000; fixed cost is `6,00,000 per
annum.

You are required to find out:

(a) Rent chargeable for single and double room per day, so that there is a margin of safety of 20% on
hire of rooms and that the rent for a double room should be kept at 120% of a single room.
(b) Evaluate the profitability of restaurant, sports centre and shopping arcade separately.

[Ans.: (a) Single Room: `655 approx. & Double Room: `786 approx. (b) `9950000, `412500, Nil]
[Hint.: The margin of safety is 20%, therefore break even point is 80%
At 80% that is at BEP the revenue (`1,36,87,500 × 80%+ `68,43,750) = `1,77,93,750
∴Total desired revenue = `1,77,93,750 ÷ 80% = `2,22,42,187.5]
[Note.: Suggested answers of ICAI has forgotten to multiply Total Variable cost by 80% & has wrongly
calculated, revenue at BEP as (`1,36,87,500 + `68,43,750) = `1,72,46,250] (15 Marks) Nov./97
Question 11: Kangan Resorts operates a lodging house with attached facilities of a shopping arcade and
restaurant on a National Highway. The following details are available:

(i) The lodging house has 40 twin-bedded rooms, which are to be rented for `200 per night on double
occupancy basis. The occupancy ratio is expected at 85% and always both the beds in the room will be
occupied. The lodging facilities are operated, for 200 days in the year during foreign tourists season time only.
(ii) As per past record the spending pattern of each tourist staying in the lodge will be as under:
`50 per day in the shopping arcade and `80 per day in the restaurant.
(iii) Ratios of variable cost to respective sales volume are:
Shops Restaurant
50% 60%
(iv) For the lodging house the variable cost on house-keeping and electricity will amount `30 per day per
occupied room.
(v) Annual fixed overhead for the entire complex is estimated at `10,00,000.

Required:

(i) Prepare an income statement for the next year.


(ii) The Lodging House Manager suggests a proposal of reducing room rent to `150 per day on double
occupancy basis, which will increase occupancy level to 95%. Should the proposal be accepted or not?

[Ans.: (i) Estimated Profit `931200 (ii) Proposal should not be accepted as it will result in reduction of profits
by `152800] (7 Marks) May/08 & Dec./02 (ICWA Final-Adapted)
Question 12: A limited company operates a lodging house with a restaurant, shops and recreational facilities
attached. Its manager has entrusted you with the planning of the coming year’s operations, more particularly
on the level of profits the company was likely to earn.
CA. Parag Gupta Ph.: +91 11 47665555 Paraggupta_ca@yahoo.co.in Costing & O.R.
World’s largest CA Final student’s consultancy group: http://groups.yahoo.com/group/costingbyparaggupta
Cost Accounting & Management - 65 -

The lodging house has 100 double-bed room, which are likely to be rented at `150 per day. The manager
expects an occupancy ratio of 75% for a period of 250 days during the tourist season. It is also anticipated
that both the beds in a room will be occupied during the period.

Each person staying in the lodging house is expected to spend on the basis of past statistics `30 per day in
the shops attached to the lodge and `60 per day in the restaurant. The recreational facilities are not charged
to the customer.

Some other relevant data available to you is as under:

(i) Variable cost to volume ratio:


Particulars Shops Restaurant
Cost of goods sold 40% 30%
Supplies 5% 15%
Others 5% 10%

(ii) For lodging house, the variable costs are `25 per day per occupied room for cleaning, laundry etc.
(iii) Annual fixed costs for the entire complex are `19,50,000.

From the above, you are required to prepare:


(a) an income statement for the coming year; and
(b) an analysis to indicate whether the manager’s suggestion of reducing the room rent to `125 per day
to enhance the occupancy ratio to 90% should be accepted.

[Ans.: (a) `1968750, (b) `2190000]


Question 13: A company operates a hotel. It is spread over six floors of a building excluding the ground floor
with a restaurant in the sixth floor. On the ground floor, the hotel operates a sports centre including a
swimming pool and a shopping arcade.

The hotel has a capacity of 100 single rooms and 20 double rooms .The average occupancy of both single
and double rooms is expected to be 80% throughout the year of 365 days. The rent for double room has been
fixed at 125% of the rent of single room. The costs are as under:

Variable costs : Single rooms `220 each per day


Double room `350 each per day
Fixed costs : Single rooms `120 each per day
Double rooms `250 each per day

The income and costs relating to the service centres are as under:
(a) Restaurant:
Estimated average sales per day `25,000.
Contribution 30% of sales.
Fixed costs `8,00,000 per annum.
(b) Sports centers:
Average number of persons expected to use the centre per day is 50.
Average contribution per day per person is `15.
Fixed costs `4,00,000 per annum.
(c) Shopping arcade
Average contribution per month `35,000.
Fixed costs `4,00,000 per annum.

Required:

(i) Calculate the rent chargeable for single and double rooms per day in such a way that the hotel
earns a margin of safety of 20% on hire of rooms.
(ii) Evaluate the profitability of the three service centers and work out the total profit of the hotel per
annum based on the rent recommended by you in (i) above.

CA. Parag Gupta Ph.: +91 11 47665555 Paraggupta_ca@yahoo.co.in Costing & O.R.
World’s largest CA Final student’s consultancy group: http://groups.yahoo.com/group/costingbyparaggupta
Service Sector - 66 -

(iii) The hotel wants to reserve the normal occupancy of ten single rooms for its regular customers by
allowing a discount of 10% on room rent. What increase in occupancy ratio is required in respect of
the remaining rooms to earn the same profit envisaged in (ii) above.
(iv) An associate company wishes to take the entire hotel complex on lease for a total rent of `175 lacs
for five years. The associate company is prepared to pay the entire lease rent in advance. Taking
the Capital recovery factor for 10% return for 5 years at 3.79, advice the management of the hotel
company whether or not the leasing arrangement should be entered into. ICWA-June/91

[Ans.: (i) Single room: `402; Double room: `502.5; (ii) Restaurant: `1937000; Sports centre: `(126250);
Shopping arcade: `20000; Hotel: `3072250 (iii) Revised occupancy level: 81.66%; (iv) Lease is acceptable]
[Hint.: The margin of safety is 20%, therefore break even point is 80%
At 80% that is at BEP the revenue (`84,68,000 × 80%+ `49,64,000) = `1,17,38,400
∴Total desired revenue = 1,17,38,400 ÷ 80% = `1,46,73,000]
[Note.: Suggested answers of ICWAI has forgotten to multiply Total Variable cost by 80% & has wrongly
calculated, revenue at BEP as (`84,68,000 + `49,64,000) = `1,34,32,000]
Question 14: A manufacturing company runs its boiler on furnace oil obtained from X oil company and Y oil
company whose depots are situated at a distance of 24 kms and 16 kms from the factory site.

Transportation of furnace oil is made by company’s own tank lorries (two) of 8 ton capacity each. Onward
trips are made only with full load and the lorries return empty. The filling time takes an average of 40 minutes
for X oil company and 30 minutes for Y oil company. The empty time in the factory is only 40 minutes for
each. The average speed of lorries work out is 24 kms per hour.

The varying operating charges average 80 paise per km covered and fixed charges gives an incidence of
`7.50 per hour of operation.

Calculate the transportation cost per ton-km for each source of furnace oil. (8 Marks) Nov/04

[Ans.: X-`0.33, Y-`0.347]


Question 15: You are required to calculate a suggested fare per passenger/km. from the following
information for a Mini Bus:

(a) Length of route: 30km.


(b) Purchase price `4,00,000.
(c) Part of above cost met by loan, annual interest of which is `10,000 p.a.
(d) Other annual charges: Insurance `15,000, Garage rent `9,000, Road Tax `3,000, Repairs &
maintenance `15,000, Administrative charges `5,000.
(e) Running Expenses: Driver & Conductor `5,000 p.m., Repairs/Replacement of tyre-tube `3,600 p.a,
Diesel and oil cost per km.`5.
(f) Effective life of vehicle is estimated at 5 years at the end of which it will have a scrap value of
`10,000.
(g) Mini Bus has 20 seats and is planned to make six no .two-way trips for 25 days p.m.
(h) Provide profit @ 20% of total revenue. (15 Marks) Nov./97

[Ans.: Rate per passenger km. 0.43 paise]


Question 16: A Cement Company transports its requirement of limestone from a quarry situated at a distance
of 6 km from the factory. Presently the company engages transport contractors for the purpose .The
Company has invited tenders from the local transport contractors and the lowest quotation received is `18 per
tonne of limestone.

The Management is concerned about the increasing cost of transport and has, therefore under its
consideration has proposal for the purchase of a fleet of trucks for being used departmentally for the transport
of limestone. You have been furnished with the following data to examine the feasibility of the proposal.

(i) The company has two options regarding purchases of trucks. They are (a) buy 10 tonne capacity
trucks or (b) buy 8 tonne capacity trucks.
(ii) Operation cost data:
CA. Parag Gupta Ph.: +91 11 47665555 Paraggupta_ca@yahoo.co.in Costing & O.R.
World’s largest CA Final student’s consultancy group: http://groups.yahoo.com/group/costingbyparaggupta
Cost Accounting & Management - 67 -

10 Tonne 8 Tonne
Particulars capacity trucks capacity trucks
Purchase price of each truck ( `) 4,30,000 4,00,000
Life in years 5 5
Scrap value at the end of 5th year of life. (`) 82,000 40,000
Km. per liter of diesel 3 4
Repairs and maintenance
Per annum per truck 47,100 38,400
Road Tax per quarter per truck ( `) 600 600
Miscellaneous fixed expenses per month (`) 3,000 3,000
Oil and sundries per 100 km run ( `) 10 10

(iii) Each truck will make 5 trips (to and fro) on an average for 24 days month.
(iv) Cost of diesel a `10m per liter.
(v) Salary of drivers: `1,600 per month per driver.
Two extra drivers will be employed to work as relievers.
(vi) Other staff required

One Mechanic @ `2,000 per month


One Fitter @ `1,600 per month
One Supervisor @ `3,200 per month

(vii) The capacity of the cement plant is 24,000 tonnes per month of limestone crush.

Required:

(i) Present a comparative cost sheet on the basis of the aforesaid data showing the transport cost
per tonne of operating 10 tonne and 8 tonne capacity trucks at full capacity utilization of the
cement plant for an average month by classifying the expenses into (a) varying with km run (b)
varying with number of trucks and (c) fixed, and suggest the best alternative out of the three
choices available namely (a) selection from two capacity trucks and (b) hiring of transportation.
(ii) Apart from cost analysis as in (i) above, what other factors may be considered by the
management before accepting the proposal for purchase of trucks. ICWA-Dec./91

[Ans.: Cost/Tonne: 10 Tonne trucks-`14.27; 8 Tonne trucks-`15.90; Hire Charges-`18.00]


Question 17: The manager of a hotel providing lodging facilities wants to expand his services to include
manual booking (reservation or cancellation) of railway tickets for his clients. He does not want to have
electronic booking due to operational difficulty. He has the following information:-

(`)/ month
Proportion of rent allocated for office space 4,000
General Telephone expenses allocated to this service 2,400
Proportion of security charges/ maintenance expenses allocated 1,600
Salary to person exclusively doing the booking of tickets 20,000
Mobile phone charges exclusive to person booking ticket 3,000
Share of general miscellaneous fixed expenses allocated 1,000
Conveyance incurred to book tickets (to and fro charges to the 4,000
nearest booking station)[fixed per month]

The manager estimates that there will be 2,500 bookings per month for 3 months of peak season, 1,000
bookings per month for 2 months of moderate business and 700 bookings per month during the remaining
period. He cannot charge more than the prevailing rate of `30 per booking charged by other agents.

Calculate the total cost per booking.

What is the estimated profit the manager hopes to achieve for the full year?

CA. Parag Gupta Ph.: +91 11 47665555 Paraggupta_ca@yahoo.co.in Costing & O.R.
World’s largest CA Final student’s consultancy group: http://groups.yahoo.com/group/costingbyparaggupta
Service Sector - 68 -

What should be the average minimum volume to justify the setting up of the new service?
(Detailed break-up of monthly revenues or costs is not essential.) (7 Marks) Nov.10-O.C.

[Ans.: Total cost per booking: `4,32,000; Estimated Profit: Nil; Average Minimum Volume: 10800 p.a. (i.e.
900 p.m.)]
Question 18: A Company presently brings coal to its factory from a nearby yard and the rate paid for
transportation of coal from the yard located 6km. away to factory is `50 per tonne. The total coal to be
handled in a month is 24,000 tonne.
The Company is considering proposal to buy its own truck and has the option of buying either a 10 tonne
capacity or a 8 tonne capacity truck.
The following information’s are available:

Particulars 10 Tonne Truck 8 Tonne Truck


Purchase price 10,00,000 8,50,000
Life (year) 5 5
Scrap value at the end of the 5th year Nil Nil
Km per liter of diesel 3 4
Repair/ Main. p.a. per truck 60,000 48,000
Other Fixed Expenses p.a 60,000 36,000
Lubricants & sundries per 100 km 20 20

Each truck will daily make 5 trips (to and fro) on an average for 24 days in month. Cost of Diesel `15 per liter.
Salary of Drivers `3, 000 per month- Two Drivers will be required for a Truck.
Other Staff expenses `1,08,000 p.a.

Present a comparative Cost Sheet on the basis of the above data showing transport cost per tonne of
operating 10 tonne and 8 tonne Truck at full capacity utilization. (12 Marks) Nov./98

[Ans.: Truck capacity 10 Tonne 8 Tonne


Cost per tonne (`) 29.87 31.59]
Question 19: Modern Airways owns a single jet aircraft and operates between EXETOWN and WYETOWN.
Flights leave EXETOWN ON Mondays and Thursdays and depart from WYETOWN on Wednesdays and
Saturdays. Modern Airways cannot afford any more flights between EXETOWN and WYETOW. Only tourist
class seats are available on its flights. An analyst has collected the following information:

Seating capacity per plane 360


Average passengers per flight 200
Flights per week 4
Flights per year 208
Average on-way fare `5,000
Variable fuel costs `1,40,000 per flight
Food service to passengers (not charged to `200 per passenger
passengers)
Commission paid to travel agents paid by Modern 8% of fare
Airways on each ticket booked on Modern Airways.
(Assume that all Modern Airways tickets are booked
by travel agents)
Fixed annual lease costs allocated to each flight `5,30,000 per flight
Fixed ground services (maintenance, check-in `70,000 per flight
,baggage handling) costs allocated to each flight
Fixed salaries of flight crew allocated to each flight `40,000 per flight

For the sake of simplicity, assume that fuel costs are unaffected by the actual number of passengers on flight

Required:

a. What is the operating income that Modern Airways makes on each one-way flight between
EXETOWN and WYETOWN?
CA. Parag Gupta Ph.: +91 11 47665555 Paraggupta_ca@yahoo.co.in Costing & O.R.
World’s largest CA Final student’s consultancy group: http://groups.yahoo.com/group/costingbyparaggupta
Cost Accounting & Management - 69 -

b. The market research department of Modern Airways indicates that lowering the average one-way
fare to `4,800 will increase the average number of passengers per flight to 212. Should Modern
Airways lower its fare?
c. Zed Tours and Travels, a tour operator, approaches Modern Airways to charter its jet aircraft
twice each month, first to take Zed’s international tourists from EXETOWN TO WYETOWN and
then bring the tourists back from WYETOWN to EXETOWN. If Modern Airways accepts the offer,
it will be able to offer only 184 (208 minus 24) of its own flights each year. The terms of the
charter are:
i. For each one-way flight Zed will pay Modern `7,50,000 to charter the plane and to use its flight
crew and ground services staff.
ii. Zed will pay for fuel costs.
iii. Zed will pay for all food costs.

On purely financial considerations, should Modern Airways accept the offer from Zed Tours and Travel? What
other considerations should Modern Airways consider in deciding whether or not charter its plane to Zed
Tours and Travels? (19 Marks) May/98

[Ans.: (a) `100000; (b) Excess contribution due to lowering of fare is `13792; (c) If, fares are not lowered
then increase in contribution is `10000, although, if fares are lowered then reduction in contribution is `3792]
[Note: In part (c), suggested answers of ICAI has assumed that revenue from chartering shall be compared
with contribution after lowering of fare. This assumption seems grossly wrong, because each part is to be
considered independently & we shall compare revenue from chartering with contribution before lowering of
fare (as done in next question of ICWA).]
Question 20: In order to develop tourism, Reliable Airline has been given permit to operate three flights to
and fro in a week between Station A and Station B. The airline operates a single aircraft of 160 seats
capacity. The normal occupancy is estimated at 60% throughout the year of 52 weeks. The one way fare is
`7,000.
The costs of operation of flights are:

Fuel cost variable `95,000 per flight


Food served on board on non-chargeable basis `130 per passenger
Fixed costs:
Aircraft lease `3,50,000 per flight
Crew, landing charges etc. `72,000 per flight
Commission 5% of fare applicable for all bookings.

Required:

(i) Calculate the net operating income per flight.


(ii) The airline expects that its occupancy will increase to; 108 passengers per flight if the fare is
reduced to `6,720. Advise whether this proposal should be implemented.
(iii) A travel agency firm proposes to charter the aircraft for one return trip (to and fro) in each month
on payment of a fixed charge of `5 lakhs per flight.
The travel agency firm will meet the fuel and food costs. Should the airline accept this proposal?

[Ans.: (i) `108920; (ii) `580432; (iii) `30920] [ICWA-June/2000]


Question 21: Aero Ltd. has hired an aircraft to specially operate between cities A and B. All the seats of the
aircraft are economy class.
The following information is available :

Seating capacity of the aircraft =320 passengers


Average number of passengers per flight =240 passengers
Average one way fare from A to B =`5,000 per passenger
Variable fuel costs per flight from A to B `90,000
Food Cost `300 per passenger
(no charge to passenger)
Commission to travel agents 10% of the fare
(All tickets are through agents)
CA. Parag Gupta Ph.: +91 11 47665555 Paraggupta_ca@yahoo.co.in Costing & O.R.
World’s largest CA Final student’s consultancy group: http://groups.yahoo.com/group/costingbyparaggupta
Service Sector - 70 -

Annual lease costs allocated to each flight `2,00,000


Ground services, baggage handling / check- in `40,000
services costs per flight A to B
Flight crew salaries per flight A to B `48,000

There is an offer from another airlines operator, Mid Air Ltd. for a stop-over at destination D, which is on the
way from A to B. Due to this, the flight will operate from A to D, then D to B.
The following terms are to be considered for the stop-over:

50 seats will be booked by Mid Air at `2,500 per ticket, whether or not Mid Air is able to sell them to its
customers. No agent’s commission is payable on these tickets.

60 new passengers will be booked by Aero's travel agents for travel from A to D at a fare of `2,000 per
passenger.

Since the stop-over wastes more time, 25 of Aero’s original passengers from A to B will drop out and seek
other airlines which fly directly from A to B.

Due to the stop-over, fuel costs will increase from `90,000 to `1,35,000, Additional airport landing/ baggage
handling charges of `19,000 per stop-over will have to be incurred by Aero Ltd.

Aero Ltd. will have to serve snacks to all passengers in the D to B sector at no charge to passenger. Each
snack will cost Aero Ltd. `200. This will be in addition to the original food at `300 served in the A to D sector.

You may assume that fuel costs are not affected by the actual number of passengers in a flight. You may
ignore non-financial considerations, additional wear and tear to aircraft due to extra landing / take-off.

Without considering Mid Air’s offer,


(i) What is the profit earned by Aero Ltd. per flight from A to B?
(ii) What is the break-even number of passengers for each flight from A to B?

Considering the effects of Mid Air’s offer,


(iii) Evaluate whether Aero should accept the offer.
(A detailed profitability statement is not essential, a relevant cost-revenue analysis would suffice)

[Ans.: (i) `6,30,000; (ii) 90 passengers; (iii) Net benefit `146000] (12 Marks) Nov./10-O.C.
[Assumption: In part (iii) 50 seats blocked by Mid Air are both for A to D & D to B. Further it is assumed that
these seats will remain fully occupied, hence food cost & snacks costs also incur on such seats]
[Note: There seems some drafting error in part (iii) because per it tickets sold are 325 ({240-15} + 50 + 60)
although seating capacity is of 320 passengers only.]

Question 22: A private university with a current enrolment of 12,000 students is reviewing cost and revenue
data for the past academic year. Student tuition is `3,600 a year. Tuition normally covers 75 per cent of
university expenditures. The remaining 25 per cent comes from endowments and contributions. During the
last academic year fixed costs amounted to `300 lakhs. The rest of the costs varied with student enrolment.
Costs have been rising more rapidly than tuition or contributions, and the university just broke even last year.
A tuition increase is being contemplated. The budget committee thinks endowment revenues and
contributions will remain constant at last year’s level for the next several years.

The fixed costs are expected to increase by `30 lakhs and the variable costs are expected to increase by 10
per cent. The president of the university tells the budget committee that the expects a new grant of `50 lakhs
in addition to the normal contributions for each of the next 5 years from a large corporation owned by an
alumnus of the university. The university has been postponing a number of major capital improvements and
building projects.

Required:

CA. Parag Gupta Ph.: +91 11 47665555 Paraggupta_ca@yahoo.co.in Costing & O.R.
World’s largest CA Final student’s consultancy group: http://groups.yahoo.com/group/costingbyparaggupta
Cost Accounting & Management - 71 -

(i) If the grant is received and tuition is raised to `4,200, how much money would the university have
available in the first year for capital improvements and building with student enrolment of 11,200
and the expected cost increases?.
(ii) If the grant is received and costs increases as predicted for the coming year, what tuition should
the university charge to break-even with its current enrolment of 12,000 students after providing
`40.40 lakhs for capital improvements?

[Ans.: `5104000; `4000 per student]


Question 23: DD amusement Park charges `4 each for all rides in the park. Variable costs amount to `0.80
per ride and fixed costs are `32, 00,000.last year’s net income was `6, 40,000 on sales of `48, 00,000. Rising
costs have cut sharply into net income for DD for the last 2 years. This year management again expects a
cost increase of 25 per cent in variable costs and 10 per cent in fixed costs. To help offset these increases,
the management is considering raising the price of a ride to `5.

Required:

(i) How many rides did DD sell last year?


(ii) If the price increase is not implemented, what is the expected net income from this year assuming
the same volume of activity?
(iii) Compute the price in difference point for the new ride price.
(iv) Compute the Break-even point for this year using the old price and the new price.
(v) Should management raise the price of a ride, if the price increase will reduce ride volume 10 per
cent from the last years’ level? In that situation, what will be the expected net income?

[Ans.: (i) 1200000; (ii) 80000 (iii) 1040000 (iv) At old price: 1173334; At new price: 880000 (v) 800000]
Question 24: A city health centre provides health and other related services to the citizens who are covered
under insurance plan. The health centre receives a payment from the insurance company each time any
patient attends the centre for consultation as under:

Consultations involved Payment from Insurance company


`
No treatment 60
Minor treatment 250
Major treatment 500

In addition, the adult patients will have to make a co-payment which is equivalent to the amount of payment
for the respective category of treatment made by the insurance company. However, children and senior
citizens are not required to make any such co-payment.

The health centre will remain open for 6 days in a week for 52 weeks in a year. Each physician treated 20
patients per day although the maximum number of patients that could have been treated by a physician on
any working day is 24 patients.

The health centre received a fixed income of `225280 per annum for promotion of health products from the
manufacturers.

The annual expenditure of the health centre is estimated as under:


Materials and consumables (100% variable) `2232000
Staff salaries per annum per employee (fixed):
Physician `450000
Assistants `150000
Administrative staff `90000
Establishment and other operating costs (fixed) `1600000
The non-financial information is as under
(i) Staff:
No. of physicians employed 6
Assistants 7
Administrative staff 2
CA. Parag Gupta Ph.: +91 11 47665555 Paraggupta_ca@yahoo.co.in Costing & O.R.
World’s largest CA Final student’s consultancy group: http://groups.yahoo.com/group/costingbyparaggupta
Service Sector - 72 -

(ii) Patient Mix:


Adults 50%
Children 40%
Senior Citizens 10%
(iii) Mix of patients appointments (%)
Consultation requiring no treatment 70%
Minor treatment 20%
Major treatment 10%

Required:

(a) Calculate the Net Income of the city health centre for the next year.
(b) Determine the percentage of maximum capacity required to be utilized next year in order to break
even. (8 Marks) Nov./08-O.C.
[Ans.:(i) `438000 (ii) 77%]
[Assumption: Patient mix and mix of patient appointments will be same in the next year.]
Question 25: A Multinational company runs a Public Medical Health Centre. For this purpose, it has hired a
building at a rent of `10,000 per month with 5% of total taking*. Health centre has three types of wards for its
patients namely. General ward, Cottage ward and Deluxe ward. State the rent to be charged to each bed-day
for different type of ward on the basis of the following information:

(i) The number of beds of each type are General ward 100, Cottage ward 50, Deluxe ward 30.
(ii) The rent of Cottage ward bed is to be fixed at 2.5 times of the General ward bed and that of Deluxe ward
bed as twice of the Cottage ward bed.
(iii) The occupancy of each type of ward is as follows:
General ward 100%, Cottage ward 80% and Deluxe ward 60%. But, in general ward there were occasions
when beds are full, extra beds were hired at a charges of `20 per bed. The total hire charges for the extra
beds incurred for the whole year amount to `12,000.
(iv) The Health Centre engaged a heart specialist from outside and on an average fees paid to him was
`15,000 per trip. He makes three trips in the whole year.
(v) The other expenses for the year were as under: `
Salary of Supervisors, Nurses, Ward boys 4,25,000
Repairs and maintenance 90,000
Salary of doctors 13,50,000
Food supplied to patients 40,000
Laundry charges for their bed linens 80,500
Medicines supplied 74,000
Cost of oxygen, X-ray etc. other than directly borne for treatment of patients 49,500
General administration charges 63,000
(vi) Provide profit @ 20% on total taking*.
(vii) The Health Centre imposes 8% service tax on rent received.
(viii) 360 days may be taken in a year. (12 Marks) Nov./06

[Ans.: General ward `32.22 `33.10*


Cottage ward `80.55 `82.76*
Deluxe ward `161.08 `165.51*
*If total taking includes Service Tax]

CA. Parag Gupta Ph.: +91 11 47665555 Paraggupta_ca@yahoo.co.in Costing & O.R.
World’s largest CA Final student’s consultancy group: http://groups.yahoo.com/group/costingbyparaggupta
Standard
Costing &
Variance
Analysis

CIMA defines standard cost as "a standard expressed in money. It is built up from an assessment of the
value of cost elements. Its main uses are providing bases for performance measurement, control by exception
reporting, valuing stock & establishing selling price."

The term static budget refers to the budget that is prepared at the beginning of the budgeting period and is
valid for only the planned level of activity.
A flexible budget provides estimates of what costs should be for any level of activity within a specified
range.
A static budget is suitable for planning, but it is inadequate for evaluating how well costs are controlled
because the actual level of activity is unlikely to equal the planned level of activity, thus resulting in “apples-
to-oranges” cost comparisons. Flexible budget, when used for performance evaluation purposes, actual
costs are compared to what the costs should have been for the actual level of activity during the period.
This enables “apples-to-apples” cost comparisons.

Direct Material Cost Variance

Material Usage Variance/ + Material Price Variance/


Material Consumption Variance/ Material Rate Variance/
Material Quantity Variance Material Spending Variance

Material Yield Variance + Material Mix Variance


Or
Material Sub-Usage Variance

CA. Parag Gupta Ph.: +91 11 47665555 Paraggupta_ca@yahoo.co.in Costing & O.R.
World’s largest CA Final student’s consultancy group: http://groups.yahoo.com/group/costingbyparaggupta
Standard Costing & Variance Analysis - 74 -

1. Total Material Cost Variance (TMCV)


TMCV = (Standard Qty × Standard Rate) – (Actual Qty Purchaseda × Actual Rate)
OR
= Standard Cost for Actual Output – Actual Cost for Actual Output
2. Material Usage Variance (MUV)
MUV = (Standard Qty for Actual Output – Actual Qty Used) × Standard Rate
3. Material Price Variance (MPV)
MPV = [Standard Price (Rate) – Actual Price (Rate)] × Actual Quantity Purchaseda
a
Note: [There are two approaches of taking Actual Quantity while calculating TMCV & MPV. (i) Quantity is
taken as quantity purchased (ii) Quantity is taken as quantity used.

Remember, since price variance is reported in the period in which it is incurred it is recommended to use
quantity purchased approach.

If quantity purchase is not equal to quantity used then, TMCV ≠ MUV + MPV
If quantity purchase is not equal to quantity used & question is silent, you are supposed to write
assumption about the approach used for taking actual quantity.]
4. Material Yield Variance (MYV)
MYV = (Actual Output/Yield – Std. Output for Actual Mix) × Std. price per unit of output
= (Actual Output – [c]) × [b]
Where, Std. price per unit of Output = Total Standard Cost of Input
Total Standard Output
OR
Material Sub-Usage Variance (MSUV)
MSUV = (Std. Quantity for actual output- Revised Std. Quantity) × Standard Price
Revised Standard Qty = Total Qty of Actual Mix × Std.
Qty
Total Qty in Std. Mix
5. Material Mix Variance (MMV)
Main Formula:
MMV = (Revised Standard Quantity – Actual Quantity) × Standard Price

Rarely used Alternate Formula:


MMV = (Std. Rate of Std. Mix – Std. Rate of Actual Mix) × Total Actual Mix
Where, Std. Rate of Std. Mix = Total Std. Cost for Actual Output
Total Std. Quantity
&, Std. Rate of Actual Mix = Total Actual Qty at Std. Rate
Total Actual Qty
 All Material Variances save Yield Variance are calculated as Std. – Actual. Yield Variance is
calculated as Actual – Std.
 All Material Variances save Yield Variance are calculated on Input quantity. Yield Variance is
calculated for Production.

Question 1: Actual Output is 90 kgs., Standard mix for material A, B, & C is 50%, 30%, & 20% @ `2 per kg,
`3 per kg and `4 per kg respectively. Standard Normal Loss is 10%. Actual quantity used is 60 kgs @ `4 per
kg of A, 50 kgs @ `2 per kg of B, & 10 kgs @ `3 per kg of C. Calculate all material variances.

[Ans.: MCV: 100(A); MPV: 60(A); MUV: 40(A); MMV: 14(F); MYV: 54(A)]
Question 2: A Ltd. produces an article by blending 2 raw materials. The following standards have been set
up for raw materials:

CA. Parag Gupta Ph.: +91 11 47665555 Paraggupta_ca@yahoo.co.in Costing & O.R.
World’s largest CA Final student’s consultancy group: http://groups.yahoo.com/group/costingbyparaggupta
Cost Accounting & Management - 75 -

Material Standard Mix Standard Price (`per kg.)


A 40% 4
B 60% 3

The standard loss in processing is 15%. During Sept’08, the company produces 1700 units of finished goods.
The position of stock and purchases for the month of Sept’08 is as under:

Material Stock on 1-9-08 Stock on 30-9-08 Purchases during Sept’08


Kg. Cost `
A 35 5 800 3400
B 40 50 1200 3000

Calculate all material variances. Assume FIFO for the issue of material. The opening stock is to be valued at
standard price.

[Ans.: MUV: 90(A); MMV: 22(A); MYV: 68(A); On the basis of purchase - MCV: 400(F); MPV: 400(F); On
the basis of usage - MCV: 286(F); MPV: 376(F)]
Question 3: The standard cost for producing 180 kgs of a product whose raw material inputs are A and B is
given below:
Standard Cost (`)
Material A 60 kgs @ `10 per kg 600
Material B 140 kgs @ `2 per kg 280
880

The actual prices of A and B were `12 and `8 per kg respectively. Consumption of B was 108 kg. The actual
output at 80% yield was 144 kg.

Calculate the following direct material variances:


(i) Mix variance
(ii) Yield variance
(iii) Price variance
(iv) Usage variance

[Ans.: (i) 144(A); (ii) 88(A); (iii) 792(A); (iv) 232(A)] (5 Marks) Nov./10-O.C.
Question 4: The standard set for a chemical mixture of a firm is as under:
Material Standard mix % Standard price per kg (`)
A 40 60
B 60 30

The standard loss in production is 10 %. During a period, the actual consumption and price paid for a
good output of 182 kg. are as under:

Material Quantity Actual price


in kg. per kg (`)
A 90 18
B 110 34

Calculate the variances.

[Ans.: MCV: 3100.30(F); MPV: 3340(F); MUV: 206.70(A); MMV: 300(A); MYV: 93.30(F)]
Question 5: The standard quantity of material required is 4 kgs. per unit of actual output. The relevant
figures are as under:

CA. Parag Gupta Ph.: +91 11 47665555 Paraggupta_ca@yahoo.co.in Costing & O.R.
World’s largest CA Final student’s consultancy group: http://groups.yahoo.com/group/costingbyparaggupta
Standard Costing & Variance Analysis - 76 -

Material A B C D
Standard mix % 30% 40% 20% 10%
Price per kg. (`) 1.25 1.50 3.50 3.00
Actual qty. used (Kg.) 1,180 1,580 830 440
Actual price per kg. (`) 1.30 1.80 3.40 3.00
Actual output: 1,000 units

Calculate price variance, mix variance, sub-usage variance and total material cost variance.

[Ans.: MCV: 620(A); MPV: 450(A); MUV: 170(A); MMV: 110.75(A); MSUV: 59.25(A)]

1. Labour Cost Variance (LCV):


LCV = Standard Labour Cost – Actual Labour Cost
OR
= (Standard Hours × Standard Rate) – (Actual Hoursb × Actual Rate)
2. Direct Labour Efficiency Variance (LEV):
LEV = (Standard Hours for actual output – Actual Hoursb) × Standard Rate
3. Labour Rate Variance (LRV):
LRV = (Standard Rate – Actual Rate) × Actual Hoursb
4. Idle Time Variance (ITV):
ITV (Always Adverse) = Idle Hours × Standard Rate
b
Note: In case we are not supposed to calculate idle time variance then we will calculate 3 labour
variances (i.e. Labour Cost, Rate & Efficiency variance) at actual hours worked otherwise, Labour Cost &
Rate variance will be calculated on actual hours paid basis & efficiency variance is calculated on actual
hours worked basis.

5. Labour Yield Variance (LYV)


LYV = (Actual Output/Yield – Std. Output for Actual Mix of Labour Worked) × Std. price per unit of
output
Where,
Std. price per unit of Output = Total Standard Cost of Input Labour Hours
Total Standard Output
OR
Labour Revised Efficiency Variance (LREV)
LREV = (Std. Hours for actual output- Revised Std. Hours Worked) × Standard Price
Revised Standard Hrs. = Total Hrs Worked of Actual Mix × Std. Hrs.
Total Hrs. in Std. Mix

6. Labour Mix Variance (Gang Variance) or (Composite Variance):


LMV = (Revised Std. Hrs. Worked – Actual Hours Worked) × Standard Price

CA. Parag Gupta Ph.: +91 11 47665555 Paraggupta_ca@yahoo.co.in Costing & O.R.
World’s largest CA Final student’s consultancy group: http://groups.yahoo.com/group/costingbyparaggupta
Cost Accounting & Management - 77 -

 Actual Hours Worked = Actual Hours Paid – Idle Time


 All Labour Variances save Yield Variance are calculated as Std. – Actual. Yield Variance is
calculated as Actual – Std.
 All Labour Variances save Yield Variance are calculated on Input hours. Yield Variance is
calculated for Production.
 Since CIMA terminology doesn’t define Labour Yield Variance, so until question specifically
demands, it is recommended to find Labour Revised Efficiency variance instead of Labour
Yield Variance.

Question 6: 100 skilled workmen, 40 semi-skilled workmen and 60 unskilled workmen were to work for 30
weeks to get a contract job completed. The standard weekly wages were `60, `36 and `24 respectively. The
job was actually completed in 32 weeks by 80 skilled, 50 semi-skilled and 70 unskilled workmen who were
paid `65, `40 and `20 respectively as weekly wages.

Find out the labour cost variance, labour rate variance, labour mix variance and labour efficiency variance.

[Ans.: LCV: 8800(A); LRV: 10240(A); LEV: 1440(F); LMV:19200(F)]


Question 7: Given the following data, compute the variances.

Skilled Semi-Skilled Unskilled


Number of workers in standard gang 16 6 3
Standard rate per hour 3 2 1
Actual number of workers in the gang 14 9 2
Actual rate of pay per hour (`) 4 3 2

In a 40- hour week, the gang as a whole produced 900 standard hours. (10 Marks)Nov/09-NC-Adapted

[Ans.: LCV: 1212(A); LEV: 212(A); LRV: 1000(A); LREV: 252(A); LMV:40(F)]
Question 8: A firm gives you the following data:
Standard time per unit 2.5 hours
Actual hours paid 2,000 hours
Standard rate of pay `2 per hour
25 % of the actual hours has been lost as idle time.
Actual output 1,000 units
Actual wages `4,500

Calculate the idle time variance.

[Ans.: LCV: 500(F); LRV: 500(A); LEV: 2000(F); ITV: 1000(A)]


[Note: All variances other than idle time variance are calculated just for practice]
Question 9: The following details are available from the records of A Ltd. engaged in manufacturing article A
for the week ended 28th Feb.

Particulars Hours Rate per hour (`) Total


Skilled labour 10 3 30
Semi-skilled labour 8 1.50 12
Unskilled labour 16 1 16
58

The actual production was 1000 articles A for which the actual hours worked and rates are given below:
Particulars Hours Rate per hour (`) Total
Skilled labour 9000 4 36000
Semi-skilled labour 8400 1.50 12600
Unskilled labour 20000 0.90 18000
66600
CA. Parag Gupta Ph.: +91 11 47665555 Paraggupta_ca@yahoo.co.in Costing & O.R.
World’s largest CA Final student’s consultancy group: http://groups.yahoo.com/group/costingbyparaggupta
Standard Costing & Variance Analysis - 78 -

From the above set of data, you are asked to calculate: (i) Labour cost variance (ii) Labour rate variance (iii)
Labour efficiency variance (iv) Labour yield variance (v) Labour mix variance.

[Ans.: 8600 (A), 7000 (A), 1600(A), 5800 (A), 4200 (F)]

FACTORY OVERHEAD VARIANCE

Variable Factory Overhead Variance Fixed Factory Overhead Variance

1. Overheads are usually measured in relation to output if details of input quantities on which these variable
overheads have been incurred are not readily available.
2. Overheads are calculated on Actual Hours/Days Worked.
3. Under-absorption = Adverse Variance & Over-absorption = Favourable Variance
4. Remember if Cost-allocation base is specified, we will only take the specified cost allocation for absorption
purpose otherwise, if question is silent, Variable overheads are absorbed on actual hours.

Budgeted Overheads
1) Std. Overhead Rate per unit =
Budget Output
Budgeted Overheads
2) Std. Overhead Rate per hour =
Budget Hours
Budgeted Hours×Actual Output
3) Std. Hours for actual output =
Budgeted Output
Budgeted Output×Actual Hours
4) Std. Output for actual time =
Budgeted Hours
5) Recovered/Absorbed Overheads = Std. Rate per unit×Actual Output
= Std. Rate per hour×Std. Hours for actual output
6) Budgeted Overheads = Std. Rate per unit×Budgeted Output
= Std. Rate per hour×Budgeted hours
7) Standard Overheads = Std. Rate per hour×Actual Hours
= Std. Rate per unit×Std. Output for actual time
8) Possible Overheads = Std. Rate per day×Actual Days
= Std. Rate per hour×Std. hrs. per day×Actual Days
9) Actual Overheads = Actual Rate per unit×Actual Output
= Actual Rate per hour×Actual Hours

o Variable Factory Overhead

1. Variable Overhead Cost Variance (VOCV):


VOCV = Standard Cost – Actual Cost
2. Variable Overhead Efficiency Variance (VOHV):
VOHV = (Budgeted units of variable overhead cost allocation base allowed for actual output - Actual
units of variable overhead cost allocation base for actual output) × Budgeted Variable
Overhead Rate
= (Standard Hours for actual output – Actual Hours) × Standard Rate

CA. Parag Gupta Ph.: +91 11 47665555 Paraggupta_ca@yahoo.co.in Costing & O.R.
World’s largest CA Final student’s consultancy group: http://groups.yahoo.com/group/costingbyparaggupta
Cost Accounting & Management - 79 -

3. Variable Overhead Expense Variance (VOEV):


VOEV = (Budgeted Variable Overhead Rate - Actual Variable Overhead Rate) × Actual units of
variable overhead cost allocation base for actual output
= (Standard Rate – Actual Rate) × Actual Hours

Question 10: XYZ Company has established the following standards for variable factory overhead.

Standard hours per unit : 6


Variable overhead per hour : `2/-
The actual data for the month are as follows:
Actual variable overheads incurred `2,00,000
Actual output (units) 20,000
Actual hours worked 1,12,000

Required:

Calculate variable overhead variances viz :


(i) Variable overhead variance.
(ii) Variable overhead budget variance.
(iii) Variable overhead efficiency variance.

[Ans.: (i) 40000(F); (ii) 24000(F); (iii) 16000(F)]


Question 11: The following information has been extracted from the books of Goru Enterprises which is using
standard costing system:

Actual output = 9,000 units


Direct wages paid = 1,10,000 hours at `22 per hour, of which 5,000
hours, being idle time, were not recorded in
production
Standard hours = 10 hours per unit
Labour efficiency variance = `3,75,000 (A)
Standard variable Overhead = `150 per unit
Actual variable Overhead = `16,00,000

You are required to calculate:

(i) Idle time variance


(ii) Total variable overhead variance
(iii) Variable overhead expenditure variance
(iv) Variable overhead efficiency variance (6 Marks) May/08

[Ans.: (i) 125000(A), (ii) 250000(A) (iii) 25000(A) (iv) 225000(A)]


Question 12: Z Ltd. uses standard costing system in manufacturing of its single product ‘M’. The standard
cost per unit of M is as follow:

Direct material - 2 meters @ `6 per meters 12.00


Direct labour - 1 hour @ `4.40 per hours 4.40
Variable overhead - 1 hour @ `3 per hour 3.00
19.40
During July 2004, 6000 units of M were produced and the related data are as under:
Direct material acquired - 19000 meters @ 5.70 per meters.
Material consumed - 12670 meters.
Direct labour - ? hour @ `? Per hour `27,950
Variable overhead incurred `20,475

The variable overhead efficiency variable is `1,500 adverse. Variable overhead are based on direct labour
hour. There are no stock of raw material in the beginning.

CA. Parag Gupta Ph.: +91 11 47665555 Paraggupta_ca@yahoo.co.in Costing & O.R.
World’s largest CA Final student’s consultancy group: http://groups.yahoo.com/group/costingbyparaggupta
Standard Costing & Variance Analysis - 80 -

You are required to compute the missing figures and work out all the relevant variances. (Nov./93)
Compute of standard cost and actual costs

[Ans.:
Material cost variance `36,300(A) Material price variance `5,700(F)
Material usage variance `4,020(A) Labour cost variance `1,550(A)
Labour rate variance `650 (F) Labour efficiency variance `2,200(A)
Variable overhead variance `2,475(A) Variable overhead efficiency variance `1,500(A)
Variable overhead budget variance `975(A)]

Question 13: Mr. M provides the following information relating to 1,000 units of product ‘ZED’ during the
month of April, 1998

Standard price per kg. of raw material – `3


Actual total direct material cost – `10,000
Standard direct labour hours – 1,600
Actual direct labour hours – 1,800
Total standard direct labour cost – `8,000
Standard variable overhead per direct labour hour – Re.1
Standard variable overhead cost per unit of ZED – `1.60
Total standard variable overheads – `1,600
Actual total variable overheads – `1,620

The material usage variance is `600 (adverse) and the overall cost variance per unit of ZED is Re.0.07
(adverse) as compared to the total standard cost per unit of ZED of `21.

You are required to compute the following:


(a) Standard quantity of raw-material per unit of ZED.
(b) Standard direct labour rate per hour.
(c) Standard direct material cost per unit of ZED.
(d) Standard direct labour cost per unit of ZED.
(e) Standard total material cost for the output.
(f) Actual total direct labour cost for the output
(g) Material price variance.
(h) Labour rate variance.
(i) Labour efficiency variance.
(j) Variable overhead expenditure variance.
(k) Variable overheads efficiency variance. (May/93)

[Ans.:
Standard quantity of raw material per unit of 3.8 Kgs. Standard direct labour rate per hour `5
ZED
Standard direct material cost per unit of ZED `11.40 Standard direct labour cost per units of `8
ZED
Standard total material cost for the output 11,400 Actual total direct labour cost for the 9,450
output
Material price variance `2,000(F) Labour rate variance `450 (A)
Labour efficiency variance `1,000(A) Variable overhead expenditure variance `180(F)
variable overhead efficiency variance `200(A)

o Fixed Overhead Variance

CA. Parag Gupta Ph.: +91 11 47665555 Paraggupta_ca@yahoo.co.in Costing & O.R.
World’s largest CA Final student’s consultancy group: http://groups.yahoo.com/group/costingbyparaggupta
Cost Accounting & Management - 81 -

 In case we are supposed to calculate Calendar Variance then we will calculate Revised Capacity
Variance along with Calendar Variance but will name it as Capacity Variance rather than Revised
Capacity Variance. We will not calculate capacity variance in such case.

1. Fixed Overhead Cost Variance (FOCV):


FOCV = Absorbed Overhead – Actual Overhead

2. Fixed Overhead Volume Variance (FOVV):


FOVV = Absorbed Overhead – Budgeted Overhead

3. Fixed Overhead Budget Variance (FOBV):


FOBV = Budgeted Overhead – Actual Overhead

4. Fixed Overhead Efficiency Variance (FOEV):


FOEV = Absorbed Overhead – Standard Overhead
= (Std. Hours for Actual Output – Actual Hrs Worked) × Std. Overhead per Hour

5. Fixed Overhead Capacity Variance (FO Cap V):


FO Cap V = Standard Overhead – Budgeted Overhead
= (Actual Hrs – Budgeted Hours) × Std. Overhead per hour

6. Fixed Overhead Revised Capacity Variance (FORCV):


FORCV = Standard Overhead – Possible Overhead
= (Actual Hrs per Day – Std. Hrs. per Day) × Actual Days Worked × Std. Overhead per
hour

7. Fixed Overhead Day/Calendar Variance (FODV):


FODV = Possible Overhead – Budgeted Overhead

= (Actual Days Worked– Std. Days) × Std. Hours per Day × Std. Overhead per hour

= (Actual Days Worked - Standard Days) × Standard Overhead per Day

Question 14: Budgeted No. of working days 24


Budgeted No. of hours per month 12,000
Fixed overhead rate Re.0.50 per hour
Actual No. of working days in June 25
Compute the calendar variance

[Ans.: FODV: 250(F)]


Question 15: Assuming the expenses to be fixed, calculate from the following data:

(a) Efficiency variance, (b) Volume variance, (c) Calendar variance and (d) Expense variance

CA. Parag Gupta Ph.: +91 11 47665555 Paraggupta_ca@yahoo.co.in Costing & O.R.
World’s largest CA Final student’s consultancy group: http://groups.yahoo.com/group/costingbyparaggupta
Standard Costing & Variance Analysis - 82 -

Budget Actual
No. of working days per month 20 22
Man hours per day 8,000 8,400
Output per man hour in units 1.0 1.2
Standard overhead rate per man hour `2
Actual fixed expenses per month `3,25,000

[Ans.: (a) 73920(F); (b) 123520(F); (c) 32000(F); (d) 5000(A)]


Question 16: You are given the following data:

Budgeted Actual
Fixed overhead for July `10,000 `10,200
Units of production in July 5,000 5,200

Standard time for one unit 4 hours


Actual hours worked 20,100 hours
Calculate all variances relating to fixed overheads
[Ans.: FOCV: 200(F); FOBV: 200(A); FOVV: 400(F); FO Cap V:50(F); FOEV: 350(F)]
Question 17: ABC limited provides the following information for April, 2002:
Budget Actual
Number of working day 20 21
Man hours 40,000 43,000
Output per manhour (unit) 3.2 3.0
Overhead – Fixed (`) 32,000 31,500
Variable (`) 1,02,400 1,14,400
Required
Compute variable overhead, fixed overhead variance and total overhead variance (12 Marks) May/02
[Ans.: Total variable overhead variable `11,200(A); Variable overhead expenditure variance `4320(A);
Variable overhead efficiency variance `6,880(A); Fixed overhead expenditure variance `500(F); Fixed
overhead volume variance `250(F) ; Fixed overhead efficiency variance `2150(A); Fixed overhead capacity
variance `800(F) ; Calendar variance `1600(F); Total overhead variance `10450(A)]

Sales Variance:

Based on Turnover (Value Method) Based on Profit (Sales Margin


Method)

Value Variance Value Variance

Volume Variance Price Variance Volume Variance Price Variance

Quantity Variance/ Mix Variance Quantity Variance/ Mix Variance


Sub-Volume Variance Sub-Volume Variance

Market Size Variance Market Share Variance

CA. Parag Gupta Ph.: +91 11 47665555 Paraggupta_ca@yahoo.co.in Costing & O.R.
World’s largest CA Final student’s consultancy group: http://groups.yahoo.com/group/costingbyparaggupta
Cost Accounting & Management - 83 -

Sales Variances are calculated in same way as that of Material Variances save Material
Variances are calculated as Std. – Actual & Sales variances are calculated as Actual – Std.

Based on Turnover:

1. Sales Value Variance (SVV)

SVV = Actual Sales - Budgeted Sales

2. Sales Volume Variance (S Vol V)

SVV = Standard Sales - Budgeted Sales


= (Actual Sales Qty - Budgeted Sales Qty) × Standard Sale Price

3. Sales Price Variance (SPV)

SPV = Actual Sales - Standard Sales


= [Actual Price (Rate) - Standard Price (Rate)] × Actual Quantity Sold

4. Sales Mix Variance (SMV)

SMV = Std. Sales - Revised Std. Sales

Based on Quantity : Products are Homogenous (Better Approach)


SMV = (Actual Qty sold - Revised Std. Quantity) × Standard Selling price
Where, Revised Std. Qty. = Total Qty of Actual Mix × Std. Quantity
Total Quantity of Std. Mix

Based on Value : Products are not Homogenous


SMV = Std. Sales - Revised Std. Sales

Where, Revised Std. Sales = Budgeted Ratio of Sales × Std. Sales


& Budgeted Ratio of Sales = Budgeted Sales of Product
Total Budgeted Sales

5. Sales Quantity Variance (SQV)

SQV = Revised Std. Sales - Budgeted Sales

Based on Profit:

These are exactly same as sales variance (based on turnover). Std. Selling Price is replaced
with Std. Margin & Actual Selling Price by Actual Margin, but Std. Margin = Std. Selling Price –
Std. Cost & Actual Margin = Actual Selling Price – Standard Cost

Question 18: Compute the sales turnover variances from the following figures: -
Product Budget Actual
Quantity Price Quantity Price
` `

A 2,000 2.50 2,400 3.00


B 1,500 5.00 1,400 4.50
C 1,000 7.50 1,200 7.00
D 500 10.00 400 10.50

CA. Parag Gupta Ph.: +91 11 47665555 Paraggupta_ca@yahoo.co.in Costing & O.R.
World’s largest CA Final student’s consultancy group: http://groups.yahoo.com/group/costingbyparaggupta
Standard Costing & Variance Analysis - 84 -

[Ans.: SVV: 1100(F); SPV: 100(F); S Vol V: 1000(F); SMV: 1000(A); SQV: 2000(F)]
Question 19: A Trident Toys Ltd. had drawn up the following sales budget for August, 2004 :-

‘Bravo’ Toys 5,000 units at `100 each


‘Champion’ Toys 4,000 units at `200 each
‘Super ‘ Toys 6,000 units at ` 180 each
The actual sales for August, 2004 were:
‘Bravo’ Toys 5,750 units at `120 each
‘Champion ‘ Toys 4,850 units at `180 each
‘Super ‘ Toys 5,000 units at `165 each

The cost per unit of Bravo , Champion and super Toys were `90, `170, `130 respectively .
Analyse the variance to show:
(a) the effect on turnover
(1) Sales price variance
(2) Sales mix variance
(3) Sales quantity variance
(4) Total sales value variance
(b) the effect on profit :
(1) Sales margin : price variance
(2) Sales margin : mix variance
(3) Sales margin : quantity variance
(4) Total sales margin variance (Nov/91)

[Ans.:
Sales price variance `57,000(A) Sales mix variance `30,200(A)
Sales Quantity Variance `95,200(F) Total sales value variance `8,000(F)
Sales margin price variance `57,000(A) Sales margin mix variance `35,800(A)
Sales margin quantity variance `18,800(F) Total sales margin variance `74,000 (A)]

Market Size Variance = (Budgeted mkt. share %) × (Actual Industry sales volume in units –
Budgeted industry sales volume in units) × (Budgeted Avg. Contribution margin per unit)

Market Share Variance = (Actual Mkt. Share % - Budgeted Mkt. Share %) × (Actual industry
sales volume) × (Budgeted Average Contribution Margin per unit)

Question 20: Super computers manufactures and sells three related PC models:

1. PC — Sold mostly to college students


2. Portable PC— Smaller version of PC positioned as home computer
3. Super PC — Sold mostly to business executives

Budgeted and actual data for 1995 is as follows:

Budget for 1995


Selling Price Variable Cost Contribution Sales Volume
per Unit per Unit margin per in Units
Unit
` ` `
PC 24,000 14,000 10,000 7,000
Portable PC 16,000 10,000 6,000 1,000
Super PC 1,00,000 60,000 40,000 2,000
10,000

CA. Parag Gupta Ph.: +91 11 47665555 Paraggupta_ca@yahoo.co.in Costing & O.R.
World’s largest CA Final student’s consultancy group: http://groups.yahoo.com/group/costingbyparaggupta
Cost Accounting & Management - 85 -

Actual for 1995


Selling Price Variable Cost Contribution Sales Volume
per Unit per Unit margin per in Units
Unit
` ` `
PC 22,000 10,000 12,000 8,250
Portable PC 13,000 8,000 5,000 1,650
Super PC 70,000 50,000 20,000 1,100
11,000

Super computers derived its total unit sales budget for 1995 from the internal management estimate of a
20% market share and an industry sales forecast by computer manufacturers association of 50,000
units. At the end of the year the association reported actual industry sales of 68,750 units.

Required:

(i) Compute the individual product and total sales volume variance.
(ii) Compute total sales quantity variance.
(iii) Compute the market size and market share variances.
(iv) Compute individual product and total sales mix variances.
(v) Comment on your results. (15 Marks) May/96

[Ans.: (i) PC: 12500000(F); Portable PC: 3900000(F); Super PC: 36000000(A); (ii) 15600000(F); (iii)
58500000(F) & 42900000(A); (iv) PC: 5500000(F); Portable PC: 3300000(F); Super PC: 44000000(A)]
[Note: Since question is silent that whether sales variances are supposed to calculated on the basis of value
or margin, we can calculate it on either basis]

Reconciliation Statement:-
Operating statement is prepared to reconcile the actual profit with the budgeted profit

Particulars Favorable Unfavorable (`)


Budgeted Profit :
Add Favorable variances & Deduct Unfavorable variances
Sales Variances :
Sales Margin price variance
Sales Margin mix variance
Sales Margin qty. variance
Total:
Cost variance :-
Material :
Price variance
Yield variance
Mix variance
Labour :
Rate variance
Mix variance
Yield variance
Idle time variance
Variable overhead variance :
Expenditure variance
Efficiency variance
Fixed overhead variance :
Expenditure variance
Efficiency variance
Capacity variance
Calendar variance
Total Cost Variances:
Actual Profit :
CA. Parag Gupta Ph.: +91 11 47665555 Paraggupta_ca@yahoo.co.in Costing & O.R.
World’s largest CA Final student’s consultancy group: http://groups.yahoo.com/group/costingbyparaggupta
Standard Costing & Variance Analysis - 86 -

1. Whenever we are following Marginal Costing instead of Absorption costing, Fixed overhead
expenditure variance = Fixed overhead cost variance & Fixed overhead volume variance or
any of its sub-parts can not be calculated in such cases. Also, while calculating sales margin
variances, Std. Margin = Std. Selling Price – Std. Variable Cost & Actual Margin = Actual
Selling Price – Std. Variable Cost
2. Standard Profit = Actual Output × Standard Margin p.u.
3. If question has provided information about Selling and administration expense, it should be
ignored. Reconciliation statement should be prepared showing actual profit before Selling
and distribution. After this, amount of selling and distribution expenses should be deducted to
arrive at the figure of actual profit for the month. Remember, Selling & distribution expense is
not an item of production cost.

Question 21: A company, which uses standard marginal costing, furnishes the following details relating to a
single product manufacturing and sold in a quarter:

Budget Actual
Sale units 6,000 6,400
(`’000) (`’000)
Sales 1500 1696
Direct materials 240 270
Direct labour 360 416
Variable overheads 600 648
Total variable costs 1200 1,334

The sales budget is based on the expectation of the company’s estimate of market share of 12%. The market
report reveals that the actual sales of the product in the whole country for the quarter is 60,000 units.
Further data are given as under:
Standard Actual
Direct material price per Kg. `8 `7.50
Direct labour rate per hour `6 `6.40

Required:

i) Compute the following variances for the quarter :


- Gross margin sales volume variance
- Market size variance
- Market share variance
. - Sales price variance
- Direct materials usage and price variance
- Direct labour efficiency and rate variances
- Variable overheads efficiency and expense variances.
ii) Prepare an operating statement reconciling the budgeted contribution with actual contribution.
(12 Marks) May/03 & (9 Marks) June/09 [Old Course-Adapted]

[Ans.: (a) Market size variance `60,000(F) , (b) Market share variance `40,000(A), (c) Gross margin sales
volume variance `20,000(F) , (d) Gross margin sales price variance `96,000(F), (e) Direct material usage
variance `32000(A), Direct Material Price variance `18,000(F), (f) Direct labour efficiency variance `6,000(A) ,
Direct labour rate variance `26,000(A), (g) Variance overhead efficiency variance `10,000(A), Variable
overhead expense variance `2,000(F) ; (2) Total actual contribution `3,62,000.]
Question 22: A single product company has furnished the following standard cost data per unit of output:

Direct materials 20 kg @ `10 per kg


Direct labour 12 hours at `5.50 per hour
Variable overheads 12 hours at `10 per hour
Fixed overheads `9,00,000 per month based on a normal volume of 60,000 direct labour hours
Selling price `600 per unit.

CA. Parag Gupta Ph.: +91 11 47665555 Paraggupta_ca@yahoo.co.in Costing & O.R.
World’s largest CA Final student’s consultancy group: http://groups.yahoo.com/group/costingbyparaggupta
Cost Accounting & Management - 87 -

The cost incurred and other relevant information for the month of November 2004 are as under:
Direct Materials used 1,00,000 kg at a cost of `10,50,000
Direct Wages paid `3,10,000 for 62,000 hours worked
Overheads `15,26,000 out of which a sum of `9,40,000 is fixed.
Actual output 4,800 units sold for `28,32,000

Assume no stocks of work-in-progress or finished goods at the beginning or the end of the month.

Required:

(i) Compute all variance


(ii) Prepare an operating statement reconciling the budgeted profit and actual profit.

[Ans.: (i) DMCV: `90000(A); DMPV: `50000(A); DMUV: `40000(A); DLCV: `6800(F); DLRV: `31000(F);
DLEV: `24200(A); VOCV: `10000(A); FOCV: `76000; FO Exp V: `40000(A); FOVV: `36000(A); FO Cap V:
``30000(F); FO Eff. V: `66000(A); SVV: `168000(A); SPV: `48000(A); S Vol. V: `120000(A); S Margin Vol. V:
`6800(A) (ii) Actual Profit: `(54000); Std. profit per unit: `34]
Question 23: The budgeted output of a single product manufacturing company for the year ending 31st
March was 5,000 units. The financial results in respect of the actual output of 4,800 units achieved
during the year were as under:
`
Direct material 29,700
Direct wages 44,700
Variable overheads 72,750
Fixed overheads 39,000
Profit 36,600
Sales 2,22,750

The standard wage rate is `4.50 per hour and the standard variable overhead rate is `7.50 per hour.
The cost accounts recorded the following variances for the year:

Variances Favourable Adverse


` `
Material price – 300
Material usage – 600
Wage rate 750 –
Labour efficiency – 2,250
Variable overhead expenses 3,000 –
Variable overhead efficiency – 3,750
Fixed overhead expense – 1,500
Selling price 6,750 –

Required:
(i) Prepare a statement showing the original budget.
(ii) Prepare the standard product cost sheet per unit.
(iii) Prepare a statement showing the reconciliation of originally budgeted profit and the actual profit.
[Ans.; Budgeted Profit: 37500; Selling price per unit: `45]

CA. Parag Gupta Ph.: +91 11 47665555 Paraggupta_ca@yahoo.co.in Costing & O.R.
World’s largest CA Final student’s consultancy group: http://groups.yahoo.com/group/costingbyparaggupta
Standard Costing & Variance Analysis - 88 -

Miscellaneous Topics :

Non Conventional Variance Analysis:

Quality Cost Variance = It represents difference between (a) actual quality cost, and (b) budgeted quality
cost.
Ex post variance analysis: Variance analysis if environment is different from that anticipated. In such cases
actual performance should be compared with a standard which reflects these changed conditions. As per
CIMA, Operating & Planning Variances are subsets of material total variance replacing traditional usage &
price variances. These variances are used to isolate variances caused by (i) unforeseen circumstances, i.e.
planning variance & (ii) operational variance, which reflects non-standard performance.

Planning variances seek to explain the extent to which the original standard needs to be adjusted in order to
reflect changes in operating conditions between the current situation and that envisaged when the standard
was originally calculated, in effect it means that the original standard is brought up to date so that it is a
realistic attainable target in current conditions. It can be controllable as well as uncontrollable.

Operating variances indicate the extent to which attainable targets (i.e. the adjusted standards) have been
achieved. Operating variances would be calculated after the planning variances have been established and
are thus a realistic way of assessing performance. Operating Variance is always controllable.

Price Planning Variance = (Std. Rate-Revised Std. Rate)× Standard Quantity on revised standard
Usage Planning Variance = (Std. Qty.- Revised Std. usage)×Std. Price
Price Operating Variance = (Prevailing Rate/Revised Std. Rate-Actual Rate)× Actual Quantity
Usage Operating Variance = (Std. usage based on revised std.-Actual Qty.)×Revised Std. Price

Case Study
XYZ Ltd. manufactures a standard animal feed.
The predetermined standards for the budget period Jan-March 2005 were set by management in October
2004.
Standard hours per tonne of product is 1.1
Standard direct labour rate per hour is `8.50
Standard usage of material per tonne of product is 1.2 tonnes
Standard price of material is `70 per tonne

Research shows that in the quarter ended 31 March 2005 the prevailing market price of material had been
`71 per tonne. Since the budget was set the wage rate had increased to `8.75 per hour, national pay award.
During the quarter modifications to plant and machinery shows that direct labour hours per unit should be
1.05 per tonne of product and that standard usage would reduce to 1.175 tonnes per tonne of product.

During the quarter ended 31 March 2005 activity and costs showed:
Actual production 15,400 tonnes
Raw material usage 16,555 tonnes
Actual cost of raw materials used `1,191,960
Actual direct labour cost 16,632 hours `143,035

Variance Analysis Report


Traditional Approach
Direct Labour
Direct Labour Cost Variance = Standard Cost for Actual Production – Actual Cost
= ((15400×1.1) hrs×`8.5) – `143035 = `955(F)
Direct Labour Rate Variance = (Standard Rate-Actual Rate)× Actual Hours
= (`8.50-`8.5999)×16632 = `1663 (A)
Direct Labour Efficiency Variance = (Std. Hours for actual Output-Actual Hours)×Std. Rate
= (16940-16632)×`8.50 = `2618 (F)
Direct Material
Direct Material Cost Variance = Standard Cost for Actual Production – Actual Cost
= ((15400×1.2) tonnes×`70) – `1191960 = `101640(F)

CA. Parag Gupta Ph.: +91 11 47665555 Paraggupta_ca@yahoo.co.in Costing & O.R.
World’s largest CA Final student’s consultancy group: http://groups.yahoo.com/group/costingbyparaggupta
Cost Accounting & Management - 89 -

Direct Material Rate Variance = (Standard Rate-Actual Rate)× Actual Quantity


= (`70-`72)×16555 = `33110 (A)
Direct Material Quantity Variance = (Std. Qty. for actual Output-Actual Qty.)×Std. Rate
= (18480-16555)×`70 = `134750 (F)
Operating & Planning Variances
Direct Material Price Operating Variance = (Prevailing Rate-Actual Rate)× Actual Quantity
= (`71-`72)×16555 = `16555 (A)
Direct Material Price Planning Variance = (Std. Rate-Revised Std. Rate)× Standard Quantity on revised
standard
= (`70-`71)×(15400×1.175) = `18095 (A)
Direct Material Usage Operating Variance = (Std. usage based on revised std.-Actual Qty.)×Revised Std.
Price
= (18095-16555)×`71 = `109340 (F)
Direct Material Usage Planning Variance = (Std. Qty.- Revised Std. usage)×Std. Price
= (18480-18095)×`70 = `26950 (F)
Summary
Planning Variances `
`
Price 18,095 (A)
Usage 26,950 (F)
8,855 (F)
Operating Variances
Price 16,555 (A)
Usage 109,340 (F)
92,785 (F)
Traditional Variance 101,640 (F)

Direct Labour
Direct Labour Rate Operating Variance = (Revised Rate-Actual Rate)× Actual Hours
= (`8.75-`8.5999)×16632 = `2495 (F)
Direct Labour Rate Planning Variance = (Std. Rate-Revised Std. Rate)× Standard hours based on revised
standard
= (`8.50-`8.75)×(15400×1.05) = `4042.50 (A)
Direct Labour Efficiency Operating Variance = (Std. hours based on revised std.-Actual Hrs.)×Revised Std.
wage rate
= (16170-16632)×`8.75 = `4042.50 (A)
Direct Labour Efficiency Planning Variance = (Std. Hrs.-Revised Std. hrs.)×Std. Rate
= (16940-16170)×`8.50 = `6545 (F)
Summary
Planning Variances `
`
Rate 4,042.50 (A)
Efficiency 6,545 (F)
2,502.50 (F)
Operating Variances
Rate 2,495 (F)
Efficiency 4,042.50 (A)
1,547.5 (A)
Traditional Variance 955 (F)

Note: In foregoing case material planning variance is controllable though labour planning variance is
uncontrollable.

CA. Parag Gupta Ph.: +91 11 47665555 Paraggupta_ca@yahoo.co.in Costing & O.R.
World’s largest CA Final student’s consultancy group: http://groups.yahoo.com/group/costingbyparaggupta
Standard Costing & Variance Analysis - 90 -

Ex post sales variances {Least important topic}:


The conventional sales volume variance reports the difference between actual and budgeted sales, priced at
the budgeted contribution per unit. The variance merely indicates whether sales volume is greater or less than
expected. It does not indicate how well sales management actual sales volume should be compared with an
ex post estimate that reflects the market conditions prevailing during that period. In such cases only total
sales margin variance be reported & it should be separated in planning and appraisal element using following
formulae:

Total sales margin variance (planning element):


= {Ex post budgeted sales volume × (Ex post selling price – Standard cost) – Original budgeted sales
volume × (Budgeted selling price – Standard cost)}

Total sales margin variance (appraisal element):


= {Actual sales volume × (Actual selling price – Standard cost)}
= Ex post budgeted sales volume × (Ex post selling price – Standard cost)}

The figure of “Ex post budgeted sales volume” for a particular product can be determined by estimating the
total market sales volume for the period and then multiplying the estimate by the target percentage of market
share. Where industry statistics are published, this calculation should be based on actual total industry sales
volume.
Question 24: C Preserves produces Jams, Marmalade and Preserves. All the products are produced in
a similar fashion; the fruits are cooked at low temperature in a vacuum process and then blended with
glucose syrup with added citric acid and pectin to help setting

Margins are tight and the firm operates, a system of standard costing for each batch of Jam.
The standard cost data for a batch of raspberry jam are

Fruits extract 400 kgs @ `16 per kg.


Glucose syrup 700 kgs @ `10 per kg.
Pectin 99 kgs. @ 33.2 per kg.
Citric acid 1 kg at `200 per kg.
Labour 18 hours @ `32.50 per hour.
Standard processing loss 3%.

The climate conditions proved disastrous for the raspberry crop. As a consequence, normal prices in the
trade were `19 per kg for fruits abstract although good buying could achieve some savings. The impact
of exchange rates for imported sugar plus the minimum price fixed for sugarcane, caused the price of
syrup to increase by 20%.

Fruit extract 428 Kgs at `18 Per Kg .


Glucose syrup 742 Kgs at `12 per Kg .
Pectin 125 Kgs at `32.8 per Kg .
Citric aid 1 Kgs at `95 per Kg .
Labour 20 hrs .at `30 per hour .

Actual output was 1,164 kgs of raspberry jam.

You are required to:

(i) Calculate the ingredients planning variances that are deemed uncontrollable.
(ii) Calculate the ingredients operating variances that are deemed controllable.
(iii) Calculate the mixture and yield variances.
(iv) Calculate the total variances for the batch. (CIMA May/88) & (11 Marks) May/05

[Ans.: (1) Total `2,600(Adverse), (2) Total `1,316.2(adverse) Price variance 583(F), Usage variance 583 (F)
Usage variance 1,899.2 (A) (3) Total 340.3 (A) Yield variance 155.9 (A), Labour operating variance 15 (A) ,
(5) Total variance 3931.2 (A)]

CA. Parag Gupta Ph.: +91 11 47665555 Paraggupta_ca@yahoo.co.in Costing & O.R.
World’s largest CA Final student’s consultancy group: http://groups.yahoo.com/group/costingbyparaggupta
Cost Accounting & Management - 91 -

[Hint: Material mix & yield variance are supposed to be calculated on revised standard rates & quantity & not
on standard rates and quantity]

Budget Ratios:

1) Efficiency Ratio = Output expressed in terms of std. hours ×100


Actual Hours spent for producing the output
= Standard Hours ×100
Actual Hours worked
= Actual Output ×100
Standard Output

2) Activity Ratio = Actual Output in Std. hours ×100


(a.k.a. Volume Ratio) Budgeted output in std. hours
= Standard Hours ×100
Budgeted Hours

3) Actual usage of budgeted capacity ratio = Actual Working Hours ×100


(a.k.a. Capacity Ratio) Budgeted Hours

= Standard Output ×100


Budgeted Output

4) Calendar Ratio = Actual No. of working days in a period ×100


No. of working days in related budgeted period
= Max. possible standard working hrs in actual days×100
Budget Hours

5) Actual Capacity Usage Ratio = Actual Hours worked ×100 0


Max. Possible no. of working hours in budget days

6) Standard Capacity Usage Ratio = Budgeted Hours ×100 0


Max. Possible no. of working hours in budget days
Reconciliation of Budget ratios:
Activity Ratio = Efficiency Ratio × Capacity Ratio

Question 25: What are the various formulae used in calculating budget ratios. (3 Marks) June/09-N.C.
Question 26: A company manufactures two products X and Y. Product X requires 8 hours to produce while
Y requires 12 hours. In April, 2004, of 22 effective working days of 8 hours a day, 1,200 units of X and 800
units of Y were produced. The company employs 100 workers in production department to produce X and Y.
The budgeted hours are 1,86,000 for the year.

Calculate Capacity, Activity and Efficiency ratio and establish their relationship. (6 Marks) Nov./04

[Ans.: Capacity ratio = 113.55%, Activity ratio = 123.87%, Efficiency ratio = 109.09%]
Question 27: A company manufactures two products X & Y. Product X requires 5 hours to produce while Y
requires 10 hours. In July 2004, of 25 effective working days of 8 hours a day, 1000 units of X and 600 units
of Y were produced. The company employs 50 workers in the production department to produce X and Y. The
budget hours are 102000 for the year.

Calculate capacity ratio, activity ratio and efficiency ratios. Also establish their interrelationship

[Ans.: 117.65%, 129.41%, 110%] (7 Marks) Nov./96

CA. Parag Gupta Ph.: +91 11 47665555 Paraggupta_ca@yahoo.co.in Costing & O.R.
World’s largest CA Final student’s consultancy group: http://groups.yahoo.com/group/costingbyparaggupta
Standard Costing & Variance Analysis - 92 -

Question 28: The following data have been obtained from the records of a machine shop for an average
month:

Budget:
No. of working days 25
Working hours per day 8
No. of direct workers 16
Efficiency One standard hour per clock hour
Down time 20%
Overheads Fixed `15,360
Variable `20,480
The actual data for the month of September 1985 are as follows :
Overhead Fixed `16,500
Variable `14,500
Net operator hours worked 1920
Standard hours produced 2112
There was a special holiday in September 1985.

Required to present reports to Departmental manager:

(i) Calculate efficiency, activity, calendar and standard capacity usages & actual capacity utilization ratio.
(ii) Setting out the analysis of variances. (Nov./85), (10 Marks) Nov./10-N.C. [Adapted]

[Ans.: (i) 110%, 82.5%, 96%, 80%, 75%;(ii) FOCV 3828(A), FOEV 1140(A), FOVV 2688(A), FOCal V 614(A),
FOCV 3226(A), FOEV 1152(F), VOCV 2396(F), VOEff V 1536(F), VOExp V 860(F)]
Question 29: Following data is available for T.T.D and Co:

Standard working hours 8 hours per day of 5 days per week


Maximum capacity 50 employees
Actual working 40 employees
Actual hours expected to be worked per four week 6,400 hours
Std. hours expected to be earned per four weeks 8,000 hours
Actual hours worked in the four week period 6,000 hours
Standard hours earned in the four week period 7,000 hours.

The related period is of 4 weeks. In this period there was a one special day holiday due to national event.
Calculate the following ratios:

(1) Efficiency Ratio, (2) Activity Ratio, (3) Calendar Ratio, (4) Standard Capacity Usage Ratio, (5) Actual
Capacity Usage Ratio. (6) Actual Usage of Budgeted Capacity Ratio

[Ans.: (1) 116.67%, (2) 109.375%, (3) 95%, (4) 80%, (5) 75%, (6) 93.75%]
[Hint: “Actual hours expected to be worked per four week” means budgeted hours, “Std. hours expected to be
earned per four weeks” means Max. Possible no. of working hours in budget period & “Standard hours earned
in the four week period” means Actual output expressed in terms of standard hours (i.e. standard hours for
actual output)]

Possible Causes of Cost Variances :


Variance Favourable Adverse
a) Material Price  Unforeseen discounts received  Price Increase
 Greater care taken in purchasing  Careless Purchasing
 Change in material standard  Change in material Standard
b)Material Usage  Material used of higher quality than  Defective material
standard  Excessive Waste
 More effective use of material  Theft
 Errors in allocating material to jobs  Stricter quality control
 Errors in allocating material to jobs
c) Labour Rate Pay  Use of apprentices or other  Wage Rate Increase
CA. Parag Gupta Ph.: +91 11 47665555 Paraggupta_ca@yahoo.co.in Costing & O.R.
World’s largest CA Final student’s consultancy group: http://groups.yahoo.com/group/costingbyparaggupta
Cost Accounting & Management - 93 -

workers at a rate of pay lower than


standard
d) Idle Time  Machine Breakdown
 Non-Availability of Labours
 Illness or injury to workers
e)Labour Efficiency  Output produced more quickly than  Lost time in excess of standard
expected, i.e., actual output in allowed
excess of standard output set for  Output lower than standard set
same no. of hrs. because of work because of deliberate restriction,
motivation, better quality of Labour lack of training, or sub standard
or equipments Labour used.
 Errors in allocating time to jobs.
f) Overhead  Savings in costs incurred  Increase in cost of services used.
Expenditure  More economical use of services Excessive use of change in type of
services used.
g) Overhead  The same reasons as for the  The same reasons as for the
efficiency labour efficiency variance have labour efficiency variance have
caused overhead recovery to be caused overhead recovery to be
different from standard. different from standard.
h) Overhead Volume  Excess of Actual time worked over  Excessive Idle time
budget.  Shortage of Plant Capacity

A/cing procedure for standard cost:

Single Plan: It contemplates analysis of variance at source i.e. the analysis of variance is done from
the original documents like invoices, labour sheets, etc. Unlike partial plan, price variance can be
analyzed at the time of receipt of material (in case of material) & usage variance can be analyzed &
recorded as & when finished output is recorded i.e. when excess materials are used. The features of
single plan can be summarized as:

i) Work-in-progress A/c is debited & credited with standard cost.


ii) Inventory is valued at standard cost
iii) All Variances are computed at any stage before debiting WIP A/c. Rate & Expenditure
variances are computed at the inception. Quantity/efficiency variances are calculated at the
time of crediting control A/cs.

1 Material Control A/c Dr. Actual Qty Purchased @ Std. Rate


Material Price Variance A/c Dr. Unfavourable Variance
To Material Price Variance A/c Favourable Variance
To A/cs Payable Actual Qty @ Actual Rate

(To record purchase of raw materials at


standard price and related
unfavourable/favourable variance)

2 Work-in-Progress Control A/c Dr. Std. Qty @ Std. Rate


Material Usage Variance A/c Dr. Unfavourable Variance
To Material Usage Variance A/c Favourable Variance
To Material Control A/c Actual Qty @ Std. Rate

(To transfer raw materials to production at


standard usage rates and related
unfavourable/favourable quantity variance)

3 Same Entry for Labour as in entry 1


4 Same Entry for Labour as in entry 2

CA. Parag Gupta Ph.: +91 11 47665555 Paraggupta_ca@yahoo.co.in Costing & O.R.
World’s largest CA Final student’s consultancy group: http://groups.yahoo.com/group/costingbyparaggupta
Standard Costing & Variance Analysis - 94 -

5 Factory Overhead Control A/c Dr. Actual Overhead


To Cash Actual Overhead
(To record expenses on overhead)

6 Work-in-Progress A/c Dr. Absorbed Overhead


Overhead Expense Variance A/c Adverse/Underapplied
Overhead Efficiency Variance A/c Adverse/Underapplied
To Overhead Expense Variance A/c Favourable/Overapplied
To Overhead Efficiency Variance A/c Favourable/Overapplied
To Factory Overhead Control A/c Actual Qty @ Actual Rate

(To increase work in process for the standard


variable overhead, and record the related
efficiency and spending variance)

1. Overhead Volume Variance will be calculated inspite of Overhead Efficiency Variance in case of Fixed
Note: Overhead.

Partial Plan: In this system variances are analyzed at the end of period. Overhead A/c will be opened
& closed on normal basis. If nothing is specified in question, Partial Plan will be followed. Some salient
features of partial plan are:

i) Work-in-progress A/c is debited with actual cost of material, actual cost of labour & actual
overhead.
ii) Work-in-progress A/c will be credited with standard cost of production.
iii) At the end of A/cing period work-in-progress A/c is credited with standard cost of unfinished
goods.
iv) After entering at iii) above is passed, debit & credit sides of work-in-progress account are
compared & difference is transferred to cost variance, which is further analyzed for reporting to
management on the basis of additional information not recorded in accounts.
1 Material Control A/c Dr. Actual Qty purchased @ Actual Rate
To General Ledger Adjustment
A/c
(To record purchase of raw materials
at actual price)

2 Work-in-Progress Control A/c Dr. Actual Qty consumed @ Actual Rate


To Material Control A/c
(To transfer raw materials to
production)

3 Same Entry for Labour as in entry 1


4 Same Entry for Labour as in entry 2

5 Factory Overhead Control A/c Actual Overhead


To General Ledger Adjustment
A/c
(To record expenses on overhead)

6 Work-in-Progress Control A/c Dr. Actual Overhead


To Factory Overhead Control A/c Actual Overhead
(To record expenses on overhead
incurred)

CA. Parag Gupta Ph.: +91 11 47665555 Paraggupta_ca@yahoo.co.in Costing & O.R.
World’s largest CA Final student’s consultancy group: http://groups.yahoo.com/group/costingbyparaggupta
Cost Accounting & Management - 95 -

7 Finished Goods A/c Dr. Standard Cost


To Work-in-Progress A/c Standard Cost
(Being the standard cost of production
transferred to finished good)

8 Cost of Sales A/c Dr. Standard Cost


To Finished Goods A/c Standard Cost
(Being the standard cost of goods
sold transferred to Cost of Sales A/c)

9 Work-in-Progress A/c Dr.


To All favourable variances Favourable/Overapplied
(Being variances carried to respective
A/cs pending investigation before
being finally disposed off)

10 All unfavourable variances Dr. Unfavourable/Underapplied


To Work-in-Progress A/c
(Being variances carried to respective
A/cs pending investigation before
being finally disposed off)

In case Non-Integrated System is adopted, expenses will be transferred to Cost Ledger Control A/c (also
known as Nominal Ledger Control A/c, General Ledger Control A/c) in spite of A/cs Payable, Wages Payable,
Cash A/c, etc. Also balance of P&L A/c is transferred to Cost Ledger Control A/c. In case of Partial Plan it
generally taken as Non-Integrated System

Disposition of Variances: (3 Marks) Nov/98


There is no unanimity of opinion in regard to disposition of variances. The following are various methods:

1) Write off all variances to P&L A/c or cost of sales every month.
2) Distribute the variance prorate of cost of sales, work-in-progress and finished good stocks.
3) Write off quantity variance to P&L A/c but the price variance may be spread over cost of sales, work-in-
progress & finished goods stock. The reason behind apportioning price variances to inventories & cost of
sales is that they represent cost although they are described as variance.

Question 30: Under the single plan, record the journal entries giving appropriate narration, with indication of
amounts of debits or credits alongside the entries, for the following transactions using the respective control
A/c.

(i) Material price variance (on purchase of materials)


(ii) Material usage variance (on consumption)
(iii) Labour rate variance. (6 Marks) Nov./06

Question 31: Material purchased 10,000 pieces at `1.10 `11,000


Materials consumed 9,500 pieces at `1.10 `10,450
Actual wages paid 2,475 hours at `3.50 `8,662.50

Actual factory expenses incurred `17,000 (Budgeted `16,500) Units produced: 900 units and sold at `60
per unit.

The standard rates and prices are as under:

Direct materials Re. 1.00 per unit Standard input 10 pieces per unit Direct labour rate `3.00 per hour

CA. Parag Gupta Ph.: +91 11 47665555 Paraggupta_ca@yahoo.co.in Costing & O.R.
World’s largest CA Final student’s consultancy group: http://groups.yahoo.com/group/costingbyparaggupta
Standard Costing & Variance Analysis - 96 -

Standard requirement 2.5 hours per unit


Overheads `6.00 per labour hour

Explain the operation of the recording of standard cost under this method.

Question 32: Standard cost sheet per unit output is as under `


Direct material 3 pcs. @ `2.15 6.45
Direct Labour:
Dept. A 2 hrs @ `1.75 3.50
Dept. B 4 hrs. @ `1.50 6.00 9.50
Overheads :
Dept. A 2 hrs. @ Re. 0.50 1.00
Dept. B 4 hrs. @ Re. 1.00 4.00 5.00
20.95
Transactions for the period are as under :
Materials purchased and consumed:
8,600 pcs. @ `2.50 each
Labour time spent
Dept. A. 5,200 hours
Dept. B. 12,000 hours
There is no change in labour rates.
Actual factory overheads are :
Dept. A. `3,000
Dept B. `12,500
Units produced:
Dept. A. 2,800
Dept. B. 2,800
Budgeted overheads:
Dept. A. `3,000
Dept. B. `12,000
Record the transactions under single plan.
Question 33[Reverse calculations]: Upasana Ltd. manufactures paint. It uses a standard costing system
and the variances are reported to the management on the fortnightly basis. A fire destroyed some important
records of the company. You have been able to collect the following information from the spoilt papers/
reports and as a result of consultation with accounting personnel in respect of a fortnight:
a) The paint requires 2 types of raw material RM 1 and RM 2 . The standard quantity of RM 2 in the final
product is 5 liters and the standard cost thereof is `36 per liter.
b) The company purchased 200 kg of RM 1 and 550 liters of RM 2 during that fortnight.
c) The standard wage rate is `24 per labor hour. Actual labor hours were 460 during that fortnight.
d) Variances are disclosed from some spoiled papers are:
i. Price Variance (RM 2 ) - `1,320 (A)
ii. Usage Variance (RM 1 ) - `240 (F)
iii. Labor efficiency variance - `1,440 (A)
e) Some incomplete ledger entries for that fortnight reveal
f)
(1) Sundry Creditors
` `
Purchase of raw materials 25,440
(2) RM 2
Opening Balance 3,600 Closing Balance 8,280
(3) RM 1
Opening Balance 0 3600
Closing Balance 1200
CA. Parag Gupta Ph.: +91 11 47665555 Paraggupta_ca@yahoo.co.in Costing & O.R.
World’s largest CA Final student’s consultancy group: http://groups.yahoo.com/group/costingbyparaggupta
Cost Accounting & Management - 97 -

(4) Works-in-progress
Opening Balance 0
RM 2 14,400 Closing Balance 0
(5) Wages
Paid & Outstanding 10,350

You have been asked to compute the meaningful variances to be presented before the management. (Key
computations should form part of the answer.) (19 Marks) Nov./95

[Ans.: Material cost variances `7200(A); Material price variance (RM 1 ) `480(F); Material usage variance
(RM 2 ) `720(A); Labour cost variance `750(A); Labour rate variance `690(F)]
Question 34 [Reverse calculations]: Rainbow Ltd. manufactures paint in batches. The company uses
standard costing system and the variances are reported weekly. You have taken the account sheet for study
for variance analysis discussion. While working coffee was spilled on these sheets and only following could
have been retrieved:
Dr. Cr.
Raw Material -1
Beg. Balance 0
18,000
Closing Balance 6,000
Raw Material -2
Beg. Balance 18,000
Closing Balance 41,400
Work in Progress
Beg. Balance 0
Raw Material -2 72,000
Closing Balance 0

Sundry Creditors
1,27,200
Wages outstanding
51,750
Quantity Variance-Material-1
1,200
Price Variance-Material-2
6,600
Efficiency Variance-Labour
7,200

Other information’s are: standard cost of Material – 2 is `180 per litre and standard quantity is 5 litres.
Standard wages rate is `24 per hour and a total 2,300 hours were worked during the week. 1,000 kg of
Material -1and 550 litres of Material-2 were purchased. Sundry creditors are for material acquisition, and
wages outstanding pertain to direct labour.

You are required to compute Material-1 Rate Variance, Material-2 Quantity Variance & Labour Spending
Variance, Standard hours allowed for production and purchase value of Material-1 for variance analysis
discussion.
[Ans.: Material – 1 rate variance = `2,400(F)
Material - 2 quantity variance = `3,600(A)
Labour spending variance = `3,450(F)
Standard hour allowed = 2000 hour
Cost of material allowed = `21,600] (11 Marks) Nov/05
[Note: A/cs in this question are misprinted in suggested answers of ICAI]
Question 35: F Manufacturing Ltd., uses the three variances method to analyses the manufacturing
overhead variances. Manufacturing overhead variances for the fiscal year just ended were computed as
follows:

CA. Parag Gupta Ph.: +91 11 47665555 Paraggupta_ca@yahoo.co.in Costing & O.R.
World’s largest CA Final student’s consultancy group: http://groups.yahoo.com/group/costingbyparaggupta
Standard Costing & Variance Analysis - 98 -

Spending `86,000 Adverse


Efficiency `36,000 Favorable
Volume `80,000 Favorable
The manufacturing overhead application rate for the year was `160 per machine hours of which `60 per
machine hour was the variable component. The year end balance in the Manufacturing Overhead Control
Account was `16,50,000 and the standard machine hours for the year were 11,300.

From the above data compute:


(i) Budgeted Machine Hours.
(ii) Actual Machine Hours.
(iii) Applied Manufacturing Overhead.
(iv) Total Amount of Fixed Overhead Cost (budgeted). (13 Marks) Nov./00

[Ans.: (i) 10500hrs (ii) 10700hrs (iii) 1680000 (iv) 922000]


[Note.: There seems some drafting error in question because Budgeted Fixed Overhead can also be
calculated as Budged Hours calculated in (i) × Standard Fixed Overhead application rate i.e. 10500 × (160-
60) = 10,50,000, although value of Budgeted Fixed Overhead as per calculation in (iv) is `9,22,000]

Miscellaneous Questions
Question 36: Following is the standard cost card of a component:

Materials 2 Units at `15 `30


Labour 3 Hours at `20 `60
Total overheads 3 Hours at `10 `30

During a particular month 10,000 units of the component were produced and the same was found to be at
60% capacity of the budget. In preparing the variance report for the month, the cost accountant gathered the
following information:
Labour `6,50,000
Variable overheads `2,00,000
Fixed overheads `3,00,000
Material price variance ` 70,000 (A)
Material cost variance ` 50,000 (A)
Labour rate variance ` 50,000 (F)
Fixed overhead expenditure variance ` 50,000 (A)

You are required to prepare from the above details:


(1) Actual material cost incurred
(2) Standard cost of materials actually consumed
(3) Labour efficiency variance
(4) Variable OH efficiency variance
(5) Variance OH expenditure variance
(6) Fixed OH efficiency variance
(7) Fixed OH capacity variance
(8) Fixed OH volume variance

[Ans.: (1) `350000; (2) `280000; (3) `100000(A); (4) 25000(A); (5) 25000(A); (6) 25000(A); (7) 75000(A); (8)
100000(A)] (14 Marks) May 99 & (14 Marks) Nov./05
Question 37: The following is the Operating Statement of a company for April, 2001.
`
Budgeted profit 1,00,000

Variances: Favourable Adverse


` `
Sales Volume 4,000

CA. Parag Gupta Ph.: +91 11 47665555 Paraggupta_ca@yahoo.co.in Costing & O.R.
World’s largest CA Final student’s consultancy group: http://groups.yahoo.com/group/costingbyparaggupta
Cost Accounting & Management - 99 -

Price 9,600
Direct Materials - Price 4,960
- Usage 6,400
Direct Labour - Rate 3,600
- Efficiency 3,600
Fixed Overhead -Efficiency 2,400
Capacity 4,000
Expense 1,400

17,000 22,960 5,960(A)


Actual profit 94,040
Additional information is as under :
Budget for the year 120,000 unit

Budgeted fixed overhead `4,80,000 per year


Standard cost of one unit of product is :
Direct materials 5kg @ `4 per kg .
Direct labour 2hour @ `3 per hour
Fixed overheads are absorbed on direct labour
Profit 25% on sales.
Note
You are required to prepare the annual financial profit/loss statement for April, 2001 in the following format
Account Qty./hour Rate/price Actual value
Sale
Direct materials
Direct labour
Fixed overheads
Total cost
Profit (13 Marks) May/01
[Ans : Profit 94040]
[Note: Although annual statements have been demanded but we will prepare financial statement for
April,’01 only, because operating statement has been provided for April 2001 only.]
Question 38[Standard Costing with equivalent production units]: A single product company has
prepared the following cost sheet based on 8,000 unit of output per month:
Direct materials 1.5 kg @ `24 per kg 36.00
Direct labour 3 hour @ `4 per hour 12.00
Factory overhead 12.00
Total 60.00

The flexible budget for factory overhead is as under:


Output (unit) 6,000 7,500 9,000 10.500
Factory overhead (`) 81,600 92,400 1,03,200 1,14,000

The actual results for the month of October 2002 are given below:
- Direct material purchase and consumed were 11,224 Kg at `2,66,570.
- Direct labour hour worked were 22,400 and direct wages paid amounted to `96,320.
- Factory overhead incurred amounted to `96,440 out of which the variable overhead is ` 2.60 per direct
labour hour worked.
- Actual output is 7,620 unit.
- Work-in-process:
Opening WIP: 300 unit:
Material 100% complete
Labour and overhead 60% complete
Closing WIP: 200 unit:
Material 50% complete
Labour and overhead 40% complete

You are required to analyze the variance. (12 Marks) Nov/02

CA. Parag Gupta Ph.: +91 11 47665555 Paraggupta_ca@yahoo.co.in Costing & O.R.
World’s largest CA Final student’s consultancy group: http://groups.yahoo.com/group/costingbyparaggupta
Standard Costing & Variance Analysis - 100 -

[Ans.: Material price variance `2806(F); Material cost variance `550(F); Material usage variance `2256(A);
Labour cost variance `6080(A); Labour efficiency variable `640(F); Variable overhead expenditure variance
`4480(A); Variable overhead efficiency variance `384(F); Total variance overhead cost variance `4096(A);
Expenditure variance `200(F); Efficiency variable `256(F); capacity variable `2560(A); Volume variance
`2304(A); Total fixed overhead cost variance `2104(A)]
Question 39[Standard Costing with equivalent production units]:
The following information relates to a manufacturing concern (`)
Material A 24,000 kgs @ `3 per kg 72,000
Material B 12,000 kgs @ `4 per kg 48,000
Wages 60,000 hours @ `4 per hour 2,40,000
Variable overheads 60,000 hours @ Re. 1 per hour 60,000
Fixed overheads 60,000 hours @ `2 per hour 1,20,000
Total cost 5,40,000
Budgeted profit 60,000
Budgeted sales 6,00,000
Budgeted production (units) 12,000

Actual (`)
Sales (9,000 units) 4,57,500
Material A consumed 22,275 kgs. 62,370
Material B consumed 10,890 kgs 44,649
Wages paid (48,000 hours) 1,91,250
Fixed overhead 1,20,900
Variable overhead 45,000
Labour hours worked 47,700
Closing work in progress 900 units
Degree of completion
Material A and B 100%
Wages and overheads 50%

You are required to:

i) Calculate all the material and labour variances.


ii) Calculate variable overhead expenditure and efficiency variances, fixed overhead expenditure and
volume variances and sales price and sales volume variances. (10 Marks) Nov./09-O.C.

[Ans.: Material price variance `3366(F); Material cost variance `8019(A); Material usage variance `11385(A);
Material Yield Variance = 11550(A); Material Mix Variance = 165(F); Labour cost variance `2250(A); Labour
efficiency variable `1800(A); Labour rate variance `750(F); Labour Idle time variance `1200(A); Variable
overhead expenditure variance `2700(F); Variable overhead efficiency variance `450(A); Fixed overhead
expenditure variance `900(A); Fixed overhead volume variance `25500(A); Sales price variance `7500(F);
Sales volume variance `150000(A)]
Question 40[Standard Costing with equivalent production units]: The standard product-cost sheet of
MSC Ltd., which plans to produce 8000 units of a product, is as under:
(`)
Direct materials (1.50 kg. @`6) 9
Direct labour (4 hrs. @ `6) 24
Variable overheads 4
Fixed overheads 12
Selling price 53
The actual results of a period are as under:
Actual orders received 8200 units
Actual sales 7500 units
Actual production 7500 units
Direct materials purchased & issued to production 12000 kg
Direct materials price/kg `6.50
Direct labour hours worked 29000
Direct labour hour rate `6.25
CA. Parag Gupta Ph.: +91 11 47665555 Paraggupta_ca@yahoo.co.in Costing & O.R.
World’s largest CA Final student’s consultancy group: http://groups.yahoo.com/group/costingbyparaggupta
Cost Accounting & Management - 101 -

Total overheads incurred `130000


Out of which variable overheads were `36000

Closing work-in-progress 300 units-Materials 100% complete & Labour & overheads 60% complete.
Opening work-in-progress 600 units-Materials 100% complete & Labour & overheads 75% complete.
The budgeted market share was 16% and the actual sales in the country for the period of this product is
60000 units.
Actual selling price `54 per unit.
Analyse the variances in as much details as possible. (ICWA-June/98)

[Ans.: Non-Conventional Analysis: Sales Margin Production Quantity Variance = 2800(A) Market size
variance: `6400(F); Market share variance: `5600(A); Conventional Analysis: Sales margin volume variance:
`2000(A); Sales margin price variance: `7500(F); Material usage variance: `7200(A); Material price variance:
`6000(A); Labour efficiency variance: `480(A); Labour rate variance: `7250(A); Variable overhead efficiency
variance: `80(A); Variable overhead expenditure variance: `7000(A); Fixed overhead efficiency variance:
`240(A); Fixed overhead volume variance: `9240(A); Fixed overhead capacity variance: `9000(A); Fixed
overhead expenditure variance: `2000(F)]
[Note: Since actual order received ≠ actual sales quantity, Market share variance will be on the basis of
actual order received and we will also calculate one further variance regarding inefficiency of production
department about fulfilling order quantity, Sales Margin Production Quantity Variance = (Actual sales quantity
– Sales order quantity) × Std. margin p.u. While calculating all other variance sales order quantity shall be
ignored.]
Question 41: The following figures are available. Find out the missing figures, giving appropriate
`
Budgeted profit 15,000
Less: Adverse variances:
Contribution price variance 10,600
Direct materials variance 1,000
Fixed overhead variance 600 (12,200)
2,800
Add: Favourable variances
Contribution quantity variance 1,800
Direct wages variance 600
Variable overhead variance 1,800 4,200
Actual profit 7,000
There is no inventory
Production units = Sales units for both actual and budget
Other Information
Standard selling price `18/unit
Standard variable cost `15/unit
Standard contribution `3/unit
Actual selling price `17/unit
Budgeted sales 10,000 units

Standard material cost p.u. = Re. 1 (which is 5 kg. @ `20 Paise/kg.).


Material usage variance = 400 (Adv.)
Actual labour hours @ actual rate = `63,000
Actual labour hours @ standard rate = `61,950
Variable overhead standard rate = `2
Standard hours of production = 4 per unit
Variable overhead at standard rate = `84,800.
Variable overhead expenditure variance = 400 (A).
Budgeted fixed overhead = `15,000.

Find out the following:

(i) Actual sales units


(ii) Actual sales rupees

CA. Parag Gupta Ph.: +91 11 47665555 Paraggupta_ca@yahoo.co.in Costing & O.R.
World’s largest CA Final student’s consultancy group: http://groups.yahoo.com/group/costingbyparaggupta
Standard Costing & Variance Analysis - 102 -

(iii) Actual quantity of raw materials used


(iv) Labour efficiency variance
(v) Actual variable overhead in rupees
(vi) Variable overhead efficiency variance
(vii) Actual Fixed Overhead
(viii) Operating profit variance (8 Marks) Nov/06

[Ans.: (i) 10,600 units (ii) 1,80,200 (iii) 55,000 Kgs (iv) 1650 (f) (v) `83,000 (vi) 2,200 (F) (vii) `15,600 (viii) for
budget profit `8,000(A) For standard profit `9,800(A)]
[Hint: Operating profit variance means summation of all cost variances with Sales margin variance]
Question 42: A company produces a product X, using raw materials A and B. The standard mix of A and B is
1:1 and the standard loss is 10% of input.
You are required to compute the missing information indicated by “?” based on the data given below:

A B Total
Standard price of raw material (`/kg.) 24 30
Actual input (kg.) ? 70
Actual output (kg.) ?
Actual price `/kg. 30 ?
Standard input quantity (kg.) ? ?
Yield variance (sub usage) ? ? 270(A)
Mix variance ? ? ?
Usage variance ? ? ?
Price variance ? ? ?
Cost variance 0 ? 1300(A)
[Ans.: (14 Marks) May/07

]
Question 43: A single product company operates a system of standard costing. The following data relate to
actual output, sales, costs and variances for a month :

Actual output 18,000 units


`
Actual sales and costs incurred :
Sales 12,15,000
Direct materials purchased and used 63,000 kg 2,04,750
Direct wages 2,12,040
Variable overheads 2,77,020
Fixed overheads 3,25,000
Total costs 10,18,810
Profit 1,96,190

Standard wage rate is `6 per hour. Budgeted output for the month is 20,000 units.
Variances are :

Direct materials — Price variance 15,750(A)


— Usage variance 27,000(A)
Direct labour — Rate variance 6,840(A)

CA. Parag Gupta Ph.: +91 11 47665555 Paraggupta_ca@yahoo.co.in Costing & O.R.
World’s largest CA Final student’s consultancy group: http://groups.yahoo.com/group/costingbyparaggupta
Cost Accounting & Management - 103 -

— Efficiency variance 10,800(F)


Variable overheads — Efficiency variance 14,400(F)
— Expense variance 3,420(A)
Fixed overheads — Expense variance 25,000(A)
Sales price variance 45,000(F)
Required :

(i) Present the original budget along with cost sheet showing the standard cost and profit per unit.
(ii) Calculate the sales gross margin volume and fixed overheads volume variances.
(iii) Prepare an operating statement reconciling the budgeted profit with actual profit.

[Ans.: (1) Standard gross margin `2,60,000 ; (2) Sales gross margin volume variance `26,000 (A) fixed
overhead volume variance `30,000(A) ; (3) Budgeted profit `2,60,000]
(12 Marks) May/04 & (11 Marks) Nov./08-O.C. [Adapted]
Question 44: Gems and co. manufacturing a product for which the standard selling price has been
ascertained as below :

Materials – 2 units at `20 40


Labour – 20 hrs. @ `2.00 40
Variable overhead 8
Fixed overhead 20
Total cost 108
Profit 32
Selling 140

During the budget period , the company could produce and sell only 8,000 units as against budget of 10,000
units . The company’s profit and loss account is presented below :
Profit and loss account for the year ended

Particular ` Particular `
To materials (16,500 units ) 3,96,000 By sales (8,000 units ) 11,20,000
To wages (1,70,000 hours ) 3,46,800
To variable overhead 60,000
To fixed overhead 1,84,000
To net profit 1,33,200
11,20,000 11,20,000

4,000 hours were lost due to power failure. There was no opening or closing work-in-progress. You are to
Reconcile the actual profit with the standard profit , in term of variance. (May/92)

[Ans.: Standard profit (8,000 @ `32) 2,56,000


Actual profit 1,33,200 ]
Question 45: Ravi, Richard, Rahim and Roop Singh are regional salesmen distributing the product of Super
Perfumes Ltd. The selling price of the product is `400 per unit. The sales quota and the standard selling
expenses for the year are:-

Salesmen Sales quota Standard selling expenses


` `
Ravi 7,50,000 2,25,000
Richard 9,00,000 2,47,500
Rahim 11,50,000 2,87,500
Roop Singh 6,00,000 2,25,000

Actual data for the year were as follows : -

CA. Parag Gupta Ph.: +91 11 47665555 Paraggupta_ca@yahoo.co.in Costing & O.R.
World’s largest CA Final student’s consultancy group: http://groups.yahoo.com/group/costingbyparaggupta
Standard Costing & Variance Analysis - 104 -

Ravi Richard Rahim Roop Singh


Days on field work 200 175 225 250
Kilometres covered 20,000 18,000 18,000 30,000
` ` ` `
Sales 8,00,000 10,00,000 10,50,000 5,20,000
Salary 80,000 80,000 80,000 80,000
Free samples 9,000 7,500 5,375 8,000
Postage and stationery 8,000 9,000 10,000 6,000
Other expenses 9,000 5,000 4,000 10,000

The salesmen are allowed conveyance allowance of `1.50 per kilometre and a daily allowance of `80 per day
for the days spent on field work. Ravi gets a commission of 6 percent on sales and others are given a
commission of 5 percent on sales. Corporate sales office expenses are chargeable at the rate of `30 per unit
sold in the case of Ravi and Richard and `40 per unit in the case of Rahim and Roop Singh. Prepare a
schedule showing the selling cost variances by salesmen.

[Ans.:Particular Ravi Richard Rahim Roop singh

Selling cost variance 20,000(A) 7,500(F) 39,375(A) 52,000(A)


The total selling cost variance (adverse)=`1,03,875]
Question 46 [Equivalent production (of output)]: Goodwill Ltd. manufactures readymade shirts of a
specific quality in lots to each special order from its overseas customers.
The standard cost for one dozen of shirt are:

Direct material (24 metres @ `11) 264


Direct labour (3hour @`49) 147
Overheads (3hour @`40) 120
Standard cost per dozen 531

During July, 2004 it worked on three orders, for which the month’s job cost record as following:

Lots no Units Materials used Hours worked


45(UK) 1,700Doz 40,440 metres 5,130
46(US) 1,200Doz 28,825 metres 2,890
47(CAN) 1,000Doz 24,100 metres 2,980

Additional information:

(a) The company bought 95,000 metres of material during July at a cost of `10,64,000. The material price
variance is recorded when material are purchased .All inventories carries as cost.
(b) Direct labour during July the amounted to `5,50,000. The employees were paid at `50 per hour.
(c) Overhead during the month amounted to `4,56,000.
(d) A total of `57,60,000 was budgeted for overhead for the year 2003-04 ,based on estimated production of
the plants normal capacity of 48,000 dozen shirts annually .
Overhead at the level of production is 40% fixed and 60% variable. Overheads is applied on the basis of
direct labour hours.
(e) There was no work-in-progress at the beginning of July. During July, lot nos. 45and 47 were completed.
All material were issued for lots no. 46 which was 80% completed as regard conversion.

Required:

(A) Computation of standard cost of production of the shirts per dozen as well as in total for lot nos. 45, 46
and 47.
(B) Find out the variation in quantity of material used and labour hours worked of each lot as well as in total.
(C) Calculate the material price variance; labour rate variance; variable overhead efficiency variance and
fixed overhead volume variance. (19 Marks) May/94

CA. Parag Gupta Ph.: +91 11 47665555 Paraggupta_ca@yahoo.co.in Costing & O.R.
World’s largest CA Final student’s consultancy group: http://groups.yahoo.com/group/costingbyparaggupta
Cost Accounting & Management - 105 -

[Ans.: (A) `477.60 per dozen; (B) Total variation Material 235(F), Labour 20(A); (C) Material price variance
`19000(A); Labour rate variance `11000(A); variable overheads efficiency variance `480(A) and fixed
overheads volume variance `16320(A)]
Question 47: The standard cost sheet of a company based on the normal output of 30,000 units for a
quarter is as under:
(`)
Direct material (4Kg. @ 2 per Kg.) 8.00
Direct wages (6 hour @`4per hour) 24.00
Overhead (50%of direct wages) 12.00
Total cost 44.00
Profit 6.00
Selling price 50.00

The budgeted fixed overhead amount to `1,44,000 per quarter and it is included in the overhead cost included
above.

On the basis of the budgeted activity of 36,000 units, the company estimated the profit for the second of the
year as under :
(`)
Direct material 2,88,000
Direct wages 8,64,000
Overhead 4,32,000
Total cost 15,84,000
Sales 18,00,000
Profit 2,16,000

The cost records revealed the following actual for the second quarter of the year:
Production 25,000 units
Direct material consumed 96,000 Kg. at `2.25 per kg.
Direct wages paid 1,60,000 hours at `4.10 per hour. Out of which 6,000 hour being idea time were not
recorded on production.
Overhead `3,32,000 out of which `1,50,000 were fixed .
Sales 25,000 units at an average price of `51.50 per unit.

You are required to:

(i) Prepare a statement of actual Profit/Loss for the second quarter of the year.
(ii) Analyse the variances and present on operating statement reconciling the budgeted profit with actual
profit. (19 Marks) Nov./94

[Ans.: Budgeted Profit: `216000;SVMV: `66000(A); SPV: `37500(F); MPV: `24000(A), MUV: `8000(F); LEff
V: `16000; LRV: `16000; ITV: `24000; VOEV: `2800(F), VOEff V: `4800(A); FOEV: `6000(A); VOEffV:
`3200; FOCap.V: `20800; Actual Profit: `83500]
[Hint: In above question, Examiner has specifically provided us Standard rate in Standard Cost card but has
committed error while calculating Total Budgeted Overhead for Second quarter. Remember Budgeted Fixed
Overhead shall not vary with change in output, which examiner has erroneously done. So, while doing this
question we have 2 options:
1st: Assuming that out of Budgeted Overhead `432000, Budgeted Fixed Overhead are still `144000 & since
Std. Overhead rate p.u./hr. is same at 36000 units as that of 30000 units, Std. Variable/Fixed overhead p.u./hr
shall also be taken as same as that at Normal capacity of 30000 units. In other words there will be no change
in Std. rate.
2nd: Assuming that out of Budgeted Overhead `432000, Budgeted Fixed Overhead are still `144000, but
since budgeted units has changed from the time standard was set, hence although Standard Overhead
p.u./hr. is same but Std. Variable/Fixed overhead p.u. will get changed and will become
Std. Variable Overhead p.u. = (432000-144000)/36000 = `8 p.u.
Std. Variable Overhead p.u. = (432000-144000)/36000*6 = `1.33 p.h.
Std. Fixed Overhead p.u. = (144000)/36000 = `4 p.u.
Std. Fixed Overhead p.u. = (144000)/36000*6 = `0.67 p.h.

CA. Parag Gupta Ph.: +91 11 47665555 Paraggupta_ca@yahoo.co.in Costing & O.R.
World’s largest CA Final student’s consultancy group: http://groups.yahoo.com/group/costingbyparaggupta
Standard Costing & Variance Analysis - 106 -

Since, there is drafting error in question while making budget, hence you are free to follow any of the 2
assumptions, but 1st assumption was followed by Suggested answers of ICAI.]
Question 48: The overhead expense budget for a cost centre is as under:
Indirect material Re.0.40 per hour
Indirect labour Re.0.60 per hour
Maintenance Re. 0.40 per hour
Power Re. 0.30 per hour
Sundries Re. 0.30 per hour
Total variable expenses `2.00 per hour
Fixed overhead budgeted `240

Budgeted output = 9,600 units or 120 standard hours.


At the end of a period the actual rates given by the accounts department are as under:

Power Re.0.32; maintenance Re.0.45; indirect labour Re.0.60; indirect material Re.0.50 and sundry
expenses Re. 0.29 per hour; total variable expenses were `2.16 per hour. The actual output is 12,160
units for which the actual hours worked are 156. The fixed expenses amounted to `250. Compute the
variances.

[Ans.: VOCV: 33A; FOVV: 64(F); FOBV: 10(A); FOCV: 54(F)]


Question 49: The Standard Cost of producing one unit of Item ‘Q’ is as under:

Direct Material -- A – 12 Kg. @ `10/- = `120


B – 5 kg. @ `6/- = `30
Direct Wages -- 5 hrs. @ `3/- = `15
Fixed Production = `35
Overheads
Total Standard Cost: = `200
Standard Gross Profit = `50
Standard Sale Price = `250

Fixed Production overhead is absorbed on expected annual output of 13,200 units.

Actual result for the month of September, 1997 are under:


Actual Production: 1,000 units
`
Sales 1,000 Units @ `250 = 2,50,000
Direct Material -- A – 11,000 kg. = 1,21,000
B – 5,200 kg. = 28,600
Direct wages 5,500 hrs. = 17,500
Fixed Overheads = 39,000
= 2,06,100
Gross profit = 43,900
You are required to calculate all variances. Material price variance is taken out at the time of receipt of
Material. Material purchases were:
12,000 kg. ‘A’ @ `11 & 5,000 kg. of ‘B’ @ `5.50 (12 Marks Nov./97

[Ans.: Material price variance `9500(A); Material cost variance `9500(F); Material usage variance `8800(F);
Material mix variance `1741.18(F); Material Yield variance `7052.82; Material purchase price variance
`9500(A); Labour cost variance `2500(A); Labour efficiency variable `1500(A); Labour Price Variance
1000(A); Fixed Overhead Expenditure variance `500(A); Efficiency variable `3500(A); Volume variance
`3500(A); Total fixed overhead cost variance `4000(A)]
Question 50: The following information is available in respect of Y Ltd. for a week:

(a) 400 kg of raw material were actually used in producing product ‘EXE’. The purchase cost thereof
being `24,800. The standard price per kg of raw material is `60. The expected output is 12 units of

CA. Parag Gupta Ph.: +91 11 47665555 Paraggupta_ca@yahoo.co.in Costing & O.R.
World’s largest CA Final student’s consultancy group: http://groups.yahoo.com/group/costingbyparaggupta
Cost Accounting & Management - 107 -

product ‘EXE’ from each kg of raw material. Raw material price variance and usage variance as
computed by cost accountant are `800 (adverse) and `600 (adverse) respectively.
(b) The week is of 40 hours. The standard time to produce one unit of ‘EXE’ is 30 minutes. The standard
wage rate is `5 per labour hour. The company employs 60 workers who have been paid hourly wage
rate as under:
Number of workers 6 8 46
Hourly Wage Rate (`) 4.80 5.20 5.00
(c) Budgeted overheads for a four-weekly period is `81,600. The actual fixed overheads spent during the
said week are `19,800.
(d) Entire output of ‘EXE’ has been sold at its standard selling price of `15 per unit.

You are required to:

(i) Compute the variances relating to labour and overheads.


(ii) Prepare a statement showing total standard costs, standard profit, and actual profit for the week.

[Ans.: Sales `70200; Standard profit `15210; Actual profit `13584] (19 Marks) May/97
Question 51: Stand cost Corporation produces three products A,B and C. The master budget called for
the sale of 10,000 units of A at `12 6,000 units of B at `15 and 8,000 units of C at `9. In addition, the
standard variable cost for each product was `7 for A, `9 for B and `6 for C. Infact, the firm actually
produced and sold 11,000 units of A at `11.50, 5,000 units of B at `15.10 and 9,000 units of C at `8.55.

The firm uses two input to produce each of the products X and Y. The standard price of material X is
`2 and for a unit of material Y is Re. 1. The materials budgeted to be used for each product were:
Products Materials

X Y
(units) (units)
A 2 3
B 4 1
C 1 4

The firm actually used 54,000 units of X at a cost of `1,09,620 and 72,000 units of Y at a cost of
`73,000.

Required:

Determine the mix, quantity and rate variances for sales as well as the yield, mix and price variance for
materials.

[Ans.: Sales mix variance `5750(A); Sales quantity variance `11750(F), Sales rate variance `9050(A);
Sales margin mix variance `2583(A); Sales margin quantity variance `9050(A), Sales margin rate
variance `7050(A); Material Yield variance `1408(A); Material mix variance `2592(A); Material Price
variance `2620(A)]
[Note: Since question is silent that whether sales variances are supposed to calculated on the basis of value
or margin, we can calculate it on either basis]
Question 52: The working results of a company for two corresponding years are shown below:
Year 1 Year 2
`in lakhs `in lakhs

Sales 1,200 1,540


Direct Material 600 648
Direct Wages and Variable Overheads 360 412
Fixed Overheads 160 300

CA. Parag Gupta Ph.: +91 11 47665555 Paraggupta_ca@yahoo.co.in Costing & O.R.
World’s largest CA Final student’s consultancy group: http://groups.yahoo.com/group/costingbyparaggupta
Standard Costing & Variance Analysis - 108 -

1,120 1,360
Profit 80 180

In year 2, there has been an increase in the selling price by 10%. Following are the details of material
consumption and utilization of direct labour hours during the two years.

Year 1 Year 2
Direct Material Consumption in m/t 5,00,000 5,40,000
Direct Labour Hours 75,00,000 80,00,000

You are required to :

(i) Keeping year 1 as base year, analyse the results of year 2 and work out the amount which each factor
has contributed to change in profit.
(ii) Find out the break even sales for both years.
(iii) Calculate the percentage increase in selling price that would be needed over the sale value of year 2 to
earn a margin of safety of 45%. (13 Marks) May/00

[Ans.: (ii) BES: Yr. 1- `800 Lacs; Yr. 2- `962.50 Lacs (iii) 4.25%]
Question 53[Equivalent Production]: Standard Cost Card of a product is as under:
(`)
Direct Materials
A 2kg @ `3 per kg 6.00
B 1kg @ `4 per kg 4.00
Direct Wages 5 hours @ `4 per hour 20.00
Variable overheads 5 hours @ `1 per hour 5.00
Fixed overheads 5 hours @ `2 per hour 10.00
Total 45.00
Standard Profit 5.00
Standard Selling Price 50.00

Budgeted output is 8,000 per month.

In October 2004, the company produced and sold 6,000 units. The actual sales value was `3,05,000. Direct
materials consumed was: Material A 14,850 kg valued at `43,065 and Material B 7,260 kg valued at `29,750.
The total direct labour hours paid were 32,000 and wages paid therefore amounted to `1,27,500. The direct
labour hours actually booked on production was 31,800. Overheads recorded were: Fixed `80,600 and
Variable `30,000. Closing work-in-progress was 600 units in respect of which metals A and B were fully
issued and labour and overheads were 50% complete.

Analyse the variances and present an operating statement showing the reconciliation between budgeted and
actual profit for the month in the following format.

Operating Statement (`)


Budgeted Profit
Sales Variance
Price
Volume
Total
Cost Variance
Direct Materials
Price
Yield
Mix
Direct Wages
Rate
CA. Parag Gupta Ph.: +91 11 47665555 Paraggupta_ca@yahoo.co.in Costing & O.R.
World’s largest CA Final student’s consultancy group: http://groups.yahoo.com/group/costingbyparaggupta
Cost Accounting & Management - 109 -

Efficiency
Idle time
Variable Overheads
Expenses
Efficiency
Fixed Overheads
Expenses
Efficiency
Idle time
Capacity
Total Cost Variance
Actual Profit

[Ans.: Actual Profit-`10585, Budgeted Profit-`40000; Equivalent units: DM-6600 & DL-6300]
[Note: If we are specifically asked to calculate Fixed Overhead Idle Time Variance then we will calculate
Capacity Variance at Actual Hours paid in spite of worked. All other variance will remain same.]
Question 54: The Standard cost card per unit of output of Product K produced by a firm is as under:
(`)
Direct Material A. (10kg @ `10) 100
B. (5 kg @ `3) 15
Direct wages (5 hrs. @ `3) 15
Fixed production overheads 25
Total Standard Cost 155
Standard Gross Profit 45
Standard Selling Price 200

The fixed production overheads has been absorbed on the expected annual output of 10,800 units produced
at a an even flow throughout the year.
During the month of November 2004, the following were the actual results for an actual production of 800
units
(`)
Sales 800 units @ `200 1,60,000
Direct Material A 7,800 kg 79,950
B 4,300 kg 11,825
Direct wages 4,200 hours 12,075
Fixed overheads 23,500
Total 1,27,350
Gross profit 32,650

The material price variance is extracted at the time of receipt of materials. Materials purchased were A.
9,000kg @ `10.25 per kg; B 5,000 kg @ `2.75 per kg.

Required:
(i) Calculate all variances.
(ii) Prepare an operating statement showing the standard gross profit, variances and the actual gross
profit.
(iii) Explain the reason for the difference in the actual gross profit given above and the actual gross profit
arrived at by you.

[Ans.: Sales-`160000; Std. Cost of Sales-`124000; Actual Profit-`32525; Actual Profit (On Issue Basis)-
`32650] ICWA-Dec/89 & (12 Marks) May/10-N.C.[Adapted]
Question 55: A small company, making a single product, produces accounts for a costing period, as
following:
(`)
Direct Material 792
Direct Wages 1,192
Variable Overhead 1,940
Fixed Overhead 1,040 4,964
CA. Parag Gupta Ph.: +91 11 47665555 Paraggupta_ca@yahoo.co.in Costing & O.R.
World’s largest CA Final student’s consultancy group: http://groups.yahoo.com/group/costingbyparaggupta
Standard Costing & Variance Analysis - 110 -

Profit 976
Sales 5,940

The original budget was in respect of 1,000 per period, but during this period only 960 units were produced
and sold.

Standard direct wages rate is `0.60 per hour and standard variable overhead rate is `1 per hour.
Cost variances during the period are as follows:
(`)
Particulars Gains Losses
Material Price - 8
Material Usage - 16
Wage rate 20 -
Labour Efficiency - 60
Variable Overhead Expenditure 80 -
Variable Overhead Efficiency - 100
Fixed Overhead Cost - 40
Sales Price 180 -

From the above information, prepare for the period the original budget and a flexible budget for the sales
achieved.

[Ans.: Sales: Std. Cost p.u.: `6; Original Budget (1000 units): 6000; Flexible Budget (960 units): 5760]
Question 56: The accountant of a company has presented the following operating statement to the General
Manager of Department ‘P’ for the month of May 2004.
(`)
Particulars Budget Actual Variance
May 2004
Sales 24,00,000 22,00,000 2,00,000
Direct Materials 6,00,000 5,20,000 80,000
Direct Labour 8,00,000 7,56,000 44,000
Factory Overheads (V) 2,00,000 1,84,000 16,000
Factory Overheads (F) 1,00,000 1,16,000 (16,000)
Selling Overheads (V) 3,00,000 2,88,000 12,000
Selling Overheads (F) 2,00,000 1,84,000 1,60,000
Total 22,00,000 20,48,000 1,52,000
Profit 2,00,000 1,52,000 (48,000)
Direct Labour Hours 1,00,000 95,000
Units of Production and Sale 20,000 18,000

The general manager was surprised to see that his operation have resulted in the adverse profit variance of
`48,000 for the month. On the basis of the budgeted profit of `10 per unit, he expected that he would make a
profit of `1,80,000 on a sale of 18,000 units of production in May 2004 instead of the budgeted profit of
`2,00,000 resulting in an adverse profit variance of `20,000 only.

You are required to :

(a) Redraft the above statement to show the original budget, flexible budget, actual expenses incurred
and variations for May 2004.
(b) Calculate all variance relating to sales, direct material, direct labour and overheads.
[Ans.: Net Profit- Original Budget: 200000; Flexible Budget: 150000; Actual: 152000]
Question 57: The following information is available from the record of Sunrise Ltd. which produces only one
product:
Budgeted Income Statement: January 1995
` ` `
Sales Revenue: (20,000 units at `5) 1,00,000
Production Costs:

CA. Parag Gupta Ph.: +91 11 47665555 Paraggupta_ca@yahoo.co.in Costing & O.R.
World’s largest CA Final student’s consultancy group: http://groups.yahoo.com/group/costingbyparaggupta
Cost Accounting & Management - 111 -

Budgeted production 20,000 units


Direct Materials:
A (10,000 Kg. @ `0.30) 3,000
B (10,000 Kg. @ `0.70) 7,000 10,000
Direct Labour
Skilled (9,000 Hrs @ `3.00) 27,000
Un-skilled (5,200 Hrs @ `2.50) 13,000 40,000
Production Overhead:
Fixed 20,000
Variable (20,000 units @ `0.50) 10,000
80,000
Add: Opening Stock (1,000 units @ `4.00) 4,000
84,000
Deduct Closing Stock (1,000 units @ `4.00) 4,000
80,000
Budgeted Profit 20,000

During January 1995 production and sales were both above budget and the following income statement was
prepared:
Income Statement: January 1995
` ` `
Sales Revenue:
(14,000 units at `5) 70,000
(8,000 units at `4.75) 38,000
1,08,000
Production Costs:
Actual production 24,000 units
Direct Materials:
A (16,000 Kg. @ `0.20) 3,200
B (10,000 Kg. @ `0.80) 8,000 11,200
Direct Labour
Skilled (13,000 Hrs @ `2.95) 38,350
Un-skilled ( 6,300 Hrs @ `2.60) 16,380 54,730
Production Overhead:
Fixed 18,020
Variable (24,000 units @ `0.625) 15,000
98,950
Add: Opening Stock (1,000 units @ `4.00) 4,000
1,02,950
Less: Closing Stock (3,000 units @ `4.00) 12,000
90,950
Actual Profit 17,050

During the period 1,000 abnormal idle hours for skilled labour due to machine breakdown was reported.
In the above statement stock is valued at standard cost of `4 per unit.

Required:

Prepare a standard costing statement analyzing the differences between the budget and the actual
performance. In your analysis include calculations of the sales volume and sales price variances; direct
material price, mix, yield and usage variances; direct labour rate, idle time and efficiency variances; overhead
expenditure and volume variances. (19 Marks) Nov./95

[Ans.: Sales volume margin variance `2000(F); Sales margin price variance `2000(A); Material price
variance `600(F); Material mix variance `1200(F); Material yield variance `1000(A); Labour rate variance
`20(F); Labour Efficiency variance `3750(A); Idle time variance `3000(A); Variable overheads exp. Variance
`3000(A); Fixed overheads Exp. Variance `1980(F); Fixed overhead volume variance `4000(F)]

CA. Parag Gupta Ph.: +91 11 47665555 Paraggupta_ca@yahoo.co.in Costing & O.R.
World’s largest CA Final student’s consultancy group: http://groups.yahoo.com/group/costingbyparaggupta
Standard Costing & Variance Analysis - 112 -

Question 58: GLOBAL LTD. is engaged in marketing of wide range of consumer goods. A, B, C and D
are the zonal sales officers for four zones. The company fixes annual sales target for them individually.
You are furnished with the following:

(1) The standard costs of sales target in respect of A, B, C and D are `5,00,000, `3,75,000, `4,00,000
and `4,25,000 respectively.
(2) A, B, C and D respectively earned `29,900, `23,500, `24,500 and `25,800 as commission at 5% on
actual sales effected by them during the previous year.
(3) The relevant variances as computed by a qualified cost accountant are as follows:

A B C D
` ` ` `
Sales price variance 4,000 (F) 6,000 (A) 5,000 (A) 2,000 (A)
Sales volume variance 6,000 (A) 26,000 (F) 15,000 (F) 8,000 (F)
Sales margin mix variance 14,000 (A) 8,000 (F) 17,000 (F) 3,000 (A)
(A) = Adverse variance and (F) = Favourable variance.

You are required to:


1) Compute the amount of sales target fixed and the actual amount of contribution earned in case of
each of the zonal sales officer.
2) Evaluate the overall performance of these zonal sales officers taking three relevant base factors and
then recommend whose performance is the best. (13 Marks) Nov./96

[Ans.: Actual Margin/Sales-15.05; 16/38; 18.78; 15.50; Rank: IV; II; I; III]
[Hint: In part 2) Actual margin is to be calculated instead of actual contribution, because actual cost is not
given.]
[Note: Suggested Answers of ICAI has assumed that sales margin quantity variance is Nil]

Behavioural aspects of Standard Costing


1. Projection of fixed overheads and estimated selling price in a Standard Cost Sheet is a circular exercise
with no added value.
In an award winning article,” COST / MANAGEMENT ACCOUNTING: THE 21ST CENTURY PARADIGM”,
published in Management Accounting (USA), December 1995, William L Ferrara argues that while preparing
a Standard Cost Sheet, one of the objectives of which is to assist management in pricing products, a
professional cannot project fixed overheads until and unless he is aware of the production quantum to be
effected. The forecast of future production can only be made if a tentative selling price of the product is known
because, in a competitive market, it is the selling price which decides the sale quantity and therefore the
production volume. The authors contend that in case the selling price is known at the time of projecting fixed
overheads then the re computation of the same is a valueless exercise.
2. Traditional costing tools like standard costing induce a static behaviour in the employees.
During the past decade and a half, various writers such as Johnson and Kaplan, Ferrara and Monden etc
have questioned the productivity and use of traditional systems such as standard costing and variance
analysis. They argue that the use of standard costing renders employees static and curbs innovation and that
companies following traditional standard costing find it difficult to improve upon standards because of severe
resistance from employees who are convinced that the established best practise cannot be improved further.
3. Fear of adverse variances forces managers to give undue importance to material price, labour rate and
efficiency and capacity utilisation. These concepts are detrimental to the modern day world class
manufacturing environment characterized by concepts of JIT and TQM.
In a World Class Manufacturing environment, characterised by Just in Time policies, the focus of the
management is to produce only as much as is required. This requires purchase of small quantities of raw
material, increase in the number of set ups and minimal importance to capacity utilisation. Policies like this
result in increased adverse variances related to raw material prices, labour efficiency and production volume.
Critics argue that the fear of such adverse variances affects goal congruence and forces managers to behave
against their company’s policies.
4. Traditional costing does not provide the management with what is the allowable cost; rather it emphasises
on the standard or actual costs.
This is looked upon as one of the major reasons for lack of innovation especially in the global era where
competition amongst companies is unprecedented. It is argued that techniques like Target costing are much
CA. Parag Gupta Ph.: +91 11 47665555 Paraggupta_ca@yahoo.co.in Costing & O.R.
World’s largest CA Final student’s consultancy group: http://groups.yahoo.com/group/costingbyparaggupta
Cost Accounting & Management - 113 -

more motivating when compared to Traditional costing since the former encourage the use of concepts like
value engineering and value analysis.
Question 59: Describe the role of cost accountant in the area of operating efficiency. (4 Marks) May/97

Ans.: Role of Cost Accountant in the area of operating efficiency: Operating efficiency relates to the efficient
use of various resources, viz material, labour, machines etc. The cost accountant can contribute towards
efficient and effective use of man, material and machine by introducing control and accounting procedures.
In the area of labour efficiency, the role of cost accountant includes reporting efficiency variances and
evaluating wage incentive schemes. In the case of material, efficiency can be increased by introducing
effective material control and accounting procedures and computing material usage variance. In the area of
machine efficiency, his objective would be to ensure optimum utilization of machine time. He may also give
his advice based on his information and analysis, to the management, towards the replacement of old
machine with the latest one. The efficiency of operating procedures will be monitored by industrial engineers
and organization & methods specialists but the cost accountant may be required to calculate the net savings
to be expected from the procedural change.

Question 60: Explain the problems concerning control of operations that a manufacturing company can be
expected to experience in using a standard costing system during periods of rapid inflation. (4 Marks) May/96

Ans.: The problems concerning control of operations that a manufacturing company can be expected to
experience in using a standard costing system during periods of rapid inflation are as follows:

(i) The formulation/setting of material standards makes assumptions about the inflation, which will
prevail in future, if this assumption is not stated clearly then it is difficult to determine how much of price
variance is due to inflation and how much is due to buying efficiency.
(ii) Price indices tend to reflect average price changes. Consequently, it is difficult for a company to
predict future costs and interpret variances if the specific rate of inflation for its inputs is considerably different
from the general rate of inflation.
(iii) Inflation may result in relative changes in the prices of inputs. Therefore, standard mixes requiring
different inputs may no longer be the most efficient mix.
(iv) If standard prices are not adjusted then the efficiency variances will be understated.
(v) Sharp rises in prices will raise questions as to whether unadjusted standards can be used in the
decision making process (e.g. Pricing decisions).
(vi) Administrative work involved in maintaining upto-date standards when prices are constantly changing
will increase.
Question 61: “Overhead variances should be viewed as interdependent rather than independent”.
Explain.

Ans.: The operations of a firm are so inter linked that the level of performance in one area of operation will
affect the performance in other areas. Improvements in one area may lead to improvements in other areas. A
sub-standard performance in one area may be compensated by a favourable performance in another area.
Because of such interdependency among activities in the firm, the managers should not jump to conclusions
merely based on the label of variances namely favourable or unfavourable. They should remember that
there is a room for trade off amongst variances. Hence, variances need to be viewed as ‘attention
directors’ rather than problem solvers. Thus, a better picture will be captured when overhead variance are not
viewed in isolation but in an integrated manner. (4 Marks) May/02

Question 62: What are the basic differences between Standard Costing and Budgetary Control?
(7 Marks) Nov/97
Ans.: Basic differences between Standard Costing and Budgetary Control are as follows:
(i) Standard costs are ascertained for material labour and overheads Here the control of each elements
of cost is effected by comparing actual costs with standard costs of actual output. Whereas budgets are
prepared for different functions like sales, production capital assets etc. of business. Budgetary control here is
concerned with the overall profitability and financial position of the business.
(ii) Range of standard costing is narrow as it is mainly confined to the control of production costs. But the
range of budgeting is wider than that of standard costing. It in fact covers sales, capital and financial
expenses as well.

CA. Parag Gupta Ph.: +91 11 47665555 Paraggupta_ca@yahoo.co.in Costing & O.R.
World’s largest CA Final student’s consultancy group: http://groups.yahoo.com/group/costingbyparaggupta
Standard Costing & Variance Analysis - 114 -

(iii) Standard costing is confined of the projection of cost accounts only whereas budgetary control
includes projection of financial accounts as well.
(iv) For exercising control, variances are computed in standard costing as well as budgetary control. But
these variances are normally recorded in different cost accounts under standard costing whereas they are not
revealed under budgets.
(v) Under standard costing various causes of variances in respect of each cost element can be analysed
in minute detail and corrective action taken accordingly.
Whereas budgetary control system deals with total expenses and revenues based on estimates.

Question 63: “Calculation of variances in standard costing is not an end in itself, but a means to an end.”
Discuss. (7 Marks) May/99

Ans.: The crux of standard costing lies in variance analysis. Standard costing is the technique whereby
standard costs are predetermined and subsequently compared with the recorded actual costs. It is a
technique of cost ascertainment and cost control. It establishes predetermined estimates of the cost of
products and services based on management’s standards of efficient operation. It thus lays emphasis on
“what the cost should be”. These should be costs are when compared with the actual costs. The difference
between standard cost and actual cost of actual output is defined as the variance.

The variance in other words in the difference between the actual performance and the standard performance.
The calculations of variances are simple. A variance may be favourable or unfavourable. If the actual cost is
less than the standard cost, the variance is favourarable but if the actual cost is more than the standard cost,
the variance will be unfavourable. They are easily expressible and do not provide detailed analysis to enable
management of exercise control over them. It is not enough to know the figures of these variances from
month to month. We infact are required to trace their origin and causes of occurrence for taking necessary
remedial steps to reduce / eliminate them.

A detailed probe into the variance particularly the controllable variances helps the management to ascertain:
(i) the amount of variance
(ii) the factors or causes of their occurrence
(iii) the responsibility to be laid on executives and departments and
(iv) corrective actions which should be taken to obviate or reduce the variances.

Mere calculation and analysis of variances is of no use. The success of variance analysis depends upon how
quickly and effectively the corrective actions can be taken on the analysed variances. In fact variance gives
information. The manager needs to act on the information provided for taking corrective action. Information is
the means and action taken on it is the end. In other words, the calculation of variances in standard costing is
not an end in itself, but a means to an end.

Investigation of variances
When deciding which variances to investigate, the following factors should be considered:
 Reliability and accuracy of the figures: Mistakes in calculating budget figures, or in recording actual
costs and revenues, could lead to a variance being reported where no problem actually exists (the
process is actually ‘in control’).
 Materiality: The size of the variance may indicate the scale of the problem and the potential benefits
arising from its correction.
 Possible interdependencies of variances: Sometimes a variance in one area is related to a variance
in another. For example, a favourable raw material price variance resulting from the purchase of a
lower grade of material, may cause an adverse labour efficiency variance because the lower grade
material is harder to work with. These two variances would need to be considered jointly before
making an investigation decision.
 The inherent variability of the cost or revenue: Some costs, by nature, are quite volatile (oil prices, for
example) and variances would therefore not be surprising. Other costs, such as labour rates, are far
more stable and even a small variance may indicate a problem.
 Adverse or favourable? Adverse variances tend to attract most attention as they indicate problems.
However, there is an argument for the investigation of favourable variances so that a business can
learn from its successes.
CA. Parag Gupta Ph.: +91 11 47665555 Paraggupta_ca@yahoo.co.in Costing & O.R.
World’s largest CA Final student’s consultancy group: http://groups.yahoo.com/group/costingbyparaggupta
Cost Accounting & Management - 115 -

 Trends in variances: One adverse variance may be caused by a random event. A series of adverse
variances usually indicates that a process is out of control.
 Controllability/probability of correction: If a cost or revenue is outside the manager’s control (such as
the world market price of a raw material) then there is little point in investigating its cause.
 Costs and benefits of correction If the cost of correcting the problem is likely to be higher than the
benefit, then there is little point in investigating further.

Variance investigation models can be classified into the following categories:


 Simple rule of thumb model: Under this approach, company managers uses simple models based on
arbitrary criteria such as percentage of standard cost approach. Any variance larger than a fixed
percentage of standard cost is investigated. It is a simple system to operate, but it ignores many of
the issues listed above. For example, it takes no account of the reliability of the figures, possible
interdependencies of variances, the inherent variability of the costs involved, recent trends in
variances, or the costs and potential benefits of investigation.
 Statistical models that do not incorporate costs and benefits of investigation.
 Statistical decision models that take into account the cost and benefits of investigation.

Investigation will indicate that variance is due to:


 Random uncontrollable factors when the operation is under control.
 Assignable causes, but the cost of investigation exceeds benefits.
 Assignable causes, but the benefits of investigation exceed the cost.

Note : The aim is to investigate only those variances in the third category.

Statistical investigation models not incorporating cost and benefits


When deciding whether to investigate a variance it is important to distinguish between variances caused by
normal random cost variation (those that occur even though the process is ‘in control’), and those caused by
genuine problems (often referred to as the process being ‘out of control’).
Some costs are inherently variable, and standards can be viewed as representing an average cost. Small
variances (adverse or favourable) can often be accounted for by the normal random variation that occurs
around this average. Larger variances are less likely to be explained by normal random variation, and are
more likely to suggest that the process is out of control.

Control charts provide a visual representation of the variation of actual costs around standard.

1. Assume actual observations when under control indicate a mean usage of 10 kg per unit with a SD of
1 kg (normally distributed).
2. Actual usage is 26000 kg for an output of 2000 units.
Therefore, average usage = 13 kg per unit.
3. Z = Actual usage (13 kg) – Expected usage (10 kg) = 3.0
SD (1 kg)
4. Normal distribution table indicates that an observation 3 SDs from the mean has a probability of
0.13% (50%-49.87%).

CA. Parag Gupta Ph.: +91 11 47665555 Paraggupta_ca@yahoo.co.in Costing & O.R.
World’s largest CA Final student’s consultancy group: http://groups.yahoo.com/group/costingbyparaggupta
Standard Costing & Variance Analysis - 116 -

5. Thus the probability of actual average material usage per unit of output being 13 kg or more when
the operation is under control is 0.13%. It is very unlikely that material usage comes from ‘in
control distribution’.
6. Statistical control charts, which rely on the above principles, can be used to monitor resources usage
and the probability that operations are out of control.

Statistical quality control charts


Figure above shows a variance control chart. Actual costs are plotted on the diagram as they occur, given as
percentages of standard cost. As long as the actual cost percentage remains within the limits no action is
taken. These small variations from standard are assumed to be due to normal random variation. If actual
costs move outside the limits, this indicates the need for corrective action.
Limits can be set on the basis of experience, or on the basis of a standard normal distribution, a technique
that is explained in “Test of Hypothesis”
A control chart provides a useful visual representation of data, and helps isolate normal random
(uncontrollable) variation in cost. It also gives a clear picture of trends in variances. However, a control chart
is only really useful for costs where an average can be established, and its use is usually restricted to
efficiency rather than expenditure variances.

Statistical decision models that take into account the cost and benefits of investigation.

1. If the process is out of control there is a benefit (B) associated with returning it to it is in - control state
(i.e. cost savings from avoiding variances in future periods). Say B = `400.
2. Let C = cost of investigation (Say C = `100).
3. Let P = probability that the process is out of control.
4. Expected benefit = PB
5. Investigate if PB > C, or P > C/B
6. P >100 /400 =0.25
7. Probability (Process is in control) = 0.0013 (from example in control charts)
8. P =1 – 0.0013 = 0.9987
9. Decision = Investigate the variance

Remember, Variances should only be investigated if the expected value of the resulting benefits exceeds the
costs of investigation.

Illustration 1
The cost accountant for X plc has reported a large adverse material usage variance for the previous month.
An initial investigation has shown that a faulty machine is causing the variance.
X plc is trying to decide whether to close down the production line for one day to allow engineers to perform a
temporary repair, at an incremental cost of `200,000. Past experience suggests that there is a 60% chance of
the temporary repair correcting the fault. If the fault is not corrected now it is estimated that an extra material
cost of `400,000 will be incurred before the fault is permanently corrected by scheduled maintenance work,
due to take place in three-months’ time.
Solution:
Probability of the process being out of control, P = 0.60
Cost savings, B = `400,000
Incremental cost of correction, C= `200,000
C/B = 2,00,000/4,00,000 = 0.50
Investigation is desirable when the probability of being out of control exceeds 0.50
Since P>C/B hence investigation is advisable.
A probability-based model appears very sensible but its major drawback is the need for accurate estimates of
probabilities. In above example, it could be questioned how the firm estimated the 60% chance of success.
CONCLUSION
Variances are calculated to assist managers in the control of a business. Effective decisions on whether to
investigate variances are crucial for successful control.

Illustration 2
A company using a detailed system of standard costing finds that the cost of investigation of variances is
`20,000. If after investigation an out of control situation is discovered, the cost of correction is `30,000. If no
investigation is made, the present value of extra cost involved is `1,50,000. The probability of the process

CA. Parag Gupta Ph.: +91 11 47665555 Paraggupta_ca@yahoo.co.in Costing & O.R.
World’s largest CA Final student’s consultancy group: http://groups.yahoo.com/group/costingbyparaggupta
Cost Accounting & Management - 117 -

being in control is 0.82 and the probability of the process being out of control is 0.18. You are required to
advise.
(i) Whether investigation of the variances should be undertaken or not;
(ii) The probability at which it is desirable to institute investigation into variances. (ICWA-June/87)
Solution:
Cost of investigation, C = `20,000
Cost of Correction, M = `30,000
Extra cost if no investigation, L = `1,50,000
Benefit of investigation, B = L – M = `1,20,000
Probability of the process being out of control, P = 0.18
C/B = 20,000/1,20,000 = 0.167
Investigation is desirable when the probability of being out of control exceeds 0.167
Since P>C/B hence investigation is advisable.

Illustration 3
A machine produces 1,00,000 standard components per day at a cost of `1.50 per unit. If the process if in
control, on an average 3% of the output is defective. If the process is out of control, the rate of defective will
be 5%. The entire cost of a defective unit is a loss.
The cost of carrying out an investigation is `600. If the process is found to be out of control after an
investigation then it costs another `400 to rectify the error. The probability of the process being in control is
0.7. (ICWA-Dec./98)
Solution:
Cost of investigation, C = `600
Cost of Correction, M = `400
Extra cost if no investigation, L = 1,00,000 × (5-3)% × 1.50 = `3,000
Benefit of investigation, B = L – M = `2,600
Probability of the process being out of control, P = 0.3
C/B = 600/2,600 = 0.231
Investigation is desirable when the probability of being out of control exceeds 0.231
Since P>C/B hence investigation is advisable.

BALANCE SCORECARD

Balanced Score Card:


Balanced Score Card is a set of financial and non-financial measures relating to a company’s critical success
factors.
It is an approach advocated by Kaplan & Norton, which provides information to management to assist in
strategic policy formulation and achievement.
The main objective of Balanced Score Card is to provide a comprehensive framework for translating a firm’s
strategic objectives into a coherent set of performance measures.

Components of a balanced score card:


A well designed balanced score card combines financial measures of past performance with measures of
firm’s drivers of future performance.
The specific objectives and measures of an organisation’s balanced score card are derived from the firm’s
vision and strategy.
There are a number of “balances” in the BSC, among which are the balance or equilibrium between four
historical domains or perspectives considered to be mutually linked in terms of strategy and performance:

1. Customer perspective i.e how customers see us? In order to translate effective internal processes
into organization success, customers/clients must be happy with the service they receive. The
Customer perspective considers the business through the eyes of the customers, measuring and
reflecting upon customer satisfaction.

CA. Parag Gupta Ph.: +91 11 47665555 Paraggupta_ca@yahoo.co.in Costing & O.R.
World’s largest CA Final student’s consultancy group: http://groups.yahoo.com/group/costingbyparaggupta
Standard Costing & Variance Analysis - 118 -

2. Internal perspective i.e. in what must the organization excel? The internal perspective focuses
attention on the performance of the key internal processes, which drive the business. The nature of
the processes is dependent on the nature of the organization.
3. Innovation and learning perspective, i.e Can we continue to improve and create value? The value
& Growth perspective is a measure of potential future performance – it direct attention to the basis of
all future success – the organization people and infrastructure. Adequate investment in these areas is
critical to all long-term success.
4. Financial, perspective i. e. How we look to our shareholders? The financial perspective measures
the results that the organisation delivers to it shareholders.

Thus, the scoreboard provides a view of an organization’s overall performance by integrating financial
measures with other key performance indicators. All these four perspective provide a balanced view of the
present and future performance of the business.

Paul Niven’s analogy of the Balanced Scorecard is that of a tree. The Learning and Growth perspective are
the roots, the trunk is the Internal Process perspective, Customers are the branches, and the leaves are the
Financial perspective. Each perspective is interdependent on those below as well as those above. It is a
continuous cycle of renewal and growth. Leaves (finances) fall to fertilize the ground and root system, which
stimulates growth throughout the organization. In this analogy, learning and growth is the foundation on which
all other perspectives are built. For example, if a hospital assesses patient satisfaction and discovers patients
aren’t satisfied (Customer Perspective), one of the strategies might be the implementation of employee
training in the area of customer service (Learning & Growth Perspective). Improved customer service through
a reduction of wait time in the emergency room (Internal Process Perspective) can ultimately improve
utilization (Financial Perspective). Refer to below-mentioned figure. There are definite cause and effects
between and among each of the four perspectives. The key is to identify the right strategies.

Employee training leads to Improved Emergency


in customer service Room Wait Times

leads to

Increased Patients leads to Increased utilization


Satisfaction

CA. Parag Gupta Ph.: +91 11 47665555 Paraggupta_ca@yahoo.co.in Costing & O.R.
World’s largest CA Final student’s consultancy group: http://groups.yahoo.com/group/costingbyparaggupta
Cost Accounting & Management - 119 -

Source: http://www.ruralcenter.org/?id=res_bsc

One of the reasons the Balanced Scorecard has been so successful is that it is a balanced approach. This
balance includes:
1. Balance between financial and non-financial indicators of success
2. Balance between internal and external constituents of the organization
3. Balance between lag and lead indicators of performance.
Internal constituents might include employees whereas external constituents might include physician groups
or insurers. Lag indicators generally represent past performance and might include customer satisfaction or
revenue. Although these measures are objective and accessible, they lack any predictive power. Lead
indicators are the performance drivers that lead to the achievement of lag indicators and often include the
measurement of processes and activities. For example, Emergency Room wait time might represent a leading
indicator of patient satisfaction. A Balanced Scorecard should contain a variety of different measures.

Process of creating a Balanced Score Card:


Step Description
1. Identify the vision i.e. where an organization is going. For example,the vision of a company
may be to dominate the market.
2. Identify the organisation’s strategies i.e. how an organization is planning to go there. For
example, strategy may be to focus on cost efficiency, high quality and fresh investment in
new technology.
3. Define critical success factors and perspective i.e. what we have to do well in each
perspective (see Note Below for illustration of perspectives and performance measures)
4. Identify measures, which will ensure that every thing is going in the expected way.
5. Evaluation of Balanced score card i.e. ensuring what we are measuring is right
6. Create action plans and plan reporting of the Balanced Score Card
7. Follow–up and manage i.e. which person should have reports and how reports should look
like

A. Innovation and Learning Perspective


Goals Performance
Technology leadership Product performance compared to competitors, number of new
products with patented technology.
Cost leadership Manufacturing overheads per quarter as a percentage of sales rate
of decrease in cost of quality per quarter
Market leadership Market share in all major markets
Research and development Number of new products, numbers of patents
B. Internal business Perspective
Goals Performance
Efficiency of manufacturing Manufacturing cycle time
process
Sales penetration Annual sales vs. Plan sales, increase in number of customers in a
unit of time.
New Product introduction Rate of new product introduction/quarter
C. Customer Perspective
Goals Performance
Price Competitive price
Delivery Number of on time delivery, lead time from receipt of order to
delivery to customers.
Quantity Own quality relative to industry standards, number of defects or
defect level
Support Response time, customer satisfaction surveys
D. Financial Perspective
Goals Performance
Sales Revenue and profit Growth
Cost Of Sales Extent to which it remained fixed or decreased each year
Profiablity Return on capital employed
Prosperity Cash Flow

CA. Parag Gupta Ph.: +91 11 47665555 Paraggupta_ca@yahoo.co.in Costing & O.R.
World’s largest CA Final student’s consultancy group: http://groups.yahoo.com/group/costingbyparaggupta
Standard Costing & Variance Analysis - 120 -

Advantages of Balanced Score Card:


1. Wholistic approach: It brings strategy and vision as the center of management focus. It helps
companies to assess overall performance, improve operational processes and enable management
to develop better plans for improvements. It provides management with a comprehensive picture of
business operations.
2. Overall Agenda: It brings together in a single management report many of the seemingly desperate
elements like customer oriented, shortening response time, improving quality etc. of competitive
agenda.
3. Objectivity: It emphasizes the need to provide the user with a set of information, which addresses all
relevant areas of performance in an objective and unbiased manner.
4. Management By Objectives: The methodology of balanced score card facilitates communication
and understanding of business goals and strategies at all levels of an organization. Thus it enables
management by objective.
5. Feedback and Learning: It provides strategic feedback and learning. The Balanced Score Card
guards against subordination. It emphasizes an integrated combination of traditional and non-
traditional performance measures.
6. System Approach: It help senior manages to consider all important performance measure
together and allows them to see whether an improvement in one area has been achieved at the
expense of another.

Question 64: “Balanced score card and performance measurement system endeavours to create a blend of
strategic measures, outcomes and drive measures and internal and external measures”. Discuss the
statement and explain the major components of a balanced score card. (4 Marks) May/05

Ans.: The balanced score card translates an organization's mission and strategy into a comprehensive set of
performance measures that provides the framework for implementing its strategy. The balanced score card
does not focus solely on achieving financial objectives. It is an approach, which provides information to
management to assist in strategic policy formulation and achievement. It emphasizes the need to provide the
user with a set of information, which addresses all relevant areas of performance in an objective and
unbiased manner. As a management tool it helps companies to assess overall performance, improve
operational processes and enables management to develop better plans for improvements.
Major components of a balanced scorecard - The components of balanced score cards varies form
business to business. A well designed balanced scorecard combines financial measures of post
performance with measures of firm's drivers of future performance. The specific objectives and measures
of an organization-balanced scorecard can be derived from the firm's vision and strategy. Generally,
balanced score card has the following four perspectives from which a company's activity can be
evaluated.
1. Financial perspective: Financial perspective measures the results that the organization delivers
to its stakeholders. The measures are: operating income, revenue growth, revenues from new
products, gross margin percentage, cost reduction in key areas, economic value added, return
on investment.
2. Customer perspective: The customer perspective considers the business through the eyes of
customers, measuring and rejecting upon customer satisfaction.
The measures are: - market share. customer satisfaction, customer retention percentage, time
taken to fulfil customer's requests.
3. Internal business perspective: The internal perspective focuses attention on the performance
of the key internal processes, which drive the business such as innovative process, operation
process and post-sales services.
4. Learning & growth perspective: The measure are:- employee education & skills levels,
employee turnover ratio, information system availability, percentage of employee suggestion
implemented etc.
Question 65: What are the elements of a Balanced Score card? Also explain how it can be used as a
Financial Planning model. (4 Marks) May/06
Question 66: Explain briefly the major components of a balanced score card.
(4 Marks) May/07 & (4 Marks) Nov./10-N.C.
Question 67: “In many organisations, initiatives to introduce balanced score card failed because efforts were
made to negotiate targets rather than to build consensus.”
CA. Parag Gupta Ph.: +91 11 47665555 Paraggupta_ca@yahoo.co.in Costing & O.R.
World’s largest CA Final student’s consultancy group: http://groups.yahoo.com/group/costingbyparaggupta
Cost Accounting & Management - 121 -

Required:
Elucidate the above statement. (8 Marks) Nov./07
Ans.: Balanced scorecard is a set of financial and non-financial measures relating to a company’s critical
success factors. It is an approach which provides information to management to assist in strategy
implementation. Therefore, the components to be included in the balanced score card must flow from
strategy. The targets should be measurable and must flow from strategy and corporate plan of the company.
It is necessary that managers should agree to the components and targets because in absence of a
consensus, managers may not commit to the targets established by the top management / the board of
directors. Moreover, the functions are interdependent and results in one functional area/perspective (e.g.
innovation and learning) have direct bearing on the results in other functional area / perspective (e.g.
customer perspective). Therefore, it is not sufficient that individual managers agree to their targets.
Successful implementation requires that the top management builds an overall consensus on the components
and targets of the balanced score card. Negotiation undermines the fundamental principle that the
components and targets should flow from strategy. As a result, an approach to establish targets through
negotiation defeats the very purpose of balanced score card.

Question 68: Explain goals and performance measure for each perspective of Balance Score Card.
(4 Marks) June/09-O.C.
Strategic Analysis of Operating Income

(1) Growth Component: It measures the change in operating income attributable solely to increase in the
quantity of output sold between 2 year i.e. it is measures the increase in revenues minus the increase in costs
from selling the more units of product. In it we assume that output prices, efficiency & capacity of Year 1 will
continue in Year 2. The calculations of growth component are similar to sales-volume variance (read in
Standard Costing).

(a) Revenue effect = (Actual output sold in Year 2 – Actual output sold in Year 1)×Output price/unit in Year 1

(b) Cost effect = (Actual Input or capacity used in Year 1 – Actual Input or capacity which would have been
used to produce output of Year 2 assuming same input output relationship that existed in
Year 1)×Input price/unit in Year 1

(2) Price Recovery Component: It measures the change in operating income attributable solely to changes
in prices of input & output between 2 years. It measures the amount by which the output price increases
outstrips input price increases. The calculations of price recovery component are similar to selling price
variance, and input price & spending variance for material, labour & overheads (read in Standard Costing).

(a) Revenue effect = (Selling Price in Year 2 – Selling Price in Year 1)×Actual output sold in Year 2
{Revenue effect is similar to Selling price variance}

(b) Cost effect = (Input price/unit in Year 1 - Input price/unit in Year 2)×Actual units of Input or capacity which
would have been used to produce output of Year 2 assuming same input output relationship
that existed in Year 1
{Cost effect is similar to Material price variance}

(3) Productivity Component: It measures the change in operating income attributable to a change in
quantity of inputs units used in Year 2 relative to the quantity of inputs that would have been used in Year 1 to
produce the Year 2 output. It measures the amount by which operating income increases by using inputs
productively to lower costs even when the prices of products are not increasing. The calculations for the
productivity variance are similar to efficiency variance (read in Standard Costing).

Cost effect = (Actual Input or capacity which would have been used to produce output of Year 2 assuming
same input output relationship that existed in Year 1 - Actual Input or capacity used in Year
2)×Input price/unit in Year 2

Question 69 [Balance Scorecard]: Kitchen King company makes a high-end kitchen range hood ‘Maharaja’.
The company presents the data for the year 2003 and 2004:

CA. Parag Gupta Ph.: +91 11 47665555 Paraggupta_ca@yahoo.co.in Costing & O.R.
World’s largest CA Final student’s consultancy group: http://groups.yahoo.com/group/costingbyparaggupta
Standard Costing & Variance Analysis - 122 -

2003 2004
1. Units or maharaja produced and sold 40,000 42,000
2. Selling Price per unit in ` 1,000 1,100
3. Total Direct Material (Square feet) 1,20,000 1,23,000
4. Direct material cost per square feet in ` 100 110
5. Manufacturing Capacity (in units) 50,000 50,000
6. Total Conversion cost in ` 1,00,00,000 1,10,00,000
7. Conversion cost per unit of capacity (6)/(5) 200 220
8. Selling and customer service capacity 300 290
customer customer
9. Total selling and customer service cost in ` 72,00,000 72,50,000
10. Cost per customer of selling and customer service capacity
(9)/(8) 24,000 25,000
Kitchen King produces no defective units, but it reduces direct material used per unit in 2004. Conversion cost
in each year depends on production capacity defined in terms of Maharaja units that can be produced. Selling
and Customer service cost depends on the number of customers that the selling and service functions are
designed to support. Kitchen King has 230 customers in 2003 and 250 customers in 2004. You are required
1. Describe briefly key elements that would include in Kitchen King’s Balance Score Card.
2. Calculate the Growth, Price-recovery and productivity component that explain the change in operating
income from 2003 to 2004. (18 Marks) Nov/05

Ans.: The change in operating income from 2003 to 2004 is analyzed as follows:
(Amount in 000 `)
Cost effect of
productivity
2003 Growth component Price recovery component 2004
Revenue 40000 2000 (F) 4200 (F) ------------ 46200
Cost 29200 600 (A) 2560 (A) 580 (F) 31780
Operating
Income 10800 1400(F) 1640 (F) 580 (F) 14420

CA. Parag Gupta Ph.: +91 11 47665555 Paraggupta_ca@yahoo.co.in Costing & O.R.
World’s largest CA Final student’s consultancy group: http://groups.yahoo.com/group/costingbyparaggupta
Budget and Budgetary
Control
(incld. Key factor
analysis, Throughput
A/cing & Synchronous
Manufacturing)
Key Factor/Limiting Factor analysis & Product Mix Decision

A key factor is defined as the factor in the activities of an undertaking which, at a particular point of time or
over a period, will limit the volume of output. Other variant terms are limiting factor, Principal Budget Factor &
scarce factor. Limiting factors are governed by both internal & external factors. It may be actual or potential. If
a factor of production is in short supply, then the best-paying product becomes that which yields the highest
contribution per unit of limiting factor.
Profitability= Contribution
Key Factor

Thus, Contribution per unit of key factor may be ascertained & maximized according to priority (ranking).
Some examples of key factors are:

I. Materials-Scarce Raw Material; Restrictions by licenses, etc.


II. Labour-General Shortage; Shortage of a particular type of labour.
III. Plant-Imbalance; Insufficient capacity due to shortage of capital, supply, etc.
IV. Management-Shortage of efficient staff; policy decisions.
V. Capital-Shortage of capital; insufficient research activity
VI. Sales-Market demand; insufficient advertisement.

Important Note 1: If no limiting factor is provided in question, but still we need to


rank the products for better allocation, we shall take CONTRIBUTION PER UNIT as
base for ranking.
Important Note 2: In case where there is more than one limiting factor, the technique
of linear programming is applied. We can also apply concepts of Throughput
Accounting for this purpose.

Question 1: A company manufactures and markets three products P, Q and R. All three products are made
from the same set of machines. Production is limited by machine capacity.

From the data given below, indicate priorities of products P, Q and R with a view to maximizing profits:

PRODUCT
P Q R
Raw material cost per unit (`)11.00 16.25 21.00
Direct labour cost per unit (`) 2.50 2.50 2.50
Other variable cost per unit (`)1.50 2.25 3.50
Selling price per unit (`)25.00 30.00 35.00
Standard machine time required 40 20 25
per unit in minutes

[Ans.: P-III; Q-I; R-II]

CA. Parag Gupta Ph.: +91 11 47665555 Paraggupta_ca@yahoo.co.in Costing & O.R.
World’s largest CA Final student’s consultancy group: http://groups.yahoo.com/group/costingbyparaggupta
Budget & Budgetary Control - 124 -
(incld. Limiting Factors, Product mix decisions,
Throughput A/cing & Synchronous Manufacturing)
Question 2: Jupiter Enterprises Ltd. is manufacturing three products A, B and C using the same raw
materials. All the products pass through departments I, II and III. Relevant details of the current production
plan are as follows:

Products
Standard cost per unit A B C
Direct materials `40 `50 `64
Direct labour:
Dept. I(Rate-`10/hr) 1 hr 1.2 hrs. 1.5 hrs.
Dept. II(Rate-`12/hr) 0.5 hr 1 hr 1 hr
Dept. III(Rate-`15/hr) 0.8 hr 1 hr 1.2 hrs.
Variable overhead `12 `11 `16
Current annual production (units) 30,000 40,000 25,000
Selling price per unit `100 `130 `175
Forecast of sales (max. units) 50,000 40,000 30,000

Fixed overheads per annum – `25 lakhs.


The labour available in department II is in short supply and cannot be increased in the near future. The
Managing Director has doubts about the correctness of the product mix proposed above. He has requested
you to examine the current production plan and give your specific recommendation.

You are required:


(a) To state whether the suggested product mix is the optimum one in the circumstances stated
above: if not, which is the optimum mix? Show workings;
(b) To indicate increase in profit, if any, if your suggestion is implemented. ICWA-June/97

[Ans.: (a) Suggested product mix is not optimum; (b) Increase in profit is `2 lakhs]
Question 3: A company producing products “PIE’ and ‘SIGMA’ using a single production process, has the
following cost data:

PIE SIGMA
Selling price per unit (`) 20 30
Variable cost per unit (`) 11 16
Machine hours required per unit production (hrs.) 1 2
Market limitation (units) 1 lakh 2.5 lakhs

Total machine hours available -4 lakhs.


Fixed cost per annum-`26 lakhs
Considering the limiting factors of machine hours and market limitations, you are required to

(a) Indicate the best combination of products to give optimum contribution;


(b) Show the additional machinery requirement to be augmented on rental basis at an annual rent of
`1.5 lakhs per machine to provide additional capacity of 30,000 hours per machine;
(c) Change in number of machines to be rented if the annual rental charges reduce to `1, 25,000 per
machine.

[Ans.: (a) Pie: 100000 units; Sigma: 150000 units; (b) 6 machines are to be taken on rental (c) 6 machines
are to be taken on rental] ICWA-June/97
Question 4: Mega Corporation manufactures and sells three products to the automobile industry. All the
products must pass through a matching process, the capacity of which is limited to 20,000 hours per annum,
both by equipment design and government regulation. The following additional information is available:

CA. Parag Gupta Ph.: +91 11 47665555 Paraggupta_ca@yahoo.co.in Costing & O.R.
World’s largest CA Final student’s consultancy group: http://groups.yahoo.com/group/costingbyparaggupta
Cost Accounting & Management - 125 -

Product - X Product – Y Product - Z


Selling price `/unit 1,900 2,400 4,000
Variable cost `/unit 700 1,200 2,800
Machining requirement hrs/units 3 2 1
Maximum possible sales – units 10,000 2,000 1,000

Required : statement showing the best possible production mix which would provide the maximum profit for
Mega Corporation together with supporting workings.

[Ans.: X :5000; Y: 2000; Z: 1000]

Question 5: A firm can produce three different products from the same raw material using the same
production facilities. The requisite labour is available in plenty at `8 per hour for all products. The supply of
raw materials, which is imported at `8 per kg. is limited to 10,400 kgs. for the budget period. The variable
overheads are `5.60 per hour. The fixed overheads are `50,000. The selling commission is 10 per cent on
sales.

a. From the following information, you are required to suggest the most suitable sales mix, which will
maximize the firms profits. Also determine the profit that will be earned at that level:

Product Market demand Selling price Labour hours Raw Material required
(units) per Unit required per Unit per Unit (Kgs)
X 8,000 30 1 0.7
Y 6,000 40 2 0.4
Z 5,000 50 1.5 1.5

b. Assume, in above situation, if additional 4,500 kgs of raw materials is made for production, should the firm
go in for further production, if it will result in additional fixed overheads of `25,000 and 25% increase in the
rates per hour for labour and variable overheads?

[Ans.: X : 8000, Y : 6000, Z : 1600 with Profit = 66160; Loss = 2300]

Question 6: Super Forging Ltd. is manufacturing three household products A, B and C and selling them in
competitive market. Details of current demand, selling price and cost structure are given below:

Particulars A B C
Expected demand (units) 10000 12000 20000
Selling price per unit (`) 20 16 10
Variable cost per unit (`)
Direct Materials (`10/kg) 6 4 2
Direct Labour (`15/hr.) 3 3 1.50
Variable Overheads 2 1 1
Fixed Overhead per unit (`) 5 4 2

The company is frequently affected by acute scarcity of raw material and high labour turnover. During the next
period it is expected to have one of the following situations:

(a) Raw materials available will be only 12100 kg.


(b) Direct labour hours available will be only 5000 hrs.
(c) It may be possible to increase sales of any one product by 25% without any additional fixed costs but
by spending `20000 on advertisement. There will be no shortage of materials or labour.

Suggest the best production plan in each case and the resultant profit that the company would earn according
to your suggestion. ICWA-June/98

CA. Parag Gupta Ph.: +91 11 47665555 Paraggupta_ca@yahoo.co.in Costing & O.R.
World’s largest CA Final student’s consultancy group: http://groups.yahoo.com/group/costingbyparaggupta
Budget & Budgetary Control - 126 -
(incld. Limiting Factors, Product mix decisions,
Throughput A/cing & Synchronous Manufacturing)

[Ans.: Net Profit (a) 117500 (b) 102000 (c) 165500]


[Hint: In part (c) of above mentioned question since we have been provided % increase in units we shall opt
the product which is going provide us highest increment in total contribution.
Now since total increase in contribution is
A: 10000×25%×9 = `22500
B: 12000×25%×8 = `24000
C: 20000×25%×5.5 = `27500
Hence we are supposed to increase sales of Product C (P/V ratio is not supposed to be seen in such cases).

However, if inspite of % increase, absolute value of increase in unit would have been provided, we would
choose Contribution per unit as our base of ranking. For example if in above it would have asked that “It may
be possible to increase sales of any one product by 2500 units without any additional fixed costs but by
spending `20000 on advertisement. There will be no shortage of materials or labour.”

In such cases we will rank our products on the basis of contribution per unit, hence we are supposed to
increase sales of Product A inspite of product C because, it is ultimately lead with highest increase in
contribution.]
Question 7[Sales mix with two key factors]: Arvind Electronics is a small scale unit manufacturing three
standard electronic components A, B and C .Current selling price and costs are as follows:

A B C
Selling price per unit `28 `60 `125
Direct materials 8 15 20
Direct labour (@ `10/hr.) 10 20 50
Variable overheads 5 10 25

Fixed overheads are budgeted at `25, 000 per annum. Direct labour availability is limited to 11,000 hours only
per year. The company has agreed to supply a minimum of 500 units of each of these products. Current
production plan of the company envisages supply of the minimum requirements and using the surplus labour
hours for production and sale of ‘C’ which has the highest margin.

Required:
(a) Profit according to current plan.
(b) Alternative plan, if any, for maximum profit.
(c) Break-even point in terms of units and sales value based on our calculations in (b).
(d) Number of units and sales value required in respect of each of the above three products to earn
24% profit after tax on capital employed of `1,00,000. Tax rate applicable to the firm is 50%.

[Ans.: (a) `42000 (b) Profit will be `52500 (c) 1500 units & `106500 (d) A: 500 units & `14000; B: 3700 units
& 222000; C: 500 & 62500] ICWA-Dec./97
Question 8: M/s Mars Ltd. is manufacturing three products. The cost details are as follows:

Products
Particulars A B C
Units ` Units ` Units `
Direct Materials 4 12 5 15 6 18
Direct Labour 5 6 6
Direct Expenses 8 9 11
25 30 35
Selling Price 35 40 50
10 10 15
No. of Units sold 20,000 40,000 20,000
Total Contribution 2,00,000 4,00,000 3,00,000
Total `9, 00,000
Less: Fixed Costs `7, 50,000
`1, 50,000

CA. Parag Gupta Ph.: +91 11 47665555 Paraggupta_ca@yahoo.co.in Costing & O.R.
World’s largest CA Final student’s consultancy group: http://groups.yahoo.com/group/costingbyparaggupta
Cost Accounting & Management - 127 -

The direct materials were all imported. Due to foreign exchange restrictions, henceforth, the company can
import only 3,00,000 units of raw materials. The company can produce in all 1,00,000 units maximum (all
products). However, they can market only 20,000 units each of product A and C. There is a local substitute
material which is available at a price of `3.75 per unit. Besides, the company has to spend `50, 000 on
intermediaries and consumables, if local substitute material is used in the production process. There was also
a third party who was willing to take a part of the plant on lease up to 50,000 units capacity of B and willing to
pay lease charges of `2,75,000.

You are required to advise the management:

(a) What should be the quantum of production/sales mix of products with existing import restrictions?
(b) Whether the company can optimize production of 1,00,000 units with local substance materials?
(c) Whether the company can enhance profits by leasing out a part of the plant to the third party and
restricting its own production? CA (Inter)/99

[Ans.: (a) A - 20000 units, B - 20000 units, C - 20000 units; (b) Yes; (c) No]
Question 9: The following particulars are taken from the records of a company engaged in manufacturing two
products, A and B, from a certain material:

Product A (per unit ) ` Product B (per


unit ) `
Sales 2,500 5,000
Material cost (`50 per kg) 500 1,250
Direct labour (`30 per hour) 750 1,500
Variable overhead 250 500

Total fixed overhead: `10, 00,000


Comment on the profitability of each product when:

(i) Total sales in value is limited.


(ii) Raw material is in short supply.
(iii) Production capacity is the limiting factor.
(iv) Total availability or raw materials is 20,000 kg. and maximum sales potential of each product is
1,000 units, find the product mix to yield maximum profits. CA (Inter) Nov/98

[Ans.: (i), (ii) & (iii) Product A is more profitable (iv) A: 1000 units; B: 400 units to yield a profit of `700000]
Question 10: A Company manufactures and sells three products A, B and C as per details given below:

Products
A B C
Direct materials `20 `16 `18
Direct labour 12 14 12
Variable overheads 8 10 6
Fixed overheads 6 6 4
46 46 40
Profit 18 14 12
Selling Price 64 60 52
Sales (Units) 10,000 5,000 8,000

(a) Rank the products according the profitability. What is the profit currently earned by the company?
(b) Production arrangements are such that if one of the products is discontinued, sales of the other two
can be increased by 50%.Management wants to discontinue ‘C’ as its contribution is least. Do you
agree? Present your recommendation supported by suitable analysis.
(c) There is also a proposal to introduce a highly profitable product ‘D’ using the existing facilities, along
with the two products retained at the original level of production. The details relating to product ‘D’ are
as follows:

CA. Parag Gupta Ph.: +91 11 47665555 Paraggupta_ca@yahoo.co.in Costing & O.R.
World’s largest CA Final student’s consultancy group: http://groups.yahoo.com/group/costingbyparaggupta
Budget & Budgetary Control - 128 -
(incld. Limiting Factors, Product mix decisions,
Throughput A/cing & Synchronous Manufacturing)
Variable cost Selling price / unit
per unit(`)
Direct Materials 12
Direct Labour 8 `48 (proposed)
Variable overheads 5

Demand for ’D’ is uncertain. Management wants to know the minimum sales that would ensure the maximum
profit as per (b). Give you recommendation, supported by workings. ICWA-June/01

[Ans.: (a) A-I; B-III; C-II with Profit of 3.46 lakhs; (b) C should not be discontinued. Product B should be
discontinued, then profit will improve to `430000 (c) Minimum Sales = 8000 units to ensure max. profit of
`430000]
Question 11: A company engaged in the manufacture of sophisticated products uses high grade raw
materials which are in short supply. During the year 1991, the company earned a profit of 12% before interest
and depreciation on turnover of `10 crores. Interest & depreciation which are fixed amounted to `75 lakhs &
`50 lakhs respectively. The product mix was as under:

Product Group Mix % to total sales P/V ratio (%) Raw Material as %
on sales value
A 30 30 40
B 20 40 50
C 50 25 36

During the year 1992, the price of the raw material is expected to increase by 10%. The company has been
able to make arrangements for the procurement of raw materials of the total value of `561 lakhs at 1992
prices. The sales potential of each product group can be increased in 1992 by 50% of 1991 sales.

Required:

(i) Set the optimal product mix for 1992.


(ii) What increase in overall price is required to raise sales value of 1992 to maintain the Margin of Safety
at 10%. (ICWA-June/83), (ICWA-June/93), (May/85-[Adapted]), (12 Marks) Nov./10-N.C.[Adapted]

[Ans.: (i) A - `450 lakhs; B - `300 lakhs, C – `500 lakhs; (ii) 1.33%]
Question 12: A company manufactures and sells three products P,Q,R whose unit selling price and cost
details are given below:

P Q R
Selling price/unit (`) 80 60 50
Direct materials /unit (`) 32 25 20
Direct labour/unit (`) 20 15 10
(@ `10/hr.)
Variable overheads/unit (`) 10 9 6
Total budgeted Fixed Overheads per
annum `5,50,000
Current sales (Units) 15,000 20,000 10,000

Required:

(a) Profit earned by the company p.a. according to estimates.


(b) Assuming that the company has only 75,000 labour hours per year, is it advisable to increase
production/sales of any one of the products by a maximum of 25% only by spending an additional
amount of `30,000 on advertisement/if so, which product should be chosen and what would be the
expected profit?
(c) A reduction of selling price by 5% will boost up sales of all products by 50%. But, direct labour hours
would continue to be 75,000. What is the best product mix for this proposition? Would you
recommend this option? Show workings. CA(Inter) Nov./01

CA. Parag Gupta Ph.: +91 11 47665555 Paraggupta_ca@yahoo.co.in Costing & O.R.
World’s largest CA Final student’s consultancy group: http://groups.yahoo.com/group/costingbyparaggupta
Cost Accounting & Management - 129 -

[Ans.: (a) `80000 (c) No]


Question 13[Specific and avoidable fixed cost]: Bloom Ltd. makes 3 products, A, B and C. The following
information is available:
(Figures in Rupees per unit)
A B C
Selling price (peak-season) 550 630 690
Selling price (off-season) 550 604 690
Material cost 230 260 290
Labour (peak-season) 110 120 150
Labour (off-season) 100 99 149
Variable production overhead 100 120 130
Variable selling overhead
(only for peak-season) 10 20 15
Labour hours required for one unit of
production 8 11 7 (hours)

Material cost and variable production overheads are the same for the peak-season and off-season. Variable
selling overheads are not incurred in the off-season. Fixed costs amount to `26,780 for each season, of which
`2,000 is towards salary for special technician, incurred only for product B, and `4,780 is the amount that will
be incurred on after-sales warranty and free maintenance of only product C, to match competition.

Labour force can be interchangeably used for all the products. During peak-season, there is labour shortage
and the maximum labour hours available are 1,617 hours. During off-season, labour is freely available, but
demand is limited to 100 units of A, 115 units of B and 135 units of C, with production facility being limited to
215 units for A, B and C put together.

You are required to:

(i) Advise the company about the best product mix during peak-season for maximum profit.
(ii) What will be the maximum profit for the off-season? (12 Marks) Nov/08-N.C.

[Ans.; (i) Best strategy for peak-season is to produce 202 units of A. (ii) Maximum profit for off-season
`4,375.]
Question 14: A farmer asks your recommendation for optimal mix of production for the coming year. The
current data is given below

ITEMS PRODUCED
A B C D
Area occupied (acres) 25 20 30 25
Yield per acre (tones) 10 8 9 12
Sales price per tonne (`) 1,000 1,250 1,500 1350
Variable cost per (acre)
Material (`) 700 600 950 900
Labour (`) 2,000 2,500 3,000 3,700
Variable overheads (`) 2,000 2,000 2,000 2,000
Fixed overhead `
Cultivation and growing 1,00,000
Harvesting and transport 2,40,000
Land revenue 90,000
Administration 1,10,000
Total 5,40,000

The land which is being used for producing items A and B can be used for either of the items but not for items
C and D. The land which is being used for producing items C and D can be used for either items but not for
items A and B.

In order to provide adequate market service, the farmer must produce each year at least 40 tonnes each of A
and B and 36 tonnes each of C and D.
CA. Parag Gupta Ph.: +91 11 47665555 Paraggupta_ca@yahoo.co.in Costing & O.R.
World’s largest CA Final student’s consultancy group: http://groups.yahoo.com/group/costingbyparaggupta
Budget & Budgetary Control - 130 -
(incld. Limiting Factors, Product mix decisions,
Throughput A/cing & Synchronous Manufacturing)

You are required to calculate the following:


(a) The profit for the current year; and
(b) The profit for the production mix which you could recommend.

[Ans.: (a) `157000; (b) `216300] C.S.- Dec./96 & (11 Marks) Nov./09-O.C.[Adapted]
Question 15: An agriculturist has 480 hectares of land on which he grows potatoes, tomatoes, peas and
carrots. Out of the total area of land, 340 hectares are suitable for all the four vegetables but the remaining
140 hectares of land are suitable only for growing peas and carrots. Labour for all kinds of farm work is
available is plenty.

The market requirement is that all the four types of vegetables must be produced with a minimum of 5000
boxes of any one variety. The farmer has decided that the area devoted to any crop should be in terms of
complete hectares and not in fractions of a hectare. The only other limitation is that not more than 113750
boxes of any one vegetable should be produced.

The relevant data concerning production, market prices and costs are as under:

Potatoes Peas Carrots Tomatoes


Annual yield:
Boxes per hectare 350 100 70 180
Costs:
Direct material per hectare `952 `432 `384 `624
Direct Labour:
Growing per hectare 1792 1216 744 1056
Harvesting and packing per box 7.20 6.56 8.80 10.40
Transport per box 10.40 10.40 8.00 19.20
Market price per box 30.76 31.74 36.80 44.55

Fixed expenses per annum:


Growing ` 124000
Harvesting 75000
Transport 75000
General administration 150000

It is possible to make the land presently suitable for peas and carrots, viable for growing potatoes and
tomatoes if certain land development work is undertaken. This work will involve a capital expenditure of `6000
per hectare which a band is prepared to finance at the rate of interest of 15% p.a. If such improvement is
undertaken, the harvesting cost of the entire crop of tomatoes will decrease on an average by `2.60 per box.

Required:

(i) Calculate, within the given constraints, the area to be cultivated in respect of each crop to achieve the
largest total profit and the amount of such total profit before land development work is undertaken.
(ii) Assuming that the other constraint continues, advise the grower whether the land development
scheme should be undertaken and if so the maximum total profit that would be achieved after the said
development scheme is undertaken. Nov./88

[Ans.: (i) Potatoes-312 hectares; Peas-50 hectares; Carrots-90 hectares; Tomatoes-28 hectares with profit of
20116240 (ii) Potatoes-325 hectares; Peas-50 hectares; Carrots-72 hectares; Tomatoes-33 hectares with
profit of 22477260]
Question 16[Key factor with no sales mix]: Kumar Co. has two plants one at Sambalpur and the other
at Bilaspur, where production of goods takes place. The basic raw material requirement is 80% of the
finished product, by weight. Such materials are available locally, but are limited to 6000 MT at `1800 per
MT at Sambalpur, and 16000 MT at `2000 per MT at Bilaspur. Any extra requirement will have to be
procured from Jamshedpur at `2500 per MT. Other particulars are as under-

CA. Parag Gupta Ph.: +91 11 47665555 Paraggupta_ca@yahoo.co.in Costing & O.R.
World’s largest CA Final student’s consultancy group: http://groups.yahoo.com/group/costingbyparaggupta
Cost Accounting & Management - 131 -

Particulars For Sambalpur Unit For Bilaspur Unit


Annual Output (MT) 12000 15000
Capacity Utilization 80% 60%
Other Variables (` Lakhs) 156 192
Fixed Costs (` Lakhs) 108 120

You are required to determine-

1. Cost break-up of each unit per MT of output.


2. Quantity of production at each unit of availability of local supplies of basic raw material only, by
keeping the same total production of the Company, as a whole.
3. Cost savings, if any, as per revised schedule of production.

[Ans.: Cost per M.T. of output : Sambalpur – `3850; Bilaspur `3680; Quantity of production : Sambalpur –
7500 M.T., Bilaspur - 19500] (12 Marks) Nov./01 & (12 Marks) Nov./10 [Adapted]

Question 17 [Key factor with no sales mix]: X Ltd. has two factories, one at Lucknow and another at Pune
producing 7,200 tonnes and 10,800 tonnes of a product against the maximum production capacity of 9,000
and 11,880 tonnes respectively at Lucknow and Pune.

10% of the raw material introduced is lost in the production process. The maximum quantity of raw material,
available locally is 6,000 and 13,000 tonnes at `720 and `729 per tonne at Lucknow and Pune respectively.
For the additional needs a supplier of Bhopal is ready to supply raw material at our factory site at `792 per
tonne.

Other variable costs of the production process are `22.32 lacks and `32.94 lacs and fixed costs are `18 lacs
and `24.84 lacs respectively for Lucknow and Pune factory.

The output is sold at a selling price of `1,450 and `1,460 per tonne by Lucknow and Pune factory
respectively.

You are required to compute the cost per tonne and net profit earned in respect of each factory.
Can you suggest any other alternative production plan for both the factories without any change in present
total output of 18,000 tonnes whereby the company may earn optimum profit? (19 Marks) Nov./97

[Ans.: Cost per tonne – Lucknow – `1380; Pune `1345; Net Profit - Lucknow `5.04 lakhs & Pune `12.42
Lakhs; Alternative production plan – Lucknow 6120 tonnes & Pune11880 tonnes.]

The theory of constraints and throughput accounting

During the 1980s Goldratt and Cox (1989) advocated a new approach to production management called
optimized production technology (OPT). OPT is based on the principle that profits are expanded by increasing
the throughput (throughput: sales –direct materials) of the plant. The OPT approach determines what
prevents throughput being higher by distinguishing between bottleneck and non-bottleneck resources. This
approach advocates that bottleneck resources/activities should be fully utilized while non bottleneck
resources/activities should not be utilized to 100% of their capacity since it would result in increase in
inventory.
A constraint is any thing that confines or limits a person’s or machines, ability to perform a project or
function. Delay in a production environment are caused by human and machine constraint. Human
constraint can be caused by an inability to understand react or perform at some higher rate of speed.
Machine constraints are called bottlenecks, are points at which processing levels are sufficiently
slow to cause the other processing mechanism in the network to experience idle time.

OPT is based on the principle that profits are expanded by increasing throughput of the plant i.e. rate at which
raw material are turned into sales. The most widely recognized management accounting system developed

CA. Parag Gupta Ph.: +91 11 47665555 Paraggupta_ca@yahoo.co.in Costing & O.R.
World’s largest CA Final student’s consultancy group: http://groups.yahoo.com/group/costingbyparaggupta
Budget & Budgetary Control - 132 -
(incld. Limiting Factors, Product mix decisions,
Throughput A/cing & Synchronous Manufacturing)
for this purpose is known in USA as Theory of Constraints (TOC). The theory was picked up and inducted into
an accounting system in the UK where it is known as Throughput Accounting (TA).

Instead of using marginal contribution to allocate resources to restricted resources, this theory uses the
concept of throughput : sales – direct materials.

More importantly, the Theory of Constraints concludes that in many cases, resources should not be thrown
heedlessly into production processes which are located before the bottleneck since this will only result in a
build up of partially finished inventory waiting for processing which is more costly to accumulate then it is to
underutilize the capacity. Traditional variance analysis focuses attention on underutilized capacity, e.g.
unfavourable volume variances, which simply encourage plant managers to produce a backlog.

Basis of introducing TOC is to reduce inventory created by non-bottleneck machines in front of bottleneck
machines.

The idea behind TOC is that raw materials is the only variable cost. Labour & variable overhead are consider
as fixed cost, except specifically provided. In it we deduct only true unit-level variable costs such as material
& power.
The theory of constraints (TOC) describes methods to maximize operating income under bottleneck situation.
The three measurements:
1. Calculate Throughput contribution = sale - direct materials cost of the goods sold.
2. Investments(inventory) = Sum of materials costs in direct materials, work – in – process, and finished
goods inventories; R & D costs; and costs of equipment and buildings.
3. Operating costs equal all costs of operations (other than direct materials) incurred to earn throughput
contribution. Operating costs include salaries and wages, rent utilities, and depreciation.
The objective of TOC is to increase throughput contribution while decreasing investments and operating
costs. TOC considers a short – run time and assumes that operating costs are fixed costs.

The important concept behind TOC is that the production rate of the entire factory is set at the pace of the
bottleneck resource. Hence, in order to achieve the best result TOC emphasizes the importance of removing
bottlenecks (or limiting factor).
Procedure of TOC (Theory of Constraints)
Step-1: Find the total requirement of resources i.e. S of units × resource required per unit. Compute it for
each department separately.
Capacity required
Step-2: Throughput accounting Ratio  TA ratio    100 for each department
Capacity available
Step-3: The highest among the TA ratio will be considered as the bottleneck factor (Other scarce resources
will be ignored).
Step-4: To obtain optimum use of the bottleneck activity we calculate contribution per bottleneck factor (a.k.a.
product return per time period on bottleneck activity) & rank accordingly.
Step-5: Prepare statement for optimum product mix for allocating the bottleneck factor.

Note: Some authors treats value derived in Step 2 as machine utilization ratio & calculates
Throughput contribution per time period
Throughput accounting ratio =
Conversion cost per time period
Question 18: Explain the theory of constraint. (4 Marks) Nov/03
Question 19: Explain the concept and aim of theory of constraints. What are the key measures of theory of
constraints? (7 Marks) May/08

Question 20: A company produces three products A, B and C. The following information is available for a
period:
A B C
Contribution 30 25 15
(Rupees per unit)
(Sales – Direct materials)
Machine hours required per unit of production:

CA. Parag Gupta Ph.: +91 11 47665555 Paraggupta_ca@yahoo.co.in Costing & O.R.
World’s largest CA Final student’s consultancy group: http://groups.yahoo.com/group/costingbyparaggupta
Cost Accounting & Management - 133 -

Hours
A B C Throughput accounting ratio
Machine 1 10 2 4 133.33%
Machine 2 15 3 6 200%
Machine 3 5 1 2 66.67%

Estimated sales demand for A, B and C are 500 units each and machine capacity is limited to 6,000 hours for
each machine.
You are required to analyze the above information and apply theory of constraints process to remove the
constraints. How many units of each product will be made? (6 Marks) Nov/08-N.C.

[Ans.: Units: A-100; B-500; C-500]


Question 21: A company produces 3 products A, B and C. The following information is available for a
period.

Production
A B C
Contribution `24 `20 `12
(Sales – Direct Materials)
Machine hours required per unit:
Machine 1 12 4 2
Machine 2 18 6 3
Machine 3 6 2 1
Estimated sales demand 200 200 200

It is given that machine capacity is limited to 3,200 hours for each machine, you are required to analyze the
above information and apply TOC process to remove the constraint.

[Ans.: Units: A-77; B-200; C-200]


Question 22: Vikram Ltd. produces 4 products using 3 different machines. Machine capacity is limited to
3,000 hours for each machine. The following information is available for February, 2009:

Products A B C D
Contribution (Sales-direct material) ` 1,500 1,200 1,000 600
Machine Hours Required/Unit:
Machine 1 10 6 2 1
Machine 2 10 9 3 1.5
Machine 3 10 3 1 0.5
Estimated Demand (units) 200 200 200 200

From the above information you are required to identify the bottleneck activity and allocate the machine time.

[Ans.: Units: A-200; B-11; C-200; D-200] (7 Marks) June/09-N.C.


Question 23: ABC Ltd makes and sells two products A and B, each of which passes through the same
automated production operations. The following estimated information is available for period 1

(i) Product unit data: A B


Direct material cost (`) 2 40
Variable production overhead cost (`) 28 4
Overall hours per product unit (hrs) 0.25 0.15

CA. Parag Gupta Ph.: +91 11 47665555 Paraggupta_ca@yahoo.co.in Costing & O.R.
World’s largest CA Final student’s consultancy group: http://groups.yahoo.com/group/costingbyparaggupta
Budget & Budgetary Control - 134 -
(incld. Limiting Factors, Product mix decisions,
Throughput A/cing & Synchronous Manufacturing)
(ii) Production/ sales of products A and B are 1,20,000 units and 45,000 units respectively. The selling prices
per unit for A and B are `60 and `70 respectively.

(iii) Maximum demand for each product is 20% above the estimated sales levels.

(iv) Total fixed production overhead cost is `14,70,000. This is absorbed by products A and B at an average
rate per hour based on the estimated production levels.

Required:

(a) Using net profit as the decision measure, show why the management of ABC Ltd argues that it is
indifferent on financial grounds as to the mix of products A and B which should be produced and sold and
calculate the total net profit for period 1.

(b) One of the production operations has a maximum capacity of 3,075 hours which has been identified as a
bottleneck which limits the overall production/ sales of products A and B. The bottleneck hours required per
product unit for product A and B are 0.02 and 0.015 respectively.

All other information detailed in (a) still applies.

Required: Calculate the mix (units) of products A and B which will maximize net profit and the value (`) of the
maximum net profit.

(c) The bottleneck situation detailed in (b) still applies. ABC Ltd has decided to determine the profit
maximizing mix of products A and B based on the throughput accounting principle of maximizing the
throughput return per production hour of the bottleneck resource. This may be measured as:

(selling price - material cost)


Throughput return per production hour =
Bottleneck hours per unit

All other information detailed in (a) and (b) still applies, except that the variable overhead cost as per (a) is
now considered to be fixed for the short/intermediate term, based on the value (`) which applied to the
product mix in (a).

Calculate the mix (units) of products A and B which will maximize net profit and the value of that net profit.

[Ans.: (a) Profit per unit = `20 under both the products (b) Profit: `3331500; (c) A: 144000 units, B: 13000
units; Net profit: `3732000] (RTP-May/07)
[Note: Assumed that variable overheads (e.g. Direct labour) are fixed in short term. They are derived from
part (a) as {120000×28 + 45000×4}]

SYNCHRONOUS MANUFACTURING

This concept of ‘synchronous manufacturing’ was started in 1984. It has been defined as: an all-
encompassing manufacturing management philosophy that includes a consistent set of principles,
procedures, and techniques where every action is evaluated in terms of the common global goal of the
organisation.

A set of seven ‘principles’ are associated with synchronous manufacturing:

1. Do not focus on balance idle capacities; focus on synchronizing the production flow.
2. The marginal value of time at a bottleneck resource is equal to the throughput rate of the products
processed by the bottleneck.
3. The marginal value of time at a non-bottleneck resource is negligible.
4. The level of utilization of a non-bottleneck resource is controlled by other constraints within the
system.
5. Resources must be utilized, not simply activated.
6. A transfer batch may not, and many times should not, be equal to the process batch.

CA. Parag Gupta Ph.: +91 11 47665555 Paraggupta_ca@yahoo.co.in Costing & O.R.
World’s largest CA Final student’s consultancy group: http://groups.yahoo.com/group/costingbyparaggupta
Cost Accounting & Management - 135 -

7. A process batch should be variable both along its route and over time.
According to synchronous manufacturing principles 2 and 3, the return on improvements at a bottleneck
resource is very high. But the return on improvement made at non-bottlenecks is marginal at best.
The synchronous manufacturing philosophy required managers to focus on those areas of operations where
there exist potential global improvements.

Question 24: Brief the principles associated with synchronous manufacturing. (5 Marks) May/10-N.C.

Budgeting Control & Performance Management

Budget is a financial and/or quantitative statement, prepared and approved prior to a defined period of time of
the policy to be pursued during that period for the purpose of attaining a given objective. It may include
income, expenditure and employment of capital.
Features:
1. Financial and/or Quantitative statement
2. Futuristic – prepared and approved prior to a defined period of time
3. Goal Oriented – for the purpose of attaining a given objective
4. Components – income, expenditure and employment of capital

The objective of budgeting are:


1. To encourage self-study in all aspects of a company’s operations.
2. To get all members of management to “put their heads” to the basic question of how the business
should be run to make them a co-ordinated team operating in unison towards clearly defined
objectives.
3. To force a definition and crystallization of company policies and aims.
4. To increase the effectiveness with which people and capital are employed.
5. To disclose areas of potential improvement in the company’s operations.
6. To stimulate study of relationship of the company to its external economic environment for improving
the effectiveness of its direction.

Budgetary Control:
Definition: Budgetary Control is defined as “the establishment of budgets, relating the responsibilities of
executives to the requirement of a policy, and the continuous comparison of actual with budgeted results
either to secure by individual action the objective of that policy or to provide a base for its revision.”
Salient features:
a) Objectives: Determining the objectives to be achieved, over the budget period, and the policy or
policies that might be adopted for the achievement of these ends.
b) Activities: Determining the variety of activities that should be undertaken for the achievement of the
objectives.
c) Plans: Drawing up a plan or a scheme of operation in respect of each class of activity in physical as
well as monetary terms for the full budget period and its parts.
d) Performance evaluation: Laying out a system of comparison of actual performance by each person,
section or department with the relevant budget and determination of causes for the discrepancies, if
any.
e) Control Action: Ensuring that corrective action will be taken where the plan is not being achieved
and, if that be not possible, for the revision of the plan.

Objectives of budgetary control system


The objectives of a budgetary control system are:
1. Definition of Goals: Portraying with precision, the overall aims of the business and determining
targets of performance for each section or department of the business.
2. Defining responsibilities: Laying down the responsibilities of each of the executives and other
personnel so that everyone knows what is expected of him and how he will be judged.
3. Basis for performance evaluation: Providing basis for the comparison of actual performance with
the predetermined targets and investigation of deviation, if any, of actual performance and expenses
from the budgeted figures. It helps to take timely corrective measures.
4. Optimum use of resources: Ensuring the best use of all available resources to maximize profit or
production, subject to the limiting factors.

CA. Parag Gupta Ph.: +91 11 47665555 Paraggupta_ca@yahoo.co.in Costing & O.R.
World’s largest CA Final student’s consultancy group: http://groups.yahoo.com/group/costingbyparaggupta
Budget & Budgetary Control - 136 -
(incld. Limiting Factors, Product mix decisions,
Throughput A/cing & Synchronous Manufacturing)
5. Coordination: Co-ordinating the various activities of the business and centralizing control, but also
facility for management to decentralise responsibility arid delegate authority.
6. Planned action: Engendering a spirit of careful forethought, assessment of what is possible and an
attempt at it. It leads to dynamism without recklessness. It also helps to draw up long range plans
with a fair measure of accuracy.
7. Basis for policy: Providing a basis for revision of current and future policies. Providing a yardstick
against which actual results can be compared.

Formulation of Policy i.e.


long-term plans

Preparation of Forecasts: Sales,


Production, Stocks, Costs, Capital
Expenditure, Cash & Master Forecasts

Consideration of alternative plans. Review of necessary actions on limiting


Selection of the plan which gives factors. Costs relating to such actions
maximum profit yet permits conditions obtained.
of long-term, financial stability.

Sales Budget Cash Budget Production Budget

Selling & Distribution Capital Expenditure Budgets for Prime Costs


Costs Budget Budget & Overhead Costs

Stock Budget Administration Costs Budget

Master Budget incld. Budgeted


P & L A/c & Balance Sheet

Accounting, variance analysis, Action by Management at all


reports & statements levels

Fig. Budgetary Control: diagram of procedure

CA. Parag Gupta Ph.: +91 11 47665555 Paraggupta_ca@yahoo.co.in Costing & O.R.
World’s largest CA Final student’s consultancy group: http://groups.yahoo.com/group/costingbyparaggupta
Cost Accounting & Management - 137 -

Role of a Budget Officer:

 Successful implementation of budgetary control system depends upon the Budget Committee.
 The Budget Committee would be composed of all functional heads and a member from the Board to
preside over and guide the deliberations.
 The Budget Committee acts through the Budget Officer whose responsibilities include:

1. Functional Budget preparation: To assist in the preparation of the various budgets by


coordinating the work of the accounts department which normally compiles the budgets, with
the relevant functional departments like Sales, Production, Plant maintenance etc.
2. Communication to Responsible Centres: To forward the budget to the individuals who are
responsible to adhere to them and to guide them in overcoming any practical difficulties in its
working.
3. Coordination: To prepare the periodical budget reports for circulation to the individuals
concerned, coordinating with them in the formulation of budgets for subsequent periods.
4. Follow-up: To determine the follow-up action to be taken on the budget reports.
5. Budget Committee Review: To prepare an overall budget working report for discussion at
the Budget Committee meetings and to ensure follow-up on the lines of action suggested by
the Committee.
6. Board Review: To prepare periodical reports for the Board meeting, comparing the budgeted
Profit and Loss Account and the Balance Sheet will the actual results.

Advantages of Budgetary Control System

1. Efficiency: It enables the management to conduct its business activities in an efficient manner.
Effective utilization of scarce resources, i.e. men, material, machinery, methods and money – is made
possible.
2. Cost Control: It is powerful instrument used by business houses for the control of their expenditure.
It inculcates the feeling of cost consciousness among workers.
3. Performance evaluation: It provides a yardstick for measuring and evaluating the performance of
individuals and their departments.
4. Standard Costing and Variance analysis: It creates suitable conditions for the implementation of
standard costing system in a business organization. It reveals the deviations to management from the
budgeted figures after making a comparison with actual figures.
5. Policy formulation: It helps in the review of current trends and framing of future policies.

Limitations of Budgetary Control System

1. Estimates: Budgets may or may not be true, as they are based on estimates. The assumptions about
future events may or may not actually happen.
2. Rigidity: Budgets are considered as rigid document. Too much emphasis on budgets may affect day-
to-day operations and ignores the dynamic state of organization functioning.
3. False Sense of Security: Mere budgeting cannot lead to profitability. Budgets cannot be executed
automatically. It may create a false sense of security that everything has been taken care of in the
budgets.
4. Lack of coordination: Staff co-operation is usually not available during budgetary control exercise.
5. Time and Cost: The introduction and implementation of the system may be expensive.

The interrelationship of budgets:

The critical importance of the principal budget factor stems from the factor that all budgets are interrelated.
For example, if sales are the principal budget factor this is the first budget to be prepared. This will then
provide the basis for the preparation of several others budgets, including the selling expenses budget and the
production budget.
However, the production budget cannot be prepared directly from the sales budget without a consideration of
stockholding policy. For example, management may plan to increase finished goods stock in anticipation of a
sales drive. Production quantities would then have to be higher than the budgeted sales level. Similarly, if a
decision is taken to reduce the level of material stocks held, it would not be necessary to purchase all of the
materials required for production.

CA. Parag Gupta Ph.: +91 11 47665555 Paraggupta_ca@yahoo.co.in Costing & O.R.
World’s largest CA Final student’s consultancy group: http://groups.yahoo.com/group/costingbyparaggupta
Budget & Budgetary Control - 138 -
(incld. Limiting Factors, Product mix decisions,
Throughput A/cing & Synchronous Manufacturing)

Using spreadsheets in budget preparation:


It is clear from just this simple example that exchange in one budget can have a knock-on effect on several
others budgets. For this reason spreadsheets are particularly useful in budget preparation. Budgetary
planning is an iterative process. Once the first set of budgets has been prepared they will be considered by
senior managers. They may require amendments to be made or they may wish to see the effect of changes in
key decision variables.
A well-designed spreadsheet model can take account of all of the budget interrelationships. This means that it
will not be an onerous task to alter decision variables and produce revised budgets for management’s
consideration.

Flexible Budget:
Meaning: It is a budget, which is designed to change in relation to level of activity by recognizing the
difference between fixed, semi-variable and variable costs.
Need: The need for preparation of flexible budgets arises in the following circumstances
 Seasonal fluctuations in sales and/or production, for example in soft drinks industry. Introduction of
new products, product design and versions on a frequent basis.
 Industries engaged in make-to-order business like shipbuilding;
 An industry which is influenced by changes in fashion; and General change in sales

Flexible budgeting may be resorted to in the following situations:

 New Business: In the case of new business venture due to its typical nature it may be difficult to
forecast the demand of a product accurately.
 Uncertain Environment: Where the business is dependent upon the mercy of nature e.g. a person
dealing in wool trade may have enough market if temperature goes below the freezing point.
 Factor market conditions: In the case of labour intensive industry where the production of the
concern is dependent upon the availability of labour.

Steps involved in the preparation of budgets

Definition of Objectives: Objectives should be defined precisely. They should be written out; areas of control
de-marketed and items of revenue and expenditure to be covered by the budget stated. This will give a clear
understanding of the plan and its scope to all those who must cooperate to make it a success.

Identification of key (or budget) factor: A key factor represents source whose availability is less than its
requirement. Such resource constraints put a limit on the organization objective of maximum profitability.
Some examples are lack of sales demand, rationing of raw material, labour shortage, plant capacity etc. For
proper budgeting, the key factor must be located and estimated properly.

Budget Committee and Controller: Formulation of a budget usually requires whole time services of a senior
executive; he must be assisted in this work by a Budget Committee, consisting of all the heads of department
along with the Managing Director as the Chairman. The Controller is responsible for coordination and
development of budget programmes and preparing the Budget Manual .

Budget Manual: The Budget manual is a schedule, document or booklet, which shows in a written form, the
budgeting organization and procedure. The manual should be well written and indexed so that a copy thereof
may be given to each department head for guidance.

Budget period: The period covered by a budget is known as budget period. Normally a calendar year or a
period coterminous with the financial year is adopted as the Budget Period. It is then sub-divided into shorter
periods – it may be months or quarters or such period as coincide with period of trading activity.

Standard of activity or output: The standards of activity levels for future period should be laid down. These
are generally based on past statistics, known market changes and current conditions and forecast of future
situations. In a progressive business, the achievement of a year must exceed those of earlier years. In
budgeting, fixing the budget of sales and capital expenditure are most important since these budgets
determine the extent of development activity.

CA. Parag Gupta Ph.: +91 11 47665555 Paraggupta_ca@yahoo.co.in Costing & O.R.
World’s largest CA Final student’s consultancy group: http://groups.yahoo.com/group/costingbyparaggupta
Cost Accounting & Management - 139 -

Specimen of Master Budget


For the year ending………………………..
Normal Capacity……………………………………………………………… standard hours (100%)
Capacity Budgeted……………………………………………………………. standard hours
Budgeted Figures
Product A (`) Product A (`) Total (`)
1. SALES
Manufacturing Costs:
Direct Labour
Direct Material
Factory Overhead
Add Opening Stock
Less Closing Stock
2. COST OF GOODS SOLD
3. Gross Profit ( 1 - 2 )
4. Selling & Distribution Costs
5. Administration Costs
6. NET PROFIT ( 3 – 4 & 5 )
7. Fixed Assets
8. Current Assets
9. CAPITAL EMPLOYED
10. Ratio of Profit to Capital Employed
11. Ratio of Sales to Capital Employed
12. Current Ratio
13. Quick Ratio
14. PROFIT APPROPRIATIONS
15. Net Profit (6 above)
Less Dividends
Transfer to General Reserve
Transfer to Asset Replacement
Reserve
Taxation
16. TOTAL APPROPRIATIONS
17. PROFIT & LOSS BALANCE ( 15 – 16 )

The Budget Manual


Effective budgetary planning relies on the provision of adequate information to the individuals involved in the
planning process.
Many of these information needs are contained in the budget manual. A budget manual is a collection of
documents that contains key information for those involved in the planning process. Typical contents could
include the following:
a) An introductory explanation of the budgetary planning and control process, including a statement of
the budgetary objective and desired results.
b) A form of organisation chart to show who is responsible for the preparation of each functional budget
and the way in which the budgets are interrelated.
c) A timetable for the preparation of each budget. This will prevent the formation of a ‘bottleneck’ with
the late preparation of one budget holding up the preparation of all others.
d) Copies of all forms to be completed by those responsible for preparing budgets, with explanations
concerning their completion.
e) A list of the organization’s account codes, with full explanations of how to use them.
f) Information concerning key assumptions to be made by managers in their budgets,
for example the rate of inflation, key exchange rates, etc.

ZERO BASE BUDGETING (ZBB):

Meaning: It is an expenditure control device where each divisional head has to justify the requirement of
funds for each head of expenditure and prepare the budget accordingly, without reference to the past budget
or achievements.
CA. Parag Gupta Ph.: +91 11 47665555 Paraggupta_ca@yahoo.co.in Costing & O.R.
World’s largest CA Final student’s consultancy group: http://groups.yahoo.com/group/costingbyparaggupta
Budget & Budgetary Control - 140 -
(incld. Limiting Factors, Product mix decisions,
Throughput A/cing & Synchronous Manufacturing)

It is an operating planning and budgeting process, which requires each manager to justify his entire budget
requests in detail from “scratch” (hence zero-base).

Features:

1. Wholistic: The technique deals practically with all the elements of budget proposals.

2. Analytical: A critical evaluation of all the ongoing activities is also done afresh together with new
proposals. Each manager has to justify whey he should spend any money at all.

3. Priority Based: This approach requires that all activities be identified as decision on packages, which
would be evaluated by systematic analysis and ranked in order of importance.

4. Review Based: an organisation should not only make decisions about the proposed new-
programmes but it should also from time to time, review the ”utility” and “appropriateness” of the
existing programmes.

5. Rational: It allows for budget reductions and expansions in a rational manner and allows re-
allocation of resources from low to high priority programme.

Steps in Zero Base Budgeting (ZBB)


ZBB involves the following:

 Objectives: Determination of a set of objectives is one of pre-requisite and essential step in the
direction of ZBB technique.

 Coverage: Deciding about the extent to which the technique of ZBB is to be applied, whether in all
areas of organisation’s activities or only in a few selected areas on trial basis.
 Decision Areas :Identify those areas where decisions are required to be taken.
 Ranking: Developing decision – package and ranking them in order of preference.
 Budgeting: Preparation of budget, that is translating decision packages into practicable units items
and allocating financial resources.

Advantages of ZBB:
The advantages of zero-base budgeting are as follows.
1. Priority allocation: It provides a systematic approach for the evaluation of different activities and
ranks them in order of preference for the allocation of scarce resource.
2. Maximum Efficiency: It ensures that the various functions undertaken by the organization are critical
for the achievement of its objectives and are being performed in the best possible way.
3. Cost Benefit Analysis: It provides an opportunity to the management to allocate resource for various
cost benefit analysis. The chances of arbitrary cuts and enhancement are thus avoided.
4. Elimination of wasteful expenditure: The areas of wasteful expenditure can be easily identified and
eliminated.
5. Goal Congruence: Department budgets are closely linked with corporate objectives.
6. Management by Objectives: The technique can also be used for the introduction and
implementation of the system of ‘management by objective’ Thus it cannot only be used for fulfillment
of the objectives of traditional budgeting but it can also be used for a variety of other purposes.

Limitations of ZBB:
1. Lack of Coordination: Various operational problems are likely to be faced in implementing the
technique of ZBB. It requires the wholehearted support from top management.
2. Old is Gold Attitude: Generally managers are reluctant to start afresh. They tend to plan for future
just by reference to past actions and budgets.
3. Time Consuming: It is time consuming as well as costly.
4. Lack of trained staff: It needs properly trained managerial personnel to do the required job.

Performance Budgeting: Meaning:

CA. Parag Gupta Ph.: +91 11 47665555 Paraggupta_ca@yahoo.co.in Costing & O.R.
World’s largest CA Final student’s consultancy group: http://groups.yahoo.com/group/costingbyparaggupta
Cost Accounting & Management - 141 -

It is the process of analysing, identifying, simplifying and crystallizing specific performance objectives, of a job
to be achieved over a period, within the framework of organizational objectives, the purposes and objectives
of the job.
The technique is characterised by its specific direction towards the business objectives of the organization.

Features and Advantages:


 Performance budgeting lays immediate stress on the achievement of specific goals over a period of
time.
 It aims at a continuous growth of the organisation so that it continues to meet the dynamic needs of
its growing clientle.
 It enables the organization to be sensitive and adaptive, preventing it from developing rigidities, which
may retard the process of growth.
 It requires the preparation of periodic performance reports, which compare budget and actual
performance to find out existing variances.

Important considerations in Performance Reporting:


The important considerations in drawing up of reports and determining their scope are the following:

Significance  Reliability – Are the facts in the report reliable?


 Cause or Effect – Does it either call for action or demonstrate the
effect of action
Timeliness  Latest time – How late can the information be and still be of use?
 Earliest Time – What is the earliest moment at which it could be
used if it were available?
 Frequency – How frequently is it required?
Accuracy  How small should be an inaccuracy, which does not alter the
significance of the information.
 What doubtful elements does it contain? Could any of them or all
together make a material difference?
Appropriateness  Receiver – is the recipient the right person to take any action that is
needed?
 Additional Information – Is there any other information which is
required to support the information to anyone else jointly interested?
Discrimination  Will anything be lost by omitting the item?
 Will any of the items gain from the omission?
 Is the responsibility for suppressing the item acceptable?
Presentation  Clarity – Is the report clear and unbiased?
 Form – Is the form suitable to the subject and to the recipient?

Performance Reports:
A. Top Management: (including Board of Directors and financial mangers)
 Balance Sheet
 Profit & Loss Statement
 Position of Stock;
 Disposition of funds or working capital
 Capital expenditure and forward commitments together with progress of projects in hand. Cash flow
statements;
 Sales, production, and other appropriate statistics

B. Sales Management
 Actual sales compared with budgeted sales to measure performance by (a) products; (b) territories;
(c) individual salesmen; and (d) customers.
 Standard profit and loss, product-wise (a) for fixing selling prices and (b) to concentrate on sales of
most profitable products.
 Selling expenses in relation to budget and sales value analysed by (a) products; (b) territories; (c)
individual salesmen; and (d) customers

CA. Parag Gupta Ph.: +91 11 47665555 Paraggupta_ca@yahoo.co.in Costing & O.R.
World’s largest CA Final student’s consultancy group: http://groups.yahoo.com/group/costingbyparaggupta
Budget & Budgetary Control - 142 -
(incld. Limiting Factors, Product mix decisions,
Throughput A/cing & Synchronous Manufacturing)
 Bad debts and accounts, which are slow and difficult in collection. Status reports on new or doubtful
customers.

C. Production management
 To Buyer:
Price variations on purchases analysed by commodities.
 To Foreman
Operational Efficiency for individual operators duly summarized as departmental averages. Labour
utilization report and causes of lost time and controllable time; Indirect shop expenses against the
standard allowed; and Scrap report
 To Works Manager
1. Departmental operating statement
2. General works operating statements (Expenses relating to all works expenses not directly
allocable or controllable by departments);
3. Plant utilization report;
4. Department scrap report; and
5. Material usage report

Special Reports: These are prepared at the request of general management or at the initiative of the
management accountant. Some examples are:
 Taxation legislation and its effect on profits Estimates of the earning capacity of a new project Break-
even analysis
 Capital budgeting decisions
 Special pricing analysis
 Make or buy certain components

Question 25: What are the advantages and limitations of Zero base Budgeting? (4 Marks) Nov./04

Question 26: “Because a single budget system is normally used to serve several purposes, there is a danger
that they may conflict with each other”. Do you agree? Discuss. (4 Marks) May/05
Ans.: A single budget system may be conflicting in planning and motivation, and planning and performance
evaluation roles as below:
Planning and motivation roles – Demanding budgets that may not be achieved may be appropriate to
motivate maximum performance but they are unsuitable for planning purposes. For these, a budget should be
a set based on easier targets that are expected to be met.
Planning and performance evaluation roles - For planning purposes budgets are set in advance of the budget
period based on an anticipated set of circumstances or environment. Performance evaluation should be
based on a comparison of active performance with an adjusted budget to reflect the circumstance under
which managers actually operated.

Question 27: Write short note on ‘Zero Base Budgeting’ as an approach towards productivity improvement.
(4 Marks) Nov./05
Question 28: Describe the process of zero-base budgeting. (4 Marks) May/07 & (5 Marks) Nov./10-N.C.

Question 29: A manufacturing company makes 4 products that are sold through 8 regional offices
countrywide. The products pass through 3 production processes in a factory. A separate market research
division monitors outside competition. This division is outside the sales management hierarchy.
As a management accountant, suggest some routine reports for performance measurement to be made to :
(a) The Sales Management
(b) The Works Manager (9 Marks) Nov./10-O.C.

CONCEPTS RELATED TO CAPACITY (Refer CAS 2 & 3)


 ‘Licenced Capacity’ is the production capacity of the plant for which license has been issued by an
appropriate authority.

 ‘Installed Capacity’ is the maximum productive capacity according to the manufacturers’ specification of
machines/equipment. In other words, it is only a theoretical capacity and is therefore, seldom achieved. In
case a product passes through different production processes and each process is having different

CA. Parag Gupta Ph.: +91 11 47665555 Paraggupta_ca@yahoo.co.in Costing & O.R.
World’s largest CA Final student’s consultancy group: http://groups.yahoo.com/group/costingbyparaggupta
Cost Accounting & Management - 143 -

capacity then the process which brings effective or ultimate production shall be considered for deciding
installed capacity. It is a measure of maximum operating capacity based on 100% efficiency with no
interruption for maintenance or other factors (i.e. Maximum no. of days in a period × No. of workers ×
Hours per day)

 ‘Practical or Achievable Capacity’ is the maximum productive capacity of a plant reduced by the
predictable and unavoidable factors of interruption pertaining to internal causes.Thus, practical capacity is
the installed capacity (-) the inevitable interruptions due to time lost for preventive maintenance, repairs,
set ups, normal delays, weekly off-days and holidays etc. It is defined as actually utilized capacity of a
plant. Practical capacity does not consider the external factors causing reduction in production e.g. lack of
orders.

 ‘Normal capacity’ is determined based on the productive capacity achieved over a period of time, say
average of three normal years out of preceding five years or expected to be achieved over a period of
time, say next three to five years. The periods influenced by abnormalities should be excluded for this
purpose. Unlike practical capacity, normal capacity allows for idleness both of plant and personnel
caused due to lack of sales orders.

Note: Since practical capacity is based on possible production & normal capacity is based on production
basing on possible sales, it is not tenable assumption that firm will produce without taking into account
sales level, normal capacity is preferable measure to calculate factory overhead absorption rate.

 ‘Expected or Short-run expectancy’ is the capacity of a plant utilized based on sales expectancy for
next period/year (i.e. in a single year only). Thus, expected capacity differs from normal capacity in the
length of time to determine capacity base. This measure doesn’t smoothen out cyclic fluctuations in sales
that are likely to occur over a period of time as it is guided by one year projection only. If there is no sales
problem it is clear that the practical capacity & short term capacity will coincide.

 ‘Actual Capacity Utilization’ is the volume of production achieved in relation to installed capacity.

 ‘Idle Capacity’ is the difference between installed capacity and the actual capacity utilization when actual
capacity utilization is less than installed capacity.

 ‘Excess Capacity Utilization’ is the difference between installed capacity and the actual capacity
utilization when actual capacity utilization is more than installed capacity.

 ‘Abnormal idle capacity’ is the difference between practical capacity and normal capacity or actual
capacity utilization whichever is higher

Note: Whenever question specifically provides budgeted capacity, we take the capacity so provided
as budgeted capacity otherwise we take budgeted capacity as normal capacity, when sales remains
reasonably constant although if sales fluctuate from one year to another, we take budgeted capacity
as expected capacity. If normal/expected capacity is missing, we take it as practical capacity.

Remember, fixed overheads absorption rate is to be always calculated on Budgeted capacity i.e.
Budgeted Fixed Overheads
Budgeted Capacity
If a question provides us fixed overhead absorption rate along with budgeted as well as actual capacity, we
will multiply it with budgeted capacity (not by actual capacity) to find Budgeted Fixed Overhead.

Question 30: Manufacturers’ Specifications - capacity per hour = 500 units


No of shifts (each shift 8 hours) = 3 shifts
Holidays in a year:
Sundays = 52 days
Other holidays = 13 days
Annual maintenance is done within these 13 holidays
Preventive Weekly Maintenance for the machine on Sunday.
Normal idle capacity for batch change over, lunch, personal need, etc. = 1 hr per shift

CA. Parag Gupta Ph.: +91 11 47665555 Paraggupta_ca@yahoo.co.in Costing & O.R.
World’s largest CA Final student’s consultancy group: http://groups.yahoo.com/group/costingbyparaggupta
Budget & Budgetary Control - 144 -
(incld. Limiting Factors, Product mix decisions,
Throughput A/cing & Synchronous Manufacturing)
Production (units) based on sales expectancy in past 5 years = 30.1, 26.9, 29.7, 24.4 and 30.2 lakh
Actual Production for the year = 30.1 lakh

[Ans.: Installed capacity = 43.8 lakh units; Practical capacity = 31.5 lakh units; Normal capacity = 30 lakh
units; Actual capacity utilization = 68.7%; Idle capacity = 31.3%; Abnormal idle capacity = 1.4 lakh units]
[Hint: It is assumed that business cycle consists of 3 years]

Question 31: XYZ Ltd. wishes to determine various capacity levels both in terms of production units and
machine-hours. One machine-hour produces 10 units of finished product. The production department in which
the machine is located normally operates 6 days a week (except Sunday) on a single, eight hour shift. The
plant is closed for 10 working days each year for holidays.
The plant is closed for 200 hours each year for its repairs and maintenance. Normal sales demand averages
20000 units a year over a 5-year period (extensive product changes are made every five years). The
expected sales volume for the next year is 19000 units. Budgeted Fixed overhead costs are `6,00,000 &
variable overhead costs at `100 per hour as the basis of determining factory overhead application rate.
Show the machine-hours and production capacity at four levels. Also calculate factory overhead application
rate.

[Ans.: Maximum capacity: 2920 hours or 29200 units; Practical capacity: 2224 hours or 22240 units; Normal
capacity: 2000 hours or 20000 units; Expected capacity: 1900 hours or 19000 units; Factory overhead
application rate per hour: `400; Factory overhead application rate per unit: `40]

Question 32: A manufacturer produces three products whose cost data are as follows:

X Y Z
Direct materials (`/ Unit) 32.00 76.00 58.50
Direct Labour:
Deptt. Rate / hour (`) Hours Hours Hours
1 2.50 18 10 20
2 3.00 5 4 7
3 2.00 10 5 20
Variable overheads (`/unit) 8 4.50 10.50

Fixed overheads (`) 4,00,000 per annum.


The budget was prepared at a time, when market was sluggish. The budgeted quantities and selling prices
are as under:

Product Budgeted quantity Selling Price / unit


(Units) (`)
X 19,500 135
Y 15,600 140
Z 15,600 200

Later, the market improved and the sales quantities could be increased by 20 per cent for product X and 25
per cent each for product Y and Z. The sales manager confirmed that the increased sales could be achieved
at the prices originally budgeted. The production manager stated that the output could not be increased
beyond the budgeted level due to the limitation of Direct labour hours in department 2.

Required:
(i) Prepare a statement of budgeted profitability.
(ii) Set optimal product mix and calculate the optimal profit. (14 Marks) Nov./07

[Ans.: (i) Profit `399500 (ii) X 23400 units, Y 19500 units, Z 10585 units; optimal profit `406450]
Question 33 [Flexible Budget]: JBC Limited, a manufacturing company having a capacity of 60,000 units
has prepared a following cost sheet:
CA. Parag Gupta Ph.: +91 11 47665555 Paraggupta_ca@yahoo.co.in Costing & O.R.
World’s largest CA Final student’s consultancy group: http://groups.yahoo.com/group/costingbyparaggupta
Cost Accounting & Management - 145 -

Direct material per unit) `12.50


Direct wages (per unit.) `5.00
Semi-variable cost `30,000 fixed plus 0.50 per unit
Factory overhead (per unit) `10.00 (50% fixed)
Selling and administration overhead (per unit) `8.00 (25% variable)
Selling price (per unit) `40

During the year 2008, the sales volume achieved by the company was 50,000 units.

The company has launched an expansion program as under:

(a) The capacity will be increased to 1,00,000 units.


(b) The cost of investment on expansion is `5 lakhs which is proposed to be financed through financial
institution at 12 per cent per annum.
(c) The depreciation rate on new investment is 10 per cent based on straight line.
(d) The additional fixed overheads will amount to `2.00 lakhs up to 80,000 units and will increase by `80,000
more beyond 80,000 units.

After the expansion, the company has two alternatives for operating the expanded plant as under:
(i) Sales can be increased up to 80,000 units by spending `50,000 on special advertisement campaign to
explore new market.
(ii) Sales can be increased up to 1,00,000 units subject to the following:
(a) Reduction of selling price by `4 per unit on all the units sold.
(b) The direct material cost would go down by 4 per cent due to discount on bulk buying.
(c) By increasing the variable selling and administration expenses by 4 per cent.

Required:
(i) Construct a flexible budget at the level 50,000 units, 80,000 units and 1,00,000 units of production and
select best profitable level of operation.
(ii) Calculate break even point both before and after expansion.
ICWA June/84[Adapted] & (9 Marks) June./09-O.C.
[Ans.: (i) Profit: 50000 units – `60000; 80000 units – `150000; 100000 units – `62000; (ii) BEP: Before
Expansion: 50000 units – 46000 units; 80000 units – 70000 units; After expansion: 100000 units – 94570
units ]
Question 34 [Flexible Budget for overhead items – Variance- Overhead absorbed and overhead
volume variance]: A manufacturing company has the following budgeted costs for one month which are
based on a normal capacity level of 40,000 hours. A departmental overhead absorption rate of `4.40 per hour
has been calculated as follows:

Overhead Fixed (`’000) Variable per hour (`)


Management and supervision 30 -
Shift premium - 0.10
ESI and pension costs 6 0.22
Inspection 20 0.25
Consumable supplies 6 0.18
Power for machinery - 0.20
Lighting and heating 4 -
Rates 9 -
Repairs and maintenance 8 0.15
Materials handling 10 0.30
Depreciation of machinery 15 -
Production administration 12 -
120
Overhead rate per hour:
Variable 1.40
Fixed 3.00
Total 4.40

CA. Parag Gupta Ph.: +91 11 47665555 Paraggupta_ca@yahoo.co.in Costing & O.R.
World’s largest CA Final student’s consultancy group: http://groups.yahoo.com/group/costingbyparaggupta
Budget & Budgetary Control - 146 -
(incld. Limiting Factors, Product mix decisions,
Throughput A/cing & Synchronous Manufacturing)
During the month of April, the company actually worked 36,000 hours producing 36,000 standard hours of
production and incurred the following overhead costs.

`’000
Management and supervision 30.0
Shift premium 4.0
ESI and pension costs 15.0
Inspection 28.0
Consumable supplies 12.7
Power for machinery 7.8
Lighting and heating 4.2
Rates 9.0
Repairs and maintenance 15.1
Materials handling 21.4
Depreciation of machinery 15.0
Production administration 11.5
Idle time 1.6
175.3

You are required to:

(a) Prepare a statement showing the flexible budget for the month of April, the actual costs and the
variance for each overhead item;
(b) Comment on each variance of `1,000 or more by suggesting possible reasons for the variances
reported;
(c) State for control purposes, with reasons to support your conclusions:
(i) Whether (b) above is adequate; and
(ii) Whether the statement prepared in request in (a) above could be improved, and if so, how:
(d) Calculate:
(i) The overhead absorbed;
(ii) The total amount under/over–spent ; and
(iii) The overhead volume variance ICWA-June/93

[Ans.: (a) Total overhead variance `4900A; (d) (i) `158400 (ii) `4900 (iii) `12000(A)]
Question 35 [Cash Budget]: Following budgeted sales values have been extracted from the budget of A.Z.
limited for the year ending 31st December 1997:

April `4,00,000
May 4,50,000
June 5,20,000
July 4,20,000
August 4,80,000

The contribution/sales ratio is 40% Fixed costs are budgeted to be `12,00,000 for the year arising at a
constant rate per month and including depreciation of `3,00,000 per annum.

40% of each month’s sales are produced in the month prior to sale, and 60% are produced in the month of
sale. 50% of the direct materials required for production are purchased in the month prior to their being used
in production.

30% of the variable costs are labour costs, which are paid in the month they are incurred.

60% of the variable costs are direct material costs. Suppliers of direct materials are paid in the month after
purchase.

The remaining variable costs are variable overhead costs. 40% of the variable overhead costs are paid in the
month they are incurred, the balance being paid in the month after they are incurred.
Fixed costs are paid in the month they incurred.

CA. Parag Gupta Ph.: +91 11 47665555 Paraggupta_ca@yahoo.co.in Costing & O.R.
World’s largest CA Final student’s consultancy group: http://groups.yahoo.com/group/costingbyparaggupta
Cost Accounting & Management - 147 -

Capital expenditure expected in June is `1,90,000.

Sales receipts for the three months of May, June and July are budgeted as follows:
May `4,01,700
June 4,50,280
July 4,25,880

The bank balance on May 1,1997 is expected to be `40,000.

Requirement:
Prepare a cash budget for A.Z. Limited.
Your budget should be in columnar format showing separately the receipts, payments and balances for each
of the months of May, June and July 1997.

[Ans.: Closing Balance – May: `92428, June: (`9860), July: 66844]


Question 36 [Production Cost budget]: The budgeted and actual cost data of M Ltd. for 6 months
from April to September, 2008 are as under:

Budget Actual
Production units 16,000 14,000
Material cost `25,60,000 `41,60,000
(1,600 MT @ `1,600) (at `1,650)
Labour cost `16,00,000 `15,99,840
(at `40 per hour) (@ `44 per hour)
Variable overhead `3,00,000 `2,76,000
Fixed overhead `4,60,000 `5,80,000

In the first half of financial year 2009-10, production is budgeted for 30,000 units, material cost per tonne will
increase from last year’s actual by `150, but it is proposed to maintain the consumption efficiency of 2008 as
budgeted. Labour efficiency will be lower by 1% and labour rate will be `44 per hour. Variable and fixed
overheads will go up by 20% over 2008 actuals.

Prepare the Production Cost budget for the period April -September, 2009 giving all the workings.
(6 Marks) Nov./08-N.C.
[Ans.: If difference in actual and standard time is also considered for calculating the lower efficiency then
Total cost: `1,02,69,708; If lower efficiency of 1% is based on budgeted efficiency then Total Cost:
`1,01,38,652; If lower efficiency of 1% is based on actual efficiency then Total Cost: `1,02,68,696]
[Hint.: We need to specifically mention in exam the assumption we are following regarding labour efficiency.]
[Note.: In Suggested answers by ICAI, while doing solution ICAI have assumed that decrease in efficiency
means reduction in time for producing same units although we shall goes by real meaning of decrease in
efficiency i.e. decrease in productivity/output & if we want to know about the increase in time for same amount
of output, then we can definitely derive it. (This logic is also substantiated by a solution by ICAI in a similar
question in Study material of ICAI & also in Nov’95). In other words Reduction in Labour efficiency by 1%
doesn’t mean that you will be able to produce same amount of units (i.e. 30000) in 101% of original labour
time, rather, it means that we will be able to make 29700 (30000×99%) units in original labour time i.e.75000
hours or in other words we can produce 30000 units (units to be produced) in 101.01% of original labour time
i.e. 75758 hours approx.]
Question 37 [Preparation of monthly cash budgets and budgeted income statement] On 30th
September, 1990, the Balance Sheet of Melodies Pvt. Ltd. retailers of musical instruments was as under:

Ordinary Shares of `10 each `20,000 Equipment (at cost) `20,000


fully paid Less: Depreciation 5,000
Reserves and Surplus 10,000 15,000
Trade creditors 40,000 Stock 20,000
CA. Parag Gupta Ph.: +91 11 47665555 Paraggupta_ca@yahoo.co.in Costing & O.R.
World’s largest CA Final student’s consultancy group: http://groups.yahoo.com/group/costingbyparaggupta
Budget & Budgetary Control - 148 -
(incld. Limiting Factors, Product mix decisions,
Throughput A/cing & Synchronous Manufacturing)
Proposed Dividend 15,000 Trade Debtors 15,000
Balance at Bank 35,000
85,000 85,000

The company is developing a system of forward planning, and on 1st October, 1990 it supplies the following
information:

Month Credit Sales Cash Sales Credit Purchases


September 1990 (actual) `15,000 `14,000 `40,000
October 1990 (Budget) 18,000 5,000 23,000
November 1990 (Budget) 20,000 6,000 27,000
December 1990 (Budget) 25,000 8,000 26,000

All trade debtors are allowed one month’s credit and are expected to settle promptly. All trade creditors are
paid in the month following delivery.

On 1st October 1990,all the equipment was replaced at a cost of `30,000.`14,000 was allowed in exchange
for the old equipment and a net payment of `16,000 was made. Depreciation is to be provided at the rate of
10% per annum.

The proposed dividend will be paid in December 1990

The following expenses will be paid:


Wages `3,000 per month
Administration `1,500 per month
Rent `3,600 for the year to 30th September 1991 (to be paid in October 1990).

The gross profit percentage on sales is estimated at 25%

You are required:


(a) to prepare a Cash budget for the month of October, November and December.
(b) To prepare Income Statement for the three months ended 31st December. Nov./90

[Ans.: (a) Closing Balance (Overdraft) – October : (9100), November : (12600), December : (31100); (b) Net
profit : `13350]
Question 38 [Production Budget]: Sportswear Ltd. manufactures sportswear shirts and shorts. The
production budget for these two products has to be prepared for the next three months, November 2010,
December 2010 and January 2010.

The following information is given :


(i) Sales volume every month will be 2% more than the previous month's volume for each product.
(ii) The company carries stock of finished garments sufficient to meet 40% of the next month's sale.
(iii) Closing stock for October 2010 was 6000 shirts and 8000 shorts.

You are required to prepare the production budget for each product for November, December 2010 and
January 2011. (5 Marks) Nov./10-O.C.
[Ans.: Shirts: 15120, 15422, 15731; Shorts: 20160, 20563, 20975]
Question 39 [Production Budget & EOQ]: A firm is engaged in the manufacture of two products ‘A’ and ‘B’.
Product A used one unit of component ‘P’ and two units of components ‘Q’. Product B uses two units of
component ‘P’, one unit of component ‘Q’ and two units of component ‘R’. Component ‘R’ which is assembled
in the factory uses one unit of component ‘Q’. Components ‘P’ and ‘Q’ are purchased from the market.
The firm has prepared the following forecast of sales and inventory for the next year.

Products
A B
Sales Units 8,000 15,000
Inventories:
At the end of the year Units 1,000 2,000
CA. Parag Gupta Ph.: +91 11 47665555 Paraggupta_ca@yahoo.co.in Costing & O.R.
World’s largest CA Final student’s consultancy group: http://groups.yahoo.com/group/costingbyparaggupta
Cost Accounting & Management - 149 -

At the beginning of the year Units 3,000 5,000


The production of both the products and the assembling of the component ‘R’ will be spread out uniformly
throughout the year.
The firm at present orders its inventory of components ‘P’ and ‘Q’ in quantities equivalent to 3 months’
consumption. The firm has been advised that savings in the provisioning of components can arise by
changing over to the ordering quantities. The firm has compiled the following data relating to the two
Components:

P Q
Price per unit ` 2.00 0.80
Order placing costs per order ` 15.00 15.00
Carrying costs p.a. 20% 20%

Required:
a) Prepare a budget of production and requirements of components for the next year.
b) Find the economic order quantity.
c) Based on the economic order quantity calculated in (b) above, calculate the savings arising from
switching over to the new ordering system both in terms of cost and reduction in working capital.
(May 1989), (11 Marks) May/06-Adapted & (7 Marks) May/10-N.C.-Adapted
[Ans.: (a) Production: A-6000 units, B-12000 units; (b) P – 1500 components, Q – 3000 components]
Question 40 [Functional Budgets]: A Company manufactures two Products A and B by making use of two
types of materials, viz., X and Y. Product A requires 10 units of X and 3 units of Y. Product B requires 5 units
of X and 2 units of Y. The price of X is `2 per unit and that of Y is `3 per unit. Standard hours allowed per
product are 4 and 3, respectively. Budgeted wages rate is `8 per hour. Overtime premium is 50% and is
payable, if a worker works for more than 40 hours a week. There are 150 workers.

The Sales Manager has estimated the sales of Product A to be 5,000 units and Product B 10,000 units. The
target productivity ratio (or efficiency ratio) for the productive hours worked by the direct worker in actually
manufacturing the product is 80%, in addition, the non-productive downtime is budgeted at 20% of the
productive hours worked. There are twelve 5-day weeks in the budget period and it is anticipated that sales
and production will occur evenly throughout the whole period.

It is anticipated that stock at the beginning of the period will be:

Product A 800 units; Product B 1,680 units. The targeted closing stock expressed in terms of anticipated
activity during the budget period are Product A 12 days sales; Product B 18 days sales. The opening and
closing stock of raw material of X and Y will be maintained according to requirement of stock position for
Product A and B.

You are required to prepare the following for the next period:

(i) Material usage and Material purchase budget in terms of quantities and values.
(ii) Production budget.
(iii) Wages budget for the direct workers. (8 Marks) Nov./04 & May/95-Adapted

[Ans.: (i) Material Purchase Budget: X - 117200 units, Y - 41480 units, Total cost of materials-`358840,
Material Usage Budget: X-108600 units at `217200; Y-38240 units at `114720 (ii) Production Budget: Product
A – 5200 units, Product B – 11320 units (iii) Wages Budget: Total wages `697680]
Question 41 [Sales, Production, Material Usage, Purchase and Manpower Budgets]: ZED Ltd.
manufactures three types of products, A,B and C and markets them at `450, `550 and `650 per unit
respectively. The current ratio of sales in quantity of A, B and C is 1: 2: 4.
Relevant Data of A, B, and C (per unit)
Product Quantity of parts required therein Labour hours Variable Overheads
Frame E F G Skill Unskilled `
A 1 10 2 8 6 8 9
B 1 2 14 10 4 6 11
C 1 6 10 2 3 6 7

CA. Parag Gupta Ph.: +91 11 47665555 Paraggupta_ca@yahoo.co.in Costing & O.R.
World’s largest CA Final student’s consultancy group: http://groups.yahoo.com/group/costingbyparaggupta
Budget & Budgetary Control - 150 -
(incld. Limiting Factors, Product mix decisions,
Throughput A/cing & Synchronous Manufacturing)

The present purchase price per part is `45, `15, `15 and `5 for frame, E, F and G respectively. The wage rate
per hour for skilled and unskilled workers is `6 and `5 respectively.

The opening stocks are as on 1.11.1995 stood at 500, 1,000, 3,000, 1,500, 1,000, 20,000 and 10,000
respectively for A,B,C frames, E,F and G. The company maintains closing stock of products and parts at 90%
of the opening stocks.

The workers work for 8 hours a day for 25 days in a month.

The share of fixed overheads per month comes to `15,75,000; `5,80,000; and `8,45,000 for production,
administration and selling and distribution respectively.
The yearly profit as projected up to October,1996 is `120 lakhs.

You are required to present the following for November, 1995:


(a) Sales budget in quantity as well as in value for A, B and C.
(b) Production budget.
(c) Parts usage budget.
(d) Purchase budget in quantity as well as in value.
(e) Manpower budget showing labour hours and wages payable for both types of workers.
ICWA-June/95 & (19 Marks) Nov./95
[Ans.: (a) Sales value product A (2500 × `450), Product B (5000 × `550) & Product C (10000 × 650); (b)
Production budget (in units) Product A, B & C – 2450, 4900 & 9700 respectively; (c) Parts usage budget
(parts of total material required) Frame-17050, E-92500, F-170500, G-88000; (d) Purchase budget (in qty as
well as value) Frame-(16900 × `45), E-(92400 × `15), F-(168500 × `15), G-(82100 × `5); (e) Wages payable
to skilled worker `(63400 × `6) and unskilled worker (107200 × `5)]
Question 42 [Production, material purchase and direct wage cost budgets and budgeted income
statement]: X Ltd. produces and markets three products- Chairs, Tables and Benches. The company is
interested in presenting its budget for the next quarter ending 31st March,1991. It expects to sell 4,200 chairs,
800 tables and 500 benches during the said period at the selling price of `50, `85 and `158 per unit
respectively. The following information’s are made available for the purpose:

(i) Material and labour requirements:

Chairs Tables Benches


Timber per unit (in cu. Ft.) 0.50 1.2 2.5
Upholstery per unit (in sq. yds.) 0.25 - -
Carpenter’s time(minutes per unit) 45 60 75
Fixed and Finisher’s time(minutes per unit) 15 15 30

Timber costs `50 per cu. Ft. and upholstery costs `20 per sq. yd. fixing and finishing material costs 5% of the
cost of timber and upholstery. Carpenter gets `6 per hour while the fixer and finisher get `4.80 per hour.
(ii) Inventory levels planned:

Timber Upholstery Chairs Tables Benches


(cu. ft) (sq. yds.) (nos.) (nos.) (nos.)
Opening 600 400 400 100 50
Closing 650 260 200 300 50

(iii) Fixed overheads would be `8,000 per month.

You are required to:


(a) Prepare a production budget showing quantities to be manufactured.
(b) Prepare a raw materials purchase budget in quantities as well as in rupees.
(c) Draw a direct wage cost budget.
(d) Present a statement showing variable cost of manufacture per unit of all three products.
(e) Find out the budgeted net income for the said quarter. (Nov./93)

CA. Parag Gupta Ph.: +91 11 47665555 Paraggupta_ca@yahoo.co.in Costing & O.R.
World’s largest CA Final student’s consultancy group: http://groups.yahoo.com/group/costingbyparaggupta
Cost Accounting & Management - 151 -

[Ans.: (a) 4000, 1000, 5000; (b) 4500 cu ft of `225000, 860 sq yds of `17200; (c) `34950; (d) 37.20, 70.20,
141.15; (e) `50025]
Question 43 [Preparation of Production and Purchase Budget]: Manufacturers Ltd. produce three
products from three basic raw materials in three departments. The company operates budgetary control
system and makes its stock to finished goods on a total cost basis. From the following data, you are required
to produce for the month of July 1986 the following budgets.

(a) Production (b) Material usage


(c ) Purchases (d) Profit and loss account for each product and in total
A B C
Sales `15,00,000 `10,80,000 `16,80,000
Stock of finished products at July 1,1986 in units 3,000 2,000 2,500
Department
I II III
Production overhead `2,39,000 `2,01,300 `3,91,200
Direct labour hours 47,800 67,100 65,200
Direct Material
M1 M2 M3
Stock at July 1, 1986 in units 24,500 20,500 17,500

The company is introducing a new system of inventory control, which should reduce stock. The forecast is
that stocks as at 31st July 1986 will be reduced as follows:
Raw materials by 10% and finished product by 20%.

Fixed production overhead is absorbed on a direct labour-hour basis. It is expected that there will be no work-
in-progress at the beginning or end of the month. Administration cost is absorbed by products at a rate of 20%
of production cost and selling and distribution cost is absorbed by products at a rate of 40% production cost.

Profit is budgeted as a percentage of total cost as follows:


Product A 25%, Product B 12½% and Product C 16⅔%

Standard cost data per unit of products:


Product
Price per unit A B C
Direct material ` Units Units Units
M1 2.00 5 - 12
M2 4.00 - 10 9
M3 1.00 5 5 -

Rate per hour A B C


Direct wages: ` Hrs. Hrs. Hrs.
Department I 2.50 4 2 2
Department II 2.00 6 2 3
Department III 1.50 2 4 6
Other variable costs `10 `20 `15
(May/86)
[Ans.: (a) A – 6900 units, B – 4600 units, C – 5500 units; (b) M 1 – 100500 units, M 2 -95500 units, M 3 – 57500
units; (c) M 1 – 98500 units of `196100, M 2 - 93450 units of `373800, M 3 – 55750 units of `55750; (d) A -
`300000, B - `120000, C - `240000, D - `660000]

Responsibility Accounting
Each manager, regardless of level, is incharge of a responsibility center. A responsibility center is a segment
of an organization whose manager is accountable for a specified set of activities. The higher the manager’s
level, the broader the responsibility center and generally, the larger the no. of his subordinates. Responsibility
Accounting is a system that measures the plans (by budgets) and actions (by actual results) of each
responsibility center. Responsibility accounting is concerned with designing reports that help motivate
managers to make decisions and to take actions that are in the best interests of the overall organization. Four
major types of responsibility centers are:

CA. Parag Gupta Ph.: +91 11 47665555 Paraggupta_ca@yahoo.co.in Costing & O.R.
World’s largest CA Final student’s consultancy group: http://groups.yahoo.com/group/costingbyparaggupta
Budget & Budgetary Control - 152 -
(incld. Limiting Factors, Product mix decisions,
Throughput A/cing & Synchronous Manufacturing)
 Cost Centre - the manager is accountable for costs only. E.g. Maintenance Manager
 Revenue Centre - the manager is accountable for revenues only. E.g. Sales Manager
 Profit Centre - the manager is accountable for revenues & costs. E.g. Department/Segmental
Manager
 Investment Centre - the manager is accountable for investments, revenues & costs. E.g. Regional
Manager responsible for responsible for investment in new projects and for revenues & costs.

Type of responsibility Manager has control over Principal performance


center measurement
Cost centre Costs (only controllable cost items) Variance analysis
Efficiency measures
Profit centre Costs (only controllable costs) Profit
Sales prices (including transfer prices)
Output volumes
Investment centre Costs (only controllable costs) Return on investment
Sales prices (including transfer prices) Residual income
Output volumes Other financial ratios
Investment in fixed & current assets

Manager’s control over responsibility centre


Type of centre Manager responsible for
Costs Revenues Profit/Loss Investment
Cost Centre Yes No No No
Revenue Centre No Yes No No
Profit Centre Yes Yes Yes No
Investment Centre Yes Yes Yes Yes

Question 44 [Responsibility Accounting Reports + Budget Variance]: Nicefit manufactures ready made
garments by a simple process of cutting the clothes in various shapes and then sewing the corresponding
pieces together to form the finished product.

The sewing Department and the cutting department report to the production manager who along with
Engineering Manager reports to the Director-Manufacturing. The Sales Manager, Publicity Manager and the
Credit Manger report to the Director-Marketing, who along with Director-Manufacturing reports to the
Managing Director of the company.

The Accounts Department reports the following for the last quarter of 1983:

Budgeted (`) Actual (`)


Bad debt Losses 5,000 3,000
Cloth used 31,000 36,000
Advertising 4,000 4,000
Audit fees 7,500 7,500
Credit reports 1,200 1,050
Sales representative travelling expenses 9,000 10,200
Sales commission 7,000 7,000
Cutting Labour 6,000 6,600
Thread 500 450
Sewing Labour 17,000 18,400
Credit Deptt. Salaries 8,000 8,000
Cutting utilities 800 700
Sewing utilities 900 950
Director Marketing salaries & Admn. Exp. 20,000 21,400
Production engineering expenses 13,000 12,200
Sales management office expenses 16,000 15,700

CA. Parag Gupta Ph.: +91 11 47665555 Paraggupta_ca@yahoo.co.in Costing & O.R.
World’s largest CA Final student’s consultancy group: http://groups.yahoo.com/group/costingbyparaggupta
Cost Accounting & Management - 153 -

Production Manager office expenses 18,000 17,000


Director Mfg. Salaries & Admn. Expenses 21,000 20,100

Using the above data, prepare Responsibility Accounting reports for the director marketing, the Director-
manufacturing and the production manager.

[Ans.: Production Manager- Budget: `56200, Actual: `63100, Direct Manufacturing- Budget: `87200, Actual:
`92300, Direct Marketing- Budget: 50200, Actual: 48950]
Question 45 [Performance Budget & Summary Report]: The following data relate to a company which had
a profit approved for selling 5000 units per month at an average selling price of `10 per unit and budgeted
variable cost of production was `4 per unit and fixed costs were budgeted at `20,000. Planned income being
`10,000 per month. Because of shortage of raw—materials the plant could produce only 4000 units and the
cost of production was increased by 0.50 per unit. Consequently `1.00 raised the selling price per unit. To
modify production processes in order to meet materials shortage, the Company incurred an expenditure of
`1,000 in Research and Development. Set out a Performance budget and a summary report there.

[Ans.: Net Profit – Original Plan: `10000, Revised Budget: `4000, Actual Result: 5000]

CA. Parag Gupta Ph.: +91 11 47665555 Paraggupta_ca@yahoo.co.in Costing & O.R.
World’s largest CA Final student’s consultancy group: http://groups.yahoo.com/group/costingbyparaggupta
Transfer Pricing

A transfer price is the amount of money that one subunit (segment, department, division and so on) of an
organization charges for goods and services to another subunit of an organization.

The transfer price creates revenue for the selling subunit and purchase cost for the buying subunit, affecting
each subunit’s operating income. The product transferred between subunits of an organization is called an
intermediate product. It can either be processed further by receiving subunit or, if transferred from
production to marketing, resold to an external customer.

Organizations have a system of transfer pricing, therefore, in order to assess the efficiency and effectiveness
of its department and divisional managers. This maybe in spite of the fact that transfer prices may be artificial
in the sense that it is felt that there is no rationale for “selling” between departments and divisions.

Aims of a transfer pricing system


 To encourage goal congruence, whereby individual managers' own goals are the same as the
goals of the company as a whole.
 To enable the realistic measurement of divisional profit.
 To sustain high level of management efforts.
 To give autonomy to managers.
 To ensure profit maximization for the company as a whole.
It may be difficult to reconcile all of these aims.
Transfer Pricing Methods
Market-based transfer prices. When there is a competitive outside market for the good or service
transferred between the divisions, the market price is often used as a transfer price. This solution is
reasonably easy to administer and provides a theoretically correct transfer price when there is no idle
capacity. However, when there is idle capacity in the selling division, the transfer price will be too high and the
buying division may inappropriately purchase from an outside supplier or cut back on volume.

The major merits of this method are:


a) Maximum Prices: In a competitive market, goods/services cannot be transferred to its users at a higher
price. Hence market prices constitute the basis for efficient production.
b) Demand and Supply Forces: Market prices take into account, the forces of demand and supply. If
intermediate products are freely saleable, in the long run, market prices will provide a good indicator of
the overall efficiency of the various divisions.
c) Opportunity Cost Recovery: Opportunity costs of transferring divisions are fully recovered.
Hence there is sufficient incentive for internal transfer, for transferring divisions operating at full capacity.
d) Objective: Market prices provide reliable measures of divisional income because these prices are
established independently rather than by individuals who have an interest in the results.

The major demerits of this method are:


a) Availability of Market Prices: There may be difficulty in obtaining just / fair market prices.
Sometimes, the intermediate product may not be saleable; in other cases direct market substitutes may
not be available for products, which are manufactured only for internal consumption.
CA. Parag Gupta Ph.: +91 11 47665555 Paraggupta_ca@yahoo.co.in Costing & O.R.
World’s largest CA Final student’s consultancy group: http://groups.yahoo.com/group/costingbyparaggupta
Cost Accounting & Management - 155 -

b) Impact of S&D Costs: There may be difficulties in determining the elements of Selling and Distribution
costs such as commission, discounts, advertisement and sales promotion etc. so that necessary
adjustment may be made in the market price to provide benefit of these expenses, to the recipient
Division.
c) Unjust Enrichment: Market Prices lead to unjust enrichment of the transferring division, particularly if
the former has sufficient spare capacity and the intermediate is not freely saleable externally.

Negotiated transfer prices (a.k.a. Shadow Pricing). It refers to the determination of transfer prices based
on active participation, involvement, co-ordination and agreement of the managers of the transferring and
recipient division. In principle, if division managers understand their own businesses and are cooperative,
negotiated transfer prices should work quite well. It is always possible in such a situation (barring
externalities) to find a transfer price that would increase each participating division’s profits. While negotiated
transfer prices can work quite well under the right conditions, if managers are uncooperative and highly
competitive, negotiations may go nowhere.

Advantages:
1. Proper Decision Making: Negotiated prices lead to business like attitude amongst divisions of the
company. The buying division may purchase from outside sources if the outside prices are lower than
the internal division’s price.
2. Autonomy and Motivation Value: Each sub-unit is considered as an independent unit.
Buyers and Sellers are completely free to deal outside the Company. This promotes sub-unit
autonomy and motivates managers.
3. Overall Company Profitability: Through properly direct negotiations, managers will be able to
determine the appropriate transfer prices that satisfy the requirements of the divisions and is in the
best interest of the Company as a whole.

Limitations:
1. Sub-optimal: The agreed transfer price may depend on the negotiating skills and bargaining powers
of the managers involved. The final result may not always be optimal.
2. Conflicts: Rather than agreement on transfer prices, negotiations can lead to conflict between
divisions and may require top-management mediation.
3. Defeat of Performance evaluation criteria: Transfer prices dependent on manager’s negotiation
skills will defeat the very purpose of performance evaluation.
4. Time and Cost: Negotiations are time consuming for the manager involved, particularly when the
number of transactions and interdependencies are large.

In order to have an effective system of transfer pricing; the following points should be kept in view;
 Price of all transfer in and out of a profit centre should be determined by negotiation between the
buyer and the seller.
 Negotiations should have access to full data on alternative source and markets and to public and
private information about market prices.
 Buyers and sellers should be completely free to deal outside the company.

Cost-based transfer prices. Transfer Price can be recorded at:


(i) Variable costs (ii) Full manufacturing cost (iii) Total Actual cost (iv) Standard Cost or at (v) Variable or Full
cost plus some arbitrary mark-up.
 Variable Manufacturing Costs:

Merits Demerits
 Simple to understand.  Cost and Prices fluctuate from time to
 Easy To Operate time, Hence Transfer prices also vary.
 No Negative Contribution to Transferring  No Contribution from transferring
Division division, since only manufacturing cost is
 Suitable if Transferring Division has recouped.
sufficient spare capacity and intermediate  Not suitable for performance evaluation.
product is not marketable.  Does not consider opportunity costs and
losses.
 Not suitable for transferring divisions
which operate at full capacity.
CA. Parag Gupta Ph.: +91 11 47665555 Paraggupta_ca@yahoo.co.in Costing & O.R.
World’s largest CA Final student’s consultancy group: http://groups.yahoo.com/group/costingbyparaggupta
Transfer Pricing - 156 -
(including Profitability analysis)

 Full Manufacturing Cost:

Merits Demerits
 Simple to understand.  Cost and Prices fluctuate from time to
 Easy to Operate. time, Hence Transfer prices also vary.
 Guaranteed Contribution to transferring  Not suitable for performance evaluation.
division, to the extent of fixed  Does not consider opportunity costs and
manufacturing cost. losses.
 Ideal if transferring division has sufficient  Not suitable for transferring divisions
spare capacity, and intermediate product is which operate at full capacity.
not marketable.

 Total Actual Costs:

Merits Demerits
 Simple to understand.  Actual Cost and Prices fluctuate from
 Easy to Operate. time to time,
 Guaranteed Contribution to transferring  Not suitable for performance evaluation
division, to the extent of fixed cost. Usable of transferring division.
if transferring division has sufficient spare  Does not Consider Opportunity costs
capacity, and intermediate product is not and losses.
marketable.  Not suitable for transferring divisions
which operate at full capacity.
 Cost efficiency of transferring division
may mean lower prices, hence no
incentive for cost reduction.

 Standard Costs:

Merits Demerits
 Simple and Easy to operate when  The Transferring divisions usually
compared with actual cost based methods. absorb variances and hence segment-
 Inventories are carried at standard costs in wise performance evaluation is not
transferring and receiving divisions. possible.
 Does not Consider Opportunity costs
and losses.

 Cost Plus mark-up: Under this method, the transfers are made at Cost + Mark-up basis. Cost may be
any variant i.e. Variable Manufacturing Costs or Full Manufacturing Costs or Total Cost (Actual) or
Standard Costs.
Mark-up added to cost may be expressed either (1) as a percentage of full cost or (2) as a percentage of
capital employed.

Total Actual cost of the product includes all production costs plus cost from other business
functions (R & D, design, marketing, distribution and customer service).

Dual Pricing: The dual pricing method uses two prices. The supplying division is credited with a price based
on total cost plus a mark- up and the receiving division is debited with marginal cost. This means that the

CA. Parag Gupta Ph.: +91 11 47665555 Paraggupta_ca@yahoo.co.in Costing & O.R.
World’s largest CA Final student’s consultancy group: http://groups.yahoo.com/group/costingbyparaggupta
Cost Accounting & Management - 157 -

selling division is allowed to earn a profit and the receiving division has the correct information in order to
make the correct selling decision to maximize the group’s profit. The difference between the two prices will be
debited to a group account- a transfer price adjustment account. At the end of the year the profits of the two
divisions, and hence of the group, will be overstated to the extent of the price difference. In order to correct
this, the total amount in the transfer price adjustment account must be subtracted from the two profits to arrive
at the correct profit for the group as a whole.

Dual pricing can also be used with market price in place of marginal cost for the receiving division. This can
aid the supplying division in a particular circumstance. For example, where market prices are very volatile and
the market price of the component suddenly collapses it may be unrealistic to expect the supplying division to
cope with the decreases. Under the circumstances the receiving division would wish to buy elsewhere if the
transfer price set was higher than the market price. So the supplying division could be credited with total cost
plus and the receiving division debited with the much reduced market price. The receiving division would then
be happy to continue to buy internally.

The dual pricing method can be effective in avoiding sub-optimal decisions but it can be administratively
cumbersome.

Advantages:
1. Incentive to Transferring Division: The transfer price will meet the performance evaluation
requirements of the transferring division since each unit transferred generates a profit (due to mark-
up). Thus the supplying division manger is motivated to transfer the intermediate product internally.
2. No unjust Enrichment: If the transfer price is set at the transferring division’s marginal cost of the
intermediate product, it will not have any contribution from the internal transfers. All the total
contribution from inter-divisional trading will be assigned to the Recipient division. Such unjust
enrichment is avoided through the use of mark-up.
3. Optimal Decisions: Since relevant cost analysis is used as the second transfer price, the transfer
pricing system automatically promotes goal congruence by leading to optimal decisions.

Disadvantages:
1. Confusing: The use of different transfer prices causes confusion, particularly when more than two
divisions are involved.
2. Artificial: Dual Transfer Prices are considered to be artificial.
3. No incentive: Fixed price with mark-up protects transferring divisions from competition. It reduces
divisional incentives to compete effectively and give them little incentive to improve their
productivity.
4. Misleading: Dual Transfer prices can result in misleading information and create a false impression
of divisional profits. There is a possibility of double-counting of profits. At the extreme, all divisions
may report profits when the company as a whole is losing money.

Two-part transfer pricing: To avoid the sub-optimal decisions that may occur when the fixed costs of one
division are perceived as variable costs by another division, a two part transfer price might be used.
Under this method, Transfer price = Marginal Cost + Lump-sum Fixed Fee
 This method is most suited when there is no market for the intermediate product, and the transferring
division has no capacity constraints.
 The Transferring Division is provided with sufficient incentive for internal transfer, since marginal costs
are fully recovered and the lump-sum fee received will reduce its losses by recovering fixed costs.
 The Recipient Division is also interested in the internal procurement since the transfer price will be less
than market price or cost of alternative option like outsourcing etc.
 This system also has a drawback the supplying division has no incentive to supply units swiftly, as
each unit does not generate a profit. The profit is made whenever the fixed fee is transferred.

Moreover, the lump-sum fixed fee constitutes a commitment if the divisions to utilize a portion of the
capacity of the transferring divisions, for an agreed compensation.

General guidelines for setting Transfer Prices:

Minimum Transfer Price: The selling division would like for the transfer price to be as high as possible. The
manager of selling division will not agree to a transfer price that is less than his or her “cost.” But what cost? If

CA. Parag Gupta Ph.: +91 11 47665555 Paraggupta_ca@yahoo.co.in Costing & O.R.
World’s largest CA Final student’s consultancy group: http://groups.yahoo.com/group/costingbyparaggupta
Transfer Pricing - 158 -
(including Profitability analysis)

the manager is rational, then the manager should realize that any transfer price that covers variable cost plus
fixed costs specific to such decision plus opportunity cost will result in an increase in segment profits. The
opportunity cost is the contribution margin that is lost on units that cannot be produced and sold as a result of
the transfer. Therefore, the lowest acceptable transfer price as far as the selling division is concerned is:
When there is idle capacity, there are no lost sales and so the total contribution margin of lost sales is zero.

Transfer Price ≥ Incremental Costs + Total contribution margin of lost sales


(i.e. Variable cost + Specific Fixed costs Total number of units transferred
to such decision)

Also Packaging Cost, Bad Debts, Selling & Distribution costs are not included (except specifically
mentioned).

Maximum Transfer Price: The buying division clearly would not voluntarily agree to a transfer price (subject
to his ability to pay) unless:
Transfer price ≤ Cost of buying from outside supplier (i.e. Market Price plus buying cost incurred)

Domestic Transfer Pricing recommendations


1) Where a competitive market exists for intermediate products & there is no idle capacity, the market
price (less adjustments for selling and distribution, packaging cost, bad debts, & collection expense to
outside customers) should be used as transfer price.

2) Where a competitive market exists for intermediate products & there is idle capacity, variable cost
should be transfer price.

3) Where no external market exists for the intermediate product, transfers should be made at two-part
transfer pricing basis.

4) Where an imperfect market for intermediate product and a small no. of products, transactions are
involved, a negotiated transfer pricing is most suitable method.

5) Where a cost-based transfer prices are used standard costs, and not the actual costs, per unit of
output should be used. If actual costs are used the selling division will be able to pass on the cost of
any inefficiency to the buying division. Using standard costs ensures that costs of inefficiencies are
borne by the selling division.

Multinational Transfer Pricing and Maximization of income through transfer pricing mechanism :
Transfer Pricing Policies of domestic companies forecast on “Goal Congruence” and “Motivation”. In
Multinational companies other factors may dominate, e.g.; Transfer Prices are used to minimize world wide
“Income Tax” and “Import Duties”.

Many organizations have divisions located in different countries. Deciding on a set of transfer prices to use
for exchange between these divisions requires consideration of additional factors. These factors are -

1) Income Tax Rates: Items produced by divisions in a low Income tax rate country may be transferred to a
division of a high Income tax Rate country at a high transfer price to minimize taxes.

2) Import Duties: Sometimes Income tax effect is offset by Import Duty. Products may be transferred into high
duty countries at an artificially low transfer price so that, assuming duty is charged ad valorem, the duty paid
will be low.

3) Income repatriation: Some countries restrict the repatriation of incomes or dividends. By increasing the
price of goods or services transferred into divisions in these countries, firms can increase the funds
repatriated out of these countries without violating income or dividend, restriction.

4) Inflation: It is possible to avoid an accumulation of fund in a country from which high inflation rates or where
an early devaluation is thought to be a probability.

CA. Parag Gupta Ph.: +91 11 47665555 Paraggupta_ca@yahoo.co.in Costing & O.R.
World’s largest CA Final student’s consultancy group: http://groups.yahoo.com/group/costingbyparaggupta
Cost Accounting & Management - 159 -

5) Penetrating a new market: Transfer prices may be set low for an affiliate that is trying to establish a
competitive advantage over a local company.

Question 1: In transfer pricing what is common conflict between a division and the company as a whole.
OR (6 Marks) May/97
“Transfer pricing is a widely debated and contested topic” – Discuss (5Marks) Nov./99
Ans.: Usually a conflict between a division of the company and the company as a whole is faced by the
management of decentralized units when products or services are exchanged among different divisions of the
company. Such a conflict becomes more significant in the case of those concerns where profitability is used
as criteria for evaluating the performance of each division.

The essence of decentralisation is reflected in the freedom to make decisions. Under such set up it is
expected that the top management should not interfere with the decision making process of its subordinates
heading different units. In other words, management of decentralized units is given autonomy with regard to
decision-making. In this system top management is expected to preserve ‘autonomy in decision making’. The
management of such companies also expects that each division should not only achieve its own objective
necessary for evaluating the performance but should also achieve the objective of goal congruence.

A divisional head in a company under aforesaid set up is free to use a price as a transfer price for goods and
services, which may provide incentive. Such a transfer price may fail to achieve the objective of ‘Goal
congruence’ (which means a perfect congruence between division’s goal and the goal of the company). In
case of failure of a division to achieve the objective of ‘goal congruence’ the management of the company
may ‘dictate their ‘transfer price’. Such an interference of management of the company is usually the main
basis of conflict between a division and the company as a whole.

Further this conflict is aggravated if the management advocates the transfer of goods and services at cost. As
such the transfer price will not reflect a good: picture about the performance of the transferring division. The
profitability of the transferring division will not be known by the use of such a transfer price.

Each division appreciates the transfer of its goods/services at usual selling price/market price so as to arrive
at the correct return/profitability figure, used for measuring the performance. There is no incentive to the
transferring division if goods and services are transferred at variable cost.

Question 2: Enumerate and briefly explain any three methods of determining transfer price.
(6 Marks) May/98
Question 3: What should be the basis of transfer pricing, if unit variable cost and unit selling price are not
constant? (4 Marks) Nov./99
Ans.: If unit variable cost and unit selling price were not constant then the main problem that would arise
while fixing the transfer price of a product would be as follows:

There is an optimum level of output for a firm as a whole. This is so because there is a certain level of output
beyond which its net revenue will not rise. The ideal transfer price under these circumstances will be that
which will motivate these managers to produce at this level of output.

Essentially, it means that some division in a business house might have to produce its output at a level less
than its full capacity and in all such cases a transfer price may be imposed centrally.
Question 4: What will be the marketable transfer pricing procedure regarding the goods transferred under the
following conditions (each condition is independent of the other)?
(i) When division are not captives of internal divisions and the divisions are free to do business both internally
and externally and when there are reasonably competitive external markets for the transferred products.
(ii) If the external market for the transferred good is not reasonably competitive. (3 Marks) Nov./00

b) Discuss the potential for maximization of income by a multinational through the use of transfer pricing
mechanism. (3 Marks) Nov./00
Question 5: Indicate the possible disadvantages of treating divisions as profit centres. (4 Marks) Nov./01
Ans.: The Possible disadvantages of treating divisions as profit centres are as follows:

CA. Parag Gupta Ph.: +91 11 47665555 Paraggupta_ca@yahoo.co.in Costing & O.R.
World’s largest CA Final student’s consultancy group: http://groups.yahoo.com/group/costingbyparaggupta
Transfer Pricing - 160 -
(including Profitability analysis)

 Divisions may compete with each other and may take decisions to increase profits at the expense of
other divisions thereby overemphasizing short term results.
 It may adversely affect co-operation between the divisions and lead to lack of harmony in achieving
organizational goals of the company. Thus it is hard to achieve the objective of goal congruence.
 It may lead to reduction I the company’s overall total profits.
 The cost of activities, which are common to all divisions, may be greater for decentralized structure
than centralized structure. It may thus result in duplication of staff activities.
 Top management looses control by delegating decision making to divisional managers. There are
risks of mistakes committed by the divisional managers, which the top management, may avoid.
 Series of control reports prepared for several departments may not be effective from the point of view
of top management.
 It may under utilize corporate competence.
 It leads to complications associated with transfer pricing problems.
 It becomes difficult to identity and defines precisely suitable profit centres.
 It confuses division’s results with manager’s performance.
Question 6: What are some goals of a ‘transfer-pricing’ system in an organization? (4 Marks) May/06

Question 7: Briefly describe the different methods of Transfer Pricing. (4 Marks) Nov./05

Question 8: Division A Division B


`per unit `per unit
Variable cost 10 15
Transfer price at market value – 20
Fixed costs 5 10
Profit 5 25
Transfer price/selling price 20 70
Division A can sell externally at `20 per unit or transfer internally to Division B at `20 per unit.
Division B receives an offer from a customer of `30 per unit for its final product.

Requirements
(a) Would Division B accept the offer of `30 per unit given the existing transfer price?
(b) Is this the correct decision from the company's point of view
(i) Division A has surplus capacity?
(ii) Division A is operating at full capacity?
Give proper explanations.

[Ans.: (a) No, there is a negative contribution of `5 per unit. (b) (i) No, co. could earn a contribution of `5 per
unit. (b) (ii) Yes, there is a negative contribution of `5 per unit, There would be goal congruence and the
manager of division B would not make a sub-optimal decision.]

Question 9: AB Cycles Ltd. has 2 divisions, A and B which manufacture bicycle. Division A produces bicycle
frame and Division B assembles rest of the bicycle on the frame. There is a market for sub-assembly and the
final product. Each division has been treated as a profit centre. The transfer price has been set at the long-
run average market price. The following data are available to each division:

Estimated selling price of final product `3,000 p.u.


Long run average market price of sub-assembly `2,000 p.u.
Incremental cost of completing sub-assembly in division `1,500 p.u.
B
Incremental cost in Division A `1,200 p.u.

Required:
(i) If Division A’s maximum capacity is 1,000 p.m. and sales to the intermediate are now 800 units, should 200
units be transferred to B on long-term average price basis.

CA. Parag Gupta Ph.: +91 11 47665555 Paraggupta_ca@yahoo.co.in Costing & O.R.
World’s largest CA Final student’s consultancy group: http://groups.yahoo.com/group/costingbyparaggupta
Cost Accounting & Management - 161 -

(ii) What would be the transfer price, if manager of Division B should be kept motivated?
(iii) If outside market increases to 1,000 units, should Division A continue to transfer 200 units to Division B or
sell entire production to outside market? (9 Marks) May/05

[Ans.: (i) 800 units should be sold as sale of intermediary and 200 units should be transferred to B (ii) `1350;
(iii) Full quantity should be sold outside as intermediary]

Question 10: A large business consultancy firm is organized in to several divisions. One of the divisions is
the Information Technology (IT) division which provides consultancy services to its clients as well as to the
other divisions of the firm. The consultants in the IT division always works in a team of three professional
consultants on each day consulting assignment. The external are to be charged a fee at the rate of `4500 for
each consulting day. The fee represents the cost plus 150% profit mark up. The break up of the cost involved
in the consultancy fee is estimated at 80% as being variable and balance is being fixed.

The textile division of the consultancy firm which has undertaken a big assignment requires the services of
two teams of IT consultants to work five days in a week for a period of 48 weeks. While the director of the
textile division intends to negotiate the transfer price for the consultancy work, the director of IT division
proposed to charge the textile division at `4500 per consultancy day.

In respect of the consultancy work of the textiles division, IT division will be able to reduce the variable cost by
`200 per consultancy day. This is possible in all cases of internal consultations because of the use of
specialized equipment.

You are required to explain the implication and set transfer prices per consulting day at which the IT division
can provide consultancy services to the textiles division such that the profit of the business consultancy firm
as a whole is maximized in each of the following scenarios:

(i) Every team of the IT division is fully engaged during the 48 week period in providing consultancy
services to external clients and that the IT division has no spare capacity of consultancy teams to
take up the textiles division assignment.
(ii) IT division will be able to spare only one team of consultants to provide services to the textile
division during the 48 week period and all other teams are fully engaged in providing services to
external clients.
(iii) A new external client has come forward to pay IT division a total fee of `1584000 for engaging the
services of two teams of consultants during the aforesaid period of 48 weeks.

[Ans.: (i) `4300 for each consulting day; (ii) `4300 for each consulting day for Ist team & `1240 for each
consulting day for IInd team i.e. `2770 (average of 4300 & 1240)(iii) Transfer Price should be `3100 (3300-
200) per consulting day] (11 Marks) Nov/08-Old Course
[Hint: In part (iii) suggested answers has assumed that both teams were idle]
Question 11: PH Ltd., has two manufacturing departments organized into separate profit centers known as
the Basic unit and processing unit. The Basic unit has a production capacity of 4,000 tonnes per month of
chem vax but at present its sales are limited to 2,000 tonnes to outside market and 1,200 tonnes to the
processing unit.

The transfer price for the year 2003 was agreed at `400 per tonne. This price has been fixed in line with the
external wholesale trade price on 1st January, 2003. However, due to heavy competition the basic unit has
been forced to reduce the wholesale trade price to `360 per tonne with effect from 1st June, 2004. This price,
however, was not made applicable to the sales made to the processing unit of the company. The processing
unit applied for revision of the price as applicable to the outside market buyers as from 1st June, 2004 but the
same was turned down by the basic unit.

The processing unit refines chem. vax and packs the output known as colour-X in drums of 50 kg. each. The
selling price of colour-X is `40 per drum. The processing unit has a potential of selling a further quantity of
16,000 drums of colour-X provided the overall price is reduced to `32 per drum. In that event it can buy the
additional 800 tonnes of chem vax from the basic unit whose capacity can be fully utilized. The outside market
will not however absorb more than the present quantity of 2,000 tonnes.

CA. Parag Gupta Ph.: +91 11 47665555 Paraggupta_ca@yahoo.co.in Costing & O.R.
World’s largest CA Final student’s consultancy group: http://groups.yahoo.com/group/costingbyparaggupta
Transfer Pricing - 162 -
(including Profitability analysis)

The cost data relevant to the operation are:

Particulars Basic unit Processing unit


` `
Raw Material / tonne 70 Transfer Prices
Variable costs / tonne 140 170
Fixed costs 3,00,000 1,20,000

Required:
i. Prepare statements showing the estimated profitability for June, 2004 for each unit and the company
as a whole on the following basis:
(a) At 80% and 100% capacity utilization of the basic unit at a market price and transfer price to the
processing unit of `400 per tonne.
(b) At 80% capacity utilization of the basic unit at the market price of `360 per tonne and the transfer
price to the processing units of `400 per tonne.
(c) At 100% capacity utilization of the basic unit at the market price and transfer price to the
processing unit of `360 per tonne.
ii. Comment on the effect of the company’s transfer pricing policy on the profitability of the processing
unit.

[Ans.: (i) (a) At 80% `464000 At 100% `480000 (b) `384000 (c) `400000]
Question 12: Tycon Ltd. has two manufacturing departments organised into separate profit centres known as
Textile unit and Process House. The Textile unit has a production capacity of 5 lacs metres cloth per month,
but at present its sales is limited to 50% to outside market and 30% to process house.

The transfer price for the year 2004 was agreed at `6 per metre. This price has been fixed in line with the
external wholesale trade price on 1st January, 2004. However, the price of yarn declined, which was the raw
material of textile unit, with effect, that wholesale trade price reduced to `5.60 per metre with effect from 1st
June, 2004. This price was however not made applicable to the sales made to the processing house of the
company. The textile unit turned down the processing house request for revision of price.

The Process house refines the cloth and packs the output known as brand Rayon in bundles of 100 metres
each. The selling price of the Rayon is `825 per bundle. The process house has a potential of selling a further
quantity of 1,000 bundles of Rayon provided the overall prices is reduced to `725 per bundle. In that event it
can buy the additional 1,00,000 metres of cloth from textile unit, whose capacity can be fully utilised.
The outside market has no further scope.

The cost data relevant to the operations are:


Textile unit ` Process house `
Raw Material (per meter) on 1st 3.00 Transfer Price
June 2004
Variable Cost 1.20 (per metre) 80 (per bundle)
Fixed Cost (per month) 412000 100000

You are required to:

(i) Prepare statement showing the estimated profitability for June, 2004 for Textile unit and Process house
and company as a whole on the following basis:
(a) At 80% and 100% capacity utilisation of the Textile unit at the market price and the transfer price to the
Processing house of `6 per metre.
(b) At 80% capacity utilisation of the Textile unit at the market price of `5.60 per meter and the transfer price
to the Processing house of `6 per meter.
(c) At 100% capacity utilisation of the Textile unit at the market price of `5.60 per metre and the transfer price
to the Processing house of `5.60 per metre.
(ii) Comment on the effect of the company’s transfer pricing policy on the profitability of Processing house.

CA. Parag Gupta Ph.: +91 11 47665555 Paraggupta_ca@yahoo.co.in Costing & O.R.
World’s largest CA Final student’s consultancy group: http://groups.yahoo.com/group/costingbyparaggupta
Cost Accounting & Management - 163 -

[Ans.: (i) (a) At 80% `308000, 117500, 425500; At 100% `488000, 12500, 505500 (b) `208000, 117500,
325500 (c) `288000, 112500, 400500 (ii) Processing house will not be interested in buying more than 150000
metres from Textile Unit.] (11 Marks) Nov/04
Question 13: Vivek Ltd., has two manufacturing divisions AD and CD. Each division operates as an
independent profit centre.

AD which produces two components BRITE and LITE has a capacity of 1,00,000 hours per annum. The
annual fixed overheads of this department amounts to `20 lacs. The product wise variable cost data are as
under:

Particulars Brite Lite


Direct materials 10 5
Direct labour and variable overheads 140 35
Total 150 40
The direct labour and variable overhead rate is `35 per hour.

AD has a permanent customer for the purchase of 15,000 units of BRITE per annum at a selling price of `300
per unit. The balance capacity is devoted to the production of LITE for which there is an unlimited sales
potential at `60 per unit.

CD assembles a product known TITE by using an imported component. The annual fixed overheads of this
division amount to `4 lakhs and the variable cost data per unit are as under:
TITE
`/Unit
Imported component 300
Direct materials 40
Direct labour and variable overheads (10 hours @ `25) 250
Total 590
The selling price of TITE is `700 per unit.

With a view to minimizing the dependence on imported components, the possibility of using the company’s
own component BRITE, which is similar to the imported component, was explored. The import substitution is
possible with slight modification in the manufacture of TITE which in that case will take two extra labour hours
per unit. This means an increase of `50 in variable costs per unit of TITE. CD envisages a production of 5,000
units per annum of TITE. You are required to present the division wise profitability and the profitability of the
company as a whole on the basis of the following conditions:

i. CD imports its requirement of 5,000 components for the manufacture of TITE.


ii. CD stops import and substitutes BRITE by drawing 5,000 units of BRITE from AD at the market price
of `300 per unit.
iii. Same situation as in (ii) above except that CD gets a relief of `50 per unit (net transfer price to CD is
`250 per unit) of BRITE to compensate the increased labour and variable overhead cost of CD.
iv. CD revises its production programme to manufacture 12,000 units of TITE by drawing 10,000 units of
BRITE from AD at `250 per unit and import the balance of 2,000 units of components at `300 per
unit. Due to installation of additional production capacity, the annual fixed overhead of CD would
increase by `7,70,000 In order to induce CD to the expansion programme, do you think a negotiated
transfer price of `240 for BRITE would be agreed by AD? Give reasons and also comment on the
best alternative (i) to (iv) for the company as a whole.

[Ans.: Profit of Co. in Alt I: `1200000; Alt II:1300000; Alt III: 1300000; Alt IV: 1400000]
Question 14: A large public sector company has several manufacturing divisions. Two of these are AJ and
DJ each of which sells most its output to customers outside the group divisions. AJ division operates at full
capacity. It can in other words expand its production capacity only at an extra cost. DJ on the other hand
operates at present 50% capacity.

DJ is actively seeking profitable ways to utilize its idle capacity, The management of DJ has been able to
secure an order for 2,000 units of its product ‘M’ to be delivered over a period of a year at a price of `105

CA. Parag Gupta Ph.: +91 11 47665555 Paraggupta_ca@yahoo.co.in Costing & O.R.
World’s largest CA Final student’s consultancy group: http://groups.yahoo.com/group/costingbyparaggupta
Transfer Pricing - 164 -
(including Profitability analysis)

each. DJ can meet this deliver schedule easily and the estimated breakup of cost of this item per unit is an
under. (`)
Components purchased from outside Market 45
Part No.35 purchased from AJ 10
Factory variable overheads costs 28
Selling overheads variable 16
Total Cost per unit 99

Besides this, the interest charges on additional capital required for manufacture of this product will amount to
`1,000 p.a.

DJ expects that part No.35 will be supplied by AJ. This part which is being manufactured by AJ is being sold
to outside customers at `15 each and AJ has a market for 20,000 units of this part at this price. Even the
production in excess of this quality can be sold by AJ to the outside customers at this price. The variable cost
of part 35 to AJ is `8.50 each.

DJ on the other hand would be able to procure part 35 from outside suppliers at `15 each, being the same
price at which AJ sells to outside. Since the payment of market price for part 35 will wipe off the profit, DJ has
asked the Chief Executive of AJ to supply the part at a price of `10 each.

In case AJ is agreeable to supply the part to DJ, it has two alternatives namely: (a) to supply 2000 parts from
its existing productive capacity by diverting supplies from outside customers or (b) to supply 2000 parts by
resorting to overtime working, AJ will incur additional production costs of `4,000 on the output of 2,000 parts.

Discuss the effect of the following decisions on each of the manufacturing divisions as well as on the overall
profitability of the company as a whole with suitable calculations:

a. AJ supplies part 35 to DJ at `10 each.


b. AJ refuses to supply part 35 at `10 each but agrees to supply the same at `15 each by diverting
supplies from outside market.
c. DJ produces product `M` by using part 35 purchases from outside market at `15 each.
d. AJ manufactures 2,000 units of the part by resorting to overtime and supplies the same to DJ at
`15 each.

[Ans.: Profit of Co. in Alt I: `131000; Alt II: 131000; Alt III: 131000; Alt IV: 140000]
Question 15: S.V.Ltd. manufactures a product which is obtained basically from a series of mixing operations.
The finished product is packaged in the company made Glass Bottles and packed in attractive cartons.

The company is organized into two independent divisions viz, one for the manufacture of the End product and
the other for the manufacture of Glass Bottles. The product manufacturing division can buy all the bottle
requirements from the Bottle manufacturing division.

The General Manager of the Bottle manufacturing division has obtained the following quotations from the
outside manufacturers for the supply of empty bottles.

Volume (Empty bottles) Total purchase value (`)


8,00,000 14,00,000
12,00,000 20,00,000

A cost analysis of the Bottle manufacturing division for the manufacture of empty bottles reveals the following
production costs.

Volume (Empty bottles) Total cost (`)


8,00,000 10,40,000
12,00,000 14,40,000
The production cost and sales value of the end product marketed by the product manufacturing division are
as under:

CA. Parag Gupta Ph.: +91 11 47665555 Paraggupta_ca@yahoo.co.in Costing & O.R.
World’s largest CA Final student’s consultancy group: http://groups.yahoo.com/group/costingbyparaggupta
Cost Accounting & Management - 165 -

Volume Total cost of end Sales value


Bottles of end product Product (Excluding cost of (Packed in bottles `)
empty bottles `)
8,00,000 64,80,000 91,20,000
12,00,000 96,80,000 127,80,000

There has been considerable discussion at the corporate level as to the use of proper price for transfer of
empty bottles from the Bottle manufacturing division to product manufacturing division. This interest is
heightened because a significant portion of the Divisional Manager’s salary is in incentive bonus based on
profit centre results.

As the corporate Management Accountant responsible for defining the proper transfer prices for the supply of
empty bottles by the bottle manufacturing division to the product manufacturing division. You are required to
show for the two levels of volumes of 8,00,000 and 12,00,000 bottles, the profitability by using (1) Market
price and (2) Shared profit relative to the costs involved basis for the determination of transfer prices. The
profitability position should be furnished separately for the two divisions and the company as a whole under
each method. Discuss also the effect of these methods on the profitability of the two divisions.

[Ans.: Profitability based on cost : On 800000 bottles, Bottle Mfg Division: `221276; Product Mfg Division:
`1378324; On 1200000 bottles, Bottle Mfg Division: `214964; Product Mfg Division: `1445036; Profitability
based on market price : On 800000 bottles, Bottle Mfg Division: `360000; Product Mfg Division: `1240000;
On 1200000 bottles, Bottle Mfg Division: `560000; Product Mfg Division: `1100000]
Question 16: A company is organized into two large Divisions. Divisions ‘A’ produces a component which is
used by Division ‘B’ in making a final product. The final product is sold for `400 each. Division A has a
capacity to produce 2,000 units and the entire quantity can be purchased by Division B.

Division A informed that due to installation of new machines, its depreciation cost had gone up and hence
wanted to increase the price of the component to be supplied to Division B to `220. Division B, however can
buy the component from the outside market at `200 each. The variable costs of Division A is `190 and fixed
costs `20 per component. The variable costs of Division B in manufacturing the final product by using the
component is `150 (excluding the component cost.)

Present statement indicating the position of each of the following situations separately.

(1) If there are no alternative used for the production facilities of A, will the company benefit if Division B
buys from outside suppliers at `200 per component?
(2) If internal facilities of A are not otherwise idle and the alternative use of the facilities will give an annual
cash operating saving of `30,000 to Division A, should Division B purchase the component from outside
suppliers?
(3) If there are no alternative use for the production facilities of Division A and the selling price for the
component in the outside market drops by `15, should Division B purchase from outside suppliers?
(4) What transfer price would you fix for the component in each of the above circumstances?

[Ans.: (i) If Component is purchased from outside, Co.’s Contribution is `100000 & If Component is
purchased from Division A, Co.’s Contribution is `120000 (ii) If Component is purchased from outside, Co.’s
Contribution is `130000 (iii) If Component is purchased from outside, Co.’s Contribution is `130000 (iii) When
there are no alternative uses Transfer Price is `190, When there are alternative uses Transfer Price is `205 &
If market price get reduced to `185 and there are no alternative uses, Transfer Price is `190]
Question 17: A company has two Division viz., LD and KD, LD operates at full capacity and KD operates at
50% Capacity.

LD produces two products, LX and LY using the same labour force for each product. The direct wage rate per
production hour is `5. During the next year, its budgeted capacity of 42,000 direct labour hours involves a
commitment to sell 6,000 kg of LY. The balance capacity will be used for the production of LX. Cost date are:
(` Kg.)
LX LY
Direct Materials 36 28

CA. Parag Gupta Ph.: +91 11 47665555 Paraggupta_ca@yahoo.co.in Costing & O.R.
World’s largest CA Final student’s consultancy group: http://groups.yahoo.com/group/costingbyparaggupta
Transfer Pricing - 166 -
(including Profitability analysis)

Direct Wages 30 20

The Company’s overhead amount to `7,56,000 per annum relating to LX and LY in proportion to their wages.
At full capacity `4,20,000 of this overhead is variable, LD prices its products with 50% mark-up on its total
costs.

KD wishes to buy 2,000 kgs of LX from LD for being processed into KX to be sold at `300 per kg. The
processing materials and wage cost are `30 per kg. and the variable overheads amount to `4 per kg. The
fixed costs amount to `1,00,000 per annum.

Prepare a report showing the profitability of LD and KD and the company as a whole for each of the following
transfer price methods:

(1) LD transfers LX at a price applicable to outside customers on the basis of total cost.
(2) LD transfers LX at a price based on total costs less credit for selling and distribution expenses of `4
per kg. which will not be incurred in respect of the sale to KD.
(3) LD transfers LX at a price based on marginal cost as reduced by `4 per kg. of selling and distribution
expenses.
(4) LD manufactures the quantity of LX required by KD by employing overtime payable at double the
normal wage rate and transfers at marginal cost less `4 per kg. being selling and distribution costs
not incurred in respect of sale to KD. LD sells the entire regular production to outside customers at
the usual price.

[Ans.: (1) Profit of Division LD: `629000, Loss of Division KD: `90000, Total Profit of Co.: `539000 (2) Profit
of Division LD: `621000, Loss of Division KD: `82000, Total Profit of Co.: `539000 (3) Profit of Division LD:
`351000, Profit of Division KD: `188000, Total Profit of Co.: `539000 (4) Profit of Division LD: `621000, Profit
of Division KD: `128000, Total Profit of Co.: `749000]
Question 18: A company has three departments viz., A, B and C.
A basic raw material is introduced into department ‘A’ where by-product BYEA emerges. The annual output of
BYEA is 3,000 tonnes. Out of this, 30% can be sold at `200 per tonne in a controlled market and the balance
of 70% can be sold in the open market at `1,200 per tonne.

Alternatively .department ‘A’ can transfer the entire quantity of production of BYEA at the opportunity cost to
department ‘B’ where, after processing, product RESP can be manufactured. One tonne of BYEA will yield
200 liters of RESP. The fixed costs of department ‘B’ will amount to `12 lacs and the variable costs will work
out to `4 per liters of RESP. The entire quantity of RESP can be sold by department ‘B’ at `15 per liter.

Department ‘B’ can also transfer its output of RESP at `15 per liter to department ‘C’ where it can be further
processed into POTS and packed in polythene sachets for sale. One liter of RESP will yield 1.6 liters of
POTS. The plant will however have to allow for a packing wastage of 5% of the input of RESP used by
department ‘C’ for manufacture of POTS.

Department ‘C’ has a contract for sale of POTS in two packing as detailed below:

Pack Selling Price (`/pack) % of Production for sale


200 Ml 2.50 75%
300ML 3.50 25%
The manufacturing cost of department ‘C’ will work out to `1.50 per liter of POTS actually produced and sold.
One liter = 1000ML.

Required:

(1) Calculate the income from the sale of BYEA if department ‘A’ elects to sell the by product BYEA to
outside customers.
(2) Using the opportunity cost principle find the profit / loss arising from the manufacture and sale of
RESP by department ‘B’.
(3) Present a statement showing the profit / loss arising from the sale of POTS manufactured by
department ‘C’ from out of RESP supplied by department ‘B’

CA. Parag Gupta Ph.: +91 11 47665555 Paraggupta_ca@yahoo.co.in Costing & O.R.
World’s largest CA Final student’s consultancy group: http://groups.yahoo.com/group/costingbyparaggupta
Cost Accounting & Management - 167 -

(4) If department ‘B’ is closed down and department ‘C’ is able to obtain RESP at `6.25 per liter in
exchange of the entire quantity of BYEA produced by department ‘A’ at the rate of 120 liters of RESP
for every tonne of BYEA supplied to outside firm, will it be more profitable for the company as a whole
to produce and sell POTS? Show the calculations.

[Ans.: (i) Dept. A: `2700000, (ii) Dept. B: `2700000 (iii) Dept. C: 912000 Ltrs., `842000 (iv) Profit under
existing arrangement `6242000 & Profit under new arrangement `2455200]

Question 19 [Theoretical Transfer Price]: P Ltd. has two divisions, S and T, S transfer all its output to T,
which finishes the work. Costs and revenues at various levels of capacity are as follows:

Output S Costs T Net revenues Profit


(i.e. revenue minus
Units ` costs incurred in T) `
600 600 2,950 2,350
700 700 3,250 2,550
800 840 3,530 2,690
900 1,000 3,780 2,780
1,000 1,200 4,000 2,800
1,100 1,450 4,200 2,750
1,200 1,800 4,350 2,550

Company profits are maximized at `2,800 with output of 1,000 units. If P Ltd. wish to select a transfer price in
order to establish S and T as profit centers, what transfer price would motivate the managers of S and T
together to produce 1,000 units , no more and no less?

P Ltd. wants that the transfer price should be set at `2.10 per unit. Comment on this proposal.

[Ans. Transfer price must be selected in the range `2.00 to `2.20 per unit(exclusive). At a transfer price of
`2.10 any increase in output above 1000 units, or any shortfall in output below this would not only reduce the
profits of the company as a whole but also the divisional profits of Divisions S and T]
Question 20 [Theoretical Transfer Price]: A Ltd. produces P by its two divisions X and Y. P is first
processed in X and then in Y. X and Y are treated as profit centers. The cost value-profit structure is as given
below:

Output X Y Profit
Units Costs ` Net revenue ` `
1000 900 4000 3100
1100 1000 4300 3300
1200 1120 4540 3420
1300 1250 4730 3480
1400 1400 4900 3500
1500 1580 5030 3450
1600 1800 5110 3310

Note: Net revenue for Y means the sale proceeds minus costs incurred in y. these costs do not include the
price of transferred material chargeable by X.
Required : (a) Discuss the problem. (b) Fix and explain the optimum transfer price for A Ltd.

[Ans.: `1.6]
Question 21: BETAGRO Ltd. which has a system of assessment of Divisional performance on the basis of
residual income has two divisions Alfa and Beta. Alfa has annual capacity to manufacture 15 lakhs Nos. of a
special component which it sells to outside customers: but has idle capacity. The budgeted residual income of
Beta is `120 lakhs while that of Alfa is `100 lakhs .Other relevant details extracted from the Budget of Alfa for
the year are:

Sales (to outside customers) 12 lakhs unit @ `180per unit


Variable cost per unit `160
CA. Parag Gupta Ph.: +91 11 47665555 Paraggupta_ca@yahoo.co.in Costing & O.R.
World’s largest CA Final student’s consultancy group: http://groups.yahoo.com/group/costingbyparaggupta
Transfer Pricing - 168 -
(including Profitability analysis)

Divisional fixed cost `80 lakhs


Capital employed `750 lakhs
Cost of capital 12%

Beta has just received a special order for which it requires components similar to the ones made by Alfa. Fully
aware of Alfa’s unutilized capacity. Beta has asked Alfa to quote for manufacture and supply of 3,00,000
numbers of the components with a slight modification during final processing. Alfa and Beta agree that this
will involve an extra variable cost of `5 per unit.

(1) Calculate the transfer price which Alfa should quote to Beta to achieve its budgeted residual income.
(2) Indicate the circumstances in which the proposed transfer price may result in a sub-optimal decision
for the BETAGRO group as a whole.

[Ans.: Minimum Transfer price to be quoted will be `175]


[Hint: Residual Income is the net operating profit after taxes on an investment center in excess of its required
profit. Residual Income = NOPAT – Cost of Capital × Investment]
Question 22: AB Ltd. manufactures foam, carpet and upholstery in its three divisions. Its operating statement
for 2001-02 showing the performance of these divisions drawn for the use of arrangement is reproduced
below: (`‘000)
Particulars Manufacturing Divisions Total

Foam Carpets Upholstery


Sales revenue 1600(A) 1200 1200 4000
Manufacturing costs
Variable 1200 700 680 2580
Fixed (Traceable) - 100 20 120
1200 800 700 2700
Gross profit 400 400 500 1300
Expenses
Administration 134 116 172 422
Selling 202 210 232 644
336 326 404 1066(B)
Net Income 64 74 96 234
Division’s Ranking 3rd 2nd 1st

(A) Sales include foam transferred to the upholstery division at its manufacturing cost `2,00,000.
(B) Common expenses of `1,30,000 and `1,00,000 on account of administration and selling respectively
stand apportioned to these divisions at 10% of gross profit in case of administration and 2.5% of sales
in case of selling expenses. Rest of `8,36,000 of the expenses are traceable to respective divisions.

The manager of the foam division is not satisfied with the above approach of presenting operating
performance. In his opinion his division is best among all the divisions. He requests the management for
preparation of revised operating statement using contribution approach and showing internal transfer at
market price.

You are required to:

(a) Draw the revised operating statement using contribution approach and pricing the internal transfer at
market price.
(b) Compute relevant ratios to show comparative profitability of these divisions and rank them in the light
of your answer at (a) above. Further, offer your comments on the collection of the manager of foam
division.
(c) State why the contribution approach and pricing of internal transfers at market price are more
appropriate in realistic assessment of the performance of various divisions. (19 Marks) May/96

[Ans.: (a) Net Income `(‘000)234; (b) Contribution margin ratio Foam 28.57%, Carpet 41.67% , Upholstery
36.67% Net contribution ratio & ranking Foam 13.33%(I) Carpets 12%(II) and Upholstery 8%(III)]

CA. Parag Gupta Ph.: +91 11 47665555 Paraggupta_ca@yahoo.co.in Costing & O.R.
World’s largest CA Final student’s consultancy group: http://groups.yahoo.com/group/costingbyparaggupta
Cost Accounting & Management - 169 -

Question 23: Your company fixes the inter-divisional transfer prices for its products on the basis of cost,
plus a return on investment in the division. The Budget for Division A for 1998-99 appears as under:
Investment in Division A

`
Fixed Assets 5,00,000
Current Assets 3,00,000
Debtors 2,00,000
Annual fixed cost of the division 8,00,000
Variable cost per unit of product 10

Budgeted volume 4,00,000 units per year


Desired ROI 28%
Determine the transfer price for Division A.

[Ans.: `12.70]
Question 24: A Company has two Divisions, Division ‘A’ and Division ‘B’. Division ‘A’ has a Budget of selling
2,00,000 Nos. of a particular component ‘x’ to fetch a return of 20% on the average assets employed. The
following particulars of Division` A are also known:

Fixed Overhead `5 lakhs


Variable Cost `1 per unit
Average Assets
Sundry Debtors `2 lakhs
Inventories `5 lakhs
Plant & equipments `5 lakhs

However, there is constraint in Marketing and only 1,50,000 units of the component ‘x’ can be directly sold to
the Market at the proposed price.

It has been gathered that the balance 50,000 units of component ‘x’ can be taken up by Division ‘B’. Division
‘A’ wants a price of `4 per unit of ‘x’ but Division ‘B’ is prepared to pay `2 per unit of ‘x’.

Division ‘A’ has another option in hand. Which is to produce only 1,50,000 units of component ‘x’. This will
reduce the holding of assets by `2 lakhs and fixed overhead by `25,000.

You are required to advise the most profitable course of action for Division ‘A’. (15 Marks) Nov./97

[Ans.: Sell 150000 units in market and transfer 50000 units to Division B as Division A’s Profit and ROCE is
increased by `25000 and 0.75% respectively]

Question 25: A company is organized on decentralized lines, with each manufacturing division operating as a
separate profit centre. Each division manager has full authority to decide on sale of the division’s output to
outsiders and to the other divisions.

Division C has always purchase its requirement of a component from Division A. But when informed that
division was increasing its selling price to ` 150, the manager of Division C decided to look at outside
suppliers.

Division C can buy the component from an outside supplier for ` 135. But Division A refuses to lower its price
in view of its need to maintain its return to the investment.

The top management has the following information:


C’s annul purchase of the component 1000 units
A’s variable cost per unit ` 120
A’s fixed cost per unit ` 20

CA. Parag Gupta Ph.: +91 11 47665555 Paraggupta_ca@yahoo.co.in Costing & O.R.
World’s largest CA Final student’s consultancy group: http://groups.yahoo.com/group/costingbyparaggupta
Transfer Pricing - 170 -
(including Profitability analysis)

Required:

(i) Will the company as a whole benefit, if Division C bought the component at ` 135 from an outside
supplier?
(ii) If A did not produce the material for C, it could use the facilities for other activities resulting in a cash
operating savings of ` 18000. Should C then purchase from outside source?
(iii) Suppose there is no alternative use of A’s facilities and the market price per unit of the component
drops by ` 20. Should C now buy from outside? (13 Marks) Nov/98

[Ans.: (i) The co. as a whole will not benefit as it will be required to incur an additional cost of `15000; (ii) It is
advisable that Division C should purchase the component from outside sources as the decision will benefit the
company by `3000; (iii) It is advisable that the Division C should buy the component from outside as the
decision will benefit the company by `5000]
Question 26: City Instrument Company (CIC) consist of the Semi- conductor Division and the Mini-computer
Division, each of which operates as an independent profit centre. Semi-conductor Division employs
craftsmen, who produce two different electronic components, the new- high performance Super chip and an
old product called Okay-chip. These two products have the following cost characteristics:

Super-chip Okay-chip
Material Parts ` 20 Parts ` 10
Labour 2 hours x ` 140 280 1/2 x ` 140 70
Annual Overhead in Semi-conductor Division is ` 4000000 all fixed. Owing to high skill level necessary for the
craftsmen, the Semi-conductor Division’s capacity is set at 50000 hours per year.

To date, only one customer has developed a product utilizing super-chip, and this customer orders a
maximum of 15000 super-chips per year at a price of ` 600 per chip. If CIC can not meet his entire demand,
the customer curtails his own production. The rest of the semi-conductor’s capacity is devoted to Okay-chips,
for which there is unlimited demand at ` 120 per chip.

The Mini-computer Division produces only one product, a process control unit, which requires a complex
circuit board imported at a price of ` 600. The control unit’s cost are:
Control Unit
Material Circuit board ` 600
Other parts 80
Labour 5 hours @ ` 100 500

The Mini-computer Division is composed of only a small assembly plant and all overhead is fixed at a total of
` 800000 per year. The current market price for the control unit is ` 1400 per unit.

A joint research project has just revealed that with minor modifications, a single super-chip could be
substituted for the circuit board currently used by the Mini- computer division. The modification would require
an extra one hour of labour by Mini-computer’s staff, for a total of 6 hour per control unit. Mini-computer has
therefore asked Semi-conductor division to declare a transfer price at which Semi-conductor division would
sell super-chip internally.

Required:

(i) Mini-computer expects to sell 5000 control units this year. From the overall view point of CIC, how
many super-chips should be transferred to Mini-computer Division to replace circuit boards?
(ii) If the demand for the control units is sure to be 5000 units, but its price is uncertain, what should be
the transfer price of super-chip to ensure proper decisions? (All other data uncharged.)
(iii) If demand for the control unit rises to 12000 units at a price of ` 1400 per unit, how many of 12000
units should be built using super-chip? (All other data unchanged.) (13 Marks) Nov/00

[Ans.: (i) 5000 units (ii) `460; (iii) 10000 units]


Question 27: A Company is engaged in the manufacture of edible oil. It has three division as under:

CA. Parag Gupta Ph.: +91 11 47665555 Paraggupta_ca@yahoo.co.in Costing & O.R.
World’s largest CA Final student’s consultancy group: http://groups.yahoo.com/group/costingbyparaggupta
Cost Accounting & Management - 171 -

(i) Harvesting oil seeds and transportation therefore to the oil mill.
(ii) Oil Mill, which process oil seeds and manufactures edible oil.
(iii) Marketing Division, which packs the edible oil in 2 kg. containers for sale at ` 150 each container.

The Oil Mill has a yield of 1000 kgs. of oil seeds during a period. The Market Division has a yield of 500 cans
of edible oil of 2 kg. each from every 1000 kg. of oil. The net weight per can is 2 kg. of oil.
The cost data for each division for the period are as under:

Harvesting Division:
Variable cost per kg of oil seed ` 2.50
Fixed cost per kg. of oil seed ` 5.00

Oil Mil Division:


Variable cost of processed edible oil ` 10.00 per kg.
Fixed cost of processed edible oil ` 7.50 per kg.

Marketing Division:
Variable cost per can of 2 kg of oil ` 3.75
Fixed cost per can of 2 kg of oil ` 8.75

The fixed cost are calculated on the basis of estimated quantity of 2000 kg of oil seeds harvested, 1000 kg
of processed oil and 500 cans of edible oil packed by the aforesaid divisions respectively during the period
under review.

The other oil mills buy the oil seeds of the same quality at ` 12.50 per kg. in the market. The market price of
edible oil processed by the oil mill, if sold without being packed in the marketing division is ` 62.50 per kg of
oil.
Required:

(i) Compute the overall profit of the company of harvesting 2000 kg of oil seeds, processing it into edible oil
and selling the same in 2 kg cans as estimated for the period under review.
(ii) Compute the transfer prices that will be used for internal transfer from
(1) Harvesting Division to Oil Mill Division and
(2) From Oil Mill Division to Marketing Division
under the following pricing methods:
(1) Shared contribution in relation to variable cost; and
(2) Market price.
(iii) Which transfer pricing method will each divisional manager prefer to use? (12 Marks) May/01

[Ans.: (i) `36250; (ii) (1) `66667 (2) `62500; (iii) Harvesting Division Manager-Market Price method, Oil Mill
Division Manager shared contribution method, Marketing Division Manager-Market Price Method]
Question 28: The two manufacturing divisions of a company is organized on profit centre basis. Division X is
the only source of a component required by Division Y for their product P. Each unit of P requires one unit of
the said component. As the demand of the product is not steady, orders for increased quantities can be
obtained by manipulating prices. The manager of Division Y has given the following forecast:

Sales per day (units) Average price per unit of P (`)


5,000 393.75
10,000 298.50
15,000 247.50
20,000 208.50
25,000 180.00
30,000 150.75

The manufacturing cost (excluding the cost of the component from Division X) of P in Division Y is ` 1406250
on first 5000 units and ` 56.25 per unit in excess of 5000 units.

CA. Parag Gupta Ph.: +91 11 47665555 Paraggupta_ca@yahoo.co.in Costing & O.R.
World’s largest CA Final student’s consultancy group: http://groups.yahoo.com/group/costingbyparaggupta
Transfer Pricing - 172 -
(including Profitability analysis)

Division X incurs a total cost of ` 562500 per day for an output up to 5000 components and the total cost will
increase by ` 337500 per day for every additional 5000 component manufactured. The manager of Division X
has set the transfer price for the component at ` 90 per unit to optimize the performance of his Division.

Required:

(i) Prepare a divisional profitability statement at each level of output, for division X and Y separately;
(ii) Find out the profitability of the company as a whole at the output level where:
(a) Division X’s net profit is maximum;
(b) Division Y’s net profit is maximum.
(iii) Find out at what level of output, the company will earn maximum profit, if the company is not
organized on profit centre basis. (15 Marks) May/02

[Ans.: (i) At 30000 components profitability of X `450000, loss of Y `990000 (ii) Operating Loss of the
company X `540000, Profitability of Company Y `397500; (iii) Maximum profit will be attained if 15000
components are produced.]
Question 29: Division Z is a profit centre, which produces four products – A B C and D. each product is sold
in the external market also. Data for the period as follows:

A B C D
Market Price Per Unit ` 150 `146 ` 140 ` 130
Variable Cost of Production Per Unit ` 130 ` 100 ` 90 ` 85
Labour Hours Required Per Unit 3 4 2 3

Production D can be transferred to division Y, but the maximum quantity that might be required for transfer is
2500 units of D.

The maximum sales in the external market are:


A 2800 units
B 2500 units
C 2300 units
D 1600 units

Division Y can purchase the same product as a slightly cheaper price of ` 125 per unit instead of receiving
transfers of product D from division Z.

What should be the transfer price for each unit for 2500 units of D, if the total labour hours available in division
Z are:
(i) 20000 hours?
(ii) 30000 hours? (13 Marks) May/00

[Ans.: Transfer Price per unit `118.34; (ii) Transfer Price per unit `99.13]
Question 30: Division A is a profit centre which produces three products X, Y and Z. Each product has
an external market.
Products X Y Z
` ` `
External market price per unit 48 46 40
Variable cost of production p.u. in 33 24 28
division A
Labour hours required per unit 3 4 2
in division A

Product Y can be transferred to Division B, but the maximum quantity that might be required for transfer is
300 units of Y.
CA. Parag Gupta Ph.: +91 11 47665555 Paraggupta_ca@yahoo.co.in Costing & O.R.
World’s largest CA Final student’s consultancy group: http://groups.yahoo.com/group/costingbyparaggupta
Cost Accounting & Management - 173 -

The maximum external sales are:


X 800 units Y 500 units Z 300 units

Instead of receiving transfers of product Y from Division A, Division B could buy similar product in the
open market at a slightly cheaper price of `45 per unit.
What should the transfer price be for each unit for 300 units of Y, if the total labour hours available in
Division A are:
(a) 3,800 hours
(b) 5,600 hours

[Ans.: (a) `44 (b) `34]


Question 31: Department x is a profit centre manufacturing products Vx, X and Xt. Each of the products can
be sold in the outside market to the extent of the following:

Vx 900 units
X 300 units
Xt 600 units

Market price per unit is ` 24, ` 23, and ` 20 for Vx, X and Xt respectively. Other details are given below:
Vx X Xt
` ` `
Variable cost of production p.u. 17 12 14
Labour hours required 3 2 4

Product Vx can be transferred to department y, but the maximum quantity that might be required for transfer
is 400 units of Vx. The Manager of department y has powers to buy the product Vx from the external market
at a much cheaper price of ` 22.

What should be the transfer price for each unit for 400 units of Vx, if the total labour hours available in
department X is:
(a) 4800 hours
(b) 6200 hours? (12 Marks) Nov/03

[Ans.: (a) `21.50 (b) `19.625]


Question 32: A Company is organised into two divisions. Division X produces a component, which is used by
division Y in making of a final product. The final product is sold for `540 each. Division X has capacity to
produce 2,500 units and division Y can purchase the entire production. The variable cost of division X in
manufacturing each component is `256.50.

Division X informed that due to installation of new machines, its depreciation cost had gone up and hence
wanted to increase the price of component to be supplied to division Y to `297 , however division Y can buy
the component from out side the market at `270 each. The variable cost of division Y in manufacturing the
final product by using the component is `202.50 (excluding component cost).

Present the statement indicating the position of each Division and the company as whole taking each of the
following situations separately:

(i) If there is no alternative use for the production facility of X, will the company benefit, if division Y buys
from out side suppliers at `270 per component.
(ii) If internal facilities of X are not otherwise idle and the alternative use of the facilities will bring an
annual cash saving of `50,625 to division X, should division Y purchase the component from outside
suppliers ?
(iii) If there is no alternative use for the production facilities of division X and the selling price for the
component in the outside market drops by `20.25, should division Y purchase from outside supplier?
(iv) What transfer price would be fixed for the component in each of the above circumstances?

CA. Parag Gupta Ph.: +91 11 47665555 Paraggupta_ca@yahoo.co.in Costing & O.R.
World’s largest CA Final student’s consultancy group: http://groups.yahoo.com/group/costingbyparaggupta
Transfer Pricing - 174 -
(including Profitability analysis)

[Ans.: (i) It will be beneficial for the company as whole to buy component from Division X (ii) & (iii) It is
beneficial to buy component from outside. (iv) Transfer price (a) `256.50 per component, (b) `276.75 per
component, (c) `256.50 per component.] (12 Marks) Nov/05

Question 33: Fasteners Ltd. is having production shops reckoned as cost centres. Each shop charges other
shops for material supplied and services rendered.

The shops are motivated through goal congruence, autonomy and management efforts. Fastner Limited is
having a welding shop and painting shop. The welding shop welds annually 75,000 purchased items with
other 1,50,000 shop made parts into 12,000 assemblies. The assemblies are having variable cost of `9.50
each and are sold in market at `12 per assembly. Out of the total production, 80% is diverted to painting shop
at same price ruling in the market. Welding shop incurs a fixed cost of `25,000 per annum. The painting shop
is having fixed cost of `30,000 and its cost of painting including transfer price from welding shop comes to `20
per unit. This shop sells all units transferred to it by welding shop at `25 per assembly.

You are required to:

(a) Find out profit of individual cost centres and overall profitability of the concern.
(b) Recommend course of action if painting shop wishes to purchase its full requirement (at market
price which is `10 per assembly) either from open market or from welding shop at market
price of `10 per assembly.
Give reasons for your recommendations

[Ans.: (a)Overall profit for the company (`5,000 + `18,000) = `23,000]


[Hint: Suggested answers has assumed that welding shop can sell only 20% of whole production to outside
market]
Question 34: Division A of Better Margins Ltd. has been given a budgeted target of selling 2,00,000
components COM 21, it manufactures at a price which would fetch a return of 25% on the average assets
employed by it. The following figures are relevant:
Fixed overhead `4,00,000
Variable cost `1 per unit
Average assets:
Sales debtors 2,00,000
Stocks 6,00,000
Plant and other assets 4,00,000

However, the marketing department of the company finds out by a survey that the maximum number of COM
21, the market can take, at the proposed price is only 1,40,000 units.

Fortunately Division B is willing to purchase the balance 60,000 units. The Manager, Division A is willing to
sell to Division B at a concessional price of `4 per unit. But the Manager, Division B is ready to pay `2.25
only per unit, as he feels he can himself make COM 21 in his Division at that price.

Rather than sell to Division B at `2.25, the Manager, Division A feels he will restrict the activity of his Division
to the manufacture and sale of 1,40,000 components only. By this, he could reduce `80,000 in stocks,
`1,20,000 of plant and other assets and `40,000 in selling and administration expenses.

As a Cost Accountant, you are asked to work out the various computations and show that selling 60,000 COM
21 to Division B at `2.25 per unit would be in the interest of the organization.

Question 35: A boatyard is divided into three profit centers whose managers are rewarded according to
results. Transactions between these profit centres are frequent.

Sales centre (S) buys and sells new boats.


If it needs to take part-exchange from a customer in order to sell a new boat, it transfers the part-exchanged
boat to B at an agreed price.

Brokerage (B) buys and sells second-hand boats:


CA. Parag Gupta Ph.: +91 11 47665555 Paraggupta_ca@yahoo.co.in Costing & O.R.
World’s largest CA Final student’s consultancy group: http://groups.yahoo.com/group/costingbyparaggupta
Cost Accounting & Management - 175 -

(i) in part-exchange from S (B names the price at which it can buy a comparable boat that is in a
suitable condition for resale to an end-user customer, but deducts the likely cost of repairs) and
(ii) from other sources, on a normal trading basis.
Repairs (R) does repairs for
(i) B (to put boats into saleable condition) and
(ii) other customers.

The following situation arises:


S can sell to a customer for `35,000 a new boat which would cost `29,000. To do so, it needs to offer
`16,000 in part-exchange for the customer’s old boat. However, the customer’s boat is estimated by R to
need repairs that will cost:
Materials `300
Labour 60 hours at `15 per hour

B can buy for `15,000 a boat comparable to the one being offered by the customer in part-exchange but
which needs no repair. B could then sell that boat for `19,000.
Other data:-

R’s labour rate per hour is made up as follows: `


Variable cost 6.00
Fixed cost 4.50 (based on 20,000 budgeted hours p.a.)
Profit 4.50
15.00
– 45% of R’s time is reserved for work from B
– Annual fixed cost is budgeted at:
S `70,000
B `80,000

You are required, in relation to the above situation, to set out the contribution to profit for each profit centre
that would result:
(i) assuming that all estimates and budgets materialized as expected,
(ii) assuming that all estimates and budgets materialized as in (i), except that the repairs undertaken
by R took an extra 10 hours and `100 of materials due to a problem not noticed by B or R.

[Ans.: (i) Contribution `540 (ii) Increased contribution `630]


Question 36: A company is organized on decentralized lines, with each manufacturing division operating as a
separate profit centre. Each division manager has full authority to decide on sale of division’s output to
outsiders or to other divisions. Division AB manufactures a single standardised product. Some output is sold
externally and remaining is transferred to division XY where it is a sub-assembly in the manufacture of the
division product. The unit cost of division AB product and division XY is as follows:

Division AB (`) Division XY (`)

Transfer from division AB to XY - 42.00


Direct material 6.00 35.00
Direct labour 3.00 4.50
Direct expenses 3.00 -
Variable manufacturing overheads 3.00 18.00
Fixed manufacturing overheads 6.00 18.00
Variable selling and packing expenses 3.00 2.50
24.0 120.00

Division AB sold 40,000 units annually at the standard price of `45 in external market. In additions to the
external sales, 10,000 units are transferred annually to division XY at internal price of Rupees 42 per unit.
Variable selling and packing expenses are not incurred by supplying division for the internal transfer of the
product. Division XY incorporates the transferred goods into more advance product. The manager of division
XY disagrees with the basis used to set the transfer price. He argues that transfer price should be made at
CA. Parag Gupta Ph.: +91 11 47665555 Paraggupta_ca@yahoo.co.in Costing & O.R.
World’s largest CA Final student’s consultancy group: http://groups.yahoo.com/group/costingbyparaggupta
Transfer Pricing - 176 -
(including Profitability analysis)

variable cost since he claims that his division is taking output that division AB should be unable to sell at price
`45.
He also submitted a report of the relationship between selling price and demand to support of his
disagreement. The report of customer demand at various selling prices for division AB and for division XY is
as follows:

Division AB
Selling price per unit (`) 30 45 60
Demand (Units) 60,000 40,000 20,000
Division XY
Selling price per unit (`) 120 135 150
Demand (Units) 15,000 10,000 5,000

The company has sufficient capacity to meet demand at various selling prices. Internal transfer demanded
units will be decided by XY division.

Required:

(i) To calculate divisional profitability and overall profitability of company if division AB transfers demanded
units to XY at price of `42.
(ii) To calculate divisional profitability and overall profitability of company if division AB transfers demanded
units to XY at variable cost.
(iii) In place of internal transfers, AB division can sell 10000 units of their product in new external market
without effecting existing market, at price `32 per unit and XY division can purchase these units at the rate of
`31 in open market. Calculate company’s profit by following above strategies.

[Ans.: (a) Profitability- Div. AB: `900000, Div. XY: `60000, Cos.: `960000; (b) Profitability: Div. AB:
`6300000, Div. XY: `405000, Cos.: `1035000] (12 Marks) June/09[Old Course]
[Note: Suggested answers of ICAI has wrongly taken Total Contribution as Total Profit & has forgotten to
subtract fixed costs while making profitability statement. Part (iii) is wrongly solved by suggested answers of
ICAI.]
Question 37: Division A of a large divisionalised organization manufactures a single standardized product.
Some of the output is sold externally whilst the remainder is transferred to Division B where it is a sub-
assembly in the manufacture of that Division’s product. The unit cost of Division A’s product are as follows:
(`)
Direct Material 4
Direct Labour 2
Direct expenses 2
Variable manufacturing overheads 2
Fixed manufacturing overheads 4
Selling and packing expenses- Variable 1
Total 15

Annually 10,000 units of the product are sold externally at the standard price of `30. In addition to the external
sales, 5,000 units are transferred annually to Division B at an internal transfer charge of `29 per unit. This
transfer price is obtained by deducting variable selling and packing expenses from the external price since
these expenses are not incurred for internal transfers.

Division B incorporates the transferred goods into a more advanced product. The unit costs of the product are
as follows: (`)
Transferred in item ( from Division A) 29
Direct material and components 23
Direct labour 3
Variable overheads 12
Fixed overheads 12
Selling and packing expenses- Variable 1
Total 80

CA. Parag Gupta Ph.: +91 11 47665555 Paraggupta_ca@yahoo.co.in Costing & O.R.
World’s largest CA Final student’s consultancy group: http://groups.yahoo.com/group/costingbyparaggupta
Cost Accounting & Management - 177 -

Division B’s manager disagrees with the basis used to set the transfer price. He argues that the transfer
should be made at variable cost plus an agreed (minimal) mark-up since he claims that his division is taking
output that Division A should be unable to sell at the price of `30.

Partly because of this disagreement, a study of the relationship between selling price and demand has
recently been made for each division by the company’s Sales Director. The resulting report contains the
following table:

Customer demand at various selling prices:


Division A
Selling price (`) `20 `30 `40
Demand (units) 15,000 10,000 5,000
Division B
Selling price (`) `80 `90 `100
Demand (units) 7,200 5,000 2,800

The Manager of Division B claims that this study supports his case. He suggests that a transfer price of `12
would give Division A a reasonable contribution to its fixed overheads while allowing Division B to earn a
reasonable profit. He also believes that it would lead to an increase of output and an improvement in the
overall level of company’s profits.

You are required to:


1. Calculate the effect of the transfer pricing system on the company’s profits, and
2. Establish the likely effect on profits if adopting the suggestion by the Manager of Division B for a
transfer price of `12. ICWA-Dec./92

[Ans.: 1. Profit for the company as a whole from the sale of the final product are reduced from `223200 (7200
units) to `205000 (5000 units) 2. Apart of Contribution of `190000 from sale of intermediate product to the
external market, contribution of Division A is `14400 and of Division B is `208800.]
[Hint: Assume that the company has sufficient capacity to meet demand at various selling prices]

Profitability analysis (Strategy-based questions)

Question 38: Opticals Ltd. makes two kinds of products, P (lenses) and Q (swimming goggles) in divisions P
and Q respectively. P is an input for Q and two units of P are needed to make one unit of Q.
The following data is given to you for a period:

P Q
`/u of P `/ of Q
Direct Materials 20 25 (Excluding P)
Direct Labour 30 35
Variable overhead 10 20
External Demand (units) 3,000 3,000
Capacity (units) 7,000 2,500
Selling Price `/u
(outside market) 100 410

If Q buys P from outside, it has the following costs:


for order quantity 2,499 or less `90 per unit for the entire quantity ordered.
for order quantity 2,500-5,000 `80 per unit for the entire quantity ordered.
for order quantity more than 5,000 `70 per unit for the entire quantity ordered.

You are required to:


i) Evaluate the best strategies for Divisions P and Q.
ii) Briefly explain the concept of goal congruence. (12 Marks) Nov./09-N.C.

[Ans.: (i) Division P should sell 3000 units in market and transfer 4000 units to Division Q and Division Q
should buy 1000 units from market]

CA. Parag Gupta Ph.: +91 11 47665555 Paraggupta_ca@yahoo.co.in Costing & O.R.
World’s largest CA Final student’s consultancy group: http://groups.yahoo.com/group/costingbyparaggupta
Transfer Pricing - 178 -
(including Profitability analysis)

Question 39: X Ltd. has two divisions, A and B, which manufacture products A and B respectively. A and B
are profit centres with the respective Divisional Managers being given full responsibility and credit for their
performance.

The following figures are presented:


Division A Division B
`Per Unit `Per Unit
Direct material cost 50 24* *(other than A)
Material A, if transferred from Division A ─ 144
Material A, if purchased from outside ─ 160
Direct labour 25 14
Variable production overhead 20 2
Variable selling overhead 13 26
Selling price in outside market 160 300
Selling price to B 144 ─
Selling price to S Ltd. ─ 250

Other Information:
To make one unit of B, one unit of component A is needed. If transferred from A, B presently takes product A
at `144 per unit, with A not incurring variable selling overheads on units transferred to B.

Product A is available in the outside market at `160 per unit from competitors.

B can sell its product B in the external market at `300 per unit, whereas, if it supplied to X Ltd.’s subsidiary, S
Ltd., it supplies at `250 per unit, and need not incur variable selling overhead on units transferred to S Ltd. S
Ltd. requires 6,000 units and stipulates a condition that either all 6,000 units be taken from B or none at all.

A (units) B (units)
Manufacturing capacity 20,000 28,000
Demand in external market 18,000 26,000
S Ltd.’s demand ─ 6,000 or zero
Assume that Divisions A and B will have to operate during the year.

What is the best strategy for:


(i) Department A?
(ii) Department B, given that A will use its best strategy?
(iii) For X Ltd. As a whole? (14 Marks) May/08

[Ans.:(i) Sell 18000 units in market and 2000 units to B. Total Contribution `1034000 (ii) Get 2,000 units from
A, sell 6,000 units to S and 20,000 to outside. Make 28,000 units @ full capacity. Total Contribution
`19,60,000 (iii) Make A transfer all output to B. Sell 6,000 units of B to S and 22,000 units to outside market.
This will make X Ltd. better off by 32,28,000 – 29,94,000 = ` 2,34,000]

Question 40: AB Ltd. has two divisions, A and B, making products A and B respectively. One unit of A is an
input for each unit of B. B has production capacity of 45,000 units and ready market for 45,000 units in both
the years 2010 and 2011. Other information available:

Division A Year
2010 2011
Capacity (production units) 50,000 50,000
Maximum demand in usual external market (units) 25,000 30,000
Special order (units) (to be fully accepted or fully 10,000 15,000
rejected)
Fixed Cost `/ annum upto 30,000 units 4,30,000 4,30,000
(Beyond 30,000 units, fixed cost increases by
`1,00,000 for every additional 10,000 units for
each year)
Variable manufacturing cost `/unit 35 35

CA. Parag Gupta Ph.: +91 11 47665555 Paraggupta_ca@yahoo.co.in Costing & O.R.
World’s largest CA Final student’s consultancy group: http://groups.yahoo.com/group/costingbyparaggupta
Cost Accounting & Management - 179 -

Variable selling cost `/unit (only for usual external 10 10


sales)
Variable selling cost `/unit (only for special order 5 5
and transfer to B)
Selling price (usual external market) `/ unit 65 65
Selling Price (only special order) `/unit 55 55

B buys input A from outside at a slightly incomplete stage at `30 per unit and incurs sub-contract charges at
`20 per unit to complete it to a stage to match the output of Division A. In 2011, subcontract charges will
increase to `30 per unit. B is willing to pay A, the price if incurs viz. `50 and `60 per unit in 2010 and 2011
respectively, provided A supplies B's full requirement. For any lesser quantity, (B will accept any quantity), B
is willing to pay A only `45 and `55 per unit in 2010 and 2011 respectively. Assume no changes in inventory
levels. In 2011, A may choose to avoid the variable selling overhead of `5 per unit on transfers to B or special
order by incurring a fixed overhead of `50,000 p.a. instead.

(i) What will be the maximum profits of A under its best strategy in 2011?
(ii) In view of the company's overall interest, calculate the customerwise units to be produced by A in
2010.
(iii) Assuming that A follows its best strategy between what values of transfer price will B be able to
negotiate with A, so that A's best strategy is unchanged in 2011. (15 Marks) May/10-O.C.

[Ans.: (i) Transfer 45000 units to B, Usual sales in external market 5000 units & incurring special fixed cost to
save variable selling costs; A’s Profit `545000; (ii) Usual Sales: 25000 units, Special order: 10000 units;
Division B: 15000 units. Alternatively, Usual Sales: 25000 units, Special order: 10000 units; Division B: 5000
st
units. But 1 alternative is recommended; Total Contribution of A & Savings of B = `170000; (iii) `57.78]
[Note: Suggested answer of ICAI has erroneously solved part (ii) for Year 2011 instead of 2010. Coincidently
units are same.]
Question 41: XYZ Ltd. has two divisions, A and B. Division A makes and sells products A, which can be sold
outside as well as be used by B. A has a limitation on production capacity, that only 1,200 units can pass
through its machining operations in one month. On an average, about 10% of the units that A produces are
defective. It may be assumed that out of each lot that A supplies, 10% are defectives.

When A sells in the outside market, the defectives are not returned, since the transportation costs make it
uneconomical for the customer. Instead, A's customers sell the defectives in the outside market at a discount.
But when B buys product A, it has to fix it into its product, which is reputed for its quality. Therefore, B returns
all the defective units to A. A can manually rework the defectives, incurring only variable labour cost and sell
them outside at `150 and not having to incur any selling costs on reworked units. If A chooses not to rework,
it can only scrap the material at `30 per unit. B can buy product A from outside at `200 per unit, but has to
incur `10 per unit, as variables transport cost. B can insist to its outside suppliers also that it will accept only
good units.

A incurs a variable selling overhead only on units (other than reworked units) sold outside. The following
figures are given for the month:

Variable cost of production-Dept. A (`/unit) 120


Variable selling overhead (`/u) 20
Selling price per unit in the outside market (`/u) 200
Current selling price to B (`/u) 190
Additional variable labour cost of reworking defectives (`/u) 100
Selling price of reworked defectives (`/u) 150
Fixed costs for the month (`) 36,000
Maximum demand from B at present (no. of units) 630

The outside demand can be freely had upto 900 units.

Given the demand and supply conditions, you are required to present appropriate calculations for the
following:

CA. Parag Gupta Ph.: +91 11 47665555 Paraggupta_ca@yahoo.co.in Costing & O.R.
World’s largest CA Final student’s consultancy group: http://groups.yahoo.com/group/costingbyparaggupta
Transfer Pricing - 180 -
(including Profitability analysis)

(i) Evaluation of the best strategy for A in the present condition.


(ii) If B can buy only upto 540 units and the outside demand is only 600 units, how much should A charge B to
maintain the same level of profit, as in (i) above? (12 Marks) June/09-N.C.

[Ans.: (i) Best strategy is to sell maximum units to outside market from which contribution is `70,800; (ii)
`192.20 per unit.]

Question 42: Bearings Ltd. makes three products, A, B and C in Divisions A, B and C respectively. The
following information is given:

A B C
Direct Materials (excluding material A for
Divisions B and C) 4 15 20 `/u
Direct labour 2 3 4 `/u
Variable overhead 1 1 1 `/u
Selling price to outside customers 15 40 50 `/u
Existing Capacity 5,000 2,500 2,500 (No. of units)
Maximum External demand 3,750 5,000 4,000 (No. of units)
Additional fixed costs that would be incurred
to install additional capacity 24,000 6,000 18,750 `
Maximum Additional units that can be
produced by additional capacity 5,000 1,250 2,250 (No. of units)

B and C need material A as their input. Material A is available outside at `15 per unit. Division A supplies the
material free from defects. Each unit of B and C requires one unit of A as the input material.

If B purchases from outside, it has to pay `15 per unit. If B purchases from A, it has to incur in addition to the
transfer price, `2 per unit as variable cost to modify it.

B has sufficient idle capacity to inspect its inputs without additional costs.

If C gets material from. A, it can use it directly, but if it gets material from outside, which is at `15, it has to do
one of the following :
(i) Inspect it at its own shop floor at `3 per unit.
Or
(ii) Get the supplier to supply inspected products and pay the supplier `2 p.u. as inspection charges.
Or
(iii) A has enough idle labour, which it can lend to C to inspect at `1 p.u. even though C purchases from
outside.

A has to fix a uniform transfer price for both B and C. The transfer price will not be known to outsiders and is
at the discretion of the Divisional Managers.
What is the best strategy for each division and the company as a whole? (12 Marks) June/09-N.C.

[Ans.: B will not pay A more than `13, C should purchase all units from A at `13. A should make 10,000 units
viz. Outside: 3750 units, To B: 3750 units, To C: 2500 units; Sales by B – 3750 units, Sales by C – 2500 units;
Individual strategy is best strategy of the co.]
[Note: Division A is having spare capacity & it has to set TP for its best strategy. Its best strategy will be to
st
charge anything equivalent or above incremental cost p.u. i.e. `7 p.u. for 1 1250 units transferred & `11.80
for additional 5000 units transferred. But when considering best strategies of B & C, maximum TP Division B
is ready to pay is `13 p.u. (15-2) & `16 p.u. (15+1). Hence A can fix any transfer price uniformly within range
`11.80-`13, but since Division A is setting TP, it is assumed that it will charge maximum TP i.e. `13 p.u.
Alternate assumptions exists & are equally good]
Question 43: Tripod Ltd. has three divisions X, Y and Z, which make products X, Y and Z respectively. For
division Y, the only direct material is product X and for Z, the only direct material is product Y. Division X
purchases all its raw material from outside. Direct selling overhead, representing commission to external
sales agents are avoided on all internal transfers. Division Y additionally incurs `10 per unit and `8 per unit on

CA. Parag Gupta Ph.: +91 11 47665555 Paraggupta_ca@yahoo.co.in Costing & O.R.
World’s largest CA Final student’s consultancy group: http://groups.yahoo.com/group/costingbyparaggupta
Cost Accounting & Management - 181 -

units delivered to external customers and Z respectively. Y also incurs `6 per unit picked up from X, whereas
external suppliers supply at Y’s factory at the stated price of `85 per unit.
Additional information is given below:
Figures `/unit

X Y Z
Direct materials (external supplier rate) 40 85 135
Direct labour 30 50 45
Sales Agent’s commission 15 15 10
Selling price in external market 110 170 240
Production capacity 20,000 30,000 40,000 units
External demand 14,000 26,000 42,000 units
You are required to discuss the range of negotiation for Managers X, Y and Z, for the number of units
and the transfer price for internal transfers. (11 Marks) Nov./08-N.C.

[Ans.: Y will sell to Z only if X sells to Y at `70 per unit and Y will supply to Z maximum 4000 units]

Question 44: M Ltd. makes two products, X and Y, in their respective divisions. Each unit of Y needs one unit
of X. Divisions X and Y are profit centres and can function according to their divisional interests.

In the external domestic market, X can sell either 6000 units at `1,000 per unit or 5000 units at `1,120 per
unit.

X has a production capacity of 7000 units, with each unit requiring 2 hours. Y also has a production and
demand of 7000 units.

Y can buy product X from outside as follows:

Order Quantity Price for the entire order


(Units) (`/u)
6001-7000 900
4001-6000 920
2001-4000 1,000
0-2000 1,120

Y resorts to bulk purchase to avail maximum possible discount.

(i) There is an export order (that may either be fully accepted or fully rejected) for X to supply 800 units @
`900 per unit.

(ii) There is an offer to hire out X's capacity of 1600 hours at `130 per hour. The hiring offer may either be
fully accepted or fully rejected.

(iii) Y will not buy from X at any price more than it will incur in the outside market. Y does not place restrictions
on quantities to be supplied by X, provided its pricing condition is not violated.

Given that any one or more of the offers may be accepted, what will be X's best strategy?
What will be the corresponding transfer price?
[A detailed cost statement is not essential. Only figures relevant for decision making are required to be
considered under each analysis.] (9 Marks) Nov./10-O.C.
[Ans.: Best strategy for X is to sell externally 5000 units @ `1120 p.u. & transfer 2000 units to Y at max. T.P.
of `920 p.u.]

CA. Parag Gupta Ph.: +91 11 47665555 Paraggupta_ca@yahoo.co.in Costing & O.R.
World’s largest CA Final student’s consultancy group: http://groups.yahoo.com/group/costingbyparaggupta
Decision Making

Relevant Costing & Differential Costing

A decision model is a format method of making choice, and it often involves both quantitative and qualitative
analyses. Management accountants work with managers by analyzing and presenting relevant data to guide
decisions.

Incremental Costing: (CIMA’s Official terminology) It is a technique used in the preparation of ad hoc
information where consideration is given to a range of graduated or stepped changes in the level or nature of
activity, and the additional costs and revenues likely to result from each degree of change are presented.

Remember, Incremental cost is the additional total cost incurred for an activity & Differential cost is the
difference in total cost between two alternatives. Sometimes these terms are used interchangeably in
practice. Similarly incremental revenues and differential revenues can be defined in terms of revenue.

Opportunity cost is a measure of the benefit of opportunity forgone when various alternatives are
considered. It is the benefit foregone or rejected by not using a limited resource in its next-best alternative
use. It is the prospective change in cost following the adoption of an alternative machine, process, raw
materials, specification or operation. The lost opportunity is a cost that the manager must take into
consideration when making a decision. For example, the (relevant) cost of going to school for an CA degree is
not only the cost of tuition, books, etc. but also the income forgone (opportunity cost) by studying rather that
working.

Quantitative & Qualitative Relevant Information:


Quantitative factors are outcomes that are measured in numerical terms. Some quantitative factors are
financial; they can be measured numerically, they can be expressed in monetary terms. Examples include
cost of direct materials, direct labour, etc. Other quantitative factors are non-financial; they can be measured
numerically, but they are not expressed in monetary terms. E.g. Increase in on time delivery from a production
process, reduction in customer waiting time from a decision to invest in additional cash dispensing machine,
reduction in the number of units of defective output delivered to customers arising from an investment in
quality inspection, reduction in new product-development time for a manufacturing company, % of on-time
flight arrivals for an airline company, etc.

Qualitative factors are outcomes that are difficult to measure accurately in numerical terms. E.g. Employee
Morale, loss of customer’s goodwill, etc.

Relevant Costs and Relevant Revenues


Relevant costs are expected future costs and relevant revenues are expected future revenues that differ
among the alternative courses of action being considered. Be sure you understand that to be relevant costs
and relevant revenues they must
 Occur in the future-every decision deals with selecting a course of action based on its expected
future results; and
 Differ among the alternative courses of action-costs and revenues that do not differ will not matter
and, hence will have no bearing on the decision being made.

Examples of costs which are not relevant are past expenditures on capital, depreciation of capital assets,
accounting gains or losses on the sale of assets, and sunk costs of any type. Sunk costs are costs made as
a result of a prior decision and are not relevant to the current decision as they cannot be altered by it.
CA. Parag Gupta Ph.: +91 11 47665555 Paraggupta_ca@yahoo.co.in Costing & O.R.
World’s largest CA Final student’s consultancy group: http://groups.yahoo.com/group/costingbyparaggupta
Cost Accounting & Management - 183 -

Unfortunately, accounting profits or losses on asset disposal result from the comparison of the sunk cost
involved in its purchase (less any depreciation) as compared to the revenue generated by disposal. The
prospect of accounting losses on disposal of an asset are often enough to discourage a manager from
making a beneficial decision to replace the asset. Idle or unused capacity also has no value in a relevant cost
decision since capacity already exists due to prior decisions and will continue to exist even it remains unused.
Shared facilities or production processes are not relevant where the process would occur regardless of the
number of intermediate products produced (greater than one).
Remember, Expenses incurred on conducting a market survey to assess the potential market or associated
with research and development activities well before the product is considered for introduction are sunk costs
for a product. Further, labour payment on piece rate system is always relevant, since there is no question of
payment on idle time in such cases.

Costs
(i.e. Cash Flows)

Relevant costs Irrelevant/Non-Relevant costs

1. Occurs in
future. Sunk costs Those future costs
2. Differ that don’t differ
among among different
alternate alternatives. E.g.
Past/Historical Committed Identical material
course of
action. costs costs costs under different
In other words, alternatives, Future
the amount by sale value of
These costs are past It includes costs which are machine under
which costs not yet paid but the
costs and cannot be different
increase and decision to incur has
changed by any alternatives,etc.
benefit already been taken in past.
current or future
decrease as a It should be contrasted with
action. E.g.
direct result of discretionary costs which
Purchase price of
a specific are avoidable costs. E.g.
material, market
management Payment to permanent
survey expenses,
decision labour.
etc.

Remember, Relevant costs are cash flows. This means that costs or charges which do not reflect
cash spending (additional), should be ignored. Hence following shall be ignored:
 Depreciation
 Notional Rent
 All absorbed overheads – Fixed overheads absorption is always irrelevant although
variable overhead costs are usually relevant (because they should be incurred at the same
rate at which they are absorbed).

Question 1: “Sunk cost is irrelevant in decision-making, but irrelevant costs are not sunk costs”. Explain with
example. (4 Marks) May/06 & (4 Marks) Nov./09-O.C.
Ans.: Sunk costs are costs that have been created by a decision made in the past and that cannot be
changed by any decision that will be made in the future. For example, the written down value of assets

CA. Parag Gupta Ph.: +91 11 47665555 Paraggupta_ca@yahoo.co.in Costing & O.R.
World’s largest CA Final student’s consultancy group: http://groups.yahoo.com/group/costingbyparaggupta
Decision Making - 184 -
(including Relevant Costing, Make or Buy,
Subcontracting, Shut Down Point, etc.)
previously purchased are sunk costs. Sunk costs are not relevant for decision making because they are past
costs.
But not all irrelevant costs are sunk costs. For example, a comparison of two alternative production methods
may result in identical direct material costs for both the alternatives. In this case, the direct material cost will
remain the same whichever alternative is chosen. In this situation, though direct material cost is the future
cost to be incurred in accordance with the production, it is irrelevant, but, it is not a sunk cost.
Question 2: Explain with one example each that sunk cost is irrelevant in making decisions, but irrelevant
costs are not sunk costs. (2 Marks) May/01
Sunk cost is a historical cost incurred in the past. In other words it is a cost resource already acquired. Further
decision in respect of this resources will not be affected by it For example, book value of machinery. Hence
sunk costs are irrelevant in decision making.
Irrelevant cost are not necessarily sunk costs. For example, when a comparison of two alternative production
methods using the same materials quantity is made, then direct materials cost is not affected by the decision
but this materials cost is not sunk cost.
Question 3: Briefly explain the concept of ‘Opportunity Costs’. (4 Marks) Nov/96
Question 4: What are incremental costs and sunk costs? Discuss. (3 Marks) Nov/98

Question 5: What is meant by Incremental Revenue? (4 Marks) Nov/97


Question 6: What are relevant costs? Identify two common pitfalls in relevant cost analysis.(4 Marks) Nov /98
Ans.: Relevant costs are those expected future costs which are essential to a decision. The two key aspect of
these costs are as follows :
1. They must be expected future costs.
2. They must be different among the alternative courses of action.
For example, in a decision relating to the replacement of an old machine, the written down value of the
existing machine is not relevant but its sale price is relevant.
Relevant cost analysis helps in drawing the attention of managers to those elements of costs which are
relevant for the decision. Two common pitfalls in relevant cost analysis are as under :-
First pitfall in relevant cost analysis is to assume that all variable costs are relevant : All variable costs are not
relevant. Even among future costs, those variable costs which will not differ under various alternatives are
irrelevant. For example, a company proposes to rearrange plant facilities and estimates its future costs under
two alternatives, as under :
Particulars Do not re-arrange Re-arrange
` `
Direct material cost/unit 10 10
Direct labour cost/unit 5 4
In the above example, the direct material cost (variable cost) remains constant under both alternatives, hence
it is irrelevant to the decision as to whether plant facilities are to be re-arranged or not.
Second pitfall is to assume that all fixed costs are irrelevant: All fixed costs are not irrelevant. If fixed
expenses remain unchanged under different alternatives such expenses are only irrelevant to the decision at
hand but if they are expected to be altered they should be considered as relevant.
For example, if the plant capacity is 50,000 units and additional 10,000 units can only be manufactured by
expanding capacity which entails additional fixed expenses of `50,000. This increase in fixed expenses in
relevant to the decision whether the firm should accept order for additional 10,000 units or not.
Question 7: Distinguish between “Marginal cost” and ‘Differential Cost”. (5 Marks) May/99
Ans.: Marginal cost represents the increase or decrease in total cost which occurs with a small change in
output say, a unit of output. In Cost Accounting variable costs represent marginal cost.
Differential cost is the change (increase or decrease) in the total cost (variable as well as fixed) due to change
in the level of activity, technology or production process or method of production.
In other words, it can be defined as the cost of one unit of product or service which would be avoided if that
unit was not produced or provided.
The main point which distinguishes marginal cost and differential as that change in fixed cost when volume of
production increases or decreases by a unit of production. In the case of differential cost variable as well as
fixed cost. i.e. both costs change due to change in the level of activity, whereas under marginal costing only
variable cost changes due to change in the level of activity.

CA. Parag Gupta Ph.: +91 11 47665555 Paraggupta_ca@yahoo.co.in Costing & O.R.
World’s largest CA Final student’s consultancy group: http://groups.yahoo.com/group/costingbyparaggupta
Cost Accounting & Management - 185 -

Question 8: Explain the concept of relevancy of cost by citing three examples each of relevant costs and
non-relevant costs. (4 Marks) Nov./08-O.C.

Question 9: Explain the concepts of Opportunity costs and Relevant costs. (4 Marks) June/09-N.C.

Question 10: What are the applications of incremental/ differential costs? (5 Marks) May/10-N.C.
Ans.: The areas in which Application of Incremental/ differential cost techniques of cost analysis can be used
for making managerial decisions are:
 Make or buy decisions.
 Dropping or adding a product line
 Whether to process a product further or not.
 Acceptance of an additional order from a special customer at lowers than existing price.
 Equipment replacement decision. original
 Optimizing investment plan.

CA. Parag Gupta Ph.: +91 11 47665555 Paraggupta_ca@yahoo.co.in Costing & O.R.
World’s largest CA Final student’s consultancy group: http://groups.yahoo.com/group/costingbyparaggupta
Decision Making - 186 -
(including Relevant Costing, Make or Buy,
Subcontracting, Shut Down Point, etc.)

Case 1. Opportunity cost of material already purchased & having no other use in current production
process and having no realizable value:- Suppose a concern was using till last year a particular type of
material P under its production process. It has already purchased 5,000 kg. of material P during last year @
`2 per kg. and placed in its godown, This material is now not used by the concern under its current production
and there is no demand of this material in the market so it is decided by the management that the concern will
either give this material to someone free of cost or throw it away without incurring further cost and get its
storage space free for other use. By chance the concern receives one offer to produce a product which will
require exactly 5,000 kg. of this material P. What will be the opportunity cost of material required to be used
for proposal under consideration?
Space for

The opportunity cost of 5,000kg. of this material P for the offer under consideration is nil, which is
lecture
notes

measured from the best alternative use of the available material, i.e., either to give this material to
someone free of cost or through it away without incurring further cost.

Case 2. Opportunity cost of material already purchased and having no other use in current production
process but having realizable value:- Suppose in the above example if 5,000kg. of material P can be sold
in the market @ `1.25.per kg. then the opportunity cost (best alternative use) of the material for the offer
under consideration will be `6,250.
Space for
lecture
notes

Case 3. Opportunity cost of material required under proposed production, only some portion thereof
is held in stock out of past purchase and having no other use in current production process but
having realizable value:- Suppose there is an offer to manufacture a particular product which will require
12,000 kg. of material P. It is currently available in the market @ `3 per kg. If the concern has already
purchased 5,000 kg. of material P during last year @ `2 per kg. and placed in its godown . This material is
now not used by the concern under its current production process, and this 5,000 kg. of material P can be
sold in the market @ `1.25 per kg. What will be the opportunity cost of material required to be used for
proposal under consideration?
Space for

The opportunity cost (best alternative use) of 12,000 kg. of material required to be used under the
lecture
notes

offer under consideration will be the sum total of the opportunity cost of material in hand i.e.
`6,250 and current purchase price of the rest 7,000 kg. of material P i.e.`21,000. In
effect the opportunity cost of the material required to be used for proposal under
consideration will be `27,250.

Case 4. Opportunity cost of material already purchased & having no other use in current production
process and having no realizable value but can be disposed by incurring some additional cost:-
Suppose a concern was using till last year a particular type of material P under its production process which is
of highly toxic nature. It has already purchased 5,000 kg. of material P during last year @ `2 per kg. and
placed in its godown. This material is now not used by the concern under its current production and it can be
disposed of by incurring an additional cost of `500. By chance the concern receives one offer to produce a
product which will require exactly 5,000 kg. of this material P. What will be the opportunity cost of material
required to be used for proposal under consideration?
The opportunity cost of 5,000 kg. of this material P for the offer under consideration is (-) `500 i.e.
Space for

there will be saving of additional disposal cost of `500.


lecture
notes

Case 5. Opportunity cost of material already purchased & held in godown and having regular use in
current production process:- Suppose a concern uses material X in its routine production. During the last
year it purchased material in bulk quantity @ `7 per kg. Out of the aforesaid purchases 40000 kg. of material
X is still in godown . The same material is now available in the market @ `9 per kg. (replacement cost).There

CA. Parag Gupta Ph.: +91 11 47665555 Paraggupta_ca@yahoo.co.in Costing & O.R.
World’s largest CA Final student’s consultancy group: http://groups.yahoo.com/group/costingbyparaggupta
Cost Accounting & Management - 187 -

is a proposal to produce one special equipment, which requires 8,000 kg. of the aforesaid material. What will
be the opportunity cost of material required to be used for proposal under consideration?
Space for

The opportunity cost of 8,000 kg. of material required to be used under the proposal under
lecture
notes

consideration will be its replacement cost i.e.`72,000.

If we will use 8,000 kg. of material X under that proposal then we have to purchase the same
material from the market to meet the requirement under the regular production process and it will then cost us
`72,000

Case 6. Opportunity cost of material already purchased and can be used in regular production as a
substitute material:- Suppose there is a proposal under consideration which require 5,000 kg. of material
A. During the last year the concern purchased material A in bulk quantity @ `7 per kg. Out of the aforesaid
purchase 5,000 kg. of material A is still in godown. If this material is not used for the proposal under
consideration it can be used in place of 5,000 kg. of a substitute material Z which is in regular use. The
current replacement cost of material Z is `8 per kg. What will be the opportunity cost of material required to
be used for proposal under consideration?
Space for

Since 5,000 kg. of material A can be used in place of 5,000 kg. of material Z. If it is not used under
lecture
notes

the given proposal then it will be used in place of material A which will other wise required to be
purchased for `40,000 so the opportunity cost of 5,000 kg. of material A will be equal to the
replacement cost of 5,000 kg. of material Z, i.e.`40,000.

Case 7. Opportunity cost of material already purchased , then can be used after suitable alteration in
place of other material or can be disposed off:- Suppose there is a proposal under consideration which
require 5,000 kg. of material A. During the last year the concern purchased material A in bulk quantity @ `10
per kg. Out of the aforesaid purchases 5,000 kg. of material A is still in godown. If this material is not used for
the proposal under consideration it can be used after suitable alteration in place of 5,000 kg. of material B
which is in regular use. The current replacement cost of material B is `8 per kg. The alteration cost of material
A to be used in place of material B is `2 per kg. and current resale value of material A is `4 per kg. What will
be the opportunity cost of material A required to be used for proposal under consideration?
Space for
lecture

Since 5,000 kg. of material A, having two uses- (i) It can be used in place of 5,000 kg. of material
notes

Band alteration cost of material A will be `10,000. (ii) It can be sold for `20,000. We have to place
the value under the better alternative (higher value of these two alternatives)

Value under alternative 1. If material A is not used under the given proposal then it will be used in place of
material B which will otherwise required to be purchased for `40,000 so the opportunity cost of 5,000 kg. of
material A will be equal to the replacement cost of 5,000 kg. of material B as reduced by the alteration cost
`10,000, i.e. alternative 1 is `30,000.

Value under alternative 2 Resale value of 5,000 kg. of material A = `20,000


The higher of the two alternatives which is `30,000 be taken as opportunity cost of material A.

Question 11: X Ltd. has been approached by a customer who would like a special job to be done for him
and is willing to pay `22,000 for it. The job would require the following materials

Material Total Units Book value of Realizable Replacement


units already in units in stock value cost `/unit
required stock `/unit `/unit
A 1,000 0 - - 6
B 1,000 600 2 2.5 5
C 1,000 700 3 2.5 4
D 200 200 4 6 9

a) Material B is used regularly by X Ltd. and if stocks were required for this job they would need to be
replaced to meet other production demand.

CA. Parag Gupta Ph.: +91 11 47665555 Paraggupta_ca@yahoo.co.in Costing & O.R.
World’s largest CA Final student’s consultancy group: http://groups.yahoo.com/group/costingbyparaggupta
Decision Making - 188 -
(including Relevant Costing, Make or Buy,
Subcontracting, Shut Down Point, etc.)
b) Materials C and D are in stock as the result of previous excess purchase and they have a restricted
use. No other use could be found for material C but material D could be used in another job as
substitute for 300 units of Material E, which currently cost `5 per unit (of which the company has no
units in stock at the moment).
What are the relevant costs of material, in deciding whether or not to accept the contract?
Assume all other expenses on this contract to be specially incurred beside the relevant cost of material
are `550.
[Ans.: Total Relevant Cost is `16000; Contract should be accepted since offer is for `22000 in relation to
relevant cost of `16000]
Question 12: A company owns a machine which was purchased three years ago for `18,000. Depreciation
based on useful life of six years with no salvage value has been recorded each year. The present written
down value of equipment is `9,000. Management is considering replacing this machine with a new machine
which will reduce the variable operating costs. The new machine will cost `7,000 and will have expected life
of three years with no scrap value. The variable operating costs are Re. 0.30 per unit of output for the old
machine and Re. 0.20 per unit for the new machine. It is expected that both the machines will be operated at
their maximum capacity of 20,000 units per annum. The current disposal or sale value of the old machinery is
`4,000.

Should the company retain or replace? Assume (exclusively for complicity) that Re. 1 of each inflow or outflow
in year 1 is equal to Re. 1 of each outflow in, say, year 3.

[Ans.: It is beneficial to replace the equip. (`3000)]


Question 13: S Limited is engaged in manufacturing activities. It has received a request from one of its
important customers to supply a product which will require conversion of material ‘M’, which is a non-moving
item.
The following details are available:

Book value of material M `60


Realisable value of material M `80
Replacement cost of material M `100

It is estimated that conversion of one unit of ‘M’ into one unit of the finished product will require one labour
hour. At present, labour is paid at the rate of `20 per hour. Other costs are as follows:
Out-of-pocket expenses `30 per unit
Allocated overheads `10 per unit

The labour will be re-deployed from other activities. It is estimated that the temporary redeployment will not
result in loss of contribution. The employees to be re-deployed are permanent employees of the company.

Required:
Estimate the minimum price to be charged from the customer so that the company is not worse off by
executing the order. (4 Marks) Nov/07

[Ans.: `110 per unit]


Question 14: Mr. X has taken a shop on lease and made a down payment of `2,50,000. Additionally, the rent
under lease amount is `96,000 per annum. If lease agreement is cancelled by Mr. X, then the initial payment
is forfeited. Mr. X plans to use the shop for the general stores business, and has estimated operations for the
next year as follows:

Sales `25,00,000
Less : Value added tax (VAT) `2,80,000
Sales after VAT 22,20,000
Cost of goods sold 12,50,000
Wages and wages related cost 2,76,000
Rent including down payment 3,46,000
Rates, lighting and insurance 2,80,000
Audit, legal and general expenses 50,000 22,02,000
CA. Parag Gupta Ph.: +91 11 47665555 Paraggupta_ca@yahoo.co.in Costing & O.R.
World’s largest CA Final student’s consultancy group: http://groups.yahoo.com/group/costingbyparaggupta
Cost Accounting & Management - 189 -

Net profit before tax 18,000

In the business, Mr. X will be devoting half of his time, however no provision has been made for his
remuneration/salary. Mr. X also has an option to sublet the shop to his friend for a monthly rent of `18,000, if
he does not use the shop himself.

You are required to:


1. Identify the sunk and opportunity cost in the above problem.
2. State most profitable decision, which should be taken by Mr. X, supporting with appropriate
calculation.(11 Marks) CIMA Foundation-Adapted, (ICWA-June/01)-Adapted & (5 Marks) Nov/09-O.C.

[Ans.: Net Relevant Income from own business: `268000; Net Relevant Rental Income: `120000, hence X
should start his own business]
Question 15: A company has prepared the following budget for the forthcoming year:

`lakhs
Sales 20.00
Direct materials 3.60
Direct labour 6.40
Factory overheads:
Variable 2.20
Fixed 2.60
Administration overheads 1.80
Sales commission 1.00
Fixed selling overheads 0.40
Total costs 18.00
Profit 2.00

The policy of the company in fixing selling prices is to charge all overheads other than the prime costs on the
basis of percentage of direct wages and to add a mark up of one - ninth of total costs for profit.

While the company is confident of achieving the budget drawn up as above, a new customer approached the
company directly for execution of a special order. The direct materials and direct labour costs of the special
order are estimated respectively at ` 36,000 and `64,000. This special order is in excess of the budgeted
sales as envisaged above. The company submitted a quotation of `2,00,000 for the special order based on its
policy. The new customer is willing to pay a price of `1,50,000 for the special order. The company is hesitant
to accept the order below total cost as, according to the company management, it will lead to a loss.

You are required to state your arguments and advise the management on the acceptance of the special
order. (7 Marks) Nov./08-N.C.
[Ans.: Net Benefit in accepting the order is of `28,000]

Question 16: A company had nearly completed a job relating to construction of a specialized equipment,
when it discovered that the customer had gone out of business. At this stage, the position of the job was as
under:
(`)
Original cost estimate 1,75,200
Cost incurred so far 1,48,500
Costs to be incurred 29,700
Progress payments received from original customer 1,00,000

After searches, a new customer for the equipment has been found. He is interested to take the equipment, if
certain modifications are carried out. The new customer wanted the equipment in its original condition, but

CA. Parag Gupta Ph.: +91 11 47665555 Paraggupta_ca@yahoo.co.in Costing & O.R.
World’s largest CA Final student’s consultancy group: http://groups.yahoo.com/group/costingbyparaggupta
Decision Making - 190 -
(including Relevant Costing, Make or Buy,
Subcontracting, Shut Down Point, etc.)
without its control device and with certain other modification. The costs of these additions and modifications
are estimated as under:

Direct material (at cost) `1,050


Direct wages Dept. : A 15 man days
Dept. : B 25 man days
Variable overheads 25% of direct wages in each department
Delivery costs `1,350
Fixed overheads will be absorbed at 50% of direct wages in each department.

The following additional information is available:


(i) The direct materials required for the modification are in stock and if not used for modification of this
order, they will be used in another job in place of materials that will now cost `2,250.
(ii) Department A is working normally and hence any engagement of labour will have to be paid at the
direct wages rate of `120 per man day.
(iii) Department B is extremely busy. Its direct wages rate is `100 per man day and it is currently
yielding a contribution of `3.20 per rupee of direct wages.
(iv) Supervisory overtime payable for the modification is `1,050.
(v) The cost of the control device that the new customer does not require is `13,500. If it is taken out, it
can be used in another job in place of a different mechanism. The latter mechanism has otherwise
to be bought for `10,500. The dismantling and removal of the control mechanism will take one man
day in department A.
(vi) If the convention is not carried out, some of the materials in the original equipment can be used in
another contract in place of materials that would have cost `12,000. It would have taken 2 man days
of work in department A to make them suitable for this purpose. The remaining materials will realize
`11,400 as scrap. The drawings, which are included as part of the job can be sold for `1,500

You are required to calculate the minimum price which the company can afford to quote for the new customer
as stated above. (12 Marks) May/01

[Ans.: Minimum price which the company can quote is `61975]


Question 17: A company has in stock, materials K which was purchased four years ago at `40,000. These
materials have become obsolete and can be sold as scrap for `10,000. There are two alternatives:

Alternative I
The material can be converted into a specialized product as per details given below:
Add materials AX : 600 kg.
Direct labour : 2000 skilled hours at `6 per hour
4000 unskilled hours at `4 per hour
Advertisement : `10,000
The resultant output of 500 units can be sold at `150 per unit.
Material AX is in regular use of the company. The company has 1000 kg. of AX in stock, purchased at `50 per
kg. Its replacement cost is `70 per kg. and the re-sale value is `40 per kg. Variable overheads for conversion
of the entire output is `2,000.

Alternative II
Material K can be used as a substitute in a regular job where it will replace the material which would
otherwise have to be purchased for `8,000. This job will earn a profit of `6,000.

The skilled labour is in short supply and they have to be diverted from other jobs where they earn a
contribution of `2 per hour. Unskilled labour is surplus to the extent of 10000 hours and they are being paid
under a contract which cannot be terminated.

You are required to present a statement advising as to how deal with the stock of material K.

[Ans.: It is suggested to scrap the material for `10000]

CA. Parag Gupta Ph.: +91 11 47665555 Paraggupta_ca@yahoo.co.in Costing & O.R.
World’s largest CA Final student’s consultancy group: http://groups.yahoo.com/group/costingbyparaggupta
Cost Accounting & Management - 191 -

Question 18: A Ltd. has been offered a contract that, if accepted, would significantly increase next year’s
activity level. The contract requires the production of 20,000 kgs. of product X and specifies a contract price of
`1,000 per kg. The resources required in the production of each kg. of X include the following:
Resources per kg. of X
Labour:
Grade 1 2 hours
Grade 2 6 hours
Material
A 2 units
B 1 liter
Grade 1 labour is highly skilled and although currently under-utilized in the firm, it is A’s policy to continue to
pay Grade 1 labour in full. Acceptance of the contract would reduce the idle time of Grade 1 labour. Idle time
payments are treated as non-production overheads.

Grade 2 is unskilled with a high turnover, and may be considered a variable cost.
The cost to A for each type of labour are: Grade 1 `40 per hour; Grade 2 `20 per hour.

The materials required to fulfill the contract would be drawn from the materials already in stock. Material A is
widely used within the firm and any usage for the contract will necessitate replacement. Material B was
purchased to fulfill an expected order that was not received. If, material B is not used for the contract, it will be
sold.

For accounting purposes FIFO is used. The various values and costs for A and B are as follows: (`p. u.)
Particulars A B
Book value 80 300
Replacement cost 100 320
Net realizable value 90 250

A single recovery rate for fixed factory overheads is used throughout the firm, even though some of these
costs could be attributed to a particular product or department. The overhead is recovered by applying a pre-
determined rate per productive labour hour. Initial estimates of next year’s activity, which exclude the current
contract, show fixed production overhead of `60,00,000 and production labour hours of 3,00,000. Acceptance
of the contract would increase fixed production overheads by `22,80,000.

Variable production overheads are accurately estimated at `30 per productive labour hour.

Acceptance of the contract would encroach on the resources used to produce and sale another product Y,
which is also made by A Ltd. It is estimated that the sale of Y would then decrease by 5,000 units in the next
year. However, this reduction in sale of Y would enable attributable fixed factory overhead of `5,80,000 to be
avoided. Information on Y is as follows: (Per unit)
Selling price `700
Labour Grade 2 4 hours
Materials relevant variable costs `120

Required:

Advise A Ltd. on the desirability of the acceptance of the contract purely on economic considerations. Show
your calculations: (15 Marks) May/02

[Ans.: Accept the contract as the pre-tax operating income is `200000]


Question 19: Asha Road Carriers is a transporting company that transports goods from on place to another.
It measures quality of
service in terms of :

(i) Time required to transport goods


(ii) On-time delivery
(iii) Number of lost or damaged cartons.

CA. Parag Gupta Ph.: +91 11 47665555 Paraggupta_ca@yahoo.co.in Costing & O.R.
World’s largest CA Final student’s consultancy group: http://groups.yahoo.com/group/costingbyparaggupta
Decision Making - 192 -
(including Relevant Costing, Make or Buy,
Subcontracting, Shut Down Point, etc.)
To improve its business prospects and performance the company is seriously considering to install a
scheduling and tracking system, which involves an annual outlay of `1,50,000, besides equipments costing
`2,00,000 needed for installation of the system. The company proposes to utilise the proceeds of the fixed
deposit maturing next month to purchase the equipment. The rate of interest at present on deposit is 10%.
The company furnishes the following information about its present and anticipated future performance:

Current Expected
On-time delivery 85% 95%
Variable costs per carton lost or damaged `50 `50
Fixed costs per carton lost `30 `30
Number of cartons lost or damaged 3000 1000

The company expects that each per cent point increase in on-time performance will result in revenue increase
of `18,000 per annum. Contribution margin of 45% is required. Should Asha Road Carries acquire and install
the new system? (11 Marks) Nov./03

[Ans.: Increase in net saving `11000, so new systems should be installed]


Question 20: B Ltd. is having a big plant where tailor-made jobs are carried out. Recently a customer has
approached them for a job as per specification supplied. B Ltd. does not want to lose the customer and is
ready to quote a lower price. The planning engineer was asked to prepare a material requirement as per the
specifications. The cost estimates worked out as under:

(1) Steel sheets 5000 kg at `15 per kg. `75000


(2) Steel rods 1000 kg at `10 per kg 10000
(3) Bearing hardware items, etc, 15000
(4) Employee Costs:
Monthly rated – grade A 400 hours at `10 4000
Monthly rated – grade B 600 hours at `8 4800
(5) Overheads:
Fabrication shop – 500 hours at `20 10000
Welding shop – 300 hours at `40 12000
Planning Engineers – 200 hours at `15 3000
Design Engineers – 100 hours at `15 1500
Total Estimated Cost 135000

Following additional information is available:

(1) The stocks of steel sheets are more than sufficient and were purchased a year ago. Present market
price of this item is `12 per kg.
(2) The steel rods were purchased five years back at `10 per kg. Present purchase price is `18 per kg.
This material is already declared as non-moving and can be sold in market as such at `15 per kg. or
can be substituted for alloy steel rods which are presently costing `17 per kg.
(3) The labour force is always moved from job to job depending upon urgency. It is likely that the above
job, if accepted, will have to be done by grade A workers alone.
(4) The fabrication shop is treated as profit centre. A transfer price of `20 per hour is used for charging to
other shops in the workshop. The fabrication shop also done jobs for outsiders whom `25 per hour is
charged. The transfer price are calculated as under:

Fabrication Welding
Variable Cost per machine hour `7 `16
Departmental Fixed Costs 6 20
Profit 7 4
Transfer Price 20 40

(5) The hourly rates of planning/design engineers are `10 per hour. However, for outside consultancy
work, it is practice to charge `15 per hour.

CA. Parag Gupta Ph.: +91 11 47665555 Paraggupta_ca@yahoo.co.in Costing & O.R.
World’s largest CA Final student’s consultancy group: http://groups.yahoo.com/group/costingbyparaggupta
Cost Accounting & Management - 193 -

The management wants to have the bare minimum cost for the job so that the opportunity of getting the order
is not lost.
Revise the cost estimate using the additional information. Give reasons for each of the revised figure used in
your calculations.

[Ans.: Total estimated relevant costs is `109300]


Question 21: You are the management accountant of publishing and printing company which has been
asked to quote for the production of programme of the local village fair. The work would be carried out in
addition to the normal work of the company. Because of existing commitments, some weekend working would
be required to complete the printing of the programme. A trainee accountant has produced the following cost
estimate based upon the resource required as specified by the production manager:
Direct Material - paper (book value) `5000
- inks (purchase price) 2400
Direct Labour - skilled 250 hours @ `4.00 1000
- unskilled 100 hours @ `3.50 350
Variable Overhead 350 hours @ `4.00 1400
Printing press depreciation 200 hours @ `2.50 500
Fixed production costs 350 hours @ `6.00 2100
Estimated department cost 400
13150
You are aware that considerable publicity could be obtained for the company if you are able to win this order
and the price quoted must be very competitive.

The following notes are relevant to the cost estimate above:

(1) The paper to be used is currently in stock at a value of `5000. It is of an unusual colour which has
not been used for some time. The replacement price of paper is `8000, whilst the scrap value of
that in stock is `2500. The production manager does not forsee any alternative use for the paper if
it is not used for the village fair programmes.
(2) The inks required are not held in stock. They would have to be purchased in bulk at a cost of
`3000. 80% of the ink purchases would be used in printing the programmes. No other use in
forseen for the remainder.
(3) Skilled direct labour is in short supply, and to accommodate the printing of the programmes, 50%
of the time required would be worked at weekends for which a premium of 25% above the normal
hourly rate is paid. The normal hourly rate is `4.00 per hour.
(4) Unskilled labour is presently under-utilised, and at present 200 hours per week are recorded as
idle time. If the printing work is carried out at weekend, 25 unskilled hours would have to occur at
this time, but the employees concerned would be given two hours time off (for which they would be
paid) in lieu of each hour worked.
(5) Variable overhead represents the cost of operating the printing press and binding machines.
(6) When not being used by the company, the printing press is hired to outside companies for `6.00
per hour. This earns a contribution of `3.00 per hour. There is unlimited demand for this facility.
(7) Fixed production costs are those incurred by and absorbed into production, using an hourly rate
based on budgeted activity.
(8) The cost of estimating department represents time spent in discussions with the village fair
committee concerning the printing of its programme.

Required:

(a) Prepare a revised cost estimate using the opportunity cost approach, showing clearly minimum price
that the company should accept for the order. Give reasons for each resources valuation in your cost
estimate.
(b) Explain why contribution theory is used as a basis for providing information relevant to decision
making.
(c) Explain the relevance of opportunity costs in decision making.

[Ans.: Revised Cost Estimate: `8625]

CA. Parag Gupta Ph.: +91 11 47665555 Paraggupta_ca@yahoo.co.in Costing & O.R.
World’s largest CA Final student’s consultancy group: http://groups.yahoo.com/group/costingbyparaggupta
Decision Making - 194 -
(including Relevant Costing, Make or Buy,
Subcontracting, Shut Down Point, etc.)
[Hint: Total unskilled hours are 100 hours, out of which 75 hours will be utilized on weekdays & 25 hours will
be utilized on weekends. Since on weekend, employees concerned would be given two hours time off (for
which they would be paid) in lieu of each hour worked, hence hours paid for carrying 25 hours work will be 75
(25+50). Therefore, total hours required to carry 100 hours work = 150 (75 weekdays + 75 weekends) & idle
time available = 200 hours. Hence relevant cost of unskilled labour will be NIL]
Question 22: A research project, to date, has cost a company ` 250000 and is under review. It is anticipated
that, should the project be allowed to proceed, it will be completed in about one year and an be sold for `
400000. The following additional information is available:

(i) Materials have just been received for ` 60000. These are extremely toxic, and if not used in the project,
have to be disposed of by special means at ` 15000.
(ii) Labour: ` 75000. The men are highly skilled. If they are released from the Research Project, they may
be transferred to the Works Department of the company and consequently the sales could increase by
`150000. the accountant estimated that the prime cost of those sales would be `100000 and the
overhead absorbed (all fixed) would amount to ` 25000.
(iii) Research staff: ` 160000. A decision has already been taken that this will be the last major piece of
research undertaken and consequently, when work on the project ceases, the staff involved will be
made redundant. Redundancy and severance pay have been estimated at ` 25000.
(iv) Share of General Building expense: `35000.

The Managing Director is not sure what is included in this amount, but the accounts staff charge similar
amounts each year to each department.

You are required to advise whether the project should allowed to proceed and explain the reasons for the
treatment of each of the amounts above in your analysis. (10 Marks) May/07
[Ans.: Project should be allowed to proceed as it will provide incremental cash inflows of `130000]
Question 23: A company manufacturing several products for regular sales has conducted a market survey at
a cost of ` 1,00,000 to introduce a new product NP. The market survey suggests that there is a demand for
the sale of 1,00,000 units of NP at `18 each for one year.

The following information has been furnished by the company:

(i) Raw Materials: Each article of NP requires one unit of each of the three types of the raw materials
namely A, B, and C. Material A is in regular use of the company and the stock are replaced as and
when exhausted. Material B is not in regular use of the company but as a result of overbuying in an
earlier contract, the company at present holds a stock of 60,000 units. Material C is used only in NP
and hence the company has to purchase the same as per the requirement of production of the new
product. The data relating to three the items of raw materials are as under:

Raw Material Current Stock Cost per unit of raw material


(units)
Original cost Current Current resale
(`) replacement cost
(`) (`)
A 1,00,000 2.00 2.50 1.75
B 60,000 3.50 3.00 1.00
C - - 6.00 5.00

(ii) Direct labour: NP requires for each article:


Skilled labour 0.25 hours at `3 per hour and unskilled labour 2 hours at `2 per hour. Due to shortage
to skilled labour, the company has, in the event of deciding to take up the production of NP, to divert
the skilled labour from some other product which earns a contribution of `2 per hour of skilled
labour. The company has a surplus of 3,00,000 hours of unskilled labour for which payment is being
made one time basis as per contract and it is not possible to terminate these surplus workers.
(iii) Additional Staff required for the manufacture of NP:
One foreman at ` 36,000 p.a.
One supervisor at `24,000 p.a.

CA. Parag Gupta Ph.: +91 11 47665555 Paraggupta_ca@yahoo.co.in Costing & O.R.
World’s largest CA Final student’s consultancy group: http://groups.yahoo.com/group/costingbyparaggupta
Cost Accounting & Management - 195 -

(iv) Machines: Two machines namely Machine Type P and Machine Type Q are required to produce
NP. Machine Type P is in regular use on the other products also and Machine Type Q is now idle. If
NP is not produced machine Type Q can be sold immediately. The relevant data relating to each
type of machine are as under:
(`)
Particulars At the start of the year At the close of the
year
Type P – Replacement cost 1,60,000 1,30,000
Resale value 1,20,000 94,000
Type Q – Replacement cost 26,000 18,000
Resale value 22,000 17,000

The company charges depreciation on Straight Line Basis.


(v) Overheads:
Fixed overheads of the company ` 18,00,000 per annum.
Variable overheads `1.50 per unit of the new product NP.

Using the concept of relevancy of costs, prepare a Cost Sheet to show the cost per unit of the new product
NP. Substantiate the figures with necessary explanation.

[Ans.: Total Cost is `1400000 & cost per unit is `14]


Question 24: A company has been making a machine to order for a customer, but the customer has since
gone into liquidation, and there is no prospect that any money will be obtained from the winding up of the
company.

Costs incurred to date in manufacturing the machine are `50,000 and progress payment of `15,000 have
been received from customer prior to the liquidation.

The sales department has found another company willing to buy the machine for `34,000 once it has been
completed. To complete the work, the following cost would be incurred:-

(a) Materials these have been bought at a cost of `6,000. They have no other use, and if the machine is
not finished, they would be sold for scrap for `2,000.
(b) Further labour costs would be `8,000. Labour is in short supply, and if the machine is not finished,
the work force would be switched to another job, which would earn `30,000 in revenue, and incur
direct cost (not including direct labour), of `12,000 and absorbed (fixed) overhead of `8,000.
(c) Consultancy fees `4,000. If the work is not completed, the consultant’s contract would be cancelled
at a cost of `1,500.
(d) General overheads of `8,000 would be added to the cost of the additional work.

Should the new customer’s offer be accepted? Prepare a statement showing the economics of the
proposition. May/91

[Ans.: In view of incremental profit of `11500, the offer of new customer can be accepted]
Question 25: SV Ltd manufacturing a product called FLOTAP. The company is organized into two divisions,
viz, Division A and Division B.

Division A manufactures FLOTAP and Division B, which manufactures the containers, packs FLOTAP in the
containers and stores them by using a special protective material called ‘germicide’. The details of the
expenses incurred by Division B during 2002 under:
(`)

CA. Parag Gupta Ph.: +91 11 47665555 Paraggupta_ca@yahoo.co.in Costing & O.R.
World’s largest CA Final student’s consultancy group: http://groups.yahoo.com/group/costingbyparaggupta
Decision Making - 196 -
(including Relevant Costing, Make or Buy,
Subcontracting, Shut Down Point, etc.)
Direct material including germicide 420,000
Direct labour 3,00,000
Supervision 48,000
Maintenance of machine 21,600
Rent of a party of warehouse used 27,000
Depreciation of machinery 90,000
Miscellaneous overheads 94,500
Administration overheads apportioned to the Division 1,44,000
Total 11,45,100

PH Ltd., a company, engaged in warehousing of a variety of products approached SV Ltd. to undertake to


manufacture the containers required on contract basis for a period of four years for 7,50,000 per annum
and/or store the packed product for a further sum of `1,50,000 per annum.

Division B uses a machine for manufacture of containers. This machine was installed four years ago at a
capital cost of `7,20,000 and it has a useful life of four more years. It can be currently sold at `1,50,000.
Division B purchased germicide worth `6,00,000 during the last year. Out of this, one-fifth was used during
the last year and the cost thereof is included in the material cost of 2002. The original purchase price of
germicide was `3,000 per tonne but, if sold now, the stock of germicide would fetch only `2,400 per tonne. Its
current replacement cost is `3,600 per tonne.

Division B hired a warehouse for storage of the product for `54,000 per annum. It uses only half of the space
and has taken only half the amount of rent into account. The remaining space of warehouse is idle.
If the contract for manufacture of the containers and the storage of the product, FLOTAP is given to PH Ltd.
Division B will be closed down. In that event the supervisory staff will be transferred to another department.
The terminal benefits to be met as regards workers will amount to `45,000.
If SV Ltd. continues to store the FLOTAP and leaves the manufacture of the containers to PH Ltd. the
machine in Division B will not be required and the storage space requirements cannot be dispensed with. The
supervisory staff will be required to be retained in Division B and only 10% of all materials will be used. The
saving on account of labour retrenchment will come to `15,000 per annum. The miscellaneous overheads will
reduced by 80%.

If SV Ltd. continues to manufacture the containers and leaves the storage of FLOTAP to PH Ltd, Division B
will retain the machine and the warehouse space for use. The supervisory staff will also be retained and 90%
of all materials will be required. The labour force will continue and the miscellaneous overheads will be
reduced by 20%

You are required to evaluate the above three proposals on four year term basis and state your
recommendations.

[Ans.: Net cash outflow in Alt. I is `1022100, Alt. II is `2012400 & Alt. III is `3512700. All alternatives result in
net cash outflow, Therefore it is in interest of SV Ltd. to continue and to manufacture containers and store
them in Division B.]
Question 26: Soft Drinks Ltd. bottles and distributes ‘Amrit’ brand cold drinks. It operates its distribution
division as a cost centre. Budgeted cost for the year ending 31st March, 1996 is as follows:
(`)
Cash Operating Costs 21,00,000
Depreciation on Fleet of Vehicles (8X `52,500) 4,20,000
Apportioned Corporate Costs 3,00,000
28,20,000

Distribution division has started operation on 1st April 1994. Each vehicle of the fleet was acquired at a cost of
`2,40,000 and had an estimated economic life of four years. Salvage value of each vehicle at the end of four
years (March 31, 1998) was estimated at `30,000.

CA. Parag Gupta Ph.: +91 11 47665555 Paraggupta_ca@yahoo.co.in Costing & O.R.
World’s largest CA Final student’s consultancy group: http://groups.yahoo.com/group/costingbyparaggupta
Cost Accounting & Management - 197 -

Countrywide Distributors Ltd. which has countrywide network for the distribution of food and beverages has
offered Soft Drinks Ltd. a three years distribution contract for `19,50,000 each year. The contract will start on
1st April 1995.

If Soft Drinks Ltd accepts the offer, it will close down its own distribution division, and will sell the delivery
vehicles. Current (April 1, 1995) disposal price of each vehicle is estimated at `75,000. Soft Drinks Ltd. will
avoid cash operating cost of `21,00,000.

Security analysts have recommended the purchase of share of Soft Drinks Ltd., security analysts are
forecasting a net profit of `6,60,000 for 1995-96 as against an estimated profit of `6,30,000 for 1994-95, the
forecast assumes that the company will continue operation of its distribution division.

Required:
(a) Tabulate a comparison of all relevant cost for next three years (1995-96 to 1997-98) for the two
alternatives-use of own distribution division or use of countrywide distributors. Recommend whether
Soft Drinks Ltd. should accept the offer of Countrywide distributors. (11 Marks) May/95
(b) Why might Soft Drinks Ltd. be reluctant to accept the offer of Countrywide distributors? (8 Marks)
(Ignore Income-tax and time value of money. Wherever appropriate, suitable assumptions to be made by you)

[Ans.: Relevant Cost: 95-96 96-97 97-98


Own Dis. 2100 2100 1860
Countrywide Dis. 1350 1950 1950]
Question 27: A Ltd. produces and markets a range of consumer durable appliances. It ensures after-sales
service through X Ltd. The big appliances are serviced at customer’s residence while small appliances are
serviced at workshop of X Ltd.

The material supplied to X Ltd. is charged above cost at 10%. X Ltd. charges customers at 25% over the
above price. For labour, the company receives 10% of the rate fixed for work done under the after-sales
service agreement and 15% of the rate fixed in case of jobs not covered under the agreement from X Ltd.
60% by value of the total work undertaken by X Ltd. was for big appliances and rest accounted for small
appliances during the previous year.

The company decides to carry out all or some of the work itself and has chosen one area in the first instance.
During the previous year the company earned a profit of `2,16,000 as detailed below from X Ltd. for the area
chosen:
(`)
Particulars Material Labour
Under after-sales service agreement 60,000 1,00,000
For jobs not covered under the agreement 20,000 36,000

The company forecasts same volume of work in that area for the ensuring period. The following three options
are under consideration of the management:

(i) To set up a local service centre to provide service for small appliances only. The existing system is
to continue for big appliances.
(ii) To set up a local service centre to provide service for big appliances only. The existing system is to
continue for small appliances.
(iii) To set up a local service centre to provide service to all appliances. The existing system then stands
withdrawn.

The relevant costs for carrying out jobs under the above options are under: (`‘000)
Particulars Option-1 Option-2 Option-3
Heat, rent light etc. 125 50 150
Management costs 108 83 150
Service staff cost 230 440 750
Transport cost 25 220 230

You are required to find out the most profitable option. (15 Marks) Nov./96

CA. Parag Gupta Ph.: +91 11 47665555 Paraggupta_ca@yahoo.co.in Costing & O.R.
World’s largest CA Final student’s consultancy group: http://groups.yahoo.com/group/costingbyparaggupta
Decision Making - 198 -
(including Relevant Costing, Make or Buy,
Subcontracting, Shut Down Point, etc.)

[Ans.: Net Benefit in Option -1 `(‘000) 184, in Option -2 `(‘000) 445, in Option -3 `(‘000) 884, hence it is
advisable to choose any of the options.]
Question 28: Mahila Griha Udyog Industries is considering to supply its products-a special range of
namkeens to a departmental store. The contract will last for 50 weeks, and the details are given below:
(`)
Material:
X (in stock-at original cost) 1,50,000
Y (on order-on contract) 1,80,000
Z (to be ordered) 3,00,000
Labour:
Skilled 5,40,000
Non-Skilled 3,00,000
Supervisory 1,00,000
General Overheads 10,80,000
Total cost 26,50,000
Price offered by departmental store 18,00,000
Net Loss 8,50,000

Should the contract be accepted if the following additional information is considered?

(i) Material X is an obsolete material. It can only be used on another product, the material for which is
available at `1,35,000 (Material X requires some adaptation to be used and cost `27,000).
(ii) Material Y is ordered for some other product which is no longer required. It now has a residual value
of `2,10,000.
(iii) Skilled labour can work on other contracts which are presently operated by semi-skilled labour at a
cost of `5,70,000.
(iv) Non-skilled labours are specifically employed for this contract.
(v) Supervisory staff will remain whether or not the contract is accepted. Only two of them can replace
other positions where the salary is `35,000.
(vi) Overheads are charged at 200% of skilled labour. Only `1,25,000 would be avoidable, if the contract
is not accepted. (12 Marks) May/99

[Ans.: Since there is a net incremental cash flow of `152000 therefore, contract should be accepted]
Question 29: Ranka Builders has been offered a contract by Excel Ltd. to build for its five special Guest
Houses for use by top management. Each Guest House will be an independent one. The contract will be for a
period of one year and the offer price is `one crore. In addition, Excel Ltd. will also provide 2 grounds of land
free of cost for the purpose of construction. The Chief Accountant of Ranka Builders has prepared an
estimate on the basis of which he has advised that the contract should not be accepted at the price offered.
His estimate was as follows:
(`Lakhs)
Land (3 Grounds at `20 lacks each) 60
Drawing and Design 7
Registration 10
Materials:
Cement and Sand 6
Bricks and Tiles 4
Steel 10
Other (including interior decoration) 10
Labour -Skilled 12
-Unskilled 8
-Supervisor’s Salary 5
Overheads- General 12
Depreciation 6
Total 150

The Accountant also provides the following information:

CA. Parag Gupta Ph.: +91 11 47665555 Paraggupta_ca@yahoo.co.in Costing & O.R.
World’s largest CA Final student’s consultancy group: http://groups.yahoo.com/group/costingbyparaggupta
Cost Accounting & Management - 199 -

Land: The total requirement of land is 3 grounds costing `20 lakhs per ground. Excel Ltd. will provide 2
grounds free of cost.
Drawing and Design: These have already been prepared and 50% of the cost has already been incurredSee
Note 1
.
Materials:
(i) Cement and sand are already in stock and in regular use. If used for this contract, they have to be
replaced at a cost of `8 lakhs.
(ii) Bricks and tiles represent purchases made several months before for a different contract. They
could be sold readily for a net `5 lakhs after meeting all further expenses.
(iii) Others: Materials worth `2 lakhs relating to interior decoration are in stock for which no alternative
use is expected in the near future. However, they can be sold for `1 lakh.
Labour:
(i) Skilled workers will be transferred to this project from another project. The Project Manager claimed
that if the men were returned to him, he could have earned the company an additional `2 lakhs in
See Note 2
term of profits .
(ii) The supervisor undertakes various tasks in the sites and his pay and continuity of employment will
not be affected by the new contract. If the contract is taken, he will devote half of his time.
Overheads:
(i) The equipment that would be used on the contract was bought one year before for `30 lakhs and is
expected to last for five years. It can also be used on other contracts and the current replacement
price will be `32 lakhs and in a year’s time it will be `25 lakhs.
(ii) The general overheads includes both specific and absorbed overheads. If the contract is not
undertaken, `4 lakhs of the same can be avoided.

Ranka Builders has also on hand another project, which would not be executed if the contract from Excel Ltd.
were to be accepted. The estimated profit on that project is `10 lakhs.
In the light of information given above, you are required to indicate with reasons whether the contract from
Excel Ltd. should be accepted or not. (10 Marks) Nov./99

[Ans.: Total relevant cost is `10850000; Contract price is `1 crore. So, the offer should not be accepted.]
[Note: 1. Suggested answers of ICAI has assumed that Designs & Drawings are 50% paid although 100%
incurred-This assumption seems grossly wrong.
2. Suggested answers of ICAI has assumed that amount of `2 lakhs is without deducting labour costs-This
assumption seems grossly wrong]
Question 30: B Ltd. is a company that has, in stock materials of type XY that cost `75,000, but that are now
obsolete and have a scrap value of only `21,000. Other than selling the material for scrap, there are only two
alternative uses for them.

Alternative I: Converting the obsolete materials into a specialized product, which would require the following
additional work and materials:
Material A 600 units
Material B 1,000 units
Direct Labour
5,000 hours unskilled
5,000 hours semi-skilled
5,000 hours highly skilled
Extra selling and delivery expenses `27,000
Extra advertising `18,000

The conversion would produce 900 units of saleable product and these could be sold for `300 per unit.
Material A is already in stock and is widely used within the firm. Although present stocks, together with orders
already planned, will be sufficient to facilitate normal activity and extra material used by adopting this
alterative will necessitate such materials being replaced immediately. Material B is also in stock, but it is
unlikely that any additional supplies can be obtained for some considerable time, because of an industrial
dispute. At the present time Material B is normally used in the production of Product Z, which sells at `390 per
unit and incurs total variable cost (excluding Material B) of `210 per unit. Each unit of Product Z uses 4 units
of Material B. The details of Material A and B are as follows:
(`)

CA. Parag Gupta Ph.: +91 11 47665555 Paraggupta_ca@yahoo.co.in Costing & O.R.
World’s largest CA Final student’s consultancy group: http://groups.yahoo.com/group/costingbyparaggupta
Decision Making - 200 -
(including Relevant Costing, Make or Buy,
Subcontracting, Shut Down Point, etc.)
Material A Material B
Acquisition cost at the time of purchase 100 per unit `10 per unit
Net realizable value 85 per unit `18 per unit
Replacement cost 90 per unit -

Alternative II: Adopting the obsolete materials for use as a substitute for a sub-assembly that is regularly
used within the firm. Details of the extra work and materials required are as follows:
Material C 1,000 units
Direct Labour:
4,000 hours unskilled
1,000 hours semi-skilled
4,000 hours highly skilled
1,200 units of the sub-assembly are regular used per quarter at a cost of `900 per unit. The adaptation of
material XY would reduce the quantity of the sub-assembly purchased from outside the firm to 900 units for
the next quarter only. However, since the volume purchased would be reduced, some discount would be lost
and the price of those purchased from outside would increase to `1,050 per unit for that quarter.
Material C is not available externally though 1,000 units required would be available from stocks; it would be
produced as extra production. The standard cost per unit of Material C would be as follows: (`)
Direct labour : 6 hours unskilled labour 18
Raw materials 13
Variable overhead : 6 hours at `1 6
Fixed overheads : 6 hours at `3 18
55

The wages rate and overheads recovery rates for B Ltd. are:
Variable overhead `1 per direct labour hour
Fixed overheads `3 per direct labour hour
Unskilled labour `3 per direct labour hour
Semi-skilled labour `4 per direct labour hour
Highly skilled labour `5 per direct labour hour

The unskilled labour is employed on a casual basis and sufficient labour can be acquired to exactly meet the
production requirements. Semi-skilled labour is part of the permanent labour force, but the company has
temporary excess supply of this type of labour at the present time. Highly skilled labour is in short supply and
cannot be increased significantly in the short-term, this labour is presently engaged in meeting the demand for
product L, which requires 4 hours of highly skilled labour. The contribution from the sale of one unit of product
L is `24.

Given the above information, you are required to present cost information advising whether the stock of
Material XY should be sold, converted into a specialized product (Alternative I) or adopt for use as a
substitute for a sub-assembly (Alternative II). (16 Marks) Nov./00
[Ans.: Net relevant revenue of Alternative I is `20000 and Alternative II is `12000]
Question 31: Engineers Ltd. is just ready to deliver a machine specially designed for Durables & Co. when it
is learnt that the latter has gone bankrupt.

An enquiry comes from another firm, Steady Enterprises, which can accept the machine meant for Durable &
Co. if certain alterations are done to suit Steady Enterprises needs and the price is attractive.

The following factors prevail:


(a) Costs incurred on the machine for Durables & Co.
Direct Material `560000
Direct Labour 400000
Variable Overhead 140000
Fixed Overhead 300000
Fixed Selling & Dist. Overhead 1500000

Notes: If the negotiation with Steady Enterprises fails, part of the material used may be dealt with as under:
(a) (I) Brass materials – could be sold as scrap for `100000.

CA. Parag Gupta Ph.: +91 11 47665555 Paraggupta_ca@yahoo.co.in Costing & O.R.
World’s largest CA Final student’s consultancy group: http://groups.yahoo.com/group/costingbyparaggupta
Cost Accounting & Management - 201 -

(II) Steel materials – could be sold as scrap for `26000, but to sell it as scrap some 100 hours labour
will be hired at `10 per hour to bring it to saleable condition.
(III) Balance Materials will have to be removed at a cost of `5000, but will have a ‘nil’ sale value.
(b) Price quoted to Durables & Co. was 1800000.
(c) To cater to Steady Enterprises needs, alteration cost will be:

Department M Department A
Direct Materials `10000 `5000
Direct Labour 10 men for 2 months @ `3000 per 6 women for 2 months @ `2000
man-month per woman-month
Variable Overhead 20% of Direct labour cost 25% of Direct labour cost
Fixed Overhead 60% of Direct labour cost 50% of Direct labour cost

Notes: (c) (I) Materials required are already in stock and valued at cost. If the work for Steady Enterprises is
not undertaken, the company has the following choice:
- Material for Department M will be used for another job.
- Material for Department A, lying as it is for some years, will remain useless unless put on quick sales
for `3000.
The present market prices for the materials for M and A are `12000 and `6000 respectively.
(III) Department M is current working at full capacity, earning a contribution of `3 towards fixed
overhead and profit per Re. 1 of labour.
(IV) Department A is presently working at 40% of its capacity, but as per agreement with the Union its
present work force of 24 women cannot be reduced. A worker in this department gets `2000 a
month as wages. In order to utilize its labour, Department A undertakes some off-loading work for
`32500 per month from a sister concern when the workload in Department A falls below 50%
capacity. Variable cost associated with off-loading work is `4000 per month. The conversion work
for Steady Enterprises will mean 25% additional workload for Department A for 2 months.
(d) The pattern and specifications of the original machine could be sold to a customer for `60000.
(e) For supervision of the job for Steady Enterprises, a temporary Supervisor would be needed for 2
months at an agreed salary of `10000. He will be a person deputed by Steady Enterprises. The
company charges all indirect and supervisory salaries to fixed overhead.
(f) Durable & Co. has already made an earnest money deposit of `180000 for the machine. As per terms
of the contract, this deposit stands forfeited and Engineers Ltd. is now free to treat the sum as
miscellaneous income.
(g) Taxation may be ignored.

Engineers Ltd. seeks your advice for the minimum price, based on relevant costs only, for the quotation it will
make to Steady Enterprises.

[Ans.: Minimum Price is `520000]


Question 32: A construction company has accepted contract AX and work thereon is about to begin.
However, the company has received an offer for another contract BX. The company cannot, due to certain
constraints, take up both the contracts simultaneously. In case the company is desirous of taking up contract
BX, it can get the first contract AX rescinded upon payment of a penalty of `70,000.
The following are the estimates relating to the two contracts:
(`)
Particulars Contract AX Contract BX
Material X-in stock at original cost 54,000 -
Material Y-in stock at original cost - 62,000
Material X-firm orders placed at original cost 76,000 -
Material X-Not yet ordered (at current cost) 1,50,000 -
Material-Z Not yet ordered (at current cost) - 1,78,000
Labour-to be engaged and paid for 2,15,000 2,75,000
Site Management Costs 85,000 85,000
Travel and other expenses 17,000 14,000
Depreciation of Plant 24,000 32,000
Interest on Capital at 12% 12,800 16,000
Head Office expenses allocated to contracts 31,690 33,100

CA. Parag Gupta Ph.: +91 11 47665555 Paraggupta_ca@yahoo.co.in Costing & O.R.
World’s largest CA Final student’s consultancy group: http://groups.yahoo.com/group/costingbyparaggupta
Decision Making - 202 -
(including Relevant Costing, Make or Buy,
Subcontracting, Shut Down Point, etc.)
Total 6,65,490 6,95,100
Contract Price 7,20,000 8,80,000
Estimated Profit 54,510 1,84,900

The following additional information is available:


- Material X is not in regular use, it can be used as substitute for other materials, which are currently
quoted at 10% less than the original cost of X.
- Material Y is in regular use and its price has doubled since it was purchased. Its net realizable value
if sold will be its new price less 15%. It can, however, be kept in store for use in other contracts to be
taken up in the next year.
- If contract AX is undertaken, a part of the plant having spare capacity can be hired out for a rental of
`15,000 for the period.
- It is the policy of the company to charge notional interest on the estimated working capital at 12%
per annum.
st
- Either of two contracts can be completed by 31 March, 2003, which is the close of the company’s
financial year.
- Site management cost is fixed.

Required:

(i) Using the relevancy of cost concept prepare a comparative statement to show the net benefit
resulting from each contract.
(ii) Advise the management of the company as to which of two contracts should be undertaken.

[Ans.: (i)Expected net benefit of Contract AX `236000 and Contract BX `219000 (ii) Continue with Contract
AX.] (16 Marks) Nov./02
Question 33: Star Bicycle Company, produced and sold 1,10,000 bicycles annually, under the brand name
‘Smart’ with a price tag `899. Like all other players in the industry, Star too was running under capacity. The
manufacturing cost of these cycles was material `300, labour `200 and Manufacturing `300, 40% of the
manufacturing cost was variable. General and administration expenses were 50% of labour cost.

Star has now received a proposal to sell 25,000 bicycles per year under the brand name ‘Jeet’ to a chain
store at a price of `800. The brand will be exclusive for the chain stores as they will market it as their own
product. Expenditure for producing ‘Jeet’ will be the same as that of ‘Smart’ as design of ‘Jeet’ will exactly be
same as that of ‘Smart’ with only some cosmetic changes. To produce ‘Jeet’ however, `6,00,000 additional
fund will be required on an average. Further it estimated that sale of ‘Jeet’ through the chain store will reduce
the sale of ‘Smart’ by 10,000 units.

You are required to calculate the relevant cost of ‘Jeet’, given that the weighted average cost of capital Star
Co. is 15%. [ICWA-Dec/02]

[Ans.: Relevant Cost of ‘Jeet’ bicycle is `623.60]


Question 34: ACE Ltd. has an inventory of 5,000 units of a product left over from last year’s production. This
model is no longer in demand. It is possible to sell these at reduced prices through the normal distribution
channels. The other alternative is to ask someone to take them on “as is where is” basis. The latter alternative
will cost the company `5,000.

The company produced 2,40,000 units of the product last year, when the unit costs were as under:
(`)
Manufacturing Cost:
Variable 6.00
Fixed 1.00 7.00
Selling and Distribution Cost:
Variable 3.00
Fixed 1.50 4.50
Total Cost 11.50
Selling Price per Unit 14.00

CA. Parag Gupta Ph.: +91 11 47665555 Paraggupta_ca@yahoo.co.in Costing & O.R.
World’s largest CA Final student’s consultancy group: http://groups.yahoo.com/group/costingbyparaggupta
Cost Accounting & Management - 203 -

Required:

Should the company scrap the items or sell them at a reduced price? If you suggest the latter, what minimum
price would be recommend? (8 Marks) May/98

[Ans.: If the company can get anything more than `2 per unit, then it is worthwhile to sell the stock of 5000
units and earn an additional contribution.]
Question 35: A firm furnishes the following information:

Capacity in Units Unit Cost Unit Price


` `
2000 40 100
3000 35 95
4000 34 94
5000 32 -
6000 31 -

At present the firm is operating at 4000 units capacity and has received an order for 2000 units from an export
market at ` 28 per unit. Should the order be accepted? (4 Marks) May/00

[Ans.: The firm should accept the order]


Question 36: ABC Ltd. an LPG cylinder manufacturing unit, gets an order from Oil Corporation for supply of
40,000 cylinders at the standard price of `700 per cylinder. Getting orders is a major problem. If an order is
not executed in full for any reason not only will ABC Ltd. lose the benefit; but it stand the risk of the order
quantity being reduced for the next quarter also. The cylinders are made in two halves and then welded
together. The cost structure is:
Materials 12.5 kg. per half
Labour `50 per half
Welding charges and fitting charges would be `30 per cylinder.

Unfortunately special steel plates are in short supply and ABC Ltd. has only stock of 500MT. Another friendly
unit which has in anticipation, made bottom halves, has offered to supply 40,000 numbers. What is the
maximum price ABC Ltd can pay to the unit for the bottom halves? Transportation and unloading etc. will cost
`5 per each half.

[Ans.: The price = `330 per bottom half. (`13200000÷40000)]


Question 37: Mr. Phillips owns a gift shop, a restaurant and a lodge in Shimla. Typically he operates these
only during the season period of four months in a year. For the past season the occupancy rate in the lodge
was 90% and level of activity in case of gift shop and restaurant at 80%.
The relevant data for the past seasons were as under:

Gift shop Restaurant Lodge


Particulars
Amount ` % Amount ` % Amount ` %
Receipts/Sales 48,000 100 64,000 100 1,80,000 100
Expenditure:
Cost of Sales 26,400 55 35,200 55 - -
Supplies 2,400 5 6,400 10 14,400 8
Insurance and Taxes 1,920 4 6,400 10 36,000 20
Depreciation 2,880 6 8,000 12.5 39,600 22
Salaries 4,800 10 4,800 7.5 25,200 14
Electricity Charges 960 2 3,200 5 13,500 7.5
Total 39,360 82 64,000 100 1,28,700 71.5
Profit 8,640 18 - - 51,300 28.5

CA. Parag Gupta Ph.: +91 11 47665555 Paraggupta_ca@yahoo.co.in Costing & O.R.
World’s largest CA Final student’s consultancy group: http://groups.yahoo.com/group/costingbyparaggupta
Decision Making - 204 -
(including Relevant Costing, Make or Buy,
Subcontracting, Shut Down Point, etc.)
Additional information:
(i) Cost of sales and supplies vary directly with the occupancy rate in case of lodge and level of activity
in case of gift shop and restaurant.
(ii) Insurance & Taxes and depreciation are for the entire period of twelve months.
(iii) Salaries paid are for the season period except a chowkidar for the lodge who is paid for the full year
at `400 per month.
(iv) Electricity charges include fixed charges of `640, `1,920 and `9,900 for gift shop, restaurant and
lodge respectively. The balance amount varies directly with occupancy rate in case of lodge and
level of activity in case of gift shop & restaurant. Fixed electric charges are for the season except in
case of lodge where `6900 is for the season and `3,000 for the entire period of twelve months.

Mr. Phillips is interested in increasing his net income. The following two options are under his consideration:-
(1) To continue the operations during the season period only by inserting advertisement in newspaper
thereby occupancy rate will reach 100% in case of lodge and 90% level of activity in respect of gift-
shop and restaurant. The costs of advertisement are estimated at `12,000.
(2) To continue operations throughout the entire period of twelve months comprising season period of
four months and off-season period of eight months. The occupancy rate is expected at 90% and
40% during season period and off-season period respectively in case of the lodge. The room rents
are bound to be reduced to 50% of the original rates during off-season period. The level of activity of
gift-shop and restaurant is expected at 80% and 30% during season and off-season period
respectively but 5% discount on the original rates will have to be offered during off-season period.

Which option is profitable? As a Cost Accountant would you like to suggest him any other alternative based
upon the above figures which can be adopted to earn more net profit. (Use incremental revenue and
differential cost approach). Nov./93

[Ans.: By adopting the Option 1, the net increase in incremental revenue by `120 (i.e. `11000-10880) over
the Option 2. Therefore, Option 1 is suggested to adopt.]
Question 38: S Ltd. is a civil engineering company based at Calcutta. Contracts are carried out under the
supervision of project managers who are send out from Head Office and remain on site for the duration of the
contract. The project managers recruits local labour, and arranges for plant and materials to be provided by
head office.

Sometimes ago, the company successfully tendered for two contracts which have now become mutually
exclusive. It is currently considering which of these to accept. Both jobs would last for 12 months.
The following information about each contract is available.
(`’000)
Particulars Nagpur Delhi
Contract price 170 180
Penalty Payment (this is condition of the tender, if offered the
job and it is not accepted) 16 8
Materials required:
In stores (at cost) 20 24
Contacted for - 36
To be ordered (at current cost) 40 34
Labour required:
Project manager’s salary 10 10
Travel, lodgings etc. 4 4
Local recruitment 70 56

Head Office:
Plant depreciation 6 6
Interest on plant 2 2
General administration 8 8

Notes:
(1) The materials which would be used on the Nagpur job have increased in money value by 60% over
their purchase cost. S Ltd. has no other use for these materials on any other contract apart from

CA. Parag Gupta Ph.: +91 11 47665555 Paraggupta_ca@yahoo.co.in Costing & O.R.
World’s largest CA Final student’s consultancy group: http://groups.yahoo.com/group/costingbyparaggupta
Cost Accounting & Management - 205 -

the Nagpur one, but they could be re-sold to other companies in the industry at 90% of their value.
Transportation and other selling cost would further decrease the cash inflow from the sale by
16.67% of the sales price.
(2) The materials for the Delhi job have no other obvious use, but could be sold for scrap if the
contract were cancelled. The scrap value would be 10% of cost and cost of transport etc. would be
paid by the scrap merchant. It is likely, however, that the material could be used next year on
another contract in substitution for a different material normally costing 20% less than the cost of
the materials to be used on the Delhi contract.
(3) Local labour can be hired as and when required.
(4) Plant is depreciated on a straight line basis, and the interest on plant charge is a nominal cost
added for accounting purpose.
(5) The two contracts would require similar plant, although more plant would be required for the Delhi
than for the Nagpur job. The plant not required on the Nagpur job would be sub-contract our by
Head Office for `2,000 per annum.
(6) Head Office administration costs are fixed at `25,000 for the coming year. This excludes project
manager’s salaries.

Required:

(i) Present the data of management in a form which will assist in making the decisions to which job to
undertake. Provide notes to explain the principles which have been used in selecting the data and to
support any calculations made.
(ii) Comment on the appropriateness of the approach used in your analysis.
(iii) List briefly any other factors which would ought to be considered before finally making the decision
in this case

Question 39: A company manufactures two products ‘AB’ and ‘CD’ by utilizing 25% and 40% of its total
capacity respectively. The cost data per unit for 1994-95 are as under:

Particulars ‘AB’ ‘CD’


Production & sales (units) 5,000 10,000
Selling price (`) 80 100
Direct material (`) 10 30
Direct labour (`5 per hour) (`) 25 20

Variable overheads are 100% on wages. Fixed overheads for 1994-95 amounted to `2,25,000.

During 1995-96, the company expects that the direct material costs will rise by 5%, the labour hourly rate will
rise by 25 paisa and variable overheads will continue to maintain same relationship with wages as was in
1994-95.

For the same volume of output as was in 1994-95, the selling price is to be enhanced by 5% in case of ‘AB’
and 4% in case of ‘CD’.

The company has the following proposals for consideration of the management for 1995-96 to improve
profitability:
(a) Utilize the balance capacity to produce ‘AB’ and to sell this increased production at the existing
selling price of `80
(b) Utilize the balance capacity to produce ‘CD’. While doing so the efficiency will however go down by
16% on account of newly recruited labour in respect of this increased production. Fixed selling price
and distribution expenses of `50,000 will have to be spent to sell this additional output.
(c) Introduce new product ‘EF’ to utilize the balance capacity. One unit of ‘EF’ can be manufactured in 7
labour hours. Direct material will cost `40 per unit. Its selling price per unit will be `145. Variable
overheads will maintain same ratio to wages as for other two products. To boost the sales of ‘EF’
special advertisement expenses of `30,000 will be spent.

The present allocation of 25% and 40% capacities for ‘AB’ and ‘CD’ cannot be changed and only the spare
capacity is required to be used for production under aforesaid proposals.
Required:

CA. Parag Gupta Ph.: +91 11 47665555 Paraggupta_ca@yahoo.co.in Costing & O.R.
World’s largest CA Final student’s consultancy group: http://groups.yahoo.com/group/costingbyparaggupta
Decision Making - 206 -
(including Relevant Costing, Make or Buy,
Subcontracting, Shut Down Point, etc.)
(i) Present a statement of profit for 1994-95.
(ii) Using incremental revenue and differential cost approach, find out which proposal is more profitable
for 1995-96.
(iii) Present a statement of profit for 1995-96 based on above recommendation.

[Ans.: (i) Profit `175000 (ii) Net incremental value of proposal A, B & C `119000, 115375, `127500
respectively; (iii) Total Profit `312500] May/10 (RTP-N.C.)-Adapted & (19 Marks) Nov./95
[Note: While calculating Net incremental value of proposals, Suggested answers of ICAI has taken increase
in profit due to inflation as well, which seems grossly wrong assumption.]
Question 40: ZED Ltd. operates two shops. Product A is manufactured in Shop-1 and customer’s jobs
against specific orders are being carried out in Shop-2. Its annual statement of income is:
(`)
Particulars Shop-1 Shop-2 Total
(Product-A) (Job Works)
Sales/Income 1,25,000 2,50,000 3,75,000
Material 40,000 50,000 90,000
Wages 45,000 1,00,000 1,45,000
Depreciation 18,000 31,500 49,500
Power 2,000 3,500 5,500
Rent 5,000 30,000 35,000
Heat and Light 500 3,000 3,500
Other Expenses 4,500 2,000 6,500
Total Costs 1,15,000 2,20,000 3,35,000
Net Income 10,000 30,000 40,000

The depreciation charges are for machines used in the shops. The rent and heat and light are apportioned
between the shops on the basis of floor area occupied. All other costs are current expenses identified with the
output in a particular shop.

A valued customer has given a job to manufacture 5,000 units of X for shop 2. As the company is already
working at its full capacity, it will have to reduce the output of product A by 50%, to accept the said job. The
customer is willing to pay `25 per unit of X. The material and labour will cost `10 and `18 respectively per
unit. Power will be consumed on the job just equal to the power saved on account of reduction of output of A.
In addition the company will have to incur additional overheads of `10,000.

You are required to compute the following in respect of this job:


(a) Differential cost; (b) Full cost; and (c) Opportunity cost; and (d) Sunk cost.
Advise whether the company should accept the job. (19 Marks) May/96

[Ans.: (a) `105250; (b) `162750; (c) `16750; (d) 12750. The company should not accept the job because
there will be a cash disadvantage `42750]

Make or Buy

Question 41: ABC Ltd. produces a variety of products each having a number of components parts. Product B
takes 5 hours to produce on a particular machine which is working at full capacity. B has a selling price of
`100 and variable cost of `60 per unit. A component part X-100 could be made on the same machine in two
hours at a variable cost of `10 per unit. The suppliers’ price for the components is `25 per unit.
Required: Advise whether the company should buy the component X-100
(If necessary make suitable assumptions) (5 Marks) May/95

[Ans.: Relevant cost (p.u.) `26 & Supplier’s price (p.u.) `25. Since the supplier’s price is less than relevant
cost of production, therefore, it is advisable to buy X-100 from outside suppliers.]
Question 42: Product ‘A’ takes five hours to produce on a particular machine and it has a selling price of ` 50
and a marginal cost of `35
On the same machine, another product ‘B’ can be made at two hours at a marginal cost of ` 5 per unit.
Supplier’s price of product ‘B’ is ` 10 per unit.
CA. Parag Gupta Ph.: +91 11 47665555 Paraggupta_ca@yahoo.co.in Costing & O.R.
World’s largest CA Final student’s consultancy group: http://groups.yahoo.com/group/costingbyparaggupta
Cost Accounting & Management - 207 -

Assuming that machine hour is the key factor, advice whether product ‘B’ could be bought out or
manufactured. (6 Marks) Nov./99

[Ans.: Product B should be bought]


Question 43: A company manufactures three components. These components pass through two of the
company’s departments P and Q. The machine hour capacity of each department is limited to 6000 hours in a
month. The monthly demand for components and cost data are as under:

Components A B C
Demand (units) 900 900 1350
` ` `
Direct Material/units 45 56 14
Direct labour/units 36 38 24
Variable Overheads/unit 18 20 12
Fixed overheads P @ `8 per hour 16 16 12
Q @ `10 per hour 30 30 10
Total 145 160 72
Components A and C can be purchased from market at `129 each and `70 each respectively.
You are required to prepare a statement to show which of the components in what quantities should be
purchase to minimize the cost. (7 Marks) Nov./02

[Ans.: 250 units of component A should be purchased from outside]


Question 44: Household Equipments Ltd. is producing a kitchen equipment from five components, three of
which are made using general purpose machines and two by manual labour. The data for the manufacture of
the equipment is as follows:
Components A B C D E Total
Machine Hours reqd. per unit 10 14 12 - - 36
Labour Hours reqd. Per unit - - - 2 1 3
Variable cost per unit (in `) 32 52 58 12 6 160
Fixed cost per unit 48 102 116 28 34 316
(apportioned)
Total component cost 80 154 174 40 40 476
Assembly cost /unit (all `40
variable)
Selling price /unit `600
The marketing department for the company anticipates 50% increase in demand during the next period.
General purpose machinery used to manufacture A, B and C is already working to the maximum capacity of
4,752 hours and there is no possibility of increasing this capacity during the next period. But, labour is
available for making components D and E also for assembly according to demand. The management is
considering the purchase of one of the components A, B or C from the market to meet the increase in
demand. These components are available in the market at the following prices;
Component A : ` 80
Component B : `160
Component C : `125
Required
(a) Profit made by the company from current operations.
(b) If the company buys any one of the components A, B or C, what is the extent of additional capacity
that can be created?
(c) Assuming 50% increase in demand during the next period, which component should buy from the
market?
(d) The increase in profit, if any, if the component suggested in (c) is purchased from the market.

[Ans.: (a) `11088; (b) A: 38.5% B: 63.6% C: 50% (c) Product C (d) Increase in profit `13134]
ICWA-Dec./95
Question 45: GG Ltd manufactures and sells equipment called water purifier. The cost data for each batch of
ten number of water purifier is as follows:

Components A B C D E
CA. Parag Gupta Ph.: +91 11 47665555 Paraggupta_ca@yahoo.co.in Costing & O.R.
World’s largest CA Final student’s consultancy group: http://groups.yahoo.com/group/costingbyparaggupta
Decision Making - 208 -
(including Relevant Costing, Make or Buy,
Subcontracting, Shut Down Point, etc.)
Machine Hours 20 28 24 - -
Labour Hours - - - 4 2

` ` ` ` `
Variable Costs 64 108 116 24 8
Fixed Cost as Apportioned 36 52 64 26 22
Assembly cost (all variable) ` 50 per batch.
Selling price ` 800 per batch

Maximum available machine capacity for making components A, B and C is 10800 hours and it con not be
increased further. Labour is available for making components D and E and for assembling the product.
Estimated increase in demand next year is 50% and fixed cost in general may increase by ` 10000.

In order to release production capacity to meet increased market demand, the company decides to purchase
one of the machine made components.

Quote ltd is the only supplier of component A, B, and C. because of incomplete records; it is unable to quote
single figure prices. Its quotation is as follows:

Component Pessim- Proba- most lik- Proba- Optimist- Proba-


Istic view ility ly view bility ic view bility
` ` ` ` ` `
A 120 0.25 110 0.5 80 0.25
B 200 0.25 130 0.5 140 0.25
C 160 0.25 140 0.5 120 0.25

It is agreed between the companies that the price of each of the components will be determined on an overall
basis based on information found in the quotation.

You are required to:

(i) Indicate, in the context of the key factor, the maximum number of batches that could be produced, if
each of the three alternative namely buying A B or C is considered (8 Marks)
(ii) Analyse the financial implication of purchase and advise which components is to be bought keeping
in view the fact that production capacity will be limited to a 50% increase. (4 Marks)
(iii) Prepare a profit statement for the period assuming that the component chosen by you is bought out
and extra production is made and sold. (4 Marks) May/00

[Ans.: Purchased from outside


A B C
(i) No. of batches produced internally
(in batches) 207.69 245.45 225
(ii) Component C should be purchased from outside as it gives maximum contribution of `91,350
(iii) Profit on manufacturing of 225 batches `51,350]
Question 46: Stirling Industries Ltd., manufactures a product ‘Z’ by making and assembling three
components A, B and C. The components are made in a machine shop using three identical machines each
of which can make any of the three components. However, the total capacity of the three machines is only
12,000 machine-hours per month and is just sufficient to meet the current demand. Labour for assembling is
available according to requirements. Further details are given below:

Components Machine-house Variable cost per Market price at which


required per unit unit the component can
be purchased if
required
A 4 `48 `64
B 5 60 75
C 6 80 110
Assembling - 30 (per unit of z) -

CA. Parag Gupta Ph.: +91 11 47665555 Paraggupta_ca@yahoo.co.in Costing & O.R.
World’s largest CA Final student’s consultancy group: http://groups.yahoo.com/group/costingbyparaggupta
Cost Accounting & Management - 209 -

Fixed costs per month amount to `50,000. Product ‘Z’ is sold at `300 per unit.

From next month onwards. The company expects the demand for ‘Z’ to rise by 25%.As the machine capacity
is limited, the company wants to meet the increase in demand by buying such numbers of A, B or C which is
most profitable.

You are asked to find out the following:


(a) Current demand and profit made by the company.
(b) Which component and how many units of the same should be bought from the market to meet the
increase in demand?
(c) Profit made by the company if suggestion in (b) is accepted. ICWA-Dec./98

[Ans.: Profit-`15600 (b) Balance 600 units of B should be purchased from market (c) `23000]
Question 47: P Ltd manufacture plastic can of a standard size. The variable cost per can is ` 4 and the
selling price is ` 10 each. The factory of the company has eight machines of identical size. Any individual
machine can produce 30 cans per hour. The factory works on a 300 days per annum basis and actual
available hour per machine per day is 7.5. The company has an order of 420000 cans from an oil company, to
supply. The yearly fixed cost of the company is ` 20 lacs. P ltd has received an order from another firm for
supplying 60000 nos. of plastic moulded toys. The price of the toys is ` 60 each and the variable cost is ` 50
each. While this order would be acceptable for supplying for total quantities only, on acceptance, a special
mould costing ` 225000 would be required to be acquired to manufacture the toys. The time study exercise
has revealed that 15 nos. of toys can be produced per hour by any of the machines;

Advice the company, with reasons in the following situations:


(i) Whether to accept the order of manufacturing moulded toys, in addition to supplying 420000 nos.
of cans or not;
(ii) Whether to accept the order manufacturing moulded toys, if the order of cans increased to
540000 nos. or not;
(iii) While a sub- contractor is willing to supply the toys, either whole or part of the required quantities
at an all inclusive rate of ` 57.5 each, what would be the minimum excess capacity needed to
justify the manufacturing of any portion of the toys order, instead of sub-contracting
(iv) The company had an understanding that the order of the cans will be increased during the year
on negotiation, and planned and manufactured 450000 cans during the year. For utilizing the
excess capacity, they also accepted the toys order and sub-contracting only 15000 nos. of toys.
(v) At the year’s end, however, it was revealed that the order of the cans could be for 480000 nos, if
it was properly negotiated. How much loss has been suffered by the company due to improper
prediction of demand and negotiation? (17 Marks) Nov./01

[Ans.: (i) Accept the order, increase in profit by `375000; (ii) No; (iii) 2000 excess machine hours are
required; (v) `67500]
Question 48: XYZ Ltd. is currently manufacturing 5,000 units of the product ‘XY 100’ annually, making full
use of its machine capacity. The selling price & total cost p. u. associated with ‘XY 100’ are as follows:

`p.u. `p.u.
Selling price 900
Cost per unit:
Direct materials 200
Variable machine costs @`100 per hr. 150
Manufacturing overhead costs 180
Marketing and administrative costs 200 730
Operating income per unit of ‘XY 100’ 170
XYZ Limited can sell additional 3,000 units of ‘XY 100’, if it can outsource those additional units. ABC Limited,
a supplier of quality products, has agreed to supply upto 6,000 units of ‘XY 100’ per year at a price of ` 650
per unit delivered at XYZ’s factory.

CA. Parag Gupta Ph.: +91 11 47665555 Paraggupta_ca@yahoo.co.in Costing & O.R.
World’s largest CA Final student’s consultancy group: http://groups.yahoo.com/group/costingbyparaggupta
Decision Making - 210 -
(including Relevant Costing, Make or Buy,
Subcontracting, Shut Down Point, etc.)
XYZ Limited can use its facility to produce an alternative product ‘XY 200’ . It can sell up to 12,000
units of ‘XY 200’ annually. Estimated selling price and total costs per unit to manufacture and sell 12,000 units
of ‘XY 200’ are as follows:
` `
Selling price per unit 600
Costs per unit :
Direct materials 200
Variable machine costs @ `100 per hr. 50
Manufacturing overhead costs 60
Marketing and administrative costs 110 420
Operating income per unit of ‘XY 200’ 180

Other information pertaining to the operating of XYZ Limited is as follows:

(a) XYZ Limited use machine hours as the basis for assigning fixed manufacturing overhead. The fixed
manufacturing overhead for the current year is `3,00,000. These costs will not be affected by the product-mix
decision.
(b) Variable marketing and administrative costs per unit for various products are as follows:
Manufactured ‘XY 100’ `80
Purchased ‘XY 100’ `40
Manufactured ‘XY 200’ `60

Fixed marketing and administrative costs for the current year is `6,00,000. These costs will not be affected by
the product-mix decision.
Calculate the quantity of each product that XYZ Limited should manufacture and/or purchase to maximize
operating income. Show your calculations. (16 Marks) May/02

[Ans.: Manufacture “XY 200” 12000 units; “XY 100” 1000 units; Purchase “XY 100” 6000 units]
Question 49: Panchwati Cement Ltd. produces ’43 grade’ cement for which the company has an assured
market. The output for 2004 has been budgeted at 1,80,000 units at 90% capacity utilization. The cost sheet
based on output( per unit) is as follows :
`
Selling price 130
Direct material 30
Component ‘EH’ 9.40
Direct wages @ `7 per hour 28
Factory overhead (50% variable) 24
Selling and distribution overheads (75% variable) 16
Administrative overhead (fixed) 5

The factory overheads are applied on the basis of direct labour hours.

To utilize the idle capacity and to improve the profitability of the company, the following proposals
were put up before the Board of Directors for consideration :

i) An order has been received from abroad for 500 units of product ’53 grade’ cement per month at
`175 per unit. The cost data are :
Direct material `56 per unit, direct labour 10 hours per unit, selling and distribution overhead applicable to
this product order is `14 per unit and variable factory overhead are chargeable on the basis of direct labour
hours.
ii) The company at present manufacture component ‘EH’ one unit of which is required for each unit of
product ’43 grade’. The cost details for 15,000 units of component ‘EH’ are as follows:
`
Direct materials 30,000
Direct labour 52,500
Variable overheads 25,500
Fixed overheads 33,000
Total 1,41,000
CA. Parag Gupta Ph.: +91 11 47665555 Paraggupta_ca@yahoo.co.in Costing & O.R.
World’s largest CA Final student’s consultancy group: http://groups.yahoo.com/group/costingbyparaggupta
Cost Accounting & Management - 211 -

The component ‘EH’ however is available for purchase at the market at `7.90 per unit.
iii) In the event of company deciding to purchase the component ‘EH’ from market, the company
has two alternative for the use of the capacity so released, which are as under :
(a) Rent out the released capacity at Re. 1 per hour.
(b) Manufacture component ‘GYP’ which can be sold at ` 8 per unit. The cost data of this component for
15,000 units are:
`
Direct materials 42,000
Direct labour 31,500
Factory variable overheads 13,500
Other variable overheads 25,500
Total 1,12,500

Required:

i) Prepare a statement showing profitability of the company envisaged in the budget.


ii) Evaluate the export order and state whether it is acceptable or not.
iii) Make an appraisal of proposal to manufacture component ‘EH’ and state whether the component
‘EH’ should be manufactured in the factory or purchased form the market. Assume that no
alternative use of spare capacity is available.
iv) Evaluate the alternative use of the spare capacity and state whether to manufacture or buy the
component ‘EH’ and if your decision is to buy the component ‘EH’, which of the two alternatives
for the use of spare capacity will you prefer? (16 Marks) Nov./04

[Ans.: (i) Profit `3168000 (ii) Acceptable (iii) ‘EH’ should be manufactured in the factory (iv) ‘EH’ should be
purchased and ‘GYP’ should be manufactured.]
Question 50: New Vistas Ltd. is in the midst or make or buy decision. One of the components P44E
produced by the company and incorporated into another product before being sold, takes 4 hours of
manufacture and its budget manufacturing cost is as under:

Direct Material `14


Direct Labour `12
Variable Overheads ` 8
Fixed Overheads `20
54 per unit

i. One of its reliable suppliers has offered to supply P44E at a guaranteed price of `50 per unit. Should
the company accept the offer?
ii. Meanwhile a new situation has arisen. As per a recent Government Regulation, if P44E is continued
to be manufactured, the company will incur additional inspection and testing charges of `56,000 per
annum and this has not been included in the existed budgeted manufacturing cost of the component.
In this situation, what should the company do?
iii. As there is an embargo in the company on additional recruitment of labour, if P44E is not
manufactured, the direct labour released will be employed in increasing the production of an existing
product which takes 8 hours to make and is sold for `90. Its budgeted manufacturing cost is as under:

Direct Material `10


Direct Labour `24
Variable Overheads `16
Fixed Overheads `40
90 per unit

iv. The financial accountant is a bit perturbed. He points out the only last year, a special machine,
exclusive for manufacture of P44E, was bought at the cost of `1 lakh. It cannot be resold or used
elsewhere. If the production of this compeonent is stopped, a sum of `90,000 has to be written off
from the profits of the year, being the book value of the discarded machine.

What are your comments?


CA. Parag Gupta Ph.: +91 11 47665555 Paraggupta_ca@yahoo.co.in Costing & O.R.
World’s largest CA Final student’s consultancy group: http://groups.yahoo.com/group/costingbyparaggupta
Decision Making - 212 -
(including Relevant Costing, Make or Buy,
Subcontracting, Shut Down Point, etc.)

[Ans.: (i) Manufacture (ii) If it is proposed to manufacture more than 3500 components, then own
manufacturing will be profitable. If it is less than 3500 components, then it is advised to purchase. (iii) As the
limiting factor is labour hours, it is better to manufacture the another existing product and purchase the
component at the guaranteed price of `50, (iv) Book cost is irrelevant cost.]
Question 51: X is a multiple product manufacturer. One product line consists of motors and the company
produces three different models. X is currently considering a proposal from a supplier who wants to sell the
company blades for the motors line.

The company currently produces all the blades it requires. In order to meet customer’s needs, X currently
produces three different blades for each motor model (nine different blades).
The supplier would charge `25 per blade, regardless of blade type. For the next year X has projected the
costs of its own blade production as follows (based on projected volume of 10,000 units):

Direct materials `75,000


Direct labour 65,000
Variable overhead 55,000
Fixed overhead: -
Factory supervision 35,000
Other fixed cost 65,000
Total production costs 2,95,000

Assume (1) the equipment utilized to produce the blades bus no alternative use and no market value, (2) the
space occupied by blade production will remain idle if the company purchases rather than makes the blades,
and (3) factory supervision costs reflect the salary of a production supervisor who would be dismissed from
the firm if blade production ceased.

(i) Determine the net profit or loss of purchasing (rather than manufacturing) the blades required for motor
production in the next year.

(ii) Determine the level of motor production where X would be indifferent between buying and producing the
blades. If the future volume level were predicted to decrease, would that influence the decision?

(iii) For this part only, assume that the space presently occupied by blade production could be leased to
another firm for `45,000 per year. How would this affect the make or buy decision?

[Ans.: (a) Loss: `20000 (b) Indifference Point is 6364 units (c) Loss: `25000] (9 Marks) June/09-O.C.
Question 52: B Ltd. produces and sells Bicycles. It also manufactures the chains for its Bicycles. It expects to
produce and sell 24000 Bicycles during 1996-97. It is considering an offer from an outside vendor to supply
any number of chains at `12 per chain.

The accountant of B Ltd. reports the following costs for producing 24000 chains:

Cost Cost per unit (`) Total Cost (`)


Direct Material 5.00 120000
Direct Labour 4.00 96000
Variable manufacturing overhead 2.00 48000
Inspection, set-up, etc. 1.00 24000
Machine rent 1.00 24000
Allocated fixed overhead 1.25 30000
14.25 342000

The following additional information is available:

(i) Inspection, set-up, etc. vary with the number of batches in which the chains are produced.
Currently chains are being produced in the batch size of 2000 units.
(ii) Direct labour cost represents wages to four workers who are exclusively engaged in the
manufacturing of chains. These workers are in permanent capacity and cannot be retrenched.
CA. Parag Gupta Ph.: +91 11 47665555 Paraggupta_ca@yahoo.co.in Costing & O.R.
World’s largest CA Final student’s consultancy group: http://groups.yahoo.com/group/costingbyparaggupta
Cost Accounting & Management - 213 -

(iii) If B Ltd. procures all its chains from outside vendor, it will not require machine which it has hired
for manufacturing chains.

Required:

(i) Assume that if B Ltd., purchases chains from outside vendor, the facility (including workers) where
the chains are currently manufactured will remain idle. Should B Ltd. accept the offer from outside
vendor at the anticipated production and sale volume of 24000 units?
(ii) Whether your decision in (i) will change if facilities can be used to upgrade the bicycles which will
result in incremental revenue of `22 per bicycle. The variable cost of upgrading would be `18 and
tooling cost would be `16000.
(iii) Assuming that facilities will be used as stated in (ii) above. Further, assume that with better planning,
B Ltd. will be able to manufacture chains in batch size of 4000 units (instead of 2000 units) if it
decides to produce chains inside. (13 Marks) Nov./95

[Ans.: (i) Should not accept order from outside vendor, because this decision will lead to reduction in profit by
`72000 (ii) Should accept the offer of alternative use of facilities for upgrading the bicycle. It will lead to in
increase of `80000 in contribution. This is more than the excess of bought out price over variable and
avoidable cost. (iii) If inspection cost decreases, then excess of bought-out price over variable and avoidable
costs would be `84000. In comparision to this, net contribution from using the internal facilities for upgrading
quality of chains will `80000. Therefore, if batch size increases and inspection cost reduces, then use of
internal facilities of updation of quality of chain is advocated. If decision to update is taken in (ii), it will
increase profit by `4000]
Question 53: A company is engaged in manufacture of an electronic gadget. It produces 24,000 such
gadgets per annum. The company also manufactures 24,000 units of a component.
The department expenses per annum is as under:

Direct Materials `38,40,000


Direct Labour 15,36,000
Indirect Labour 7,20,000
Inspection and testing 4,80,000
Lighting 40,000
Power 4,80,000
Insurance 30,000
Depreciation (fixed) 96,000
Miscellaneous fixed expenses 54,000

If the company stops manufacturing the components and buys the same from market, the saving in the
department budget would be as under:

Direct Materials 20%


Direct Labour 25%
Indirect labour 20%
Inspection & testing 25%
Power 25%

The purchase price of the component is `70 each.

Required:

i. State whether the company should make or buy the components.


ii. The company has received an offer of 12,000 units of the electronic gadget at the price of `245 each.
If the offer is accepted by the company, the capacity will be fully utilized and the components have to
be purchased. Should the company then make or buy the component? (Nov./84)

[Ans.: (i) Co. should make the component {Net Benefit `6 per unit} (ii) Accept Export order and buy the
component (Net cash inflow `36000)]

CA. Parag Gupta Ph.: +91 11 47665555 Paraggupta_ca@yahoo.co.in Costing & O.R.
World’s largest CA Final student’s consultancy group: http://groups.yahoo.com/group/costingbyparaggupta
Decision Making - 214 -
(including Relevant Costing, Make or Buy,
Subcontracting, Shut Down Point, etc.)
Subcontracting

Question 54: Lee Electronics manufacture four types of electronic products, A, B, C, D. All these products
have a good demand in the market. The following figures are given to you:

A B C D
Material cost (`/u) 64 72 45 56
Machining Cost (`/u)
(@ `8 per hour) 48 32 64 24
Other variable costs (`/u) 32 36 44 20
Selling Price (`/u) 162 156 173 118
Market Demand (Units) 52,000 48,500 26,500 30,000

Fixed overheads at different levels of operation are:

Level of operation Total Fixed


(In production hours) Cost (`)

Upto 1,50,000 10,00,000


1,50,001-3,00,000 10,50,000
3,00,001-4,50,000 11,00,000
4,50,001-6,00,000 11,50,000

At present, the available production capacity in the company is 4,98,000 machine hours. This capacity is not
enough to meet the entire market demand and hence the production manager wants to increase the capacity.
The company wants to retain the customers by meeting their demands through alternative ways. One
alternative is to sub-contract a part of its production. The sub-contract offer received is as under:
A B C D
Sub-contract Price (`/u) 146 126 155 108
The company seeks your advice in terms of products and quantities to be produced and/or sub-contracted, so
as to achieve the maximum possible profit. You are required to also compute the profit expected from your
suggestion. Nov./90 [Adapted] & (18 Marks) Nov. 09-N.C.

[Ans.: Make : A – 35000 units; D – 30000 units; Subcontract: A – 17000; B – 48500 units; C – 26500 units;
Total Profit `2324000; Alternatively, Make : A – 10000 units; D – 30000 units; Subcontract: A – 42000; B –
st
48500 units; C – 26500 units; Total Profit `2324000, but 1 alternative is preferable]
Question 55 [Bottleneck resource]: A company manufactures two products EXE and WYE, which
pass through two of its departments exclusively used for them. A market research study conducted by the
company reveals that the company can sale either 38,500 units of EXE or 31,500 units of WYE in a year. The
manufacturing cost and selling price details are as under:
EXE WYE

Selling price per unit 375 540


Costs:
Department 1: Direct materials 58 100
Direct labour 5 hours 50 7.5 hours 75

Department 2: Direct materials 21 26


Direct labour 7.5 hours 90 10 hours 120

Overheads: Department 1 Department 2

Variable overhead rate per DLH `2.40 `3.60


Fixed overheads `5, 00,000 `10, 00,000
Budgeted direct labour hours 1,75,000 2,80,000

Since the quantity which can be sold exceeded the production capacity, the company has been considering
the use of sub-contracting production facilities. Accordingly, when tenders were floated, two contractors

CA. Parag Gupta Ph.: +91 11 47665555 Paraggupta_ca@yahoo.co.in Costing & O.R.
World’s largest CA Final student’s consultancy group: http://groups.yahoo.com/group/costingbyparaggupta
Cost Accounting & Management - 215 -

responded as under:

Contractor DS offers to produce up to a maximum of 17,500 units of EXE or 14,000 units of WYE
in a year for the type of work done by department 1 of the company. The price charged by DS is `138 per unit
of EXE and `212 per unit of WYE. These prices included the cost of direct materials used in department 1
of the company.

Contractor DW can produce up to a maximum of 11,200 units of EXE and 7,000 units of WYE in a year for
the type of work done by department 2 of the company. The price charged by DW is `150 per unit of EXE and
` 192 per unit of WYE. These prices included the cost of direct materials used in department 2 of the
company.

Required:

(1) If the company does not wish to use the sub-contracting facility, which of the two product and
in what quantity should be produced and sold by the company by using its own manufacturing capacity
to earn maximum profit? Calculate the resultant maximum profit.
(2) If the company wishes to produce either 38,500 units of EXE or 31,500 units of WYE by using sub-
contracting facility, state which of the two products should be produced to maximise the profits. Calculate the
resultant maximum profit. (16 Marks) May/03

[Ans.: (i) EXE should be produced internally as the resultant profit earned (`25,95,000) is maximum; (ii) WYE
should be produced as it yield `35,07,328 as profit]
Question 56[Overtime and Subcontract + Bottleneck resource + 2 Key Factors for 2 products]: A
company manufactures two products P and Q. Both the products pass through the company’s two
departments, A and B. The market demand for a month is 2500 units of P and 2000 units of Q. The company
has normal capacity of 600 hours in department A and 520 hours in department B per month. Overtime is
acceptable upto 50% of normal hours in each department.

The details relating to the product are as under:

Product
P Q
Direct material cost per unit ` 10 5
Fixed overheads per month `18000 6400

Department
A B
Direct labour time per unit (minutes)
Product: P 6 12
Q 18 12
Direct wage rate per hour
Normal time ` 10 12
Overtime ` 15 18

In the event of the company not being able to fulfill the demand for want of capacity, the balance quantity of
the product can be sold by buying from a sub-contractor, who was agreed to supply product P at ` 18 and
product Q at ` 12 per unit.

Required:

(i) Calculate the quantity of each product to be manufactured and/ or to be sub- contracted in a most
economical way to fulfilling the market demand.
(ii) Present a statement showing the total cost involved in your solution in (i) above. (16 Marks) May/01

[Ans.: Total Cost is `81314]


Question 57: X Ltd. has incurred losses during the past five years. Its projection for the year 2002 is also not
very encouraging. The management is seriously considering the closure of the only manufacturing unit.
CA. Parag Gupta Ph.: +91 11 47665555 Paraggupta_ca@yahoo.co.in Costing & O.R.
World’s largest CA Final student’s consultancy group: http://groups.yahoo.com/group/costingbyparaggupta
Decision Making - 216 -
(including Relevant Costing, Make or Buy,
Subcontracting, Shut Down Point, etc.)
However, it is quite open to getting the products o a sub-contracting basis and to continue its administrative
and marketing functions. Currently, four products are being manufactured and sold by catering do different
markets. The management is also willing to sacrifice any of these products to ensure survival.
The projections for the four products for 2002 are:
(`in crores)
A (`) B (`) C (`) D (`)
Sales 72.0 54.0 84.0 60.0
Costs:
Material 48.0 30.0 54.0 36.0
Labour 18.0 12.0 30.0 30.0
Allocated Overheads:
Manufacturing 6.0 4.8 7.2 4.8
Admin. & Selling 2.4 1.2 3.6 2.4
Total Cost 74.4 48.0 94.8 73.2
Profit / Loss) (2.4) 6.0 (10.8) (13.2)
The projected volume and sub-contracting chares are:
A B C D
Volume (‘000 nos.) 2,000 1,500 3,000 2,000
Sub-contracting charges / unit (`) 80 70 90 130

Manufacturing, administrative and selling overheads consists of staff salaries, rent, essential maintenance
and tax payable to the local authorities.
In case the management decides to discontinue the manufacturing operations a minimum notice period of 3
months will be required to be given to the staff as well as to the landlords of the manufacturing unit and
offices. You may assume that both the manufacturing as well as the administrative and selling overheads are
fixed in nature, and that in the notice period mentioned above, these expenses would continue to be incurred.
 Assume that labour costs are related to the volume of operations and do not involve any notice period for
discontinuance;
 Assume that the costs are incurred and revenues earned evenly in each of the calendar months.
Based on the above, you are required to advise the management on the best option out of the options under
its consideration, viz.:
(i) Issue notices to the staff, the landlords of manufacturing unit and offices on the first day of the year and
discontinue all the operations on that very day.
(ii) Issue notices as above on the first day of the year and continue the operations till the end of the notice
period (only profitable products need to be continued).
(iii) Issues notices to the staff and the landlord, only in the manufacturing unit, resort to sub-contracting and
to continue the administrative and marketing functions. (Sub-contracting is needed to be done on profitable
products only). (17 Marks) Nov./01

[Ans.: The third option is most viable]

Export Pricing

Question 58: C Ltd. an Indian company, has entered into an agreement of strategic alliance with Z Inc.
of United States of America for the manufacture of personal computers in India. Broadly, the terms of
agreement are:
(i) Z will provide C with kits in a dismantled condition. These will be used in the manufacture of the
personal computer in India. On a value basis, the supply, in terms of the FOB price will be 50%
thereof.
CA. Parag Gupta Ph.: +91 11 47665555 Paraggupta_ca@yahoo.co.in Costing & O.R.
World’s largest CA Final student’s consultancy group: http://groups.yahoo.com/group/costingbyparaggupta
Cost Accounting & Management - 217 -

(ii) C will procure the balance of materials in India.


(iii) Z will provide to C with designs and drawings in regard to the materials and supplies to be procured
in India. For this, C will pay Z a technology fee of `3 crores.
(iv) Z will also be entitled total royalty at 10% of the selling price of the computers fixed for sales in India
as reduced by the cost of standard items procured in India and also the cost of imported kits from Z.
(v) C will furnish to Z detailed quarterly returns.

Other information available:


(i) FOB price agreed $510.
Exchange rate to be adopted $1 = `47.059
[Note: In making calculations, the final sum may be rounded to the next rupees)
(ii) Insurance and freight – `500 per imported kit;
(iii) Customs duty leviable is 150% of the CIF prices; but as a concession, the actual rate leviable has
been fixed at 30% of CIF.
(iv) The technology agreement expires with the production of 2,00,000 computers;
(v) The quoted price on kits includes a 20% margin of profits on cost to Z.
(vi) The estimated cost of materials and supplies to be obtained in India will be 140% of the cost of
supplies made by Z.
(vii) 48% of the value in rupees of the locally procured goods represent cost of the standard items.
(viii) Cost of assembly and other overheads in India will be `2,000 per personal computer.

Required: Calculate the selling price, of a personal computer in India bearing in mind that C has targeted a
profit of 20% to itself on the selling price (12 Marks) Nov/01

[Ans.: Selling Price per computer `43000]


Question 59: X Ltd having an installed capacity of one lakh units of a product is currently operating at 70%
utilization. At current level of input price, the F.O.B. cost per unit, taking credit of applicable export incentive
workout as follows:

Capacity Utilization 70% 80% 90% 100%


FOB cost per unit (`) 97 92 87 82

The company has received three foreign offers as under:


Sources A: 5000 units @ ` 55 per unit FOB
Sources B: 10000 units @ ` 52 per unit FOB
Sources C: 10000 units @ ` 51 per unit FOB

Required:
Advice the company whether it should accept any or all the export orders. (10 Marks) Nov./07

[Ans.: All the 3 proposals should be accepted as they will result in a profit of `80000 and idle capacity of 5000
units.]
Shut Down Point

Shut Down is a case where the business would be closed temporarily. In this case the some fixed costs will
continue to incur.
Avoidable Fixed Cost – Shut Down Cost (i.e. saving in fixed cost)
Shut Down Point (units) =
Contribution per unit
Avoidable Fixed Cost – Shut Down Cost (i.e. saving in fixed cost)
Shut Down Point (in `) =
P/V Ratio
CA. Parag Gupta Ph.: +91 11 47665555 Paraggupta_ca@yahoo.co.in Costing & O.R.
World’s largest CA Final student’s consultancy group: http://groups.yahoo.com/group/costingbyparaggupta
Decision Making - 218 -
(including Relevant Costing, Make or Buy,
Subcontracting, Shut Down Point, etc.)
If demand is more than calculated units we must continue the business, otherwise it is preferable to shut it
down (subject to non financial considerations).
Here, Shut down cost can be sub-divided into 3 parts:
 Costs incurred on suspension of operations.
 Costs incurred during continued shut down.
 Costs incurred in resuming operation after re-opening.

Question 60 [Shut down/Continue point]: A firm incurs a fixed cost of `1,20,000 at 60% capacity. At 0%
capacity, fixed cost is only `40, 000. If its VC Ratio is 80%, find out the Shutdown point.

[Ans.: `4,00,000/-]

Question 61: The selling price per unit of a product is `14. For the forthcoming period, the demand will be
only 5,000 units. The fixed expenses at 50% activity (5,000 units) will be `30,000. The company is thinking of
shutting down operations, in which case an additional amount of `2,000 will have to be incurred for shutting
down and only `20,000 of the above fixed costs can be avoided.

What should be the variable cost per unit to recommend a shut down?

[Ans.: If variable cost per unit is above 10.40, it is recommended to shut-down] (6 Marks) Nov./10-O.C.
Question 62: G Ltd. produces and sells 95000 units of 'X' in a year at its 80% production capacity. The
selling price of product is `8 per unit. The variable cost is 75% of sales price per unit. The fixed cost is
`3,50,000. The company is continuously incurring losses and management plans to shut-down the plant. The
fixed cost is expected to be reduced to `1,30,000. Additional costs of plant shut-down are expected at
`15,000.

Should the plant be shut-down? What is the capacity level of production of shut-down point?

[Ans.: Yes it is recommended to shut down the plant; Shut down point = 102500 units; Capacity utilization at
shut down = 86.32%] (5 Marks) Nov./10-N.C.
Question 63 [Shut down/Continue point]: A paint manufacturing company manufacture 2,00,000 per
annum medium–sized tins of “Spray Lac Paints” when working at normal capacity. It incurs the
following costs of manufacturing per unit:
(`)
Direct Material 7.80
Direct Labour 2.10
Variable overheads 2.50
Fixed overheads 4.00
Product Cost per unit 16.40

The selling price is `21 per tin and variable selling and administrative expenses is 60 paise per tin.

During the next quarter only 10,000 units can be produced and sold. Management plans to shut down the
plant estimating that the fixed manufacturing cost can be reduced to `74,000 for the quarter. When the plant
is operating, the fixed overheads are incurred at a uniform rate throughout the year. Additional costs of plant
shutdown for the quarter are estimated at `14,000.
(a) Express your opinion, as to whether the plant should be shut down during the quarter, and
(b) Calculate the shut down point for the quarter in terms of number of tins.

[Ans.: 14000 tin] (6 Marks) Nov/08-NC[Adapted]


Question 64 [Shut down/Continue point]: The annual budget of a company at 60% and 80% levels of
performance is as under:

CA. Parag Gupta Ph.: +91 11 47665555 Paraggupta_ca@yahoo.co.in Costing & O.R.
World’s largest CA Final student’s consultancy group: http://groups.yahoo.com/group/costingbyparaggupta
Cost Accounting & Management - 219 -

Level of Performance 60% 80%


`(‘000) `(‘000)
Direct Material 360 480
Direct Labour 480 640
Production Overhead 252 276
Administration Overhead 124 132
Selling & Distribution Overhead 136 148
1352 1676

The company is in great difficulties at the present moment in selling its products and is now operating at 50%
level.

The sales revenue for the year is estimated at `9,90,000. The Directors are seriously considering
suspending operations till the market picks up.

Market Research undertaken by the Company reveals that there is every indication that in about twelve
months’ time, the sales will pick up and the company can comfortably operate at 75% level of performance
and earn a sales revenue of `18 lakhs in that year.

The Sales personnel of the company do not want to suspend operations for fear of adverse reactions in the
market; but the Directors want to decide the issue purely on financial consideration.
If the manufacturing and other operations of the company are suspended for a year, it is estimated
that:

a. The present fixed costs could be reduced to `2,20,000 per annum.


b. The settlement cost of personnel not required would amount to `1,50,000.
c. The maintenance of plant has to go on and that would cost `20,000 per annum.
d. On resuming operations, the costs connected with opening after a shut-down would amount to `80,000.

Submit a report to the Directors and indicate therein, based on purely financial consideration, whether it would
be advisable or not to suspend the company’s operation in the current year.

[Ans.: Co. will incur a loss of `200000 if it continue producing @ 50% & will incur a loss of `470000 if it shut
its operations temporarily]
Question 65: Supreme ltd, which manufactures the component EXCEL, has achieved a turnover of ` 600000
for the calendar year 2002. The manager of the company has informed that the company has worked at the
profit volume ratio of 25% and the margin of safety of 20%. But he feels due to severe competition, the selling
price is to be reduced to maintain the same volume of sales for the year 2003. He does not expect any
change in variable costs. He expects that due to cost reduction programme, the profit volume ratio and
margin of safety will be 20% and 30% respectively and considerable saving in Fixed cost for 2003.

Even if the company prefers to shut down its operations for 2003, it expects to incur a minimum fixed cost of `
60000. You are expected to:

(i) Present the comparative statement for the year 2002 and 2003 showing under marginal costing.
(ii) What will be minimum sales required, if it decides to shut down its unit in 2003. (11 Marks) Nov./03

[Ans.: (i) Contribution for 2002 `150000 and 2003 `112500; (ii) `93750]
Question 66: TQM Limited makes engines for motor cars for its parent company and for two other motor car
manufacturers.

On 31st December, the company has sufficient work order for January and one further order for 21,000
engines. Due to recession in the economy, no further order are expected until May when it is hoped economic
prospect for the motor car industry will have improved. Recently factory has been working at only 75% of full
capacity and the order for 21000 engines represents about one month production at this level of activity.

The board of directors are currently considering following two options :

CA. Parag Gupta Ph.: +91 11 47665555 Paraggupta_ca@yahoo.co.in Costing & O.R.
World’s largest CA Final student’s consultancy group: http://groups.yahoo.com/group/costingbyparaggupta
Decision Making - 220 -
(including Relevant Costing, Make or Buy,
Subcontracting, Shut Down Point, etc.)
(i) Complete the order in February and close the factory in March and April.
OR
(ii) Operate at 25 per cent of full capacity for each of three months of February, March and April.
The costs per month at different levels of activities are as follows:

At 75% (`) At 25% (`) Idle(`)


Direct material 5,25,000 1,75,000 -
Direct labour 5,23,600 1,73,250 -
Factory overhead:
Indirect material 8,400 4,900 4,900
Indirect labour 1,01,500 59,500 -
Indirect expenses:
Repair & maintenance 28,000 28,000 -
Other expenses 52,500 34,400 26,600
Office overhead:
Staff salaries 1,48,400 98,000 67,550
Other overheads 28,000 19,950 11,200

Other information is as follows:

— Material cost and labour cost will not be incurred where there is no production.
— On the reopening of the factory, one time cost of training and engagement of new personnel would be
`65,800 and overhauling cost of plant would be `14,000.
— Parent company can purchase engines from open market at reasonable price.

Required:

To express your opinion, along with calculations, as to whether the plant should be shut down during the
month of March and April or operate 25% of full capacity for three months. (11 Marks) June/09-O.C.

[Ans.: TQM Ltd. is recommended to operate at 75% capacity in Feb. and shut down in March & April]

Divestment Strategy
Divestment involves a strategy of selling off or shedding business operations to divert the resources, so
released, for other purposes. Selling off a business segment or product division is one of the frequent forms
of divestment strategy. It may also include selling off or giving up the control over a subsidiary where by
the wholly owned subsidiaries may be floated as independently quoted companies .
Reason for Divestment Strategy
1. In case of a firm having an opportunity to get more profitable product or segment but have resource
constrain, it may selling off it’s unprofitable or less profitable division and utilised the recourse so released.
Cost Benefit Analysis & Capital Budgeting Method are the useful tool for analysing this type of situation.
2. In case of purchase of new business, it may be found that some of the part of the acquired business is
not upto the mark. In such type of situation disposal of the unwanted part of the business is more desirable
than hold it.
3. In case where any business segment or product or subsidiary is pull down the profit of the whole organisation,
it is better to cut down of that operation of the product or business segment or subsidiary.
4. If managing of the organization is very constrained, it is good to dispose off the unwanted and undesirable
activity of the organization, which involve large management skill so that it can concentrated on the core
activities of the organization.
5. In the situation where the firm suffering from loss, selling off or divestment policy is one suitable option to exit in
the current position and to go for turn around strategy.

Question 67: What is divestment strategy? Highlight the main reasons for divestment?(4 Marks)June/09-N.C.

Question 68: E Ltd. is engaged in the manufacturing of three products in its factory. The following budget
estimates are prepared for 2009-10:

CA. Parag Gupta Ph.: +91 11 47665555 Paraggupta_ca@yahoo.co.in Costing & O.R.
World’s largest CA Final student’s consultancy group: http://groups.yahoo.com/group/costingbyparaggupta
Cost Accounting & Management - 221 -

Products
A B C
Sales (Units) 10,000 25,000 20,000
Selling Price p.u. (`) 40 75 85
Direct Materials p.u. (`) 10 14 18

Direct wages pu. @ `2 p.hr. 8 12 10


Variable overhead pu. (`) 8 9 10
Fixed overhead (`) p.u. 16 18 20
Profit/Loss -2 22 27

After the finalisation of the above manufacturing schedule, it is observed that presently only 80% capacity
being utilised by these three products. The production activities are made at the same platform and it may be
interchangeable among products according to requirement. In order to improve the profitability of the
company the following three proposals are put for consideration:

(a) Discontinue product A and capacity released may be used for either product B or C or equally shared. The
fixed cost of product A is avoidable. Expected changes in material cost and selling price subject to the
utilisation of product A’s capacity are as under
Product B : Material cost increased by 10% and selling price reduced by 2%.
Product C : Material cost increased by 5% and selling price reduced by 5%.

(b) Discontinue product A and divert the capacity so released and the idle capacity to produce a new product
D for meeting export demand whose per unit cost data are as follows:
(`)
Selling Price 60
Direct Material 28
Direct wages @ 3 p. hr. 12
Variable overheads 6
Fixed Cost (Total) 1,05,500

(c) Product A, B and C are continuously run and hire out the idle capacity fixing a price in such a way that the
same rate of profit per direct labour hour is obtained in the original budget estimates.

Required:

(i) Prepare a statement of profitability of products A, B and C in existing situation.


(ii) Evaluate the above proposals independently and calculate the overall profitability of the company under
each proposal.
(iii) What proposal should be accepted, if the company wants to maximise its Profit?
May/10 (RTP-N.C.)-Adapted & (10 Marks) May/10-N.C.
[Ans.: `1070000; (ii)(a) `1424800, (b) `1378250, (c) `1337500; (iii) Proposal (c)]

Inventory Decisions
Formulae

1. Minimum level of inventory = Re-order level - (Average rate of consumption  lead time)

2. Maximum level of inventory = Re-order level + Re-order quantity –


(Minimum consumption  Minimum re-order period)

3. Re-order level = Maximum re-order period  Maximum Usage


Or
Minimum level or safety stock level + (Average or normal rate of
consumption  Average time to obtain fresh supplies).

CA. Parag Gupta Ph.: +91 11 47665555 Paraggupta_ca@yahoo.co.in Costing & O.R.
World’s largest CA Final student’s consultancy group: http://groups.yahoo.com/group/costingbyparaggupta
Decision Making - 222 -
(including Relevant Costing, Make or Buy,
Subcontracting, Shut Down Point, etc.)
4. Average inventory level = Minimum + ½ Re- order quantity
(or)
( Maximum level + Minimum level )  2

2  Annual consumption (A)  ordering cost per order (Co)


5. EOQ = Carrying cost per unit per annum (Ch)

6. Total ordering cost = No. of order  Co

Annual carrying or storing or holding cost = Quantity per order  2  Ch

Total ordering & carrying cost (Known as Relevant Storing cost)

= 2  Annual consumption  Co  Ch

7. Buffer stock = ROL - Consumption during the lead time.


If the result is negative then it is known as Stock Out Quantity

Stock out quantity = Consumption during the lead time - ROL

Expected Stock out quantity =  ( Present Stock out quantity - increase in ROL)
Probability
Stock out cost = Expected Stock out quantity  CS ( Stock-out cost p.u.) Storing cost =
Increase in stock  Ch .

8. The best stock policy : Select the min of annual carrying/storing/holding + total ordering cost +
stock out cost. ( objective minimization of total cost )

Question 69: Kartik Co. is organized into two processes. Raw material is introduced into Process A and its
output becomes the raw material for Process B. The finished goods of Process B is sold in the market.
Process A has a capacity to process an input of 200000 kg of raw material per annum. The normal scrap is
10% and 5% of input in Process A and Process B respectively. The realizable value of scrap is Re.1 and `2
per kg respectively for processes A and B. The operating data for a year are as under:

Process A Process B
Direct Wages `2200000 2100000
Overheads `956000 1345800

There are three suppliers of raw material whose price quotations and terms are as under:

Supplier Price `/kg Terms


P 10.00 Maximum quantity offered is 120000 kg.
Q 11.20 Maximum quantity offered is `160000 kg.
R 11.00 `11 only when the entire quantity of 200000 kg is ordered. For any quantity less
than 200000 kg the price charged is `11.60 per kg
In each case, the raw material is to be collected from the supplier’s factory. The variable transport cost for
bringing the raw material if `1.20 per kg, `1 per kg and Re. 1 per kg for P, Q and R respectively. The annual
fixed transport cost will be `200000 irrespective of the supplier from whom the material is purchased.
The output emerging from Process B can be sold to three customers at the prices and terms given below:

Customer Price `/kg Discount Condition


K 65 2% Maximum quantity acceptable to K is 80000 kg
L 64 2% Maximum quantity acceptable to L is 160000 kg
M 61.80 - Provided the entire production of the company is sold to M

CA. Parag Gupta Ph.: +91 11 47665555 Paraggupta_ca@yahoo.co.in Costing & O.R.
World’s largest CA Final student’s consultancy group: http://groups.yahoo.com/group/costingbyparaggupta
Cost Accounting & Management - 223 -

In the case of customers K and L, fixed delivery costs of `5000 in total per month will be incurred. The
variable delivery costs in respect of customers K and L respectively are `2.60 and `1.44 per kg. However,
customer M will collect the output from the company’s factory at his own cost.

Required:

1. Indicate with supporting calculations the choice of (a) Suppliers and (b) customers
2. Based on your recommendation above, prepare a statement showing the process-wise costs and
profit of the company for the year.

[Ans.: Profit is `1484000]


Question 70: The Stock Control Policy of Sakti Co. is that each stock is ordered twice a year, the quantum of
each order being one-half of the year’s forecast demand. The Materials manager, however, wishes to
introduce a policy in which for each item of stock, Re-order Levels and EOQ is calculated.

For one of the items X, the following information is available:

Forecast Annual Demand 3600 units


Cost per unit `100
Cost of Placing an order `40
Stockholding Cost 20% of the average stock value
Lead Time 1 month
Buffer stock to cover fluctuation in demand 100 units

If the new policy is adopted, calculate for stock Item X, - (a) Re-order Level that would be set by the Materials
manager (b) Anticipated Reduction in value of the average stock investment (c) Anticipated reduction in total
inventory costs in the first and subsequent years. (10 Marks) Nov./01

[Ans.: (a) 400 units (b) `74000 (c) `3680]

Question 71: X Ltd. presently has its inventory turnover (based on Cost of Goods Sold ÷ Average Inventory)
at 10 times p.a. as compared with the industry average of 4. Average Sales are `450000 p.a. Variable Cost of
Sales are 70% of Sales and Fixed Costs are `10000 per annum. Carrying Costs of Inventory (excluding
financing costs) are 5% per annum. Sales force complained that low inventory levels are resulting in lost sales
due to Stock–outs. The Sales manager has made an estimate based on stock-out reports as under:

Inventory Policy Inventory Turnover Sales


Current 10 `450000
A 8 `500000
B 6 `540000
C 4 `565000

On the basis of the above estimates and assuming a 40% tax rate and an after-tax required return of 20% on
investment in inventory, which policy would you recommend.

[Ans.: Policy as its net benefit is ``75034]

Miscellaneous

Question 72: A company can produce and sell at its maximum capacity 20,000 units of a product. The sale
price is ` 100. The present sale is 15,000 units. To produce over 20,000 units and upto another 10,000 units
some balancing equipments are to be installed at a cost of ` 10 lakhs and the same will have a life span of 10
years.

The current cost structure is as under:


Direct material 30% of sale value
Direct labour 20% of sale value

CA. Parag Gupta Ph.: +91 11 47665555 Paraggupta_ca@yahoo.co.in Costing & O.R.
World’s largest CA Final student’s consultancy group: http://groups.yahoo.com/group/costingbyparaggupta
Decision Making - 224 -
(including Relevant Costing, Make or Buy,
Subcontracting, Shut Down Point, etc.)
Variable overheads ` 20 per unit
Profit ` 15 per unit

The present cost is estimated to go up due to price escalation as under:


10% in direct material from present level of 30%
25% in Direct labour from present level of 20%
` 50000 in fixed overheads per year.

There is a concrete proposal from a party to take 10000 units additionally over the present level of output on a
long term basis at a unit price of ` 90. Apart from the investment of ` 10 lakhs, as above, the fixed overheads
will increase by ` 50000 due to additional administrative expenses.

The company is in a dilemma as to whether to accept the order for 10000 units or to use the present unused
capacity of 5000 units for which there will be additional selling expenditure of ` 50000.
Ignore financial charges and give your recommendation. (14 Marks) Nov./98

[Ans.: Attaining the maximum capacity by incurring additional selling expenditure is the best proposal]
Question 73: Unique Product manufactures and sells in a year 20000 units of a particular product to definite
customer at a price of ` 100 per unit. The concern has a capacity to produce 25000 units of the product per
annum. To produce beyond 25000 units per annum, the concern will have to install a new equipment at a cost
of ` 15 lakhs. The equipment will have a life span of 10 years and will have no residual value. There is an
offer from a client to purchase 10000 units of the product regularly at a price of ` 90 per unit. The order, if
accepted, will have to be over and above the existing level of production of 20000 units.
The cost structure is as under:
Per Unit `
Direct material 30
Direct labour 20
Variable overhead 10
Profit 20

During the coming year, it has been estimated that the cost of direct material, as compared to the current year
will be increased by 10%. Because of certain wage agreement direct labour cost will increase by 25%. Fixed
overhead will increase by 10%. If the new order of 10000 units is accepted, fixed overhead will increase
further by ` 60000 due to increased administrative charges.

You are required to analyze whether the concern should accept the order or instead of that try to secure order
for the balance unused capacity, as available now, through some sales promotion expenses which will be `
50000 per annum. Ignore financial charges for the new investment. (8 Marks) May/00

[Ans.: It is advisable to produce and sell 25000 units @ `100 per unit and utilize full production capacity, as
only then the profit would be maximum i.e. `310000]
Question 74: A company manufacturing chemicals furnishes the following data of their activities for the year
1993-94. The company manufactures three products namely Ethylene, EDC, and VCL. Ethylene is consumed
for making EDC and EDC is consumed for making VCL. One metric ton of Ethylene is required to make one
metric ton of EDC and one metric ton of EDC is required for making one metric ton of VCL. The other
particulars:

Ethylene EDC VCL


Production capacity per annum (metric tons) 25000 30000 30000
Cost per Metric ton:
Variable Costs `20 `30 `40
Product Fixed Costs 20 30 40
Common Fixed Costs 10 15 20
Total 50 75 100
Selling Price per Metric Ton (`) 150 300
Sales per annum (Metric tons) 10000 15000

CA. Parag Gupta Ph.: +91 11 47665555 Paraggupta_ca@yahoo.co.in Costing & O.R.
World’s largest CA Final student’s consultancy group: http://groups.yahoo.com/group/costingbyparaggupta
Cost Accounting & Management - 225 -

The company restricts the manufacture of all its products only to the extent of the sales demand. The
management is concerned with the low capacity utilization. In order to achieve fuller utilization of plant
capacity, the company entered into negotiations with various parties. As a result of the negotiations, X who
buys one-third of the current sales volume of VCL, offers to buy 20000 metrics tons of VCL per annum of
`250 per metric ton provided the entire quantity of 20000 metrics is sold to him. This purchase is for the
captive consumption of X and therefore will not affect the market price of VCL. X also offers to supply EDC for
manufacture of VCL to the extent of 5000 metric tons at a price of `125 per metric ton. The company can also
buy EDC from open market at `140 per metric ton if the order is for 10000 metric tons or more.

The bases of various costs given above are as follows:


(i) Variable costs exclude the cost of internally consumed Ethylene in the manufacture of EDC and costs
of EDC consumed in the manufacture of VCL.
(ii) Fixed Costs are based on normal capacity production.
(iii) The product fixed costs can be avoided only if there is nil production of the product concerned.
(iv) Common fixed costs are to be incurred irrespective of production and sales.
(v) No closing stocks are maintained.

You are required to: (a) Draw up a statement of profitability in respect of the year 1993-94 as originally
envisaged by the company.
(b) If the company decides to accept the offer of X to buy 20000 metric tons of VCL at `250 per metric ton and
if the balance quantity of production of VCL can be sold in the market show the revised statement of
profitability of the company. (14 Marks) Nov/94

[Ans.: (a) Profit `250000 (b) Profit `1050000]


[Note.: Suggested answers by ICAI has taken an assumptions that Normal Capacity is Annual Production
Capacity & co. will stop selling 10000 metric tons of EDC to outside customers]
Question 75 [Second Shift]: Agrocaps Ltd. engaged in manufacturing agricultural machinery, is preparing its
annual budget for the coming year. The company has a metal pressing capacity of 20000 hours, which will be
insufficient for manufacture of all requirements of components A, B, C and D.

The company has the following choices:


(i) Buy the components entirely from outside suppliers.
(ii) Buy from outside suppliers and/or use a partial second shift.

The data for the current year are given below:


Standard production cost per unit
Component A B C D
Requirements in units 2000 3500 1500 2800
Variable Costs: ` ` ` `
Direct materials 37 27 25 44
Direct wages 10 8 22 40
Direct expenses 10 20 10 60
Fixed overhead 5 4 11 20
Total production cost 62 59 68 164

Direct expenses relate to the use of the metal presses which cost `10 per hour to operate. Fixed overheads
are absorbed as a percentage of direct wages.

Supply of all or any part of the total requirements can be obtained at the following prices, each delivered to
the factory:
Component ` Component `
A 60 C 52
B 59 D 168
Second shift operations would increase direct wages by 25 per cent over the normal shift and fixed overhead
by `500 for each 1000 (or part thereof) second shift hours worked.

You are required to present, with calculations:


(b) Which component, and in what quantities should be manufactured in the 20000 hours of press time

CA. Parag Gupta Ph.: +91 11 47665555 Paraggupta_ca@yahoo.co.in Costing & O.R.
World’s largest CA Final student’s consultancy group: http://groups.yahoo.com/group/costingbyparaggupta
Decision Making - 226 -
(including Relevant Costing, Make or Buy,
Subcontracting, Shut Down Point, etc.)
available?
(c) Whether it would be profitable to make any of the balance of components required on a second shift
basis instead of buying them from outside suppliers. Nov./92

[Ans.: All requirements of D & A can be manufactured and only 600 units of product B can be manufactured.
The balance requirement of product B i.e. 2900 units will have to be bought-out or manufactured in the
second shift.]
Question 76[Second Shift]: A company has a capacity of 40,000 hours per annum for manufacture of four
components required for assembly of a product. The data are as under:

Components P Q R S
` ` ` `
Materials 64.75 47.25 43.75 77.00
Labour (`8.75 per hour) 17.50 35.00 17.50 105.00
Direct Expense 17.50 14.00 38.50 70.00
Fixed Overheads 8.75 7.00 19.25 35.00

The data relating to the number of components required per annum and the prices of the components, if
purchased from the market, are as under:

P Q R S
No. of components required 2,400 4,800 1,200 2,400
Purchase price per component (`) 105 103 91 294

If the company resorts to working a second shift to manufacture its requirement of components, it will
increase the labour costs by 25% over normal shift. In addition the fixed costs will increase by `875 per 1,000
hours or parts thereof of second shift working.

You are required to prepare statements to show:


i. Which of the components and in what quantities should be manufactured in 40,000 hours.
ii. Whether it will be profitable to manufacture any balance quantity of components by second shift
operation.

[Ans.: (i) S-2400, P-2400, Q-1600 (ii) Second shift is not recommended]
Question 77: R ltd will produce 300000 kgs of S and 600000 kgs of Y from an input of 900000 kgs of raw
material Z.
The selling price of S is ` 8 per kg and that of Y is ` 6 per kg. Processing cost amount to ` 54 lacs per month
as under:

Raw material Z 900000 kgs


At ` 3 per kg ` 2700000
Variable processing cost ` 1800000
Fixed processing cost ` 900000
Total 5400000

There is an offer to purchase 60000 kgs of Y additional at a price of ` 4 per kgs. The existing market for Y not
be affected by accepting the offer. But the price of S is likely to be decreased uniformly at all sales.

Find the minimum reduced average price for S to sustain the increased sales. (7 Marks) Nov./99

[Ans.: `7.91]

Question 78: AB Ltd. manufactures product ‘X’. the company operates single shift of 8 hours for 300 days in
a year. The capital employed in the business is `18 crores.
The manufacturing operations of the company comprise of four production departments. The company at
present produces 9,000 units of product ‘X’ at maimum capacity. However, the capacity utilization of all the
four departments are not equal and the present individual capacity utilizations are as under:
CA. Parag Gupta Ph.: +91 11 47665555 Paraggupta_ca@yahoo.co.in Costing & O.R.
World’s largest CA Final student’s consultancy group: http://groups.yahoo.com/group/costingbyparaggupta
Cost Accounting & Management - 227 -

Department Capacity Utilisation %


A 75
B 100
C 70
D 50

The present return on capital of the company has gone down to 10% from the earlier cut-off rate of 15% due
to increased cost of production.
As the company cannot operate more than one shift, the management is considering two alternative
proposals to increase the return on capital employed.

Alternative I
To hire out the surplus capacity of departments A, C and D. The cost and revenue projections are as under:

Department Hire Charges per Hour Incremental Cost per Hour


A 2,500 2,000
C 1,800 1,500
D 1,600 1,200

Alternative II
To increase the installed capacity of the factory to 12,000 units by adding plant and machinery in department
B at a capita cost of `4 crore. Any Balance surplus capacity in other departments after meeting the increased
volume to be hired out as per alternative I. The additional units would fetch incremental revenue of `1,600 per
unit.

You are required to evaluate the two proposals and suggest to the management, which of the two proposals
is to be accepted. (10 Marks) May/00
[Ans.: ROI for Alternative I is 10.533% and Alternative II is 10.53%]
Question 79 [Joint Cost]: P. W. Perfume Company manufactures various qualities of perfumes and
colognes. One popular line of colognes includes three products that result from a joint production
process. Below are data from the most recent month of production:

Product Sales Quantity Joint Cost Total cost


Price cost after
split off
Evergreen `40 10,000 `28 `20 `48
Morning Flower `100 6,000 `28 `40 `68
Evening Flower `150 4,000 `28 `50 `78
As the Controller, you are called into the Presidents Office with the Director of Marketing. The President
says, “I don’t understand your product cost report. Either, we are selling our largest-volume product at a
loss or the product cost data are all wrong. Now what is it?”

Required:

(i) Respond to the Presidents question.


(ii) Another company has just introduced a product that competes directly with Morning Flower to
compete successfully with the other company’s product, the price of Morning Flower cologne must be
reduced to `60. Should the company do so and sell below cost?
(iii) If P. W. Perfume Company has a policy of maintaining a gross margin of 20 per cent on sales, what
would your answer be in response to the price reduction in part (ii)?
(iv) What is the minimum price for which Morning Flower can self and still meet the 20 per cent product gross
margin for the group of products? (13 Marks) Nov./00
[Ans.: (i) Profit earned from Evergreen `83333, Morning Flower `150000 & Evening Flower `166667; (ii) The
co. should sell Morning Flower below cost; (iii) A reduction in sale price would result in a loss of revenue of
`140000; (iv) Minimum price per unit is `83.33]
Question 80 [Joint Cost]: Inorganic Chemicals purchase salt and processes it into more refined product
such as caustic soda, chlorine and polyvinyl chloride (PVC). For the month of October, 1998, the firm

CA. Parag Gupta Ph.: +91 11 47665555 Paraggupta_ca@yahoo.co.in Costing & O.R.
World’s largest CA Final student’s consultancy group: http://groups.yahoo.com/group/costingbyparaggupta
Decision Making - 228 -
(including Relevant Costing, Make or Buy,
Subcontracting, Shut Down Point, etc.)
purchased salt for ` 80000, conversion cost incurred were ` 120000 upto the split-off point, at which time two
salable products were produced: Caustic soda and chlorine. Chlorine could be further processed into PVC.
Production and other relevant information for the month of October,1998 are as follows:

Production Sales Sale price per ton


Caustic soda 2400 tons 2400 tons ` 100
Chlorine 1600 tons - -
PVC 1000 tons 1000 tons ` 400

The full production of chlorine was further processed at an incremental cast of ` 40000 to yield 1000 tons of
PVC. There were no by-products or scrap from this further processing of chlorine. The organization did not
have any opening or closing stocks of any of the above commodities for October, 1998.
There is a very active market for chlorine. The firm could have sold its entire production for October, 1998 at `
150 per ton.

You are required to calculate:

(i) How the joint cost of ` 200000 would be allocated between caustic soda and chlorine under each of
the methods, viz., (a)sales value at split-off: (b) physical measure (tons): and (c) estimate net realizable
value?
(ii) The gross margin percentage of (a) caustic soda and (b) PVC under the three methods given in (i)
above.
(iii) Daily Swimming pool ltd. offers to purchase 1600 tons of chlorine in November,1998 at 150 per ton.
This would mean that no PVC would be produced that month. Will the acceptance of the offer affect
the operating income for November, 1998?

[Ans.: Caustic Soda Chlorine


(i) (a) Sale value at split off 240000 240000
(b) Sales in tons 2400 1600
(ii) Sale Value at split off Physical Measure(tons) Estimated net realizable value(`)
(a)Caustic Soda 58.33% 50% 66.67%
(b) PVC 65% 70% 60%
(iii) Incremental operating income (`120000)]
Question 81: A company manufactures two products. Each product passes through two departments A and
B before it becomes a finished product. The data for a year are as under:

Product Aristocrat Deluxe


(i) Maximum sales potential in units 7400 10000
(ii) Product unit data:
Selling price per unit `90 `80
Machine hour per unit:
Dept. A Hours 0.50 0.30
Dept. B Hours 0.40 0.45
(iii) Maximum capacity of Department A is 3400 hours and of department B is 3840 hours.
(iv) Maximum quantity of direct material available is 17000 Kg. Each product requires 2 Kg of
direct materials. The purchase price of direct material is ` 5 per Kg.
(v) Variable costs are budgeted at `50 per hour for department A and ` 60 per hour for
department B.

In view of aforesaid production capacity constraints, the company has decided to produce only one of the two
products during the year under review.

Required:

(i) Which of the two products should be produce and sold in the year under review to maximize the profit.
State the number of units of that product and the resultant contribution.
(ii) The surplus capacity available in Department A or Department B after manufacture of either Aristocrat or
Deluxe is proposed to be hired out to earn a contribution of `40 per hour in the case of Department A and

CA. Parag Gupta Ph.: +91 11 47665555 Paraggupta_ca@yahoo.co.in Costing & O.R.
World’s largest CA Final student’s consultancy group: http://groups.yahoo.com/group/costingbyparaggupta
Cost Accounting & Management - 229 -

` 60 per hour in the case of department B. Prepare a statement to show whether Aristocrat or Deluxe
should now be produced to maximize the total contribution. Calculate such total contribution.
(iii) The company has been advised to produced 4250 units of each product and also to hire out to hire out
the surplus capacity of Department A and/or Department B. You are required to examine the feasibility of
this proposal and to prepare a budget analysis showing to total contribution for the year.

[Ans.: (i) 8500 units of Deluxe model should be produced; (ii) Total contribution: Aristrocrat `278000, Deluxe
`272900; (iii) Total Contribution `264400] (16 Marks) May/04
Question 82: Zilmil ltd makes two product ‘Brightly’ and ‘Lightly’. Both the products use the same labour
force, the size of which is restricted to 78000 hours per month. Brightly needs 2 hours per unit to make
whereas lightly needs one hour. The estimated production and sale, manufacturing and selling expenses per
month are as follows:
Brightly Lightly
Production and sale (in nos.) 12000 16000 40000 48000
Cost per month (`) 3400000 3800000 6200000 6680000

The company is considering pricing option in a highly competitive market. It has estimated sales demand at
various selling prices:
Brightly:
Selling price per unit (`) 276 272 268 264 260 254
Sales demand per month 12000 14000 16000 18000 20000 22000
Lightly:
Selling price per unit (`) 163 162 161 160 154 152
Sales demand per month 40000 42000 44000 46000 48000 50000

You are required to compute profit maximizing price and quantity for each product.
ICWA June/94-Adapted & (11 Marks) May/06

[Ans.: Brightly-16000 units, `268 per unit; Lightly – 46000 units, `160 per unit]
Question 83: A manufacturing company has an installed capacity of 150000 units per annum. Its cost
structure is given below:
(Per unit)
`
Variable cost 10
Labour (Minimum ` 100000 per month) 10
Overheads 4

Fixed overheads: ` 192300 per annum

Semi-variable overheads ` 60000 per annum at 75% capacity, which increased by 4000 per annum for every
5% increase in capacity utilization for the year as a whole.

The capacity utilization for the next year is estimated at 75% for three month, 80% for six month and 90% for
the remaining part of the year. If the company is planning to have a profit of 20% on the selling price,
calculate the selling price per unit (12 Marks) May/10 – O.C.[Adapted] & (12 Marks) Nov./03

[Ans.: Selling price per unit `32.90]


Question 84 [Bottleneck resource]: X Ltd. supplies spare parts to an air craft company Y Ltd. The
production capacity of X Ltd. facilitates production of any one spare part for a particular period of time. The
following are the cost and other information for the production of the two different spare parts A and B:

Per unit Part A Part B


Alloy usage 1.6 kgs 1.6 kgs
Machine time : Machine A 0.6 hrs 0.25 hrs
Machine time : Machine B 0.5 hrs 0.55 hrs
Target Price (`) 145 115

CA. Parag Gupta Ph.: +91 11 47665555 Paraggupta_ca@yahoo.co.in Costing & O.R.
World’s largest CA Final student’s consultancy group: http://groups.yahoo.com/group/costingbyparaggupta
Decision Making - 230 -
(including Relevant Costing, Make or Buy,
Subcontracting, Shut Down Point, etc.)
Total hours available : Machine A 4,000 hours
Machine B 4,500 hours

Alloy available is 13,000 kgs. @ `12.50 per kg.

Variable overheads per machine hours :


Machine A : `80
Machine B : `100

You are required to identify the spare part which will optimise contribution at the offered price.
If Y Ltd. reduces target price by 10% and offers `60 per hour of unutilised machine hour, what will be the total
contribution from the spare part identified above? (8 Marks) May/10-N.C.

[Ans.: (i) Total Contribution-Part A: `179982, Part B: `162500; Hence Part A (ii) `153369]
Question 85 [Bottleneck resource]: TP Ltd. produces a product which passes through two processes -
cutting and finishing.
The following information is provided :

Cutting Finishing
Hours available per annum 50,000 60,000
Hours needed per unit of product 5 12
Fixed operating costs per annum excluding direct material 10,00,000 10,00,000

The selling price of the product is `1,000 per unit and the only variable cost per unit is direct material, which
costs `400 per unit. There is demand for all units produced.

Evaluate each of the following proposals independent of each other:

(i) An outside agency s willing to do the finished operation of any number of units between 5,000
and 7,000 at `400 per unit.
(ii) An outside agency is willing to do the cutting operation of 2,000 units at `200 per unit
(iii) Additional equipment for cutting can be bought for `10,00,000 to increase the cutting facility by
50,000 hours, with annual fixed cost increased by `2 lacs. (4 Marks) Nov./10-O.C.

[Ans.: Current Profit `10 lacs; (i) Increase in profit `10 lacs; (ii) Increase in costs `4 lacs; (iii) Initial cost
`2 lacs & Increase in annual costs `2 lacs]
Question 86: A Company manufactures two products ‘X’ and ‘Y’. Company’s fixed cost per annum is `5
lacs. These products are sold for `288 per unit of ‘X’ and `432 per unit of ‘Y’. Standard cost data are:

Product ‘X’ Product ‘Y’


` `
Direct Raw Material 40 80
Direct wages `8 per hour in Departments:
1 48 72
2 24 48
3 72 ―
4 ― 96
Variable overhead 32 28

The Company operates 8 hours shift for 300 days in a year. Number of workers engaged by each
department is given below:

Department 1 2 3 4
No. of Workers 45 24 27 36

Required:

CA. Parag Gupta Ph.: +91 11 47665555 Paraggupta_ca@yahoo.co.in Costing & O.R.
World’s largest CA Final student’s consultancy group: http://groups.yahoo.com/group/costingbyparaggupta
Cost Accounting & Management - 231 -

(a) How many units of each product should be manufactured and what is the resultant maximum profit,
if numbers of employees cannot be increased or transferred?
(b) If only one product is to be manufactured by the Company, which of the products would give the
maximum profit and what is the amount of such profit? (11 Marks) May/06

[Ans.: (a) Product X-7200 units, Product Y-6000 units, Maximum Profit `666400; (b) Product Y-7200 units,
Product X – 4800 units, Maximum Profit `623200]
Question 87: Carcare Corporation has just today paid for and installed a special machine for polishing cars at
one of its prestigious outlets. It is the first day of the company’s fiscal year. The machine costs `20,000. Its
annual operating costs total `15,000 exclusive of depreciation. The machine will have a four-year useful life
and a zero terminal disposal value.

After the machine has been used for one day, a machine salesman walks in. He offers a different machine
that promises to do the same job at a yearly operating cost of `9,000, exclusive of depreciation. The new
machine will cost `24,000 in cash, duly installed. The “old” machine is unique and can be sold outright for only
`10,000 minus `2,000 removal cost. The new machine, like the old one, will have a four-year useful life and
zero terminal disposal value.

For simplicity, ignore income taxes, interest and present value considerations.

Sales, all in cash, will be `1,50,000 annually and other cash costs will be `1,10,000 annually, regardless of
this decision.

Required:

(a) Prepare a statement of cash receipts and disbursements for each of the four years under both
alternatives. What is the cumulative difference in cash flows for the four years taken together?
(b) Prepare Income Statements for each of the four years under both alternatives. Assume straight-line
depreciation. What is the cumulative difference in operating income for the four years taken together?
(c) What are the irrelevant items in your presentations in requirements (a) and (b)? Why are they irrelevant?
(d) Suppose the cost of the “old” machine was `10,00,000 rather than `20,000. Nevertheless, the old
machine can be sold outright for only `10,000 minus `2,000 removal cost. Would the net differences in
requirements (a) and (b) change? Explain.
(e) “To avoid a loss, we should keep the old machine.” What is the role of book value in decisions about
replacement of machines? (19 Marks) Nov./98

[Ans.:
Keep Old Machine Buy New machine
nd rd th
Year 1 2 ,3 &4 All 4 years Year 1 2nd, 3rd & 4th All 4 years
year each year each
(a) Net Cash 5 25 80 (5) 31 88
inflow
(b) 20 80 13 25 88 8
Operating
Income
(c) Purchase cost of old machine (`20000), Sale Revenue (`150000) and other cash costs (`110000).
(d) The net difference will not change.]
Question 88: A firm needs a component in an assembly operation. If it wants to do the manufacture itself, it
would need to buy a machine for ` 4 lakhs which will last for 4 years with no salvage value. Manufacturing
cost in each of the 4 year would be `6 lakhs, `7 lakhs, `8 lakhs & `10 lakhs respectively. If the firm had to buy
the components from a supplier, the cost would be ` 9 lakhs, `10 lakhs, 11 lakhs and ` 14 lakhs respectively
in each of the four year. However, the machine would occupy floor space which would have been used for
another machine. This latter machine would be hired at no cost to manufacture an item, the sale of which
would produce net cash flows in each of the four year of ` 2 lakhs. It is impossible to find room for both the
machine and there are no other external effects. The cost of capital is 10 % and the present value factor for
each of the four year is 0.909, 0.826, 0.751 and 0.683 respectively.

CA. Parag Gupta Ph.: +91 11 47665555 Paraggupta_ca@yahoo.co.in Costing & O.R.
World’s largest CA Final student’s consultancy group: http://groups.yahoo.com/group/costingbyparaggupta
Decision Making - 232 -
(including Relevant Costing, Make or Buy,
Subcontracting, Shut Down Point, etc.)
Should the firm make the component or buy from outside. (10 Marks) May/99

[Ans.: There is a saving of `14800 in buying the component from outside]


Question 89: A company produces main product “Super” and a co-product “Mild”. The main product is sold
entirely to its collaborator, but the product ”Mild” is sold at the local market. The company increased its
capacity as a result of which the output of “Mild” increased to 15000 m/t per annum at a price of ` 1000 pt.

However in the face of increased competition to sell the entire output of 15000 m/t of “Mild” the company will
have to reduce the sale price by ` 50 pt. every year for next five year and thereafter the price will at ` 750 pt.
As an alternative the company can convert “Mild” into “Medium” at a variable cost of ` 200 per (metric) tone.
However to enter the market the sale price will have to be ` 1200 pt in the first year and ` 1300 pt, in the
second year.

The sale of Medium will be 1000m/t in the first and there upon going up by 1000m/t each year. The company
will have to invest ` 30 lakhs in capital outlay to produce “Medium”

You are required to present the projected sales volume (quantity and value) of products “Mild” and “Medium”
and also appraise the investment of ` 30 lakhs at 12% per annum for the period of next 5 years.

Year Present value of Rupee one at 12% p.a.


1 0.89
2 0.79
3 0.71
4 0.64
5 0.57 (11 Marks) May/00

[Ans.: The company should opt for second option as the NPV (`477.31 lacs) is better than the first option
(`464.925 lacs)]

Question 90: A consumer goods manufacturer uses large volume of tin containers, which are sold on a
returnable basis to their local distributors, who are required to deposit `25% per tin, refundable on return of
the tins. The company incurs a cost of `32 per tin, which, depending upon its condition on return, can be used
six to eight times. Unusable tins are sold as scrap at `8 per tin, normally, 15,000 tins are scrapped each
month.

The company has received a suggestion from an employee to convert such scrapped tins into usable lids for
the container, as a cost reduction proposal. Following data is available concerning this proposal:

(a) Each rejected tin can be converted into 5 lids of acceptable quality, after rejections.
(b) Cost of conversion into lids is `50 per 100 pcs.
(c) Each tin weighs 1 kg and each lid weighs 120 gms.
(d) Scrapped lids and other off-cuts of the tin can be sold @ `5/kg.
(e) Company’s requirement of lids is one lakh per month, which it currently buys at `2 per pc.

Required:

I. An evaluation of the proposal, with supporting working and your recommendation whether or not to
accept the proposal.
II. A statement of estimated savings that will accrue to the company, if the proposal is accepted.

[Ans.: (a) Proposal should be accepted (b) Savings of `270000]

Question 91: Ret Ltd., a retail store buys computers from Comp Ltd. and sells them in retail. Comp Ltd. pays
Ret Ltd. a commission of 10% on the _selling price at which Ret sells to the outside market. This commission
is paid at the end of the month in which Ret Ltd. submits a bill for the commission. Ret Ltd. sells the
computers to its customers at its store at `30,000 per piece Comp Ltd. has a policy of not taking back
computers once dispatched from its factory. Comp Ltd. sells a minimum of 100 computers to its customers.

CA. Parag Gupta Ph.: +91 11 47665555 Paraggupta_ca@yahoo.co.in Costing & O.R.
World’s largest CA Final student’s consultancy group: http://groups.yahoo.com/group/costingbyparaggupta
Cost Accounting & Management - 233 -

Comp Ltd. charges prices to Ret Ltd. as follows:


`29,000 per unit, for order quantity 100 units to 140 units.

`26,000 per unit, for the entire order, if the quantity is 141 to 200 units. Ret Ltd. cannot order less than 100 or
more than 200 units from Comp Ltd.

Due to the economic recession, Ret Ltd. will be forced to offer as a free gift, a digital camera costing it `4,500
per piece, which is compatible with the computer. These cameras are sold by another Co., Photo Ltd. only in
boxes, where each box contains 50 units. Ret Ltd. can order the cameras only in boxes and these cameras
cannot be sold without the computer.

In its own store, Ret Ltd. can sell 110 units of the computer. At another far of location, Ret Ltd. can sell upto
80 units of the computer (along with its free camera), provided it is willing to spend `5,000 per unit on
shipping costs. In this market also, the selling price that each unit will fetch is `30,000 per unit.

You are required to:

(i) State what is Ret's best strategy along with supporting calculations.
(ii) Compute the break-even point in units, considering only the above costs. (13 Marks) June 09-N.C.

[Ans.: Buy 150 units from Company Ltd & sell 110 at store & 40 outside; (ii) 175 units.]

CA. Parag Gupta Ph.: +91 11 47665555 Paraggupta_ca@yahoo.co.in Costing & O.R.
World’s largest CA Final student’s consultancy group: http://groups.yahoo.com/group/costingbyparaggupta
Miscellaneous Theory Chapters

TOTAL QUALITY MANAGEMENT (TQM)


TQM is composed of three paradigms:
Total = Quality involves everyone and all activities in the company.
Quality = Conformance to Requirements (Meeting Customer Requirements).
Management = Quality can and must be managed.

Definition:
TQM is defined as a set of concepts and tools for getting all employees focused on continuous improvement
in the eyes of the customer. Quality is an important aspect of world-class manufacturing. The success of
Japanese companies is grass rooted in their long-term commitment to improvement of quality. A world class
manufacturing approach demands that the quality must be designed into product and the production
process, rather than an attempt to remove poor quality by inspection.
 The TQM approach highlights the need for a customer-oriented approach to management reporting,
eliminating some or more of traditional reporting practices.
 TQM seeks to increase customer satisfaction by finding the factors that limit current performance.
 The emphasis of TQM is to design and build quality in the product, rather than allow defectives and
then inspect and rectify them. The focus is on the causes rather than the symptoms of poor quality.
Though the goal is zero defects, the methodology is one of continuous improvement.

Three core concepts of TQM:


1. Quality Control (QC): It is concerned with the past, and deals with data obtained from previous
production, which allow action to be taken to stop the production of defective units.
2. Quality Assurance (QA): It deals with the present, and concerns the putting in place of system to
prevent defects from occurring.
3. Quality Management (QM): It is concerned with the future, and manages people in the process of
continuous improvement to the products and services offered by the organization.

Various stages / steps to be taken in the implementation of TQM:


Stage 1: Identification of customers/customer groups: through a team approach (a technique called
Multi-voting), the firm should identify major customer groups. This helps to prioritize the list of customers and
provides a focus of services.
Stage 2: Identifying customer expectations: Once the major customer groups are identified, their
expectations are listed. The question to be answered is – what does the customer expect from the firm?
Stage 3: Identifying customer decision-making requirements and product utilities: Where the focus is
on quality improvement, the overriding need is to stay close to the customers and follow their suggestions. In
this way, a decision-support system can be developed, incorporating both financial and non-financial
information, which seeks to satisfy user requirements. Hence, the firm finds outs the answer to – what are
customer’s decision – making requirements and product utilities? The answer is sought by listing out
managerial perceptions and not by actual interaction with the customers.
Stage 4: Identifying perceived problems in decision-making process and product utilities: Using
participative processes such as brainstorming and multi-voting, the firm seeks to list out its perception of
problem areas and shortcomings in meeting customer requirements.
This will list out areas of weakness where the greatest impact could be achieved through the implementation
of improvements. The firm identifies the answer to the question – what problem areas do we perceive in the
decision-making process?
Stage 5: Comparision with other organizations and bench marking: Detailed and systematic internal
deliberations allow the firm to develop a clear idea of their own strengths and weaknesses and of the areas of
most significant deficiency. The benchmarking exercise allows the firm to see how other companies are
coping with similar problems and opportunities.
Stage 6: Customer feedback: Stages 1 to 5 provide information base, developed without reference to the
customer. This is rectified at stage 6 with a survey of representative customers, which embraces their views
on perceived problem areas. Interaction with the customers and obtaining their views, helps the firm in
correcting its own perceptions and refining its processes.

CA. Parag Gupta Ph.: +91 11 47665555 Paraggupta_ca@yahoo.co.in Costing & O.R.
World’s largest CA Final student’s consultancy group: http://groups.yahoo.com/group/costingbyparaggupta
Cost Accounting & Management - 235 -

Stage 7 & 8: The identification of improvement opportunities and Quality Improvement Process: The
outcomes of the customer survey, benchmarking and internal analysis provides the inputs for stages 7 and 8,
i.e. the identification of improvement opportunities and the implementation of a formal improvement process.
This is done through a six-step process called PRAISE, for short.

“PRAISE Analysis”
The identification of improvement opportunities and implementation of quality improvement process (Stages 7
and 8) of the TQM Process is through a six-step activity sequence, identified by the acronym ‘PRAISE’.

Step Activity Elements


 Areas of Customer Dissatisfaction
1 Problem Identification  Absence of Competitive Disadvantage
 Complacency Regarding Present Arrangements
Prioritize problems and Opportunities by
2 Ranking  Perceived Importance, and
 Ease of Measurement and solution
 Ask “why?” to identify possible causes. Keep asking “why?” to
move beyond the symptoms and to avoid jumping to premature
3 Analysis calculations.
 Ask “what?” to consider potential considerations.
 Ask “how much?” to quantify cause and effect.
 Use Creative thinking to generate potential solution.
 Operationalise these solutions by identifying Barriers to
4 Innovation
implementation, Available enablers, and People whose co-
operation must be sought
 Implement the preferred solution
5 Solution  Take appropriate action to bring about the required changes
 Reinforce with training and documentation back-up
 Monitor the effectiveness of actions
 Establish and interpret performance indicators to track
6 Evaluation
 progress towards objectives
 Identify the potential for further improvements & return to step 1

Difficulties experienced at each step in the PRAISE process:


Step Activity Difficulties Remedies
 Effects of a problem are apparent but
 Participative approaches like
problem themselves are difficult to
brainstorming, multi-voting,
Problem identify.
1 panel discussion
Identification  Problem may be identifiable, but it is
 Quantification and precise
difficult to identify a measurable
definition of problem
improvement opportunity
 Difference in perception of individuals
in ranking.
 Participative approach
 Difference in preferences based on
2 Ranking  Subordination of individual to
functions e.g. production, finance,
group interest
marketing etc.
 Lack of consensus between individuals
Adoption of adhoc approaches and quick-
3 Analysis Lateral Thinking Brainstorming
fix solutions
 Lack of creativity or expertise
Systematic evaluation of all
4 Innovation  Inability to operationalise ideas, i.e.
aspects of each strategy
convert thoughts into action points
 Effective internal
communication
5 Solution Resistance from middle mangers  Training of personnel and
managers
 Participative approach
CA. Parag Gupta Ph.: +91 11 47665555 Paraggupta_ca@yahoo.co.in Costing & O.R.
World’s largest CA Final student’s consultancy group: http://groups.yahoo.com/group/costingbyparaggupta
Miscellaneous Theory Topics - 236 -

 Problems in implementation
Effective control system to track
6 Evaluation  Lack of measurable data for
actuals Feedback system
comparison of expectations with actual

Central to the PRAISE system are (a) quality control – the search for continuous improvements in quality –
and (b) total employee involvement – the co-operation and commitment of employees. This dual
approach provides a single focus – the customer – whose increased satisfaction remains the primary goal of
the procedure.

Fundamental principles associated with four P’s of quality improvement


The problems listed for the PRAISE process, may lead to P of TQM process. It is possible that the
organization is led to Total Quality Paralysis instead of improvement. To avoid such disruption and paralysis
the following principles (called the four P’s) of TQM should be followed:
1. People: Some individuals are not ideally suited to the participatory process of TQM.
This will be inferred from lack of enthusiasm, non-attendance at TQM meetings, failure to complete
delegated work, remaining a “Mute Spectator” at TQM meetings etc. To avoid misdirection, TQM
teams should consist of team spirited individuals who have a flair for accepting and meeting
challenges.
2. Process: It is essential to approach problem-solving practically and to regard the formal process as a
system designed to prevent participants from jumping to conclusions. As such it will provide a means
to facilitate the generation of alternatives while ensuring that important discussion stages are not
omitted.
3. Problem: Problems need to be approached in a bite-sized chunks, with teams tackling solvable
problems with a direct economic impact, allowing for immediate feedback together with recognition of
contribution made by individual participants.
4. Preparation: Additional courses on creative thinking and statistical processes are needed in order to
give participants a greater appreciation of the diversity of the process.

This training must quickly be extended beyond the immediate accounting circle to include employees at
supervisory levels and also who are involved at the data input stage.

A three-point action plan for implementation of the process is:


1. Bite-sized chunks: Big improvement opportunities are generally complex and require extensive inter
departmental co-operation. The choice of a relatively small problem in the first instance provides a
greater chance of success. Therefore, the TQM team has to proceed from small to big issues
gradually.
2. Solvable problem: The problem selected should not be trivial, but it should be one with a potential
impact and a clear improvement opportunity. Measurable progress towards implementation should be
accomplished within a reasonable time in order to maintain the motivations of participants and
advertise the success of the improvement itself.
3. Recognition of participants: The successful projects and team members should receive appropriate
recognition throughout the enterprise. Prominent individuals should be rewarded for their efforts as a
measure of personal recognition and as encouragement to others. The reward may be recognition
itself, and sometimes monetary/non-monetary prizes may also be given.

Six Cs of TQM
1. Commitment - If a TQM culture is to be developed, so that quality improvement becomes normal
part of everyone's job, a clear commitment, from the top must be provided. Without this all else fails.
2. Culture - Training lies at the centre of effecting a change -in culture and attitudes. Negative
perceptions must be changed to encourage individual contributions.
3. Continuous improvement - TQM is a process, not a program, necessitating that we are committed
in the long term to the never ending search for ways to do the job better.
4. Co-operation: The on-the-job experience of all employees must be fully utilized and their
involvement and co-operation sought in the development of improvement strategies and associated
performance measures.
5. Customer focus: Perfect service with zero defects in all that is acceptable at either internal or
external levels.

CA. Parag Gupta Ph.: +91 11 47665555 Paraggupta_ca@yahoo.co.in Costing & O.R.
World’s largest CA Final student’s consultancy group: http://groups.yahoo.com/group/costingbyparaggupta
Cost Accounting & Management - 237 -

6. Control: Documentation, procedures and awareness of current best practice are essential if TQM
implementations are to function appropriately the need for control mechanisms is frequently overlooked,
in practice.

Control in the TQM


The fundamental principles of TQM focus on continuous improvement, which enhance the satisfaction of
customer requirements. A properly defined control function is essential for achievement of TQM objectives.

This control is prevalent in the TQM process in the following areas:


1. Process definition: The definition of the process, inputs and outputs gives a framework for the
writing of procedures and standard methods and also provides a focus for improvement opportunities.
The clear definition and documentation of procedures facilitates job flexibility, makes control easier
and increases the level of productivity.
2. Database: Documentation of key data on processes is an important step in TQM. By charting
processes for each activity, establishing time barriers, constraints, priorities degrees of difficulty and
expected improvement times, a critical database is established.
3. Quality Manual: It defines the basic philosophy of the organization, the structure and responsibilities
of managers and departments and the relationship between them. It also contains the methods to be
used to ensure quality, including the composition of teams and the audit procedure to be adopted.
4. Improved decision-making: By providing a sound control environment, which supports business
decisions with appropriate measurement and analysis, the controllership function pursues complete
customer satisfaction. The aim is to achieve acknowledged industry leadership for excellence of
process, personnel and service.
5. Control and continuous improvement: TQM facilitates not only control, but also continuous
improvement. The monitoring of the data around a process will allow modifications which makes it in-
control and capable. As changes or improvements are made they are documented and the system
updated so that everyone uses the current best method.
6. Use of Control reports: Diagrams, Statistical quality control charts and cost of quality reports are
prepared for periodic review of the TQM system in operation. The deviation from expected costs are
analyzed for suitable corrective action. The various types of costs to be reported are (i) Prevention
Costs; (ii) Appraisal Costs; (iii) Internal failure costs; and (iv) External Failure Costs.

There are a measure of all costs directly associated with the achievement of complete conformance to
product quality requirements. These are not just the cost of quality management or inspection function.
Specifically quality costs are the sum total of

a. Prevention Costs - (Quality Engineering, Quality planning).


b. Appraisal Costs - Cost of appraising product for conformance tor equirements.
c. Failure Costs - Costs incurred by failure to conform to requirements
I. Internal
II. External
Each of them, broadly, consists of:

a. Preventive Costs * Design Quality Assurance

* Test Equipment Design

* Supplier Quality assurance

* Vendor rating

* Central Quality Organization (with a strong Quality)


Engineering function).
* Training

* Quality Planning

CA. Parag Gupta Ph.: +91 11 47665555 Paraggupta_ca@yahoo.co.in Costing & O.R.
World’s largest CA Final student’s consultancy group: http://groups.yahoo.com/group/costingbyparaggupta
Miscellaneous Theory Topics - 238 -

* Development Testing
* Process capability studies
* Product Reliability tests
b. Appraisal Costs - * Inspection
* Testing
* Supplier Quality Assurance
* Test and inspection
equipment maintenance,
calibration and repair.
* Environmental and
Reliability Testing.
* Production time spent in checking and sorting product
* Depreciation of inspection and Test Equipment.
c. Failure Costs - * Scrap
i. Internal ( In Plant) * Rework
* Excess material provisioning/ procurement.
* Concession and Salvage
* Sub-standard performance
* Additional inspection, testing and assembly.
* Trouble shooting
* Reinspection and / or Retest
* Defect investigation
* Modifications necessitated by defects/failures
* Waiting time due to rectification, modification
ii) External (in user’s place) * Warranty

* LOSS OF CUSTOMER CONFIDENCE (LOSS OF


BUSINESS- applicable to export orders, local customers.
* Sending repair crew to repair at Customer’s premises

* Bringing back faulty goods, their repair and re transmittal

* Cost of replacement

The above approach has been called the “ PAF Model”.

User Quality Costs:

In this approach an attempt is made to determine the costs incurred by the user when the purchased
materials or equipment has problems. Such non – quality costs can be broadly grouped under seven
categories as given below:

CA. Parag Gupta Ph.: +91 11 47665555 Paraggupta_ca@yahoo.co.in Costing & O.R.
World’s largest CA Final student’s consultancy group: http://groups.yahoo.com/group/costingbyparaggupta
Cost Accounting & Management - 239 -

Category of user Example Categories of User Quality Costs


Quality Cost
Cost of repairs 1A Parts and material for failed items and any
associated items which also must be replaced.
1B Labour for replacing the failed items and Sociated
items.
Cost of effectiveness loss 2A idle direct labour before and during a shutdown
and during startup of a process
2B Extra defective product made before, during the
immediately after process shutdown
Cost of maintaining 3A Equipment parts and materials
extra capacity because
of expected failure 3B Direct and indirect labour
Cost of damages 4A injuries to personnel
caused by a failed item 4B Training new personnel when a
replacement is required
Lost income 5A Profit on production lost during downtime of
failed item
5B Monetary penalties incurred because downtime
due to a failed item causes the user to miss
schedules or impose other inconveniences on his
customer
Extra investment cost 6A Special installation and /or Compared to

competing products “running in” requirements.


Special checkout and maintenance equipment
6B
Extra operating & 7A Lower functional output per
maintenance cost compared cycle of operation
to competing products 7B Special power and fuels

Remedies to correct misdirection in TQM

TQM may become misdirected on the following grounds:


1. Focus on documentation process and ill-measurable outcomes
2. Emphasis on quality assurance rather than improvement; and
3. Internal-focus, which is at odds with the alleged customer orientation.

This can be correction by reviving the customer focus with total employee involvement (TEI), oriented towards
organizational goals. This will involve the following areas of thrust:
1. Loyalty to the vision of the company through the pursuit of tough, visible goals.
2. Recognition of satisfied customers and motivated employees as the true assets of a company.
3. Delegation of decision-making to the point of responsibility by eliminating hierarchical ties of authority
to allow direct and speedy response to customer needs.
CA. Parag Gupta Ph.: +91 11 47665555 Paraggupta_ca@yahoo.co.in Costing & O.R.
World’s largest CA Final student’s consultancy group: http://groups.yahoo.com/group/costingbyparaggupta
Miscellaneous Theory Topics - 240 -

4. Decentralization of management to make best use of the creative energy of the workforce.
Question 1: Define Total Quality Management? What are the six Cs for successful implementation of
TQM? (4 Marks) May/04, (4 Marks) May/05 & (6 marks) May/07
Question 2: Discuss the benefits accruing from the implementation of a Total Quality Management
programme in an organization. (4 Marks) Nov./08
Answer: The benefits accruing from the implementation of a Total Quality Management programme in an
organisation are:
1. There will be increased awareness of quality culture in the organization.
2. It will lead to commitment to continuous improvement.
3. It will focus on customer satisfaction.
4. A greater emphasis on team work will be achieved
Question 3: How does Total Quality Management (TQM) facilitates value addition in an organization?
4 Marks) Nov./02
Question 4: Explain four P’s of quality improvement principle. (4 Marks) Nov./09-O.C.

Question 5: List out the remedies available for difficulties experienced during implementation of PRAISE.
(4 Marks) Nov./10-N.C.
Question 5: What are the critical success factors for the implementation of a 'Total Quality Management'
programme? (5 Marks) Nov./09-N.C.
Question 6: A Company manufactures a single product, which requires two components. The company
purchases one of the components from two suppliers: X limited and Y limited. The price quoted from X ltd is `
180 per hundred units of the component and it is found that on an average 3% of the total receipt from this
supplier is defective. The corresponding quotation from Y ltd is ` 174 per hundred units, but the defective
would go up to 5%. If the defectives are not detected, they are utilized in production causing a damage of `
180 per hundred units of the component.

The company intends to introduce a system of inspection for the components on receipt. The inspection cost
is estimated ` 24 per hundred units of the component. Such an inspection will be able to detect only 90% of
the defective components received. No payment will be made for components found to be defective in
inspection.

Required:
(i) Advise whether inspection at the point of receipt is justified?
(ii) Which of the two suppliers should be asked to supply?
(Assume total requirement is 10000 units of the component) (10 Marks) Nov./07

[Ans.: If not inspected:Cost per 100 good component is X Ltd. `191.13, Y Ltd. `192.63; If inspected: Cost per
100 good component is X Ltd `205.86, Y Ltd. `201.13 Hence inspection is not justified. (ii) X Ltd]
Question 7: Eastern Switching Co. (ESC) produces telecommunications equipment Charles, ESC’s
president, believes that product quality is the key to gaining competitive advantage. Laurent implemented a
total quality management (TQM) program with an emphasis on customer satisfaction. The following
information is available for the first Year (2004) of the TQM program compared with the previous year.

2003 2004
Total number of units produced and sold 10,000 11,000
Units delivered before scheduled delivery data 8,500 9,900
Number of defective units shipped 400 330
Number of customer complaints other than for defective units 500 517
Average time form when customer places for defective unit to
When unit is delivered to the customer 30 days 25 days
Number of units reworked during production 600 627
Manufacturing lead time 20 days 16 days
Direct and indirect manufacturing labor - hours 90,000 1,10,000

Required:
CA. Parag Gupta Ph.: +91 11 47665555 Paraggupta_ca@yahoo.co.in Costing & O.R.
World’s largest CA Final student’s consultancy group: http://groups.yahoo.com/group/costingbyparaggupta
Cost Accounting & Management - 241 -

1. For each of the years 2003 and 2004, calculate


a. Percentage of defective units shipped.
b. On-time delivery rate
c. Customer complaints as a percentage of units shipped
d. Percentage of units reworked during production
2. On the basis of your calculations in requirement I, has ESC’s performance on quality and timeliness
improved?
3. Philip Larkin, a member of ESC’s board of directors, comments that regardless of the effect
That the program has had on quality, the output per labor-hour has declined between 2003 and 2004.
Larkin believes that lower output per labor-hour will lead to an increase in cost and lower operating
income.
a. How did Larkin conclude that output per labor-hour declined in 2004 relative to 2003?
b. Why might output labor-hour decline in 2004?
c. Do you think that a lower output per labor-hour will decrease operating income in 2004? Explain briefly.

[Ans.:
1. 2003 2004
a. Percentage of defective units 400 = 4% 330 = 3%
shipped 10,000 11000

b. Customer complaints as a 500 = 5% 517 =4.7%


percentage of units shipped 10000 11000
c. On-time delivery 8500 = 85% 9900 = 90%
10000 11000

d. Percentage of units reworked 600 =6% 627 =5.7%]


during production 10000 11000
Question 8: TQ Ltd. implemented a quality improvement programme and had the following results:

2007 2008
(Figures in `’000)
Sales 6,000 6,000
Scrap 600 300
Rework 500 400
Production inspection 200 240
Product warranty 300 150
Quality training 75 150
Materials inspection 80 60
You are required to:
(i) Classify the quality costs as prevention, appraisal, internal failure and external failure and express each
class as a percentage of sales.
(ii) Compute the amount of increase in profits due to quality improvement.

[Ans.: (i) Classification of Quality Costs Figures `’000


% of % of
2007 sales 2008 sales

Sales 6,000 6,000


Prevention
Quality training 75 1.25 150 2.5
Appraisal
Product Inspection 200 240
Materials
Inspection 80 60
280 4.67 300 5
Internal Failure
Scrap 600 300
CA. Parag Gupta Ph.: +91 11 47665555 Paraggupta_ca@yahoo.co.in Costing & O.R.
World’s largest CA Final student’s consultancy group: http://groups.yahoo.com/group/costingbyparaggupta
Miscellaneous Theory Topics - 242 -

Rework 500 400


1100 18.33 700 11.67
External Failure
Product warranty 300 5 150 2.5
1755 29.25 1300 21.67
(ii) Cost reduction was effected by 7.58% (29.25 – 21.67) of sales, which is an increase in profit by `4,55,000.
(6 Marks) Nov/08-NC& ICWA-June/03 [Adapted]
Question 9: Your company plans to operate department d at normal capacity next year producing one lakh
units of product P. Assuming no defective works, these units can be manufactured in 2.5 lakhs labour hours
at a cost of Re.0.50 per hour, factory overhead would amount to `1,50,000 of which `50,000 would be fixed
five units of materials can be purchased in two qualities; a high quality at `1.05 per unit or a lower quality at
`0.80 per unit.

Under expected conditions, using high quality materials 10% of the work will be defective requiring complete
replacement of the material, additional labour costs and variable overhead, scrap materials recovered from
defectives production could be sold at Re.0.30 per unit of high quality material used.

As an alternative to this arrangement, the use f the lower quality material is being considered by this would
require an extra operation to be performed on it. An additional machine and tooling would be needed at a cost
of `3, 000 per annum. The additional operation would take half an hour for each unit of product P produced,
not talking defective work into a account.

It is estimated that 20% of the work would be defective all of which would require complete replacement.
Scrap material from the lower quality material could be sold for `5, 000.
Present information to management indicating the more profitable course of action.

[Ans.: Had there been no defectives for production of 1,00,000 pieces of P 1,00,000X5=5,00,000 units of raw
material would be required. In case of high quality material , defective being 10% total raw material required is
5,00,000 units/0.90 =5,55,556 units. In case of lower quality material, defective being 20%, total raw material
requirement is 5,00,000 units/0.08 =6,25,000 units. Similarly labour and variable overhead requirement are to
be adjusted accordingly.
I. Using high quality materials (scrap 10%) (`)
Cost of 1,00,000 pieces of P 8,66,666

II. Using lower quality materials (scrap 10%) (`)


Cost of 1,00,000 pieces of P 8,74,250
Analysis: Hence the high quality material should be used.
Question 10: A company manufactures a component on batches of 2000 each. Each component is tested
before being sent to the agents for sales .Each components can be tested at the factory at a cost of `25. If
any component is found to be defective, it can be rectified by spending `200. In view of the large demand for
the components and the sophisticated system of manufactures, a proposal came up that the practice of pre –
testing of the components be dispensed with to save costs. In that event, any defective component is
received back from the customer under warranty, the cost of rectification and redespatch will be `400 per
component. State at what percentage of manufacture of components will the company find it cheaper to pre-
test each component. (ICWA-Dec/00)

[Ans.:-Let the defectives be’d’


(I) Total Cost `(2000x25)+200d
(II) Percentage of defectives to total 250/2000*100 =12.5%
components
Analysis: If defectives exceed 12.5% of the total number of components per-testing is recommended.
Question 11: A company manufactures a single product, the estimated costs of which are as follow:

Direct Materials `10 each, Direct wages 8 hours at Re.0.50 per hour
Overhead absorption rate `1.75 per hour.(50% fixed overhead include)
During this period 1,000 units will be produced and sold as follow:-

CA. Parag Gupta Ph.: +91 11 47665555 Paraggupta_ca@yahoo.co.in Costing & O.R.
World’s largest CA Final student’s consultancy group: http://groups.yahoo.com/group/costingbyparaggupta
Cost Accounting & Management - 243 -

900 units of first at `30 each


50 units of second at `20 each
50 units of third at `10 each

Present information to management showing the loss due to the production of inferior units.
By reprocessing the inferior units taking the full reprocessing time of a further 8 hours and adding further
material Costing `4 per unit these,” seconds” and “thirds’ can be converted into firsts. Present information to
the management.
[Note: No change in the profit position hence this need not be considered.]

Question 12: Carlon Ltd. makes and sells a single product; the unit specifications are as follows:

Direct Materials X : 8 sq. metre at ` 40 per square metre


Machine Time : 0.6 Running hours
Machine cost per gross : `400
hour
Selling price : `1,000

Carlon Ltd. requires to fulfil orders for 5,000 product units per period. There are no stock of product units
at the beginning or end of the period under review. The stock level of material X remains unchanged
throughout the period.

Carlon Ltd. is planning to implement a Quality Management Programme (QMP). The following additional
information regarding costs and revenues are given as of now and after implementation of Quality
Management Programme.

Before the implementation of QMP After the implementation


1. 5% of incoming material from suppliers scrapped due to 1. Reduced to 3%.
poor receipt and storage organisation.
2. 4% of material X input to the machine process is wasted 2. Reduced to 2.5%
due to processing problems.
3. Inspection and storage of Material X costs Re. 1 per 3. No change in the unit rate
square metre purchased.
4. Inspection during the production cycle, calibration 4. Reduction of 40% of the
checks on inspection, equipment vendor rating and existing cost.
other checks cost `2,50,000 per period
5. Production Qty. is increased to allow for the 5. Reduction to 7.5%
downgrading of 12.5% of the production units at the final
inspection stage. Down graded units are sold as
seconds at a discount of 30% of the standard selling
price.
6. Production Quantity is increased to allow for return from 6. Reduction to 2.5%
customers (these are replaced free of charge) due to
specification failure and account for 5% of units actually
delivered to customer.
7. Product liability and other claims by customers is 7. Reduction to 1%.
estimated at 3% of sales revenue from standard product
sale.
8. Machine idle time is 20% of Gross machine hrs used 8. Reduction to 12.5%.
(i.e. running hour = 80% of gross/hrs.).
9. Sundry costs of Administration, Selling and Distribution 9. Reduction by 10% of the
total – `6,00,000 per period. existing.
10. Prevention programme costs `2,00,000 10. Increase to `6,00,000.

CA. Parag Gupta Ph.: +91 11 47665555 Paraggupta_ca@yahoo.co.in Costing & O.R.
World’s largest CA Final student’s consultancy group: http://groups.yahoo.com/group/costingbyparaggupta
Miscellaneous Theory Topics - 244 -

The Total Quality Management Programme will have a reduction in Machine Run Time required per
product unit to 0.5 hr.

Required:

(a) Prepare summaries showing the calculation of (i) Total production units (pre inspection), (ii)
Purchase of Materials X (square metres), (iii) Gross Machine Hours.
In each case, the figures are required for the situation both before and after the implementation of
the Quality Management Programme so that orders for 5,000 product units can be fulfilled.
(b) Prepare Profit and Loss Account for Carlon Ltd. for the period showing the profit earned both before
and after the implementation of the Total Quality Programme. (16 Marks) May/05

[Ans.: (a)
After TQM
Existing Programme
Total units before inspection 6,000 5,541
Total purchases 52,632 46,871
Gross time 4,500 3,167
(b) Net profit 3,67,088

Question 13: Burdoy Plc has a dedicated set of production facilities for component X. A just in time system is
in place such that no stock of materials: work-in-progress or finished goods are held.

At the beginning of the period 1, the planned information relating to the production of component X through
the dedicated facilities as follows:

(i) Each unit of component X has input materials: 3 units of material A at `18 per unit and 2 units of
material B at `9 per unit.
(ii) Variable cost per unit of component X (excluding materials) is `15 per unit worked on.
(iii) Fixed cost of the dedicated facilities for the period `162,000.
(iv) It is anticipated that 10% of the units of X worked on in the process will be defective and will be
scrapped.

It is estimated that customer will require replacement (free of charge) of faulty units of components X at the
rate of 2% of the quantity invoiced to them in fulfillment of orders.

Burdoy Plc is pursuing a total quality management philosophy. Consequently all losses will be treated as
abnormal in recognition of a zero defect policy and will be valued at variable cost of production. Actual
statistics or each period 1 to 3 for component X are shown in appendix 3.1. No change have occurred from
the planned price level for materials, variable overhead or fixed overhead costs.

Required:-

(a) Prepare an analysis of the relevant figure provided in appendix 3.1 to show that the period 1 actual
results were achieved at the planned level in respect of (i) Quantities and losses and (ii) Unit cost
levels for materials and variable costs.
(b) Use your analysis from (a) in order to calculate the value of the planned level of each of internal and
external failure cost for period 1.
(c) Actual free replacements of component X to customers were 170 units and 40 units in period 2 and 3
respectively. Other data relating to period 2 and 3 is shown in appendix 3.1.

Burdoy Plc authorized additional expenditure during period 2 and 3 as follows:


Period 2: Equipment accuracy checks of `10,000 and staff training of `5,000
Period 3: Equipment accuracy checks of `10,000 plus &5,000 of inspection costs; also staff training cost of
`5,000 plus `3,000 on extra planned maintenance of equipment.

CA. Parag Gupta Ph.: +91 11 47665555 Paraggupta_ca@yahoo.co.in Costing & O.R.
World’s largest CA Final student’s consultancy group: http://groups.yahoo.com/group/costingbyparaggupta
Cost Accounting & Management - 245 -

Required:-

(i) Prepare an analysis for each of period 2 and 3 which reconciles the number of components invoiced
to customer with those worked on in the production process. The analysis should show the changes
from the planned quantity of process losses and change from the planned quantity of replacement of
faulty components in customer hands.
(ii) Prepare a cost analysis for each of period 2 and 3 which shows actual internal failure costs, external
failure costs, appraisal costs and prevention costs.
(iii) Prepare a report which explains the meaning and internal relationship of the figure in Appendix 3.1
and the analysis in (a), (b), and (c) (i) (ii). The report should also give examples of each cost type and
components on their use in the monitoring and progressing of the TQM policy being pursed by
Burdoy Plc.
Appendix 3.1 Actual Statistics for component X
Period 1 Period 2 Period 3
Invoices to customers (units) 5,400 5,500 5,450
Worked on in the process 6,120 6,200 5,780
Total Costs:
Material A and B (`) 440,640 446,400 416,160
Variable cost of production (`)
(excluding material cost) 91,800 93,000 86,700
Fixed cost 162,000 177,000 185,000

[Ans:
(a) (i) Units
Components worked on in the process 6120
Less: planned defective units 612
replacements to customer (2% X 5400) 108
Components invoiced to customers 5400
Therefore actual result agree with planned results

(ii) Planned components cost = (3 X `18 for material A) + (2 X `9 for material B) + `15 variable cost =`87
Comparing with the data in appendix:
Materials = `440 640/6120 =`72
Variable overhead = `91 800/6120 = `15
This indicates that prices were at the planned levels.

(b) Internal failure costs = `53 244(612 units X `87)


External failure costs = `9396 (108 units X `87)

(c) (i) Period 2 (units) Period 3 (units)


Components invoiced to customers 5500 5450
Planned replacement (2%) 110 109
Unplanned replacement 60 (170-110) -69 (40-109)
Components delivered to customers 5670 5490
Planned process defects (10% of worked on in
the process) 620 578
Unplanned defects (difference to agree with
with final row) -90 -288
Components worked on in the process 6200 5780

(ii) Period 2(`) Period 3(`)


Internal failure costs 46,110 (620-90) X`87 25,230 (578-288) X `87
External failure costs 14,790 (110+60) X `87 3,480 (109-69) X `87
Appraisal costs 10,000 15,000
Prevention costs 5,000 8,000 ]

CA. Parag Gupta Ph.: +91 11 47665555 Paraggupta_ca@yahoo.co.in Costing & O.R.
World’s largest CA Final student’s consultancy group: http://groups.yahoo.com/group/costingbyparaggupta
Miscellaneous Theory Topics - 246 -

PRICING DECISIONS & PARETO ANALYSIS

PRICING OF FINISHED PRODUCT:

I. Cost Plus pricing:


It is the most widely used method of pricing a product as it ensures that the selling price is greater than the
total cost of a product. This method helps business firms to generate profits and survive in the future.

Under cost plus pricing the selling prices of a product are determined based on its estimated cost plus a fixed
profit margin. Here ‘cost’ means full cost at current output and wage levels since these are regarded as most
relevant in price determination.

The unit cost of the product can be determined by using different methods viz. total cost, manufacturing cost
or variable / incremental cost. The percentage of mark up to be added to estimated cost also vary and
depends upon the cost figures used.

Cost Determination: For cost determination purposes, the following principles are adopted:
1. Cost Classification: Costs may be classified into:
Manufacturing, Administration and Selling & Distribution Costs; (Under Absorption costing) or
Variable and Fixed Costs (Under Marginal costing)
2. Size of the unit and scale of operations:
Small manufacturers: An individual manufacturer may take his cost of production into account and
arrive at a price at which the products are to be sold in the concerned region.
Medium and large manufacturers: A manufacturer having several factories all over the country may
determine the weighted average cost of the factories so as to arrive at a uniform ex-factory price for
the country as a whole. If commodities are in short supply, high cost of individual units may have to
be allowed in the price. However, in the case of high cost producers, the profit element may have to
be reduced to encourage them to reduce their costs.
3. Uniform costing for whole industry:
The price may be fixed after taking into account the cost or representative unit from the industry,
which may fall within the range of lowest cost unit and highest cost unit.
The factories in the industry may be classified into (i) Small size (ii) Medium Size; and (iii) Large size.
Representative samples are drawn and costs are determined by reference to the distribution of the
factories. For example, the costs of medium size factories can be taken into account if this group
forms the greater part of the industry.
4. Determination of Fixed costs:
Variable costs can be easily determined on a per unit basis. However, fixed costs per unit will have to
be ascertained with informed judgment.
Fixed cost per unit should normally be based on the level of production and capacity utilization likely
to be achieved, i.e. Normal Capacity or Capacity based on Sales expectancy.
Any assumption of low utilization may result in over estimating the cost. Conversely, a high utilization
assumption may result in under estimating the cost.

It is therefore, desirable that the level of production and capacity utilization, which are likely to apply in
the near future, should be arrived at with utmost care on realistic basis keeping in view both the past
performance and the future demand.
5. Depreciation:
If a firm wants to survive and stay in business, it has to maintain its fixed capital intact so that its fixed
assets may be replaced at the end of their useful working life out of the funds generated from profits
retained in the business.
In a period of relatively stable price levels, depreciation based on historical cost of fixed assets would
be adequate for achieving this object.
In periods when the price level is continuously changing, the firm may not be left with adequate funds
generated out of accumulated depreciation at the end of the life of the plant to replace it at a higher
price.
Hence depreciation should be properly included as part of cost so as to leave sufficient profits for
asset replacement.
CA. Parag Gupta Ph.: +91 11 47665555 Paraggupta_ca@yahoo.co.in Costing & O.R.
World’s largest CA Final student’s consultancy group: http://groups.yahoo.com/group/costingbyparaggupta
Cost Accounting & Management - 247 -

Advantages of Cost plus pricing


1. Guaranteed Contribution: When full costs plus basis is used for pricing, the firm earns a guaranteed
contribution equivalent to fixed costs plus profit margin. Even, profit margin is taken as nil, fixed costs
included in prices will guarantee minimum contribution.
2. Assured Profit: Cost plus is a fair method of price fixation. The business executives are convinced
that the price fixed will cover the cost.
3. Reduced risks and uncertainties: If price is greater than cost, the risk is covered. This is true when
normal expected capacity basis of cost estimation is used. The decision- maker may accept a pricing
formula that seems reasonable for reducing uncertainty.
4. Most suitable in long run: Cost plus pricing is ideal in the long run since there are no permanent
opportunity costs. The effect of seasonal fluctuations is ironed out and prices are established based
on normal long run costs.
5. Considers market factors: Cost plus pricing does not mean that market forces are ignored.
The mark up added to the cost to make a price reflect the well-established customs of trade, which
guide the price fixer towards a competitive price.
6. Full Recovery of all costs of the product: For long-run pricing decisions, full costs of the product
informs managers of the minimum costs to be recovered so as to continue in business rather than
shut down.
7. Price Stability: Price fixation based on full costs of the product promotes price stability, because it
limits the ability of sales person to cut prices. Price stability facilitates planning.
8. Simplicity: A full cost formula for pricing does not require a detailed analysis of cost- behaviour
patterns to separate costs into fixed and variable components for each product. It is simple to
operate.

Disadvantages of Cost plus pricing


1. Ignores demand: Cost plus pricing ignores demand and fails to take into account the buyers’ needs
and willingness to pay, which govern the sales volume obtainable at each series of prices.
2. Ignores competition: It fails to reflect competition adequately.
3. Arbitrary Cost allocation: It assumes that the costs have been estimated with exact
accuracy. This assumption is not true particularly in multi-product firms where the common
costs are allocated arbitrarily.
4. Ignores opportunity costs: For many decisions incremental cost plays a vital role in pricing, rather
than full costs. This aspect is ignored. Also opportunity costs, most relevant for decision-making are
summarily ignored.
5. Price-Volume relationships: Since the fixed overheads are apportioned on the basis of volume of
production, the cost will be more if sales volume is less and vice-versa. The increase or decrease in
sales volume is dependent on price. Thus it is a vicious circle- cost plus mark up is based on sales
volume & sales volume is based on price.

II. Rate of Return Pricing


Rate of return pricing is used when each division is treated as an Investment Centre. Determination of return
on capital employed is one of the most crucial aspects in price fixation and performance evaluation of
Investment Centres. The firm should determine an average mark-up on cost, which is necessary to produce a
desired rate of return on its investment.
The issues to be considered are:
 Basis on which the capital employed is computed Components to be covered in the return on capital
Fairness of the rate or return.
 The fairness of the rate of return varies from industry to industry and from time to time and is primarily
dependent on the risks involved. In following fair rate of return, the desirability of earning adequate
profits to plough back into business should be kept in mind.
 It would be correct to assume that allowing the industry to earn adequate return on the capital
employed would attract additional capital and increase the number of factories and production of all
commodities which must ultimately lead to competition and reduction in costs and prices.

III. Variable Costs Pricing:


Selling prices are fixed above variable costs in order to generate contribution. However, in the short run,
selling prices may be equal to variable cost or sometimes even below variable cost. Some illustrative
situations are:

CA. Parag Gupta Ph.: +91 11 47665555 Paraggupta_ca@yahoo.co.in Costing & O.R.
World’s largest CA Final student’s consultancy group: http://groups.yahoo.com/group/costingbyparaggupta
Miscellaneous Theory Topics - 248 -

 Products / Materials are perishable in nature. Launch of new product at competitive prices.
 Sales of old and defective stocks, seconds sales, etc.
 Disposal of accumulated stocks, where market prices have fallen (to save carrying costs)
 Sale of one product with reduced margin, to boost sales of other products having higher profit margin.

IV. Pricing above marginal cost, but below total cost.


In periods of recession, a firm may sell its articles at a price less than the total cost but above the marginal
cost for a limited period. The advantages of this policy arise due to avoidance of shut-down. Thus the benefits
are:

1. The firm can continue to produce and use the services of skilled employees who are well trained and
will be difficult to re-employ later if discharged.
2. Plant and machinery can be prevented from deterioration through idleness.
3. The firm would be ready to take advantage of improved business conditions later. The firm can
continue in the market and reduce loss of market share to Competitors.
Such pricing policy is necessarily restricted to the short run. When business conditions improve in the long
run, such pricing below total cost but above marginal costs is not advisable.

V. Differential selling prices:


Use of differential selling price, which is above marginal cost but below total cost is primarily intended to
absorb surplus capacity. It can be achieved in any of the following ways:
 Different Markets – Export Pricing: The firm producing a branded article may use the surplus
capacity to produce the same article to be sold above variable cost in a different market, e.g. export
sales. The articles sold in the home market will recover all fixed expenses. Since price reduction in
the home market is injurious to the normal sales, it is not resorted to. Any reduction in the selling
prices in the export market will not affect the price prevailing in the home market.
 Different products: The firm may produce and sell one product, which covers the entire fixed
overheads and use the surplus capacity to produce another product, which may be sold at a price
above its marginal cost. The overall profitability will thus increase. The manufacture of the second
product should be confined to surplus capacity and it should not have the possibility of becoming a
major product at the low price at which it is sold. If it becomes so, there will be a reduction in profit.

VI. Competitive Pricing:


When a company fixes its price mainly on the consideration of what its competitors are charging, its pricing
policy is called Competitive Pricing or competition-oriented pricing.
The company need not charge the same price as charged by its competitors. But under such a pricing
method the Company keeps its prices lower or higher than its competitors by a certain percentage.
Competitive price so determined does not maintain a rigid relation between its price, cost or demand.
The Company’s own costs or demand may change, but it will maintain its price because its competitors
maintain their prices. Conversely, the Company will change its price when its competitors change their price,
even if its own costs or demand have not altered.
The different types of competitive pricing are
 Going rate pricing
 Sealed bid pricing

VII. Incremental pricing:


Incremental pricing involves comparison of the impact of decisions on revenues and cost. If a pricing decision
results in a greater increase in revenue than costs, it is favourable. It also means that consideration is being
given to other objectives of the business. Thus profitability can be set as the matter of primary consideration
and then the decision can be adjusted to bring it in consonance with the other decision of the business.
Incremental pricing analyses all aspects of decision-making as listed below:
Relevant Cost Analysis: This technique considers changes in costs rather than in average cost. Overhead
allocations are irrelevant. Incremental revenue inflows and cost outflows are included for decision-making.
Product-Line Relationship Analysis: This technique necessitates consideration being given to possible
complementary relations in demand. Sale of one product may lead to the sale of a complementary product.
This overall effect on profitability has to be evaluated.
Opportunity Cost Analysis: the incremental revenue should cover the opportunity cost and also generate
surplus. A price which results in an incremental revenue which in turn merely covers the incremental costs is
not sufficient. If the opportunity foregone is greater than incremental revenue, the decision is not sound.
CA. Parag Gupta Ph.: +91 11 47665555 Paraggupta_ca@yahoo.co.in Costing & O.R.
World’s largest CA Final student’s consultancy group: http://groups.yahoo.com/group/costingbyparaggupta
Cost Accounting & Management - 249 -

Time Factor Analysis: The decision should take into account the short run and long run effects. A high price
may increase its immediate profits but may lead to loss of revenue in the long run owing to competitors
snatching the business.
CVP Analysis: In fixing prices, consideration should be given to price volume relationship.
The responsiveness of the market to the price should be such that the volume is increased to achieve
full utilization of plant capacity.
Risk Analysis: Consideration should also be given to the evaluation of uncertainty and risk factor. The
decision taken should be able to maximize the expected value based on probability theory.

VIII. New Product Pricing:


New Product Pricing is generally a difficult decision because of the uncertainty involved in the estimation of
their demand. For determining optimal prices, the following procedure may be adopted by a firm.
Market Survey: Experimental sales are conducted in different markets using different prices to see which
price is suitable. For example, choose three different markets and by using the same amount of sales
promotional activities, ascertain what is the right price.
Price Volume Relationship: The relationship between price and volume should be ascertained, using the
concept of elasticity of demand. The extent of volume increase due to price reduction and vice-versa, can
be reasonably quantified through such analysis.
Incremental Contribution Approach: For decision making, the firm should adopt the incremental
contribution approach i.e. additional total contribution from additional sales quantity. The firm can increase
its prices as long as there is further incremental contribution. Such analysis may prove that the highest
prices yielding the largest unit contributory margin need not necessarily maximize the profits. A lower price
may well go to maximize the profits.
PRICING STRATEGIES

Pricing strategy is defined as a broad plan of action by which an organization intends to reach its goal.
Some illustrative strategies are:
 Expanding product lines that enjoy substantial brand equity.
 Offer Quantity discounts to achieve increase in sales volume.

Some types of pricing strategies, which a firm can adopt are:


 Market entry Strategies – New Product Pricing – Skimming or penetration Pricing
 Discount Strategies – Differentials and discounts to Dealers, Distributors and Customers etc.
 Price Discrimination Strategies – based on customers, time, product version etc. Geographic
Pricing Strategies

A. Skimming Pricing Strategy:


It is a policy of high prices during the early period of a product’s existence and in the later years the prices are
gradually reduced. It is an attempt to exploit those segments of the market that are relatively insensitive to
price changes. For example, high initial price may be charged to take advantage of the novelty appeal of a
new product when the demand is initially inelastic. It offers a safeguard against unexpected future increase in
costs, or a large fall in demand after the novelty appeal has declined. This policy should not be adopted when
the substitutes are already available in the market.

The reasons for following such a policy are:


1. Inelastic Demand: The demand is likely to be inelastic in the earlier stages till the product is
established in the market. The firm can take advantage of high prices.
2. Sales Boost: The change of high price in the initial periods serves to skim the cream of the market
that is relatively insensitive to price. The gradual reduction in price in the later years will tend to
increase the sales.
3. Assured Profit: This method is preferred in the beginning because in the initial periods when the
demand for the product is not known the price covers the initial cost of production. Contribution
is guaranteed.
4. Cost Revenue Matching: High initial capital outlays, needed for manufactures, results in high cost of
production. Also, the manufacturer has to incur huge promotional activities resulting in increased
costs. High initial prices will be able to finance the cost of production. Gradually, the economies of
scale and savings in costs are passed on to customers.

B. Penetration Pricing Strategy

CA. Parag Gupta Ph.: +91 11 47665555 Paraggupta_ca@yahoo.co.in Costing & O.R.
World’s largest CA Final student’s consultancy group: http://groups.yahoo.com/group/costingbyparaggupta
Miscellaneous Theory Topics - 250 -

It is a policy of using a low price as the principal instrument for penetrating mass markets early.
This method is used for pricing a new product and to popularize it initially.
Profits may not be earned in the initial stages. However, prices may be increased as and when the product is
established and its demand picks up.
The low price policy is introduced for the sake of long-term survival and profitability and hence it has to
receive careful consideration before implementation. It needs an analysis of the scope for market expansion
and hence considerable amount of research and forecasting is necessary before determining the price.

The circumstances in which penetrating Pricing can be adopted are:


1. Elastic demand: The demand of the product is high, when price is low. Hence lower prices mean
large volumes and hence more profits.
2. Mass Production: When there are substantial savings in large-scale production, increase in demand
is sustained by the adoption of low pricing policy.
3. Frighten off competition: The prices fixed at a low level act as an entry barrier to the prospective
competitors. The use of this policy by existing concerns will discourage the new concerns to enter the
market. This pricing policy is also know as “stay-out-pricing”

PARETO ANALYSIS

PARETO ANALYSIS is a rule that recommends focus on the most important aspects of the decision making,
in order to simplify the process of decision –making.
It is based on the 80:20 phenomenon, first observed by Vilfredo Pareto, an Italian economist.
He noticed that 80% of the wealth of Milan was owned by 20% of its citizen. This pattern of 80:20 or
approximations like 70:30 can be observed in many different business situations.
The management can use it in a number of different circumstances to direct management attention to the key
control mechanism or planning aspect. It helps to clearly establish top priorities and to identify both profitable
and unprofitable targets.

Usefulness of Pareto Analysis: Pareto analysis is useful to


1. Prioritize problems, goals and objectives. Identify root causes
2. Select and define key quality improvement programs. Select key customer relations and service
programs. Select key employee relations improvement programs.
3. Select and define key performance improvement programs. Maximize research and product
development time.
4. Verify operating procedures and manufacturing processes
5. Sales/distribution of Products or services.
6. Allocate physical, financial and human resources.

Situations where Pareto Analysis can be applied:


Pareto analysis is applicable in the presentation of Performance Indicators data through selection of
representative process characteristics that truly determine or directly or indirectly influence or conform the
desired quality or performance result or outcome. It is generally applicable to the following business
situations:
1. Product Pricing
 Where a company sells a number of products, it may not be possible to analyse cost-
volume-price- profit relationships for all products.
 Pareto Analysis is used for analyzing the firm’s estimated sales revenues from various
products and it might indicate that approximately 80% of its total sales revenue is earned
from about 20% of its products.
 This helps top management to delegate the pricing decision for approximately 80% of its
products to the lower managerial levels. Top management can concentrate on pricing
decisions for the important 20% products, which are essential for the company’s survival.
 Sophisticated pricing methods can be adopted for the important products while for other
products cost based pricing methods may be used.
2. Customer profitability Analysis:
 The modern business thinking is to recognize the customer and satisfy his requirements.
Hence instead of analyzing products, customers can be analysed for their relative profitability
to the organization.
 It is often found that approximately 20% of customers generate 80% of the profits.
CA. Parag Gupta Ph.: +91 11 47665555 Paraggupta_ca@yahoo.co.in Costing & O.R.
World’s largest CA Final student’s consultancy group: http://groups.yahoo.com/group/costingbyparaggupta
Cost Accounting & Management - 251 -

 Such analysis is useful for evaluation of the portfolio of customer profile, and decision making
such as whether to continue serving a customer group, what is the extent of promotion
expenses to be incurred etc.
3. ABC Analysis – Stock Control:
Raw material stock control, it is found that only a few of the goods in stock make up most of the
value. About 70% of the materials value is due to high priced materials which constitute only 20% of
the quantity.
These materials are classified into A, B and C categories based on their importance. Control is
directed primarily over ‘A’ category items by setting EOQ, Stock levels, Surprise Stock Verification
procedures etc.
The outcome of such analysis is that by concentrating on small proportion of stock items that jointly
accounts for 80% of the total value, a firm will be able to control most of the monetary investment in
stocks.
4. Activity Based Costing
Activity Based Costing involves the identification of cost drivers for various items of Overhead
expenses. Generally, 20% of the firm’s cost drivers are responsible for 80% of the total cost.
By analyzing, monitoring and controlling those cost drivers that attribute to high costs, a better control
and understanding of overhead will be obtained.
5. Quality Control
 Pareto analysis can be extended to discover from an analysis of defect report or customer
complaints which ‘vital few’ causes are responsible for most of the reported problems.
 Generally 80% of reported problems are traceable to 20% of the underlying causes. By
concentrating one’s efforts on rectifying the vital 20%, one can have the greatest
immediate impact on product quality.
 Pareto Analysis indicates how frequently each type of failure (defect) occurs. The purpose of
the analysis is to direct management attention to the areas where the best return can be
achieved by solving most of quality problems, perhaps just with a single action.
Question 14: Enumerate the circumstances which are favourable for the adoption of a penetrating pricing
policy. (4 Marks) May/99, (4 Marks) May/01 & (3 Marks) May/04
Question 15: Outline the features of penetration pricing strategy.
(7 Marks) May/10-O.C.[Adapted] & (6 Marks) Nov./06
Ans.: (i) Penetration Pricing: It is a policy of using a low price as the principal instrument for penetrating
mass markets early.
(ii) This method is used for pricing a new product and to popularize it initially.
(iii) Profits may not be earned in the initial stages. However, prices may be increased as and
when the product is established and its demand picks up.
(iv) The low price policy is introduced for the sake of long term survival and profitability and
hence it has to receive careful consideration before implementation. It needs an analysis of
the scope for market expansion and hence considerable amount of research and forecasting
are necessary before determining the price.
(v) The circumstances in which penetrating pricing can be adopted are:
Elastic demand: The demand of the product is high when price is low. Hence, lower prices mean
large volumes and hence more profits.
Mass Production: When there are substantial savings in large-scale production, increase in
demand is sustained by the adoption of low pricing policy.
Frighten off competition: The prices fixed at a low-level acts as an entry barrier to the
prospective competitors. The use of this policy by existing concerns will discourage the new
concerns to enter the market. This pricing policy is also known as “stay-out-pricing”.
Question 16: Explain the concept of cost plus pricing. What are its advantages and disadvantages?
(8 Marks) May/00
Question 17: State the merits of cost-plus contracts. (3 Marks) May/04
Question 18: Describe two pricing practices in which non-cost reasons are important, when setting
prices. (3 Marks) Nov/00
Ans.: Two pricing practices in which non-cost reasons are important when setting price are:
(i) Price discrimination and (ii) Peak load pricing.

CA. Parag Gupta Ph.: +91 11 47665555 Paraggupta_ca@yahoo.co.in Costing & O.R.
World’s largest CA Final student’s consultancy group: http://groups.yahoo.com/group/costingbyparaggupta
Miscellaneous Theory Topics - 252 -

(i) Price discrimination: This is the practice of charging to some customers a higher price than that
charged to other customers e.g. Airlines tickets for business travellers and LTC travellers are
priced differently.
(ii) Peak load pricing: This pricing system is based on capacity constraints. Under this pricing
system a higher price for the same service or product is demanded when it approaches physical
capacity limits e.g. telephones, tele-communication, hotel, car rental and electric utility industries
are charged higher price at their peak load.
Question 19: What is Price Discrimination? Under what circumstances it is possible?
(4 Marks) May/10-N.C.
Ans.: Price discrimination is charging different prices with respect to customers, products, places and time.
Price discrimination is possible if the following conditions are satisfied:
(a) the maker must be capable of being segmented for price discrimination;
(b) the customers should not be able to resell the product of the segment paying higher price; and
(c) the chance of competitors’ underselling in the segment of higher prices should not be possible.
Question 20: What is ‘Price Skimming Policy’ and at what situation it should be exercised.
(3 Marks) May/02 & (4 Marks) Nov./04 & (2 Marks) Nov./09-N.C.
Question 21: State the pricing strategy that you would advise in the following situations which are
independent of each other :
(i) A new product is to be launched. It has had high promotional expenditure and its demand in the market is
not known.
(ii) A new product is to be launched. It is to be mass manufactured.
(iii) A product which has an external market demand is to be transferred to another division of the same
company. For the external market, variable selling costs of `10 per unit and fixed selling costs amounting to
`10 lacs p.a. are incurred. These costs are not applicable to divisional transfers. The divisional transfer can
take up only 20% of the output produced.
(iv) A special one-time order for the use of idle capacity is offered. This order will not impact the existing sales
of the company. The product has competition in the market.
(v) There is stock of a discontinued product. It has severe competition and the product is perishable.

[Ans.: (i) Skimming Pricing (because of inelastic demand); (ii) Peneteration Pricing; (iii) Incremental
Pricing i.e. Incremental Costs + Opportunity Costs (Selling costs will not be charged); (iv) Incremental
Pricing; (v) Variable cost pricing (Selling Price is variable cost or sometimes even below variable costs)]
(7 Marks) Nov./10-O.C.
Question 22: How Pareto analysis is helpful in pricing of product in the case of firm dealing with multi-
products? (3 Marks) Nov./05
Question 23: What is Pareto Analysis? Name some applications. (5 Marks)May/08
Question 24: Explain the usefulness of Pareto analysis and its application to business situations.
(4 Marks) Nov./03
Question 25: Explain different types of Competitive pricing? (4 Marks) May/05
Question 26: (a) State the general guidelines to be used in adopting a pricing policy in a manufacturing
organization. (3 Marks) Nov./08-O.C.
(b) Enumerate the uses of Pareto Analysis. (3 Marks) Nov./08-O.C.
Question 27: Calculate the selling price per unit to earn a return of 12% net on capital employed (net of tax
@ 40%). The cost of production and sales of 80000 units are :
Variable cost including material cost `9,60,000
Fixed overheads `5,00,000
The fixed portion of capital employed is `12 lakhs and the varying portion is 50% of sales turnover.

[Ans.: `23.61] (6 Marks) May/05 & (4 Marks) Nov./10-N.C.

Question 28 [Pricing Decisions]: LMV Limited manufactures product Z in departments. A and B which also
manufacture other products using same plant and machinery. The information of product Z is as follows
Items Department A (`) Department B (`)
Direct material per unit 30 25

CA. Parag Gupta Ph.: +91 11 47665555 Paraggupta_ca@yahoo.co.in Costing & O.R.
World’s largest CA Final student’s consultancy group: http://groups.yahoo.com/group/costingbyparaggupta
Cost Accounting & Management - 253 -

Direct labour per


(`10 per hour) 30 40
Overhead rates
Fixed 8 per hour 4 per hour
Variable 6 per hour 3 per hour
Value of Plant and Machinery 25 lakhs 15 lakhs

Overheads are recovered on the basis of direct labour hours. Variable selling and distribution overheads
relating to product Z are amounting to `30,000 per month. The product requires a working capital of `4,00,000
at the target volume of 1,500 units per month occupying 30 per cent of practical capacity.

You are required


i) To calculate the price of product Z to yield a contribution to cover 21 percent rate of return on
investment.
ii) Set the minimum selling price of the product if (1) the product is well established in the market; (2) the
product is first time launched in the market. (7 Marks) Nov./09-O.C.

[Ans.: Required Selling price p.u. `193.67]


Question 29 [Pricing Decisions]: Hind Metals manufactures an alloy product ‘Incop' by using iron and
Copper. The metals pass through two plants; X and Y. The company gives you the following details for the
manufacture of one unit of Incop:

Material : Iron 10 kgs @ `5 per kg.


Copper: 5 kg @ `8 per kg.
Wages : 3 hours @ `15 per hour in plant X.
5 hours @ `12 per hour in plant Y.
Overhead recovery : On the basis of direct labour hours.
Fixed overhead : `8 per hour in Plant X.
`5 per hour in Plant Y.
Variable Overhead : `8 per hour in Plant X.
`5 per hour in Plant Y.
Selling Overhead : (fully variable)- `20 per unit.

i) Find out the minimum price to be fixed for the alloy, when the alloy is new to the market. Briefly explain
this pricing strategy.
ii) After the alloy is well established in the market. What should be the minimum selling price? Why?

[Ans.: (i) `264 p.u.; Peneteration Pricing; (ii) `313] (6 Marks) Nov./09-N.C.
Question 30: [Pareto Analysis]: In the table below the level of retail sales (`’000) and closing stock (` ‘000)
for the last trading year are given for two pharmacies (A and Z ) operated by XYZ Ltd.

Category Sales Stock


A Z A Z
OTC medicine and healthcare 175 120 35 40
Toiletries 150 100 60 65
Photographic 125 60 20 12.5
Food / Drink 100 75 20 20
Baby- Care 50 25 10 5
Sanitary products 50 25 10 5
Foot Care 30 20 2 5
Cosmetics etc 25 30 40 45
Hair care 25 10 10 5
Perfumery 20 10 10 7.5
Other 50 25 13 5
Total 800 500 230 215

Required: Prepare a Pareto analysis for the following.


(1) The retail shop sales data.
CA. Parag Gupta Ph.: +91 11 47665555 Paraggupta_ca@yahoo.co.in Costing & O.R.
World’s largest CA Final student’s consultancy group: http://groups.yahoo.com/group/costingbyparaggupta
Miscellaneous Theory Topics - 254 -

(2) The stock data for A and Z.

[Ans.: XYZ Ltd.


Sales in A (rearranged for the purpose of ranking)
Rank Category Sales (`’000) Cum. Sales(`’000) %
1 OTC 175 175 21.9
2 Toiletries 150 325 40.6
3 Photo 125 450 56.3
4 Food/ Drink 100 550 68.8
5 Baby 50 600 75.0
5 San. Prod. 50 650 81.3
5 Other 50 700 81.3
8 Foot Care 30 730 91.3
9 Cosmetics 25 755 94.4
10 Hair-care 25 780 97.5
11 Perfume 20 800 100.0

Stock in A (rearranged for the purpose of ranking)


Rank Category Stock(`’000) Cum. Stock(`’000) %
1 Toiletries 60 60 26.1
2 Cosmetics 40 100 43.5
3 OTC 35 135 58.7
4 Photo 20 155 67.4
4 Food/ Drink 20 175 76.1
6 Other 13 188 81.7
7 Baby 10 198 86.1
7 San. Prod. 10 208 90.4
7 Hair 10 218 94.8
7 Perfume 10 228 99.1
11 foot care 2 230 100.0

Sales in Z (Rearranged for ranking)


Rank Category Sales(`’000) Cum. Sales(`’000) %
1 OTC 120 120 24
2 Toiletries 100 220 44
3 Food/ Drink 75 295 59
4 Photo 60 355 71
5 Cosmetics 30 385 77
6 Baby 25 410 82
6 San. Prod. 25 435 87
6 Other 25 460 92
9 Foot care 20 480 96
10 Hair 10 490 98
11 Perfume 10 500 100

Stock in Z (Rearranged for ranking)


Rank Category Stock (`’000) Cum. Stock(`’000) %
1 Toiletries 65 65 30.2
2 Cosmetics 45 110 51.2
3 OTC 40 150 69.8
4 Food/ Drink 20 170 79.1
5 Photo 12.5 182.5 84.9
6 Perfume 7.5 190 88.4
7 Baby 5 200 93.0
7 San. Prod. 5 200 93.0
7 foot care 5 205 95.3
7 Hair 5 210 97.7
7 Other 5 215 100.0

CA. Parag Gupta Ph.: +91 11 47665555 Paraggupta_ca@yahoo.co.in Costing & O.R.
World’s largest CA Final student’s consultancy group: http://groups.yahoo.com/group/costingbyparaggupta
Cost Accounting & Management - 255 -

BENCHMARKING

Benchmarking is the process of identifying and learning from the best practices anywhere in the world.
 It is powerful tool for continuous improvement in performance.
 It involves comparing firm’s products, services or activities against other best performing organization,
either internal or external to the firm. The objective is to find out how the product, service or activity
can be improved and ensure that the improvements are implemented.
 It attempts to identify an activity that needs to be improved and finding a non-rival organization that is
considered to represent world-class best practice and studying how it performs the activity.

Different types of Bench marking:


1. Competitive Bench marking: It involves the comparison of one’s own products, processes
and business results with that of competitors. Bench marking partners are drawn from the same
industry. To protect confidentiality it is common for the companies to undertake this type of
benchmarking through associations of third parties.
2. Strategic Benchmarking: It involves a systematic process by which a company seeks to improve its
overall performance by examining the long-term strategies. It involves comparing high-level aspects
such as developing new products and services, core competencies etc. It is similar to process
benchmarking in nature but differs in its scope and depth.
3. Global benchmarking: It is a benchmarking through which distinction in international culture,
business processes and trade practices across companies are bridged and their ramification for
business process improvement are understood and utilized. Globalisation and advances in
information technology leads to use of this type of benchmarking.
4. Process benchmarking: It involves the comparison of an organisation’s critical business processes
and operations against best practice organization that performs similar work or deliver similar
services.
5. Functional benchmarking: This is used when organizations look to benchmark with partners drawn
from different business sectors or areas of activity to find ways of improving similar functions or
work processes. This sort of benchmarking can lead to innovation and dramatic improvements.
6. Internal Benchmarking: It involves seeking partners from with the same organization, for example
from business units located in different areas. The main advantages are (a) Easy access to sensitive
data and information (b) Availability of standardized data; and (c) Lesser requirement of time and
resources. However, real innovation may be lacking.
7. External Benchmarking: It involves seeking help of outside organization that are known to be best in
class. It provides opportunities of learning from those who are at leading edge. However, this type of
benchmarking may take up more time and resource to ensure the comparability of data and
information. The credibility of the findings and the development of sound recommendation.
a) Intra-Group Benchmarking: Here the groups of companies in the same industry agree that similar
units within the cooperating companies will pool data on their process. The processes are
benchmarked against each other at or near operation level. ‘Improvement Task forces’ are
established to identify and transfer best practice to all members of the group.
b) Inter-industry benchmarking: In Inter-industry benchmarking a non-competing business with
similar process is identified and asked to participate in a benchmarking exercise. For example, a
publisher of schoolbook may approach a publisher of university level books to establish
benchmarking relationship.

Stages in the process of Benchmarking


Stage 1: Planning
a) Determination of benchmarking goal statement: This requires identification of areas to be
benchmarked, which uses the following criteria:
Benchmark for Customer Satisfaction Benchmark for improving Bottom line
 Consistency of product or service  Waste and Reject levels
 Process Cycle time.  Inventory levels
 Delivery Performance  Work in progress
 Responsiveness to Customers  Cost of Sales

CA. Parag Gupta Ph.: +91 11 47665555 Paraggupta_ca@yahoo.co.in Costing & O.R.
World’s largest CA Final student’s consultancy group: http://groups.yahoo.com/group/costingbyparaggupta
Miscellaneous Theory Topics - 256 -

Requirement.  Sales per employee


 Adaptability to Special Needs.
b) Identification of best performance: The next step is seeking the best. To arrive at the best is both
expensive and time consuming, so it is better to identify a company, which has recorded performance
success in a similar area.
c) Establishment of the benchmarking or process Improvement team: This should include persons
who are most knowledgeable about the internal operations and will be directly affected by changes
due to benchmarking.
d) Defining the relevant benchmarking measures: Relevant measures will not include the measures
used by the organization today but they will be refined into measures that comprehend the true
performance differences. Developing good measurement is key to successful benchmarking.
Stage 2: Collection of data and information: This involves the following steps
 Compile information and data on performance. They may include mapping processes.
 Select and contact partners
 Develop a mutual understanding about the procedures to be followed and, if necessary,
Benchmarking Protocol with partners.
 Prepare questions
 Distribute schedule of questions to each partner
 Undertake information avid data collection by chose method for example, interview, site- visits,
telephone, fax and e-mail.
 Collect the findings to enable analysis.
Stage 3: Analysis of findings:
 Review the findings and produce tables, Charts and graphs to support the analysis
 Identify gaps in performance between our organization and better performers.
 Seek explanations for the gaps in performance. The performance gaps can be positive negative or
zero.
 Ensure that comparisons are meaningful and credible. Communicate the findings to those who are
affected.
 Identify realistic opportunities for improvements. The negative performance gap indicates an
undesirable competitive position and provides a basis for performance improvement. If there is no
gap it may indicate a natural position relative to the performance being benchmarked. The zero
position should be analysed for identifying means to transform its performance to a level of superiority
or positive gap.
Stage 4: Recommendations:
1. Making recommendations:
 Deciding the feasibility of making the improvements in the light of the conditions that apply within own
organization.
 Agreement on the improvements that are likely to be feasible
 Producing a report on the Benchmarking in which the recommendations are included.
 Obtaining the support of key stakeholder groups for making the changes needed. Developing action
plans for implementation.
2. Implementing recommendations:
 Implement the action plans
 Monitor performance
 Reward and communicate successes
 Keep key stakeholders informed of progress.
Stage 5: Monitoring and reviewing: This involves:
a) Evaluating the benchmarking process undertaken and the results of the improvement against
objectives and success criteria plus overall efficiency and effectiveness.
b) Documenting the lessons learnt and make them available to others
c) Periodically re-considering the benchmarks for continuous improvement.

Pre-requisites for successful benchmarking

1. Commitment: Senior managers should support benchmarking and must be committed to continuous
improvements.

CA. Parag Gupta Ph.: +91 11 47665555 Paraggupta_ca@yahoo.co.in Costing & O.R.
World’s largest CA Final student’s consultancy group: http://groups.yahoo.com/group/costingbyparaggupta
Cost Accounting & Management - 257 -

2. Clarity of Objectives: The objectives should be clearly defined at the preliminary stage.
Benchmarking teams have a clear picture of their organization’s performance before approaching
others for comparisons.
3. Appropriate Scope: The scope of the work should be appropriate in the light of the objectives
resources, time available and the experience level of those involved.
4. Resources: Sufficient resources must be available to complete projects within the required time
scale.
5. Skills: Benchmarking teams should have the right skills and competencies.
6. Communication: Stakeholder, particularly staff and their representatives are to be kept informed of
the reasons for benchmarking.

Difficulties in implementation of Benchmarking

1. Time consuming: Benchmarking is time consuming and at times difficult. It has significant
requirement of staff time and company resources. Companies often waste time in benchmarking non-
critical functions.
2. Lack of Management Support: Benchmarking implementation require the direct involvement of the
senior manager etc. The drive to be best in the industry or world cannot be delegated.
3. Resistance from employees: It is likely that there may be resistance from employees.
4. Paper Goals: Companies can become preoccupied with the measure. The goal becomes not to
improve process but to match the best practices at any cost.
5. Copy-paste attitude: The key element in benchmarking is the adaptation of a best practice to tailor it
to a company’s needs and culture. Without that step, a company merely adopts another company’s
process. This approach condemns benchmarking to fail.

Benchmarking Code of Conduct:

To contribute to efficient, effective, and ethical benchmarking, individuals agree for themselves and their
organization to abide by the following principles for benchmarking with other organizations.
1. Principle of Legality: Avoid discussion or actions that might lead to or imply an interest in restraint of
trade; market or customer allocation schemes, price fixing dealing arrangements bid rigging, bribery
or misappropriation. Do not discuss costs with competitors if costs are an element of pricing.
2. Principles of Exchange: Be willing to provide the same level of information that you request in any
benchmarking exchange.
3. Principle of Confidentiality: Treat benchmarking interchange as something confidential to the
individuals and organizations involved. Information obtained must not be communicated outside the
partnering organization without prior consent of participating benchmarking partners. An
organization’s participation in a study should not be communicated externally without their
permission.
4. Principle of Use: Use information obtained through benchmarking partnering only for the purpose of
improvement of operations with the partnering companies themselves. External use or
communications of a benchmarking partner’s name with their data of observed practices requires
permission of that partner. Do not, as a consultant of client, extend one company’s benchmarking
study findings to another without the first company’s permission.
5. Principle of first part Contact: Initiate contacts, whenever possible, though a benchmarking contact
designated by the partner company. Obtain mutual agreement with the contact on any hand off of
communication or responsibility to other parties.
6. Principle of Third Party Contact: Obtain an individual’s permission before providing their name in
response to a contact request.
7. Principle of Preparation: Demonstrate commitment to the efficiency and effectiveness
benchmarking process with adequate preparation at each process particularly, at initial partnering
contact.

Question 31: What are benchmarking code of conduct? (4 Marks) May/10-O.C. & (3 Marks) Nov./04

Question 32: Describe the four types of bench marking of critical success factors. (4 Marks) Nov./08-O.C.
Question 33: Explain briefly stages involved in the process of Bench marking (5 Marks) Nov./09-O.C.

CA. Parag Gupta Ph.: +91 11 47665555 Paraggupta_ca@yahoo.co.in Costing & O.R.
World’s largest CA Final student’s consultancy group: http://groups.yahoo.com/group/costingbyparaggupta
Miscellaneous Theory Topics - 258 -

JUST IN TIME & BACK FLUSHING

Question 34: What is JIT? Explain how it eliminates wastage of resources. (4 Marks) Nov./03
Or
Describe Just-in-time system. (6 Marks) Nov./08-N.C.
Ans.: Just-in-time (also popularly known as lean production or stockless production) is can be described as
an approach with the objective of producing the right part in the right place at the right time (in other words,
“just in time”).
JIT is, in its most basic form, the systematic elimination of waste – overproduction, waiting, transportation,
inventory, motion, over-processing, defective units – and the implementation of the concepts of continuous
flow and customer pull.
The philosophy of JIT is simple - inventory is defined to be waste. It is best described as a philosophy of
management, dedicated to the elimination of waste. JIT applies primarily to repetitive manufacturing
processes in which the same products and components are produced over and over again. An activity that
consumes resources but creates no value for the customer. Muda (vulgar Japanese word for “waste”) is
divided into Muda 1 and Muda 2. Muda 1 is waste that creates no value but in unavoidable with current
technologies and policies. An example would be the payroll process. Muda 2 creates no value and can be
eliminated. An example would be shop-floor labor reporting. Waste results from any activity that adds cost
without adding value, such as the unnecessary moving of materials, the accumulation of excess inventory, or
the use of faulty production methods that create products requiring subsequent rework. JIT is an inventory
strategy implemented to improve the return on investment of a business by reducing in-process inventory
(increasing the inventory turnover rate) and its associated carrying costs.
Generally, only a small fraction of the total time and effort actually
adds value for the end customer

Value Adding
Activities
Necessary Non-
Value Adding
Activities

Waste Activities

The basic elements of JIT were developed by Toyota in the 1950's, and became known as the Toyota
Production System (TPS). JIT was well-established in many Japanese factories by the early 1970's. JIT
began to be adopted in the U.S. in the 1980's (General Electric was an early adopter), and the JIT/lean
concepts are now widely accepted and used. Some companies in India which are following the JIT system
are HONDA MOTORS, AT & T & SIEMENS.

Question 35: What are key elements of JIT? Explain in details.


Ans.: 1. Reduced Setup Cost and Times : from months to hours thus making small-lot production
economically viable; achieved by organizing procedures, using carts, and training workers to do their own
setups. Aim for single digit setup times (less than 10 minutes) or "one-touch" setup -- this can be done
through better planning, process redesign, and product redesign. A good example of the potential for
improved setup times can be found in auto racing, where a NASCAR pit crew can change all four tires and
put gas in the tank in under 20 seconds. The pit crew’s efficiency is the result of a team effort using
specialized equipment and a coordinated, well-rehearsed process. Similarly in industries for semi-versatile
machinery such as big stamping presses

2. Small-Lot production & reduced lead timesA : reducing setup times allows economical production of smaller
lots; close cooperation with suppliers is necessary to achieve reductions in order lot sizes for purchased
items, since this will require more frequent deliveries; lead times can be reduced through close cooperation
with suppliers, possibly by inducing suppliers to locate closer to the factory. Control systems such as a
kanban (card) system (or other signaling system) should be used to convey parts between work stations in
small quantities (ideally, one unit at a time).

CA. Parag Gupta Ph.: +91 11 47665555 Paraggupta_ca@yahoo.co.in Costing & O.R.
World’s largest CA Final student’s consultancy group: http://groups.yahoo.com/group/costingbyparaggupta
Cost Accounting & Management - 259 -

A
[ Lead Time is the amount of time between placing of an order & the receipt of the goods ordered.]

Reduced
Setup Cost
and Times
Supply Chain Small-Lots &
Management reduced lead
times

Key
Elements of Cellular
Material
handling JIT manufacturing
systems & working cells

Total Quantity Employee


Control involvement &
empowerment

3. Cellular manufacturing & working cells3 : production lead times can be reduced by moving work stations
closer, together applying group technology and cellular manufacturing2 concepts, reducing queue length
(reducing the number of jobs waiting to be processed at a given machine), and improving the coordination
and cooperation between successive processes. The general idea is that the machines needed to carry out
the manufacture of a particular product family1 are arranged in the form of discrete ‘mini’ assembly line. The
machines are grouped together in a sequential order required by processing so that there is an even,
constant and balanced flow of materials throughout the entire production process without queuing by
machines or returning to stores.
E.g., Traditional manufacturing operations are organized by functional specialty into plant departments that all
presses are in one department, all milling machines in another, welding machines in another and so forth.
Workers are spread out along linear shaped production lines and inevitably produce goods in large batches or
lots. Batch oriented linear production means that an operation completes all of the shop order and then
moves it to the next operation for further processing. In a JIT cellular organization, just the opposite is the
norm. Production is organized by product rather than by function with equipment dedicated or partially
dedicated to a family of products. Production in such "cells" is conducted sequentially in the order in which
operations must be performed to produce the end item. Workers in this type of environment are in close
proximity to one another and are performing operations on lots of one.
[1. Product family: Products with similar production requirements.
2. Cellular manufacturing: The layout of machines of different types performing different operations in a tight sequence, typically in a U-shape, to permit
single-piece flow and flexible deployment of human effort by means of multi-machine working.
3. Working cell: A small cluster of machines, which can be run by a single machine operator.]

4. Employee involvement and empowerment: workers should be trained to operate several machines, to do
several specialized tasks, to perform minor maintenance tasks, for house keeping and to perform quality
inspections. Use machine and worker idle time to maintain equipment and prevent breakdowns. In general,
JIT requires teams of competent, empowered employees who have more responsibility for their own work.

5. Implement a zero defects quality program : A quality at the source program must be implemented to give
workers the personal responsibility for the quality of the work they do, the authority to stop production when
something goes wrong & if possible to immediately fix it & If the defect cannot be readily fixed to halt the
entire line by pulling a cord (called jidoka).

6. Stabilize and level the Master Production Schedule (MPS)4 with uniform plant loading: create a uniform
load on all work centers through constant daily production (establish freeze windows to prevent changes in
the production plan/schedule for some period of time) and mixed model assembly (produce roughly the same
mix of products each day, using a repeating sequence if several products are produced on the same line).
Meet demand fluctuations through end-item inventory rather than through fluctuations in production level.
Use of a stable production schedule also permits the use of backflushing to manage inventory: an end item’s
bill of materials is periodically exploded to calculate the usage quantities of the various components that were
used to make the item, eliminating the need to collect detailed usage information on the shop floor.
[4. Master Production Schedule: It specifies the quantity of each finished unit of products to be produced, and the time at which each unit will be required.]

CA. Parag Gupta Ph.: +91 11 47665555 Paraggupta_ca@yahoo.co.in Costing & O.R.
World’s largest CA Final student’s consultancy group: http://groups.yahoo.com/group/costingbyparaggupta
Miscellaneous Theory Topics - 260 -

7. Supplier involvement, supplier quality assurance and: the manufacturer should treat its supplier as a long-
term partners; they often must be trained in ways to reduce setup times, inventories, defects, machine
breakdowns, etc. in order to enable them to take responsibility for delivering the best possible parts/services
to the manufacturer in a timely manner. Errors leading to defective items must be eliminated, since there are
no buffers of excess parts.

Question 36: What do you mean by Kanban Production Control System & how does it differ from traditional
systems? Give appropriate examples.
Ans.: A kanban or “pull” production control system uses simple, visual signals to control the movement of
materials between work centers as well as the production of new materials to replenish those sent
downstream to the next work center. As implemented in the Toyota Production System, a kanban is a card
that is attached to a storage and transport container. It identifies the part number and container capacity,
along with other information, and is used to provide an easily understood, visual signal that a specific activity
is required.
Egs. of Pull system:
1. Imagine a supermarket, where there are cards attached to all items on shelves. At the checkout, the
cashier takes the cards off the items and sends them to the warehouse, who send replacement items. The
warehouse also uses the same system, and when a carton is picked off the shelves, the card attached to it is
sent further back upstream.
2. Kanban system implementation might be a "three-bin system" for the supplied parts (where there is no in-
house manufacturing) — one bin on the factory floor, one bin in the factory store and one bin at the suppliers'
store. The bins usually have a removable card that contains the product details and other relevant information
— the kanban card. When the bin on the shop floor is empty, the bin and kanban card are returned to the
store. The store then replaces the bin on the factory floor with a full bin, which also contains a kanban card.
The store then contacts the supplier and returns the now empty bin with its kanban card. The suppliers
inbound product bin with its kanban card is then delivered into the factory store completing the final step to
the system. Thus the process will never run out of product and could be described as a loop, providing the
exact amount required, with only one spare so there will never be an issue of over-supply. This 'spare' bin
allows for the use, transport and uncertainty in supply that are inherent in the system.
Eg. Of Push System:
Consider the alternative. Every Friday, the warehouse sends three cartons of frozen peas to each
supermarket. The number of cartons delivered is based on a calculation, done at some time in the past, on
average usage. This means that some supermarkets will run out, whilst others will have peas coming out of
their ears! This is a push system.

Many manufacturers have implemented electronic kanban systems. Electronic kanban systems, or E-Kanban
systems, help to eliminate common problems such as manual entry errors and lost cards. E-Kanban systems
can be integrated into enterprise resource planning (ERP) systems.

In some pull systems, other signaling approaches are used in place of kanban cards. For example, an empty
container alone (with appropriate identification on the container) could serve as a signal for replenishment.
Similarly, a labeled, pallet-sized square painted on the shop floor, if uncovered and visible, could indicate the
need to go get another pallet of materials from its point of production and move it on top of the empty square
at its point of use.

A kanban system is referred to as a pull-system, because the kanban is used to pull parts to the next
production stage only when they are needed. In contrast, an MRP5 system (or any schedule-based system)
is a push system, in which a detailed production schedule for each part is used to push parts to the next
production stage when scheduled. Thus, in a pull system, material movement occurs only when the work
station needing more material asks for it to be sent, while in a push system the station producing the material
initiates its movement to the receiving station, assuming that it is needed because it was scheduled for
production. The weakness of a push system (MRP) is that customer demand must be forecast and
production lead times must be estimated. Bad guesses (forecasts or estimates) result in excess inventory
and the longer the lead time, the more room for error. The weakness of a pull system (kanban) is that
following the JIT production philosophy is essential, especially concerning the elements of short setup times
and small lot sizes, because each station in the process must be able to respond quickly to requests for more
materials.
[5. Material Requirements Planning (MRP) is software based production planning and inventory control system used to manage manufacturing processes.]

CA. Parag Gupta Ph.: +91 11 47665555 Paraggupta_ca@yahoo.co.in Costing & O.R.
World’s largest CA Final student’s consultancy group: http://groups.yahoo.com/group/costingbyparaggupta
Cost Accounting & Management - 261 -

Question 37: How does the JIT approach help in improving an organization’s profitability? (4 Marks) May/07
OR
Explain, how the implementation of JIT approach to manufacturing can be a major source of competitive
advantage. (4 Marks) Nov./08-O.C.
Ans.: JIT approach helps in the reduction of costs/increase in prices as follows:
(i) Least scrapping - Immediate detection of defective goods being manufactured so that early correction is
ensured with least scrapping.
(ii) Reduced WIP costs - Eliminates/reduces WIP between machines within working cell.
(iii) Reduced Overhead Costs - Overhead costs from non-value added activities in the form of rentals for
inventory, insurance, maintenance costs etc. are reduced.
(iv) Higher Selling Price - Higher product quality ensured by the JIT approach leads to higher premium in the
selling price.
(v) Early Detection - Detection of problem areas due to better production/scrap reporting/labour tracing and
inventory accuracy lead to reduction in costs by improvement.

Question 38: Explain in brief the JIT approach for reducing WIP inventory.
OR
Difference in operating speeds of machines may lead to higher WIP inventory. How does JIT system resolve
this issue?
Ans.: JIT approach for reducing WIP inventory:
At times, there may be huge differences between the operating speeds of different machines, e.g. Process 1
of machinery may produce 180 components per hour whereas Process II machine may finish only 135 units
per hour. This affects cost in following manner:
 Piling up of WIP inventory: Work-in-process inventory builds up in front of the slowest machines.
 Delayed tracing of Defectives: Defective parts produced by an upstream machine may not be
discovered until the next downstream machine operator finds them later. By that time, the upstream
machine may have created more defective parts, all of which must now be destroyed or reworked.
In JIT philosophy, there are two ways to resolve the above problems.
1. Kanban Card: Explained in detail in Question 3
2. Working Cells: Explained in detail in Question 2 Point 3

Question 39: What are the advantages of working cells in JIT environment?
Ans.: The establishment of working cells has the following advantages:
 The individual machine operator takes each output part from machine to machine within the cell; and
thus there is no way for WIP to build up between machines.
 The operator can immediately identify defective output which otherwise is difficult for each machine of
the cell. The smaller machines used in a machine cell are generally much simpler than the large
automated machinery they replace. Hence maintenance costs are reduced.
 It is much easier to reconfigure the production facility when it is necessary to produce different
products, avoiding the large expense of carefully repositioning and aligning equipment.

Question 40: How does JIT help in shortening set-up and operation times?
OR
Outline the JIT approach for shortening set-up and operation times.
Ans.: Long set-ups and operation time involve indirect costs like product obsolescence, inventory carrying
costs, and many defective products (because problems may not be discovered until a large number of items
have already been completed). This problem will be resolved under JIT by adopting the following steps.
1. Test data: A videotape of a typical set is prepared for analysis purposes.
2. Evaluation: A team of industrial engineers and machine users examine this tape, spotting and gradually
eliminating steps that contribute to a lengthy set-up.
3. Motion and time study: By eliminating unnecessary production steps and improving others after a number
of iterations, it is possible to achieve substantially lower set-up times than before.
4. Effects: Reduction in set-up time has the following effects: Reduction in the amount of work-in-process,
Reduction in the number of products that can be produced before, defects are identified and fixed, thereby
reducing scrap costs.

Question 41: List a few benefits associated with JIT system.


Ans.: Benefits associated with JIT system are:

CA. Parag Gupta Ph.: +91 11 47665555 Paraggupta_ca@yahoo.co.in Costing & O.R.
World’s largest CA Final student’s consultancy group: http://groups.yahoo.com/group/costingbyparaggupta
Miscellaneous Theory Topics - 262 -

1. Reduction in Inventory levels: Unnecesary piling up of Raw Materials, WIP and finished goods are
avoided. The focus is on production and purchase as per the firm’s requirements.
2. Reduction in Wastage of Time: Wastage of time in various ways like inspection time, machinery set-up
time, storage time, queue time, defectives rework time etc., are reduced.
3. Reduction in Scrap Rates: There will be sharp reductions in the rates of defectives or scrapped units. The
workers themselves identify defects and take prompt action to avoid their recurrence.
4. Reduction in Overhead Costs: By reducing unnecessary (non value-added) activities and the associated
time and cost-drivers, overheads can be greatly reduced e.g. material handling costs, rework costs, facility
costs etc.

Question 42: Explain the Impact of JIT on Product Prices.


Ans.: When a company achieves a higher level of product quality, along with ability to deliver products on the
dates required, customers may be willing to pay a premium. If customers are highly sensitive to quality or
delivery reliability (which are the benefits of JIT), it may be possible to increase price substantially.
If customers place a higher degree of importance on other factors, then there will be no opportunity for a price
increase. In case all firm in an industry adopt JIT, they offer the same level of quality and service. JIT
philosophy, in such cases, just keeps a company from losing sales to its competitors.
The impact of a JIT system on product pricing is primarily driven by customer’s perceived need for higher
product quality and reliable delivery times, as well as the presence of competitors with JIT system, the same
installation, and operational base.

Question 43: Identification of Machine Cells under JIT for systematic Overhead Cost Allocation.
Ans.: a. A working Cell or a machine Cell is a small cluster of machines, which can be run by a single
machine operator.
b. It designed to produce either a single product or a single component that goes into a similar product line.
Therefore all costs generated by the machine cell can be charged directly to the only product it produces.
c. When a company completely changes over to the use of machine cells in all locations, the cost related to
all the cells can now be changed directly to products. The balance costs left may be assigned to the
Overhead Cost Pool and identified with the products through Activity Based Costing. This results in more
accurate product costs.
Some examples of shift from Overheads to Direct Machine Cell Costs are:
a. Depreciation: Depreciation of each machine in a machine cell can be changed directly to a product. It may
be possible to depreciate a machine based on its actual use, rather than charging off a specific amount per
month.
b. Electricity: Power used by the machine in a cell can be separately metered and charged directly to the
products that pass through the cell. Excess electricity cost charged to the facility as whole has to be charged
to an overhead cost pool for allocation.
c. Material handling: In a JIT system, most material handling cost are limited since machine operators move
parts around within their machine cells. Only costs for materials handling between cells and charged to an
overhead cost pool for allocation.
d. Operating Suppliers: Supplies are used mostly with the machine cells to the majority of item sin this
expense category can be separately tracked by individual cell and charged to products directly.
e. Repairs and maintenance: All maintenance costs incurred for machinery can be grouped into machine
cells. By having the maintenance staff, charge their time and materials to these cells, these costs can be
charged straight to products. Maintenance work on the facility will be charged to an Overhead cost pool.
f. Supervision: If supervision is by machine cell, the cost of the supervisor can be split among the cells
supervised. However the cost of general facility management as well as of any support staff, must still be
charged to an overhead cost pool.
With such a higher proportion of direct costs associated with each product managers have much more
relevant information about the true cost of each product manufactured.

Question 44: What do you mean by ‘Back flushing’ in JIT system? Explain briefly the problems with back
flushing that must be corrected before it will work properly.
(4 Marks) May/10-N.C. & (4 Marks) Nov./04 & (5 Marks) June/09-OC
Ans.: Backflush accounting is a product costing approach, used in a Just-In-Time (JIT) operating
environment, in which costing is delayed until goods are finished. Standard costs are then flushed backward
through the system to assign costs to products. The result is that detailed tracking of costs is eliminated.
Journal entries to inventory accounts may be delayed until the time of product completion or even the time of
sale, and standard costs are used to assign costs to units when journal entries are made, that is, to flush

CA. Parag Gupta Ph.: +91 11 47665555 Paraggupta_ca@yahoo.co.in Costing & O.R.
World’s largest CA Final student’s consultancy group: http://groups.yahoo.com/group/costingbyparaggupta
Cost Accounting & Management - 263 -

costs backward to the points at which inventories remain. On completion of finished goods, its quantity is
multiplied by components required per item produced (as per bills of material). Resultant value is subtracted
from beginning inventory; balance inventory is closing stock/inventory. The principle of backflushing is that
whenever one part is made, the stock of the components on the part's bill of material (BOM)8 can be reduced
by the quantity on the bill of material. If, for instance, the stock of car batteries is held beside the car assembly
line, the stock of car batteries can be reduced by one every time a car is made (typically the component stock
is reduced when the work order is booked into stock). It can be argued that backflush accounting simplifies
costing since it ignores both labour variances and work-in-progress.
The problems of back flushing that must be corrected before it will work properly are:
1. Accurate BOM - Bill of material accuracy must be maintained at 100% including scrap factors.
2. Proper changes in BOM - Any changes to bills of material must be timed to match the backflush of
the changed process.
3. Inventory accuracy - Stock checking is more difficult as computer record must be checked with stock
on the shelf plus all materials and ingredients that have been issued but not yet backflushed.
4. Scrap reporting - All scrap and re-work must be reported as soon as it happens.
5. Production reporting - Every single jobs requiring any stocked materials must have a work order
opened and backflushed.
6. Lot Tracing – Lot tracing is impossible under the back flushing system. This is required when a
manufacturer needs to keep records of which production lots were used to create a product in case
all the items in a lot need to be recalled. Only a picking system (issue material from stores on a pick
list based on a works order system) can adequately record this information. Some computer systems
allow picking and backflushing system to coexist.
[8. BOM File specifies sub-assemblies components and materials required for each finished good.]

Question 45: Discuss the treatment of material, labour, overheads, & WIP in back flush accounting.
Ans.: Traditional & Standard costing systems use sequential tracking costing system for accounting costs.
It has trigger points7, corresponding to journal entries as transaction occur i.e. purchase, issue of material,
production, overhead absorption, completion of finished goods, etc.
An alternative approach to sequential tracking is backflush costing. Backflush accounting system simplifies
the accounting records by avoiding the need to follow the movement of materials and work-in-progress
through the manufacturing process within the organization.

There are several variants of backflush accounting, one of them is that inspite of separate raw materials and
WIP A/c there is single Raw and in-Process A/c (RIP). The use of standard costs and variances is likely to
be incorporated into the accounting entries. Transfers from raw and in-progress account to finished goods (or
cost of sales) will probably be made at standard cost. The difference between the actual inputs and the
standard charges from the raw and in-progress account will be recorded as a residual variance which will be
recorded in the profit and loss account. Thus, it is essential that standard costs are a good surrogate for
actual costs if large variances are to be avoided.

Conversion costs are recorded as incurred in backflush accounting system. All indirect expenses (Direct
Labour is treated as indirect cost) are treated as period costs in it. Direct Labour and overhead are
combined into temporary A/c i.e. Conversion cost control A/c. This A/c accumulates the actual conversion
cost on debit side and applied conversion cost on the credit side.

Naturally, management will still be eager to ascertain the cause of any variances which arise from the
inefficient usage of materials, labour, and overheads. However, investigations are far more likely to be
undertaken using non-financial performance indicators as opposed to detailed cost variances.
Backflush accounting is ideally suited to a just-in-time philosophy and is employed where the overall cycle
time is relatively short and inventory levels are low.

[7. Trigger point = point at which entries are made in A/cing system. It may be receipt of materials or completion/sale of goods. If system is full JIT,
completion is trigger point.]

Question 46 [JIT]: The Evans Corporation manufactures wireless telephone. Evans is deciding whether to
implement a JIT production system. Which would require annual tooling costs of `150000. Evans estimated
that the following annual benefits would arise from JIT production.

a. Average inventory would decline by `700000. from `900000 to `200000.

CA. Parag Gupta Ph.: +91 11 47665555 Paraggupta_ca@yahoo.co.in Costing & O.R.
World’s largest CA Final student’s consultancy group: http://groups.yahoo.com/group/costingbyparaggupta
Miscellaneous Theory Topics - 264 -

b. Insurance space materials – handing , and setup costs, which currently total `200000,would decline
by 30%.
c. The emphasis on quality inherent in JIT system would reduce rework costs by 20%. Evans currently
incurs `3,50,000 on rework.
d. Better quality would enable Evans to raise the selling prices of its products by `3 per unit Evans sells
30,000 units each year.

Evans’s required rate to return on inventory investment is 12% pr year.

Required;

Calculate the net benefit or cost to the Evans Corporation from implementing a JIT production system.
Suppose Evans implements JIT productions. (a) Give examples of performance measures Evans could use to
evaluate and control JIT production. (b) What is the benefits to Evans of implementing an enterprise resource
planning (ERP) system?

[Ans.: Annual Relevant Costs of Current Production System and JIT Production System for Evans
Corporation.
Relevant Relevant
Costs under Costs under
Current JIT
Production Production
Relevant Items System System
Total net incremental costs `6,58,000 `5,04,000
Annual difference in favor of JIT production `1,54,000
Question 47 [JIT]: X Video Company sells package of blank video tapes to its customer. It purchases video
tapes from Y Tape Company @ `140 a packet. Y Tape Company pays all freight to X Video Company. No
incoming inspection is necessary because Y Tape Company has a superb reputation for delivery of quality
merchandise. Annual demand of X Video Company is 13,000 packages. X Video Co. requires 15% annual
return on investment. The purchase order lead time is two weeks. The purchase order is passed through
Internet and it costs `2 per order. The relevant insurance, material handling etc ` 3.10 per package per year.
X Video Company has to decide whether or not to shift to JIT purchasing. Y Tape Company agrees to deliver
100 packages of video tapes 130 times per year (5 times every two weeks) instead of existing delivery system
of 1,000 packages 13 times a year with additional amount of `0.02 per package. X Video Co. incurs no stock
out under its current purchasing policy. It is estimated X Video Co. incurs stock out cost on 50 video tape
packages under a JIT purchasing policy. In the event of a stock out, X Video Co. has to rush order tape
packages which costs `4 per package. Comment whether X Video Company should implement JIT
purchasing system.

Z Co. also supplies video tapes. It agrees to supply @ `13.60 per package under JIT delivery system. If video
tape purchased from Z Co., relevant carrying cost would be `3 per package against `3.10 in case of
purchasing from Y Tape Co. However Z Co. doesn’t enjoy so sterling a reputation for quality. X Video Co.
anticipates following negative aspects of purchasing tapes from Z Co.

- To incur additional inspection cost of 5 paisa per package.


- Average stock out of 360 tapes packages per year would occur, largely resulting from late deliveries. Z Co.
cannot rush order at short notice. X Video Co. anticipates lost contribution margin per package of `8 from
stock out.
- Customer would likely return 2% of all packages due to poor quality of the tape and to handle this return an
additional cost of `25 per package.
- Comment whether X Video Co places order to Z Co. (12 Marks) Nov/05

[Ans.: (i)
Particulars Current Policy JIT
` `
Total relevant cost 18,32,076 18,21,925.15
Comments:. Hence, a JIT purchasing policy should be adopted by the company.
Comments : The comparative costs are as follows,
CA. Parag Gupta Ph.: +91 11 47665555 Paraggupta_ca@yahoo.co.in Costing & O.R.
World’s largest CA Final student’s consultancy group: http://groups.yahoo.com/group/costingbyparaggupta
Cost Accounting & Management - 265 -

Under current policy ` 18,32,076.00


Under purchase under JIT ` 18,21,925.10
Under purchase from Z Co Ltd ` 1,87,342.00
Packages should be bought from Z Co as it is the cheapest.]

MANUFACTURING RESOURCE PLANNING & ENTERPRISE RESOURCE PLANNING

MANUFACTURING RESOURCES PLANNING(MRP I&II)

It is a part of production operation system. Management has to develop a lot of strategies for production plan.
In early 1960’s a material acquisition plan was first introduced known as Material Requirement Plan ( MRP-I).
MRP-2 is latest all-round development of that plan.
A brief history of MRP –1
Material requirement planning is a computerized production scheduling system which takes the forward
schedule of final product requirements (the master production schedule) and translates it progressively into
the numbers of sub-assemblies, components and raw materials required at each stage of the manufacturing
cycle.
It is a management information system providing a basis for production decisions when what is manufactured
has a composite structure and when lead items are important features. Obviously, the ability of the system to
deliver what is required in the correct place at the correct time will be dependent on the quality of information
which is put into the computer model.

AIMS OF MATERIAL REQUIREMENT PLANNING:


1. Determine for final products namely, what should be produced and at what time.
2. Ascertaining the required units of production of sub-assemblies.
3. Determining the requirement for materials based on an up-to-date bill of materials file (BOM).
4. Computing inventories, WIP, batch sizes and manufacturing and packaging lead times.
5. Controlling inventory by ordering bought-in components and raw materials in relation to the orders
received or forecast rather than the more usual practice of ordering from stock-level indicators.
Benefits: Detailed forecast of the inventory position is highlighted period by period.

DATA REQUIREMENTS TO OPERATE MATERIAL REQUIREMENT PLANNING SYSTEM:

The master Production schedule: This schedule specifies the quantity of each finished unit of products to
be produced, and the time at which each unit will be required.
The Bill of material file: The bill of material file specifies the sub-assemblies, components and materials
required for each finished good.
The inventory file: This file maintains details of items in hand for each sub-assemblies, components and
materials required for each finished goods.
The routing file: This file specifies the sequence of operations required to manufacture components, sub-
assemblies and finished goods.
The master parts file: This file contains information on the production time of sub- assemblies and
components produced internally and lead times for externally acquired items.

Method of operation of material requirement planning system:


A material requirements planning (MRP) system is a computer based inventory information system which is
used to plan and control raw material and component parts inventories.
Like all computer-based information system, MRP systems can be divided into following:
o Pre-requisite information
o System input
o System processing
o System output

Pre-requisite information and system input:


1. The master production schedule (MPS) file states the production goal, generally for a week time, in terms
of desired units of production. MRP system first focuses on the forecasted units of production and timing of

CA. Parag Gupta Ph.: +91 11 47665555 Paraggupta_ca@yahoo.co.in Costing & O.R.
World’s largest CA Final student’s consultancy group: http://groups.yahoo.com/group/costingbyparaggupta
Miscellaneous Theory Topics - 266 -

finished goods demand and the determines the demand for materials, components and sub-assemblies at
each stages of production. This makes MRP a push system in which once the scheduled production starts,
the output of each department is pushed through the system to the next department for processing or into
inventory to be retrieved later.
2. The bill of materials (BOM) file contains information about how the production of the finished goods is
undertaken. A bill of material structure is used:
a) To assess all of the raw materials and component parts required to complete a product, and,
b) To describe the multiple levels of assembly or manufacturing necessary to complete a unit of finished
product.
3. The inventory records files of the MRP system defines current levels of finished goods, raw materials, and
component parts inventory at the beginning of some planning period. During the planning period, the
organization may receive units of raw materials, components parts, sub-assemblies, and even finished goods
inventory from suppliers, vendors, and subcontractors. These planned inventory receipts and delivery lead
times are included in the inventory records file so that their addition can be appropriately considered in the
time bucket of their arrival.

Pre-requisites for successful operation of MRP:


1. Strict adherence to the schedule: The successful operation of MRP system requires a strict adherence to
the latest production and purchasing schedules. Workers must be educated to understand the importance of
schedule adherence, and controls should be in place to ensure this adherence.
2. Accurate data base: Data accuracy is vital to the system. If a plan is based on inaccurate data it may be
impossible to adhere to the schedule. For example, if the bill of materials file is not updated to reflect any
changes in product composition it will be impossible to adhere to the schedule.

MRP- II
When the scope of MRP-1 is developed further, which includes
1. Planning of raw material
2. Planning of component & sub- assemblies
3. Compute the other resources e.g. machine or labour capacity
4. To create a full integrated plan for management
then it is known as Manufacturing resources planning ( MRP – 2)
MRPII (also written MRP-2 ) adds the MRP schedule into a capacity planning system and then builds the
information into a production schedule. It is also seen as a link between strategic planning and manufacturing
control. The sequence of events is as follows:
From that document, a manufacturing, plan is developed based upon inputs from purchasing & production.
Adjustments may be necessary to allow for production rates. Possible inventory levels in seasonal trades &
the size of the workforce. The manufacturing plan leads into a detailed master production schedule which is
akin to the original philosophy of MRP already outlined.

If correctly applied, MRPII provides a common data base for the different function units
such as manufacturing, purchasing and finance within a firm.

ENTERPRISE RESOURCE PLANNING (ERP)

ERP refers to software, which integrates all departments and functions across a company into a single
computer system that can serve all those needs of different departments.
It combines all computerized departments together with the help of a single integrated software program that
uses a single database so that various departments can more easily share information and communicate with
each other.

Need for ERP:


1. Complete Automation and Faster Service: ERP automates the tasks involved in performing a
business process such as order fulfillment, which involves taking an order from a customer,
shipping it and billing for it. The order process moves like a bolt of lightning through the organization,
and customers get their orders faster and with fewer errors than before. Similarly, the major
business processes like employee benefits or financial reporting can be speeded up.
2. Standardized Processes: Manufacturing companies find that multiple business units (departments)
across the company adopt different methods and computer systems for the same product.

CA. Parag Gupta Ph.: +91 11 47665555 Paraggupta_ca@yahoo.co.in Costing & O.R.
World’s largest CA Final student’s consultancy group: http://groups.yahoo.com/group/costingbyparaggupta
Cost Accounting & Management - 267 -

Standardizing these using a single integrated computer system can save time and increase
productivity.
3. Integrated Financial Data: ERP creates a single version of the financial position and
performance that cannot be questioned because everyone is using the same system. It is very useful
in analyzing the performance and deviations of different business units (responsibility centres)
rather than obtaining individual reports from each such business unit.
4. Standardised HR Information: HR may not have a unified, simple method for tracking employee
time and communicating with them about benefits and services. ERP can help companies with
multiple business units in this regard.
5. Tailor-made: ERP systems are designed as per the requirements of individual companies based on
the nature, scale and methods of operations. It is superior to other standardized application packages
(software), which may not be fully useful to a multifaceted company.
6. Information Management: A good MIS should avoid information overload, ERP helps proper
information management since all data are made available at one place, accessible to different users
based on their individual requirements.

Components of ERP:
The following may be identified as the primary components (sub-systems) of ERP system:
Sales and marketing Shop floor control
Master scheduling Accounts payable/receivable
Material requirement Planning Logistics
Capacity requirement planning Asset Management
Bill of materials Purchasing Financial Accounting

Features of ERP:
1. Integrated: ERP facilitates company-wide information integration covering all functional areas like
manufacturing, selling and distribution, payables, receivables, inventory, accounts, human
resources, etc. ERP provides complete integration of systems not only across departments but also
across companies under the same management.
2. Information Sharing: ERP bridges the information gap across organizations.
3. Project Management: ERP is the solution for better project management
4. E-Com Facilities: ERP allows automatic introduction of technologies like Electronic Fund Transfer
(EFT), Electronic Data Interchange (EDI), Internet, Intranet, Video Conferencing, E-Commerce etc.
5. Business Decision Making Solution: ERP provides business intelligence tools like Decision
Support systems (DSS), Executive Information System (EIS), Reporting, Data Mining and Early
warning systems (Robots) for enabling people to make better decisions. It eliminates most business
problems like material shortage, productivity enhancements, customer service, cash management,
inventory problems, quality problems, prompt delivery etc.
6. Futuristic: ERP not only addresses the current requirements of the company but also provides the
opportunity of continually improving and refining business processes.

Benefits of ERP:
1. Product Costing: ERP system supports advanced costing methods like Standard Costing,
Actual Costing, Activity based costing, thereby helping in determination of cost products accurately.
2. Cost Monitoring and Control: ERP can integrate all costing methods and information with finance.
This provides the company with essential financial information or monitoring and controlling costs.
3. Planning and Managing: The ERP system simplifies complicated logistics and helps in planning for
and managing different divisions in different locations as a single unit.
4. Information Flow: The advanced utility of the ERP system helps in processing the flow of product
and financial information in several different ways.
5. Efficient Database Management: The ERP system aids in the efficient managing of data on
warehouse, suppliers customers etc. required to run an organization effectively and profitably.
6. Inventory Management: Inventory reporting supports all reporting of specific and general types of
stock transactions like stock transfers, reclassifications, ID changes and physical inventory results.
Also ERP can manage stock and purchase requisitions selections of appropriate locations for
receipts, inventory valuation, warehouse management and cost accounting.
7. Customer Satisfaction: ERP system defines the logistics processes flexibly and efficiently to
deliver the right product from the right warehouse to the right customer at the right time – every

CA. Parag Gupta Ph.: +91 11 47665555 Paraggupta_ca@yahoo.co.in Costing & O.R.
World’s largest CA Final student’s consultancy group: http://groups.yahoo.com/group/costingbyparaggupta
Miscellaneous Theory Topics - 268 -

time, thereby satisfying the customers. It also supports planning transportation, confirmation,
dispatch and proof of delivery processing. Additionally, it ensures better after sales service.
8. Competitive Edge: ERP system helps a company to gain the Competitive Edge by (a) enabling the
company to respond quickly and accurately to change in market conditions; (b) improving business
process (c) ensuring quality control; (d) improved and objective production planning; intranet and
Extranet Solutions.

Question 48: Write short notes on Material Requirement Planning (5 Marks) May/02
Question 49: Mention the data required to operate the material requirement planning system.
(4 Marks) Nov./10-N.C.
Question 50: State the major features of Enterprise Resource Planning (ERP)
(4 Marks) Nov./07, (3 Marks) May/03 & (4 Marks) Nov./02
Question 51: State the benefits accruing from Enterprise Resource Planning (ERP)
(4 Marks) May/04 & (4 Marks) Nov./10-O.C.
Question 52: (i) What do you mean by ERP? (2 Marks) Nov./06
(ii) Name six benefits of ERP in an enterprise. (3 Marks) Nov./06

COMPUTER-AIDED MANUFACTURING

The manufacturing process is carried out by a range of machinery that, together with its concomitant
software, comes under the collective heading of computer–aided manufacturing (CAM).
Maximum elements of CAM are computer numerical control (CNC) and robotics.
CNC machines are programmable machine tools. These are capable of performing a number of machining
tasks, e.g. cutting, grinding, moulding, bending etc.
A program stores all the existing manufacturing activities and set-up instructions for a particular machine or
bank of machines, providing facility of changing its configuration in a matter of seconds via the keyboard;
changes to existing configurations and new configurations are easily accommodated. CNC therefore offers
great flexibility, and reduces set-up times.
Human operators will tire and are error prone. CNC machines are able to repeat the same operation
continuously in identical manner, with high accuracy level.
For Example the car producer, found that the time taken to completely retool car body panel jigs in their
intelligent body assembly system (IBAS) fell from 12 months to less than
3 months by reprogramming the process machinery by computer and using computerised jig robots.

BUSINESS PROCESS RE-ENGINEERING

Business process re-engineering involves examining business processes and making substantial changes in
the day to day operation of the organisation. It involves the redesign of work by changing the activities.
A business process consists of a collection of activities that are linked together in a co- ordinated &
Sequential manner to achieve goal & objective.
For example, material handling might be classed as
 Scheduling production,
 Storing materials,
 Processing purchase orders,
 Inspecting materials, and,
 Paying suppliers.
The aim of business process re-engineering is to improve the key business process in an organisation by
focusing on
 Simplification,
 Cost reduction,
 Improved quality, and,
 Enhanced customer satisfaction

CA. Parag Gupta Ph.: +91 11 47665555 Paraggupta_ca@yahoo.co.in Costing & O.R.
World’s largest CA Final student’s consultancy group: http://groups.yahoo.com/group/costingbyparaggupta
Cost Accounting & Management - 269 -

UNIFORM COSTING

When several undertakings start using the same costing principles and/or practices they are said to be
following uniform costing. The basic idea behind uniform costing is that the different concerns in an industry
should adopt a common method of costing and apply uniformly the same principles and techniques for better
cost comparison and common good. The principles and methods of compilation, analysis, apportionment and
absorption of overheads differ from one concern to the other in the same industry; but if a common or uniform
pattern is adopted by all, it helps mutually in cost control and cost reduction. Therefore, it is necessary that a
uniform method of costing should be adopted by the member unit of an industry.

Objectives of Uniform Costing: The main objectives of Uniform Costing are as follows :—
1. To facilitate the comparison of costs and performances of different units in the same industry; it
provides objective basis.
2. To eliminate unhealthy competition among the different units of an industry.
3. To improve production capacity level and labour efficiency by comparing the production costs of
different units with each other.
4. To provide relevant cost information/data to the Government for fixing and regulating prices of the
products.
5. To bring standardization and uniformity in the operation of participating units.
6. To reduce production, administration, selling and distribution costs, and to exercise con- trol on fixed
costs.

Essential requisites for the installation of Uniform Costing System: A successful system of uniform
costing has the following requirements :—
(1) Co-operation among member units. There should be co-operation and a policy of give and take
amongst the different units in the industry which are to use uniform costing system. Without any
reservations, every participating member should be willing to supply the required information to a
central body appointed by themselves.
(2) Spirit of sharing experience. The bigger units should be prepared to share their experiences with the
smaller concerns so that the latter may be able to improve their performances.
(3) No rivalry. There should be no rivalry or jealousy amongst the participating members.
(4) Free exchange of ideas. Ideas should be freely exchanged amongst the participating members in
order to enable them to judge their efficiency with reference to other units.
(5) Leveling the key factors. Certain key factors such as methods and principles of cost accounting
peculiar to individual units should be carefully noted. These key factors will give different costs of
different undertakings in the same industry which will make cost comparison difficult. Thus efforts
should be made to locate the key factors which create differences. These varying factors should be
leveled out by giving a weight age on an equitable basis.
(6) Cost manual. A cost manual which lays down the recommended cost accounting plan should be
circulated amongst the participating members by the central body.
(7) Adequate information for categorization. Some sort of categorization should be made both in respect
of long-term and short-term differences in order to make comparison useful. To distinguish such
differences, adequate qualitative and quantitative information should be available.
(8) Central organisation. There should be central organization for the collection, co-ordination and
presentation of information. The method of collection and presentation of information should be
properly laid down and clearly indicated.
(9) Proper explanation of the scheme. The scheme must be properly explained to all concerned. This is
necessary to convince about its utility and make all concerned to work for its success.

Advantages of Uniform Costing: The advantages accruing from the use of uniform costing system are as
follows:
i. The management of each firm will be saved from the exercise of developing and introducing a costing
system of its own.
ii. A costing system devised by mutual consultation and after considering the difficulties and
circumstances prevailing in different firms is readily adopted and successfully implemented.
iii. It facilitates comparison of cost figures of various firms to enable the firms to identify their weak and
strong points besides controlling costs.
iv. Optimum achievement of efficiency is attempted by all the firms by utilizing the experience of other
concerns in the industry.

CA. Parag Gupta Ph.: +91 11 47665555 Paraggupta_ca@yahoo.co.in Costing & O.R.
World’s largest CA Final student’s consultancy group: http://groups.yahoo.com/group/costingbyparaggupta
Miscellaneous Theory Topics - 270 -

v. Standing in the industry of each firm will be known by making a comparison of its cost data with
others.
vi. Services of cost consultants or experts may be available jointly to each firm in the industry by sharing
their experiences and expenses.
vii. Research and development benefits of bigger firms may be made available to smaller firms.
viii. It helps in the reduction of labour turnover, as a uniform wage system is the pre-condition of a uniform
costing system.
ix. It helps Trade Associations in negotiating with the Government for any assistance or concession in
the matters of taxation, exports, subsidies, duties and prices determination etc.
x. Unhealthy competition is avoided among the firms in the same industry in framing pricing policies and
submitting tenders.
xi. Prices fixed on the basis of uniform costing are representative of the whole industry and thus are
reliable.
xii. Uniform costing provide a basis for the comparative assessment of the performance of two firms in
the same industry but in different sectors.
xiii. It helps the Government in regulating the prices of essential commodities such as bread, sugar,
cement, steel etc.

Limitations of Uniform Costing:


i. Sometimes it is not possible to adopt uniform standards, methods and procedures of costing in
different firms due to differing circumstances in which they operate. Hence, the adoption of uniform
costing becomes difficult in such firms.
ii. Disclosure of cost information and other data is an essential requirement of a uniform costing system.
Many firms do not wish to share such information with their competitors in the same industry.
iii. Small firms in an industry believe that uniform costing system is only meant for big and medium size
firms, because they cannot afford it.
iv. It induces monopolistic trend in the business, due to which prices may be increased artificially and
supplies withheld.

Question 53: What are the requisites for the installation of a uniform costing system?
(4 Marks) Nov./08-N.C. & (4 Marks) May/10-N.C.

Question 54: What is uniform costing? Why it is recommended? (4 Marks) June/09- N.C.

INTER-FIRM COMPARISON

It is technique of evaluating the performance, efficiency, costs and profits of firms in an industry. It consists of
voluntary exchange of information/data concerning costs, prices, profits, productivity and overall efficiency
among firms engaged in similar type of operations for the purpose of bringing improvement in efficiency and
indicating the weaknesses. Such a comparison will be possible where uniform costing is in operation.
An inter-firm comparison indicates the efficiency of production and selling, adequacy of profits, weak spots in
the organisation, etc. and thus demands from the firm’s management an immediate suitable action. Inter-firm
comparison may enable the management to challenge the standards which it has set for itself and to improve
upon them in the light of the current information gathered from more efficient units. Such a comparison may
be carried out in electrical industry, printing firms, cotton spinning firms, pharmaceuticals, cycle
manufacturing, etc.
Requisites of inter-firm comparison system: The following requisites should be considered while installing
a system of inter-firm comparison:
1. Centre for Inter-Comparison — For collection and analyzing data received from member units, for
doing a comparative study and for dissemination of the results of study a Central body is necessary. The
functions of such a body may be:
i. Collection of data and information from its members;
ii. Dissemination of results to its members;
iii. Undertaking research and development for common and individual benefit of its members;
iv. Organising training programmes and publishing magazines.
2. Membership - Another requirement for the success of inter-firm comparison is that the firms of
different sizes should become members of the Centre entrusted with the task of carrying out inter-firm
comparison.

CA. Parag Gupta Ph.: +91 11 47665555 Paraggupta_ca@yahoo.co.in Costing & O.R.
World’s largest CA Final student’s consultancy group: http://groups.yahoo.com/group/costingbyparaggupta
Cost Accounting & Management - 271 -

3. Nature of information to be collected - Although there is no limit to information, yet the following
information useful to the management is in general collected by the Centre for inter-firm comparison.
i. Information regarding costs and cost structures.
ii. Raw material consumption.
iii. Stock of raw material, wastage of materials, etc.
iv. Labour efficiency and labour utilisation.
v. Machine utilisation and machine efficiency.
vi. Capital employed and Return on capital.
vii. Liquidity of the organisation.
viii. Reserve and appropriation of profit.
ix. Creditors and debtors.
x. Methods of production and technical aspects.
4. Method of Collection and presentation of information - The Centre collects information at fixed
intervals in a prescribed form from its members. Sometimes a questionnaire is sent to each member; the
replies of the questionnaire received by the Centre constitute the information/data. The information is
generally collected at the end of the year as it is mostly related with final accounts and Balance Sheet. The
information supplied by firms is generally in the form of ratios and not in absolute figures. The information
collected as above is stored and presented to its members in the form of a report. Such reports are not made
available to non-members.

Advantages of Inter-firm comparison::


a) Such a comparison gives an overall view of the industry as a whole to its members– the present
position of the industry, progress made during the past and the future of the industry.
b) It helps a concern in knowing its strengths or weaknesses in relation to others so that remedial
measures may be taken.
c) It ensures an unbiased specialized reporting on particular problems of the concern.
d) It develops cost consciousness among members of the industry.
e) It helps Government in effecting price regulation.
f) It helps to improve the quality of products manufactured and to reduce the cost of production. It is
thus advantageous to the industry as well as to the society.

Limitations of inter-firm comparison


a) Top management feels that secrecy will be lost.
b) Middle management is usually not convinced with the utility of such a comparison.
c) In the absence of a suitable Cost Accounting System, the figures supplied may not be reliable for the
purpose of comparison.
d) Suitable basis for comparison may not be available.

Types of Comparisons
The following are the three types of comparisons made for this purpose:
(i) Comparison of Management Ratios. The management ratios are those which are linked to sales,
profits and assets of a business. These ratios are meant to provide the management in a nutshell, a
comparative picture of its operating performance, financial result, growth, liquidity etc. compared with
those of other firms in the industry or trade. These ratios are worked out on the basis of figures
supplied by each member. In the pyramid of ratios (given on previous page) the apex ratio is profit
related to the capital employed, which takes into account the various factors affecting the business.
The ratios worked out are useful to the management to the extent that the comparison reflects the
earning capacity, return on capital employed, earnings on fixed assets ,liquidity, growth etc, of the
business vis-à-vis others and on the basis of this information it can act for future improvement.
(ii) Comparison of cost ratios. Management may not be satisfied with the ratios calculated in (i)They
would like to go a step further to make inter-firm comparison more meaningful and to find out how
they are doing in relation to others as regards the cost of production. In a competitive world cost
ratios will assume greater importance for the simple reason that cost reduction becomes a
compelling necessity when there is acute competition. The members of the Association will, under
this type of inter-firm comparison, have to disclose much more information than they will be required
to do in case of (i). the advantages of cost ratio comparison will be more marked in the areas where
cost reduction is visualized.
(iii) Comparison of Technical Data. This of comparison will be of special interest to industries working in
highly competitive economies. Such comparison will gradually lead to rationalization of industry. It is

CA. Parag Gupta Ph.: +91 11 47665555 Paraggupta_ca@yahoo.co.in Costing & O.R.
World’s largest CA Final student’s consultancy group: http://groups.yahoo.com/group/costingbyparaggupta
Miscellaneous Theory Topics - 272 -

visualized that technical comparison will be in the realm of quantity of materials used, their
utilization, process involved, machinery used, and certain other technical aspects of production. The
following are the main ratios which are calculated for this purpose:

1. Quantity of Raw Material Consumed 2. Cost of Raw Materials Consumed


Man Hours / Machine House Man Hours / Machine Hours

3. Cost of Raw Materials Consumed 4. Cost of Scrap


Quantity Produced Cost of Raw Materials Consumed

5. Quantity of Scrap 6. Quantity Produced


Quantity of Raw Materials Consumed Rated Capacity

7. Quantity Produced 8. Cost of Rejected Material


Main Hours/Machine Hours Cost of Production

9. Cost of Reworking 10. Loss on Process


Cost of Production Cost of Material

11. Idle Time Hours 12. Overtime Hours


Total Available Time Main Hours

13. Cost of Idle Time 14. Cost of Overtime


Direct Labour Cost Direct Labour Cost

15. Power Units Consumed 16. Cost of Machine Maintenance


Machine Hours Cost of Production

17. Cost of Maintenance of Other Factory Assets


Cost of Production

Note: Where different processes are involved in the manufacturing of a product, ratios could be
worked out for each process.

Question 55: Make an assessment of the comparative position of firms A,B and C after calculating relevant
ratios on the basis of the following information:

Firm A Firm B Firm C


` ` `

Inventory (31-12-2006) 10,00,000 15,00,000 20,00,000


Sales (for 2006) 66,00,000 83,25,000 89,60,000
Cost of Goods Sold (for 2006) 60,00,000 75,00,000 80,00,000
Expenses of Management 5,00,000 7,50,000 10,00,000
Receivables (31-12-2006) 13,20,000 24,97,500 35,84,000

SOLUTION

Inventory Turnover Ratio = Cost of goods sold


Inventory

Firm A: `60,00,000 = 6 times


`10,00,000

Firm B: `75,00,000 = 5 times


`15,00,000

Firm C: `80,00,000 = 4 times

CA. Parag Gupta Ph.: +91 11 47665555 Paraggupta_ca@yahoo.co.in Costing & O.R.
World’s largest CA Final student’s consultancy group: http://groups.yahoo.com/group/costingbyparaggupta
Cost Accounting & Management - 273 -

`20,00,000

Form the above we see that inventory turnover ratio of firm A is better than that of B and C. Firm C
has the lowest ratio, i.e., it has the slowest moving stock.

Average Collection Period = Receivables x 360


Sales

Firm A: `13,20,000 x 360=72 days


`66,00,000

Firm B: `24,97,500 x 360=108 days


`83,25,000

Firm C: `35,84,000 x 360=144 days


`89,60,000

The average number of days credit allowed to customers is 72 days in firm A and 144 days in firm C, which is
just the double of A. It indicates that firm A is following a sound credit policy whereas firms B and C are
following a liberal credit policy. It is possible that firms B and C may have given credit to weak customers and
they are not making the payment in time.

Inventory turnover ratio and average collection period indicates that firm A is making an efficient use of its
working capital as compared to firms B and C. C’s position in this regard is the weakest.

Calculation of the Amount of Net Profit


Firm A Firm B Firm C
` ` `
Sales 66,00,000 83,25,000 89,60,000
Less: Cost of Goods sol 60,00,000 75,00,000 80,00,000
Gross Profit 6,00,000 8,25,000 9,60,000
Less: Expenses of Management 5,00,000 7.50,000 10,00,000
Net Profit 1,00,000 75,000 40,000
(Loss)
Percentage of Gross Profit to Sales 9.1% 9.9% 10.7%

Firm A is earning a net profit of `1,00,000 in spite of the low percentage of gross profit. This is
because of less expenses of management. On the hand, C is suffering a loss of `40,000 in spite of the
highest percentage of gross profit. This is because of the highest figure of management expenses. Firms B
and C should try to curtail the expenses of management and increase the inventory turnover ratio to make an
improvement in their performance.

To conclude, performance of firm A is better than the performance of firms B & C.

Question 56: Two companies in the same industry show the following data as adopted from the annual
financial statements:

X Y
`(Lakhs) `(Lakhs)
Total Sales Income 24.00 25.92
Variable Cost 15.60 17.80
Fixed Costs 4.80 5.00
Capital Employed 12.00 13.00

(a) Compare : (i) Capital turnover , (ii) Profit before tax as % of sales value, (iii) Profit before tax as %
on capital employed, (iv) Gross marginal income as percentage of sales, (v) Break-even sales,
and (vi) margin of safety as % on sales.
(b) Suppose you are the Accountant of Company X, comment on working. Besides information given
in X, do you feel any other interesting information which can be derived from the above?

CA. Parag Gupta Ph.: +91 11 47665555 Paraggupta_ca@yahoo.co.in Costing & O.R.
World’s largest CA Final student’s consultancy group: http://groups.yahoo.com/group/costingbyparaggupta
Miscellaneous Theory Topics - 274 -

SOLUTION (a)
X Y
`(Lakhs) `(Lakhs)
Sales 24.00 25.92
Less: Variable Cost 15.60 17.80
Gross Marginal Income 8.40 8.12
Less: Fixed Cost 4.80 5.00
Profit 3.60 3.12

(i) Capital Turnover

= Sales 24 = 2 times 25.92 = 1.99 times


Capital employed 12 13.00

(ii) % of Profit on Sales

= Profit before tax x 100 3.60 x 100 = 15% 3.12 x 100 = 12%
Sales 24.00 25.92

(iii) of profit on Capital Employed

= Profit before tax x 100 3.60 x 100 = 30% 3.12 x 100 = 24%
Capital employed 12 13

(iv) % of Gross Marginal


Income to Saes

= Gross Marginal Income x 100 8.40 x 100 = 35% 8.12 x 100 = 31.327%
Capital employed 24.00 25.92

(v) Break- even Sales

= Fixed Cost 4.80 = `13.71 5.00 = `15.96


P/V Ratio or 35% Lakhs 31.327% lakhs
Gross Marginal Income %

(vi) Margin of Safety as % on Sales

= Margin of Safety x 100 10.29 x 100 = 42.88% 9.96 x 100 = 38.43%


Sales 24.00 25.92

(b) The company X is better placed as revealed by all six ratios calculated above. Although the quantum
of sales is more in Y, the profit realized is less. Further return on capital employed can be calculated:

= Return x 100 3.60 x 100 = 30% 3.12 x 100 = 24%


Capital employed 12.00 13.00

Question 57: The standard ratios for the industry and the ratios of company X are given. Indicate the
company’s strengths and weaknesses as shown by your analysis.

Industry Company X

CA. Parag Gupta Ph.: +91 11 47665555 Paraggupta_ca@yahoo.co.in Costing & O.R.
World’s largest CA Final student’s consultancy group: http://groups.yahoo.com/group/costingbyparaggupta
Cost Accounting & Management - 275 -

Current Assets/Current Liabilities 2.4 2.67


Sales/Debtors 8.0 10.00
Sales/Stock 9.8 3.33
Sales/Total Assets 2.0 1.43
Net Profit/Sales 3.3% 2.1%
Net Profit/Total Assets 6.6% 3.0%
Net Profit/Net Worth 10.7% 4.8%
Total Debt/Total Assets 63.5% 37.7%

SOLUTION
(i) Current ratio indicates better position as current assets are comparatively higher than current
liabilities of a similar industry. However, the current assets may be proportionately higher due to
excessive stock as had been reflected in ratio (iii).
(ii) Debtors turnover ratio indicates that the industry in general allows 1.5 (12 * 8) months credit to
customers but Company X allows 1.2 (12 * 10) month credit to customers. This indicated
marginally controlled credit facilities. This shows effective credit policy and collection policy
though there is a scope for development of potential customers with further review of credit policy.
(iii) Stock turnover ratio indicates that stock is alarmingly high as the industry’s norm J.22 months
(12*9.8) sales as against 3.6 months (12 * 3.33) sales of Company. This has not been properly
reflected in current ratio as the current liabilities might have not been also equally high. But
positively the stock is abnormally high.
(iv) Asset turnover ratio indicates that assets are comparatively higher to its turnover indicating
overstocking or under-utilisation of fixed assets. This ratio further indicates over-stocking.
(v) Net profit ratio is lower as compared to standard ratio. This indicates higher cost of production
and less earnings before interest and tax. This requires either increase in sales realization or
reduction in the cost of manufacture to ensure a reasonable return on investment.
(vi) Net profit / worth ratio indicates that the capital structure of the X Company is having very low
debt/ equity ratio. The earning capacity and earning per share is also very low.

(vii) Total Debt/ Total Asset Turnover ratio is lower for X Company which indicates low capita gearing.
The total assets are substantially high leading to this ratio lower than the standard even though
the current ratio is high.

On the above basis it can be said that stock is very high though there is good control over debtors and credit
control policy. There is need for increasing the operational efficiency of production. Debts can be increased.
The company is over capitalized due to high stock and fixed assets. The company should make attempt to
reduce operating expenses to increase the net profit ratio. The company’s performance is not satisfactory as
compared to standard of the industry and there exists scope for improvements.

Direct product profitability (DPP)

As traditional absorption costing, which normally uses labour hours as a basis for absorption, is rarely suitable
for service and retail organisations other methods had to be devised. One relatively new way of spreading
overheads in retail organisations, which is used in the grocery trade in particular, is direct product profitability
(DPP). DPP started in the USA in the 1960s at General Electric, and was then taken up and used by Proctor
and Gamble in the 1980s. In 1985 the Food Marketing Institute in the USA laid down a standard approach to
the system and two years later DPP was taken up by the Institute of Grocery Distribution in the U.K. The
system described below was introduced in the late 1980s and has since undergone transformation as activity
based costing.
In recent years DPP has developed considerably in parallel with activity-based costing. DPP has become
much more sophisticated and is now very similar to activity-based costing. One of the reasons for its
development during the 1990s has been the development of EPOS and EFTPOS (electronic point of sale and
electronic funds transfer) systems that have enabled access to the detailed data needed for direct product
cost and profitability calculations.
Benefits of DPP
 Better cost analysis
 Better pricing decisions

CA. Parag Gupta Ph.: +91 11 47665555 Paraggupta_ca@yahoo.co.in Costing & O.R.
World’s largest CA Final student’s consultancy group: http://groups.yahoo.com/group/costingbyparaggupta
Miscellaneous Theory Topics - 276 -

 Better management of stores and warehouse space


 The rationalisation of product ranges.

Direct product profitability statement : Retail organisations traditionally deducted the bought- in cost of
goods from the selling price to give a gross margin. The gross margin is useless measure for controlling the
costs of the organisation itself or making decisions about the profitability of the different products. This is
because none of the costs generated by the retail organisation itself are included in its calculation. For
example, it does not include the storage costs of the different goods and these costs vary considerably from
one good to another. A method was needed which related the indirect costs to the goods according to the
way the goods used or created these costs.
Indirect costs, for DPP may be analysed into basic cost categories as follows:
(i) Overhead cost : This is incurred through an activity that is not directly linked to a particular product.
(ii) Volume related cost : The cost is incurred in relation to the space occupied by products.
This includes storage and transport costs.
(iii) Product batch cost : This cost is often a time based cost. If product items (that is a number of identical
products which are handled together as a batch) are stocked on shelves a labour time cost is incurred.
(iv) Inventory financing costs : This is the cost of tying up money in stock and is the cost of the product
multiplied by interest rate per day or per weak.
Customer profitability analysis : In many organisations it is just as important to cost customers as it is to cost
products. Different customers or groups of customers differ in their profitability. This is a relatively new
technique that ABC makes possible because it creates cost pools for activities. Customers use some
activities but not all, and different groups of customers have different ‘activity profiles’.
Service organisations, such as a bank or a hotel, in particular need to cost customers. A bank’s activities for a
customer will include the following types of activities:
 Withdrawal of cash
 Unauthorised overdraft
 Request for a statement
 Stopping a cheque
 Returning a cheque because of insufficient funds.
Different customers or categories of customers will each use different amounts of these activities and so
customer profitability profiles can be built up, and customers can be charged according to the cost to serve
them. A hotel may have activities that are provided for specific types of customers, such as well laid-out
gardens, a swimming pool and a bar. Older guests may appreciate and use the garden, families use the
swimming pool and business guests use the bar. If the activities are charged to the relevant guests a correct
cost per bed occupied can be calculated for this type of category. This will show the relative profitability and
lead to strategies for encouraging the more profitable guests.
Even a manufacturing organisation can benefit from costing its customers. Not all customers cost the same to
serve even if they require the same products. Some customers may be located a long way from the factory
and transport may cost more. Other customers may be disruptive and place rush orders that interrupt
production scheduling and require immediate, special trans- port. Some customers need after sales service
and help with technical matters, etc.
Benefits of customer profitability analysis.
1. It helps the supplier to identify which customers are eroding overall profitability and which customers
are contributing to it.
2. It can help to provide a basis for constructive dialogue between buyer and seller to improve margins.

CA. Parag Gupta Ph.: +91 11 47665555 Paraggupta_ca@yahoo.co.in Costing & O.R.
World’s largest CA Final student’s consultancy group: http://groups.yahoo.com/group/costingbyparaggupta
Linear Programming
Problems

Linear Programming is a mathematical technique for determining the optimal allocation of resources
achieving the specified objective when there are alternative uses of the resources like money, manpower,
materials, machines and other facilities. The objective in resource allocation may be either cost minimization
or profit maximization.

Methods of solving Linear Programming Problems

General LPP Transportation Problem Assignment Problem

Graphical Method Simplex Method

Steps in formulation of LPP:


1) The information stated in the problem is summarized in a table.
2) Identify the variables required & denote them by the symbols.
3) Formulate the objective function to be optimized (maximized or minimized) as a linear function of the
variables.
4) Express the linear constraints mathematically in terms of variables.
5) Add the non-negative constraints from the consideration that –ve values of the variables do not have
any valid significance.

1. A common mistake in LP is to use the gross profit per unit instead of the contribution margin per
unit in a maximizing function for a manufacturing firm. Similarly, it is a mistake to use the full
(absorption) cost of good sold per unit instead of the variable cost per unit in a cost minimizing
function. These mistakes are usually caused by relying on a traditional income statement, which
deducts cost of good sold from sales in determining gross profit.
2. Fixed costs or revenues that do not vary as units are produced or sold are not relevant in an LP
problem.
3. If question has asked just to formulate the problem then Objective function is meant for maximizing
profit (not just contribution) or minimize total cost (not just variable cost), although if question has
asked to find the optimum solution then we will maximize contribution or minimize variable costs &
will adjust fixed costs after solving the whole question

Rules for Graphical Method:


1) Istly, formulate the LPP (steps are defined supra).
2) Graph each of the linear constraints by treating each linear inequation as an equation.
3) Identify the feasible region of the solution i.e. the area which satisfies all of the linear constraints
simultaneously. Ensure that region is bounded. If the region is not bounded, either there are
additional hidden conditions which can be used to bound the region or there is no solution to the
problem.
Note: ICAI’s study material has suggested that the region drawn shall be bounded for both, problems of

CA. Parag Gupta Ph.: +91 11 47665555 Paraggupta_ca@yahoo.co.in Costing & O.R.
World’s largest CA Final student’s consultancy group: http://groups.yahoo.com/group/costingbyparaggupta
Linear Programming - 278 -

maximization as well as minimization, but as per an email clarification from BOS of ICAI (posted on group
on 14th July’09), we don’t need to find hidden conditions.
4) Find the coordinates of the corner points of the feasible region as the optimum solution lies at one of
the corners of the feasible region [Extreme Point Theorem].
5) Compute the value of the objective function at each point obtained in Step 4).
6) The set of values corresponding to the maximum (or minimum) value of the objective function is the
solution of the linear programming problem when it is a maximization (or minimization) problem.

Assign some arbitrary value to objective function (this value can be obtained by taking the LCM of the co-
efficients of x & y in the objective function) & draw the line for the equation representing the objective
function. Common points obtained from drawing a line parallel to this line (touching feasible region along
any edge) which is farthest from origin represents the optimal solution in problems of maximization.

Question 1: Maximize, Z = 3x + 5y
Subject to constraints,
x + 2 y ≤ 20
x + y ≤ 15
y≤ 8
x, y ≥ 0
Solve the LPP under graphical method.

[Ans.: Zmax = 55 when x is 10 and y is 5]


Question 2: Minimize, Z= 2x + 3y
Subject to constraints,
-x + 2y ≥ 4
x+y≥6
x + 3y ≥ 9
x, y ≥ 0.
Solve the LPP under graphical method.

[Ans.: Zmin = 15.33 when x is 8/ 3 and y is 10/ 3 ]


Question 3: XYZ chemical company is producing two products A and B. The processing times are 3
hours and 4 hours per unit for A on operations one and two respectively and 4 hours and 2 hours per unit
for B on operations on one and two respectively. The available time is 36 hours and 28 hours for
operation one and two respectively. The product A can be sold at `3/- profit per unit and B at `8/- profit
per unit. Solve for maximum profit programme by graphical & simplex method.

[Ans.: Profit is `72, no. of units produced for A is nil & no. of units produced for B is 9]
Question 4: A manufacturer can produce two different products, A and B during a given time period.
Each of these products requires four different manufacturing operations: Grinding, Turning, Assembling &
Testing. The manufacturing requirements in hours per unit of product are given below for A & B

A B
Grinding 1 2
Turning 3 1
Assembling 6 3
Testing 5 4

The available capacities of these operations in hours for the given time period are: Grinding, 30; Turning,
60; Assembly, 200; Testing 200. The contribution to profit is `2 for each unit of A and `3 for each unit of
B. The firm can sell all that it produces at the prevailing market price. Formulate the problem as a
linear programming model to maximize profit by graphical method.

[Ans.: Profit is `54 when no. of units produced for A is 18 & B is 6]

CA. Parag Gupta Ph.: +91 11 47665555 Paraggupta_ca@yahoo.co.in Costing & O.R.
World’s largest CA Final student’s consultancy group: http://groups.yahoo.com/group/costingbyparaggupta
Operations Research - 279 -

Question 5: A firm makes two products X and Y, and has a total production capacity of 16 tonnes per day. X
and Y are requiring the same production capacity. The firm has a permanent contract to supply at least 3
tonnes of X and 6 tonnes of Y per day to another company. Each tonne of X require 14 machine hours of
production time and each tonne of Y requires 20 machine hours of production time. The daily maximum
possible number of machine hours is 280. All the firm's output can be sold, and the profit made is `20 per
tonne of X and `25 per tonne of Y.
Required:
Formulate a linear programme to determine the production schedule for maximum profit by using graphical
approach and calculate the optimal product mix and profit. (6 Marks) Nov./10-N.C.
[Ans.: Profit is `366.67 when no. of tonnes produced for X is 20/3 & Y is 28/3]
Question 6 (Diet Problem): A diet for a sick person must contain at least 4000 units of vitamins,
50 units of minerals and 1400 units of calories. Two foods A and B are available at a cost of `4/- and
`3/- per unit respectively. If one unit of A contains 200 units of vitamins, I unit of mineral and 40 calories
and one unit of food B contains 100 units of vitamins, 2 units of minerals and 40 calories. What
combination of food be used to have least cost?

[Ans.: Least cost is `110 when units purchased for A is 5 & B is 30]
Question 7: A company that produces soft drinks has a contract that requires that a minimum of 80 units of
the chemical A and 60 units of the chemical B into each bottle of the drink. The chemicals are available in a
prepared mix from two different suppliers. Supplier X 1 has a mix of 4 units of A & 2 units of B that costs `10,
and supplier X 2 has a mix of 1 unit of A and 1 unit of B that costs `4. How many mixes from company X 1 and
company X 2 should the company purchase to honour contract requirement & yet minimize cost? Solve by
both graphical as well as Simplex Method.

[Ans.: Cost will be `260 when co. purchases 10 mixes from X 1 & 40 mixes from X 2 ]
[Note: In minimization problems, ICAI’s Study material has suggested finding hidden conditions to bound the
region, but as per an email clarification from ICAI (posted on group on 14th July’09), we don’t need to find
hidden conditions]
Question 8: A company manufactures two products A & B, involving three departments-Machining,
Fabrication, & Assembly. The process time, profit/unit and total capacity of each department is given in the
following table:

Machining Fabrication Assembly Profit


(Hours) (Hours) (Hours) (`)
A 1 5 3 80
B 2 4 1 100
Capacity 720 1,800 900

Set-up LPP to maximize profit. What will be the product mix at maximum profit level? (9 Marks) May/05

[Ans.: x = 120 units & y = 300 units and the maximum profit is `39600]
Question 9: The budgeted data relating to two products manufactured by a Co. for a month are as under:

Product A Product B
Selling price 300 200
Variable manufacturing cost 160 60
Sales commission 60 40

Each unit of product incurs costs in the company’s two departments P and Q. The total capacity
available for the month under review is budgeted to be 1,400 hours in department P and 2,000 hours in
department Q. The capacity costs amount to `14,000 and `20,000 respectively per month for P and Q
irrespective of the level of usage made of it. The number of hours required in each of these
departments to complete one unit of output is as under:

CA. Parag Gupta Ph.: +91 11 47665555 Paraggupta_ca@yahoo.co.in Costing & O.R.
World’s largest CA Final student’s consultancy group: http://groups.yahoo.com/group/costingbyparaggupta
Linear Programming - 280 -

A B
Department P 2 4
Department Q 5 4

The maximum output which the company can sell in the month is restricted to 400 units of either of the
products. You are required to formulate the Linear Programming (LP) model and solve it graphically to
determine the optimal product mix and profit. (8 Marks) May/04

[Ans.: Optimal Profit `7000]


Question 10: A company produces two types of belts; X and Y. Profits on these types are `2 & `1.5 each
belt respectively. A belt of type X requires twice as much time as a belt of type Y. The company can
produce at the most 1000 belts of type Y per day. Materal for 800 belts only per day is available. At the
most 400 buckles for belts of type X & 700 for those of type Y are available per day. How many belts of each
type should the company produce so as to maximize the profit? (7 Marks) ICWA June/07-[Adapted]

[Hint: Let time taken to make one belt of type Y is t, therefore total time available in one day is 1000t. Now we
know, type x requires twice as much time as a belt of type y therefore total time for producing both these belts
is 2tx + yt. Therefore, 2x+y ≤ 1000]

[Ans.: 200 belts of type X and 600 belts of type Y]


Question 11: A manufacturer of patent medicines is preparing a production plan on medicines, A & B. There
are sufficient raw materials available to make 20000 bottles of A and 40000 bottles of B, but there are only
45000 bottles into which medicines can be put. Further, it takes 3 hours to prepare enough material to fill
1000 bottles of A, it takes 1 hour to prepare enough material to fill 1000 bottles of B & there are 66
hours available for this operation. The profit is `8 per bottle for A & `7 per bottle for B. How should the
manufacturer schedule his production in order to maximize his profit?

[Hint: Equation for bold line will be 3x/1000 + y/1000 ≤ 66]

[Ans.: 10500 bottles of A and 34500 bottles of B, max. profit `325500]


Question 12: In a chemical industry two products A and B are made involving two operations. The production
of B also results in a by-product C. The product A can be sold at a profit of `3 per unit and B at a profit of `8
per unit. The by-product C has a profit of `2 per unit. Forecasts show that up to 5 units of C can be sold. The
company gets 3 units of C for each unit of B produced. The manufacturing times are 3 h per unit for A on
each of the operation one & two and 4h and 5h per unit for B on operation one and two respectively. Because
the product C results from producing B, no time is used in producing C. The available times are 18h and 21h
of operation one and two respectively. The company desires to know that how much A and B should be
produced keeping C in mind to make the higher profit. Formulate LP model for this problem.

[Ans.: Maximize Z = 3x 1 + 8x 2 + 2 x 3
Subject to constraints
3x 1 + 4x 2 ≤ 18
3x 1 + 5x 2 ≤ 21
x 3 ≤ 5, x 3 = 3x 2
x 1 , x 2 , x 3 ≥ 0] (10 Marks) May/01
[Hint: Since the ratios of the number of units produced between B and C is 1:3, therefore x 2 = 1/ 3 x 3 ]
Question 13: Distinguish between a slack variable & an artificial slack variable in linear programming.
(3 Marks) May/03
Slack variable: In order to convert every constraint of the type ‘less than equal to’ in a LP problem into an
equality constraint, so that solution of the problem can be arrived, we add a variable to each such
constraint. The variable so added in each constraint is known as slack variable. Slack Variables represent idle
or unused resources. The contribution per unit of a slack variable is always taken as zero in objective
function. A slack variable is always non negative.
Constraint 3x + 2y can be written as 3x + 2y + s 1 here s 1 is a slack variable and is +ve.

Artificial variable: In order to convert constraints of the type ‘greater than equal to’ equality for finding the
solution of the L. P. problem, we first subtract a surplus variable and then add a variable. This variable is also
CA. Parag Gupta Ph.: +91 11 47665555 Paraggupta_ca@yahoo.co.in Costing & O.R.
World’s largest CA Final student’s consultancy group: http://groups.yahoo.com/group/costingbyparaggupta
Operations Research - 281 -

added in the constraints of the type ‘equal to’ to start with the initial feasible solution. The variable added in
the constraints as explained above is known as artificial variable. Artificial variable is a fictitious variable
and cannot have any physical or economic meaning. It is intentionally introduced to form an initial solution for
further iterations. It has an infinitely large cost coefficient. Artificial variables are always positive.
Constraints 3x + 4y ≥ 50 & 2x + 6y = 40 can be written as 3x + 4y - s 1 + A 1  50 & 2x + 6y + A 2 = 40;
Here s 1 is a surplus variable and A 1 & A 2 are artificial variables. s 1 , A 1 & A 2 are +ve.

Rules for Simplex Method:


1) Istly, formulate the LPP (steps are defined supra).
2) Add Slack Variable. (The ≤ type inequality can be transformed into equalities by addition of slack
variables.)
3) Deduct Surplus Variable & Add Artificial Slack Variable. (This is done to convert ≥ type inequalities
into equalities.)
[Similar to Slack Variable, Surplus Variable is subtracted to convert inequality into equality but as it
has -1 as its coefficient it cannot be directly used as basic variable. To specify a basic variable an
artificial variable is generally added with an infinitely large cost coefficient (M)]

For Initial Simplex tableau:


3.1 Setup the Simplex tableau. Coefficient values from
Profit/Cost per unit constraint equations

Profit/Cost Cj 0 0 M
Fixed Per unit Quantity Replacement
Ratio (C B ) Basic X1 (b i ) Ratio
X2 S1 S2 A1
Variables
0 s1 1 0

M A1 0 1

Zj 0 M
Net Evaluation Row (C j -Z j ) 0 0
Unit Matrix
Where, s 1 is slack variable, s 2 is surplus variable & A 1 is Artificial Slack Variable.

3.2 In the first row i.e. C j , we are supposed to write coefficients of variables in the objective function.
3.3 Basic Variables (a.k.a. Program Variables): It contains artificial slack variables (A 1 , A 2 , A 3 , etc.) &
slack variables (s 1 , s 2 , s 3 , etc.). It never contains surplus variables (i.e. negative slack variables).
3.4 Profit/Cost per unit (C B ): Coefficients of basic variables in objective function.
3.5 Write unit matrix as coefficients of basic variables.
3.6 In Z j row, below basic variables, copy Profit/Cost per unit & write zeroes in Net Evaluation Row
(NER).
Step 3.6 is basically part of optimality test but has been deliberately written earlier just to do questions
easily.

3.7 Fill the table below non-basic variables by coefficients values from constraint equations.
3.8 Quantity column (b i ) (a.k.a. solution values): These represent constraint values written in
constraint equations.
3.9 Optimality Test:
a) NER: Add the products of Profit/Cost per unit (C B ) with respective column coefficients & write it in
Z j row i.e., Z j =∑C B .a ij , where a ij are the matrix element in the ith row and jth column. Now
subtract it from objective function’s coefficient of respective column (C j ) i.e. C j -Z j
b) For maximization problems: Check that whether there is any +ve NER, it indicates the magnitude
of opportunity cost of not including 1 unit of respective column variables (in other words, it
represents the net profit which would result from introducing one unit of variable to the product).
For minimization problems: Check that whether there is any -ve NER.
[Note: If all values of NER are other than positive (or other than negative) in case of maximization
(or minimization) problem the calculated simplex table is optimal one & no further iterations are
required, otherwise step 3.10 will be performed.]

CA. Parag Gupta Ph.: +91 11 47665555 Paraggupta_ca@yahoo.co.in Costing & O.R.
World’s largest CA Final student’s consultancy group: http://groups.yahoo.com/group/costingbyparaggupta
Linear Programming - 282 -

Deriving a Revised Tableau for Improved/Optimal Solution:

3.10 Selection of entering variable: For maximization problem - The column with highest +ve NER is
key column (In case of tie choose arbitrarily). For minimization problem - The column with
highest -ve NER is key column (In case of tie choose arbitrarily). The variable heading this
column is entering variable.
3.11 Selection of leaving variable: Divide Quantity (b i ) by corresponding elements of key column &
write it in Replacement Ratio column. The row having least non-negative replacement ratio is
key row, & the current variable for this row is leaving variable. The element at the intersection of
key row & the key column is known as pivot/key element & is encircled.
3.12 Fixed ratio: This is calculated for all rows (variables) other than Key row.
Fixed ratio = row element in key column ÷ key no.
3.13 Setup a new updated simplex table.
a. Basic Variables:
For key row – Leaving variable will be replaced by entering variable.
For Non key rows - Variable will remain same as that of preceding simplex table.
b. Profit/Cost per unit (C B ): Coefficients of basic variables in objective function.
c. Write unit matrix as coefficients of basic variables.
d. In Z j row, below basic variables, copy Profit/Cost per unit & write zeroes in Net Evaluation Row
(NER).
e. Key Row [for non-basic variable column & quantity column (b i )]: Divide all the nos. in the key row
by the key no.
f. Non Key Rows [for non-basic variable column & quantity column (b i )]:
Old row no. – (Corresponding no. in key row × Corresponding fixed ratio)
g. Perform Optimality test (Same as Step 3.9)
 Marginal Value of Resource (a.k.a. shadow price or opportunity cost) is the value of NER under
slack variables (s 1 , s 2 , s 3 , etc.). From the optimal (or other) solution profit gets reduced (or cost
gets increased) by the amount this value is multiplied units of no. of units of such slack
variables introduced.
 Fractions of simplex table must be retained & should not be decimalized.

Question 14: The final simplex table for the problem is given below:
Maximize, Z = 3x1 + 4x2 + x3
Subject to, x1 + 2x2 + 3x3  90 (constraint for operation 1)
2x1 + x2 + x3  60 (constraint for operation 2)
3x1 + x2 + 2x3  80 (constraint for operation 3)
x1,x2,x3 ≥ 0

3 4 1 0 0 0
Programme Profit Quantity x1 x2 x3 s1 s2 s3
x2 4 40 0 1 10/6 4/6 -1/3 0
x1 3 10 1 0 -1/3 -1/3 2/3 0
s3 0 10 0 0 8/6 8/6 -10/6 1
(Cj - Zj) 0 0 -28/6 -10/6 - 2/3 0

Find the solution, maximum profit, idle capacity and the loss of total contribution of every one unit reduced
from the right hand side of the constraints. Nov./88

[Ans.: x 1 = 10; x 2 = 40; x 3 = 0; Maximum Z = 190; Idle Capacity = 10 hours in Operation 3; Loss of Total
Contribution = `5/3 per hour and `2/3 per hour in Operation 1 & 2 respectively.]
Question 15: Three grades of coal A, B and C contains phosphorus and ash as impurities. In a
particular industrial process, fuel up to 100 ton (maximum) is required which could contain ash not more than
3% and phosphorus not more than 0.03%. It is desired to maximize the profit while satisfying these
conditions. There is an unlimited supply of each grade. The percentage of impurities and the profits of each
grades are as follows:

CA. Parag Gupta Ph.: +91 11 47665555 Paraggupta_ca@yahoo.co.in Costing & O.R.
World’s largest CA Final student’s consultancy group: http://groups.yahoo.com/group/costingbyparaggupta
Operations Research - 283 -

Coal Phosphorus Ash (%) Profit in ` (per ton)


(%)
A .02 3.0 12.00
B .04 2.0 15.00
C .03 5.0 14.00

You are required to formulate the Linear Programming (LP) model to solve it by using simplex method to
determine optimal product mix and profit. (11 Marks) Nov./05

[Ans.: The optimal solution is X 1 = 40, X 2 = 40 & X 3 = 20 with maximum Z = 1360]

Question 16: A gear manufacturing company makes two types of gears – A and B. Both gears are processed
on 3 machines, Hobbing M/c, Shaping M/c and Grinding M/c. The time required by each gear and total time
available per week on each M/c is as follows:
Gear (A) Gear (B) Available
Machine (Hours) (Hours) Hours
Hobbing M/c 3 3 36
Shaping M/c 5 2 60
Grinding M/c 2 6 60
Other data:
Selling price (`) 820 960
Variable cost (`) 780 900
Determine the optimum production plan and the maximum contribution for the next week by simplex method.
The initial table is given below:

Qty\Cj 40 60 0 0 0
Cj Variable x1 x2 x3 x4 x5
0 x3 36 3 3 1 0 0
0 x4 60 5 2 0 1 0
0 x5 60 2 6 0 0 1

[Ans.: Optimum Z = 660 with X 1 = 3 and X 2 = 9] (7 Marks)


May/07
Question 17: Solve by Simplex Method:
Minimize, Z = 20x 1 +10 x 2
Subject to, x 1 + x 2 ≥ 10
3x 1 + 2x 2 ≥ 24
x 1 ,x 2 ≥ 0 Nov./87
[Ans.: Zmin = 120 when x 1 is 0 and x 2 is 12]
Question 18: The following is a linear programming problem. You are required to set up the initial simplex
tableau. (P lease do not attempt further iterations or solution) :
Maximise
100x 1 + 80x 2 Note
Subject to
3x 1 + 5x 2 ≤ 150
x 2 ≤ 20
8x 1 + 5x 2 ≤ 300
x 1 + x 2 ≥ 25
x1, x2 ≥ 0 (6 Marks) Nov./09-N.C.
[Ans.:
Qty\Cj 100 80 0 0 0 0 -M
Cj Variable x1 x2 s1 s2 s3 s4 A1
0 s1 150 3 5 1 0 0 0 0
0 s2 20 0 1 0 1 0 0 0
0 s3 300 8 5 0 0 1 0 0
-M A1 25 1 1 0 0 0 -1 1]

CA. Parag Gupta Ph.: +91 11 47665555 Paraggupta_ca@yahoo.co.in Costing & O.R.
World’s largest CA Final student’s consultancy group: http://groups.yahoo.com/group/costingbyparaggupta
Linear Programming - 284 -

[Note: In the above question “100x 1 + 80x 2 ” was misprinted as “100x 1 = 80x 2 ” in question paper. This
mistake was corrected by ICAI in their suggested answers, so I am using correct word itself]
Question 19: You are given the following linear program. Introduce appropriate variables and restate the
problem to set up the simplex tableau. (Do not attempt further solution.)
Maximise:
8x 1 + 4x 2 – 3x 3 + 10x 4
s.t.
2x 1 – x 2 + x 3 + 2x 4 ≥ 40
3x 1 – x 2 + x 4 ≤ 90
2x 1 + x 2 + x 4 = 60
x1, x2, x3, x4 ≥ 0 (5 Marks) Nov./10-O.C.

[Ans.:
Qty\Cj 8 4 -3 10 0 -M 0 -M
Cj Variable x1 x2 x3 x4 s1 A1 s2 A2
-M A1 40 2 -1 1 2 -1 1 0 0
0 s2 90 3 -1 0 1 0 0 1 0
-M A2 60 2 1 0 1 0 0 0 1]

Question 20: A company produces two products, x 1 and x 2 with respective unit contributions of `8 and `6.
Each product passes through machining operations in two machining centres, MI and MII, whose capacities
are limited to 60 and 48 hours respectively with corresponding slack variables s 1 and s 2 .
The following table gives the values for an method for an interaction under the simplex maximizing the
contribution:

x1 x2 s1 s2
Basic Variables
x1 1 0 1/3 -1/6 (MI constraint)
x2 0 1 -1/6 1/3 (MI constraint)

You are required to:


(i) Evaluate if this iteration represents the optimal solution.
(ii) Find out what will be the optimal contribution. (7 Marks) May/10-O.C.

[Ans.: (i) Yes (ii) Optimal contribution will be `132 (60×5/3 + 48×⅔)]
Additional Question (iii): Find out quantity of output of 2 products at optimal contribution.
[Ans.: Quantities of x 1 and x 2 will be 12 units & 6 units respectively]
[Hint: While formulating the problem, assume coefficients as a & b for variables x 1 & x 2 in 1st constraint and
coefficients as c & d for variables x 1 & x 2 in 2nd constraint. Now solve the simplex table & compare 3rd table of
your simplex table from the abovementioned simplex table to arrive at values of a, b, c & d. Use this value of
a, b, c & d and place them in 3rd table (quantity column) to arrive at values of quantities.]

Miscellaneous Questions

Question 21 (Transportation Problem): The following matrix gives the unit cost of transporting the product
from production plant P 1 , P 2 & P 3 to destinations D 1 , D 2 & D 3 . Plants P 1 , P 2 & P 3 have a maximum
production of 65, 24 and 111 respectively and destinations D 1 , D 2 & D 3 must receive at least 60, 65 & 75
units respectively:

To D1 D2 D3 Supply

From
P1 400 600 800 65
P2 1,000 1,200 1,400 24
P3 500 900 700 111
Demand 60 65 75 200

CA. Parag Gupta Ph.: +91 11 47665555 Paraggupta_ca@yahoo.co.in Costing & O.R.
World’s largest CA Final student’s consultancy group: http://groups.yahoo.com/group/costingbyparaggupta
Operations Research - 285 -

You are required to formulate the above as a linear programming problem.


(Only formulation is needed. Please do not solve). (9 Marks) Nov./08-N.C.

[Ans.: Minimize, Z = 400x 11 + 600x 12 + 800x 13 + 1000x 21 + 1200x 22 + 1400x 23 + 500x 31 + 900x 32 + 700x 33
Subject to, x 11 + x 12 + x 13 ≤ 65
x 21 + x 22 + x 23 ≤ 24
x 31 + x 32 + x 33 ≤ 111
x 11 + x 21 + x 31 ≥ 60
x 12 + x 22 + x 32 ≥ 65
x 13 + x 23 + x 33 ≥ 75
where, xij ≥ 0 (i,j = 1 to 3)]
[Note: If bolded words are absent in question, we will use = in Demand & supply constraints in spite of ≥ & ≤]
Question 22 (Transportation Problem): Transport Ltd. provides tourist vehicles of 3 types- 20-seater vans,
8-seater big cars & 5-seater small cars. These seating capacities are excluding the drivers. The company has
4 vehicles of the 20-seater van type, 10 vehicles of the eight-seater big car types 20 vehicles of the 5-seater
small car types. These vehicles have to be used to transport employees of their client company from their
residences to their offices and back. All the residences are in the same housing colony. The offices are at two
different places, one is the Head Office and the other is the Branch. Each vehicle plies only one round trip per
day, if residence to office in the morning and office to residence in the evening. Each day, 180 officials need
to be transported in Route I (from residence to Head Office & back) and 40 officials need to be transported in
Route II (from Residence to Branch office & back). The cost per round trip for each type of vehicle along each
route is given below.
Figs-`/round trip
20-seater vans 8-seater big cars 5-seater small cars
Route I-
600 400 300
Residence-Head Office & Back
Route II-
500 300 200
Residence-Branch Office & Back

You are required to formulate the information as LPP with objective of minimizing the total cost of hiring
vehicles for the client company, subject to the constraints mentioned above. (10 Marks) May/08

[Hint: Minimize, Z=600x 1 + 400x 2 + 300X 3 + 500y 1 + 300y 2 + 200y 3


Subject to, x1 + y1 ≤ 4
x 2 + y 2 ≤ 10
x 3 + y 3 ≤ 20
20x 1 + 8x 2 + 5x 3 = 180
20y 1 + 8y 2 + 5y 3 = 40
x 1 , x 2 , x 3 , y 1 , y 2 , y 3 ≥ 0]
Question 23 (Trim Problem): The Fine Paper Company produces rolls of paper used in cash registers. Each
roll of paper is 500 ft. in length and can be produced in widths of 1,2,3 and 5 inch. The company’s production
process results in 500 feet rolls that are 12 inches in width. Thus company must cut its 12 inch roll to the
desired width. It has six basic cutting alternatives as follows:

Cutting No. of Rolls Waste


Alternative (inches)
1” 2” 3” 5”
1 6 3 0 0 0
2 0 3 2 0 0
3 1 1 1 1 1
4 0 0 2 1 1
5 0 4 1 0 1
6 4 2 1 0 1

The maximum demand requirements for the four rolls are as follows:

CA. Parag Gupta Ph.: +91 11 47665555 Paraggupta_ca@yahoo.co.in Costing & O.R.
World’s largest CA Final student’s consultancy group: http://groups.yahoo.com/group/costingbyparaggupta
Linear Programming - 286 -

Roll Width (inches) Demand Requirements (Rolls)


1 3000
2 2000
3 1500
5 1000
The company wishes to minimize the waste generated by its production meeting its demand requirements.
Formulate LP model.

[Ans.: Minimize, x 3 + x 4 + x 5 + x 6
Subject to, 6x 1 + x 3 + 4x 6 3000
3x 1 + 3x 2 + x 3 + 4x 5 + 2x 6 2000
2x 2 + x 3 + 2x 4 + x 5 + x 6 1500
x 3 + x 4  1000
x j ≥ 0; where j = 1 to 6]
Question 24: The Delhi Florist Company is planning to make up floral arrangements for the upcoming
festival. The company has available the following supply of flowers at the costs shown:

Type Number Available Cost per flower


Red Roses 800 `0.20
Gardenias 456 `0.25
Carnations 4000 `0.15
White Roses 920 `0.20
Yellow Roses 422 `0.22

These flowers can be used in any of the four popular arrangements whose makeup and selling prices are as
follows:

Arrangement Requirement Selling Price


Economy 4 red roses `6
2 gardenias
8 carnations
May time 8 white roses `8
5 gardenias
10 carnations
4 yellow roses
Spring colour 9 red roses `10
10 carnations
9 white roses
6 yellow roses
Deluxe rose 12 red roses `12
12 white roses
12 yellow roses
Formulate a LPP which allows the florist company to determine how many units of each arrangement should
be made up in order to maximize profits assuming all arrangements are sold. Nov./90

[Ans.: Maximize, 3.5x 1 + 2.77x 2 + 3.58x 3 + 4.56x 4


Subject to, 4x 1 + 9x 3 + 12x 4 800
2x 1 + 5x 2 456
8x 1 + 10x 2 + 10x 3 4000
8x 2 + 9x 3 + 12x 4  920
4x 2 + 6x 3 + 12x 4  422
x j ≥ 0; where j = 1 to 4]
Question 25 (Blending Problems): A refinery makes 3 grades of petrol (A, B, C) from 3 crude oils (d, e, f)
Crude f can be used in any grade but the others satisfy the following specifications.

CA. Parag Gupta Ph.: +91 11 47665555 Paraggupta_ca@yahoo.co.in Costing & O.R.
World’s largest CA Final student’s consultancy group: http://groups.yahoo.com/group/costingbyparaggupta
Operations Research - 287 -

Grade Specifications Selling Price per liter


A Not less than 50% crude d 8.0
Not more than 25% crude e
B Not less than 25% crude d 6.5
Not more than 50% crude e
C No specifications 5.5

There are capacity limitations on the amount of the three crude elements that can be used;

Crude Capacity Price per litre


D 500 9.5
E 500 5.5
F 300 6.5

It is required to produce the maximum profit.

[Hint:
D E F Sell Price per liter
A X A1 X A2 X A3 8
B X B1 X B2 X B3 6.5
C X C1 X C2 X C3 5.5
Cost price per liter 9.5 5.5 6.5
Capacity 500 500 300

Z = 8(X A1 +X A2 +X A3 ) – (9.5X A1 +5.5X A2 +6.5X A3 ) + 6.5(X B1 +X B2 +X B3 ) – (9.5X B1 +5.5X B2 +6.5X B3 ) +


5.5(X C1 +X C2 +X C3 ) – (9.5X C1 +5.5X C2 +6.5X C3 )
i.e. Z = -1.5 X A1 + 2.5 X A2 + 1.5 X A3 - 3.0 X B1 + 1.0 X B2 – 4.0 X C1 - 1.0 X C3
Subject to constraints,
X A1 ≥ 0.50 (X A1 +X A2 +X A3 )
X A2 ≤ 0.25 (X A1 +X A2 +X A3 )
X B1 ≥ 0.25 (X B1 +X B2 +X B3 )
X B2 ≤ 0.50 (X B1 +X B2 +X B3 )
X A1 +X B1 +X C1 ≤ 500
X A2 +X B2 +X C2 ≤ 500
X A3 +X B3 +X C3 ≤ 300
X ij ≥0 where i= A,B & C & j = 1,2 & 3]

Question 26: A manufacturer produces three products Y 1 , Y 2 , Y 3 from three raw materials X 1 , X 2 and X 3 .
The cost of raw materials X 1 , X 2 and X 3 is `30, `50 and `120 per kg respectively and they are available in a
limited quantity viz. 20 kgs of X 1 , 15 kgs of X 2 and 10 kgs of X 3 . The selling price of Y 1 , Y 2 and Y 3 is `90,
`100 and `120 per kg respectively. In order to produce 1 kg of Y 1 ½ kg of X 1 , ¼ kg of X 2 and ¼ kg of X 3 are
required. Similarly to produce 1 kg of Y 2 , 3/7 kg X, 2/7 kg of X 2 and 2/7 kg of X 3 and to produce 1 kg of Y 3 , 2/3
kg of X 2 and 1/3 kg of X 3 will be required.
Formulate the linear programming problem to maximize the profit. (10 Marks) Nov./00

[Ans.: Maximize, Z = 32.50y 1 + 38.57y 2 + 46.67y 3


Subject to 7y 1 + 6y 2 ≤ 280
21y 1 + 24y 2 + 56y 3 ≤ 1260
21y 1 + 24y2 + 28y 3 ≤ 840
y 1 , y 2 , y 3 ≥ 0]
Question 27 (Planning-production & financing) Consider a company that must produce two products over
a production period of three months of duration. The company can pay for materials and labour from two
sources:
The firm faces three decisions:
(1) How many units should it produce of Product 1?
(2) How many units should it produce of Product 2?
(3) How much money should it borrow to support the production of the two products?
In making these decisions, the firm wishes to maximize the profit contribution subject to the conditions stated

CA. Parag Gupta Ph.: +91 11 47665555 Paraggupta_ca@yahoo.co.in Costing & O.R.
World’s largest CA Final student’s consultancy group: http://groups.yahoo.com/group/costingbyparaggupta
Linear Programming - 288 -

below:
(i) Since the company’s products are enjoying a seller’s market, it can sell as many units as it
can produce. The company would therefore like to produce as many units as possible
subject to production capacity and financial constraints. The capacity constrains, together
with cost and price data, are given in Table -1.

Capacity, Price and Cost data


Product Selling Price Cost of Production Requirement Hours per unit in
( Per unit) ( Per unit) Department
A B C
1 14 10 0.5 0.3 0.2
2 11 8 0.3 0.4 0.1
Available hours per production period of three months 500 400 200

(ii) The available company funds during the production period will be `3 lakhs.
(iii) A bank will give loans up to `2 lakhs per production period at an interest rate of 20 percent per
annum provided the company’s acid (quick) test rat io is at least 1 to 1 while the loan is
outstanding. Take simplified acid-test ratio given by
Surplus cash on hand after production + Accounts receivable
Bank Borrowing + Interest accrued thereon

(iv) Also make sure that the needed funds are made available for meeting the production costs.

Formulate the above as a Linear Programming Problem. (20 Marks) Nov./92

[Ans.: Maximize, Z = 4x 1 + 3x 2 – 0.05 x 3


Subject to, 0.5x 1 + 0.3x 2 ≤ 500
0.3x 1 + 0.4x 2 ≤ 400
0.2x 1 + 0.1x 2 ≤ 200
10x 1 + 8x 2 – x 3 ≤ 300000
-4x 1 - 3x 2 + 0.05x 3 ≤ 300000
x 3 ≤ 200000
x 1 , x 2 , x 3 ≥ 0]
[Hint: Quick ratio clearly specifies that sales & interest are on accrual basis & not on cash basis.]
Question 28 (Agriculture Application): An agriculturist has a farm with125 acre. He produces Radish,
Muttar and Potato. Whatever he raises is fully sold in the market. He gets `5 for Radish per kg., `4 for muttar
per kg., & `5 for potato per kg. The average yield is 1,500 kg. of Radish per acre;1,800 kg. of muttar per acre
and 1,200kg. of Potato per acre. To produce each 100 kg. of Radish and muttar and to produce each 80kg. of
potato, a sum of `12.50 has to be used for manure. Labour required for each acre to raise the crop is 6 man
days for Radish and Potato each and 5 man days for Muttar. A total of 500 man days of labour at the rate of
`40 per man day are available.
Formulate this as a LPP model to maximize the Agriculturist’s total profit. (10 Marks) May/97

[Ans: Z = 7072.5x 1 + 6775x 2 + 5572.5x 3


Z constraint in detail, 7500x 1 + 7200x 2 + 6000x 3 - 187.5x 1 - 225x 2 – 187.5x 3 - 240x 1 - 200x 2 - 240x 3
Subject to constraints,
x 1 + x 2 + x 3 ≤ 125
6x 1 + 5x 2 + 6x 3 ≤ 500
x 1 ,x 2 ,x 3 ≥ 0]
Question 29 (Production Runs): An oil refinery can blend three grades of crude oil to produce quality A and
quality B petrol. Two possible blending processes are available. For each production run, the older process
uses 5 units of crude Q, 7 units of crude P and 2 units of crude R and produces 9 units of A and 7 units of B.
The newer process uses 3 units of crude Q, 9 units of crude P and 4 units of crude R to produce 5 units of A
and 9 units of B.
Because of prior contract commitments, the refinery must produce at least 500 units of A and at least 300
units of B for the next month. It has 1,500 units of crude Q, 1,900 units of crude P and 1,000 units of crude R.
For' each unit of A, refinery receives `60 while for each unit of B, it receives `90.
CA. Parag Gupta Ph.: +91 11 47665555 Paraggupta_ca@yahoo.co.in Costing & O.R.
World’s largest CA Final student’s consultancy group: http://groups.yahoo.com/group/costingbyparaggupta
Operations Research - 289 -

Formulate the problem as linear programming model so as to maximize the revenue. (4 Marks) Nov./09-O.C.

[Ans.: Maximize, Z = 60(9x 1 + 5x 2 ) + 90(7x 1 + 9x 2 )


Subject to, 7x 1 + 9x 2 ≤ 1900
5x 1 + 3x 2 ≤ 1500
2x 1 + 4x 2 ≤ 1000
9x 1 + 5x 2 ≥ 500
7x 1 + 9x 2 ≥ 300
x 1 , x 2 ≥ 0]
Question 30: A Computer Company produces three types of models, which are first required to be machined
and then assembled. The time (in hours) required for these operations for each model is given below:
Model Machine Time Assembly Time
P III 20 5
P II 15 4
Celeron 12 3
The total available machine time and assembly time are 1,000 hours and 1,500 hours respectively. The
selling price and other variable costs for three models are:

P III P II Celeron
Selling Price (`) 3,000 5,000 15,000
Labour, Material and
other Variable Costs (`) 2,000 4,000 8,000

The company has taken a loan of `50,000 from a Nationalised Bank, which is required to be repaid on
1.4.2001. In addition, the company has borrowed `1,00,000 from XYZ Cooperative Bank. However, this bank
has given its consent to renew the loan.
The balance sheet of the company as on 31.3.2001 is as follows:

Liabilities ` Assets `
Equity Share Capital 1,00,000 Land 80,000
Capital reserve 20,000 Buildings 50,000
Profit & Loss Account 30,000 Plant & Machinery 1,00,000
Long-term Loan 2,00,000 Furniture etc. 20,000
Loan from XYZ Cooperative 1,00,000 Cash 2,10,000
Bank
Loan from Nationalized 50,000
Bank
Total 5,00,000 Total 5,00,000

The company is required to pay a sum of `15,000 towards the salary. Interest on long-term loan is to be paid
every month@ 18% per annum. Interest on loan from XYZ Cooperative and Nationalised Banks may be taken
as `1,500 per month. The company has already promised to deliver three P III, Two P II and five Celeron type
of computers to M/s. ABC Ltd. next month. The level of operation I the company is subject to the availability of
cash next month.
The Company Manager is willing to know that how many units of each model must be manufactured next
month, so as to maximize the profit.
Formulate a linear programming problem for the above. (20 Marks) May/93 [Adapted] & (10 Marks) May/01

[Hint: Maximize Z = 1000x 1 + 1000x 2 + 7000x 3 – (`15,000 + `3,000 + `1,500)


s.t.c, 20x 1 + 15x 2 + 12x 3 ≤ 1000
5x 1 + 4x 2 + 3x 3 ≤ 1500
2000x 1 + 4000x 2 + 8000x 3 ≤ `140500
Where x 1 ≥ 3 ,x 2 ≥ 2 ,x 3 ≥ 5 ]
[Assumption: Since, loan from cooperative bank will be repaid on 1st April,’01 Interest on loan represents
interest on loan from nationalized bank only.]
Question 31: A firm produces three products A,B & C. Its uses two types of raw materials I & II of which 5000

CA. Parag Gupta Ph.: +91 11 47665555 Paraggupta_ca@yahoo.co.in Costing & O.R.
World’s largest CA Final student’s consultancy group: http://groups.yahoo.com/group/costingbyparaggupta
Linear Programming - 290 -

and 7500 units respectively are available. The raw material requirements per unit of product are given below:

Raw Material Requirement per unit of product


A B C
I 3 4 5
II 5 3 5

The labour time for each unit of product A is twice that of product B and three times that of product C.
The entire labour force of the firm can produce the equivalent of 3000 units of A. The minimum demand of the
three products is 600, 650 and 500 units respectively. Also, the ratios of the number of units must be
equal to 2:3:4. Assuming the profits per unit of A,B & C as `50, 50 and 80 respectively.
Formulate the problem as linear programming model in order to determine the number of units of each
product which will maximize the profit. (10 Marks) Nov./97

[Hint: 1. Let time taken to produce one unit of Product A is t, therefore total time available is 3000t. Now we
know, the labour time for each unit of product A is twice that of product B and three times that of product C,
therefore total time for producing all three products is tx 1 + tx 2 /2+ tx 3 /3 . Therefore, x 1 + x 2 /2+ x 3 /3 ≤ 3000
2. Since the ratios of the number of units produced must be equal to 2:3:4, therefore
1
/ 2 x 1 = 1/ 3 x 2 and 1/ 3 x 2 = 1/ 4 x 3 or 3x 1 = 2x 2 and 4x 2 = 3x 3 ]
Question 32: Renco Foundries is in the process of drawing up a Capital Budget for the next three years. It
has funds to the tune of `100000 which can be allocated across the projects A, B, C, D and E. The net cash
flows associated with an investment of `1 in each project are provided in the following table:

Cash Flow at Time


0 1 2 3
From inv. A -` 1 +` 0.5 +` 1 ` 0
From inv. B `0 -` 1 +` 0.5 +` 1
From inv. C -` 1 +`1.2 `0 `0
From inv. D -`1 `0 `0 +`1.9
From inv. E `0 `0 -` 1 +`1.5

Note: Time 0 = present, Time 1 = 1 year from now, Time 2 = 2 years from now, Time 3 = 3 years from now.
For example, ` 1 invested in investment B requires a ` 1 cash outflow at time 1 and returns ` 0.50 at time 2
and ` 1 at time 3.
To ensure that the firm remains reasonably diversified, the firm will not commit an investment exceeding
`75000 in any project. The firm cannot borrow funds; therefore the cash available for investment at any time is
limited to cash on hand. The firm will earn interest at 8% per annum by parking the uninvested funds in money
market instruments. Assume that the returns from investments can be immediately reinvested. For example,
the positive cash flow received from project C at time 1 can immediately be reinvested in project B.
Required: Formulate an LP that will “Maximize cash on hand at time 3”. (20 Marks) Nov./95

[Ans.: Maximize Z = b + 1.9d + 1.5e + 1.08S 2


Subject to Constraints,
a + c + d + S 0 = 100000
0.5a + 1.2c + 1.08 S 0 = b + S 1
a + 0.5b + 1.08S 1 = e + S 2
a, b, c, d, e ≤ 75000
Where a, b, c, d, e, and S i (i = 0, 1, 2) ≥ 0]
Question 33: Let us assume that you have inherited `1,00,000 from your father-in-law that can be invested in
a combination of only two stock portfolio, with the maximum investment allowed in either portfolio set
at `75,000. The first portfolio has an average rate of return of 10%, whereas the second has 20% In terms
of risk factors associated with these portfolios. The first has a risk rating of 4 (on a scale from 0 to 10),
and the second has 9. Since you wish to maximize your return, you will not accept an average rate of return
below 12% or a risk factor above 6. Hence, you then face the important question. How much should you
invest in each portfolio?
Formulate this as a linear programming problem and solve it by Graphic Method.

[Hint: For finding bounded region apply ‘true false’ technique] (10 Marks) May/99 & Nov./08-RTP-O.C.

CA. Parag Gupta Ph.: +91 11 47665555 Paraggupta_ca@yahoo.co.in Costing & O.R.
World’s largest CA Final student’s consultancy group: http://groups.yahoo.com/group/costingbyparaggupta
Operations Research - 291 -

[Ans.: Co. should invest `60000 in first portfolio and `40000 in second portfolio to achieve the maximum
average rate of return of `14000]
Question 34: A manufacturer of three products tries to follow a policy of producing those which contribute
most to fixed cost and profit. However, there is a policy of recognizing certain minimum sales requirements.
Currently these are:
Product Unit per week
X 20
Y 30
Z 60

There are three producing departments. The product time in hour per unit in each department and the total
times available for each week in each department are:

Time required per product(in hrs.)


Department/Product X Y Z Total hours
1 0.25 0.20 0.15 420
2 0.30 0.40 0.50 1048
3 0.25 0.30 0.25 529

The contribution per unit of product X,Y,Z is `10.50, `9.00 and `8.00 respectively. The company has
scheduled 20 units of X, 30 units of Y and 60 units of Z for production in the following week.
You are required to state:
(a) Whether the present schedule is an optimum one from a profit point of view and if it is not what it
should be;
(b) The recommendations that should be made to the firm about their production facilities (following the
answer to (a) above)

[Hint.: Solve it: Maximize, Z = 10.5x 1 + 9x 2 + 8x 3 + 960


Subject to, 0.25x 1 + 0.20x 2 + 0.15x 3 ≤ 400
0.30x 1 + 0.40x 2 + 0.40x 3 ≤ 1000
0.25x 1 + 0.30x 2 + 0.25x 3 ≤ 500
x 1 , x 2 , x 3 ≥ 0; where X = x 1 + 20, Y = x 2 +30, Z = x 2 + 60]
[Note: In situations where inequalities are like 0 ≤ x ≥ 20, lower bounds shall be introduced i.e. we shall
introduced a new variable like x 1 = x - 20 so computations can be made easy.]
Question 35: The costs and selling prices per unit of two products manufacturing by a company are as
under:
Product A (`) B (`)
Selling Price 500 450
Variable costs
Direct Materials @ 25 per kg 100 100
Direct Labour @ 20 per kg[Note1] 80 40
Painting @ `30 per hour 30 60
Variable Overheads: 190 175
Fixed costs @ `17.50/D.L.Hr 70 35
Total costs 470 410
Profit 30 40
In any month the maximum availability of input is limited to the following:
Direct Materials 480 kgs
Direct Labour hours 400 hours
Painting hours 200 hours
Required:
(i) Formulate a linear programme to determine the production plan which maximizes the profits by
using graphical approach.
(ii) State the optimal product mix and the monthly profit derived from your solution in (i) above.
(iii) If the company can sell the painting time as `40 per hour as a separate service, show what the
modification will be required in the formulation of the linear programming problem. You are
required to re-formulate the problem but not to solve. (11 Marks) Nov./08-O.C.
CA. Parag Gupta Ph.: +91 11 47665555 Paraggupta_ca@yahoo.co.in Costing & O.R.
World’s largest CA Final student’s consultancy group: http://groups.yahoo.com/group/costingbyparaggupta
Linear Programming - 292 -

[Ans.:(i) Maximize, Z (Contribution) = 100 x + 75 y


subject to, 4 x + 4 y ≤ 480
4 x + 2 y ≤ 400
x + 2 y ≤ 200
where, x, y ≥ 0
& Maximum profit is `4000 [`11000(contribution) – `7000(fixed cost [Note 2])] when production of A is 80 units &
production of B is 40 units.
(iii) Maximize, Z (Contribution) = 90 x + 55 y {Painting hours @ 40 per hour is taken in computation}
subject to, 4 x + 4 y ≤ 480
4 x + 2 y ≤ 400
x + 2 y ≤ 200
where, x, y ≥ 0]
[Note: 1. In the above question Direct Material was written twice in question paper [i.e. Direct Labour was also
written as Direct Material]. This mistake was corrected by ICAI in their suggested answers, so I am using
correct word itself i.e. Direct Labour.]
2. Here company’s fixed costs are on allocation basis, so fixed costs (absorbed) on individual products are
not required to be calculated for individual products. We will directly calculate fixed cost as `7000 (i.e. 400
D.L. Hours @ `17.50 per hour).
3. It is advisable to go through part (iii) after going through concepts of Relevant costing]
Question 36: Explain the limitations of linear programming. (4 Marks) May/04 & (5 Marks) Nov./00

Answer: Important limitations of linear programming problems are as follows:


1. Linear Constraints: In some situations it is not possible to express both the objective function and
constraints in linear form. E.g. Setup time is often non-linear constraint and is independent of the
quantity produced.
2. Lack of Synergy emphasis: Joint interactions between some activities leads in comparatively less
resource usage than sum of these quantities resulting from each activity being performed individually.
From Linear programming it is not possible to handle such situations.
3. Fractions: Linear Programming may yield fractional valued answers for the decision variables,
whereas it may happen that only integer values of the variables are logical. Rounding-off the values
obtained may not result into an optimal solution. E.g. No. of units produced.
4. Uncertain Conditions: It is applicable only in static situations & objective function and the constraints
equations should change during the period of study. Furthermore, these coefficients may actually be
random variables, each with an underlying probability distribution for the values. Such problems can’t
be solved using LPP.
5. Heavy calculations: For large problems having many constraints, the computational difficulties are
enormous, even when assistance of large digital computers is available.
6. Single Objective: LPP deals with problems that have a single objective. Real life problems may
involve multiple and even conflicting objectives.
Question 37: Enumerate the industrial applications of linear programming. (4 Marks) May/03

Answer: The industrial applications of linear programming are:


 Product mix problems
 Production scheduling
 Blending problems
 Transportation & distribution problems.

Question 38: What are practical applications of linear programming. (7 Marks) May/07

Answer: Linear programming can be used to find optional solutions under constraints.
In production:
 product mix under capacity constraints to minimize costs/maximize profits along with marginal
costing.
 Inventory management to minimize holding cost, warehousing / transporting from factories to
warehouses etc.
Sensitivity Analysis: By providing a range of feasible solutions to decide on discounts on selling price,
decisions to make or buy.
CA. Parag Gupta Ph.: +91 11 47665555 Paraggupta_ca@yahoo.co.in Costing & O.R.
World’s largest CA Final student’s consultancy group: http://groups.yahoo.com/group/costingbyparaggupta
Operations Research - 293 -

Blending: Optional blending of raw materials under supply constraints.


Finance: Portfolio management, interest/receivables management.
Advertisement mix: In advertising campaign – analogous to production management and product mix.
Assignment of personnel to jobs and resource allocation problems.
However, the validity will depend on the manager’s ability to establish a proper linear relationship among
variables considered.

Least Important Topics

Situation Treatment
Graphical: If for a given question two or more extreme points yields same value of
objective function it is multiple optimal solution.
Multiple Optimal
Simplex: For any optimal solution if NER of a non-basic variable is ZERO, it indicates
Situation
that this variable can become incoming variable & there are more than 1 optimal
solution.
Graphical: If a solution satisfies all constraints and the non-negativity conditions it is
called feasible solution, otherwise it is infeasible solution
Infeasibility
Simplex: If artificial variable persists in the optimal solution table as basic variable with
non-zero quantity it is a infeasible solution.
Graphical: For maximization type of LPP, unboundedness occurs when there is no
constraint on the solution so that one or more of the decision variables can be increased
Unboundedness indefinitely without violating any of the constraints.
Simplex: If all Replacement Ratios are –ve i.e. Outgoing variable cannot be identified, it
is an unbounded solution.
Only Simplex: It occurs if there is a tie in the replacement ratios for determining outgoing
variable; the next tableau would give a degenerate solution. In case of degeneracy, the
Degeneracy
outgoing variable should be selected on an arbitrary basis. The variable not selected as
outgoing variable will bear “0” in the quantity column in the subsequent table.

Dual

Any LPP can be re-formulated into what is known as its dual. We can convert the Minimization Problem into
Maximization Problem & vice versa. This is known as duality.

Method of Deriving Dual:


1) Convert equations in inequalities (e.g. x+y=10 can be written as x+y≥10 & x+y≤10)
2) Convert LPP in Standard form i.e.
i) All variable should be non-negative.
ii) For Maximization Type Problems → All constraints should be of “≤” type.
For Minimization Type Problems → All constraints should be of “≥” type.
iii) If we have m constraints & n variables in primal (initial primal), we must have n constraints & m
variables in Dual. Exception: It will not be so if we have equation in our initial LPP [i.e. we have
processed Step i) & have converted equation in inequations]. In such case we will introduce new
variable for similar variables [introduced by virtue of step i)].

Question 39: Write short notes on the characteristics of the dual problem. (Nov./96)
Ans.:
If Primal Then Dual
(i) Objective is to maximize (i) Objective is to minimize
(ii) Objective is to minimize (ii) Objective is to maximize
(iii) No. of constraints (iii) No. of variables
(iv) No. of variables (iv) No. of constraints
(v) Primal constraints “≤” type (v) Dual constraints “≥” type
(vi) Primal constraints “≥” type (vi) Dual constraints “≤” type
(vii) Primal variable x j unrestricted in sign (vii) Dual constraint j is = type
(viii) Primal constraint i is = type (viii) Dual variable y i is unrestricted in sign
(ix) R.H.S. of constraints (ix) Coefficients of Objective function
CA. Parag Gupta Ph.: +91 11 47665555 Paraggupta_ca@yahoo.co.in Costing & O.R.
World’s largest CA Final student’s consultancy group: http://groups.yahoo.com/group/costingbyparaggupta
Linear Programming - 294 -

(x) Coefficients of Objective function (x) R.H.S. of constraints


(xi) Primal is infeasible (xii) Dual is unbounded
(xii) Dual is unbounded (xii) Primal is infeasible
(xiii) NER of Optimal table of primal (xiii) Quantity column of Optimal table of Dual
(xiv) Quantity column of Optimal table of Primal (xiv) NER of Optimal table of Dual

Question 40: Formulate the dual for the following linear program:
Maximize: 100x 1 + 90x 2 + 40x 3 + 60 x 4
Subject to, 6x 1 + 4x 2 + 8x 3 + 4 x 4 ≤ 140
10x 1 + 10x 2 + 2x 3 + 6x 4 ≤ 120
10x 1 + 12x 2 + 6x 3 + 2x 4 ≤ 50
x 1 , x 2 , x 3 ,x 4 ≥ 0;
(Only formulation is required. Please do not solve.) (6 Marks) June/09-N.C.

[Ans.: Dual will be: Minimize, G = 140a + 120b + 50c


Subject to,
6a + 10b + 10c ≥ 100
4a + 10b + 12c ≥ 90
8a + 2b + 6c ≥ 40
4a + 6b + 2c ≥ 60
& a, b, c ≥ 0
Question 41: XYZ company has three departments - Assembly, painting and packing with the capability of
making three types of almirahs. An almiras of Type 1 requires one hour of assembly, 40 minutes of painting
and 20 minutes of packing time respectively. Similarly, almirah of type II needs 80 minutes of assembly, 20
minutes of painting and one hour of packing time respectively. The last type requires 40 minutes each of
assembly, painting and packing time. The total times available at assembly, painting, and packing
departments are 600 hours, 400 hours, and 800 hours respectively. Determine the number of each type of
almirahs that should be produced in order to maximize the profit. The unit profits for types I, II and III are `40,
80 and 60 respectively.
Suppose the manager is thinking of renting the productive capacities of the three departments to another
almirah manufacturer – Steel Racks Company. Steel Racks is interested in minimizing the rental charges. On
the other hand the XYZ company would like to know the worth of a productive hour to them, in each of the
departments to determine the rental rates. Formulate the problem as a LPP. Explain clearly. May/90

[Ans.: Type I almirah = 0 units; Type II almirah = 200 units; Type III almirah = 500 units; Maximum profit =
`46000; Rental rates per hour for assembly, painting & packing departments are `50, `40 & `Nil respectively.
Minimal total rental charges = `46000]

CA. Parag Gupta Ph.: +91 11 47665555 Paraggupta_ca@yahoo.co.in Costing & O.R.
World’s largest CA Final student’s consultancy group: http://groups.yahoo.com/group/costingbyparaggupta
The Transportation
Problem

Stages in solving transportation problems

Steps for Stage 1 (IBFS)


1) If problem is not balanced, introduce dummy row/column to balance it.
[Note: Balancing means ensuring that the total of availability and requirement is equal. If total is not
equal then we need to make them equal, by introducing dummy row/column.]
2) In case of problems of maximization, Convert profit matrix into loss matrix. This is done by
subtracting each entry in the table from largest no. in the table.
3) Assign M or ∞ to prohibited routes (i.e. routes which are not available) etc.
4) Find Initial Basic Feasible Solution using methods defined below.

[Important Note: 1. Though we can also use step 2 before step 1(i.e. we can also convert profit matrix in loss
matrix before balancing the problem), but, it is recommended to apply the steps in above defined order only.
2. If question is silent, we need to find IBFS using VAM (because it provides least of all values of transportation
cost found by different method.]
Steps in Northwest Corner Method

1. Allocation always starts from upper left hand corner (i.e. North side first row-first column).
Assignment is made in such a way that the resources available are exhausted or demand is fully
satisfied.
2. If the resources available are fully exhausted then we move down to second next row and continue
CA. Parag Gupta Ph.: +91 11 47665555 Paraggupta_ca@yahoo.co.in Costing & O.R.
World’s largest CA Final student’s consultancy group: http://groups.yahoo.com/group/costingbyparaggupta
The Transportation Problem - 296 -

the process till the whole demand is exhausted.


3. If the first allocation completely satisfies the demand then we move to the next column of the same
row, and continue the allocation process till all availability and requirements are met.
4. The procedure is repeated till all row availability and column requirements are met.

Steps in Least Cost Method

1. Allocation always starts from the cell whose transportation cost per unit is least.
2. The lowest cell is filled as much as possible in view of the availability and destination requirement of
its column.
3. Then we move to the next lowest cell and so on continue the procedure in view of the remaining
availability of demand and supply.
4. The procedure is repeated till all row availability and column requirements are met.
5. In case of tie for the lowest cell during allocation, choice may be made for a row or column by which
maximum requirement is exhausted.

Steps in Vogel’s method

1. Find the difference between two least cost cells in every row and column.
2. Identify the row or column with the highest of the difference. It is in this row or
column where allocation should be made.
3. In the row or column selected in step 2, identify the least cost cell. It is in this cell allocation should
be made.
4. If there is a tie amongst the largest differences, the choice may be made for a row or column which
has least cost. In case there is a tie in cost cell also, choice may be made for a row or column by
which maximum requirement is exhausted.
5. Hatch that row or column containing this cell whose totals have been exhausted so that this row or
column is ignored in further consideration.
6. Re-compute the row & column differences for the reduced TLP go ahead to Step 3. Continue the
above steps till all allocations are made.

Question 1: What are the common methods of obtaining initial feasible solution in a transportation
problem? (3 Marks) Nov./02 & (3 Marks) Nov./08-O.C.

Steps for Stage 2 (Optimality Test)


It can be applied to a transportation table if it satisfies the following conditions:

a) It contains exactly m+n-1 allocations, where m & n represent the no. of rows & columns of the table.

{Note: (i) In case degeneracy occurs (i.e. allocations are less than m+n-1), infinitely small
allocation(s) i.e. ε is/are introduced in least cost & independent cell to non-degenerate the
solution. If least cost cell(s) is/are not independent, then cell(s) with next lower cost(s) is/are
chosen.

(ii) Degeneracy can occur at initial basic feasible solution or at a later stage. ε may be removed
once the purpose is over.

(iii) The quantity, ε is considered to be so small that if it is transferred to an occupied cell, it does not
change the quantity of allocation, i.e. x ij - ε =x ij ; x ij + ε =x ij ; ε - ε = 0; ε + ε = ε; x ij × ε =x ij }

b) These allocations are independent i.e. loop can not be performed by them.

{Note: (i) Closed path or loop should have even no. of turns & is formed with an allocation on
each corner, which in turn is a join of horizontal & vertical (not diagonal) lines. A loop may not
involve all the allocations.

(ii) IBFS obtained by Northwest Corner rule, Least cost method or VAM are always in independent
CA. Parag Gupta Ph.: +91 11 47665555 Paraggupta_ca@yahoo.co.in Costing & O.R.
World’s largest CA Final student’s consultancy group: http://groups.yahoo.com/group/costingbyparaggupta
Operations Research - 297 -

positions though they may be m+n-1 or less than m+n-1 in no.}

The Stepping-Stone Method  LEAST Important Topic

1. Start with an arbitrary empty cell (a cell without allocation a.k.a. non-basic variables) and allocated +1
unit to this cell.
2. In order to maintain conditions of requirement & supply (a.k.a. rim constraints), 1 unit will be
deducted/added to basic variables (i.e. cells having allocations).
3. Calculate Net change (a.k.a. opportunity cost or net evaluation) in transportation cost as a result of
this perturbation.
4. Calculation Net evaluation for every empty cell.
5. If any cell evaluation is -ve, the cost can be reduced so that the solution under consideration can be
improved i.e. it is not optimal. On the other hand if all net evaluation are +ve, the given solution is
optimal one. If any net evaluation is Zero, the given solution is optimal but there exist other optimal
solutions as well.
6. If solution is not optimal, the cell with largest –ve opportunity cost should be selected & allocate
maximum units to this route (subject to rim constraints).
7. Iterate towards optimal solution, if further savings are possible.

Modified Distribution Method (MODI) -Preferable  MOST Important Topic

1. Form UV Matrix. It is formed


(a) By adding a column to the RHS of transportation table labeled u i & row in the bottom to it
labeled v j .
(b) Assign any value arbitrarily to a row or column variable u i or v j.
(c) Taking each cost cell (u i +v j )=c ij , calculate individually all values of u i & v j

2. Calculate Net evaluation table, opportunity cost for each unallocated cell i.e. ij =i.e. c ij -(u i +v j )
{Also, Opportunity cost = Actual cost – Implicit cost}

3. Check for signs of ij . If all ij are +ve, the given solution is optimal one. If any ij is Zero, the
given solution is optimal but there exist other optimal solutions as well. If there are –ve ij, it means
this solution is not optimal.

4. If solution is not optimal, the cell with largest –ve opportunity cost should be selected. Form a closed
loop starting and ending at the selected worst –ve cell and reallocate the solution (maintaining rim
constraints).

E.g. If Reallocation is to done by transferring the max. amount to the ticked cell, the rule for obtaining
max. amount (say θ max ) that can be transferred to the ticked cell is derived as:

3-θ θ

θ max = min of 3 - θ=0 =3


1+θ 9-θ 9 - θ=0

4-θ θ

9-θ θ max = min of 4–θ=0 =4


6+θ 8–θ=0
9–θ=0
7+θ 8-θ

5. Iterate towards optimal solution, if further savings are possible.

Question 2: What do you mean by Degeneracy in transportation problem? How this can be solved?
(4 Marks) May/10-N.C. & (4 Marks) Nov./09-O.C.
Question 3: How do you know whether an alternative solution exists for a transportation problem?
CA. Parag Gupta Ph.: +91 11 47665555 Paraggupta_ca@yahoo.co.in Costing & O.R.
World’s largest CA Final student’s consultancy group: http://groups.yahoo.com/group/costingbyparaggupta
The Transportation Problem - 298 -

(4 Marks) Nov./09-N.C.
Question 4: In an unbalanced minimization transportation problem, with positive unit transport costs from 3
factories to 4 destinations, it is necessary to introduce a dummy destination to make it a balanced
transportation problem. How will you find out if a given solution is optimal? (4 Marks) May/10-O.C.

[Ans.: (i) Yes (ii) Check for signs of ij . If all ij are +ve, the given solution is optimal one. If any ij is
Zero, the given solution is optimal but there exist other optimal solutions as well. If there are –ve ij, it means
this solution is not optimal.]
Question 5: Obtain the IBFS for the following using (i) North West Corner Rule (ii) LCM (iii) VAM.
Warehouse-1 Warehouse-2 Warehouse-3 Supply
Factory- 1 6 8 4 14
Factory- 2 4 9 8 12
Factory- 3 1 2 6 5
Demand 6 10 15 -

Question 6: Goods manufactured at 3 plants, A, B and C are required to be transported to sales outlets X, Y
and Z. The unit costs of transporting the goods from the plants to the outlets are given below:

Sales outlets\Plants A B C Total Demand


X 3 9 6 20
Y 4 4 6 40
Z 8 3 5 60
Total supply 40 50 30 120

You are required to:


(i) Compute the initial allocation by North-West Corner Rule.
(ii) Compute the initial allocation by Vogel’s approximation method and check whether it is optional.
(iii) State your analysis on the optionality of allocation under North-West corner Rule and Vogel’s
Approximation method. (10 Marks) May/08

[Ans.:
(i) Initial allocation under NW corner rule is: (ii) Initial allocation under VAM is:
Plant Sales Outlet Units Initial Cost Plant Sales Outlet Units Initial Cost
A X 20 60 A X 20 60
A Y 20 80 A Y 20 80
B Y 20 80 B Z 50 150
B Z 30 90 C Y 20 120
C Z 30 150 C Z 10 100
460 460
It is optimal solution
(iii) The solution under VAM is optimal with a zero in R 2 C 2 which means that the cell C 2 R 2 which means that
the cell C 2 R 2 can come into solution, which will be another optimal solution. Under NWC rule the initial
allocation had C 2 R 2 and the total cost was the same `460 as the total cost under optimal VAM solution. Thus,
in this problem, both methods have yielded the optimal solution under the 1st allocation. If we do an optimality
test for the solution, we will get a zero for ij in C 3 R 2 indicating the other optimal solution which was
obtained under VAM.]
Question 7 (Minimization-Unbalanced): A company has three plants located at A, B and C. The production
of these plants is absorbed by four distribution centres located at X, Y, W and Z. The transportation cost per
unit has been shown in small cells in the following table:

Distribution Centers X Y W Z Supply


Factories (Units)
A 6 9 13 7 6000
B 6 10 11 5 6000

CA. Parag Gupta Ph.: +91 11 47665555 Paraggupta_ca@yahoo.co.in Costing & O.R.
World’s largest CA Final student’s consultancy group: http://groups.yahoo.com/group/costingbyparaggupta
Operations Research - 299 -

C 4 7 14 8 6000
18000
Demand (Units) 4000 4000 4500 5000 17500

Find the optimum solution of the transportation problem by applying Vogel's Approximation Method.
[Ans.: Total Cost `129500] (8 Marks) Nov./10-N.C.

Question 8 (Minimization-Unbalanced): A product is manufactured by four factories A, B, C & D. The unit


production costs are `2, `3, `1 and `5 respectively. Their daily production capacities are 50, 70, 30 and 50
units respectively. These factories supply the product to four stores P, Q, R, S. The demand made by these
stores are 25, 35, 105 and 20 units respectively. Unit transportation cost in rupees from each factory to each
store is given in the following table:

Stores
Factories P Q R S
A 2 4 6 11
B 10 8 7 5
C 13 3 9 12
D 4 6 8 3

Determine the extent of deliveries from each of the factories to each of the stores so that the total cost
(production and transportation together) is minimum. (10 Marks) May/02

[Ans.: Total Cost `1465]


Question 9 (Minimization-unbalanced-degeneracy): A company has four terminals U, V, W, and X. At the
start of a particular day 10, 4, 6, and 5 trailers respectively are available at these terminals. During the
previous night 13, 10, 6, and 6 trailers respectively were loaded at plants A, B, C, and D. The company
dispatcher has come up with the costs between the terminals and plants as follows:

Plants
A B C D
Terminals

U 20 36 10 28
V 40 20 45 20
W 75 35 45 50
X 30 35 40 25

Find the allocation of loaded trailers from plants to terminals in order to minimize transportation cost.
(10 Marks) May/97
[Ans.:
Terminal Plant Cost (`)
U A 80
U C 60
V B 60
V D 20
W B 210
X D 125
Transportation Cost 555]

Question 10 (Minimization-balanced-degeneracy): The cost per unit of transporting goods from factories
X, Y, Z to destinations A, B and C, and the quantities demanded and supplied are tabulated below. As the
company is working out the optimum logistics, the Govt. has announced a fall in oil prices. The revised unit
costs are exactly half the costs given in the table. You are required to evaluate the minimum transportation
cost.

CA. Parag Gupta Ph.: +91 11 47665555 Paraggupta_ca@yahoo.co.in Costing & O.R.
World’s largest CA Final student’s consultancy group: http://groups.yahoo.com/group/costingbyparaggupta
The Transportation Problem - 300 -

Factories\Destinations A B C Supply
X 15 9 6 10
Y 21 12 6 10
Z 6 18 9 10
Demand 10 10 10 30

[Ans.: Min. transportation cost is `105] (6 Marks) June/09-N.C.

Question 11 (Minimization-balanced-degeneracy): Find optimal solution for the following problem

Warehouse-1 Warehouse-2 Warehouse-3 Supply


Factory-1 50 30 220 1
Factory-2 30 45 170 3
Factory-3 250 200 50 4
Demand 4 2 2

[Ans.: Optimal Cost is `640]

Question 12 (Degeneracy - ε is introduced at next least cost cell): The initial allocation of a transportation
problem, alongwith the unit cost of transportation from each origin to destination is given below. You are
required to arrive at the minimum transportation cost by the Vogel’s Approximation method and check for
optimality.
(Hint : Candidates may consider u 1 = 0 at Row 1 for initial cell evaluation)

[Ans,: Optimal Cost = `204] (6 Marks) May/07


[Hint for solution: R 4 C 2 has the least cost (cost = 3), but this is not independent. The next least cost cell
R 4 C 3 (cost = 5) is independent, hence we will select it to resolve the degeneracy.]
Question 13 (Minimization-unbalanced-degeneracy): A company has factories at A, B and C which
supply warehouses at D, E, F and G. Monthly factory capacities are 160, 150 and 190 units respectively.
Monthly warehouse requirements are 80, 90, 110 and 160 units respectively. Unit shipping costs (in rupees)
are as follows:

To
From D E F G
A 42 48 38 37
B 40 49 52 51
C 39 38 40 43

Determine the optimum distribution for this company to minimize shipping costs.

[Ans.: Minimal Cost is `17050]


Question 14: Find optimal solution for the following transportation cost problem.

CA. Parag Gupta Ph.: +91 11 47665555 Paraggupta_ca@yahoo.co.in Costing & O.R.
World’s largest CA Final student’s consultancy group: http://groups.yahoo.com/group/costingbyparaggupta
Operations Research - 301 -

Go-down 1 Go-down 2 Go-down 3 Go-down 4 Go-down 5 Go-down 6 Supply


Factory- 1 4 6 9 2 7 8 10
Factory- 2 3 5 4 8 10 0 12
Factory- 3 2 6 9 8 4 13 4
Factory- 4 4 4 5 9 3 6 18
Factory- 5 9 8 7 3 2 14 20
Demand 8 8 16 3 8 21 -

[Ans.: `242]
NOTE: SIMILAR QUESTION ↑ IS WRONGLY SOLVED BY STUDY MATERIAL OF ICAI
Question 15 (Prohibited routes): Solve the following transportation problem

Go-down 1 Go-down 2 Go-down 3 Go-down 4 Go-down 5 Go-down 6 Stock availability


Factory 1 7 5 7 7 5 3 60
Factory 2 9 11 6 11 * 5 20
Factory 3 11 10 6 2 2 8 90
Factory 4 9 10 9 6 9 12 50
Demand 60 20 40 20 40 40
Note: It is not possible to transport any quantity from factory 2 to go-down 5. State whether the solution
derived by you is unique. (Nov./1989)

[Ans.: Total Cost `1120]


Question 16 (Multiple Optimal Solutions): A Compressed Natural Gas (CNG) company has three plants
producing gas and four outlets. The cost of transporting gas from different production plants to the outlets,
production capacity of each plant & requirement at different outlet is shown in the following cost-matrix table.

Outlets
Plants A B C D Capacity of Production
X 4 6 8 6 700
Y 3 5 2 5 400
Z 3 9 6 5 600
Requirement 400 450 350 500 1700

Determine a transportation schedule so that the cost is minimized.


The cost in the cost-matrix is given in thousand of rupees. (10 Marks) Nov./01

[Ans.: Minimum costs `7350 thousands]


Question 17 (Maximization-unbalanced-degeneracy-multiple solution): A particular product is
manufactured in factories A, B, C and D; and is sold at centres 1, 2, and 3. The cost in ` of product per unit
and capacity in kgms per unit time of each plant is given below:

Factory Cost (`) per unit Capacity (kgms) per unit


A 12 100
B 15 20
C 11 60
D 13 80

The sale price in `per unit and the demand in kgms per unit time are as follows:

Sales Centre Sale Price (`) per unit Demand (kgms) per unit
1 15 120
2 14 140
3 16 60

Find the optimal sale distribution. (10 Marks) Nov./97


CA. Parag Gupta Ph.: +91 11 47665555 Paraggupta_ca@yahoo.co.in Costing & O.R.
World’s largest CA Final student’s consultancy group: http://groups.yahoo.com/group/costingbyparaggupta
The Transportation Problem - 302 -

[Ans.:
From Factory To Sales Centre Quantity Total Profit
A 1 100 300
B 2 20 -20
C 3 60 300
D 1 20 40
D 2 60 60
Dummy 2 60 0
680]

Question 18 (Investment decision): XYZ and co. has provided the following data seeking your advice on
optimum investment strategy:

Beginning of the year Net Return Data (in paise) of Selected Amount available
Investments (Lacs)
P Q R S
1 95 80 70 60 70
2 75 65 60 50 40
3 70 45 50 40 90
4 60 40 40 30 30
Maximum Investment (lacs) 40 50 60 60 -

The following additional information are also provided:


 P, Q, R, and S represent the selected investments.
 The company decided to have four years investment plan.
 The policy of the company is that amount invested in any year will remain so until the end of the
fourth year.
 The values (paise) in the table represent net return on investment of one Rupee till the end of the
planning horizon (for example, a Rupee invested in Investment P at the beginning of year 1 will
grow to `1.95 by the end of the fourth year, yielding a return of 95 paise).
Using the above, determine the optimum investment strategy. (10 Marks) Nov/96

[Ans.: Total investment `13000000]


Question 19 (Maximization-unbalanced): ABC Enterprises is having three plants manufacturing dry-cells,
located at different locations. Production cost differs from plant to plant. There are five sales offices of the
company located in different regions of the country. The sales prices can differ from region to region. The
shipping cost from each plant to each sales office and other data are given by following table:

Production Data Table


Production cost per unit Max. capacity in no. of units Plant no.
20 150 1
22 200 2
18 125 3

Shipping Cost and Demand & Sales Prices table


Shipping Costs
Sales Office 1 Sales Office 2 Sales Office 3 Sales Office 4 Sales Office 5
Plant 1 1 1 5 9 4
Plant 2 9 7 8 3 6
Plant 3 4 5 3 2 7
Demand & Sales Prices
Sales Office 1 Sales Office 2 Sales Office 3 Sales Office 4 Sales Office 5
Demand 80 100 75 45 125
Sales Price 30 32 31 34 29

Find the production and distribution schedule most profitable to the company. (10 Marks) Nov/98

CA. Parag Gupta Ph.: +91 11 47665555 Paraggupta_ca@yahoo.co.in Costing & O.R.
World’s largest CA Final student’s consultancy group: http://groups.yahoo.com/group/costingbyparaggupta
Operations Research - 303 -

[Ans.:
Plant Sales Office Units Total Profit (`)
1 1 50 450
1 2 100 1100
2 4 25 225
2 5 125 125
2 Dummy 50 0
3 1 30 240
3 3 75 750
3 4 20 280
Total 3170]

Question 20 (Maximization-unbalanced): A company has three factories and four customers. The
company furnishes the following schedule of profit per unit on transportation of its goods to the customers in
rupees:
Customers
Factory A B C D Supply
P 40 25 22 33 100
Q 44 35 30 30 30
R 38 38 28 30 70
Demand 40 20 60 30

You are required to solve the transportation problem to maximize the profit and determine the resultant of
optimal profit. (8 Marks) May/03
[Ans.: Maximum Profit `5130]
Question 21 (Maximization-unbalanced-degeneracy): A leading firm has three auditors. Each auditor can
work up to 160 hours during the next month, during which time three projects must be completed. Project 1
will 130 hours, project 2 will take 140 hours, and project 3 will take 160 hours. The amount per hour that can
be billed for assigning each auditor to each project is given in Table 1:
Table1
Project
Auditor 1 2 3
` ` `
1 1200 1500 1900
2 1400 1300 1200
3 1600 1400 1500

Formulate this as a transportation problem and find the optimal solution. Also find out the maximum total
billings during the next month. (10 Marks) May/95

[Ans.: Auditor Project Billing Amount (`)


1 3 304000
2 2 143000
3 1 208000
3 2 42000
Total billings 697000]

Miscellaneous Questions:
Question 22 (Overtime Production): A company has factories at A, B, and C, which supply warehouses at
D, E, F and G. The factories capacities are 230, 280, and 180 respectively for regular production. If overtime
production is utilised, the capacities can be increased to 300, 360 and 190 respectively. Increment unit
overtime costs are `5, `4, and `6 respectively. The current warehouse requirements are 165, 175, 205 and
165 respectively. Unit shipping costs in rupees between the factories and the warehouses are

CA. Parag Gupta Ph.: +91 11 47665555 Paraggupta_ca@yahoo.co.in Costing & O.R.
World’s largest CA Final student’s consultancy group: http://groups.yahoo.com/group/costingbyparaggupta
The Transportation Problem - 304 -

To
D E F G
From
A 6 7 8 10

B 4 10 7 6

C 3 22 2 11

Determine the optimum distribution for the company to minimize costs.

[Ans.:
From To Quantity
A D 40
A E 175
A F 15
B D 115
B G 165
C F 180
A1 Dummy 70
B1 D 10
B1 Dummy 70
C1 F 10]

Question 23 [Redundancy cost]: A company has seven manufacturing units situated in different parts of the
country. Due to recession it is proposed to close four of these and to concentrate production in the remaining
three units. Production in these units will actually be increased from present level and would require an
increase in the personnel employed in them. Personnel at the closed units expressed their desire from
moving to any one of the remaining units and the company is willing to provide them removal (transfer) costs.
The retraining expenses would have to be incurred as the technology in these units are different. Not all
existing personnel can be absorbed by transfer and a number of redundancies will arise, cost of redundancy
is given as a general figure at each unit closed.

A B C D
No. Employed 200 400 300 200
(These units A, B, C and D are to be closed)
Retraining costs in ` ‘000/person A B C D
Transfer to E 0.5 0.4 0.6 0.3
F 0.6 0.4 0.6 0.3
G 0.5 0.3 0.7 0.3
Removal cost in `‘000/person A B C D
Transfer to E 2.5 3.6 3.4 4.7
F 2.4 4.6 3.4 1.7
G 2.5 2.7 3.3 2.7
Redundancy payments in `‘000/person A B C D
6.0 5.0 6.0 7.0
Additional personnel required at E F G
Units
Required 350 450 200

(i) Obtain a solution to the problem of the cheapest means to transfer personnel from units closed to
those units which will be expanded.
(ii) State with reason whether or not the solution obtained is optimal and unique.
(iii) State the costs of the initial and final solution. [ICWA /1990]

[Ans.: (i)

CA. Parag Gupta Ph.: +91 11 47665555 Paraggupta_ca@yahoo.co.in Costing & O.R.
World’s largest CA Final student’s consultancy group: http://groups.yahoo.com/group/costingbyparaggupta
Operations Research - 305 -

From To Quantity Total Cost (`‘000s)


A E 200 600
B E 100 400
B G 200 600
B Dummy 100 500
C E 50 200
C F 250 1000
D F 200 400
3700
(ii) Since there exist Zero ij hence, solution is optimal but not unique. (iii) Costs are same i.e.`3700000]

Trans-shipment Model (LEAST IMPORTANT)

The trans-shipment model recognizes that it may be cheaper to ship through intermediate or transient
nodes before reaching the final destination. This concept is more general than that of the regular
transportation model, where direct shipments only are allowed between a source and a destination.
In the tras-shipment model, as each source or destination is a potential point of supply as well as demand,
total supply, say of B units is added to the actual supply of each source as well as to the actual demand at
each destination. Also the ‘demand’ at each source & ‘supply’ at each destination is set equal to B.
Therefore, we may assume the supply & demand of each location to be fictitious one. These quantities (B)
may be regarded as buffer stocks & each of these buffer stocks should at least be equal to the total
supply/demand in the given problem.

Question 24: A firm has two factories X and Y and three retail stores A, B and C. The number of units of a
product available at factories X and Y are 200 and 300 respectively, while demanded at retail stores are
100, 150 and 250 respectively. Rather than shipping directly from sources to destinations, it is decided to
investigate the possibility of trans-shipment. Find the optimal shipping schedule. The transportation costs in
rupees per unit are given below.

Factory Retail Store


X Y A B C
X 0 6 7 8 9
Factory
Y 6 0 5 4 3
A 7 2 0 5 1
Retail Store B 1 5 1 0 4
C 8 9 7 6 0

[Ans.: Factory X supplies 100 units each to retail stores A and B. Factory Y supplies 50 units to retail store
B and 250 units to C.]
Question 24: Table 1 represents the supply from the plants, the requirement at the distribution centres and
the unit transportation costs.

Table 1 – Distribution Centres


A B C Supply
Plants 1 11 13 25 150
2 13 15 35 300
Requirement 150 150 150

When each plant is also considered a destination and each distribution centre is also considered as origin,
some additional cost data are necessary, which are given in the tables below:

Table 2 - To Plants Table 3 - To dist. centres Table 4 - To plants


1 2 A B C 1 2
From 1 0 75 From A 0 33 11 From A 13 25
Plant 2 11 0 Dist. B 11 0 13 Dist. B 35 13
Centre C 75 13 0 Centre C 55 65

CA. Parag Gupta Ph.: +91 11 47665555 Paraggupta_ca@yahoo.co.in Costing & O.R.
World’s largest CA Final student’s consultancy group: http://groups.yahoo.com/group/costingbyparaggupta
The Transportation Problem - 306 -

Find the optimal shipping schedule for the trans-shipment problem.

[Ans.: Plant 2 supplies 450 units each to Distribution Center A & B; Plant 1 transports 150 units to
distribution centre A. Distribution centre A sends 150 units to C; Total cost of transshipment `7,500]

CA. Parag Gupta Ph.: +91 11 47665555 Paraggupta_ca@yahoo.co.in Costing & O.R.
World’s largest CA Final student’s consultancy group: http://groups.yahoo.com/group/costingbyparaggupta
The Assignment Problem

The assignment problem like transportation is another special case of LPP. In general, it is concerned with
one to one basis when n jobs are to be assigned to n facilities with a view to optimizing the resource
required. The emphasis in an assignment problem is on determining how assignment should be made in
order that the total cost involved is minimized or alternatively, total value is maximized when pay-offs are
given in terms of say profits.

Steps for solving assignment problem (Using Hungarian Assignment Method-HAM):

1) If problem is not balanced i.e. m≠n, introduce dummy row/column to balance it.
Where m= no. of rows & n= no. of columns.
2) In case of problems of maximization, Convert profit matrix into loss matrix. This is done by
subtracting each entry in the table from largest no. in the table.
[Important Note: Though we can also use step 2 before step 1(i.e. we can also convert profit matrix in loss

matrix before balancing the problem), but, it is recommended to apply the steps in above defined order

only]
3) Assign M or ∞ to prohibited assignments, etc.
4) Subtract the minimum element of each row from all the elements in that row.
[Note: Step 4 is to calculate Men/Machine/Facility/etc. opportunity costs with regards to the respective

jobs/tasks/etc. (i.e. penalty for assigning to a facility, machine, etc. in spite of least cost assignment.)]
5) Subtract the minimum element of each column from all the elements in that column. The resulting
matrix is starting matrix.
[Note: Step 5 is to calculate total opportunity cost table.]
6) Draw the min. no. of lines (both horizontal & vertical) to cover all zeros.
7) Now check that whether n i.e. order of matrix & no. of lines drawn in Step 6 are same or not. If yes, go
to step 10, otherwise go to step 8 (for improving the solution).
[Note: Order of matrix means no. of rows/column in a matrix]
8) Find the smallest uncovered cost element (by the lines drawn in step 7). Subtract this element from all
such (uncovered) elements & add it to the elements with two lines through it (i.e. intersection of
horizontal & vertical lines). Do not alter the elements through which one line passes.
[Note: Step 8 gives us revised opportunity cost table]
9) Repeat Steps 6-8 until we get minimum no. of lines equal to n.
10) Find a row with only one zero in it, & make an assignment (by encircling it) to this zero. Draw a vertical
line through the column containing this zero. This eliminates any confusion of making any further
assignment in that column. Repeat this step until no row with single zero is left.
11) Find a column with only one zero in it, & make an assignment to this zero. Draw a horizontal line
through the row containing this zero. This eliminates any confusion of making any further assignment in
that row. Repeat this step until no column with single zero is left.
12) If at this stage, there are still zeroes which are not assigned or crossed, then go to step 13 otherwise
the solution is optimal.
13) If any row/column consists more than one zero, make an assignment to any zero arbitrarily & draw 2
CA. Parag Gupta Ph.: +91 11 47665555 Paraggupta_ca@yahoo.co.in Costing & O.R.
World’s largest CA Final student’s consultancy group: http://groups.yahoo.com/group/costingbyparaggupta
The Assignment Problem - 308 -

lines i.e. one horizontal & one vertical line through the row & column containing this zero.
14) Repeat steps 10) to 13) until no further movement is possible. The resulting solution is optimal.
15) Determine the total cost with reference to the original cost table.

Question 1: In an assignment problem to assign jobs to men to minimise the time taken, suppose that one
man does not know how to do a particular job, how will you eliminate this allocation from the solution?
(4 Marks) Nov./09-N.C.
Question 2 (Minimization balanced): A Marketing Manager has 4 subordinates and 4 tasks. The
subordinates differ in efficiency. The tasks also differ in their intrinsic difficulty. His estimates of the time each
subordinate would take to perform each task is given in the matrix below. How should the task be allocated
one to one man so that the total man-hours are minimized? (7 Marks) Nov./04

I II III IV
1 16 52 34 22
2 26 56 8 52
3 76 38 36 30
4 38 52 48 20

[Ans.: Minimum time taken = 82 hours]


Question 3: Find the optimal solution for the assignment problem with the following cost matrix:

Marketing Division
Executive N E W S
A 14 20 11 19
B 12 10 15 9
C 16 19 18 15
D 17 13 15 14

[Ans.: Total minimum cost `49] (10 Marks) Nov./03


Question 4 (Maximization-balanced): A manager has 4 subordinates and 4 tasks. The subordinates differ
in efficiency. His estimate of the production each would do is given in the table. How the task should be
allocated one to one man, so that total production is maximized.

Subordinates
Task I II III IV
1 8 26 17 11
2 13 28 4 26
3 38 19 18 15
4 19 26 24 10

[Ans.: Optimal Production is 114 units]


Question 5 (Maximization-balanced): An organization producing 4 different products viz. A, B, C, and D
having 4 operators viz. P, Q, R and S, who are capable of producing any of the four products, works
effectively 7 hours a day. The time (in minutes) required for each operator for producing each of the
product are given in the cells of the following matrix along with profit (`per unit):

Products
Operators A B C D
P 6 10 14 12
Q 7 5 3 4
R 6 7 10 10
S 20 10 15 15
Profit (`/unit) 3 2 4 1

CA. Parag Gupta Ph.: +91 11 47665555 Paraggupta_ca@yahoo.co.in Costing & O.R.
World’s largest CA Final student’s consultancy group: http://groups.yahoo.com/group/costingbyparaggupta
Operations Research - 309 -

Find the optimal assignment of operators to products which will maximize the profit.

[Ans.: Operator Product Profit


P A 210
Q C 560
R B 120
S D 28] (10 Marks), May/96 & (10 Marks), Nov./97 - Adapted
Question 6 (Minimization-unbalanced): A has one surplus truck in each cities A, B, C, D & E and one
deficit truck in each of the cities 1, 2, 3, 4, 5 & 6. The distance between the cities in kilometers is shown in the
matrix below.
Cities 1 2 3 4 5 6
A 12 10 15 22 18 8
B 10 18 25 15 16 12
C 11 10 3 8 5 9
D 6 14 10 13 13 12
E 8 12 11 7 3 10

Find the assignment of trucks from the cities in surplus to cities in deficit so that the total distance covered
by vehicles in minimum.

[Ans.: Optimum Distance is 34 kms]


Question 7 (Multiple optimal solutions): Solve the minimal assignment problem whose effectiveness matrix
is
Men
Jobs 1 2 3 4
I 2 3 4 5
II 4 5 6 7
III 7 8 9 8
IV 3 5 8 4
[Ans.: Optimal cost is `20/-]
Question 8 (Multiple Optimal Solutions): The cost matrix giving selling costs per unit of a product by
salesman A, B, C and D in Regions R 1 , R 2 , R 3 and R 4 is given below:
(i) Assign one salesman to one region to minimize selling cost.
(ii) If the selling price of the product is `200 per unit and variable cost excluding the selling cost given
in the table is `100 per unit, find the assignment that would maximize the contribution.
(iii) What other conclusion can you make from the above?

A B C D
R1 4 12 16 8
R2 20 28 32 24
R3 36 44 48 40
R4 52 60 64 56
(8 Marks) Nov./08-N.C.
[Ans.: R 1 ->A, R 2 ->B, R 3 ->C and R 4 ->D with minimum selling cost as `136; (ii) R 1 ->A, R 2 ->B, R 3 ->C and
R 4 ->D with maximum contribution as `264]
Question 9 (Prohibited routes): A BPO Co. is taking bids for 4 routes in the city to ply pick-up and drop
cabs. Four companies have made bids as detailed below: Bids for Routes (`) :

Company/Routes R1 R2 R3 R4
C1 4,000 5,000 --- ---
C2 --- 4,000 --- 4,000
C3 3,000 --- 2,000 ---
C4 --- --- 4,000 5,000

Each bidder can be assigned only one route. Determine the minimum cost.
CA. Parag Gupta Ph.: +91 11 47665555 Paraggupta_ca@yahoo.co.in Costing & O.R.
World’s largest CA Final student’s consultancy group: http://groups.yahoo.com/group/costingbyparaggupta
The Assignment Problem - 310 -

[Ans.: `15000/-] (10 Marks) Nov./95 & (6 Marks) Nov./06


Question 10 (Prohibited routes): WELLDONE Company has taken the third floor of a multi- storied building
for rent with a view to locate one of their zonal offices. There are five main rooms in this floor to be
assigned to five managers. Each room has its own advantages and disadvantages. Some have windows;
some are closer to the washrooms or to the canteen or secretarial pool. The rooms are of all different sizes
and shapes. Each of the five managers was asked to rank their room preferences amongst the rooms 301,
302, 303, 304 and 305. Their preferences were recorded in a table as indicated below:

MANAGER
M1 M2 M3 M4 M5
302 302 303 302 301
303 304 301 305 302
304 305 304 304 304
* 301 305 303 *
* * 302 * *

Most of the managers did not list all the five rooms since they were not satisfied with some of these rooms
and they have left off these from the list. Assuming that their preferences can be quantified by numbers,
find out as to which manager should be assigned to which room so that their total preference ranking is a
minimum. (May/1988 & Nov./1990)

[Ans.: M 1 →302, M 2 →304, M 3 →303, M 4 →305 and M 5 →301 with total minimum ranking as 7]
Question 11 (Unbalanced-Prohibited routes): A factory is going to modify a plant layout to install four
new machines M 1 , M 2 , M 3 and M 4 . There are 5 vacant places J, K, L, M and N available. Because of
limited space machine M 2 cannot be placed at L and M 3 cannot be placed at J. The cost of locating
machine to place in Rupees is shown below:

(`)
J K L M N
M1 18 22 30 20 22
M2 24 18 - 20 18
M3 - 22 28 22 14
M4 28 16 24 14 16

Determine the optimal assignment schedule in such a manner that the total costs are kept at a minimum.

[Ans.: M 1 →J, M 2 →K, M 3 →N, & M 4 →M with total minimum cost of `64] (7 Marks) June/09-O.C.
Question 12 (Maximization-Prohibited route): Imagine yourself to be the Executive Director of a 5-star
Hotel which has four banquet halls that can be used for all functions including weddings. The halls were all
about the same size and the facilities in each hall differed. During a heavy marriage season, 4 parties
approached you to reserve a hall for the marriage to be celebrated on the same day. These marriage
parties were told that the first choice among these 4 halls would cost `10000 for the day. They were also
required to indicate the second, third and fourth preferences and the price that they would be willing to pay.
Marriage party A & D indicated that they won’t be interested in Halls 3 & 4. Other particulars are given in the
following table:

Revenue/Hall
Halls
Marriage Party 1 2 3 4
A 10000 9000 X X
B 8000 10000 8000 5000
C 7000 10000 6000 8000
D 10000 8000 X X

Where X indicates that the party does not want that hall.
CA. Parag Gupta Ph.: +91 11 47665555 Paraggupta_ca@yahoo.co.in Costing & O.R.
World’s largest CA Final student’s consultancy group: http://groups.yahoo.com/group/costingbyparaggupta
Operations Research - 311 -

Decide on an allocation that will maximize the revenue to your hotel. (10 Marks) May/95

[Ans.: Marriage Party Revenue (`)


A Hall 2 9000
B Hall 3 8000
C Hall 4 8000
D Hall 1 10000]

Question 13 (Airline scheduling): An airline operates seven days a week has time-table as shown
below. Crews must have a minimum layover of 5 hours between flights. Obtain the pairing of flights that
minimizes layover time away from home. For any given pairing the crew will be based at the city that results
in smaller layover. For each pair also mention the town where the crew should be based.

Delhi Jaipur Jaipur Delhi


Flight no. Departure Arrival Flight no. Departure Arrival
1 7.00 8.00 101 8.00 9.15
2 8.00 9.00 102 8.30 9.45
3 13.30 14.30 103 12.00 13.15
4 18.30 19.30 104 17.30 18.45

[Ans.: Minimum Layover time is 52.5 hours] (10 Marks) May/97-Adapted & (10 Marks) May/00-Adapted
Question 14 (Data entry job allocation): A firm employs typists for piecemeal work on an hourly basis.
There are five typists available and their charges and speeds are different. According to an earlier
understanding, only one job is given to one typist and the typist is paid for full hours even if he works for a
fraction of an hour. Find the least cost allocation for the following data:

Typist Rate/Hour Pages/Hour Job No. of Pages


A `5 12 P 199
B `6 14 Q 175
C `3 8 R 145
D `4 10 S 298
E `4 11 T 178

[Ans.: Typist Job Cost(`) (10 Marks) Nov./96


A T 75
B R 66
C Q 66
D P 80
E S 112
399]
Question 15 (Traveling salesmen-LEAST IMPORTANT): A travelling salesman has to visit 5 cities. He
wishes to start from a particular city, visit each city once and return to his starting point. The travelling cost
for each city from a particular city is given below:

To City
A B C D E
A X 4 7 3 4
B 4 X 6 3 4
From City C 7 6 X 7 5
D 3 3 7 X 7
E 4 4 5 7 X

What is the sequence of visit of the salesman, so that the cost is minimum?
[Hint: Route A to B, B to D, D to A shall not be taken because D is not allowed to follow A, till C & E are

CA. Parag Gupta Ph.: +91 11 47665555 Paraggupta_ca@yahoo.co.in Costing & O.R.
World’s largest CA Final student’s consultancy group: http://groups.yahoo.com/group/costingbyparaggupta
The Assignment Problem - 312 -

processed. So we will follow next best solution i.e. A-E-C-B-D-A] (I.C.W.A. Final, June/97-Adapted)

Question 16 (Replacement decisions): Average time taken by an operator on a specific machine is


tabulated below. The management is considering replacing one of the old machines by a new one and the
estimated time for operation by each operator on the new machine is also indicated.

Machines
Operators 1 2 3 4 5 6 New
A 10 12 8 10 8 12 11
B 9 10 8 7 8 9 10
C 8 7 8 8 8 6 8
D 12 13 14 14 15 14 11
E 9 9 9 8 8 10 9
F 7 8 9 9 9 8 8

(a) Find out an allocation of operators to the old machines to achieve a minimum operation time.
(b) Reset the problem with the new machine and find out the allocation of the operators to each machine
and comment on whether it is advantageous to replace an old machine to achieve a reduction in
operating time only.
(c) How will the operators be reallocated to the machines after replacement?

[Ans.: Old Allocation A-3,B-4,C-6,D-1,E-5,F-2 with time = 49 New Allocation A-3,B-4,C-6,D-New,E-5,F-1 with
time =47]
Question 17: A company has four zones open and four marketing managers available for assignment. The
zones are not equal in sales potentials. It is estimated that a typical marketing manager operating in each
zone would bring in the following Annual sales:
Zones `
East 2,40,000
West 1,92,000
North 1,44,000
South 1,20,000

The four marketing managers are also different in ability. It is estimated that working under the same
conditions, their yearly sales would be proportionately as under:

Manager M : 8
Manager N : 7
Manager O : 5
Manager P : 4

Required:
If the criterion is maximum expected total sales, find the optimum assignment and the maximum sales.

[Ans.: Assignment Sales (`)


M – East 80,000
N – West 56,000
O – North 30,000
P – South 20,000
1,86,000] (11Marks) Nov./07 & (10 Marks) May/98-Adapted
{Caution: Please make sure that you do the above question in accordance with answer
of similar questions solved by me or provided by C.A. Institute’s Suggested Answers –
Please abstain from using any other method suggested by any other teacher of Delhi}

Question 18 (Liquidity management): X holds stock of different companies. For a certain problem he is
compelled to sell off four of his holdings. Since he wants the money over the next five months, he sells not
more than one stock in any month. He has estimated the sale proceeds in each of the next five months as
follows:
CA. Parag Gupta Ph.: +91 11 47665555 Paraggupta_ca@yahoo.co.in Costing & O.R.
World’s largest CA Final student’s consultancy group: http://groups.yahoo.com/group/costingbyparaggupta
Operations Research - 313 -

(`‘000)
June July Aug Sep Oct
S1 13 16 14 19 17
S2 18 20 13 18 12
S3 17 15 10 22 18
S4 19 18 14 21 15
Find what will be the optimum plan of X and how much money can he realize by sales?

[Ans.: Optimum sale proceeds are `78000]


Question 19 (Market research and assignment): The market research team of the Look Forward Company
requires some household data from four different cities. The team has to perform this job in two days-
the next Saturday and Sunday. It plans to spend half a-day in each of the cities. The relevant data are given
here:

Probability of a Household
Contact
Day and Time City 1 City 2 City 3 City 4
Saturday Morning 0.32 0.85 0.16 0.64
Saturday Evening 0.60 0.56 0.95 0.80
Sunday Morning 0.70 0.35 0.40 0.62
Sunday Evening 0.10 0.72 0.64 0.90
Number of households expected to interview 150 100 200 200

How should the team plan its visit to the four cities so that the expected response may be maximized? State
this expected response.

[Ans.: Expected response is 560]


Question 20: A hospital has to pay nurses for 40 hours a week. One nurse is assigned to one patient. The
cost per hour for each of the nurses is given below:
(i) Find the nurse-patient combination to minimize cost to the hospital.
(ii) How much does each nurse earn per week?

Nurse\Patient W X Y
K 10 10 30
L 30 10 20
M 20 30 20

Suppose that a new patient Z is admitted, and that a new nurse N is appointed. The new patient is charged
`40 per hour by each of the existing nurses. The new nurse charges `50 per hour irrespective of the patient.
(iii) What would be your revised calculations?
(iv) Comment on the new solution. (8 Marks) May’10-O.C.

[Ans.: (i) K→W; L→X; M→Y (ii) 400, 400, 800 (iii) K→W; L→X; M→Y; N→Z & 400, 400, 800, 2000]
Question 21: A city corporation has decided to carry out road repairs on main four arteries of the
city. The government has agreed to make a special grant of `50 lakhs towards the cost with a condition that
the repairs must be done at the lowest cost and quickest time. If conditions warrant, then a supplementary
token grant will also be considered favorably. The corporation has floated tenders and 5 contractors have sent
in their bids. In order to expedite work, one road will be awarded to only one contractor.

Cost of Repairs (`Lakhs)


Contractors/Road R1 R2 R3 R4
C1 9 14 19 15
C2 7 17 20 19
C3 9 18 21 18
C4 10 12 18 19
C5 10 15 21 16

CA. Parag Gupta Ph.: +91 11 47665555 Paraggupta_ca@yahoo.co.in Costing & O.R.
World’s largest CA Final student’s consultancy group: http://groups.yahoo.com/group/costingbyparaggupta
The Assignment Problem - 314 -

(i) Find the best way of assigning the repair work to the contractors and the costs.
(ii) If it is necessary to seek supplementary grants, then what should be amount sought?
(iii) Which of the five contractors will be unsuccessful in his bid?
(iv) If C1 unable to accept any work, find best assignment.

[Ans.: C1-R3, C2-R1, C3-Dummy, C4-R2, C5-R4; Grant of `4 Lakhs; C3]

CA. Parag Gupta Ph.: +91 11 47665555 Paraggupta_ca@yahoo.co.in Costing & O.R.
World’s largest CA Final student’s consultancy group: http://groups.yahoo.com/group/costingbyparaggupta
Network Analysis –
PERT/CPM

PERT – Program Evaluation & Review Technique – It is generally used for those projects where time
required to complete various activities are not known as a priori. It is probabilistic model & is primarily
concerned for evaluation of time. It is event oriented.

CPM – Critical Path Analysis – It is a commonly used for those projects which are repetitive in nature &
where one has prior experience of handling similar projects. It is a deterministic model & places emphasis on
time & cost for activities of a project.

 A project can be defined as a set of large number of activities or jobs (with each activity
consuming time & resources) that are performed in a certain sequence determined.
 A network is a graphical representation of a project, depicting the flow as well as the sequence of
well-defined activities & events.
 An activity (Also known as task & job) is any portion of a project which consumes time or
resources and has definable beginning & ending.
 Event (Also known as node & connector) is the beginning & ending points of an activity or a
group of activities.

Steps for drawing CPM/PERT network: (4 Marks) Nov./10-N.C.


1. Analyze & break up of the entire project into smaller systems i.e. specific activities and/or events.
2. Determine the interdependence & sequence of those activities.
3. Draw a network diagram.
4. Estimate the completion time, cost, etc. for each activity.
5. Identify the critical path (longest path through the network).
6. Update the CPM/PERT diagram as the project progresses.

Network Representation:

Each activity of the project is represented by arrow pointing in direction of progress of project. The events of
the network establish the precedence relationship among different activities.
Three rules are available for constructing the network.

Rule 1. Each activity is represented by one & only one, arrow.

Rule 2. Each activity must be identified by two distinct events & No two or more activities can have the same
tail and head events.
Following figure shows how a dummy activity can be used to represent two concurrent activities, A & B.
Dummy activity is a hypothetical activity which takes no resource or time to complete. It is represented by
broken arrowed line & is used for either distinguishing activities having common starting & finishing events or
to identify & maintain proper precedence relationship between activities that are not connected by events.

CA. Parag Gupta Ph.: +91 11 47665555 Paraggupta_ca@yahoo.co.in Costing & O.R.
World’s largest CA Final student’s consultancy group: http://groups.yahoo.com/group/costingbyparaggupta
Network Analysis - PERT / CPM - 316 -
(including resource allocation & resource leveling)

lecture notes
Space for

Inserting dummy activity in one four ways in the figure, we maintain the concurrence of A & B, and provide
unique end events for the two activities (to satisfy Rule 2).

Rule 3. To maintain correct precedence relationship, the following questions must be answered as each
activity is added to the network:
(a) What activities must be immediately precede the current activity?
(b) What activities must follow the current activity?
(c) What activities must occur concurrently with the current activity?

The answers to these questions may require the use of dummy activities to ensure correct precedence’s
among the activities. For example, consider the following segment of a project:
1. Activity C starts immediately after A and B has been completed.
2. Activity E starts only after B has been completed.
Part (a) of the figure above, shows the incorrect representation of the precedence relationship because it
requires both A & B to be completed before E can start. In part (b) the use of dummy rectifies situation.

Question 1: What do you mean by a dummy activity? Why it is used in networking? (4 Marks) May/08
[Ans.: Dummy activity is a hypothetical activity which takes no resource or time to complete. It is represented
by broken arrowed line and is inserted in the network to clarify an activity pattern under the following
situations.
(a) For distinguishing activities having common starting & finishing events
(b) To identify & maintain proper precedence relationship between activities that are not connected by
events.
(c) To bring all “loose ends” to a single initial and single terminal event.]
Question 2: Explain the following in the context of a network:
(i) Critical path
(ii) Dummy activity (4 Marks) June/09-N.C.
Question 3 (Application of Dummy): Depict the following dependency relationships by means of network
diagrams.(The Alphabets stands for activities)
1. A and B control F; B and C control G.
CA. Parag Gupta Ph.: +91 11 47665555 Paraggupta_ca@yahoo.co.in Costing & O.R.
World’s largest CA Final student’s consultancy group: http://groups.yahoo.com/group/costingbyparaggupta
Operations Research - 317 -

2. A controls F, G and H; B controls G and H with H controlled by C.


3. A controls F and G; B controls G while C controls G and H.
4. A controls F and G; B and C control G with H depending upon C.
5. F and G are controlled by A; G and H are controlled by B with H controlled by B and C.
6. A and B control F; B controls G while C controls G and H.
Question 4: Develop a network based on the following information;

Activity Immediate predecessors


A -
B -
C A
D B
E C,D
F D
G E
H F
Question 5 [Difficult network-Try it at only after proper practice]: Construct the project network
comprised of activities A to L with the following precedence relationships:
(a) A,B and C, the first activities of the project can be executed concurrently
(b) A & B precede D
(c) B precedes E,F,H
(d) F and C precede G
(e) E and H precede I & J
(f) C,D,F and J precede K
(g) K precede L
(h) I, G, and L are terminal activities of the project.

[Hint: Network diagram of this question is same as that of Question 16]


Question 6 [Difficult network-Try it at only after proper practice]: Construct the project network
comprised of activities A to P that satisfies the following precedence relationships:
(a) A,B and C, the first activities of the project can be executed concurrently
(b) D,E and F follow A
(c) I and G follow both B and D
(d) H follows both C & G
(e) K and L follow I
(f) J succeeds both E and H
(g) M and N succeed F, but cannot start until both E and H are completed.
(h) O succeeds both M and I
(i) P succeeds J,L and O
(j) K,N and P are the terminal activities of the project.
Question 7: A publisher has a contract with an author to publish a textbook. The simplified (activities)
associated with the production of the textbook are given below. The author is required to submit to the
publisher a hard copy and a computer file of the manuscript. Develop the associated network for the project.

Activity Predecessor(s) Duration (weeks)


A: Manuscript proofreading by editor - 3
B: Sample pages preparation - 2
C: Book cover design - 4
D: Artwork preparation - 3
E: Author’s approval of edited manuscript and sample pages A,B 2
F: Book formatting E 4
G: Author’ review of formatted pages F 2
H: Author’s review of artwork D 1
I: Production of printing plates G,H 2
J: Book production and binding C,I 4

CA. Parag Gupta Ph.: +91 9891 432 632 Paraggupta_ca@yahoo.co.in Costing & O.R.
World’s largest CA Final student’s consultancy group: http://groups.yahoo.com/group/costingbyparaggupta
Network Analysis - PERT / CPM - 318 -
(including resource allocation & resource leveling)

Question 8: A project consists of a series of tasks labeled A, B,.……...., H, I with the following relationships
(W < X,Y means X and Y cannot start until W is completed; X,Y < W means W cannot start until both X and Y
are completed). With this notation construct the network diagram having the following constraints:
A < D,E; B,D < F; C < G; B < H; F,G < I
Question 9 (Concurrent Activities-LEAST IMPORTANT-Do after proper practice of network diagrams):
The footing of a building can be completed in four consecutive sections. The activities for each section include
(1) digging, (2) placing steel, and (3) pouring concrete. The digging of one section cannot start until that of the
preceding section has been completed. The same restriction applies placing steel & pouring concrete.
Develop the project network.

Numbering the Events (Fulkerson’s Rule)


1. The initial event which has all outgoing arrows with no incoming arrow is numbered “1”.
2. Delete all the arrows coming out from node “1”. This will convert some more nodes into initial events.
Number these events as 2, 3, 4, ….
3. Delete all the arrows going out from these numbered events to create more initial events. Assign the
next numbers to these events.
4. Continue until the final or terminal node, which has all arrows coming in with no arrow going out is
numbered.

Determination of time to complete each activity:


The CPM system of networks omits the probabilistic consideration and is based on a Single Time Estimate of
the average time required to execute the activity.

In PERT analysis, there is always a great deal of uncertainty associated with the activity durations of any
project. Therefore, t e estimated time is better described by a probability distribution than by a single estimate.
Three time estimates (from beta probability distribution) are made as follows:
1) The Optimistic Time Estimate (t o ): Shortest possible time in which an activity can be completed in
ideal conditions. No provisions are made for delays or setbacks while estimating this time.
2) The Most Likely Time (t m ): It assumes that things go in normal way with few setbacks.
3) The Pessimistic Time (t p ): The max. possible time if everything go wrong & abnormal situations
prevailed. However, major catastrophes such as earthquakes, labour troubles, etc. are not taken into
account.
The expected time (mean time) for each activity can be approximated using the weighted average i.e.

t o  4tm  t p
Expected Time (t e ) =
6

Forward Pass Computation: It is the process of tracing the network from START to END. It gives the earliest
start & finish times for each activity.
Earliest event time (E j ): The time that event j will occur if the preceding activities are started as early as
possible. E j is the max. of the sums E i + t ij involving each immediately precedent event i & intervening event
ij.

Backward Pass Computation: It is the process of tracing the network starting from LAST node & moving
backward.
Latest event time (L j ): The latest time that event i can occur without delaying completion of project beyond its
earliest time. L i is the min. of the differences L i - t ij involving each immediately precedent event j &
intervening event ij.
E.g. of Earliest event time & Latest event time:

Activity: 1-2 1-3 2-3 2-5 3-4 3-6 4-5 4-6 5-6 6-7
Duration (weeks): 15 15 3 5 8 12 1 14 3 14

CA. Parag Gupta Ph.: +91 11 47665555 Paraggupta_ca@yahoo.co.in Costing & O.R.
World’s largest CA Final student’s consultancy group: http://groups.yahoo.com/group/costingbyparaggupta
Operations Research - 319 -

 In accordance with Zero Slack Convention, if no schedule date for completion of the project is
specified, then we take L=E for the terminal event of the project.
 It is a convention to keep the earliest allowance time of the START event as zero.
 Flexibility of non critical in case of event is known as slack & in case of activity is term as
float.(Though some writers have used these terms interchangeably).

The critical path can be identified by determining the following four parameters for each activity:
 EST - earliest start time: the earliest time at which the activity can start given that all its precedent
activities must be completed first = E i
 EFT - earliest finish time, equal to the earliest start time for the activity plus the time required to
complete the activity = EST(i-j) + t ij
 LFT - latest finish time: the latest time at which the activity can be completed without delaying
(beyond its targeted completion time) the project = L j
 LST - latest start time, equal to the latest finish time minus the time required to complete the activity =
LFT(i-j) - t ij

CRITICAL PATH: The critical path is the path through the project network in which none of the activities
have float (total float is zero) i.e. A critical path satisfies following 3 conditions:
 EST =LST
 EFT=LFT
 E j – E i = L j – L i = t ij

The duration of project is fixed by the time taken to complete the path through the network with the greatest
total duration. This path is known as critical path & activities on it are known as critical activities. A delay in the
critical path delays the project. Similarly, to accelerate the project it is necessary to reduce the total time
required for the activities in the critical path.

Dummy Activity may or may not lie on critical path.

Question 10 (Critical Path): Tasks A, B, C,….., H, I constitute a project. The precedence relationships are
A<D; A<E; B<F; D<F; C<G; C<H; F<I; G<I
Draw a network to represent the project and find the minimum time of completion of the project when time, in
days, of each task is as follows:

Task: A B C D E F G H I
Time: 8 10 8 10 16 17 18 14 9
Also identify the critical path.
[Ans.: Critical path is 1-2-4-5-6 with 44 days]

Question 11: At the end of activity 6-7, a product is to be launched and the date has been announced for the
inaugural function, based on the normal duration of activities as given in the network below. Activities have

CA. Parag Gupta Ph.: +91 9891 432 632 Paraggupta_ca@yahoo.co.in Costing & O.R.
World’s largest CA Final student’s consultancy group: http://groups.yahoo.com/group/costingbyparaggupta
Network Analysis - PERT / CPM - 320 -
(including resource allocation & resource leveling)

been subcontracted by the project manager to contractors A, B, C, D, E, F, G and H as indicated in the table
below. Each subcontractor offers a discount on his contract price for each day given to him in addition to the
normal days indicated in the network. What will be the maximum discount that the project manager may earn
for the company without delaying the launch of the product?

Activity Contractor Discount (`)/ Day


1–2 A 300
1–3 B 200
1–4 C 1,200
2–5 D 500
3–5 E 400
4–6 F 1,000
5–6 G 600
6–7 H 500

[Ans.: Maximum discount, without delaying the launch of the product, will be of `2600 on extending activity 2-
5 by 2 days & 3-5 by 4 days] (7 Marks) Nov./10-O.C.

Question 12: A project consists of seven activities for which relevant data are given below:
(i) Draw the network (5 Marks)
(ii) Name and highlight the critical path. (1 Mark) Nov./06& Nov/96[Adapted]

Activity Preceding Activity Activity Duration (Days)


A - 4
B - 7
C - 6
D A,B 5
E A,B 7
F C,D,E 6
G C,D,E 5

[Ans.: B, E, F = 20 days]
Question 13 (Forward and Backward Pass): A project schedule has the following characteristics:

Activity Time (weeks) Activity Times (week)


1-2 4 5-6 4
1-3 1 5-7 8
2-4 1 6-8 1
3-4 1 7-8 2
3-5 6 8-9 1
4-9 5 8-10 8
9-10 7

(i) Construct the PERT network


(ii) Compute E and L for each event;
(iii) Float for each activity; and
(iii) Find critical path and its duration. (10 Marks) May/2000
[Note: Float is to be calculated only after going through below text]

[Ans.: Critical path is 1-3-5-7-8-10 with 25 weeks]

CA. Parag Gupta Ph.: +91 11 47665555 Paraggupta_ca@yahoo.co.in Costing & O.R.
World’s largest CA Final student’s consultancy group: http://groups.yahoo.com/group/costingbyparaggupta
Operations Research - 321 -

The total float time for an activity is the time between its earliest and latest start time, or between its earliest
and latest finish time. It is the amount of time that an activity can be delayed past its earliest start or earliest
finish without delaying the project. = LST-EST or LFT-EFT = LFT-EST-t ij = LFT- (EST+t ij )

The slack time or slack of an event in a network is the difference the latest event time & earliest event time
i.e. L i -E i

The free float time of an activity is equal to the amount by which its duration can be increased without
affecting either the project time or the time available for the subsequent activities. It indicates the value by
which an activity can be delayed beyond the earliest starting point without affecting the earliest start, &
therefore, the total float of the activities following it. = Total Float ij – (Slack of event j)

The independent float time of an activity is the amount by which the duration of an activity could be extended
without affecting the total project time, the time available for subsequent activities or the time available for the
preceding activities. = [Free Float ij – (Slack of event i)] or ZERO, whichever is higher. Also EST of
following activity – LFT of preceding activity – Duration of current activity or Zero, whichever is higher.

The interfering float time is the part of total float which causes a reduction in the float of successor activities.
It is that portion of the activity float which cannot be consumed without affecting adversely the float of the
subsequent activity or activities. = LFT – (EST of following activity) or ZERO, whichever is higher.

While calculating floats, for just for our simplifying computations, we can write values of Slack of event j
in column wherein we are supposed to write interfering float.

Negative Float in a PERT Network: In a CPM network, it is assumed that L=E at the final event but in PERT
network, there may be a scheduled date at which a project is expected to complete. This value of the
scheduled time, Ts, is often taken as the time for the final event.
When L < E, a negative float results.
In such cases, critical activity is not specified by the condition of zero float along the path. The critical path is
the path of least float for such kind of PERT network in which backward pass is based on the scheduled date.

Subcritical Activity: Activity having next higher float than the critical activity.

Supercritical Activity: These Activities have negative float. It results when activity duration is more than time
available. It indicates abnormal situation requiring as to how to compress the activity.

Subcritical path: The path with the next least floats than critical path is subcritical path.

Float Calculation

Event Oriented Activity Oriented


Here Activities are shown as i-j, where i is tail Here Activities are generally shown via
event & j is head event e.g. 1-2, 2-3, etc. here alphabets e.g. A, B, C, etc.
activity is between 1-2 though starting/tail
event(i) is 1 & head/ending event(j) is 2.

Question 14 (Floats): The utility data for a network are given below. Determine the total, free, independent
and interfering floats and identify the critical path.
Activity: 0-1 1-2 1-3 2-4 2-5 3-4 3-6 4-7 5-7 6-7
Duration: 2 8 10 6 3 3 7 5 2 8
[Ans.: Critical Path is 0-1-3-6-7 with 27]

CA. Parag Gupta Ph.: +91 9891 432 632 Paraggupta_ca@yahoo.co.in Costing & O.R.
World’s largest CA Final student’s consultancy group: http://groups.yahoo.com/group/costingbyparaggupta
Network Analysis - PERT / CPM - 322 -
(including resource allocation & resource leveling)

Question 15: For the network given below, compute E and L for each event & determine the total, free,
independent and interfering floats and identify the critical path.

Question 16: The following table gives the activities in a construction project and the time duration of each
activity:
Activity Preceding activity Normal Time (Days)
A - 16
B - 20
C A 8
D A 10
E B, C 6
F D, E 12
Required:
(i) Draw the activity network of the project.
(ii) Find critical path.
(iii) Find the total float and free-float for each activity. (6 Marks) Nov/07
[Ans.: (ii) A-C-E-F = 42 days.(iii) Total Float A-0, B-4, C-0, D-4, E-0, F-0; Free Float A-0, B-4, C-0, D-4, E-0,
F-0]
Question 17: Given is the following information regarding a project:

Activity A B C D E F G H I J K L
Dependence - - - AB B B FC B EH EH CDFJ K
Duration (days) 3 4 2 5 1 3 6 4 4 2 1 5

Draw the Network Diagram and identify the Critical Path and Project Duration. (12 Marks) Nov./94
Find the three types of float (viz. Total, Free and Independent) for each activity.
[Ans.: B-H-J-K-L = 16 days]

Probability Estimate: It is used to calculate the probability of completing the time within given
duration (Using Normal Distribution):
T1 – Tcp
Z=
t
Where, Z = Standard Normal Variate
T 1 = Duration in which we wish to complete the project
T cp = Duration on critical path
σ t = Standard Deviation of the earliest finish of network = Square root of sum of variance of all
activity durations of critical path, where
2
 tp – t o 
(σ2t)
Variance Distribution, =  
 6 
Question 18: If the critical path of a project is 20 months alongwith a standard deviation of 4 months, what is
the probability that the project will be completed within: (a) 20 months (b) 18 months (c) 24 months?
[Ans.: 0.50, 0.31, 0.84]
Question 19: PERT calculation yield a project length of 60 weeks with variance of 9. Within how many
weeks would you expect the project to be completed with probability of 0.99? (That is the project length that
you would expect to be exceeded only by 1% of time if the project were repeated many time in an identical
manner).
[Ans.: 67 weeks]

CA. Parag Gupta Ph.: +91 11 47665555 Paraggupta_ca@yahoo.co.in Costing & O.R.
World’s largest CA Final student’s consultancy group: http://groups.yahoo.com/group/costingbyparaggupta
Operations Research - 323 -

Question 20 [Calculation of variance in 2 critical paths]: Consider the network shown below. The three
time estimates for the activities are given along the arrows. Determine the critical path. What is the probability
that the project will be completed in 20 days?

[Ans.: 0.6844]

 In case there are two critical paths, variance of separate activities of both of them shall be added for
calculating σ t , but for calculating Z, we will take higher of two σ t taken above (As in above question).
 In case of event variance, if there are two longest paths, higher of the two would picked up.

Question 21: Consider the schedule of activities and related information as given below, for the construction
of a plant:

Activity Expected Time Variance Expected Cost


(Months) (Millions of `)
1-2 4 1 5
2-3 2 1 3
3-6 3 1 4
2-4 6 2 9
1-5 2 1 2
5-6 5 1 12
4-6 9 5 20
5-7 7 8 7
7-8 10 16 14
6-8 1 1 4

Assuming that the cost and time required for one activity is independent of the time an cost of any other
activity are expected to follow normal distribution.
Draw a network based on the above data and calculate:
(a) Critical path
(b) Expected cost of construction of the plant.
(c) Expected time required to build the plant.
(d) The standard deviation of the expected time. (10 Marks) May/01

[Ans.: (a) 1-2-4-6-8; (b) `80 millions; (c) 20 months; (d) 3 months]
Question 22: The time estimate (in weeks) for the activities of a PERT network are given below:

Activity to tm tp
1-2 1 1 7
1-3 1 4 7
1-4 2 2 8
2-5 1 1 1
3-5 2 5 14
4-6 2 5 8
5-6 3 6 15

(a) Draw the project network and identify all the paths through it.
(b) Determine the expected project length.
(c) Calculate the standard deviation and variance of the project length.
(d) What is the probability that the project will be completed.
CA. Parag Gupta Ph.: +91 9891 432 632 Paraggupta_ca@yahoo.co.in Costing & O.R.
World’s largest CA Final student’s consultancy group: http://groups.yahoo.com/group/costingbyparaggupta
Network Analysis - PERT / CPM - 324 -
(including resource allocation & resource leveling)

1. at least 4 weeks earlier than expected time?


2. no more that 4 weeks later than expected time?
(e) If the project due date is 19 weeks, what is the probability of not meeting the due date?
(f) The probability than the project will be completed on schedule if the scheduled completion time is 20
weeks.
(g) What should be the scheduled completion time for the probability of completion to be 90%? (Nov./91)

[Ans.: (c) Variance = 9 & Standard Deviation = 3; (d) (i) 0.0918 (ii) 0.9082 (e) 0.2514 (f) 0.8413 (g) 20.84]
Question 23: A company is launching a new product and has made estimates of the time for the various
activities associated with the launch as follows:

Times (Days)
Activity Predecessor Optimistic Most Likely Pessimistic
A NONE 1 3 5
B NONE 3 4 5
C A,B 1 3 11
D B 3 3 9
E A 1 2 3
F C 2 5 14
G E,F 2 3 4
H D,F 2 2 2
I G,H 10 10 10

Required :
i) Draw the network diagram.
ii) Calculate the expected time and variance of each activity.
iii) Find out the expected length of critical path and its standard deviation.
iv) Find the probability that the launching will be completed in 27 days.
v) Find the duration, which has 95% probability of completion. (8 Marks) Nov./09-O.C.

[Ans.: (iii) Critical Path is B-C-F-G-I with expected project duration of 27 days & standard deviation of 2.646
days; (iv) 50% (v) 31 days approx.]
[Note: Answer suggested by ICAI for this question consist myriads of errors.]
Question 24: Given the following project network, determine:
1. Earliest expected completion time for each event
2. Latest allowable completion time for each event
3. Slack time for each event
4. Critical Path
5. The probability that project will be completed on schedule, if scheduled completion time is 38

(8 Marks) Nov./04
[Hint: Critical Path is 1-2-4-5-7 and probability = 7.93%]

CA. Parag Gupta Ph.: +91 11 47665555 Paraggupta_ca@yahoo.co.in Costing & O.R.
World’s largest CA Final student’s consultancy group: http://groups.yahoo.com/group/costingbyparaggupta
Operations Research - 325 -

Question 25: A project consists of seven activities and the time estimates of the activities are furnished as
under:

Activity Optimistic Most likely Pessimistic


Days Days Days
1-2 4 10 16
1-3 3 6 9
1-4 4 7 16
2-5 5 5 5
3-5 8 11 32
4-6 4 10 16
5-6 2 5 8

Required:
(i) Draw the network diagram.
(ii) Identify the critical path and its duration.
(iii) What is the probability that project will be completed in 5 days earlier than the critical path
duration?
(iv) What project duration will provide 95% confidence level of completion (Z 0.95 =1.65)?
Given (11 Marks) Nov/08-O.C.

Z 1.00 1.09 1.18 1.25 1.33


Probability 0.1587 0.1379 0.1190 0.1056 0.0918

[Ans.: (ii)Critical Path is 1→3→5→6 & its duration is 25 days (iii) Probability = 11.90%, (iv) 32 days (approx)]
Remember, while doing questions like above,
Hence for calculating probability

whenever we have been


provided with a normal
table as an extra sheet
in examination for
[generally a green We would check value of z (e.g.
colour sheet], we would 1.18) in the extra sheet table and
calculate probability, find probability (i.e. 0.3810), now
treating that probability we will reduce this value from 0.5
has been provided for to get the probability of the
abovesaid region (i.e. 0.5-0.3810 =
0.1190)
But, if in spite of extra sheet of normal table, value of z has been provided as a part of question itself,
(as it was in above question)
Hence for calculating probability

If probability is
increasing with increase
for
in value of z but all
We would check value of z (e.g.
probabilities are less
1.18) in the values given with the
than 0.5, then area has
question and find probability (i.e.
been provided for
0.3810), now we will reduce this
value from 0.5 to get the
probability of the abovesaid region
(i.e. 0.5-0.3810 = 0.1190)

CA. Parag Gupta Ph.: +91 9891 432 632 Paraggupta_ca@yahoo.co.in Costing & O.R.
World’s largest CA Final student’s consultancy group: http://groups.yahoo.com/group/costingbyparaggupta
Network Analysis - PERT / CPM - 326 -
(including resource allocation & resource leveling)

Hence for calculating probability


we would first reduce value of
If probability is probability by 0.5 to make get
increasing with increase probability for
in value of z but all
probabilities are greater
than 0.5, then area has
been provided for
Now, we will do the same steps as
done in above case.
Hence for probability for

But if probability is is
decreasing with always equal to probability for
increase in value of z
(as in above question),
then area has been
provided for

We will simply check value of z (i.e.


1.18) in the value in the question
and its probability itself will be
our answer i.e. 0.1190)

Question 26: A project consists of the following activities, whose time estimates are given against each as
under:

Estimated duration (weeks)


Activity Optimistic Most likely Pessimistic
1-2 3 6 15
1-3 2 5 14
1-4 6 12 30
2-5 2 5 8
2-6 5 11 17
3-6 3 6 15
4-7 3 9 27
5-7 1 4 7
6-7 4 19 28
Required :

(i) Draw the project net work.


(ii) Find the expected duration and variance of each activity.
(iii) Determine the critical path and the expected project duration.
(iv) What is the probability that the project will be completed in 38 weeks?
(v) What project duration will have 95% chance of completion. (Z 0.95 = 1.65)

Given : Z 0.21 0.41 0.82 (8 Marks) May/03


0.0832 0.1591 0.2939

[Ans.: (iii) Critical path 1→2→6→7, Expected project duration is 36 weeks. (iv) 66% (v) 44 weeks]

Question 27: An Engineering Project has the following activities, whose time estimates are listed below:

CA. Parag Gupta Ph.: +91 11 47665555 Paraggupta_ca@yahoo.co.in Costing & O.R.
World’s largest CA Final student’s consultancy group: http://groups.yahoo.com/group/costingbyparaggupta
Operations Research - 327 -

Activity Estimated Duration (in months)


(i-j) Optimistic Most Likely Pessimistic
1-2 2 2 14
1-3 2 8 14
1-4 4 4 16
2-5 2 2 2
3-5 4 10 28
4-6 4 10 16
5-6 6 12 30

(a) Draw the project network and find the critical path.
(b) Find the expected duration and variance for each activity. What is the expected project length?
(c) Calculate the variance and standard deviation of the project length.
(d) What is the probability that the project will be completed at least eight months earlier than expected
time?
(e) If the project due date is 38 months, what is the probability of not meeting the due date?

Given:
z 0.50 0.67 1.00 1.33 2.00
P 0.3085 0.2514 0.1587 0.0918 0.0228

[Ans.: (i) Critical path 1-3-5-6; (ii) Expected project length 34 months; (iii) Variance 36 months, Standard
Deviation 6; (iv) 9.18%; (v) 25.14%] (4 Marks)May/10-NC[(i) & (ii) part],(10 Marks)Nov./01 & (7 Marks)Nov/05
Question 28: The following information is given:

Activity (1-2) (2-3) (2-4) (3-5) (4-6) (5-6) (5-7) (6-7)


Pessimistic time (in weeks) 3 9 6 8 8 0 5 8
Most likely time (in weeks) 3 6 4 6 6 0 4 5
Optimistic time (in weeks) 3 3 2 4 4 0 3 2

Draw the Network diagram for the above. Calculate:


1) Variance of each activity.
2) Critical path and expected project length.
3) The probability that the project will be completed in 23 weeks.

Given that:
Z value: 1.90 1.91 1.92 1.93 1.94
Probability: 0.9713 0.9719 0.9726 0.9732 0.9738

[Ans.: (i) 0, 1, 4/9, 4/9, 4/9, 0, 1/9, 1; (ii) 20 weeks; (iii) 97.26%] (10 Marks) May/98
Question 29: A civil engineering firm has to bid for the construction of a dam. The activities and their time
estimates are given below:

Activity Optimistic Most likely Pessimistic


1-2 14 17 25
2-3 14 18 21
2-4 13 15 18
2-8 16 19 28
3-4 (dummy) 0 0 0
3-5 15 18 27
4-6 13 17 21
5-7 (dummy) 0 0 0
5-9 14 18 20
6-7 (dummy) 0 0 0
6-8 (dummy) 0 0 0
7-9 16 20 41
8-9 14 16 22

CA. Parag Gupta Ph.: +91 9891 432 632 Paraggupta_ca@yahoo.co.in Costing & O.R.
World’s largest CA Final student’s consultancy group: http://groups.yahoo.com/group/costingbyparaggupta
Network Analysis - PERT / CPM - 328 -
(including resource allocation & resource leveling)

The policy of the firm with respect to submitting bids is to bid the minimum amount that will provide a 95% of
probability of at best breaking-even. The fixed costs for the project are eight lakhs and the variable costs are
9000 every day spent working on the project. The duration is in days and the costs are in rupees.
What amount should the firm bid under this policy? (You may perform the calculations on duration etc., up to
two decimal places) (May/90)

[Ans.: `1574000]
Question 30 [Negative float-LEAST IMPORTANT]: The optimistic, most likely and pessimistic times of the
activities of a project are given below. Activity 40-50 must not start before 22 days, while activity 70-90 must
end by 35 days. The scheduled completion time of the project is 46 days. Draw the network and determine
the critical path. What is the probability of completing the project in scheduled time?

Activity t o -t m -t p Activity t o -t m -t p
10-20 4-8-12 50-70 3-6-9
20-30 1-4-7 50-80 4-6-8
20-40 8-12-16 60-100 4-6-8
30-50 3-5-7 70-90 4-8-12
40-50 0-0-0 80-90 2-5-8
40-60 3-6-9 90-100 4-10-16

[Ans.: 10-20-40-50-70-90-100 = 46 days; Probability is 50%]


Question 31 [Probability at non-critical event- Calculation for this question is very tedious]: A PERT
network is shown below. The activity times in days are given along with the arrows. The scheduled times for
some important events are given along the nodes. Determine the critical path and probabilities of meeting the
scheduled dates for the specified events. Tabulate the results and determine slack for each event.

[Ans.: Critical Path is 1-3-4-8-11-12 with project completion time as 26.51 days, Probability of completing the
project in the scheduled completion time of 24 days is 29.54%, Probability that event 3 will occur on
scheduled date is 20.44% & Probability of meeting schedule date of event 5 is less than or equal to 54.89%
with minimum of 1.8%]
Question 32: Distinction between PERT and CPM (5 Marks) Nov/98 & (5 Marks) Nov/07

PERT CPM
1. PERT is used for non-repetitive jobs like planning 1. CPM is used for repetitive job like building a house
the assembly of the space. 2. It is a deterministic model.
2. it is a probabilistic model. 3. It is activity-oriented as the result or calculations
3. It is event-oriented as the results of analysis are are considered in terms of activities or operations of
expressed in terms of events or distinct points in time the project.
indicative of progress. 4. It is applied mainly for construction and business
4. It is applied mainly for planning and scheduling problems.
research programmes. 5. CPM does not incorporate statistical analysis in
5. PERT incorporates statistical analysis and thereby determining time estimates, because time is precise
determines the probabilities concerning the time by and known.
which each activity or entire project would be 6. It is difficult to use CPM as a control device for the
CA. Parag Gupta Ph.: +91 11 47665555 Paraggupta_ca@yahoo.co.in Costing & O.R.
World’s largest CA Final student’s consultancy group: http://groups.yahoo.com/group/costingbyparaggupta
Operations Research - 329 -

completed. simple reason that one must repeat the entire


6. PERT serves as useful control device as it assists evaluation of the project each time the changes are
management in controlling a project by calling introduced into the network.
attention to such delays

Question 33: A Project Manager has to manage various projects. For each project given below, you are
required to advise him whether to use PERT or CPM and briefly state the reason:
(i) Project K is yet to begin. The manager has recently successfully handled similar projects. He is able to
break down the project into smaller modules and knows when he may comfortably finish each module.
(ii) Project L has been sanctioned some fixed amount. Though the manager is familiar about what time it will
take, he expects pressure towards the end to finish the project slightly earlier, by deploying additional
resources of the company.
(iii) Project M is new to the manager. He has never handled such a project. He can break up the project into
smaller modules, but even then, he is not sure of their exact times.
(iv) Project N has a limitation on the skilled workforce available. But the manager knows from earlier
experience, the slack on each event in the project. He is confident of handling the bottleneck of labour.
(v) Project O is a research project, bound to produce immense benefit to the company in future.

[Ans.: (i) CPM; (ii) CPM; (iii) PERT; (iv) CPM; (v) PERT] (5 Marks) May/10-O.C.

Project Crashing
There are usually compelling reasons to complete the project earlier than the originally estimated duration of
critical path computed on the normal basis of a new project.
Direct Cost: This is the cost of the materials, equipment and labour required to perform the activity. When the
time duration is reduced the project direct cost increases.
Indirect Cost: It consists of two parts: fixed cost and variable cost. The fixed cost is due to general and
administrative expenses, insurance, etc. Variable indirect cost consists of supervision, interest on capital, etc.

The total project cost is the sum of the direct & the indirect costs.
Optimum duration is the project duration at which total project cost is lowest.

Steps of Project Crashing:

Initialization Steps

Step 1: Prepare Slope table and calculate maximum possible reduction for every activity & cost slope (i.e.
crashing cost per unit time) for each activity according to the following formulas:
C  Nc
Activity Cost Slope = c
Nt  Ct
Where, C c = Crash Cost = Direct cost that is anticipated in completing an activity within crash time.
N c = Normal Cost = This is the lowest possible direct cost required to complete an activity
N t = Normal Time = Min. time required to complete an activity at normal cost.
C t = Crash Time = Min. time required to complete an activity.
Maximum Possible reduction (i.e. the number of time periods of crashing availability) = N t - C t

Step 2: Enumerate all the paths through the project network, and list them with their normal time durations in
the path table. Identify the critical path(s) as those with longest duration, and mark the critical path(s) in
Decision table.
Step 3: Identify the normal project duration, the normal project cost and the normal critical path in the first row
of the cost table, labeled as iteration 0.

Iteration Steps

CA. Parag Gupta Ph.: +91 9891 432 632 Paraggupta_ca@yahoo.co.in Costing & O.R.
World’s largest CA Final student’s consultancy group: http://groups.yahoo.com/group/costingbyparaggupta
Network Analysis - PERT / CPM - 330 -
(including resource allocation & resource leveling)

Step 4: Select that subset of critical activities which, when compressed in parallel, enable all current critical
paths to become shorter, and do so at the least group cost slope, where the group cost slope for a subset of
critical activities is the sum of the cost slope for activities in the group.

Step 5: Crash the selected critical activities until one or both of the following two conditions occurs: (i) one (or
more) of the crashed activities becomes fully crashed (i.e. is reduced to crash time); or (ii) a new path
becomes critical.

Step 6: Record the selected activities, number of time periods compressed, the new project duration, the
group cost slope for the selected activities, the added cost resulting from the compression, the new total direct
cost, and the new critical path (if any) as items in the decision table & cost table for this iteration. Update the
slope table and the path table to reflect the reduction in path lengths resulting from the selected crashing.

Step 7: Repeat steps 4 through 6 until all activities on some (any) critical path become fully crashed. At this
point the Decision table is complete, as no further time reduction is possible. Plot the time-cost trade-off graph
by linear interpolation between the time/cost pairs which occur in each row of the cost table.

If question has asked to calculate optimum duration & not minimum duration, we will perform crashing
only up to the iteration where direct cost slope is equal to indirect cost slope (because at this point total
cost will be minimum).

Question 34: The following network gives the duration in days for each activity:

(i) You are required to list the critical paths.


(ii) Given that each activity can be crashed by a maximum of one day, choose to crash any four activities
so that the project duration is reduced by 2 days. (6 Marks) Nov./09-N.C.

[Ans.: (i) 1-2-5-6; 1-3-5-6; 1-3-4-5-6; 1-4-5-6 with 15 days; (ii) Either 1-2, 1-3, 1-4, 5-6 or 1-2, 3-5, 4-5, 5-6 or
1-3, 1-4, 2-5, 5-6 or 2-5, 3-5, 4-5, 5-6]
Question 35: A network is given below:
(i) Name the paths and give their total duration. (3 Marks)
(ii) Give three different ways of reducing the project above duration by four days. (3 marks) Nov./06

[Ans.: Paths and their duration are 1-2-5-8-9; 1-3-4-7-8-9; 1-3-4-6-7-8-9; 1-3-5-8-9 with 15 weeks]
[Hint: Time duration of activity 3-5 was missing in question paper, so best way to assume its duration as
exactly 4 weeks, because in that case all paths will become critical paths. You may assume other time
durations as well.]
CA. Parag Gupta Ph.: +91 11 47665555 Paraggupta_ca@yahoo.co.in Costing & O.R.
World’s largest CA Final student’s consultancy group: http://groups.yahoo.com/group/costingbyparaggupta
Operations Research - 331 -

Question 36: A project is composed of seven activities as per the details given below:

Activity Normal Time Crash Time Normal Cost Crash Cost


(Days) (Days) (`) (`)
1-2 4 3 1500 2000
1-3 2 2 1000 1000
1-4 5 4 1875 2250
2-3 7 5 1000 1500
2-5 7 6 2000 2500
3-5 2 1 1250 1625
4-5 5 4 1500 2125

Indirect cost per day of the project is `500.


Required:
(a) Draw the project network.
(b) Determine the critical path and its duration.
(c) Find the optimum duration and the resultant cost of the project. (8 Marks) May/04

[Ans.: (b) Longest path is 13 days; (c) Optimal project duration of 10 days.]

Question 37: The normal time, crash time and crashing cost per day are given for the following network:

Activity Normal Time Crash Time Crashing Cost


(Days) (Days) (`/day)
1-2 18 14 40
1-3 23 22 20
2-3 8 5 60
2-4 10 6 40
3-4 3 2 80
4-5 8 6 50

(i) Crash the project duration in steps and arrive at the minimum duration. What will be the critical path and
the cost of crashing?
(ii) If there is an indirect cost of `70 per day, what will be the optimal project duration and the cost of
crashing?

[Ans.: (i) Min. Duration is 30 days & total cost of crashing is `360 (ii) Optimal Duration is 31 days & cost of
crashing is `280] (10 Marks) Nov./08-N.C.
Question 38: A small project consists of jobs as given in the table below. Each job is listed with its normal
time and a minimum or crash time (in days). The cost (in `per day) for each job is also given:

Job (i-j) Normal Duration (in Minimum (crash) Cost of Crashing


days) Duration (in days) (`per day)
1-2 9 6 20
1-3 8 5 25
1-4 15 10 30
2-4 5 3 10
CA. Parag Gupta Ph.: +91 9891 432 632 Paraggupta_ca@yahoo.co.in Costing & O.R.
World’s largest CA Final student’s consultancy group: http://groups.yahoo.com/group/costingbyparaggupta
Network Analysis - PERT / CPM - 332 -
(including resource allocation & resource leveling)

3-4 10 6 15
4-5 2 1 40

(i) What is the normal project length and the minimum project length?
(ii) Determine the minimum crashing cost of schedules ranging from normal length down to, and
including, the minimum length schedule. That is, if L = Length of the schedule, find the costs of schedules
which are L, L-1, L-2 and so on.
(iii) Overhead costs total `60 per day. What is the optimum length schedule in terms of both crashing and
overhead cost? List the schedule duration of each job for your solution. (10 Marks) May/02

[Ans.: (i) Critical path is 1→3→4→5, Normal Project length = 20 days, Minimum Project Length = 12 days; (ii)
Optimum total costs = `1030; (iii) Optimum duration of the project is 15 days.]
Question 39: The following table gives data on normal time and cost and crash time and cost for a project.
(a) Draw the network and identify the critical path.
(b) What is the normal project duration and associated cost?
(c) Find out total float for each activity.
(d) Crash the relevant activities systematically and determine the optimum project time and cost.

Activity Normal Crash


Time Cost Time Cost
(Week) (`) (Week) (`)
1-2 3 300 2 400
2-3 3 30 3 30
2-4 7 420 5 580
2-5 9 720 7 810
3-5 5 250 4 300
4-5 0 0 0 0
5-6 6 320 4 410
6-7 4 400 3 470
6-8 13 780 10 900
7-8 10 1000 9 1200
4220
Indirect costs are `50 per week.

[Ans.: (b) Normal Project Duration is 32 weeks with cost of `5820; (d) Optimum Project Duration is 29 weeks
with cost of `5805] (12 Marks) June/09 [Old Course-Adapted] & ICWA Final (Dec-1988)
Question 40: A small project is having seven activities. The relevant data about these activities is given
below:

Activity Dependence Normal Crash Normal cost Crash Cost


duration duration (`) (`)
(Days) (Days)
A - 7 5 500 900
B A 4 2 400 600
C A 5 5 500 500
D A 6 4 800 1000
E B,C 7 4 700 1000
F C,D 5 2 800 1400
G E,F 6 4 800 1600

(i) Find out the normal duration and the minimum duration.
(ii) What is the percentage increase in cost to complete the project in 21 days? (10 Marks) Nov./97

[Ans.: (i) Normal duration 25 days, minimum duration 18 days; (ii) 15.5%]

CA. Parag Gupta Ph.: +91 11 47665555 Paraggupta_ca@yahoo.co.in Costing & O.R.
World’s largest CA Final student’s consultancy group: http://groups.yahoo.com/group/costingbyparaggupta
Operations Research - 333 -

Miscellaneous Topics (CPM and PERT)

Question 41: Write short notes on resource smoothing and resource leveling.
(5 Marks each) May/99, May/00, May/02, Nov./02, May/05

Resource smoothing is a resource scheduling technique used for smoothing peak resource requirement
during different periods of project network. Under this technique, the constraint may be the total project
duration. It helps to estimate the resource requirements for various projects. In resource smoothing, time
scaled diagram of various activities of project and their floats along with their resource requirements are used.
Floats on non critical activities are utilized & these activities are rescheduled or shifted (while the project
duration remains unchanged) so that a uniform demand on resources is achieved.

Resource Levelling (a.k.a. resource allocation) is an operation of resource scheduling wherein constraint may
be availability of certain resources. Here project time is varied for maximum utilization of resources i.e. project
duration is not treated as an invariant, but the demand on certain specified resources should not go beyond a
specified level. The maximum demand of a resource should not exceed the available limit at any point of time.
Non critical activities are rescheduled by utilizing their floats.

Points which are worth noting for developing the algorithm for resource allocation:
1. Halt when both resources and activities are available.
2. Prior to allocation at a halt, up date the E.S.T., E.F.T. and float of the activities not allocated at earlier
halt time and their succeeding activities. The repercussions may have to be traced right up to the last
event.
3. (a) priorities are assigned on the basis of floats e.g. 1st priority to activity with least float, 2nd to the
activity with the next higher float and so on.
(b) In case of tie in floats, assign priorities on the basis of man-days if the activities e.g., 1st priority to
the activity with highest M × D.
(c) In case of tie in man-days even, assign 1st priority to the activity with highest M (gang size).
(d) In case of tie in M’s even, assign 1st priority to the activity with lower i, where i is the tail event
number of the activity.
4. When an activity requires more than one man, it may so happen during allocation that the activity
requires more number of persons than that available at the halt time under consideration. In such
cases, the resources are allocated to the job with next priority for which they are sufficient.
5. During the floating out of activities, the float of an activity may go negative which means that the
project duration is going to be extended beyond the critical path. Once the float of an activity
becomes negative, there from the float criterion for ascertaining priorities is invalidated. The priorities
are then fixed on the basis of M x D, gang size and lower i criteria respectively.

Question 42: The Madras Construction Company is bidding on a contract to install a line of microwave
towers. It has identified the following activities, along with their expected time, predecessor restrictions, and
worker requirements:

Activity Duration, Weeks Predecessor Crew size, workers


A 4 None 4
B 7 None 2
C 3 A 2
D 3 A 4
E 2 B 3
F 2 B 3
G 2 D,E 3
H 3 F,G 4

The contract specifies that the project must be completed in 14 weeks. This company will assign a fixed
number of workers to the project for its entire duration, and so it would like to ensure that the minimum
number of workers is assigned and that the project will be completed in 14 weeks. Find a schedule which will
do this.
CA. Parag Gupta Ph.: +91 9891 432 632 Paraggupta_ca@yahoo.co.in Costing & O.R.
World’s largest CA Final student’s consultancy group: http://groups.yahoo.com/group/costingbyparaggupta
Network Analysis - PERT / CPM - 334 -
(including resource allocation & resource leveling)

[Hint.: The maximum number of workers to be assigned to the project is 6] (10 Marks) May/95

Question 43 (Resource Smoothing): A network with the following activity durations and manpower
requirement is given. Analyze the project from point of view of resources to bring out the necessary steps
involved in the analysis and smoothing of resources.

Activity: 1-2 2-3 2-4 3-5 4-6 4-7 5-8 6-8 7-9 8-10 9-10
Duration (weeks): 2 3 4 2 4 3 6 6 5 4 4
No. of Men required: 4 3 3 5 3 4 3 6 2 2 9

[Ans.: Demand for men will decrease to 15 to 11]

Question 44 (Resource Smoothing): Consider a project consisting of 14 activities having the duration and
resource requirement shown below. Analyze the project and smoothen the requirement of the resources.

Activity Duration (Weeks) Masons (M) Labourers (L)


1-2 2 1 2
2-3 3 2 2
2-4 4 3 2
2-5 2 1 3
3-10 4 2 2
4-6 2 3 2
4-7 4 3 3
5-9 4 5 3
6-8 2 1 2
7-9 5 1 3
8-9 3 - 4
9-11 2 1 1
10-11 3 1 2
11-12 2 1 2

[Ans.: Demand for masons will decrease to 8 & 10 for Labourers]

Question 45 (Resource Allocation): The following information is available:

Activity No. of days No. of men reqd. per


day
A 1-2 4 2
B 1-3 2 3
C 1-4 8 5
D 2-6 6 3
E 3-5 4 2
F 5-6 1 3
G 4-6 1 8

(a) Draw the network and find the critical path.


(b) What is the peak requirement of Manpower? On which day(s) will this occur?
(c) If the maximum labour available on any day is only 10, when can the project be completed?

[Ans.: (i) Critical Path is AD = 10 days (ii) Peak requirement is 11 men, required on days 7 and 9 (iii) the
project can be completed in 11 days.] (9 Marks) May/08
Question 46 (Resource Allocation): For a project consisting of several activities, the durations and required
resources for carrying out each of the activities and their availabilities are given below:
(a) Draw the network, identify critical path and compute the total float for each of the activities.
(b) Find the project completion time under the given resource constraints.

CA. Parag Gupta Ph.: +91 11 47665555 Paraggupta_ca@yahoo.co.in Costing & O.R.
World’s largest CA Final student’s consultancy group: http://groups.yahoo.com/group/costingbyparaggupta
Operations Research - 335 -

Resources required
Activity Equipment Operators Duration (Days)
1-2 X 30 4
1-3 Y 20 3
1-4 Z 20 6
2-4 X 30 4
2-5 Z 20 8
3-4 Y 20 4
3-5 Y 20 4
4-5 X 30 6
Resource availability:
No. of operators = 50, equipment X = 1, equipment Y = 1, equipment Z = 1

[Ans.: Critical Path is 1-2-4-5 with duration of 14 days, Project requires 21 days for completion under given
constraints] (Nov./85)

Questions on Resource Smoothing and Resource Leveling are rarely being asked in Examination.

Question 47 (Updating): A company had planned its operations as follows:

Activity: 1-2 2-4 1-3 3-4 1-4 2-5 4-7 3-6 5-7 6-8 7-8
Duration (Days): 7 8 8 6 6 16 19 24 9 7 8
(i) Draw the network and find the critical paths.
(ii) After 15 days of working, the following progress is noted:
(a) Activities 1-2, 1-3 and 1-4 completed as per original schedule.
(b) Activity 2-4 is in progress and will be completed in 4 more days.
(c) Activity 3-6 is in progress and will need 17 more days to complete.
(d) The staff at activity 3-6 is specialized. They are directed to complete 3-6 and undertake an activity 6-7,
which will require 7days. This rearrangement arose due to a modification in a specialization.
(e) Activity 6-8 will be completed in 4 days instead of the originally planned 7 days.
(f) There is no change in the other activities.
Update the network diagram after 15 days of start of work based on the assumption given above. Indicate the
revised critical paths alongwith their duration. (11 Marks) May/07

[Ans.: Critical Path 1→2→4→7→8 = 42 days; (ii) Critical path 1→3→6→7→8 = 47 days]

CA. Parag Gupta Ph.: +91 9891 432 632 Paraggupta_ca@yahoo.co.in Costing & O.R.
World’s largest CA Final student’s consultancy group: http://groups.yahoo.com/group/costingbyparaggupta
Simulation

According to Shannon “Simulation is the process of designing a model of the real system by conducting
experiments with this model for the purpose of designing a model of the real system.” It is imitation of reality.
Maruti car training center with various crossings and signals is a simulated model of city traffic system.

The imitation of reality which may be in the physical form or in the form of mathematical equations may be
called simulation. The aircraft simulator to train pilots on new models of aircraft represents the use of physical
model as a means of experimentation. The mathematical models of real-life situations in investment analysis,
scheduling or inventory control can be experimented; these are known as symbolic simulation models. These
models can be either deterministic or probabilistic. The deterministic models can provide answers to ‘what if’
type of questions in business problems. The probabilistic simulation models deal with random phenomenon
and the method of simulation applied is known as Monte Carlo Simulation.

Application of simulation in practical situations


Simulation is quite versatile & commonly applied technique for solving decision problems. It has been applied
successfully to a wide range of problems, as given below:
1. Locating of ambulances.
2. Study of projects involving risky investments.
3. Effect of environment on health & group behaviour.
4. Design of computer systems of future needs.
5. Design of weapon systems, war strategies & tactics.
6. New Shop floor management before it is implemented.
7. Design of queuing systems.
8. Design of optimal replenishment policy.
9. Scheduling of production process.
10. Aircraft management & routing.
11. Scheduling of bank tellers & location of bank branches.
12. Routing & dispatching when roads are not secured.
13. Deployment of fire stations & many other problems.

Advantages
1. Behavioral Analysis - Simulation can be used to investigate the behavior of problems which are too
complex to be modeled mathematically.
2. Fairly Simple - The simulation technique is easier to use than mathematical models and is considered
superior to mathematical models.
3. Flexible - Simulation models are comparatively flexible and can be modified to accommodate the
changing environment of the real situation.
4. Time saving – It can compress the performance of a system over several years involving large
calculations into few minutes e.g. the effects of ordering, advertising or other policies over many
months or years can be obtained by computer simulation in short time.
5. Imparting training - It has advantageously been used for training the operating and managerial staff in
operation of complex plans. It is always advantageous to train people on simulation models before
putting into their hands the real system.
6. Foresee Hindrances - Through simulation, management can foresee the difficulties and bottlenecks
which may come up due to the introduction of new machines, equipment or process.

Limitations (4 Marks) Nov/02 & (3 Marks) May/04


1. Not an Optimizing technique - Simulation does not produce optimum results. It simply allows us to
select the best of the alternative systems examined.
CA. Parag Gupta Ph.: +91 11 47665555 Paraggupta_ca@yahoo.co.in Costing & O.R.
World’s largest CA Final student’s consultancy group: http://groups.yahoo.com/group/costingbyparaggupta
Operations Research - 337 -

2. Costlier on occasions – It is, by no means, a cheap method of analysis. In a number of situations,


such as corporate planning, simulation is comparatively costlier and time consuming.
3. Quantification – In number of situations it is not possible to quantify all the variables that affect the
behavior of the system.
4. Time Consuming – On occasions simulation is very much time consuming process.
5. Useful in long term only – Reliable results are possible only if the simulation is continued for a long
period.

Monte Carlo Simulation (a.k.a. Computer Simulation)


The Monte Carlo method of simulation was developed by two mathematicians John Von Neuman and
Stainslaw Ulam. The technique employs random numbers and is used to solve problems that involve
probability and wherein physical experimentation is impracticable and formulation of mathematical
formula is impossible.

The steps involved carrying out Monte Carlo simulation are:


1. Setting up a probability distribution for variables to be analysed.
2. Building a cumulative probability distribution for each random variables.
3. Generating random numbers and then assigning an appropriate set of random numbers to represent
value or range (interval) of values for each random variable. If the cumulative probabilities are in two
digits the range of random numbers to be assigned are 00 to 99; and if in three digits, the range is
from 000 to 999, and so on.
4. Conducting the simulation experiment using random sampling.
5. Repeating the Step 4 until the required number of simulation runs has been generated.
6. Designing and implementing a course of action and maintaining control.

Question 1: State major reason for using a simulation technique to solve a problem and also describe basic
steps in a general simulation process. (4 Mark) Nov/06, (4 Marks) Nov/03 & (5 Marks) June/09-O.C.

Question 2: Write short note on Monte Carlo Simulation.


(1 Mark) Nov/06, (5 Marks) Nov/01 & (1 Mark) Nov/03
Question 3: How would you use Monte Carlo Simulation method in inventory control?
(3 Marks) Nov/03 & (4 Marks) May/08
Question 4: What are the steps involved in carrying out Monte Carlo Simulation model?
(4 Marks) Nov./10-N.C.
Question 5: How can simulation be applied in practical situations? (4 Marks) Nov./10-O.C.
Question 6 (Stock simulation): Bright Bakery keeps stocks of a popular brand of cake. Previous experience
indicates the daily demand as given here.

Daily Demand 0 10 20 30 40 50
Probability 0.01 0.20 0.15 0.50 0.12 0.02

Consider the following sequence of random numbers;


48 78 19 51 56 77 15 14 68 09
(i) Using the sequence simulate the demand for next 10 days.
(ii) Find out the stock simulation if owner of the Bakery decides to make 30 cakes every day. Also estimate
the daily Average demand for the cakes. (10 Marks) Nov./98 & (10 Marks) Nov./99

[Ans.: Average Daily Demand = 22 cakes.]


Question 7: A company manufactures around 200 mopeds. Depending upon the availability of raw
materials and other conditions, the daily production has been varying from 196 mopeds to 204
mopeds, whose probability distribution is as given below:

Production/day 196 197 198 199 200 201 202 203 204
Probability 0.05 0.09 0.12 0.14 0.20 0.15 0.11 0.08 0.06

CA. Parag Gupta Ph.: +91 11 47665555 Paraggupta_ca@yahoo.co.in Costing & O.R.
World’s largest CA Final student’s consultancy group: http://groups.yahoo.com/group/costingbyparaggupta
Simulation - 338 -

The finished mopeds are transported in a specially designed three storied lorry that can
accommodate only 200 mopeds. Use the following 15 random numbers 82, 89,78,24, 53, 61, 18, 45,
04, 23, 50, 77, 27, 54, 10 to simulate the process to find out :
(i) What will be the average number of mopeds waiting in the factory?
(ii) What will be the average number of empty space on the lorry?

[Ans.: (i) 2.8 (ii) 0.27] (8 Marks) ICWA Dec/06, (10 Marks) May/98 [Adapted] & (10 Marks) May/99
Question 8 (Queuing Problem): Dr. STRONG is a dentist who schedules all her patients for 30
minutes appointments. Some of the patients take more or less than 30 minutes depending on the type of
dental work to be done. The following summary shows the various categories of work, their
probabilities and the time needed to complete the work :

Category Time required (mts) Probability of category


Filling 45 0.40
Crown 60 0.15
Cleaning 15 0.15
Extraction 45 0.10
Checkup 15 0.20

Simulate the dentist’s clinic for four hours and determine the average waiting time for the patients as well
as the idleness of the doctor. Assume that all the patients show up at the clinic at exactly their scheduled
arrival time starting at 8.00 A.M. Use the following random numbers handling the above problem:
40 82 11 34 25 66 17 79 (12 Marks) Nov/90

[Ans.: Avg waiting time = 35.625 minutes; Dentist’s idle time = nil]
Question 9 (Service Sector): A single counter ticket booking centre employs one booking clerk. A
passenger on arrival immediately goes on the booking counter for being served if the counter is free. If, on
the other hand, the counter is engaged, the passenger will have to wait. The passengers are served on first
come first served basis. The time of arrival and the time of service varies from one minute to six minutes. The
distribution of arrival and service time is as under:

Arrival/Service Arrival Service


Time (Minutes) (Probability) (Probability)
1 0.05 0.10
2 0.20 0.20
3 0.35 0.40
4 0.25 0.20
5 0.10 0.10
6 0.05 -

Required:
(i) Simulate the arrival and service of 10 passengers starting from 9 A.M. by using the following
random numbers in pairs respectively for arrival and service. Random numbers 60 09 16 12 08
18 36 65 38 25 07 11 08 79 59 61 53 77 03 10.
(ii) Determine the total duration of
(1) Idle time of booking clerk and
(2) Waiting time of passengers

[Ans.: (1) 6 Mins. (2) 6 Mins.]


(6 Marks) May/10-O.C.[Adapted], (8 Marks) Nov/08-O.C. & (7 Marks) May/03 [Adapted]
Question 10 (Service Sector): At a small store of readymade garments, there is one clerk at the counter
who is to check bills, receive payments and place the packed garments into fancy bags. The arrival of
customer at the store is random and service time varies from one minute to six minutes, the frequency
distribution for which is given below:

CA. Parag Gupta Ph.: +91 11 47665555 Paraggupta_ca@yahoo.co.in Costing & O.R.
World’s largest CA Final student’s consultancy group: http://groups.yahoo.com/group/costingbyparaggupta
Operations Research - 339 -

Time between arrivals Frequency Service Time Frequency


1 5 1 1
2 20 2 2
3 35 3 4
4 25 4 2
5 10 5 1
6 5 6 0

The store starts work at 11 a.m. and closes at 12 noon for lunch and the customers are served on the "first
came first served basis".
Using Monte Carlo simulation technique, find average length of waiting line, average waiting time, average
service time and total time spent by a customer in system.
You are given the following set of random numbers, first twenty for arrivals and last twenty for service :

64 04 02 70 03 60 16 18 36 38
07 08 59 53 01 62 36 27 97 86
30 75 38 24 57 09 12 18 65 25
11 79 61 77 10 16 55 52 59 63

[Ans.: Average length of waiting line is 1.3 customers, Average waiting time is 2.8 mins, Average service
time is 2.7 mins; Time a customer spends in system = 2.8 + 2.7 = 5.5 minutes.] (9 Marks) Nov./09-O.C.
[Note: Few terminologies & their meaning in Queuing System:
(i) Queue length – The average number of customers in the queue waiting to get service. This excludes
the customer(s) being served.
(ii) System length – The average number of customers in the system including those waiting as well
those being served.
(iii) Waiting time in the queue – The average time for which a customer has to wait in the queue to get
service.
(iv) Total time in the system – the average time spent by a customer in the system from the moment he
arrives till he leaves the system. It is taken to be waiting time + service time.]
Question 11: Ramu and Raju are workers on a two-station assemble line. The distribution of activity times at
their stations is as follows:-

Time in Sec. Time frequency for Ramu Time frequency for Raju
10 4 4
20 6 5
30 10 6
40 20 7
50 40 10
60 11 8
70 5 6
80 4 4

(a) Simulate operation of the line for eight times. Use the random numbers given below:

Operation 1 Operation 2
14 61 36 97
01 82 76 41
95 00 55 56
44 03 25 34

(b) Assuming Raju must wait until Ramu completes the first item before starting work, will he have to wait to
process any of the other eight items? Explain your answer, based upon your simulation.

[Ans.: (b) No]


Question 12: The occurrence of rain in a city on a day is dependent upon whether or not it rained
on the previous day. If it rained on the previous day, the rain distribution is given by:

CA. Parag Gupta Ph.: +91 11 47665555 Paraggupta_ca@yahoo.co.in Costing & O.R.
World’s largest CA Final student’s consultancy group: http://groups.yahoo.com/group/costingbyparaggupta
Simulation - 340 -

Event Probability
No rain 0.50
1 cm. Rain 0.25
2 cm. Rain 0.15
3 cm. Rain 0.05
4 cm. Rain 0.03
5 cm. Rain 0.02
If it did not rain the previous day, the rain distribution is given by:

Event Probability
No rain 0.75
1 cm. Rain 0.15
2 cm. Rain 0.06
3 cm. Rain 0.04

Simulate the city’s weather for 10 days and determine by simulation the total days without rain as well as
the total rainfall during the period. Use the following random numbers:
67 63 39 55 29 78 70 06 78 76
for simulation. Assume that for the first day of the simulation it had not rained the day before.

[Ans.: It didn’t rain on 6 out of 10 days. Total rainfall during the period is 5cm.] (10 Marks) Nov./93
Question 13: The output of a production line is checked by an inspector for one or more of three different
types of defects, called defects A, B and C. If defect A occurs, the item is scrapped. If defect B or C
occurs, the item must be reworked. The time required to rework a B defect is 15 minutes and the time
required to rework a C defect is 30 minutes. The probabilities of an A, B and C defects are 0.15, 0.20 and
0.10 respectively. For ten items coming off the assembly line, determine the number of items without any
defect, the number scrapped and the total minutes of rework time. Use the following random numbers:

RN for defect A 48 55 91 40 93 01 83 63 47 52
RN for defect B 47 36 57 04 79 55 10 13 57 09
RN for defect C 82 95 18 96 20 84 56 11 52 03

[Ans.: No defect in 5 items, one item is scrapped & Rectification time required on reworking is 90 minutes]
(10 Marks) May/94 & (7 marks) May/04 [Adapted]
Question 14 (Purchase Quantity Decision): A book-store wishes to carry Systems Analysis and Design in
stock. Demand is probabilistic and replenishment of stock takes 2 days (i.e., if an order is placed in March 1,
it will be delivered at the end of the day on March 3). The probabilities of demand are given below:

Demand (daily) : 0 1 2 3 4
Probability : 0.05 0.10 0.30 0.45 0.10

Each time an order is placed, the store incurs an ordering cost of `10 per order. The store also incurs a
carrying cost of `0.50 per book per day. The inventory carrying cost is calculated on the basis of stock at the
end of each day. The manger of the book-store wishes to compare two options for his inventory decision:
A. Order 5 books, when the inventory at the beginning of the day plus orders outstanding is less than 8
books.
B. Order 8 books, when the inventory at the beginning of the day plus orders outstanding is less than 8
books.
Currently (beginning of the 1st day) the store has stock of 8 books plus 6 books ordered 2 days ago and
expected to arrive next day. Using Monte-Carlo simulation for 10 cycles, recommend which option the
manager should choose?
The two digits random numbers are given below:
89, 34, 78, 61, 63, 81, 39, 16, 13, 73 (10 Marks) May/00, (7 Marks) Nov./04 [Adapted]

[Ans.: Manager should go for Option B, as its Total Cost `42.5 is less than Option A’s `59.5]
[Hint: Assume that the demand occurring during the day can be met out of stock received at the end of the
day]

CA. Parag Gupta Ph.: +91 11 47665555 Paraggupta_ca@yahoo.co.in Costing & O.R.
World’s largest CA Final student’s consultancy group: http://groups.yahoo.com/group/costingbyparaggupta
Operations Research - 341 -

Question 15: A company uses a high grade book raw material. The consumption pattern is probabilistic as
given below and it takes two months to replenish stocks:

Consumption per month (tons) : 1 2 3 4


Probability : 0.15 0.30 0.45 0.10

The cost of placing an order is `1000 and the cost of carrying stocks is `50 per month per ton. The average
carrying costs are calculated on the stocks held at the end of each month.
The company has two options for purchase of raw material as under:
Option I. Order 5 tons, when the closing inventory of the month plus orders outstanding is less than 8 tons.
Option II. Order 8 tons, when the closing inventory of the month plus orders outstanding is less than 8 tons.
Currently on 1st April 2002, the company has a stock of 8 tons of raw materials plus 6 tons ordered two
months ago. The order quantity is expected to be received next month.
Using the random numbers given below, simulate 12 months consumption till 31-3-2003 and advise the
company as to which purchase option should be accepted such that the inventory costs are minimum.
Random numbers are:
88, 41, 67, 63, 48, 74, 27, 16, 11, 64, 49, 21
(ICWA Dec. 98-Adapted)&(16 Marks) ICWA Dec/07-Adapted

[Ans.: Co. should go for Option II, as its Total Cost `5350 is less than Option I’s `7200]
[Hint: Since raw material is ordered in 3rd month after calculation of closing stock, it will be received on 5th
months but after calculation of closing stock. Since closing stock of 5th month & opening stock of 6th month
will always be same, hence we will treat that order has been received in 6th month. Similar assumption will
apply for other reorder points as well.]
Question 16: A Publishing house has brought out a new monthly magazine, which sells at `37.5 per copy.
The cost of producing it is `30 per copy. A Newsstand estimates the sales pattern of the magazine as follows:

Demand Copies Probability


0 < 300 0.18
300 < 600 0.32
600 < 900 0.25
900 < 1200 0.15
1200 < 1500 0.06
1500 < 1800 0.04

The newsstand has contracted for 750 copies of the magazine per month from the publisher.
The unsold copies are returnable to the publisher who will take them back at cost less `4 per copy for
handling charges.
The newsstand manager wants to simulate of the demand and profitability. The following random number
may be used for simulation:
27, 15, 56, 17, 98, 71, 51, 32, 62, 83, 96, 69.
You are required to-
(i) Allocate random numbers to the demand pattern forecast by the newsstand.
(ii) Simulate twelve months sales and calculate the monthly and annual profit/loss.
(iii) Calculate the loss on lost sales. (8 Marks) Nov./05 & (June/97)ICWA-Adapted

[Ans.: Loss on lost sale = `15750]


[Hint: In statistical terminology, 0<300 is used for “exclusive method” of class interval & it means 0-300. To
calculate demand, we shall find class mid-points/marks i.e. average of upper Limit and lower limit]
Question 17 (Perishable Goods): A retailer deals in a perishable commodity. The daily demand and supply
are variables. The data for the past 500 days, show the following demand and supply:

Supply Demand
Availability (kg.) No. of days Demand (kg.) No. of days
10 40 10 50
20 50 20 110
30 190 30 200

CA. Parag Gupta Ph.: +91 11 47665555 Paraggupta_ca@yahoo.co.in Costing & O.R.
World’s largest CA Final student’s consultancy group: http://groups.yahoo.com/group/costingbyparaggupta
Simulation - 342 -

40 150 40 100
50 70 50 40

The retailer buys the commodity at `20 per kg. and sells it at `30 per kg. Any commodity remains at the end
of the day, has no saleable value. Moreover, the loss (unearned profit) on any unsatisfied demand is `8 per
kg. Given the following pair of random numbers, simulate 6 days sales, demand and profit.
(31,18); (63,84);(15,79);(07,32);(43,75);(81,27).
The first random number in the pair is for supply and the second random number is for demand viz. in the first
pair (31,18), use 21 to simulate supply and 18 to simulate demand. (10 Marks) Nov/00

[Ans.: `400]
Question 18: For a washing powder manufacturing factory, frequency distribution of contribution (=sales
price-variable cost) per unit annual demand and requirement of investment were found as follows:

Annual Demand Contribution per unit Required investment


Units Probability ` Probability ` Probability
20000 0.05 3.00 0.10 1750000 0.25
25000 0.10 5.00 0.20 2000000 0.50
30000 0.20 7.00 0.40 2500000 0.25
35000 0.30 9.00 0.20
40000 0.20 10.00 0.10
45000 0.10
50000 0.05

Consider the random numbers 93,03,51,59,77,61,71,62,99,15 for using Monte-Carlo simulation for 10 runs to
estimate the percentage of return on investment (ROI%) defined as

ROI % =
Cash Inflow X 100
Investment
Recommend an optimum investment strategy based on modal value of ROI%.

[Ans.: Modal value of ROI % is 12.25, the optimal investment strategy is to investment `2000000]
(10 Marks) Nov./95 & (10 Marks) May/01[Adapted]
Question 19: ABC Cooperative Bank receives and disburses different amount of cash in each month. The
bank has an opening cash Balance of Rs. 15 crores in the first month. Pattern of receipts and disbursements
from past data is as follows:

Monthly Cash receipts Monthly Cash disbursements


Rs. In Crores Probability Rs. In Crores Probability
30 0.20 33 0.15
42 0.40 60 0.20
36 0.25 39 0.40
99 0.15 57 0.25

Simulate the cash position over a period of 12 months.


Required:
(i) Calculate probability that the ABC Cooperative Bank will fall short in payments.
(ii) Calculate average monthly shortfall.
(iii) If ABC Bank can get an overdraft facility of Rs. 45 crores from other Nationalized banks.
What is the probability that they will fall short in monthly payments?
Use the following sequence (rowwise) of paired random numbers.
1778 4316 7435 3123 7244 4692 5158 6808 9358 5478 9654 0977

[Ans.: (i) 0.83; (ii) Average shortfall is 399 ÷ 12 = Rs. 33.25 crores; Alternatively, average shortfall is 399 ÷ 10
= Rs. 39.90 crores (iii) 0.42] (7 Marks) May/10-N.C.
[Hint: Payments may be divided into 2 parts viz. Amount paid & amount due but not paid in current month.]

CA. Parag Gupta Ph.: +91 11 47665555 Paraggupta_ca@yahoo.co.in Costing & O.R.
World’s largest CA Final student’s consultancy group: http://groups.yahoo.com/group/costingbyparaggupta
Operations Research - 343 -

Question 20: The Ever alert Ltd., which has a satisfactory preventive maintenance system in its plant, has
installed a new Hot Air Generator based on electricity instead of fuel oil for drying its finished products. The
Hot Air Generator requires periodicity shutdown maintenance. If the shut down is scheduled yearly, the cost
of maintenance will be as under:

Maintenance cost (`) Probability


15,000 0.3
20,000 0.4
25,000 0.3

The costs are expected to be almost linear i.e. if the shut down is scheduled twice a year the maintenance
cost will be double.
There is no previous experience regarding the time taken between break downs. Costs associated with break
down will vary depending upon the periodicity of maintenance. The probability distribution of break down cost
is estimated as under:
Breakdown costs `Per annum Shutdown once a year Shutdown twice a year
75,000 0.2 0.5
80,000 0.5 0.3
1,00,000 0.3 0.2

Simulate the total costs maintenance and breakdown cost- and recommend whether shutdown overhauling
should be restored to once a year or twice a year? (ICWA-Dec/96)

[Ans.: The average annual cost will only be `1.06 lakhs as against `1.20 lakhs when shutdown is twice a
year hence shutdown maintenance/overhauling once a year will be more economical]
[Hint: Since random nos. were not provided, assume them at your own. In such answer may vary.]
Question 21 (Network simulation-Do after PERT/CPM): A project consists of 7 activities. The time for
performance of each of the activity is as follows:-

Activity Immediate Time Probability


A - 3 0.2
4 0.6
5 0.2
B - 4 1.0
C A 1 1.0
D B,C 4 0.8
5 0.2
E D 3 0.1
4 0.3
5 0.3
6 0.3
F D 5 0.2
7 0.8
G E,F 2 0.5
3 0.5

a) Draw a network and identify critical path using expected time.


b) Simulate the project for 5 times using random number and find the critical paths?

68 13 09 20 73 07 72
99 93 18 24 22 07 29
57 33 49 65 92 98 00
57 12 31 96 85 92 91
77 37 34 11 27 10 59

CA. Parag Gupta Ph.: +91 11 47665555 Paraggupta_ca@yahoo.co.in Costing & O.R.
World’s largest CA Final student’s consultancy group: http://groups.yahoo.com/group/costingbyparaggupta
Learning Curve
Theory

The Theory of learning curve was first introduced by T.P. Wright of Curtiss—Wright, Buffalo, U.S.A. engaged
in production of airframes.

CIMA Terminology: The learning curve is “The mathematical expression of the phenomenon that when
complex & labour-intensive procedures are repeated, unit labour times tend to decrease at a constant rate.
The learning curve models mathematically this reduction in unit production time.”

More specifically, the learning curve theory states that the cumulative average time per unit produced is
assumed to decrease by a constant percentage every time total output of product doubles. For
instance, with an 80% learning curve, the cumulative average time per unit of output will fall to 80% of what it
was before, every time output is doubled.

E.g. If the first unit of output requires 100 hours & an 70% learning curve applies, the production times would
be as follows.:
Cumulative Average-Time Learning Model (Doubling Approach)
Cum. No. of Cum. Average Total time Incremental Incremental Incremental
units time per unit required (Hrs.) time taken units time taken
produced (Hrs.) (Total Hours) produced (Hrs. per unit)
1 100.0 100.0 (100×1) 100 1 100 (100÷1)
2 (1×2) 70.0 (100×70%) 140.0 (70×2) 40 1 40 (40÷1)
4 (2×2) 49.0 (70×70%) 196.0 (49×4) 56 2 28 (56÷2)
8 (4×2) 34.3 (49×70%) 274.4 (34.3×8) 78.4 4 19.6 (78.4÷4)

Graph of Learning Curve:

Cumulative Average-Time Learning Model (Equation Approach)


Cum. No. of units Cum. Average time Cumulative Total time Incremental time taken/Individual
produced (X) per unit (Hrs.)1 required (Hrs.) unit time for Xth unit
1 100 100 100
2 70 140 40 (140-100)
3 56.8 170.4 30.4 (170.4–140)
4 49 196 25.6 (196-170.4)
5 43.7 218.5 22.5 (218.5-196)

CA. Parag Gupta Ph.: +91 11 47665555 Paraggupta_ca@yahoo.co.in Costing & O.R.
World’s largest CA Final student’s consultancy group: http://groups.yahoo.com/group/costingbyparaggupta
Operations Research - 345 -

6 39.8 238.8 20.3 (238.8-218.5)


7 36.7 256.9 18.1 (256.9-238.8)
8 34.3 274.4 17.5 (274.4-256.9)
1
The mathematical relationship underlying the cumulative average-time learning model is
Y x =aXb
Where, Y = cumulative average time per unit to produce x units.
X = number of units made so far (cumulative no. of units)
a = time for the first unit
log (learning ratio)
b = the learning coefficient or the index of learning =
log 2
log 0.70 0.1549
(For e.g. for 70% learning curve, b = = = -0.5146 )
log 2 0.3010

Average labour cost / time of Ist 2N units


Learning ratio/percentage =
Average labour cost / time of Ist N units

 Learning Curve Table gives us factor for learning curve, which is used as value of Xb in the
above-mentioned formula of learning curve.
 If the initial output was a batch of units rather than just one, the table of factors would be derived
in the same way except that we firstly use factor according to batch & hours calculated on
individual units.

Uses of Learning curve:

(i) CVP Analysis: It helps in analyzing cost-volume profit relationship and is useful for cost estimates and
forecasting.
(ii) Budgeting: It helps in preparing realistic budgets and profit planning.
(iii) Pricing: It helps in pricing, particularly in a tender when it is known that the tender
consists of several repetitive jobs.
(iv) Decisioning: It helps design engineers in making decision, based upon expected rates of
improvement.
(v) Setting Standards: It helps in setting standards in learning phase.
(vi) Human Resource planning: Its knowledge helps in manpower planning for contract of long
duration or for repetitive clerical work.

Limitations of learning curve theory:

1. The learning curve is useful only for new operations where machines do not constitute a major
part of the production process. It is not applicable to all productions. E.g. New and experienced
workmen.
2. The learning curve assumes that the production will continue without any major interruptions. If for
any reason the work in interrupted, the curve may be deflected or assume a new slopes.
3. Changes other than learning may affect the learning curve. For example, improvement in facilities,
arrangements, and equipment as well as personnel morale and performance may be factors
influencing the curve. On the other hand, negative developments in employee attitudes may also
affect the curve and reverse or retard the progress of improvement.
4. The characteristic 80 percent learning curve as originally obtaining in the air force industry in U.S.A.
has been usually accepted as the percentage applicable to all industries. Studies show that
there cannot be a unique percentage which can be universally applied.
5. Normally, the trade unions will not accept gradual reduction in production time per unit. In this type of
situation, management may try to establish a low standard time per unit from the onset. This will lead
to adverse efficiency variances, until learning effect has taken place.
6. If there is productivity bonus, incorporation of learning effect may frustrate workers in learning stage,
as it may appear as a threat to the size of bonus, which they may earn.

CA. Parag Gupta Ph.: +91 11 47665555 Paraggupta_ca@yahoo.co.in Costing & O.R.
World’s largest CA Final student’s consultancy group: http://groups.yahoo.com/group/costingbyparaggupta
Learning Curve Theory - 346 -

Experience Curve : It is a term applied to the ‘corporate embodiment’ of the shop floor, managerial &
technological learning effects within an organization & it expresses the way in which the average cost per unit
of production changes over time due to technological & organizational changes to factory size, product,
design, materials used & so on, not just ‘learning’ by skilled workers.

Important Points in regards to logarithms:


 Y=xa can also be written as log Y = a log x
 log (x × y) = log x + log y
 log (x ÷ y) = log x – log y
 log 1 = 0, log 10 = 1 & log 100 = 2

Question 1: Explain the concept of ‘Learning curve’. How can it be applied for Cost Management?
(2 Marks) Nov/06, (4 Marks) May/06, (4 Marks) May/07 & (4 Marks) Nov/07
Question 2: Explain the concept of Learning curve and discuss its relevance to pricing decisions.
(4 Marks) May/04
Question 3: What are the distinctive features of learning curve theory in manufacturing environment?
(4 Marks) May/03, (5 Marks) Nov/07, (4 Marks) Nov./10-N.C.
Answer: As the production quantity of a given item is doubled, the cost of the item decreases at a fixed rate.
This phenomenon is the basic premise on which the theory of learning curve has been formulated. As the
quantity produced doubles, the absolute amount of cost increase will be successively smaller but the rate of
decrease will remain fixed. It occurs due to the following distinctive features of manufacturing environment:
(a) Better tooling methods are developed and used.
(b) More productive equipments are designed and used to make the product.
(c) Design bugs are detected and corrected.
(d) Engineering changes decrease over time.
(e) Earlier teething problems are overcome.
(f) Rejections and rework tend to diminish over time.
Question 4: The following information is provided by a firm. The factory manager wants to use appropriate
average learning rate on activities so that he may forecast costs and prices for certain levels of activity.
i) A set of very experienced people feed data into the computer for processing inventory records in the
factory. The manager wishes to apply 80% learning rate on data entry and calculation of inventory.
ii) A new type of machinery is to be installed in the factory. This is patented process and the output may
take a year for full fledged production. The factory manager wants to use a learning rate on the
workers at the new machine.
iii) An operation uses contract labour. The contractor shifts people among various jobs once in two days.
The labour force performs one task in 3 days. The manager wants to apply an average learning rate
for these workers.
You are required to advise to the manager with reasons on the applicability of the learning curve theory on the
above information. (4 Marks) Nov./09-O.C.
[Ans.: (i), (ii) & (iii) Don’t apply Learning curve theory]
Question 5: A company which has developed a new machine has observed that the time taken to
manufacture the first machine is 600 hours. Calculate the time which the company will take to manufacture
the second machine if the actual learning curve rate is (i) 80% and (ii) 90%. Explain which of the two learning
rates will show faster learning. (3 Marks) Nov./08-O.C.

[Ans.: (i) 360 hours (ii) 480 hours; 80% is shows faster learning]
Question 6: (a)Your company has been approached by a customer to supply four units of a new product
made to the customer’s individual specification. The company experiences a 90% learning rate. The
estimated labour time for the first unit of this product is 150 hours and the company’s direct labour cost is `5
per hour. Estimate the labour cost for this order.
(b) After receiving the first order, if the customer places a repeat order, what will be the labour cost for the
second order.
(c) If the customer had ordered all eight units at the same time, calculate the labour cost per unit for the
combined order.

[Ans.: (a) 2430 (b) 1944 (c) 4374]

CA. Parag Gupta Ph.: +91 11 47665555 Paraggupta_ca@yahoo.co.in Costing & O.R.
World’s largest CA Final student’s consultancy group: http://groups.yahoo.com/group/costingbyparaggupta
Operations Research - 347 -

Question 7: A factory has a special offer to produce 4 units of a labour intensive product by using its existing
facilities after the regular shift timings. The product can be produced by using only overtime hours which
entails normal rate plus 25%, so that usual production is not affected. Two workers are interested in taking up
this additional job every evening after their usual shift is over. One is an experienced man who has been
working on a similar product. His normal wages are `48 per hour. The other worker is a new person who
earns `42 an hour as normal wages. He can be safely considered to have a learning curve ratio of 90% for
this work. The company wants to minimize labour cost for the order and only one person is to be chosen for
the job. The experienced man will take 20 hours for the first unit while the new worker will take 30 hours for
the first unit. Evaluate who should be chosen for the job. (5 Marks) Nov./10-O.C.
[Ans.: Employing experienced person will lead into savings of `303 (5103-4800)]

Question 8: The Gifts Company makes mementos for offering chief guests and other dignitaries at functions.
A customer wants 4 identical pieces of hand-crafted gifts for 4 dignitaries invited to its function.
For this product, the Gifts Company estimates the following costs for the 1st unit of the product
`/unit
Direct variable costs (excluding labour) 2,000
Direct labour (20 hours @ `50 per hour) 1,000
90% learning curve ratio is applicable and one labourer works for one customer's order.
i) What is the price per piece to be quoted for this customer if the targeted contribution. is `1,500 per
unit ?
ii) If 4 different labourers made the 4 products simultaneously to ensure faster delivery to the customer,
can the price at (i) above be quoted? Why? (6 Marks) Nov./09-N.C.

[Ans.: (i) `4310 per piece (ii) `4500 per piece]


Question 9: In a particular industry, a 60% learning curve applies. The first item of a new product took 100
hours to make. How long should the 6th item take?

[Ans.: 7.55 hours]


Question 10: ABC Co. Ltd. has observed that a 80% learning curve ratio applies to all labour-related costs
realating Model X, which has been recently introduced in the market. It is expected that first unit will take a
time of 10 minutes. What is the expected time per unit (i) when cumulative production is 20 units, and (ii)
when cumulative production is 10 units?

[Ans.: (i) 3.812 minutes & (ii) 4.764 minutes]


Question 11: The 1st batch of 50 units of Product A took 80 hours to make. The company now wishes to
estimate, what average time per unit will be if the total output for product A is 125 units. An 80% learning
curve applies.

[Ans.: 1.2 hours]


Question 12: A customer requires separate price quotations for each of the following possible order:
Order Units
First 100
Second 60
Third 40
It is expected that in first order 4 hours per unit will be consumed. A 80% learning curve applies. Following
learning curve table for 80% learning curve is given:
X 1.0 1.3 1.4 1.5 1.6 1.7 1.8 1.9 2.0
Y% 100.0 91.7 89.5 87.6 86.1 84.4 83.0 81.5 80.0
Find out average time per unit:
(a) when two orders are expected.
(b) When three orders are expected.

[Ans.: (a) 3.44 (b) 3.2]


Question 13: M Ltd. manufactures a special product purely carried out by manual labour. It has a capacity of
20000 units. It estimates the following cost structure:

CA. Parag Gupta Ph.: +91 11 47665555 Paraggupta_ca@yahoo.co.in Costing & O.R.
World’s largest CA Final student’s consultancy group: http://groups.yahoo.com/group/costingbyparaggupta
Learning Curve Theory - 348 -

Direct Material 30 `/unit


Direct Labour (1 hour/unit) 20 `/unit
Variable overhead 10 `/unit

Fixed overheads at maximum capacity are `150000.


It is estimated that at the current level of efficiency, each unit requires one hour for the first 5000 units.
Subsequently it is possible to achieve 80% learning rate. The market can absorb the first 5000 units at `100
per unit. What should be the minimum selling price acceptable for an order of 15000 units for a prospective
client? (7 Marks) May/08

[Ans.: Min. Selling Price is `50.40]


Question 14: PQ Ltd. makes and sells a labour-intensive product. Its labour force has a learning rate of 80%,
applicable only to direct labour and not to variable overhead.
The cost per unit of the first product is as follows:
Direct materials 10000
Direct labours 8000 (@`4 per hour)
Variable overheads 2000
Total variable cost 20000
PQ Ltd. has received an order from X Ltd. for 4 units of the product. Another customer, Y Ltd. is also
interested in purchasing 4 units of the product. PQ Ltd. has the capacity to fulfill both the orders. Y Ltd.
presently purchases this product in the market for `17,200 and is willing to pay this price per unit of PQ's
product. But X Ltd. lets PQ choose one of the following options :
(i) A price of `16,500 per unit for the 4 units it proposes to takes from PQ.
Or
(ii) Supply X Ltd.'s idle labour force to PQ, for only 4 units of production, with PQ having to pay only Re. 1 per
labour hour to X Ltd.'s workers. X Ltd.'s workers will be withdrawn after the first 4 units are produced. In this
case, PQ need not use its labour for producing X Ltd.'s requirement. X Ltd. assures PQ that its labour force
also has a learning rate of 80%. In this option, X. Ltd. offers to buy the product from PQ at only `14,000 per
unit.
X and Y shall not know of each other's offer.
If both orders came before any work started, what is the best option that PQ may choose?
Present suitable calculations in favour of your argument. (8 Marks) June/09-N.C.

[Ans.: PQ will earn `2832 extra if it chooses Option (i)]


Question 15: ABC Ltd. is a company engaged in the provision of small quantities of specialized equipment to
industrial customers. The work involved in the manufacture of the output calls for a high level of manual skill,
and since no product has a long technological life, it is thought that a 70% learning curve applies to all output
made by the company.
A customer has asked for the delivery of eight units of a product which has never before been made. An
estimate of the direct cost of the first unit has been derived as follows:
Materials `400
Direct labour (250 hours) 1250
1650
The company adds a contribution margin of 160% on direct cost to cover fixed costs and provide a profit
margin.
Required:
(a) State the formula for the 70% learning curve; and
(b) Use the formula to calculate the estimated direct costs and sales price of the ten units of product.

[Ans.: 2033.59 per unit]


Question 16: A company has 10 direct workers, who work for 25 days a month of 8 hours per day. The
estimated down time is 25% of the total available time. The company received an order for a new product.
The first unit of the new product requires 40 direct labour hours to manufacture the product. The company
expects 80% (index is -0.322) learning curve for this type of work. The company uses standard absorption
costing and the cost data are as under:

Direct materials `60 per unit


CA. Parag Gupta Ph.: +91 11 47665555 Paraggupta_ca@yahoo.co.in Costing & O.R.
World’s largest CA Final student’s consultancy group: http://groups.yahoo.com/group/costingbyparaggupta
Operations Research - 349 -

Direct labour `6 per direct labour hour


Variable overheads Re. 1 per direct labour hour
Fixed overheads `7500 per month
Required:
(i) Calculate the cost per unit of the first order of 30 units.
(ii) If the company receives a repeat order for 20 units, what price to be quoted to yield a profit of
25% on selling price? (8 Marks) Nov/02 & CIMA(London)-Adapted

[Ans.: (i) 220.56 (ii) 212.88]


Question 17: ABC Ltd. has designed a new type of sailing boat, for which the cost and sales price of the first
boat to be produced has been estimated as follows:
Materials `5000
Labour (800 hrs. X `5 per hour) 4000
Overhead (150% of labour cost) 6000
15000
Profit mark-up (20%) 3000
Sales price 18000
It is planned to sell all the yachts at full cost plus 20%. An 80% learning curve is expected to apply to the
production work. A customer interest in buying the yachts, but thinks `18000 is too high price to pay. He might
want to buy 2, or even 4 of the yachts during the next six months.
He has asked the following questions:
(a) If he paid `18000 for the first yacht, what price would he have to pay later for the second yacht?
(b) Could ABC Ltd. quote same unit price for two yachts, if the customer ordered two at the same time?
(c) If the customer bought two yachts now at one price, what would be the price per unit for a third and a
fourth yacht, if he ordered them separately later on?
(d) Could ABC Ltd. quote a single unit price for (i) four yachts; (ii) eight yachts; if they were all ordered
now.
Assuming that there are no other prospective customers for the yachts, how would these questions be
answered?

[Ans.: (a) 13200 (b) 15600 (c) 11760 (d) 13680 and 12144]
rd th
[Hint: In part (c) 2 yachts are being ordered at same time, so price for 3 & 4 yacht will be 23520÷ 2]
Question 18: An electronics firm which has developed a new type of fire-alarm system has been asked
to quote for a prospective contract. The customer requires separate price quotations for each of the
following possible orders:

Order Number of fire-alarm systems


First 100
Second 60
Third 40

The firm estimates the following cost per unit for the first order:

Direct materials `500


Direct labour
Deptt. A (Highly automatic) 20 hours at `10 per hour
Deptt. B (Skilled labour) 40 hours at `15 per hour
Variable overheads 20% of direct labour
Fixed overheads absorbed:
Deptt. A `8 per hour
Deptt. B `5 per hour

Determine a price per unit for each of the three orders, assuming the firm uses a mark up of 25% on total
costs and allows for an 80% learning curve. Extract from 80% Learning curve table:

X 1.0 1.3 1.4 1.5 1.6 1.7 1.8 1.9 2.0


Y% 100.0 91.7 89.5 87.6 86.1 84.4 83.0 81.5 80.0

CA. Parag Gupta Ph.: +91 11 47665555 Paraggupta_ca@yahoo.co.in Costing & O.R.
World’s largest CA Final student’s consultancy group: http://groups.yahoo.com/group/costingbyparaggupta
Learning Curve Theory - 350 -

X represents the cumulative total volume produced to date expressed as a multiple of the initial order. Y is the
learning curve factor, for a given X value, expressed as a percentage of the cost of the initial order.
(11 Marks) May/10-N.C., (8 Marks) May/05, ICWA Nov/05 & CIMA – Adapted
[Ans.: (i) Price (P.V) of 1st order (100 units) = 2275 (ii) (P.V) of 2nd order (60 units) = 1848.64 (iii) (P.V) of 3rd
order (40 units) = 1764.40]
Question 19: A company has accepted an order for making 15 items of a specialized machine at a price of
`4 lacs each. The delivery is to be completed within 4 months. The company works 23 days a month and the
normal direct wages per day amounts to `10000. However, in case of need, the company can work overtime
up to 8 days during the said period at double the normal rate of wages. Overheads amount to `12000 per
normal working day but no overheads are charged on overtime working days. The material cost is `240000
per machine. The company has estimated that it will take 10 working days to manufacture the first machine.
The company is expected to experience a learning effect of 90% (b=-0.152). The contract stipulates a penalty
of `40000 per machine delivered beyond the schedule of 4 months.
You are required to calculate the costs and advise the company whether it is preferable to work only during
normal working days and pay penalty for any delayed delivery of the machines or to work overtime to avoid
paying penalty. (ICWA June, 2000)

[Ans.: Additional profit is `6250]


Question 20: A firm has developed a product for which the following standard cost estimates have been
made for first batch to be manufactured in Month 1:
Standard Cost for the batch
500 labour hours @ 8 per hour `4000
55 units of direct materials @ `100 per unit 5500
Variable overhead 500 hours @ 15 per hour 7500
17000
From experience the firm knows that labour will benefit from a learning benefit and labour time will be
reduced. This is expected to approximate to an 80% learning curve and to follow the general function.
y = axb
Where y = average labour hours per batch
a = number of labour hours for the first batch
x = cumulative number of batches
and b = learning coefficient.
(The learning coefficient is found as follows:
b = log (1 – proportionate decrease) ÷ log 2
The coefficient for an 80% learning curve is b = -0.322.)
In addition, the growing expertise of labour is expected to improve the efficiency with which materials are
used. The usage of materials is expected to approximate to a 95% learning curve and to follow the general
function.
y = axb
Where y = average material quantity per batch
a = number of material quantity for the first batch
and x and b are explained previously.
The actual production for the first six months was as follows:
Month 1 20 batches Month 4 24 batches
Month 2 30 batches Month 5 33 batches
Month 3 25 batches Month 6 28 batches
During Month 6 the following results were recorded for the last batch made:
Actual results of last batch
Labour hours 115
Direct wages `978
Direct materials (41 units) 3977
Variable overhead 1685
You are required:
(a) to calculate the learning coefficient for materials;
(b) to derive the Standard Cost of the last batch in Month 6;
(c) to calculate what variances have arisen in connection with the last batch;
(d) to explain what information the variances provide for management.

CA. Parag Gupta Ph.: +91 11 47665555 Paraggupta_ca@yahoo.co.in Costing & O.R.
World’s largest CA Final student’s consultancy group: http://groups.yahoo.com/group/costingbyparaggupta
Operations Research - 351 -

[Ans.: (a) -0.074 (b) `6021 (c) LRV = 58V, LEV = 140A, MPV = 123F, MVV = 322A, VOExp. V = 40F, VOEff.
V = 262A]

Please go through concepts of Absorption Costing before doing Question 16 and


Question 19, through Standard Costing before doing Question 20, & through Direct
Costing before doing Question 14

CA. Parag Gupta Ph.: +91 11 47665555 Paraggupta_ca@yahoo.co.in Costing & O.R.
World’s largest CA Final student’s consultancy group: http://groups.yahoo.com/group/costingbyparaggupta
Time Series Analysis
& Forecasting

Forecasting is a basic tool to help managerial decision making. Managerial decisions are seldom made in the
absence of some form of forecasting. For e.g. a manager is interested in finding out his likely sales in the year
2010 or as a long-term planning in 2020 so that he could adjust his production accordingly and avoid the
possibility of either unsold stocks or inadequate production to meet the demand. However, the first step in
making estimates for the future consists of gathering information from the past. In this connection one usually
deals with statistical data which are collected, observed or recorded at successive intervals of time. Such data
are generally referred to as ‘time series’. For e.g., if we observe production, sales, population, imports,
exports, etc. at different point of time, say, over the last 5 or 10 years, the set of observations formed shall
constitute time series.

Utilities of Time Series Analysis:


(1) It helps in evaluating current accomplishments.
(2) It helps in planning future operations.
(3) It facilitates comparison.
(4) It helps in understanding past behavior.

Various Components of Time Series:


A time series is the result of a number of movements which are caused by numerous economic, political,
natural and other factors. The analysis of time series means decomposing the past data into components and

CA. Parag Gupta Ph.: +91 11 47665555 Paraggupta_ca@yahoo.co.in Costing & O.R.
World’s largest CA Final student’s consultancy group: http://groups.yahoo.com/group/costingbyparaggupta
Operations Research - 353 -

then projecting them forward. A time series typically has four components, though on occasions only one or
two of these may eclipse the others.
1. Secular Trend (T) (a.k.a. Long term trend) - Trend is the general direction in which something tends to
move. It is the relatively consistent movement of a variable over a long period. For e.g. a glance at the sales
of a popular soft drink manufacturer is likely to reveal an increasing trend.
2. Seasonal Variations (S) - Seasonal variations are those periodic movements in business activity which
occur regularly within a definite period, may be every week or month or quarter. Since these variations repeat
during a period of 12 months they can be predicted fairly accurately. For example, toy manufacturers have
sales increases before Christmas.
3. Cyclical Variations (C) - These are caused by business cycles or trade cycles. Cyclic fluctuations are
long-term movements that represent consistently recurring rise and decline in activity. For e.g. Sales may be
low due to overall subdued economic activity.
4. Irregular Variations (I) (a.k.a. Random movements) – These are residual, or erratic movements that do
not have any set pattern and are usually caused by unpredictable reason, like earthquake, fire, wars, etc.

Models for Decomposition of Time Series


(a) Multiplicative Model: It is assumed that there is a multiplicative relationship between these four
components i.e. it is assumed that any particular value in a series is the product of factors that can be
attributed to the various components. Symbolically, Y = T×S×C×I

(b) Additive Model: In this approach we assumes that all the components of the time series are independent
of one another. For e.g. it assumes that trend has no effect on the seasonal component, no matter how high
or low this value may become. Symbolically, Y = T+S+C+I
Note: In multiplicative model S, C, and I indexes are expressed as decimal per cents.

Question 1: What is trend? What are the various methods of fitting a straight line to a time series?
(3 Marks) Nov/08
Question 2: Name the various methods of fitting a straight line to a time series and briefly explain any two of
them. (5 Marks) June/09
Question 3: Identify the characteristics movement such as regular, irregular, cyclical, seasonal, long-term
trend, short-term etc. of time series in the following situations:
(i) A factory delaying its production due to demolition of factory shed in earthquake.
(ii) An era of Depression in business.
(iii) The country needs more and more food grains due to constant growth of population.
(iv) Decline in death rate due to availability of proper health care facilities.
(v) A continuous increase in demand of small cars.
(vi) A demand of gold products is increasing during the festival time. (3 Marks) May/10

[Ans.: (i) Irregular (ii) Cyclical (iii) Long term trend (iv) Long term trend (v) Long term trend (vi) Seasonal]

Trend Projections

The various methods that can be use for determining trend are:
(a) Freehand or graphic method, (b) Semi-Average method,
(c) Moving average method, and (d) Method of least squares.

(a) Freehand or graphic method: It is the simplest method of fitting a trend line. The procedure of obtaining
a straight line trend by this method is given below.
(i) Plot the time series on the graph.
(ii) Examine carefully the direction of the trend based on the plotted information (dots).
(iii) Draw the straight line which will best fit to the data according to personal judgment. The line now
shows the directions of the trends.

Question 4: Fit a trend line to the following data by the freehand method:

CA. Parag Gupta Ph.: +91 11 47665555 Paraggupta_ca@yahoo.co.in Costing & O.R.
World’s largest CA Final student’s consultancy group: http://groups.yahoo.com/group/costingbyparaggupta
Time series analysis & Forecasting - 354 -

Year Production of steel Year Production of steel


(in million tones) (in million tones)
1987 20 1992 25
1988 22 1993 23
1989 24 1994 26
1990 21 1995 25
1991 23

(b) Semi-Average Method: Here we break the series into two parts, take the average (i.e. arithmetic mean)
of each part. We thus get two points. Each point is plotted at the mid-point of the class interval covered by the
respective part and then connect the points by a straight line.

Note: 1. In case total no. of years are in odd no., then while breaking the series in two parts we will leave out
the middle year, although while preparing graph all years will be taken on x-axis (including middle year).
2. In case the parts formed on breaking the series in two parts are in even no., then while centering the
average, 1st July of middle 2 year will be taken. E.g. two parts formed are [1999, 2000, 2001 & 2002] and
[2003, 2004, 2005 & 2006], in such case average will be centered corresponding to 1st July 2000 & 1st July
2004.
Question 5: Fit a trend line to the following data by the method of semi-average:

Year Sales of Firm A Year Sales of Firm A


(Thousand Units) (Thousand Units)
1989 102 1993 108
1990 105 1994 116
1991 114 1995 112
1992 110

[Hint: Avg. of Ist 3 years is 107 & Avg. of last 3 years is 112]
Question 6: Fit a trend line to the following data by the method of semi-average:

Production from 1999-2006:


Year X Y Year X Y
1999 0 10 2003 4 20
2000 1 12 2004 5 25
2001 2 18 2005 6 23
2002 3 20 2006 7 32

[Hint: Avg. of Ist 4 years is 15 & Avg. of last 4 years is 25]


Question 7: Fit a trend line by the method of semi-averages to the data given below. Estimate the sales for
1996. If the actual sale for that year is Rs. 520 lakhs, account for the difference between the two figures.

Year Sales (Rs. Lakhs) Year Sales (Rs. Lakhs)


1988 412 1992 479
1989 438 1993 482
1990 444 1994 490
1991 454 1995 500
[Hint: Avg. of Ist 4 years is 437 & Avg. of last 4 years is 487.75]
(c) Moving average method: This method consists of taking arithmetic mean of the values of a certain time
span (no. of years or any other period) and placing it at the centre of the time span. We then repeat the
procedure by dropping the first yearly figures of the values and adding the figure of the next which we had
previously not added; thus we move the span and its centre move by one year; then we compute and place a
new average. We continue this until we exhaust the series.

CA. Parag Gupta Ph.: +91 11 47665555 Paraggupta_ca@yahoo.co.in Costing & O.R.
World’s largest CA Final student’s consultancy group: http://groups.yahoo.com/group/costingbyparaggupta
Operations Research - 355 -

Since the moving average method is most commonly applied to data which are characterized by cyclical
movements, it is necessary to select a period for moving average which coincides with the length of the cycle
such as 3-yearly moving average, 5-yearly moving average, etc.

Note: Method of centering the moving averages (for even period):- If we are calculating 4-yearly moving
average we will first take four-yearly totals and of these totals we will again take 2-yearly totals and divide
these totals by 8. Alternatively, we can first take four-yearly totals and of these totals, calculate 4-yearly
moving average by dividing by 4 & then we will again take 2 yearly average out of this 4-yearly moving
average.

Question 8: (a) Calculate the 3-yearly moving averages of the production figures given below to determine
the trend values. Find the short-term fluctuations.

Year Production (In Met. Tons) Year Production (In Met. Tons)
1981 15 1989 63
1982 21 1990 70
1983 30 1991 74
1984 36 1992 82
1985 42 1993 90
1986 46 1994 95
1987 50 1995 102
1988 56

(b) Construct 5-yearly moving averages of the number of students studying in a college shown below:

Year No. of Students Year No. of Students


1987 332 1992 405
1988 317 1993 410
1989 357 1994 427
1990 392 1995 405
1991 402 1996 431

[Ans.: (a) Short term fluctuations:

Years 1982 1983 1984 1985 1986 1987 1988 1989 1990 1991 1992 1993 1994
Multiplicative 95.45 103.45 100.00 101.62 100.00 98.68 99.41 100.00 101.45 98.23 100.00 101.12 99.3
Additive -1 1 0 0.67 0 -0.67 -0.33 0 1 -1.33 0 1 -0.67

(b) Years 1989 1990 1991 1992 1993 1994


5-Year moving avg: 360.0 374.6 393.2 407.2 409.8 415.6]

ICAI’s study material has used multiplicative model in some questions & additive model in other
questions while eliminating trend & calculating seasonal indices, so we can adopt any of the two
models if question is silent, after giving prompt assumption note clarifying the relationship model
used. (Clarified by ICAI via mail - posted on group on 13th Jan’10)

Question 9: Find out the three year moving averages starting from 1989.

Year: 1989 1990 1991 1992 1993 1994 1995 1996 1997 1998 1999 2000
Sales: 10 15 20 25 15 12 15 24 15 24 15 24

[Ans.: Years 1990 1991 1992 1993 1994 1995 1996 1997 1998 1999
3 Year moving avg: 15 20 20 17 14 17 18 20 18 21]

CA. Parag Gupta Ph.: +91 11 47665555 Paraggupta_ca@yahoo.co.in Costing & O.R.
World’s largest CA Final student’s consultancy group: http://groups.yahoo.com/group/costingbyparaggupta
Time series analysis & Forecasting - 356 -

Question 10: Calculate 5-yearly and 7-yearly moving averages for the following data of the number of
commercial and industrial failures in a country during 1980-1995.

Year No. Of Failures Year No. Of Failures


1980 23 1988 9
1981 26 1989 13
1982 28 1990 11
1983 32 1991 14
1984 20 1992 12
1985 12 1993 9
1986 12 1994 3
1987 10 1995 1

Also plot the actual and trend values on a graph. (RTP-Nov/08)

[Ans.: Years 1982 1983 1984 1985 1986 1987 1988 1989 1990 1991 1992 1993
5 Year moving avg: 25.8 23.6 20.8 17.2 12.6 11.2 11 11.4 11.8 11.8 9.8 7.8
7 Year moving avg: - 21.9 20.0 17.6 15.4 12.4 11.6 11.6 11.1 10.1 9.0 -]

Question 11: Estimate the trend values using the data given below by taking a four-yearly moving average
and eliminate the trend:

Year Value Year Value


1983 24.1 1990 45.3
1984 25.1 1991 39.3
1985 27.3 1992 41.3
1986 28.3 1993 42.2
1987 28.1 1994 46.4
1988 29.1 1995 46.6
1989 30.1 1996 49.2

[Ans.: Years 1985 1986 1987 1988 1989 1990 1991 1992 1993 1994
Multiplicative 102.25 102.17 98.42 93.78 87.12 120.86 97.01 97.76 97.66 102.86
Additive 0.60 0.60 -0.45 -1.93 -4.45 7.82 -1.21 -0.86 -1.01 1.29]
Question 12: From the following data calculate 3-yearly, 5-yearly and 7-yearly moving averages and plot
data on the graph:
Year 1981 1982 1983 1984 1985
Cyclical Fluctuations +2 +1 0 -2 -1

Year 1986 1987 1988 1989 1990


Cyclical Fluctuations +2 +1 0 -2 -1

Year 1991 1992 1993 1994 1995


Cyclical Fluctuations +2 +1 0 -2 -1

[Ans.: Years 1982 1983 1984 1985 1986 1987 1988 1989 1990 1991 1992 1993 1994
3 Year moving avg: 1.00 -0.33 -1.00 -0.33 0.67 1.00 -0.33 -1.00 -0.33 0.67 1.00 -0.33 -1.00
5 Year moving avg: - 0 0 0 0 0 0 0 0 0 0 0 -
7 Year moving avg: - - 0.43 0.14 -0.28 -0.43 -0.14 -0.43 0.14 -0.28 -0.43 - -]

d) Method of least squares: (i) Linear Trend: The trend line drawn by this method is the line which is an
average of the movement through time of the original data. The following regression equation for a straight
line can be used to express the relationship:
Y c = a + bX

CA. Parag Gupta Ph.: +91 11 47665555 Paraggupta_ca@yahoo.co.in Costing & O.R.
World’s largest CA Final student’s consultancy group: http://groups.yahoo.com/group/costingbyparaggupta
Operations Research - 357 -

We will solve following equations to yield the values of parameters a and b of the above equation.
ΣY = Na + b ΣX
ΣXY = a ΣX + b ΣX2
 Y c = the trend value (which is to be predicted)
 a = the Y-axis intercept
 b = slope of trend line
 X = the independent variable, the time
 ΣY = summation of value of dependent variable (the variable whose values are to be
forecasted)
 N = No. of data points
 ΣX = summation of value of the independent variable (time in this case)
 ΣXY = summation of products of the X and corresponding values
 ΣX2 = summation of the square of values of the independent variable

Question 13:

Year 1996 1997 1998 1999 2000


Y 270 285 295 315 300

Fit a linear trend to these data using method of least squares:


[Ans.: Y c = 299 +15x (origin: year 1998) (x = 1- year unit)]
Question 14: Below are given the figures of production(in thousand quintals ) of a sugar factory:

Year 1976 1977 1978 1979 1980 1981 1982


Production 80 90 92 83 94 99 92
(In ‘000 qtl.)

(i) Fit a straight line trend to these figures.


(ii) Plot these figures on graph and show the trend line.
(iii) Eliminate the trend.

[Ans.: (i) Y c = 90 +2x (origin: year 1979) (x = 1- year unit)


(iii) Years 1976 1977 1978 1979 1980 1981 1982
Multiplicative 95.24 104.65 104.54 92.22 102.17 105.32 95.83
Additive -4 4 4 -7 2 5 -4]
Question 15: Below are given the figures of production (in thousand quintals) of a sugar factory.

Year Production Year Production


(Thousand Quintals) (Thousand Quintals)
1975 77 1980 91
1977 88 1981 98
1978 94 1984 90
1979 85

(i) Fit a straight line by the ‘least squares’ method and tabulate the trend values.
(ii) Eliminate the trend using additive model. What components of the time series are thus left over?
(iii) What is the monthly increase in the production of sugar? (RTP-Nov/08-Adapted)

[Ans.: (i) Y c = 88.803 + 1.38x (origin: year 1979) (x = 1- year unit); (ii) After eliminating the trend we are left
with cyclical & irregular variations. The seasonal variations will be absent as the data given is annual; (iii) The
monthly increase in the production of sugar is b÷12 i.e. 1.38÷12 = 0.115 thousand qtl]
Question 16: Fit a straight line trend by the method of least squares to the following data. Assuming that the
same rate of change continues, what would be the predicted earnings for the year 1988?

Year 1979 1980 1981 1982 1983 1984 1985 1986


Earnings 38 40 65 72 69 60 87 95
(Rs. In Lakhs)

CA. Parag Gupta Ph.: +91 11 47665555 Paraggupta_ca@yahoo.co.in Costing & O.R.
World’s largest CA Final student’s consultancy group: http://groups.yahoo.com/group/costingbyparaggupta
Time series analysis & Forecasting - 358 -

(RTP-June/09-Adapted)

[Ans.: (i) Y c = 62.0833 + 7.3333x (origin: year 1982) (x = 1- year unit); Alternatively, Y c = 65.75 + 7.3333x
(Origin year: Mid of 1982-83) (Deviations from 1982.5); (ii) 106.087]
Question 17: Year: 1985 1986 1987 1988 1989 1990
Y 15 14 18 20 17 24
Fit a trend to these data using method of least squares.

[Ans.: Y c = 17.2 + 1.6x (origin: year 1987) (x = 1- year unit); Alternatively, Y c = 18 + 1.6x (Origin year: Mid of
1987-88) (Deviations from 1987.5)]
(ii) Non-Linear trend (a.k.a. Higher degree polynomial trends): When we plot original time series data on
a graph preparatory to the analysis we may find that a curved line is more appropriate to the data. Then we
draw a non-linear trend. The simple example of the non-linear trend is the second degree parabola, the
equation of which is written in the form:
Y c = a + bX + cX2
When numerical values of a, b, and c have been derived, the trend value for any year may be computed by
substituting in the equation the value of X for that year. The value of a, b and c can be determined by solving
the following three normal equations simultaneously:
ΣY = Na + b ΣX + c ΣX2
ΣXY = a ΣX + b ΣX2 + c ΣX3
ΣX2Y = a ΣX2 + b ΣX3 + c ΣX4
Note: If trend equations are asked to be converted to monthly values from annual value, we will divide ‘a’ by
12 and ‘b’ by 144 (i.e. 12×12) & in non-linear trend equation, we will divide ‘a’ by 12 and ‘b’ by 144 (i.e.
12×12) ‘c’ by 1728 (i.e. 12×12×12).
Question 18: The prices of a commodity during 1990-95 are given below. Fit a parabola Y=a + bX + cX2 to
these data. Estimate the price of the commodity for the year 1996.

Year Prices Year Prices


1990 100 1993 140
1991 107 1994 181
1992 128 1995 192

Also plot the actual and trend values on the graph.

[Ans.: Y c = 126.657 + 18.042x + 1.786x2 (origin: year 1992) (x = 1- year unit); Likely price of commodity for
the year 1996 is Rs. 227.401]
Question 19: Fit quadratic trend:

Year 1995 1996 1997 1998 1999


Production of 2 4 8 14 22
cars(‘0000)

[Ans.: Y c = 8 + 5x + x2 origin: year 1997) (x = 1- year unit)]


Question 20: The trend of the annual sales of ABC Co. Ltd. is described by the following equation:
Y c = 30 + 3.6 X (origin 1981, X unit= 1 year, Y unit= annual sales)
Convert the equation on monthly basis.
[Ans.: Y c = 2.5 + 0.025x]
Detrending: The process of eliminating the trend is referred to as detrending and the trend itself can be
represented either as a straight line or some type of smooth curve.
What is left after elimination of trend is short term variation, expressed as:
T  S  C I
Under multiplicative model,  S  C I
T
CA. Parag Gupta Ph.: +91 11 47665555 Paraggupta_ca@yahoo.co.in Costing & O.R.
World’s largest CA Final student’s consultancy group: http://groups.yahoo.com/group/costingbyparaggupta
Operations Research - 359 -

And, Under additive model, [T + S + C + I] -T = S + C + I.

Calculation of Seasonal Variations

To obtain a statistical description of pattern of seasonal variation it will be desirable to first free the data from
the effects of trend, cycles & irregular variation. Once the other components have been eliminated, we can
study seasonal variations, which give a clear idea about the relative position of each season on the basis of
which it is possible to plan for the season.
There are many techniques available for computing an index of seasonal variation; following are some of the
most popularly used:
1. Method of Simple Averages (Weekly, Monthly or Quarterly).
2. Ratio-to-Trend method (a.k.a. Percentage-to-Trend method).
3. Ratio-to-Moving Average method (a.k.a. Percentage of Moving Average method).
4. Link Relative Method (a.k.a. Pearson’s method)

1. Method of Simple Averages: Steps for calculating index are:


(a) Arrange data by months, quarters, etc. & find monthly/quarterly totals.
(b) Obtain yearly average for every month/quarter by dividing totals by the no. of years.
(c) Calculate average of monthly/quarterly averages by dividing the total of monthly/quarterly averages by
12/4.
(d) Calculate Seasonal Indices: Multiplicative model:-
Monthly or quarterly average
Seasonal Index   100
Grand average of the months or quarters

Additive model:- Monthly or quarterly average – Grand average of the months or quarters

(e) Seasonal Index Adjustment: Multiplicative model:- If total of seasonal indices, in case of
monthly/quarterly data is not equal to 1200/400, each seasonal index is multiplied by the factor:
1200/400
Sum of seasonal indices
Additive model:- If total of seasonal indices, in case of monthly/quarterly data is not equal to 0, each
seasonal index is subtracted by the factor:
Total of Seasonal Indices
No. of months/qua rters
Note: In Method of Simple averages for calculating seasonal index, we assume that trend is absent.

Question 21: Assuming that trend is absent, determine if there is any seasonality in the data given below:

Year 1st Quarter 2 nd Quarter 3rd Quarter 4th Quarter


1991 3.7 4.1 3.3 3.5
1992 3.7 3.9 3.6 3.6
1993 4.0 4.1 3.3 3.1
1994 3.3 4.4 4.0 4.0

What are the seasonal indices for various quarters?

[Ans.: Quarters 1st 2nd 3rd 4th


Multiplicative 98.66 110.74 95.30 95.30
Additive -0.050 0.400 -0.175 -0.175]
Question 22: Consumption of monthly electric power in millions of kwh for street lighting in a big city during
1990-94 is given below:

CA. Parag Gupta Ph.: +91 11 47665555 Paraggupta_ca@yahoo.co.in Costing & O.R.
World’s largest CA Final student’s consultancy group: http://groups.yahoo.com/group/costingbyparaggupta
Time series analysis & Forecasting - 360 -

Year Jan. Feb. Mar. Apr. May June July August Sept. Oct. Nov. Dec.
1990 318 281 278 250 231 216 223 245 269 302 325 347
1991 342 309 299 268 249 236 242 262 288 321 342 364
1992 367 328 320 287 269 251 259 284 309 345 367 394
1993 392 349 342 311 290 273 282 305 328 364 389 417
1994 420 378 370 334 314 296 305 330 356 396 422 452
Find out seasonal variation by the method of monthly averages.

[Ans.: Months Jan Feb Mar Apr May Jun Jul Aug Sep Oct Nov Dec
Multiplicative 116.14 103.88 101.61 91.57 85.44 80.33 82.79 90.05 97.88 109.13 116.52 124.66
Additive 51.1 12.3 5.1 -26.7 -46.1 -62.3 -54.5 -31.5 -6.7 28.9 52.3 78.1]

Question 23: Using the method of monthly averages determine the monthly indices for the following data of
production of a commodity for the year 1979,1980,1981.

Months 1979 1980 1981


Production in lacs of tons
January 12 15 16
February 11 14 15
March 10 13 14
April 14 16 16
May 15 16 15
June 15 15 17
July 16 17 16
August 13 12 13
September 11 13 10
October 10 12 10
November 12 13 11
December 15 14 15

[Ans.: Months Jan Feb Mar Apr May Jun Jul Aug Sep Oct Nov Dec
Multiplicative 104.9 97.5 90.2 112.2 112.2 114.6 119.5 92.6 82.9 78.0 87.8 107.3
- - - -
Additive 0.67 0.33 1.33 1.67 1.67 2 2.67 -1 2.33 -3 1.67 1
2. Ratio-to-Trend Method: Steps for calculating index are:
(a) By using the method of least squares, the trend values are obtained.
(b) Multiplicative model:- The original time series is to be divided by the trend values obtained earlier. These
figures are to be transformed into percentages. Now these percentage figures have three components of the
time series, viz. seasonal, cyclical and irregular as the trend has been eliminated.
Additive model:- The trend value obtained should be subtracted original time series.
(c) These figures are to be averaged for each month or quarter or for any other time period in which the
original data are available. This process will eliminate the effects of both cyclical and irregular movements. It
may be noted that while averaging the figures, median should be preferred to the arithmetic mean. This is
because the latter gives undue weightage to extreme values, which are mainly on account of seasonal
swings.
(d) Seasonal Index Adjustment: Multiplicative model:- If total of seasonal indices, in case of monthly/quarterly
data is not equal to 1200/400, each seasonal index is multiplied by the factor:
1200/400
Sum of seasonal indices

CA. Parag Gupta Ph.: +91 11 47665555 Paraggupta_ca@yahoo.co.in Costing & O.R.
World’s largest CA Final student’s consultancy group: http://groups.yahoo.com/group/costingbyparaggupta
Operations Research - 361 -

Additive model:- If total of seasonal indices, in case of monthly/quarterly data is not equal to 0, each seasonal
index is subtracted by the factor:
Total of Seasonal Indices
No. of months/qua rters

Question 24: Find seasonal variation by the ratio-to-trend method for the following data:

Year Quarters
Ist 2nd 3rd 4th
1997 75 60 54 59
1998 88 65 63 80
1999 95 74 66 85
2000 100 78 73 93
2001 118 100 88 110

[Ans.: Quarters 1st 2nd 3rd 4th


Multiplicative 122.96 94.07 83.14 99.83
Additive 17.6 -4.6 -13.6 0.6]
Question 25: Find seasonal variation by the ratio-to-trend method from the data given below:

Year 1st Quarter 2 nd Quarter 3rd Quarter 4th Quarter


1990 30 40 36 34
1991 34 52 50 44
1992 40 58 54 48
1993 54 76 58 62
1994 80 92 86 82

[Ans.: Quarters 1st 2nd 3rd 4th


Multiplicative 92.048 117.364 102.125 88.463
Additive -3.9 9.1 1.3 -6.5]
Question 26: Following data relate to sales of TULSIAN Ltd.

Year Quarterly Sales (Rs. Lakhs)


Ist 2nd 3rd 4th
1997 8 16 24 32
1998 48 36 24 12
1999 48 16 32 64
2000 72 108 144 36
2001 56 28 84 112

Required: Seasonal indices by Ratio to Trend Method.

[Ans.: Quarters 1st 2nd 3rd 4th


Multiplicative 107.90725 89.65125 111.72525 90.71125]
3. Ratio-to-Moving Average Method: Steps for calculating index are:
(a) Calculate 12-months/4-quarters moving total for the time series
(b) Compute 12-months/4-quarters moving average by dividing each total by 12/4 & centre the moving
averages, it will smooth out S × I and the remainder will be T × C. In other words, the use of the moving
average gives the trend and cycle, T × C.
(c) Calculate the percentage of the actual value to the moving-average value for each month/quarter in the
T  S  C I
time series having a 12-month/4-quarter average entry. Symbolically,  S I
TC
CA. Parag Gupta Ph.: +91 11 47665555 Paraggupta_ca@yahoo.co.in Costing & O.R.
World’s largest CA Final student’s consultancy group: http://groups.yahoo.com/group/costingbyparaggupta
Time series analysis & Forecasting - 362 -

(d) Collect all the percentage of actual to moving-average values & arrange them by months/quarters. These
figures are to be averaged for each month or quarter. This process will eliminate the effects of irregular
movements. It may be noted that while averaging the figures, median should be preferred to the arithmetic
mean. This is because the latter gives undue weightage to extreme values, which are mainly on account of
seasonal swings. Sometimes modified mean is used as an average for each month. The modified mean is
calculated by discarding the highest and lowest values for each quarter and averaging the remaining values.
S I
Symbolically, S
I
(e) Seasonal Index Adjustment: If total of seasonal indices, in case of monthly/quarterly data is not equal to
1200/400, each seasonal index is multiplied by the factor:
1200/400
Sum of seasonal indices
Question 27: Calculate seasonal indices by the ‘ratio to moving average’ method from the following data:
Year 1st Quarter 2 nd Quarter 3rd Quarter 4th Quarter
1992 68 62 61 63
1993 65 58 66 61
1994 68 63 63 67

[Ans.: Quarters 1st 2nd 3rd 4th


Multiplicative 105.30 95.21 100.97 98.52
Additive 3.359375 -3.015625 0.609375 -0.953125]
Question 28: Apply ratio to moving average method to calculate seasonal indices from the following data:

Year 1985 1986 1987 1988


January 10 11 10 12
February 12 11 12 13
March 13 12 11 13
April 15 13 12 15
May 16 14 13 16
June 16 14 15 18
July 17 15 15 20
August 18 15 17 20
September 18 15 18 21
October 19 16 20 22
November 22 18 22 24
December 22 20 24 25

[Ans.: Months Jan Feb Mar Apr May Jun Jul Aug Sep Oct Nov Dec
Multiplicative* 70.25 74.76 75.46 83.99 88.81 96.24 103.16 106.87 109.78 116.71 133.47 140.49
*Calculation of Seasonal index is based on Median]
4.The Link Relative Method: This method is relatively complex as it involves the averaging of the link
relatives. The use of this method to calculate seasonal indices involves the following steps:
(a) As a first step, link relatives of the seasonal figures are to be calculated. A link relative is the value of one
season expressed as a percentage of the preceding season.
Figure for current month/quar ter
Link relative   100
Figure for preceding month/quar ter
(b) The link relatives for each month or quarter are to be averaged. For this purpose, we can use either the
mean or the median. As the arithmetic mean gives undue importance to extreme values, the median is
preferable.
(c) The average link relatives are now to be converted into chain relatives on the basis of the first season
(month or quarter).
The formula to calculate chain relatives is:
Link relative of the current month/quar ter  Chain relative of the preceding month/quar ter
100
As this will not add to 100 due to trend effect, some adjustment is needed.
CA. Parag Gupta Ph.: +91 11 47665555 Paraggupta_ca@yahoo.co.in Costing & O.R.
World’s largest CA Final student’s consultancy group: http://groups.yahoo.com/group/costingbyparaggupta
Operations Research - 363 -

(d) The adjusted chain relatives will be calculated by subtracting correction factor (CF) from chain relatives.
This correction factor is used on the assumption that there is a linear trend. Correction factor for every
month/quarter will be calculated in following way:
In monthly series, it will be
For January: 0÷12 of [(tentative) Chain relative for January-100]
For February: 1÷12 of [(tentative) Chain relative for January-100]
For March: 2÷12 of [(tentative) Chain relative for January-100], and so forth.

In quarterly series, it will be


For 1st Quarter: 0÷4 of [(tentative) Chain relative for 1st Quarter-100]
For 2nd Quarter: 1÷4 of [(tentative) Chain relative for 1st Quarter -100]
For 3rd Quarter: 2÷4 of [(tentative) Chain relative for 1st Quarter -100], and so forth.
(e)This is the final step wherein we have to ensure that the adjusted chain relatives add up to 1,200 if the data
are monthly or to 400 if the data are quarterly. The figures thus arrived at are the required seasonal indices.

Question 29: Apply the method of link relatives to the following data and calculate seasonal indices.

Year 1st Quarter 2 nd Quarter 3rd Quarter 4th Quarter


1990 6.0 6.5 7.8 8.7
1991 5.4 7.9 8.4 7.3
1992 6.8 6.5 9.3 6.4
1993 7.2 5.8 7.5 8.5
1994 6.6 7.3 8.0 7.1

[Ans.: Quarters 1st 2nd 3rd 4th (RTP-June/09-Adapted)


Seasonal Indices 88.18 94.01 113.21 104.60]
Question 30: Calculate the seasonal indices from the following data using the link relative method.

Year Q1 Q2 Q3 Q4
1995 65 58 56 61
1996 68 63 63 67
1997 70 59 56 52
1998 60 55 51 58

[Ans.: Quarters 1st 2nd 3rd 4th


Seasonal indices 109.33 97.78 93.89 99.00]

Deseasonalized Data

By deseasonalized data we mean the data which show how things would have been or would be if there were
no seasonal fluctuations. Before we can identify either the trend or cyclical components of time series, we
must eliminate seasonal variation. To deseasonalize a time series, we divide each of the actual values in the
series by the appropriate seasonal index (expressed as a fraction of 100).
T  S  C I
Under multiplicative model,  T  C I
S
And, Under additive model, [T + S + C + I] -S = T + C + I.
Question 31: Deseasonalize the following data with the help of the seasonal data given against

Quarters Cash Balance (`000 Rs.) Seasonal Index


Jan - March 360 90
April – June 550 110
July – Sept. 490 100
Oct – Dec 600 120

[Ans.: 400, 500, 490, 500]

CA. Parag Gupta Ph.: +91 11 47665555 Paraggupta_ca@yahoo.co.in Costing & O.R.
World’s largest CA Final student’s consultancy group: http://groups.yahoo.com/group/costingbyparaggupta
Time series analysis & Forecasting - 364 -

ICAI’s study material has used multiplicative model in some questions & additive model in other
questions while eliminating trend & calculating seasonal indices, so we can adopt any of the two
models if question is silent, after giving prompt assumption note clarifying the relationship model
used. (Clarified by ICAI via mail - posted on group on 13th Jan’10)

Forecasting

Qualitative Forecasting Quantitative Forecasting


Useful when historical data are unavailable. Uses historical data.

Time Series Casual Forecasting


Involves projection of future values of a Involves determination of
variable based entirely on the past and factors which relate to the
present observation of that variable. variable to be predicted.

Various forecasting methods using time series.


(i) Mean Forecast: In this method we forecast the value of the series to be equal to the mean of the
series. This method is not adequate as trend effects and the cyclical effects are not taken into account in
this.
(ii) Naïve forecast: In this method, high correlation between successive pairs of values in a time series
is assumed & we forecast the value, for the time period t, to-be equal to the actual value observed in the
previous period t that is, time period (t–1):
(iii) Linear Trend Forecast: In this method , a linear relationship between the time and the response
value has been found from the linear relationship.
Y t = a + bX
where X will be found from the value of t and a and b are constants.
(iv) Non-linear Trend Forecast: In this method, a non-linear relationship between the time and the
response value has been found again by least-squares method. Then the value, for the time period t, will
be calculated from the non-linear equation . i.e.,
Y t = a + bX + cX2
where X-value will be calculated from the value of t.
(v) Forecasting will Exponential Smoothing: Dealt later.

Question 32: Discuss various forecasting methods using time series. (5 Marks) Nov./10

Exponential Smoothing: This is a very popular scheme to produce a smoothed Time Series. Whereas in
Moving Averages the past observations are weighted equally, Exponential Smoothing assigns exponentially
decreasing weights as the observation get older. In other words, recent observations are given relatively more
weight in forecasting than the older observations.
Making Forecast,
u t = u t-1 + α(Y t-1 -u t-1 )
Also, Current Forecast = Last period’s forecast + α(Last period’s actual value – Last period’s forecasted
value)
Also, Current Forecast = Last period’s forecast + α(Forecasting error)
Where, α is smoothing constant & 0≤α≤1.
Choice of α: In selecting an appropriate value of the smoothing constant, the objective is to obtain the most
accurate forecast. The difference between an actual value and a forecast value is called forecast error. A
measure of overall error of the forecasts made is the mean of square differences between actual and
forecasted values i.e. Mean Squared Error, MSE

 Y  ut-1    forecast error 


2 2
t-1
MSE  
n n

CA. Parag Gupta Ph.: +91 11 47665555 Paraggupta_ca@yahoo.co.in Costing & O.R.
World’s largest CA Final student’s consultancy group: http://groups.yahoo.com/group/costingbyparaggupta
Operations Research - 365 -

Question 33: The actual figures for the sales of bleaching, powder for the year 2008 are shown below in
the table below. The forecasts or the estimates using Exponential smoothing are required , α =0.3.
Initial forecast may be taken as 29.6.

Month Jan. Feb Mar Apr May Jun Jul Aug Sep Oct Nov Dec
Sales 33 31 34 32 37 36 34 32 41 44 44 50

[Ans.: Month Jan Feb Mar Apr May Jun Jul Aug Sep Oct Nov Dec
Forecast 29.6 30.6 30.7 31.7 31.8 33.4 34.2 34.1 33.5 35.7 38.2 39.9]

Question 34: The demand for a particular item during the ten months of a year is as given below. The
manager is considering how well the exponential smoothing serves as an appropriate technique in forecasting
the demand of this item. She is testing three values of the smoothing constant α = 0.2, α = 0.5 & α = 0.8. You
are required to (a) calculate forecasted value using each of the given values, assuming the initial forecast as
208, and (b) calculate MSE for each of these series of estimates and suggest which of them is most
appropriate.

Month 1 2 3 4 5 6 7 8 9 10
Sales 213 201 198 207 220 232 210 217 212 225

[Ans.: Months 1 2 3 4 5 6 7 8 9 10 11 MSE


α = 0.2 Forecast 208.0 209.0 207.4 205.5 205.8 208.7 213.3 212.7 213.5 213.2 215.6 109.501
α = 0.5 Forecast 208.0 210.5 205.8 201.9 204.4 212.2 222.1 216.1 216.5 214.3 219.6 111.946
α = 0.8 Forecast 208.0 212.0 203.2 199.0 205.4 217.1 229.0 213.8 216.4 212.9 222.6 120.922

Since MSE is least for α = 0.2, therefore, this smoothing constant is most preferable.]

Trend-adjusted Exponential smoothing:


u t = u t-1 + α(Y t -u t-1 )
t   (ut -ut-1 ) + (1- )t-1
 1 
ut'  ut    1 t
  
Here, u t gives the smoothened average at time period t t gives smoothed trend & ut' gives the necessary
forecast for the next period at time period t.

Question 35: Exponential Smoothing for series with Trend.


Assumed : Initial Trend = -3.00, Initial Average = 750.00, α = 0.1

Period 0 1 2 3 4 5 6 7 8 9 10 11 12
Sales 720 670 680 740 720 940 1020 1220 1260 1300 1190 1080

[Ans.: Year 2 3 4 5 6 7 8 9 10 11 12
Forecast 717 704.6 696.2 701.3 701.7 746.3 800.3 886.3 967.3 1043.8 1086.3
Forecast error 47 24.6 -43.79 -18.75 -238.28 -273.71 -419.63 -373.71 -332.74 -146.23 6.3]

CA. Parag Gupta Ph.: +91 11 47665555 Paraggupta_ca@yahoo.co.in Costing & O.R.
World’s largest CA Final student’s consultancy group: http://groups.yahoo.com/group/costingbyparaggupta
Sampling & Test of
Hypothesis

Sampling

Population: All items being considered for study.


When we describe characteristics of population, they are called parameters. E.g. If mean marks in Costing &
O.R. of all CA Final Students are 50, in this case 50 marks is a characteristic of the population all CA Final
Students & can be call a population parameter.

Sample: A part or portion of the population chosen for study.


When we describe the characteristics of a sample, they are called statistics/sample statistics. E.g. If we say
that mean marks in Costing & O.R. of CA Final students studying from Parag Gupta are 65, in this case 65
marks is a characteristic of the sample “Students of Parag Gupta” hence would be sample statistics.
Statisticians use lowercase Roman letters to denote sample statistics & Greek or capital letters for
population parameters.

Key Notations used in Chapter


N = Population Size
n = Sample Size
p = Proportion of Population
p = Proportion of Sample
x = Mean of Sample
μ = Population Mean
Standard Score (a.k.a. standard normal variate / z
z =
transformation/test static)
σ = Population standard deviation
s = Sample standard deviation
x = Standard error of the mean

p = Standard error of proportion


Estimated population standard deviation

[Remember, when population standard deviation
 = is not known, we assume that sample standard
deviation is equal to population standard
deviation].

x = Estimated Standard error of the mean

Sampling distribution of the mean: This is distribution of all the sample means. In statistical terminology,
the sampling distribution we would obtain by taking all the samples of a given size is a theoretical sampling
distribution. Statisticians have developed formulas for estimating the characteristics of these theoretical
sampling distributions, making it unnecessary for us to collect large no. of samples.
CA. Parag Gupta Ph.: +91 11 47665555 Paraggupta_ca@yahoo.co.in Costing & O.R.
World’s largest CA Final student’s consultancy group: http://groups.yahoo.com/group/costingbyparaggupta
Operations Research - 367 -

Standard Error: Variability in sample means (or proportions) is measured in terms of standard errors.
Standard error is the same basic concept as standard deviation; both represent a typical distance from the
mean. Standard error of sample mean/proportion is shorthand notation of “Standard deviation of the
distribution of sample means/proportion”.

Calculation of Standard Error:


When population is finite & When population is infinite
n÷N > 0.05 or n÷N ≤ 0.05
Standard error of the population mean:

1. When σ (population standard deviation is  N n 


known) x   x 
n N 1 n
 
2. When σ (population standard deviation is 
 N n



not known) [  =s] x 
n

N 1
x 
n
Standard error of the population proportion:
p(1  p)
3. When p (population proportion is known) N.A. p 
n
4. When p (population proportion is not 
p(1  p)
known)
N.A. p 
n

As per study material of ICAI & suggested answers Nov’09 (although Suggested answers of
June’09 is contradicting it), whenever we are using single sample t test & when population is infinite
or n÷N ≤ 0.05, formula of calculating standard error will be

 If sample standard deviation is not provided in question and we are supposed to calculate



sample standard deviation for calculating standard error, then x 
n
 s
 If sample standard deviation is already been provided to us, then x 
n 1

Characterizing the normal distribution


Every bell-shaped curve (normal distribution) has certain properties. You can use these properties to help
determine the relative standing of any particular result in the distribution. The following is a list of properties
shared by every normal distribution. These properties are explained in more detail in the following sections.
 The shape of the curve is symmetric.
 It has a bump in the middle, with tails going off to the left and right.
 The mean is directly in the middle of the distribution. The mean of the population is designated by the
Greek letter μ.
 The mean and the median are the same value, due to symmetry.
 The standard deviation represents a typical (almost average) distance between the mean and all of
the data. The standard deviation of the population is designated by the Greek letter σ.
 About 95% of the values are within two standard deviations of the mean.
A normal distribution is symmetric, meaning that if you fold it in half right down the middle, the two halves are
mirror images of each other. Because its curve is symmetric, the mean (the balancing point) and the median
(the point where half of the data lie on either side) are equal, and they both occur at the middle of the
distribution.

CA. Parag Gupta Ph.: +91 11 47665555 Paraggupta_ca@yahoo.co.in Costing & O.R.
World’s largest CA Final student’s consultancy group: http://groups.yahoo.com/group/costingbyparaggupta
Sampling & Test of Hypothesis - 368 -

Variability in a distribution is measured and marked off in terms of number of standard deviations
.
The 68-95-99.7 rule, or three sigma rule, or empirical rule, states that for a normal distribution, almost all
values lie within 3 standard deviations of the mean.
 About 68% (actually, 68.27%)of the values lie within 1 standard deviation of the mean (or between
the mean minus 1 times the standard deviation, and the mean plus 1 times the standard deviation). In
statistical notation, this is represented as: μ ± σ.
 About 95% (actually, 95.45%)of the values lie within 2 standard deviations of the mean (or between
the mean minus 2 times the standard deviation, and the mean plus 2 times the standard deviation).
The statistical notation for this is: μ ± 2σ.
 Almost all (actually, 99.73%) of the values lie within 3 standard deviations of the mean (or between
the mean minus 3 times the standard deviation and the mean plus 3 times the standard deviation).
Statisticians use the following notation to represent this: μ ± 3σ.

Estimates: 1. Point Estimate: Here we assume that sample mean x is the best estimator of the population
mean μ. Further, the most frequently used estimator of the population standard deviation σ is the sample
standard deviation s.
2
 ( x  x)
Variance of Sample, s 
2

n 1
[We use n-1 instead of n so that any biasness (as an estimator of population variance) would tend to be low.]

2. Interval Estimates: An interval estimate describes a range of values within which a population parameter
is likely to lie.

Confidence Level: The probability that we associate with an interval estimate.


Confidence Interval: It is the range of the estimate we are making. Confidence Interval can be expressed as:
For Normal Distribution: x  z x , where

x  z x is Upper limit of confidence level & x  z x is Lower limit of confidence level.

& for Student t Distribution: x  t x , where

x  t x is Upper limit of confidence level & x  t x is Lower limit of confidence level.

Note.: 1. Confidence interval for proportions are same except that p will be used in spite of x .
2. When population standard deviation is not known,  will be used in spite of by µx
x
We will calculate Interval estimates using:
Student t distribution:
 Whenever sample size is 30 or less.
 Population standard deviation is not known.
 Population is normal or approximately normal

Normal Distribution: In all other cases

Question 1: The Greensboro Coliseum is considering expanding its seating capacity and needs to know both
the average number of people who attend events there and the variability in this number. The following are
the attendances (in thousands) at nine randomly selected sporting events. Find point estimates of the mean
and the variance of the population from which the sample was drawn.
8.8 14.0 21.3 7.9 12.5 20.6 16.3 14.1 13.0
CA. Parag Gupta Ph.: +91 11 47665555 Paraggupta_ca@yahoo.co.in Costing & O.R.
World’s largest CA Final student’s consultancy group: http://groups.yahoo.com/group/costingbyparaggupta
Operations Research - 369 -

[Ans.: x = 14.278 thousand people, s2=21.119 (thousands of people)2]

Question 2: In a sample of 400 textile workers, 184 expressed extreme dissatisfaction regarding a
prospective plan to modify working conditions. Because this dissatisfaction was vehement enough to allow
management to interpret plan reaction as being highly undesirable, they were curious about the proportion of
total workers harboring this sentiment. Give the point estimate of this proportion.

[Ans.: 0.46]
Question 3: From a population known to have a standard deviation of 1.4, a sample of 60 individuals is
taken. The mean for this sample is found to be 6.2.
(a) Find the standard error of the mean.
(b) Establish an interval estimate around the sample mean, using one standard error of the mean.

[Ans.: (a) 0.181 (b) 6.2 ± 0.181]


Question 4: For a population with a known variance of 185, a sample of 64 individuals leads to 217 as an
estimate of the mean.
(a) Find the standard error of the mean.
(b) Establish an interval estimate that should include the population mean 68.3 percent of the time.

[Ans.: (a) 1.70 (b) 217 ± 1.70]


Question 5: The manager of the Neuse River Bridge is concerned about the number of cars “running” the toll
gates and is considering altering the toll-collection procedure if such alteration would be cost-effective. She
randomly sampled 75 hours to determine the rate of violation. The resulting average violations per hour were
7. If the population standard deviation is known to be 0.9, estimate an interval that has a 95.5 percent chance
of containing the true mean.

[Ans.: 7 ± 0.208]
Question 6: In what way may an estimate be less meaningful because of
(a) A high confidence level?
(b) A narrow confidence level?

[Ans.: (a) High confidence level produces wide intervals, so we sacrifice precision to gain confidence. (b)
Narrow intervals result from low confidence levels, so we sacrifice confidence to gain precision.]
Question 7: Upon collecting a sample of 250 from a population with standard deviation of 13.7, the mean is
found to be 112.4.
(a) Find a 95 percent confidence interval for the mean.
(b) Find a 99 percent confidence interval for the mean.

[Ans.: (a) 112.4 ± 1.697 (b) 112.4 ± 2.234]

Finite Population:
Question 8: From a population of 540, a sample of 60 individuals is taken. From this sample, the mean is
found to be 6.2 and the standard deviation 1.368.
(a) Find the estimated standard error of the mean.
(b) Construct a 96 percent confidence interval for the mean.

[Ans.: (a) 0.167 (b) 6.2 ± 0.342]


Question 9: Joel Friedlander is a broker on the New York Stock Exchange who is curious about the time
between the placement and execution of a market order. Joel sampled 45 orders and found that the mean
time to execution was 24.3 minutes with a standard deviation of 3.2 minutes. Help Joel by constructing a 95
percent confidence interval for the mean time to execution.

[Ans.: 24.3 ± 0.945 minutes]

CA. Parag Gupta Ph.: +91 11 47665555 Paraggupta_ca@yahoo.co.in Costing & O.R.
World’s largest CA Final student’s consultancy group: http://groups.yahoo.com/group/costingbyparaggupta
Sampling & Test of Hypothesis - 370 -

Question 10: Chief of Police Kathy Ackert has recently instituted a crackdown on drug dealers in her city.
Since the crackdown began, 750 of the 12368 drug dealers is Rs.250000. The standard deviation of the dollar
value of drugs for these 750 dealers is Rs.41000. Construct for Chief Ackert a 90 percent confidence interval
for the mean dollar value of drugs possessed by the city’s drug dealers.

[Ans.: Rs.250000 ± Rs.2380]


Proportion:
Question 11: When a sample of 70 retail executives was surveyed regarding the poor November
performance of the retail industry, 65 percent believed that decreased sales were due to unseasonably warm
temperatures, resulting in consumer’s delaying purchase of cold-weather items.
(a) Estimate the standard error of the proportion of retail executives who blame warm weather for low
sales.
(b) Find the upper and lower confidence limits for this proportion, given a confidence level equal to 0.95.

[Ans.: (a) 0.0570 (b) 0.65 ± 0.1117]


Question 12: Michael Jordon, a professional basketball player, shot 200 foul shots and made 174 of them.
(a) Estimate the standard error of the proportion of all foul shouts that Micheal makes.
(b) Construct a 98 percent confidence interval for the proportion of all foul shots that Micheal makes.

[Ans.: (a) 0.0238 (b) 0.87 ± 0.0555]


Question 13: The owner of the Home Loan Company randomly surveyed 150 of the company’s 3000
accounts and determined that 60 percent were in excellent standing.
(a) Find a 95 percent confidence interval for the proportion in excellent standing.
(b) Based on part (a), what kind of interval estimate might you give for the absolute number of accounts
that meet the requirement of excellence, keeping the same 95 percent confidence interval?

[Ans.: (a) 0.6 ± 0.0784 (b) 1800 ± 235.2 accounts]


Question 14: For the following sample sizes and confidence levels, find the appropriate t values for
constructing confidence intervals:
(a) n=28; 95%.
(b) n=8; 98%.
(c) n=10; 95%.
(d) n=25; 99%.
(e) n=10; 99%.

[Ans.; (a) 2.052 (b) 2.998 (c) 2.262 (d) 2.797 (e) 3.250]
Question 15: Northern Orange County has found, much to the dismay of the county commissioners, that the
population has a severe problem with dental plaque. Every year the local dental board examines a sample of
patients and rates each patient’s plaque buildup on a scale from 1 to 100 with 1 representing no plaque and
100 representing a great deal of plaque. This year, the board examined 21 patients and found that they had
an average Plaque Rating Score (PRS) of 72 with a standard deviation of 6.2. Construct for Orange County a
98 percent confidence interval for the mean PRS for Northern Orange County.

[Ans.: 72 ± 3.5]
Question 16: The following sample of eight observations is from an infinite population with normal
distribution:
75.3 76.4 83.2 91 80.1 77.5 84.8 81.0
a) Find the mean.
b) Estimate the population standard deviation.
c) Construct a 98 percent confidence interval for the mean.

[Ans.: (a) 81.1625 (b) 5.1517 (c) 81.1625 ± 5.4606]

Test of Hypothesis
CA. Parag Gupta Ph.: +91 11 47665555 Paraggupta_ca@yahoo.co.in Costing & O.R.
World’s largest CA Final student’s consultancy group: http://groups.yahoo.com/group/costingbyparaggupta
Operations Research - 371 -

As a consumer in this age of information, when you hear a claim being made (for example, "Our ice cream
was the top choice of 80% of taste testers"), you basically have three options:
 Believe it automatically (or go the other way and reject it outright)
 Conduct your own test to verify or refute the claim

Believing results without question (or rejecting them out of hand) isn't wise; the only times you may want to do
this are when the source has already established a good (or bad) name with you or the result simply isn't that
important (after all, you can't go around checking every single claim that comes your way).

Doing a Hypothesis Test


A hypothesis test is a statistical procedure that's designed to test a claim. Typically, the claim is being made
about a population parameter (one number that characterizes the entire population). Because parameters
tend to be unknown quantities, everyone wants to make claims about what their values may be. For example,
the claim that 25% (or 0.25) of all women have varicose veins is a claim about the proportion (that's the
parameter) of all women (that's the population) who have varicose veins (that's the variable, having or not
having varicose veins).

Defining what you're testing


To get more specific, the varicose vein claim is that the parameter, the population proportion (p), is equal to
0.25. (This claim is called the null hypothesis(H o ).) If you're out to test this claim, you're questioning the claim
and have a hypothesis of your own (called the research hypothesis, or alternative hypothesis). You may
hypothesize, for example, that the actual proportion of women who have varicose veins is lower than 0.25,
based on your observations. Or, you may hypothesize that due to the popularity of high heeled shoes, the
proportion may be higher than 0.25. Or, if you're simply questioning whether the actual proportion is 0.25,
your alternative hypothesis is, "No, it isn't 0.25."

For example, if the claim is that the average time to make a Maggie Noodle is two minutes, the statistical
shorthand notation for the null hypothesis in this case would be as follows: H o : μ = 2.
What's the alternative?
 The population parameter is not equal to the claimed value (H 1 : μ≠ 2).
 The population parameter is greater than the claimed value (H 1 : μ > 2).
 The population parameter is less than the claimed value (H 1 : μ < 2).

How do you know which hypothesis to put in H o and which one to put in H 1 /H a ? Typically, the null
hypothesis says that nothing new is happening; the previous result is the same now as it was before,
or the groups have the same average (their difference is equal to zero). In general, you assume that
people's claims are true until proven otherwise. In general, when hypothesis testing, you set up H o
and H a so that you believe H o is true unless your evidence (your data and statistics) shows you
otherwise.

After you've set up the hypotheses, the next step is to collect your evidence and determine whether your
evidence corroborates the claim made in H o . Remember, the claim is made about the population, but you
can't test the whole population; the best you can usually do is take a unbiased and accurate sample.

This standardized version of your statistic is called a test statistic, and it's the main component of a
hypothesis test. The general procedure for converting a statistic to a test statistic (standard score) in the case
of means/proportions:
Test of Hypothesis : One-Sample Tests
Converting to a Standard Score (a.k.a. standard normal variate / z transformation / test static)
x  H0
z
x
In case of proportions:

CA. Parag Gupta Ph.: +91 11 47665555 Paraggupta_ca@yahoo.co.in Costing & O.R.
World’s largest CA Final student’s consultancy group: http://groups.yahoo.com/group/costingbyparaggupta
Sampling & Test of Hypothesis - 372 -

p  pH0
z
p
pH 0 (1  pH 0 )
where,  p 
n

Two-tail Test

Left-tail Test

Right-tail Test

Type-1 errors - Rejecting a null hypothesis when it is true.


Type-2 errors – Accepting a null hypothesis when it is false.

Interpreting the significance level [a.k.a. alpha level (α)]


The purpose of hypothesis testing is not to question the computed value of the sample statistic but to make a
judgment about the difference between the sample statistic and a hypothesized population parameter.
If significance level is 5% it means that the difference between sample statistic and the hypothesized
population parameter is so large(significant) that it or a larger difference would occur, on the average, only
five or fewer times in every 100 samples when the hypothesized population parameter is correct.

E.g. If difference observed between the sample mean x & the hypothesized population mean μ is 4.5% & our
level of significance is 5%, this means that we will reject the null hypothesis since the difference between
sample statistic and the hypothesized population parameter is so large that it would occur, on the average,
only 4.5 times in every 100 samples (less than 5 times) when the hypothesized population parameter is
correct.

While testing hypothesis, confidence interval is made around H0 & not x . It is rather tested that whether
x
CA.lies within
Parag interval
Gupta or not.
Ph.: +91 11 47665555 Paraggupta_ca@yahoo.co.in Costing & O.R.
World’s largest CA Final student’s consultancy group: http://groups.yahoo.com/group/costingbyparaggupta
Operations Research - 373 -

Summary of Five-Step Process for Hypothesis Testing under the Standardized Scale:
1. Decide whether there is a two-tailed or a one-tailed test. State your hypotheses. Select a level of
significance appropriate for this decision.
2. Decide which distribution (t or z) is appropriate & find the critical value(s) for the chosen level of
significance from appropriate table.
3. Calculate the standard error of the sample statistic. Use the standard error to convert the observed
value of the sample statistic to a standardized value.
4. Sketch the distribution and mark the position of the standardized sample value and the critical
value(s) for the test.
5. Compare the value of the standardized sample statistic with the critical value(s) for this test and
interpret the result.

Question 17: Write a short note on the procedure in hypothesis testing. (5 Marks) Nov./08
Question 18: If we reject a hypothesized value because it differs from a sample statistic by more than 1.75
standard errors, what is the probability that we have rejected a hypothesis that is in fact true.?

[Ans.: 0.0802]
Question 19: Sports and media magnate Ned Sterner is interested in purchasing the Atlanta Stalwarts, if he
can be reasonably certain that operating the team will not be too costly. He figures that average attendance
would have to be about 28500 fans per game to make the purchase attractive to him. Ned randomly chooses
64 home games over the past 4 years and finds from figures reported in Sporting Reviews that average
attendance at these games was 26100. A study he commissioned the last time he purchased a team showed
that the population standard deviation for attendance at similar events had been quite stable for the past 10
years at about 6000 fans. Using 2 standard errors as the decision criterion, should Ned purchase the
Stalwarts? Can you think of any reason(s) why your conclusion might not be valid?
[Ans.: x =26100, x L =27000, xU =30000, so Ned should not purchase the Stalwarts. If σ has increased, the
conclusion might not be valid.]
Question 20: An automobile manufacturer claims that a particular model gets 28 miles to the gallon. The
Environmental Protection Agency, using a sample of 49 automobiles of this model, finds the sample mean to
be 26.8 miles per gallon. From previous studies, the population standard deviation is known to be 5 miles per
gallon. Could we reasonably expect (within 2 standard errors) that we could select such a sample if indeed
the population mean is actually 28 miles per gallon?
[Ans.: x =26.8 mpg, x L =26.57, x U =29.43, so such a sample is not unreasonable.]

Question 21: For the following cases, specify which probability distribution to use in a hypothesis test:

(a) H o :μ = 27, H 1 :μ≠27, x =33,  = 4, n =25
(b) H o :μ = 98.6, H 1 :μ>98.6, x =99.1, σ = 1.5, n =50

(c) H o :μ = 3.5, H 1 :μ<3.5, x =2.8,  = 0.6, n =18
(d) H o :μ = 382, H 1 :μ≠382, x =363, σ = 68, n =12

(e) H o :μ = 57, H 1 :μ>57, x =65,  = 12, n =42

[Ans.: (a) t with 24 df. (b) Normal (c) t with 17 df (d) Normal (e) t with 41df (so we use normal table)]
Large Sample size

Question 22: Atlas Sporting Goods has implemented a special trade promotion for its propane stove and
feels that the promotion should result in a price change for the consumer. Atlas knows that the promotions
should result in a price change for the consumer. Atlas knows that before the promotion began, the average
retail price of the stove was Rs.44.95, with a standard deviation of Rs.5.75. Atlas samples 25 of its retailers
after the promotion begins and finds the mean price for the stove is now Rs.42.95. At a 0.02 significance
level, does Atlas have reason to believe that the average retail price to the consumer has decreased?

CA. Parag Gupta Ph.: +91 11 47665555 Paraggupta_ca@yahoo.co.in Costing & O.R.
World’s largest CA Final student’s consultancy group: http://groups.yahoo.com/group/costingbyparaggupta
Sampling & Test of Hypothesis - 374 -

[Ans.: z = -1.74 ( x =Rs.42.95), z L = -2.05 ( x L =Rs.42.59), so don’t reject H o . Atlas should not believe the
price has decreased]
Question 23: Hinton Press hypothesizes that the life of its largest web press is 14500 hours, with a known
standard deviation of 2100 hours. From a sample of 25 presses, the company finds a sample mean of 13000
hours. At a 0.01 significance level, should the company conclude that the average life of the presses is less
than the hypothesized 14500 hours?

[Ans.: z = -3.57 ( x =13000 hours), z L = -2.33 ( x L =13521 hours), so reject H o . The average life is significantly
less]
Question 24: The average commission charged by full-service brokerage firms on a sale of common stock is
Rs.144, with a standard deviation of Rs.52. Joel Freelander has taken a random sample of 121 trades by his
clients and determined that they paid an average commission of Rs.151. At a 0.10 significance level, can Joel
conclude that his client’s commissions are higher than the industry average?
[Ans.: z = 1.48 ( x =Rs.151), z U = 1.28 ( xU =Rs.150), so reject H o . Their commissions are significantly
higher.]
Question 25: Prior to the 1973 oil embargo and subsequent increases in the price of crude oil, gasoline
usage in the Unites States has grown at a seasonally adjusted rate of 0.57% per month, with a standard
deviation of 0.10% per month. In 15 randomly chosen months between 1975 and 1985, gasoline usage grew
at an average rate of only 0.33% per month. At a level 0.01 level of significance, can you conclude that the
growth in the use of gasoline had decreased as a result of the embargo and its consequences?
[Ans.: z = -9.30 ( x =0.33 percent), z L = -2.33 ( x L =0.51 percent), so reject H o . The growth rate has
decreased significantly.]
Question 26: The data-processing department at a large life insurance company has installed new color
video display terminals to replace the monochrome units it previously used. The 95 operators trained to use
the new machines averaged 7.2 hours before achieving satisfactory level of performance. Their sample
variance was 16.2 squared hours. Long experience with operators on the old monochrome terminals showed
that they averaged 8.1 hours on the machines before their performances were satisfactory. At the 0.01
significance level, should the supervisor of the department conclude that the new terminals are easier to learn
to operate?
[Ans.: Z = -2.1679 ( x =7.2), Z L = -2.33 ( x L =7.13), so don’t reject H o . The new terminals are not easier to
learn to operate.]
Proportions

Question 27: Grant Inc., a manufacturer of women’s dress blouses, knows that its brand is carried in 19
percent of the women’s clothing stores east of the Mississippi River. Grant recently sampled 85 women’s
clothing stores on the West Coast and found that 14.12 percent of the stores carried the brand. At the 0.04
level of significance, is there evidence that Grant has poorer distribution on the West Coast than it does east
of the Mississipi?
[Ans.: z = -1.15 ( p =0.1412), z L = -1.75 ( p L =0.1155), so don’t reject H o . There is no evidence that West
Coast distribution is significantly worse.]

Question 28: Feronetics specializes in the use of gene-splicing techniques to produce new pharmaceutical
compounds. It has recently developed a nasal spray containing interferon, which it believes will limit the
transmission of the common cold within families. In the general population, 15.1 percent of all individuals will
catch a rhinovirus-caused cold once another family member contracts such a cold. The interferon spray was
tested on 180 people, one of whose family members subsequently contracted a rhinovirus-caused cold. Only
17 of the test subjects developed similar colds.
(a) At a significance level of 0.05, should Feronetics conclude that the new spray effectively reduces
transmission of colds?
(b) What should it conclude at α = 0.02?
(c) On the basis of these results, do you think Feronetics should be allowed to market the new spray?
Explain.
[Ans.: (a) z = -2.12 ( p =0.0944), z L = -1.64 ( p L =0.1072), so reject H o . Yes, they should conclude that
transmission is reduced.
CA. Parag Gupta Ph.: +91 11 47665555 Paraggupta_ca@yahoo.co.in Costing & O.R.
World’s largest CA Final student’s consultancy group: http://groups.yahoo.com/group/costingbyparaggupta
Operations Research - 375 -

(b) z L = -2.05 ( p L =0.0963), so the conclusion is unchanged.


(a) Not necessarily: Among other reasons, we have been given no information about potential adverse
side effects of the spray.]
Question 29: A manufacturer claimed that at least 95% of the equipment which he supplied to a factory
conformed to specifications. An examination of a sample of 200 pieces of equipment revealed that 18 were
faulty. Test this claim at a significance level of (i) 0.05 (ii) 0.01. RTP-June/09
[Ans.: (i) z = -2.596 ( p =0.91), z L = -1.64 ( p L =0.925), so reject H o . Hence manufacturer’s claim is rejected.
(ii) z = -2.596 ( p =0.91), z L = -2.33 ( p L =0.914), so reject H o . Hence manufacturer’s claim is rejected.]

Question 30: A ketchup manufacturer is in the process of deciding whether to produce a new extra-spicy
brand. The company’s marketing-research department used a national telephone survey of 6000 households
and found that extra-spicy ketchup would be purchased by 335 of them. A much more extensive study made
2 years ago showed that 5 percent of the households would purchase the brand then. At a 2 percent
significance level, should the company conclude that there is an increased interest in the extra-spicy flavor?
[Ans.: z = 2.07 ( p =0.05583), z U = 2.05 ( p U = 0.05577), so reject H o . Interest has increased significantly
(although just barely).]
Small Sample size
Question 31: Given a sample mean of 83, a sample standard deviation of 12.5, and a sample size of 22, test
the hypothesis that the value of the population mean is 70, against the alternative that it is more than 70. Use
the 0.025 significance level.
[Ans.: t = 4.766 ( x =83), t U = 2.080 ( x U =75.67), so reject H o ]

Question 32: In the past, a machine has produced pipes of diameter 50 mm. To determine whether the
machine is in proper working order, a sample of 10 pipes is chosen, for which mean diameter is 53
mm and the standard deviation is 3 mm. Test the hypothesis that the machine is in proper working order,
given that the critical value of the test statistic from the table is 2.26. (4 Marks) Nov./09-N.C.
[Ans.: t = 3 , t CRIT = ±2.26 ,so reject H o . Machine may not be working properly.]
Question 33: A factory manager contends that the mean operating life of light bulbs of his factory is 4200
hours. A customer disagrees and says it is less.
The mean operating life for a random sample of 9 bulbs is 4000 hours, with a sample standard deviation of
201 hours.
Test the hypothesis of the factory manager, given that the critical value of the test static as per the table is (-)
2.896. (6 Marks) June/09-N.C.
[Ans.: t = -2.81 ( x =4000 hours), t L = -2.896 ( x L =3994.198 hours), so accept H o . The average life is not
significantly less]
Question 34: A potato chips manufacturing company decided that the mean net weight per pack of its
product must be 90 grams. A random sample of 16 packets yields a mean weight of 80 grams with standard
deviation of 17.10 grams. Test the hypothesis that the mean of the whole universe is less than 90, use level of
significance of (a) 0.05(b) 0.01. (5 Marks) Nov./10-N.C.
[Ans.: t = -2.2649 ( x =80 grams); (i) t L = -1.753 ( x L =82.26 grams), so reject H o . The average weight is
significantly less (ii) t L = -2.602 ( x L =78.52 grams), so don’t reject H o . The average weight is not significantly
less]

Question 35: Realtor Elaine Snyderman took a random sample of 12 homes in a prestigious suburb of
Chicago and found the average appraised market value to be Rs.780000, with a standard deviation of
Rs.49000. Test the hypothesis that for all homes in the area, the mean appraised value is Rs.825000, against
the alternative that it is less than Rs.825000. Use the 0.05 level of significance.
[Ans.: t = -3.0456 ( x =780000), t L = -1.796 ( x L =798466), so reject H o ]

Question 36: A television documentary on overeating claimed that Americans are about 10 pounds
overweight on average. To test this claim, eighteen randomly selected individuals were examined, and their
average excess weight was found to be 12.4 pounds, with a sample standard deviation of 2.7 pounds. At a
significance level of 0.01, is there any reason to doubt the validity of the claimed 10-pound value?

CA. Parag Gupta Ph.: +91 11 47665555 Paraggupta_ca@yahoo.co.in Costing & O.R.
World’s largest CA Final student’s consultancy group: http://groups.yahoo.com/group/costingbyparaggupta
Sampling & Test of Hypothesis - 376 -

[Ans.: t = 3.665 ( x =12.4), t CRIT = ±2.898 ( x L =8.102, x U =11.898), so reject H o . The claim doesn’t appear to
be valid.]

Test of Hypothesis :Two-Sample Tests

Hypothesis Testing for differences between means


Difference between sample means → x1  x2
The mean of the sampling distribution of the distribution of the difference between sample means is
symbolized  x1 x 2 Remember it is always equal to  x1   x 2
Large Sample Sizes:
 12  22
 x1 x 2  
n1 n2
Where,  x1 x 2 =Standard error of the difference between two means
 12 = Variance of Population 1
 22 = Variance of Population 2
n1 = Size of sample from Population 1
n2 = Size of sample from Population 2
Similar to way done earlier, if the two population standard deviations are not know, we can estimate the

standard error of the difference between two means by taking  =s & formula for the estimated standard
error of the difference between two means becomes
 2  2

1 2
 x1x 2  
n1 n2

Where,  x1 x 2 = Estimated Standard error of the difference between two means
 2
 1 = Estimated variance of Population 1
 2
 2 = Estimated variance of Population 2
( x 1  x 2 )  ( 1   2 ) H 0
Standardized scale, z  
 x1 x 2

Small Sample Sizes: (i) If sample size is 30 or less but population standard deviation is known: Use normal
distribution & calculate standard error is calculated in same way as that in Large Sample sizes.
(ii) If sample size is 30 or less but population standard deviation is not known: Use t distribution and standard
error is calculated as
2 2
 n1   1  n2   2 1 1
 x1 x 2   
n1  n2  2 n1 n2
Note: Degree of freedom will be n1  n2  2
( x 1  x 2 )  ( 1   2 ) H 0
Standardized scale, t  
 x1x 2
Hypothesis Testing for differences between proportions
pˆ (1  pˆ ) pˆ (1  pˆ )
Estimated standard error of the difference between two proportions, ˆ p  p  
1 2
n1 n2
n1p1  n2 p2
where, Combined proportion of two samples, pˆ 
n1  n2

CA. Parag Gupta Ph.: +91 11 47665555 Paraggupta_ca@yahoo.co.in Costing & O.R.
World’s largest CA Final student’s consultancy group: http://groups.yahoo.com/group/costingbyparaggupta
Operations Research - 377 -

( p1  p 2 )  ( p1  p2 )H0
Standardized scale, z  
 p p
1 2

Large Sample size


Question 37: Two independent samples of observations were collected. For the first sample of 60 elements,
the mean was 86 and the standard deviation 6. The second sample of 75 elements had a mean of 82 and a
standard deviation of 9.
(a) Compute the estimated standard error of the difference between the two means.
(b) Using  = 0.01, test whether the two samples can reasonably be considered to have come from
populations with the same mean.
[Ans.: (a) 1.2961 (b) z = 3.08 ( x1  x2 = 4), z CRIT = ±2.57 (( x1  x2 ) CRIT = ± 3.331), so reject H o ]

Question 38: A sample of 32 money-market mutual funds was chosen on January 1, 1993, and the average
annual rate of return over the past 30 days was found to be 3.23%, with a sample standard deviation of
0.51%. A year earlier, a sample of 38 money-market funds showed an average rate of return of 4.36%, with a
sample standard deviation of 0.84%. Is it reasonable to conclude (at  = 0.05) that money-market interest
rates declined during 1992?
[Ans.: z = 6.92 ( x1  x2 =1.13 percent), zU = 1.64 (( x1  x2 ) U =0.27 percent), so reject H o ]

Question 39: Notwithstanding the Equal Pay Act of 1963, in 1993 it still appeared that men earned more than
women in similar jobs. A random sample of 38 male machine-tool operators found a mean hourly wage of
Rs.11.38, with a standard deviation of Rs.1.84. A random sample of 45 female machine-tool operators found
their mean wage to be Rs. 8.42, with a standard deviation of Rs.1.31. On the basis of these samples, is it
reasonable to conclude (  = 0.01) that the male operators are earning over Rs.2.00 more per hour than the
female operators?
[Ans.: z=2.69 ( x M  x W =Rs.2.96), zU =2.33 (( x M  x W ) U =Rs.2.83), so reject H o . The male operators earn
significantly more than Rs.2.00 above what the female operators earn.]
Small Sample size

Question 40: A consumer-research organization routinely selects several car models each year and
evaluates their fuel efficiency. In this year’s study of two similar subcompact models from two different
automakers, the average gas mileage for 12 cars of brand A was 27.2 miles per gallon, with a standard
deviation of 3.8 mpg. The nine brand B cars that were tested averaged 32.1 mpg, with a standard deviation of
4.3 mpg. At  = 0.01, should it conclude that brand B cars have higher average gas mileage than do brand A
cars?
[Ans.: t = -2.628 ( x A  x B = -4.9 mpg), t L = -2.539 (( x A  x B ) L = -4.4734 mpg), so reject H o . Brand B has
significantly higher mileage.]
Question 41: To celebrate their first anniversary, Randy Nelson decided to buy a pair of diamond earring for
his wife Debbie. He was shown nine pairs with marquise gems weighing approximately 2 carats per pair.
Because of differences in the colors and qualities of the stones, the prices varied from set to set. The average
price was Rs.2990, with a sample standard deviation of Rs.370. He also looked at six pairs with pear-shaped
stones of the same 2-carat approximate weight. These earrings had an average price of Rs.3065, with a
standard deviation of Rs.805. On the basis of this evidence, can Randy conclude (at a significance level of
0.05) that pear-shaped diamonds cost more, on average than marquise diamonds?
[Ans.: t = -0.2267 ( x1  x2 = -Rs.75), t L = -1.771 (( x1  x2 ) L = -Rs.586), so do not reject H o . The pear-shaped
stones are not significantly more expensive.]
Question 42: Because refunds are paid more quickly on tax returns that are filed electronically, the
Commissioner of the Internal Revenue Service was wondering whether refunds due on returns filed by mail
were smaller than those due on returns filed electronically. Looking only at returns claiming refunds, a sample
of 17 filed by mail had an average refund of Rs.563, with a standard deviation of Rs.378. The average refund
claimed on 13 electronically filed return was Rs.958, with a standard deviation of Rs.619. At  = 0.01, do
these data support the commissioner’s speculation?

CA. Parag Gupta Ph.: +91 11 47665555 Paraggupta_ca@yahoo.co.in Costing & O.R.
World’s largest CA Final student’s consultancy group: http://groups.yahoo.com/group/costingbyparaggupta
Sampling & Test of Hypothesis - 378 -

[Ans.: t = -2.084 ( x M  x E = -Rs.395), t L = -2.467 (( x M  x E ) L = -Rs.467.59), so do not reject H o . The data do


not support the commissioner’s speculation.]

Testing Differences between Means with Dependent (or paired) Samples

Question 43: The data below are a random sample of nine firms chosen from the “Digest of Earning
Reports”, in The Wall Street Journal on Feb 6 2008:
(a) Find the mean change in earnings per share between 2007 & 2008.
(b) Find the standard deviation of the change and the standard error of the mean.
(c) Were average earnings per share different in 2007 & 2008? Test at  = 0.02

Firm 1 2 3 4 5 6 7 8 9
2007 earnings per share 1.38 1.26 3.64 3.50 2.47 3.21 1.05 1.98 2.72
2008 earnings per share 2.48 1.50 4.59 3.06 2.11 2.80 1.59 0.92 0.47
[Ans.: t= -0.5428 ( x = -0.19), tCRIT  2.896( xCRIT  1.01) so do not reject H o .]

Question 44: Nine computer-components dealers in major metropolitan areas were asked for their prices on
two similar dot-matrix printers with standard widths and near-letter-quality fonts. The results of this survey are
given below. At  = 0.05, is it reasonable to assert that, on average, the Apson printer is less expensive than
HP Printer?
Dealer 1 2 3 4 5 6 7 8 9
Apson (Rs.) 350 419 385 360 405 395 389 409 375
HP (Rs.) 370 425 369 375 389 385 395 425 400
[Ans.: t = 0.981 ( x = 5.11), tU  1.80( xU  9.69) so do not reject H o . Apson is not significantly less
expensive]
Question 45: The sales data of an item in six shops before and after a special promotional campaign are as
under:
Shops A B C D E F
Before Campaign 53 28 31 48 50 42
After Campaign 58 29 30 55 56 45
Can the compaign be judged to be a success?
Test at 5% level of significance using t-test. (RTP-Nov/08)

[Ans.: t = -2.782, tL  2.015 so reject H o ]

Question 46: Divya is a production supervisor on the disk-drive assembly line at XY Ltd. XY Ltd. recently
subscribed to an easy listening music service at its factory, hoping that this would relax the workers and lead
to greater productivity Divya is skeptical about this hypothesis and fears the music will be distracting, leading
to lower productivity. She sampled weekly production for the same six workers before the music was installed
and after it was installed. Her data are given below. At  = 0.02, has the average production changed at all?
Employee 1 2 3 4 5 6
Week without music 219 205 226 198 209 216
Week with music 235 186 240 203 221 205

[Ans.: t= 0.478 ( x = 2.83), tCRIT  3.365( xCRIT  19.95) so do not reject H o .]

Proportion

Question 47: On Friday, 11 stocks in a random sample of 40 of the roughly 2500 stocks traded on the New
York Stock Exchange advanced; that is, their price of their shares increased. In a sample of 60 NYSE stocks
taken on Thursday, 24 advanced. At  = 0.10, can you conclude that a smaller proportion of NYSE stocks
advanced on Friday than did on Thursday?
[Ans.: z = -1.283 ( p1  p 2  0.125), zL  1.28(( p1  p 2  0.1247) , so reject H o , a smaller proportion
advanced on Friday]
Question 48: A coal-fired power plant is considering two different systems for pollution abatement. The first
system has reduced the emission of pollutants to acceptable levels 68 percent of the time, as determined
CA. Parag Gupta Ph.: +91 11 47665555 Paraggupta_ca@yahoo.co.in Costing & O.R.
World’s largest CA Final student’s consultancy group: http://groups.yahoo.com/group/costingbyparaggupta
Operations Research - 379 -

from 200 samples. The second, more expensive system has reduced the emission of pollutants to acceptable
levels 76 percent of the time, as determined 250 air samples. If the expensive system is significantly more
effective than the inexpensive system in reducing pollutants to acceptable levels, then the management of
power plant will install the expensive system, Which system will be installed if management uses a
significance level of 0.02 in making its decision?
[Ans.: z = -1.89 ( p1  p 2  0.08), zL  2.05(( p1  p 2 )L  0.0869) , so don’t reject H o , install the less
expensive system.]
Question 49: Two different areas of a large eastern city are being considered as sites for day-care centers.
Of 200 households surveyed in one section, the proportion in which the mother worked full-time was 0.52. In
the other section, 40 percent of the 150 households surveyed had mothers working at full-time jobs. At the
0.04 level of significance, is there a significant difference in the proportions of working mothers in the two
areas of the city?
[Ans.: z = 2.23 ( p1  p 2  0.12), zCRIT  2.05(( p1  p 2 )CRIT  0.1105) , so reject H o , proportions are
significantly different]

Chi-square and analysis of variance


Chi-square distribution is use to examine the difference among more than two sample proportions,
ANOVA enables us to test for the significance of the differences among more than two sample means.

(Oi  Ei )2
Chi-square,  2  
Ei
where, Ei =Expected frequency & Oi is Observed frequency

Number of degrees of freedom = (no. of rows-1)×(no. of columns-1)

Question 50: Given the following dimensions for contingency tables, how many degrees of freedom will the
chi-square statistic for each have?
(a) 5 rows, 4 columns.
(b) 6 rows, 2 columns.
(c) 3 rows, 7 columns.
(d) 4 rows, 4 columns.

[Ans.: (a) 12 (b) 5 (c) 12 (d) 9]


Question 51: A brand manager is concerned that her brand’s share may be unevenly distributed throughout
the country. In a survey in which the country was divided into four geographic regions, a random sampling of
100 consumers in each region was surveyed, with the following results:
Region
NE NW SE SW TOTAL
Purchase the brand 40 55 45 50 190
Do not purchase 60 45 55 50 210
Total 100 100 100 100 400

Develop a table of observed and expected frequencies for this problem.


(a) Calculate the sample  2 value.
(b) State the null and alternative hypothesis.
(c) If the level of significance is 0.05, should the null hypothesis be rejected?

[Ans.: (a)  2 = 5.012 (b) H 0 : region and purchasing are independent. H 1 : region and purchasing are
dependent. (c) U2 =7.815]

Question 52: To see if silicon chip sales are independent of where the U.S. economy is in the business cycle,
data have been collected on the weekly sales of Zippy Chippy, a Silicon Valley firm, and on whether the U.S.

CA. Parag Gupta Ph.: +91 11 47665555 Paraggupta_ca@yahoo.co.in Costing & O.R.
World’s largest CA Final student’s consultancy group: http://groups.yahoo.com/group/costingbyparaggupta
Sampling & Test of Hypothesis - 380 -

economy was rising to a cycle peak, at a cycle peak, falling to a cycle trough, or at cycle trough. The results
are:

WEEKLY CHIP SALES


Economy High Medium Low TOTAL
At peak 20 7 3 30
At trough 30 40 30 100
Rising 20 8 2 30
Falling 30 5 5 40
TOTAL 100 60 40 200

Calculate a table of observed and expected frequencies for this problem.


(a) State the null and alternative hypothesis.
(b) Calculate the sample  2 value.
(c) At the 0.10 significance level, what is your conclusion?

[Ans.: (a) H 0 : sales and economy are independent. H 1 : sales and economy are dependent. (b)  2 = 34.597
(c) U2 =10.645, so we reject H 0 ]

Question 53: A newspaper publisher, trying to pinpoint his market’s characteristics, wondered whether
newspaper readership in the community is related to reader’s educational achievement. A survey questioned
adults in the area on their level of education and their frequency of readership. The results are shown in the
following table.

LEVEL OF EDUCATION ACHIEVEMENT


FREQUENCY Professional College High school Did not TOTAL
OF or graduate grad complete high
READERSHIP postgraduate school
Never 10 17 11 21 59
Sometimes 12 23 8 5 48
Morning or 35 38 16 7 96
evening
Both editions 28 19 6 13 66
TOTAL 85 97 41 46 269

At the 0.10 significance level, does the frequency of newspaper readership in the community differ according
to the reader’s level of education?

[Ans.:  2 = 32.855, U2 =14.684, so we reject H 0 Different levels of education do correspond to different
frequencies of readership.]
Question 54: The contingency table below summarize the results obtained in a study conducted by a
research organization with respect to the performance of four competing brands of tooth paste among the
users
Brand A Brand B Brand C Brand D Total
No. of Cavities 9 13 17 11 50
One of five 63 70 85 82 300
More than five 28 37 48 37 150
Total 100 120 150 130 500
Test the hypothesis that incidence of cavities is independent of the brand of the tooth paste used. Use level of
significance 1% and 5%.

[Ans.:  2 = 1.911, U2 =12.59, so we accept H 0. Hence, incidence of cavities is independent of the brand of
the tooth paste used.]

CA. Parag Gupta Ph.: +91 11 47665555 Paraggupta_ca@yahoo.co.in Costing & O.R.
World’s largest CA Final student’s consultancy group: http://groups.yahoo.com/group/costingbyparaggupta
Operations Research - 381 -

Question 55: Given below in the contingency table for production is three shifts and the number of defective
good turn out- Find the value of  2 . Is it possible that the number defective goods depends on the shifts run
by them, No of Shifts:
Shift I Week II Week III Week Total
I 15 5 20 40
II 20 10 20 50
III 25 15 20 60
60 30 60 150
[Ans.:  2 = 3.647, U2 =9.488, so we accept H 0. Hence, the number of defective does not depends on the
shift run by the factory] (RTP-June/09)

Analysis of Variance (ANOVA)


One Way Classification:

Sum of Squares for Columns, SSC =  n( x  x ) 2

SSC
Mean Sum of Squares for Columns, MSC 
c 1
Sum of Squares due to error, SSE =  ( x
i j
ij  x j )2

SSE
Mean Sum of Squares due to error, MSE 
t c
MSC
Variance Ratio, F 
MSE
Degree of Freedom between Columns = c-1
Degree of Freedom between Rows = t-c

where, x = grand mean i.e. mean of means,


c = total no. of columns
r = total no. of rows
i = no. of row
j = no. of column

Question 56: The three samples below have been obtained from national populations with equal variances.
8 7 12
10 5 9
7 10 13
14 9 12
11 9 14
Test the hypothesis at 5% level that the population means are equal.
(The table value of F at 5% level of significance for v 1 =2 and v 2 =12 is 3.88)

[Ans.: F = 4, The table values of F at 5% level of significance for (2, 12) degrees of freedom is 3.88. The
calculated value of F is more than the table value of F. Hence, The null hypotheses is rejected. We, therefore
conclude that the population means are not equal.]
Question 57: Below are given the yield (in kg.) per acre for 5 trial plots of 4 varieties of treatment. Carry out
an analysis of variance and state conclusion
Treatment
Plot no. 1 2 3 4
1 42 48 68 80
2 50 66 52 94
3 62 68 76 78
CA. Parag Gupta Ph.: +91 11 47665555 Paraggupta_ca@yahoo.co.in Costing & O.R.
World’s largest CA Final student’s consultancy group: http://groups.yahoo.com/group/costingbyparaggupta
Sampling & Test of Hypothesis - 382 -

4 34 78 64 82
5 52 70 70 66

[Ans.: F = 8.3, The table values of F at 5% level of significance for (3, 16) degrees of freedom is 3.24. The
calculated value of F is more than the table value of F. Hence, The null hypotheses is rejected. Hence, The
treatment does not have same effect.] (RTP-Nov/08)
[Note: ICAI has assumed that one way classification should be used in above question instead of manifold
classification]
Question 58: The manager of an assembly line in a clock manufacturing plant decided to study how different
speeds of the conveyor belt affect the rate of defective units produced in an 8-hour shift. To examine this, he
ran the belt at four different speeds for five 8-hour shifts each and measured the number of defective units
found at the end of each shift. The results of the study follow:

Defective Units per Shift


Speed 1 Speed 2 Speed 3 Speed 4
37 27 32 35
35 32 36 27
38 32 33 33
36 34 34 31
34 30 40 29
(a) Calculate the mean number of defective units, x , for each speed; then determine the grand mean, x .
(b) Estimate the population variance (the between column variance) (i.e. MSC).
(c) Calculate the variances within the samples and estimate the population variance based upon these
variances (the within column variance) (i.e. MSE).
(d) Calculate the F ratio. At the 0.05 level of significance, do the four conveyor-belt speeds produce the same
mean rate of defective clocks per shift?
[Ans.: (a) x j =36.31, 31,35,31; x = 33.25 (b) 34.5833 (c) 7.375 (d) F= 4.69, F U = 3.24, so we reject H o ]

Manifold Classification:
T2
Correction Factor, C 
r c
(  x j )2
Sum of Squares between columns, SSC   C
nj
(  xi )2
Sum of Squares between rows, SSR   C
ni
r c
Total Sum of Squares, SST   xij2  C
i 1 j 1

Sum of Squares due to Error, SSE = SST-{SSC+SSR}


Degree of Freedom between Columns = c-1
Degree of Freedom between Rows = r-1
Degree of Freedom for total = rc-1
Degree of Freedom between errors = [rc-1]-[(c-1)+(r-1)]

where, T = grand total


c = total no. of columns
r = total no. of rows
i = no. of row
j = no. of column
Question 59: A farmer applies three types of fertilizers on 4 separate plots. The figures on yield per acre are
tabulated below:

CA. Parag Gupta Ph.: +91 11 47665555 Paraggupta_ca@yahoo.co.in Costing & O.R.
World’s largest CA Final student’s consultancy group: http://groups.yahoo.com/group/costingbyparaggupta
Operations Research - 383 -

Yield
Fertilizers/Plots A B C D Total
Nitrogen 6 4 8 6 24
Potash 7 6 6 9 28
Phosphates 8 5 10 9 32
Total 21 15 24 24 84
Find out if the plots are materially different in fertility as also if the three fertilizers make any material
difference in yields.

[Ans.: ANOVA TABLE


Sources of variation d.f S.S MSS Variance Ratio (F)
Rows (Fertilizers) 2 8 MSR=4 2.4
Columns (Plots) 3 18 MSC=6 3.6
Errors 6 10 MSE=1.667
Total 11 36
F (0.05, 3, 6) is 4.76 & F (0.05, 2, 6) is 5.14. The calculated value of F is less than the table value of F. Hence, The
null hypotheses is accepted.]
Question 60: For the following data representing the number of units of production per day turned out by five
workers using from machines, set-up the ANOVA table (Assumed Origin at 20).
Workers Machine Type
A B C D
1. 4 -2 7 -4
2. 6 0 12 3
3. -6 -4 4 -8
4. 3 -2 6 -7
5. -2 2 9 -1 (RTP-June/09)

[Ans.: ANOVA TABLE


Sources of variation d.f S.S MSS Variance Ratio (F)
Rows (Workmen) 4 161.5 40.38 6.576
Columns (Machine) 3 33.8 112.93 18.39
Errors 12 73.7 6.14
Total 19 574
F (0.05, 4, 12) is 3.259 & F (0.05, 3, 12) is 3.49. The calculated value of F is more than the table value of F. Hence,
The null hypotheses is rejected.]

CA. Parag Gupta Ph.: +91 11 47665555 Paraggupta_ca@yahoo.co.in Costing & O.R.
World’s largest CA Final student’s consultancy group: http://groups.yahoo.com/group/costingbyparaggupta
-a-

Table for Areas Under the Standard Normal Curve


from 0 to Z (Type II)
[P (0 ≤ X ≤ x) = n (0 ≤ Z ≤ z)]

z 0.00 0.01 0.02 0.03 0.04 0.05 0.06 0.07 0.08 0.09
0.0 0.0000 0.0040 0.0080 0.0120 0.0160 0.0199 0.0239 0.0279 0.0319 0.0359
0.1 0.0398 0.0438 0.0478 0.0517 0.0557 0.0596 0.0636 0.0675 0.0714 0.0753
0.2 0.0793 0.0832 0.0871 0.0910 0.0948 0.0987 0.1026 0.1064 0.1103 0.1141
0.3 0.1179 0.1217 0.1255 0.1293 0.1331 0.1368 0.1406 0.1443 0.1480 0.1517
0.4 0.1554 0.1591 0.1628 0.1664 0.1700 0.1736 0.1772 0.1808 0.1844 0.1879
0.5 0.1915 0.1950 0.1985 0.2019 0.2054 0.2088 0.2123 0.2157 0.2190 0.2224
0.6 0.2257 0.2291 0.2324 0.2357 0.2389 0.2422 0.2454 0.2486 0.2517 0.2549
0.7 0.2580 0.2611 0.2642 0.2673 0.2704 0.2734 0.2764 0.2794 0.2823 0.2852
0.8 0.2881 0.2910 0.2939 0.2967 0.2995 0.3023 0.3051 0.3078 0.3106 0.3133
0.9 0.3159 0.3186 0.3212 0.3238 0.3264 0.3289 0.3315 0.3304 0.3365 0.3389
1.0 0.3413 0.3438 0.3461 0.3485 0.3508 0.3531 0.3554 0.3577 0.3599 0.3621
1.1 0.3643 0.3665 0.3686 0.3708 0.3729 0.3749 0.3770 0.3790 0.3810 0.3830
1.2 0.3849 0.3869 0.3888 0.3907 0.3925 0.3944 0.3962 0.3980 0.3997 0.4015
1.3 0.4032 0.4049 0.4066 0.4082 0.4099 0.4115 0.4131 0.4147 0.4162 0.4177
1.4 0.4192 0.4207 0.4222 0.4236 0.4251 0.4265 0.4279 0.4292 0.4306 0.4319
1.5 0.4332 0.4345 0.4357 0.4370 0.4382 0.4394 0.4406 0.4418 0.4429 0.4441
1.6 0.4452 0.4463 0.4474 0.4484 0.4495 0.4505 0.4515 0.4525 0.4535 0.4545
1.7 0.4554 0.4564 0.4573 0.4582 0.4591 0.4599 0.4608 0.4616 0.4625 0.4633
1.8 0.4641 0.4649 0.4656 0.4664 0.4671 0.4678 0.4686 0.4693 0.4699 0.4706
1.9 0.4713 0.4719 0.4726 0.4732 0.4738 0.4744 0.4750 0.4756 0.4761 0.4767
2.0 0.4772 0.4778 0.4783 0.4788 0.4793 0.4798 0.4803 0.4808 0.4812 0.4817
2.1 0.4821 0.4826 0.4830 0.4834 0.4838 0.4842 0.4846 0.4850 0.4854 0.4857
2.2 0.4861 0.4864 0.4868 0.4871 0.4875 0.4878 0.4881 0.4884 0.4887 0.4890
2.3 0.4893 0.4896 0.4898 0.4901 0.4904 0.4906 0.4909 0.4911 0.4913 0.4916
2.4 0.4918 0.4920 0.4922 0.4925 0.4927 0.4929 0.4931 0.4932 0.4934 0.4936
2.5 0.4938 0.4940 0.4941 0.4943 0.4945 0.4946 0.4948 0.4949 0.4951 0.4952
2.6 0.4953 0.4955 0.4956 0.4957 0.4959 0.4960 0.4961 0.4962 0.4963 0.4964
2.7 0.4965 0.4966 0.4967 0.4968 0.4969 0.4970 0.4971 0.4972 0.4973 0.4974
2.8 0.4974 0.4975 0.4976 0.4977 0.4977 0.4978 0.4979 0.4979 0.4980 0.4981
2.9 0.4981 0.4982 0.4982 0.4983 0.4984 0.4984 0.4985 0.4985 0.4986 0.4986
3.0 0.4987 0.4987 0.4987 0.4988 0.4988 0.4989 0.4989 0.4989 0.4990 0.4990
3.1 0.4990 0.4991 0.4991 0.4991 0.4992 0.4992 0.4992 0.4992 0.4993 0.4993
3.2 0.4993 0.4993 0.4994 0.4994 0.4994 0.4994 0.4994 0.4995 0.4995 0.4995
3.3 0.4995 0.4995 0.4995 0.4996 0.4996 0.4996 0.4996 0.4996 0.4996 0.4997
3.4 0.4997 0.4997 0.4997 0.4997 0.4997 0.4997 0.4997 0.4997 0.4997 0.4998
3.5 0.4998 0.4998 0.4998 0.4998 0.4998 0.4998 0.4998 0.4998 0.4998 0.4998
3.6 0.4998 0.4998 0.4999 0.4999 0.4999 0.4999 0.4999 0.4999 0.4999 0.4999
3.7 0.4999 0.4999 0.4999 0.4999 0.4999 0.4999 0.4999 0.4999 0.4999 0.4999

CA. Parag Gupta Ph.: +91 9891 432 632 Paraggupta_ca@yahoo.co.in Costing & O.R.
World’s largest CA Final student’s consultancy group: http://groups.yahoo.com/group/costingbyparaggupta
-b-

Student’s t Distribution
The following table provides the values of t  that
correspond to a given upper-tail area  and a
specified number of degree of freedom (i.e. n-1).

Degree of Upper-tail area 


Freedom(ν) 0.25 0.2 0.15 0.1 0.05 0.025 0.01 0.005 0.0025 0.001 0.0005
1 1.000 1.376 1.963 3.078 6.314 12.710 31.820 63.660 127.300 318.300 636.600
2 0.816 1.061 1.386 1.886 2.920 4.303 6.965 9.925 14.090 22.330 31.600
3 0.765 0.978 1.250 1.638 2.353 3.182 4.541 5.841 7.453 10.210 12.920
4 0.741 0.941 1.190 1.533 2.132 2.776 3.747 4.604 5.598 7.173 8.610
5 0.727 0.920 1.156 1.476 2.015 2.571 3.365 4.032 4.773 5.893 6.869
6 0.718 0.906 1.134 1.440 1.943 2.447 3.143 3.707 4.317 5.208 5.959
7 0.711 0.896 1.119 1.415 1.895 2.365 2.998 3.499 4.029 4.785 5.408
8 0.706 0.889 1.108 1.397 1.860 2.306 2.896 3.355 3.833 4.501 5.041
9 0.703 0.883 1.100 1.383 1.833 2.262 2.821 3.250 3.690 4.297 4.781
10 0.700 0.879 1.093 1.372 1.812 2.228 2.764 3.169 3.581 4.144 4.587
11 0.697 0.876 1.088 1.363 1.796 2.201 2.718 3.106 3.497 4.025 4.437
12 0.695 0.873 1.083 1.356 1.782 2.179 2.681 3.055 3.428 3.930 4.318
13 0.694 0.870 1.079 1.350 1.771 2.160 2.650 3.012 3.372 3.852 4.221
14 0.692 0.868 1.076 1.345 1.761 2.145 2.624 2.977 3.326 3.787 4.140
15 0.691 0.866 1.074 1.341 1.753 2.131 2.602 2.947 3.286 3.733 4.073
16 0.690 0.865 1.071 1.337 1.746 2.120 2.583 2.921 3.252 3.686 4.015
17 0.689 0.863 1.069 1.333 1.740 2.110 2.567 2.898 3.222 3.646 3.965
18 0.688 0.862 1.067 1.330 1.734 2.101 2.552 2.878 3.197 3.610 3.922
19 0.688 0.861 1.066 1.328 1.729 2.093 2.539 2.861 3.174 3.579 3.883
20 0.687 0.860 1.064 1.325 1.725 2.086 2.528 2.845 3.153 3.552 3.850
21 0.686 0.859 1.063 1.323 1.721 2.080 2.518 2.831 3.135 3.527 3.819
22 0.686 0.858 1.061 1.321 1.717 2.074 2.508 2.819 3.119 3.505 3.792
23 0.685 0.858 1.060 1.319 1.714 2.069 2.500 2.807 3.104 3.485 3.767
24 0.685 0.857 1.059 1.318 1.711 2.064 2.492 2.797 3.091 3.467 3.745
25 0.684 0.856 1.058 1.316 1.708 2.060 2.485 2.787 3.078 3.450 3.725
26 0.684 0.856 1.058 1.315 1.706 2.056 2.479 2.779 3.067 3.435 3.707
27 0.684 0.855 1.057 1.314 1.703 2.052 2.473 2.771 3.057 3.421 3.690
28 0.683 0.855 1.056 1.313 1.701 2.048 2.467 2.763 3.047 3.408 3.674
29 0.683 0.854 1.055 1.311 1.699 2.045 2.462 2.756 3.038 3.396 3.659
30 0.683 0.854 1.055 1.310 1.697 2.042 2.457 2.750 3.030 3.385 3.646
40 0.681 0.851 1.050 1.303 1.684 2.021 2.423 2.704 2.971 3.307 3.551
50 0.679 0.849 1.047 1.299 1.676 2.009 2.403 2.678 2.937 3.261 3.496
60 0.679 0.848 1.045 1.296 1.671 2.000 2.390 2.660 2.915 3.232 3.460
80 0.678 0.846 1.043 1.292 1.664 1.990 2.374 2.639 2.887 3.195 3.416
100 0.677 0.845 1.042 1.290 1.660 1.984 2.364 2.626 2.871 3.174 3.390
120 0.677 0.845 1.041 1.289 1.658 1.980 2.358 2.617 2.860 3.160 3.373
∞ 0.674 0.842 1.036 1.282 1.645 1.960 2.326 2.576 2.807 3.090 3.291

CA. Parag Gupta Ph.: +91 9891 432 632 Paraggupta_ca@yahoo.co.in Costing & O.R.
World’s largest CA Final student’s consultancy group: http://groups.yahoo.com/group/costingbyparaggupta
-c-

Upper-tail area 
df 0.995 0.99 0.975 0.95 0.9 0.75 0.5 0.25 0.1 0.05 0.025 0.02 0.01 0.005
1 0.000 0.000 0.001 0.004 0.016 0.102 0.455 1.323 2.706 3.841 5.024 5.412 6.635 7.879
2 0.010 0.020 0.051 0.103 0.211 0.575 1.386 2.773 4.605 5.991 7.378 7.824 9.210 10.597
3 0.072 0.115 0.216 0.352 0.584 1.213 2.366 4.108 6.251 7.815 9.348 9.837 11.345 12.838
4 0.207 0.297 0.484 0.711 1.064 1.923 3.357 5.385 7.779 9.488 11.143 11.668 13.277 14.860
5 0.412 0.554 0.831 1.145 1.610 2.675 4.351 6.626 9.236 11.071 12.833 13.388 15.086 16.750
6 0.676 0.872 1.237 1.635 2.204 3.455 5.348 7.841 10.645 12.592 14.449 15.033 16.812 18.548
7 0.989 1.239 1.690 2.167 2.833 4.255 6.346 9.037 12.017 14.067 16.013 16.622 18.475 20.278
8 1.344 1.647 2.180 2.733 3.490 5.071 7.344 10.219 13.362 15.507 17.535 18.168 20.090 21.955
9 1.735 2.088 2.700 3.325 4.168 5.899 8.343 11.389 14.684 16.919 19.023 19.679 21.666 23.589
10 2.156 2.558 3.247 3.940 4.865 6.737 9.342 12.549 15.987 18.307 20.483 21.161 23.209 25.188
11 2.603 3.053 3.816 4.575 5.578 7.584 10.341 13.701 17.275 19.675 21.920 22.618 24.725 26.757
12 3.074 3.571 4.404 5.226 6.304 8.438 11.340 14.845 18.549 21.026 23.337 24.054 26.217 28.300
13 3.565 4.107 5.009 5.892 7.042 9.299 12.340 15.984 19.812 22.362 24.736 25.472 27.688 29.819
14 4.075 4.660 5.629 6.571 7.790 10.165 13.339 17.117 21.064 23.685 26.119 26.873 29.141 31.319
15 4.601 5.229 6.262 7.261 8.547 11.037 14.339 18.245 22.307 24.996 27.488 28.259 30.578 32.801
16 5.142 5.812 6.908 7.962 9.312 11.912 15.339 19.369 23.542 26.296 28.845 29.633 32.000 34.267
17 5.697 6.408 7.564 8.672 10.085 12.792 16.338 20.489 24.769 27.587 30.191 30.995 33.409 35.718
18 6.265 7.015 8.231 9.390 10.865 13.675 17.338 21.605 25.989 28.869 31.526 32.346 34.805 37.156
19 6.844 7.633 8.907 10.117 11.651 14.562 18.338 22.718 27.204 30.144 32.852 33.687 36.191 38.582
20 7.434 8.260 9.591 10.851 12.443 15.452 19.337 23.828 28.412 31.410 34.170 35.020 37.566 39.997
21 8.034 8.897 10.283 11.591 13.240 16.344 20.337 24.935 29.615 32.671 35.479 36.343 38.932 41.401
22 8.643 9.542 10.982 12.338 14.041 17.240 21.337 26.039 30.813 33.924 36.781 37.659 40.289 42.796
23 9.260 10.196 11.689 13.091 14.848 18.137 22.337 27.141 32.007 35.172 38.076 38.968 41.638 44.181
24 9.886 10.856 12.401 13.848 15.659 19.037 23.337 28.241 33.196 36.415 39.364 40.270 42.980 45.559
25 10.520 11.524 13.120 14.611 16.473 19.939 24.337 29.339 34.382 37.652 40.646 41.566 44.314 46.928
26 11.160 12.198 13.844 15.379 17.292 20.843 25.336 30.435 35.563 38.885 41.923 42.856 45.642 48.290
27 11.808 12.879 14.573 16.151 18.114 21.749 26.336 31.528 36.741 40.113 43.195 44.140 46.963 49.645
28 12.461 13.565 15.308 16.928 18.939 22.657 27.336 32.620 37.916 41.337 44.461 45.419 48.278 50.993
29 13.121 14.256 16.047 17.708 19.768 23.567 28.336 33.711 39.087 42.557 45.722 46.693 49.588 52.336
30 13.787 14.953 16.791 18.493 20.599 24.478 29.336 34.800 40.256 43.773 46.979 47.962 50.892 53.672

CA. Parag Gupta Ph.: +91 9891 432 632 Paraggupta_ca@yahoo.co.in Costing & O.R.
World’s largest CA Final student’s consultancy group: http://groups.yahoo.com/group/costingbyparaggupta
-d-

F Table for alpha=.10 .


df for numerator(v 1 )
df for
denominator(v 2 ) 1 2 3 4 5 6 7 8 9 10 12
1 39.863 49.500 53.593 55.833 57.240 58.204 58.906 59.439 59.858 60.195 60.705
2 8.526 9.000 9.162 9.243 9.293 9.326 9.349 9.367 9.381 9.392 9.408
3 5.538 5.462 5.391 5.343 5.309 5.285 5.266 5.252 5.240 5.230 5.216
4 4.545 4.325 4.191 4.107 4.051 4.010 3.979 3.955 3.936 3.920 3.896
5 4.060 3.780 3.619 3.520 3.453 3.405 3.368 3.339 3.316 3.297 3.268
6 3.776 3.463 3.289 3.181 3.108 3.055 3.014 2.983 2.958 2.937 2.905
7 3.589 3.257 3.074 2.961 2.883 2.827 2.785 2.752 2.725 2.703 2.668
8 3.458 3.113 2.924 2.806 2.726 2.668 2.624 2.589 2.561 2.538 2.502
9 3.360 3.006 2.813 2.693 2.611 2.551 2.505 2.469 2.440 2.416 2.379
10 3.285 2.924 2.728 2.605 2.522 2.461 2.414 2.377 2.347 2.323 2.284
11 3.225 2.860 2.660 2.536 2.451 2.389 2.342 2.304 2.274 2.248 2.209
12 3.177 2.807 2.606 2.480 2.394 2.331 2.283 2.245 2.214 2.188 2.147
13 3.136 2.763 2.560 2.434 2.347 2.283 2.234 2.195 2.164 2.138 2.097
14 3.102 2.726 2.522 2.395 2.307 2.243 2.193 2.154 2.122 2.095 2.054
15 3.073 2.695 2.490 2.361 2.273 2.208 2.158 2.119 2.086 2.059 2.017
16 3.048 2.668 2.462 2.333 2.244 2.178 2.128 2.088 2.055 2.028 1.985
17 3.026 2.645 2.437 2.308 2.218 2.152 2.102 2.061 2.028 2.001 1.958
18 3.007 2.624 2.416 2.286 2.196 2.130 2.079 2.038 2.005 1.977 1.933
19 2.990 2.606 2.397 2.266 2.176 2.109 2.058 2.017 1.984 1.956 1.912
20 2.975 2.589 2.380 2.249 2.158 2.091 2.040 1.999 1.965 1.937 1.892
21 2.961 2.575 2.365 2.233 2.142 2.075 2.023 1.982 1.948 1.920 1.875
22 2.949 2.561 2.351 2.219 2.128 2.061 2.008 1.967 1.933 1.904 1.859
23 2.937 2.549 2.339 2.207 2.115 2.047 1.995 1.953 1.919 1.890 1.845
24 2.927 2.538 2.327 2.195 2.103 2.035 1.983 1.941 1.906 1.877 1.832
25 2.918 2.528 2.317 2.184 2.092 2.024 1.971 1.929 1.895 1.866 1.820
26 2.909 2.519 2.307 2.174 2.082 2.014 1.961 1.919 1.884 1.855 1.809
27 2.901 2.511 2.299 2.165 2.073 2.005 1.952 1.909 1.874 1.845 1.799
28 2.894 2.503 2.291 2.157 2.064 1.996 1.943 1.900 1.865 1.836 1.790
29 2.887 2.495 2.283 2.149 2.057 1.988 1.935 1.892 1.857 1.827 1.781
30 2.881 2.489 2.276 2.142 2.049 1.980 1.927 1.884 1.849 1.819 1.773
40 2.835 2.440 2.226 2.091 1.997 1.927 1.873 1.829 1.793 1.763 1.715
60 2.791 2.393 2.177 2.041 1.946 1.875 1.819 1.775 1.738 1.707 1.657
120 2.748 2.347 2.130 1.992 1.896 1.824 1.767 1.722 1.684 1.652 1.601

CA. Parag Gupta Ph.: +91 9891 432 632 Paraggupta_ca@yahoo.co.in Costing & O.R.
World’s largest CA Final student’s consultancy group: http://groups.yahoo.com/group/costingbyparaggupta
-e-

F Table for alpha=.05 .


df for numerator(v 1 )
df for
denominator(v 2 ) 1 2 3 4 5 6 7 8 9 10 12
1 161.448 199.500 215.707 224.583 230.162 233.986 236.768 238.883 240.543 241.882 243.906
2 18.513 19.000 19.164 19.247 19.296 19.330 19.353 19.371 19.385 19.396 19.413
3 10.128 9.552 9.277 9.117 9.014 8.941 8.887 8.845 8.812 8.786 8.745
4 7.709 6.944 6.591 6.388 6.256 6.163 6.094 6.041 5.999 5.964 5.912
5 6.608 5.786 5.410 5.192 5.050 4.950 4.876 4.818 4.773 4.735 4.678
6 5.987 5.143 4.757 4.534 4.387 4.284 4.207 4.147 4.099 4.060 4.000
7 5.591 4.737 4.347 4.120 3.972 3.866 3.787 3.726 3.677 3.637 3.575
8 5.318 4.459 4.066 3.838 3.688 3.581 3.501 3.438 3.388 3.347 3.284
9 5.117 4.257 3.863 3.633 3.482 3.374 3.293 3.230 3.179 3.137 3.073
10 4.965 4.103 3.708 3.478 3.326 3.217 3.136 3.072 3.020 2.978 2.913
11 4.844 3.982 3.587 3.357 3.204 3.095 3.012 2.948 2.896 2.854 2.788
12 4.747 3.885 3.490 3.259 3.106 2.996 2.913 2.849 2.796 2.753 2.687
13 4.667 3.806 3.411 3.179 3.025 2.915 2.832 2.767 2.714 2.671 2.604
14 4.600 3.739 3.344 3.112 2.958 2.848 2.764 2.699 2.646 2.602 2.534
15 4.543 3.682 3.287 3.056 2.901 2.791 2.707 2.641 2.588 2.544 2.475
16 4.494 3.634 3.239 3.007 2.852 2.741 2.657 2.591 2.538 2.494 2.425
17 4.451 3.592 3.197 2.965 2.810 2.699 2.614 2.548 2.494 2.450 2.381
18 4.414 3.555 3.160 2.928 2.773 2.661 2.577 2.510 2.456 2.412 2.342
19 4.381 3.522 3.127 2.895 2.740 2.628 2.544 2.477 2.423 2.378 2.308
20 4.351 3.493 3.098 2.866 2.711 2.599 2.514 2.447 2.393 2.348 2.278
21 4.325 3.467 3.073 2.840 2.685 2.573 2.488 2.421 2.366 2.321 2.250
22 4.301 3.443 3.049 2.817 2.661 2.549 2.464 2.397 2.342 2.297 2.226
23 4.279 3.422 3.028 2.796 2.640 2.528 2.442 2.375 2.320 2.275 2.204
24 4.260 3.403 3.009 2.776 2.621 2.508 2.423 2.355 2.300 2.255 2.183
25 4.242 3.385 2.991 2.759 2.603 2.490 2.405 2.337 2.282 2.237 2.165
26 4.225 3.369 2.975 2.743 2.587 2.474 2.388 2.321 2.266 2.220 2.148
27 4.210 3.354 2.960 2.728 2.572 2.459 2.373 2.305 2.250 2.204 2.132
28 4.196 3.340 2.947 2.714 2.558 2.445 2.359 2.291 2.236 2.190 2.118
29 4.183 3.328 2.934 2.701 2.545 2.432 2.346 2.278 2.223 2.177 2.105
30 4.171 3.316 2.922 2.690 2.534 2.421 2.334 2.266 2.211 2.165 2.092
40 4.085 3.232 2.839 2.606 2.450 2.336 2.249 2.180 2.124 2.077 2.004
60 4.001 3.150 2.758 2.525 2.368 2.254 2.167 2.097 2.040 1.993 1.917
120 3.920 3.072 2.680 2.447 2.290 2.175 2.087 2.016 1.959 1.911 1.834

CA. Parag Gupta Ph.: +91 9891 432 632 Paraggupta_ca@yahoo.co.in Costing & O.R.
World’s largest CA Final student’s consultancy group: http://groups.yahoo.com/group/costingbyparaggupta
-f-

LOGARITHMS
Mean Differences
0 1 2 3 4 5 6 7 8 9
1 2 3 4 5 6 7 8 9
10 0000 0043 0086 0128 0170 0212 0253 0294 0334 0374 4 8 12 17 21 25 29 33 37

11 0414 0453 0492 0531 0569 0607 0645 0682 0719 0755 4 8 11 15 19 23 26 30 34
12 0792 0828 0864 0899 0934 0969 1004 1038 1072 1106 3 7 10 14 17 21 24 28 31
13 1139 1173 1206 1239 1271 1303 1335 1367 1399 1430 3 6 10 13 16 19 23 26 29
14 1461 1492 1523 1553 1584 1614 1644 1673 1703 1732 3 6 9 12 15 18 21 24 27
15 1761 1790 1818 1847 1875 1903 1931 1959 1987 2014 3 6 8 11 14 17 20 22 25

16 2041 2068 2095 2122 2148 2175 2201 2227 2253 2279 3 5 8 11 13 16 18 21 24
17 2304 2330 2355 2380 2405 2430 2455 2480 2504 2529 2 5 7 10 12 15 17 20 22
18 2553 2577 2601 2625 2648 2672 2695 2718 2742 2765 2 5 7 9 12 14 16 19 21
19 2788 2810 2833 2856 2878 2900 2923 2945 2967 2989 2 4 7 9 11 13 16 18 20
20 3010 3032 3054 3075 3096 3118 3139 3160 3181 3201 2 4 6 8 11 13 15 17 19

21 3222 3243 3263 3284 3304 3324 3345 3365 3385 3404 2 4 6 8 10 12 14 16 18
22 3424 3444 3464 3483 3502 3522 3541 3560 3579 3598 2 4 6 8 10 12 14 15 17
23 3617 3636 3655 3674 3692 3711 3729 3747 3766 3784 2 4 6 7 9 11 13 15 17
24 3802 3820 3838 3856 3874 3892 3909 3927 3945 3962 2 4 5 7 9 11 12 14 16
25 3979 3997 4014 4031 4048 4065 4082 4099 4116 4133 2 3 5 7 9 10 12 14 15

26 4150 4166 4183 4200 4216 4232 4249 4265 4281 4298 2 3 5 7 8 10 11 13 15
27 4314 4330 4346 4362 4378 4393 4409 4425 4440 4456 2 3 5 6 8 9 11 13 14
28 4472 4487 4502 4518 4533 4548 4564 4579 4594 4609 2 3 5 6 8 9 11 12 14
29 4624 4639 4654 4669 4683 4698 4713 4728 4742 4757 1 3 4 6 7 9 10 12 13
30 4771 4786 4800 4814 4829 4843 4857 4871 4886 4900 1 3 4 6 7 9 10 11 13

31 4914 4928 4942 4955 4969 4983 4997 5011 5024 5038 1 3 4 6 7 8 10 11 12
32 5051 5065 5079 5092 5105 5119 5132 5145 5159 5172 1 3 4 5 7 8 9 11 12
33 5185 5198 5211 5224 5237 5250 5263 5276 5289 5302 1 3 4 5 6 8 9 10 12
34 5315 5328 5340 5353 5366 5378 5391 5403 5416 5428 1 3 4 5 6 8 9 10 11
35 5441 5453 5465 5478 5490 5502 5514 5527 5539 5551 1 2 4 5 6 7 9 10 11

36 5563 5575 5587 5599 5611 5623 5635 5647 5658 5670 1 2 4 5 6 7 8 10 11
37 5682 5694 5705 5717 5729 5740 5752 5763 5775 5786 1 2 3 5 6 7 8 9 10
38 5798 5808 5821 5832 5843 5855 5866 5877 5888 5899 1 2 3 5 6 7 8 9 10
39 5911 5922 5933 5944 5955 5966 5977 5988 5999 6010 1 2 3 4 5 7 8 9 10
40 6021 6031 6042 6053 6064 6075 6085 6096 6107 6117 1 2 3 4 5 6 8 9 10

41 6128 6138 6149 6160 6170 6180 6191 6201 6212 6222 1 2 3 4 5 6 7 8 9
42 6232 6243 6253 6263 6274 6284 6294 6304 6314 6325 1 2 3 4 5 6 7 8 9
43 6335 6345 6355 6365 6375 6385 6395 6405 6415 6425 1 2 3 4 5 6 7 8 9
44 6435 6444 6454 6464 6474 6484 6493 6503 6514 6522 1 2 3 4 5 6 7 8 9
45 6532 6542 6551 6561 6571 6580 6590 6599 6609 6618 1 2 3 4 5 6 7 8 9

46 6628 6637 6646 6656 6665 6675 6684 6693 6702 6712 1 2 3 4 5 6 7 7 8
47 6721 6730 6739 6749 6758 6767 6776 6785 6794 6803 1 2 3 4 5 5 6 7 8
48 6812 6821 6830 6839 6848 6857 6866 6875 6884 6893 1 2 3 3 4 5 6 6 8
49 6902 6911 6920 6928 6937 6946 6955 6964 6972 6981 1 2 3 4 4 5 6 7 8
50 6990 6998 7007 7016 7024 7033 7042 7050 7059 7067 1 2 3 3 4 5 6 7 8

51 7076 7083 7093 7101 7110 7118 7126 7135 7143 7152 1 2 3 3 4 5 6 7 8
52 7160 7168 7177 7185 7193 7202 7210 7318 7226 7235 1 2 3 3 4 5 6 7 7
53 7243 7251 7259 7267 7275 7284 7292 7300 7308 7316 1 2 2 3 4 5 6 6 7
54 7324 7332 7340 7348 7356 7364 7372 7350 7355 73 1 2 2 3 4 5 6 6 7
55 7404 7412 7419 7427 7435 7443 7451 7459 7466 7474 1 2 2 3 4 5 5 6 7

56 7482 7490 7497 7505 7513 7520 7528 7536 7543 7551 1 2 2 3 4 5 5 6 7
57 7559 7566 7574 7582 7589 7597 7604 7612 7619 7627 1 2 2 3 4 5 5 6 7
58 7634 7642 7619 7657 7664 7672 7679 7686 7694 7701 1 1 2 3 4 4 5 6 7
59 7709 7716 7723 7731 7738 7745 7752 7760 7767 7774 1 1 2 3 4 4 5 6 7
60 7782 7789 7796 7803 7810 7818 7825 7832 7839 7846 1 1 2 3 4 4 5 6 6

61 7853 7860 7868 7875 7882 7889 7896 7903 7910 7917 1 1 2 3 4 4 5 6 6
CA. Parag Gupta Ph.: +91 9891 432 632 Paraggupta_ca@yahoo.co.in Costing & O.R.
World’s largest CA Final student’s consultancy group: http://groups.yahoo.com/group/costingbyparaggupta
-g-

Mean Differences
0 1 2 3 4 5 6 7 8 9
1 2 3 4 5 6 7 8 9
62 7924 7931 7938 7945 7952 7959 7966 7973 7980 7987 1 1 2 3 3 4 5 6 6
63 7993 8000 8007 8014 8021 8028 8035 8041 8048 8055 1 1 2 1 1 2 5 5 6
64 8062 8069 8075 8082 8089 8096 8102 8109 8116 8122 1 1 2 3 3 4 5 5 6
65 8129 8136 8142 8149 8156 8162 8169 8176 8182 8189 1 1 2 3 3 4 5 5 6

66 8195 8202 8209 8215 8222 8228 8235 8241 8248 8254 1 1 2 3 3 4 5 5 6
67 8261 8267 8274 8280 8287 8293 8299 8306 8312 8319 1 1 2 3 3 4 5 5 6
68 8325 8331 8338 8344 8351 8357 8363 8370 8376 8382 1 1 2 3 3 4 4 5 6
69 8388 8395 8401 8407 8414 8420 8426 8432 8439 8445 1 1 2 2 3 4 4 5 6
70 8451 8457 8463 8470 8476 8482 8488 8494 8500 8506 1 1 2 2 3 4 4 5 6

71 8513 8519 8525 8531 8537 8543 8549 8555 8561 8567 1 1 2 2 3 4 4 5 5
72 8573 8579 8585 8591 8597 8603 8609 8615 8621 8627 1 1 2 2 3 4 4 5 5
73 8633 8639 8645 8651 8657 8663 8669 8675 8681 8686 1 1 2 2 3 4 4 5 5
74 8692 8698 8704 8710 8716 8722 8727 8733 8739 8745 1 1 2 2 3 4 4 5 5
75 8751 8756 8762 8768 8774 8779 8785 8791 8797 8802 1 1 2 2 3 3 4 5 5

76 8808 8814 8820 8825 8831 8837 8842 8848 8854 8859 1 1 2 2 3 3 4 5 5
77 8865 8871 8876 8882 8887 8893 8899 8904 8910 8915 1 1 2 2 3 3 4 4 5
78 8921 8927 8932 8938 8943 8949 8954 8960 8965 8971 1 1 2 2 3 3 4 4 5
79 8976 8982 8987 8993 8998 9004 9009 9015 9020 9025 1 1 2 2 3 3 4 4 5
80 9031 9036 9042 9047 9053 9058 9063 9069 9074 9079 1 1 2 2 3 3 4 4 5

81 9085 9090 9096 9101 9106 9112 9117 9122 9128 9133 1 1 2 2 3 3 4 4 5
82 9138 9143 9149 9154 9159 9165 9170 9175 9180 9186 1 1 2 2 3 3 4 4 5
83 9191 9196 9201 9206 9212 9217 9222 9227 9232 9238 1 1 2 2 3 3 4 4 5
84 9243 9248 9253 9258 9263 9269 9274 9279 9284 9289 1 1 2 2 3 3 4 4 5
85 9294 9299 9304 9309 9315 9320 9325 9330 9335 9340 1 1 2 2 3 3 4 4 5

86 9345 9350 9355 9360 9365 9370 9375 9380 9385 9390 1 1 2 2 3 3 4 4 5
87 9395 9400 9405 9410 9415 9420 9425 9430 9435 9440 0 1 1 2 2 3 3 4 4
88 9445 9450 9455 9460 9465 9469 9474 9479 9484 9489 0 1 1 2 2 3 3 4 4
89 9494 9499 9504 9509 9513 9518 9523 9528 9533 9538 0 1 1 2 2 3 3 4 4
90 9542 9547 9552 9557 9562 9566 9571 9576 9581 9586 0 1 1 2 2 3 3 4 4

91 9590 9595 9600 9605 9609 9614 9619 9634 9628 9633 0 1 1 2 2 3 3 4 4
92 9638 9643 9647 9652 9657 9661 9666 9671 9675 9680 0 1 1 2 2 3 3 4 4
93 9685 9689 9694 9699 9703 9708 9713 9717 9722 9727 0 1 1 2 2 3 3 4 4
94 9731 9736 9741 9745 9750 9754 9759 9763 9768 9773 0 1 1 2 2 3 3 4 4
95 9777 9782 9786 9791 9795 9800 9805 9809 9814 9818 0 1 1 2 2 3 3 4 4

96 9823 9827 9832 9836 9841 9845 9850 9854 9859 9863 0 1 1 2 2 3 3 4 4
97 9868 9872 9877 9881 9886 9890 9894 9899 9903 9908 0 1 1 2 2 3 3 4 4
98 9982 9917 9921 9928 9930 9934 9939 9943 9948 9952 0 1 1 2 2 3 3 4 4
99 9956 9961 9965 9969 9974 9978 9983 9987 9991 9996 0 1 1 2 2 3 3 3 4

CA. Parag Gupta Ph.: +91 9891 432 632 Paraggupta_ca@yahoo.co.in Costing & O.R.
World’s largest CA Final student’s consultancy group: http://groups.yahoo.com/group/costingbyparaggupta
-h-

ANTILOGARITHMS
Mean Differences
0 1 2 3 4 5 6 7 8 9
1 2 3 4 5 6 7 8 9
.00 1000 1002 1005 1007 1009 1012 1014 1016 1019 1021 0 0 1 1 1 1 2 2 2
.01 1023 1026 1018 1030 1033 1035 1038 1040 1042 1045 0 0 1 1 1 1 2 2 2
.02 1047 1050 1052 1054 1057 1059 1062 1064 1067 1069 0 0 1 1 1 1 2 2 2
.03 1072 1074 1076 1079 1081 1084 1086 1089 1091 1094 0 0 1 1 1 1 2 2 2
.04 1096 1099 1102 1104 1107 1109 1112 1114 1117 1119 0 1 1 1 1 2 2 2 2
.05 1122 1125 1127 1130 1132 1135 1138 1140 1143 1146 0 1 1 1 1 2 2 2 2

.06 1148 1151 1153 1156 1159 1161 1164 1167 1169 1172 0 1 1 1 1 2 2 2 2
.07 1175 1178 1180 1183 1186 1189 1191 1194 1197 1199 0 1 1 1 1 2 2 2 2
.08 1202 1205 1208 1211 1213 1216 1219 1222 1225 1227 0 1 1 1 1 2 2 2 3
.09 1230 1233 1236 1239 1242 1245 1247 1250 1253 1256 0 1 1 1 1 2 2 2 3
.10 1259 1262 1265 1268 1271 1274 1276 1279 1282 1285 0 1 1 1 1 2 2 2 3

.11 1288 1291 1294 1297 1300 1303 1306 1309 1312 1315 0 1 1 1 2 2 2 2 3
.12 1318 1321 1324 1327 1330 1333 1337 1340 1343 1346 0 1 1 1 2 2 2 2 3
.13 1349 1352 1355 1358 1361 1364 1368 1371 1374 1377 0 1 1 1 2 2 2 2 3
.14 1380 1384 1387 1390 1393 1396 1400 1403 1406 1409 0 1 1 1 2 2 2 2 3
.15 1413 1416 1419 1422 1426 1429 1432 1435 1439 1442 0 1 1 1 2 2 2 2 3

.16 1445 1449 1452 1455 1459 1462 1466 1469 1472 1476 0 1 1 1 2 2 2 3 3
.17 1479 1483 1486 1489 1492 1493 1496 1500 1503 1507 0 1 1 1 2 2 2 3 3
.18 1514 1517 1521 1524 1528 1531 1535 1538 1542 1545 0 1 1 1 2 2 2 3 3
.19 1549 1552 1556 1560 1563 1567 1570 1574 1578 1581 0 1 1 1 2 2 2 3 3
.20 1585 1589 1592 1596 1600 1603 1607 1611 1614 1618 0 1 1 1 2 2 2 3 3

.21 1622 1626 1629 1633 1637 1641 1644 1648 1652 1656 0 1 1 2 2 2 3 3 3
.22 1660 1663 1667 1671 1675 1679 1683 1687 1690 1694 0 1 1 2 2 2 3 3 3
.23 1698 1702 1706 1710 1714 1718 1722 1726 1730 1734 0 1 1 2 2 2 3 3 4
.24 1738 1742 1746 1750 1754 1758 1762 1766 1770 1774 0 1 1 2 2 2 3 3 4
.25 1778 1782 1786 1791 1795 1799 1803 1807 1811 1816 0 1 1 2 2 2 3 3 4

.26 1820 1824 1828 1832 1837 1841 1845 1849 1854 1858 0 1 1 2 2 3 3 3 4
.27 1862 1866 1871 1875 1879 1884 1888 1892 1897 1901 0 1 1 2 2 3 3 3 4
.28 1905 1910 1914 1919 1923 1928 1932 1936 1941 1945 0 1 1 2 2 3 3 4 4
.29 1950 1954 1959 1963 1968 1972 1977 1982 1986 1991 0 1 1 2 2 3 3 4 4
.30 1995 2000 2004 2009 2014 2018 2023 2028 2032 2037 0 1 1 2 2 3 3 4 4

.31 2042 2046 2051 2056 2061 2065 2070 2075 2080 2084 0 1 1 2 2 3 3 4 4
.32 2089 2094 2099 2104 2109 2113 2118 2123 2128 2133 0 1 1 2 2 3 3 4 4
.33 2138 2143 2148 2153 2158 2163 2168 2173 2178 2183 0 1 1 2 2 3 3 4 4
.34 2188 2193 2198 2203 2208 2213 2218 2223 2228 2234 1 1 2 2 3 3 4 4 5
.35 2239 2245 2249 2254 2259 2265 2270 2275 2280 2286 1 1 2 2 3 3 4 4 5

.36 2291 2296 2301 2307 2312 2317 2323 2328 2333 2339 1 1 2 2 3 3 4 4 5
.37 2344 2350 2355 2360 2366 2371 2376 2381 2386 2393 1 1 2 2 3 3 4 4 5
.38 2399 2404 2410 2415 2421 2427 2432 2438 2443 2449 1 1 2 2 3 3 4 4 5
.39 2455 2460 2465 2472 2477 2483 2489 2495 2500 2506 1 1 2 2 3 3 4 5 5
.40 2512 2518 2523 2529 2535 2541 2547 2553 2559 2564 1 1 2 2 3 4 4 5 5

.41 2570 2576 2582 2588 2594 2600 2606 2612 2618 2624 1 1 2 2 3 4 4 5 5
.42 2630 2636 2642 2649 2655 2661 2667 2673 2679 2685 1 1 2 2 3 4 4 5 6
.43 2692 2698 2704 2710 2716 2723 2729 2735 2742 2748 1 1 2 3 3 4 4 5 6
.44 2754 2761 2767 2773 2780 2786 2793 2799 2805 2812 1 1 2 3 3 4 4 5 6
.45 2818 2825 2831 2838 2844 2851 2858 2864 2871 2877 1 1 2 3 3 4 5 5 6

.46 2884 2891 2897 2904 2911 2917 2924 2931 2938 2944 1 1 2 3 3 4 5 5 6
.47 2951 2958 2965 2972 2979 2985 2992 2999 3006 3013 1 1 2 3 3 4 5 5 6
.48 3020 3027 3034 3041 3048 3055 3062 3069 3076 3083 1 1 2 3 4 4 5 6 6
.49 3090 3097 3105 3112 3119 3126 3133 3141 3148 3155 1 1 2 3 4 4 5 6 6
.50 3162 3170 3177 3184 3192 3199 3206 3214 3221 3228 1 1 2 3 4 4 5 6 8

.51 3236 3243 3251 3258 3266 3273 3281 3289 3296 3304 1 2 2 3 4 5 5 6 8
.52 3311 3319 3327 3334 3342 3350 3357 3365 3373 3381 1 2 2 3 4 5 5 6 8

CA. Parag Gupta Ph.: +91 9891 432 632 Paraggupta_ca@yahoo.co.in Costing & O.R.
World’s largest CA Final student’s consultancy group: http://groups.yahoo.com/group/costingbyparaggupta
-i-

Mean Differences
0 1 2 3 4 5 6 7 8 9
1 2 3 4 5 6 7 8 9
.53 3388 3396 3404 3412 3420 3428 3436 3443 3451 3459 1 2 2 3 4 5 6 6 8
.54 3467 3475 3483 3491 3499 3508 3516 3524 3532 3540 1 2 2 3 4 5 6 6 8
.55 3548 3556 3565 3573 3581 3589 3597 3606 3614 3622 1 2 2 3 4 5 6 7 8

.56 3631 3639 3648 3656 3664 3673 3681 3690 3698 3707 1 2 3 3 4 5 6 7 8
.57 3715 3724 3733 3741 3750 3758 3767 3776 3784 3793 1 2 3 3 4 5 6 7 8
.58 3802 3811 3819 3828 3837 3846 3855 3864 3873 3882 1 2 3 4 4 5 6 7 8
.59 3890 3899 3908 3917 3926 3936 3945 3954 3963 3972 1 2 3 4 5 5 6 7 8
.60 4074 4083 4093 4102 4111 4121 4130 4140 4150 4159 1 2 3 4 5 6 7 8 9

.61 4074 4083 4093 4102 4111 4121 4130 4140 4150 4159 1 2 3 4 5 6 7 8 9
.62 4169 4178 4188 4198 4207 4217 4227 4236 4256 4256 1 2 3 4 5 6 7 8 9
.63 4266 4276 4285 4295 4305 4315 4325 4335 4345 4355 1 2 3 4 5 6 7 8 9
.64 4365 4375 4385 4395 4406 4416 4426 4436 4446 4457 1 2 3 4 5 6 7 8 9
.65 4467 4477 4487 4498 4508 4519 4529 4539 4550 4560 1 2 3 4 5 6 7 8 9

.66 4571 4581 4592 4603 4613 4624 4634 4645 4656 4667 1 2 3 4 5 6 7 9 10
.67 4677 4688 4699 4710 4721 4732 4742 4753 4764 4775 1 2 3 4 5 7 8 9 10
.68 4786 4797 4808 4819 4831 4842 4853 4864 4875 4887 1 2 3 4 6 7 8 9 10
.69 4898 4909 4920 4932 4943 4955 4966 4977 4989 5000 1 2 3 5 6 7 8 9 10
.70 5012 5023 5035 5047 5058 5070 5082 5093 5105 5117 1 2 4 5 6 7 8 9 11

.71 5129 5140 5152 5164 5176 5188 5200 5212 5224 5236 1 2 4 5 6 7 8 10 11
.72 5248 5260 5272 5284 5297 5309 5321 5333 5346 5358 1 2 4 5 6 7 9 10 11
.73 5376 5383 5395 5408 5420 5433 5445 5458 5470 5483 1 3 4 5 6 8 9 10 11
.74 5495 5508 5521 5534 5546 5559 5572 5585 5598 5610 1 3 4 5 6 8 9 10 11
.75 5623 5636 5649 5662 5675 5689 5702 5715 5728 5741 1 3 4 5 7 8 9 10 11

.76 5754 5768 5781 5794 5808 5821 5834 5848 5861 5875 1 3 4 5 7 8 9 11 12
.77 5888 5902 5916 5929 5943 5957 5970 5984 5998 6012 1 3 4 5 7 8 10 11 12
.78 6026 6039 6053 6067 6081 6095 6109 6124 6138 6152 1 3 4 6 7 8 10 11 13
.79 6166 6180 6194 6209 6223 6237 6252 6266 6281 6295 1 3 4 6 7 9 10 11 13
.80 6310 6324 6339 6353 6368 6383 6397 6412 6427 6442 1 3 4 6 7 9 10 12 13

.81 6457 6471 6486 6501 6516 6531 6546 6561 6577 6592 2 3 5 6 8 9 11 12 14
.82 6607 6622 6637 6653 6668 6683 6699 6714 6730 6745 2 3 5 6 8 9 11 12 14
.83 6761 6776 6792 6808 6823 6839 6855 6871 6887 6902 2 3 5 6 8 9 11 13 14
.84 6918 6934 6950 6966 6982 6998 7015 7031 7047 7063 2 3 5 6 8 10 11 13 15
.85 7079 7096 7112 7129 7145 7161 7178 7194 7211 7228 2 3 5 7 8 10 12 13 15

.86 7244 7261 7278 7295 7311 7328 7345 7362 7379 7396 2 3 5 7 8 10 12 13 15
.87 7413 7430 7447 7464 7482 7499 7516 7534 7551 7568 2 3 5 7 9 10 12 14 16
.88 7586 7603 7621 7638 7656 7674 7691 7709 7727 7745 2 4 5 7 9 11 12 14 16
.89 7762 7780 7798 7816 7834 7852 7870 7889 7907 7925 2 4 5 7 9 11 13 14 16
.90 7943 7962 7980 7998 8017 8035 8054 8072 8091 8110 2 4 6 7 9 11 13 15 17

.91 8128 8147 8166 8185 8204 8222 8241 8260 8279 8299 2 4 6 8 9 11 13 15 17
.92 8318 8337 8356 8375 8395 8414 8433 8453 8472 8492 2 4 6 8 10 12 14 15 17
.93 8511 8531 8551 8570 8590 8610 8630 8650 8670 8690 2 4 6 8 10 12 14 16 18
.94 8710 8730 8750 8770 8790 8810 8831 8851 8872 8892 2 4 6 8 10 12 14 16 18
.95 8913 8933 8954 8974 8895 9016 9036 9057 9078 9099 2 4 6 8 10 12 15 17 19

.96 9120 9141 9162 9183 9204 9226 9247 9268 9290 9311 2 4 6 8 11 13 15 17 19
.97 9333 9354 9376 9397 9419 9441 9462 9484 9506 9528 2 4 7 9 11 13 15 17 20
.98 9550 9572 9594 9616 9638 9661 9683 9705 9727 9750 2 4 7 9 11 13 16 18 20
.99 9772 9795 9817 9840 9863 9886 9908 9931 9954 9977 2 5 7 9 11 14 16 18 20

CA. Parag Gupta Ph.: +91 9891 432 632 Paraggupta_ca@yahoo.co.in Costing & O.R.
World’s largest CA Final student’s consultancy group: http://groups.yahoo.com/group/costingbyparaggupta

You might also like